Sie sind auf Seite 1von 711

MyPasTest

Main Navigation
Question Browser: MRCP 1
Home
Subscriptions Question Browser Exam Builder Saved Exams
» MRCP 1 Session Progress
• Question Browser
Questions Correct 0
• Timed Test A 56-year-old woman, who is known to suffer from
rheumatoid arthritis, complains she has had recurrent Questions Incorrect 0
• Mock Exam
haemoptysis for over 5 years. She has never smoked and Questions Total 0
• Past Papers only takes a non-steroidal anti-inflammatory agent.
Questions Percentage 0%
• Random Questions According to her, she coughs up phlegm every day and at
times this contains streaks of fresh blood. She has no More
• My Performance
known respiratory disease, but tends to get frequent chest
• Media Bank infections that are relieved by a course of antibiotics. What
• New Multimedia is the most likely diagnosis?
Reference: Normal Values
Online Extras Lung cancer
Haematology
Library Bronchiectasis
Community Immunoglobulins
Tuberculosis
Help Pulmonary embolism Biochemistry
PasTest Store Atypical pneumonia Diabetes

235 Endocrinology
My Account
Blood gases
Profile
CSF
Newsletters
My Career and Exams
Order History
Learning Goals
Question Filters

Security

Change Password
Sign Out

© 2011 PasTest Ltd | About Us | Contact Us | Help

file:///E|/Shakhawan/Respiratory%20S/1.htm[3/13/2012 3:55:57 PM]


MyPasTest

Main Navigation
Question Browser: MRCP 1
Home
Subscriptions Question Browser Exam Builder Saved Exams
» MRCP 1 Session Progress
• Question Browser
Questions Correct 1
• Timed Test A 56-year-old woman, who is known to suffer from
rheumatoid arthritis, complains she has had recurrent Questions Incorrect 0
• Mock Exam
haemoptysis for over 5 years. She has never smoked and Questions Total 1
• Past Papers only takes a non-steroidal anti-inflammatory agent.
Questions Percentage 100 %
• Random Questions According to her, she coughs up phlegm every day and at
times this contains streaks of fresh blood. She has no More
• My Performance
known respiratory disease, but tends to get frequent chest
• Media Bank infections that are relieved by a course of antibiotics. What
• New Multimedia is the most likely diagnosis?

Online Extras Lung cancer


Library Bronchiectasis Your answer
Community Tuberculosis
Help Pulmonary embolism
PasTest Store Atypical pneumonia

My Account

Profile
Some 3–4% of patients with rheumatoid arthritis
Newsletters
develop bronchiectasis. This is characterised by
My Career and Exams recurrent haemoptysis. The history of expectorating
Order History phlegm on most days and frequent chest infections are Reference: Normal Values
suggestive of the diagnosis. A high-resolution computed
Learning Goals tomography (CT) scan of her lungs will establish the Haematology
Question Filters diagnosis.
Immunoglobulins
Biochemistry
Security 235
Diabetes
Change Password
Sign Out Endocrinology

Blood gases

CSF

© 2011 PasTest Ltd | About Us | Contact Us | Help

file:///E|/Shakhawan/Respiratory%20S/1a.htm[3/13/2012 3:55:59 PM]


MyPasTest

Main Navigation
Question Browser: MRCP 1
Home
Subscriptions Question Browser Exam Builder Saved Exams
» MRCP 1 Session Progress
• Question Browser
Questions Correct 1
• Timed Test A 26-year-old, previously healthy, naval diving officer is
admitted to the hospital with a sudden-onset, left-sided Questions Incorrect 0
• Mock Exam
chest pain. A chest X-ray confirms a small left-sided Questions Total 1
• Past Papers pneumothorax. The patient is slightly breathless and his
Questions Percentage 100 %
• Random Questions oxygen saturation on air is 95%. He smokes 5–10
cigarettes a day. Which of the following statements is More
• My Performance
true?
• Media Bank
• New Multimedia Simple aspiration is the first-line treatment if the
patient is symptomatic Reference: Normal Values
Online Extras
If admitted for observation, he does not need oxygen Haematology
Library
Once the pneumothorax has resolved he can go back
Community Immunoglobulins
to work, including resuming his diving duties
Help Smoking cessation has no role in management, apart Biochemistry
PasTest Store from in the prevention of COPD in later life
Diabetes
He should be treated with a small (10–14 F)-sized
chest drain Endocrinology
My Account
Blood gases
Profile 236
CSF
Newsletters
My Career and Exams
Order History
Learning Goals
Question Filters

Security

Change Password
Sign Out

© 2011 PasTest Ltd | About Us | Contact Us | Help

file:///E|/Shakhawan/Respiratory%20S/2.htm[3/13/2012 3:56:01 PM]


MyPasTest

Main Navigation
Question Browser: MRCP 1
Home
Subscriptions Question Browser Exam Builder Saved Exams
» MRCP 1 Session Progress
• Question Browser
Questions Correct 1
• Timed Test A 26-year-old, previously healthy, naval diving officer is
admitted to the hospital with a sudden-onset, left-sided Questions Incorrect 0
• Mock Exam
chest pain. A chest X-ray confirms a small left-sided Questions Total 1
• Past Papers pneumothorax. The patient is slightly breathless and his
Questions Percentage 100 %
• Random Questions oxygen saturation on air is 95%. He smokes 5–10
cigarettes a day. Which of the following statements is More
• My Performance
true?
• Media Bank
• New Multimedia Simple aspiration is the first-line Your answer
treatment if the patient is symptomatic
Online Extras
If admitted for observation, he does
Library not need oxygen
Community Once the pneumothorax has resolved
Help he can go back to work, including
resuming his diving duties
PasTest Store
Smoking cessation has no role in
management, apart from in the
My Account prevention of COPD in later life

Profile He should be treated with a small (10–


14 F)-sized chest drain
Newsletters
My Career and Exams
Order History Reference: Normal Values
Learning Goals A small pneumothorax in a person with no previous Haematology
Question Filters chest disease should be aspirated if the patient is
breathless. A pneumothorax can be classed as small or Immunoglobulins
large depending on the presence of a visible rim of < 2
Biochemistry
Security cm or ≥ 2 cm between the lung margin and the chest
wall. Ideally, an asymptomatic patient with a small, Diabetes
Change Password primary spontaneous pneumothorax can be discharged
Sign Out with the advice to return if he becomes symptomatic. Endocrinology

Blood gases
If a patient is admitted for observation, high-flow
oxygen increases the absorption of air from the pleural CSF
cavity. Smoking increases the chances of a recurrence.
Diving is contraindicated unless the patient has
undergone pleurectomy.

236

© 2011 PasTest Ltd | About Us | Contact Us | Help

file:///E|/Shakhawan/Respiratory%20S/2a.htm[3/13/2012 3:56:02 PM]


MyPasTest

Main Navigation
Question Browser: MRCP 1
Home
Subscriptions Question Browser Exam Builder Saved Exams
» MRCP 1 Session Progress
• Question Browser
Questions Correct 1
• Timed Test A 46-year-old woman is admitted to hospital with a left
basal, community-acquired pneumonia. She is on the Questions Incorrect 0
• Mock Exam
appropriate antibiotics. She is still pyrexial four days after Questions Total 1
• Past Papers admission and a chest X-ray confirms a left pleural
Questions Percentage 100 %
• Random Questions effusion. The house officer has performed a diagnostic tap.
Which of the following is an indication to insert a More
• My Performance
chest drain?
• Media Bank
• New Multimedia Pleural fluid protein level more than 50% of serum
protein level Reference: Normal Values
Online Extras
Pleural fluid LDH more than 60% of serum LDH Haematology
Library
Haemorrhagic pleural fluid
Community Immunoglobulins
Pleural fluid pH < 7.2
Help Biochemistry
Pleural fluid glucose < 1.6 mmol/l
PasTest Store Diabetes
237
Endocrinology
My Account
Blood gases
Profile
CSF
Newsletters
My Career and Exams
Order History
Learning Goals
Question Filters

Security

Change Password
Sign Out

© 2011 PasTest Ltd | About Us | Contact Us | Help

file:///E|/Shakhawan/Respiratory%20S/3.htm[3/13/2012 3:56:04 PM]


MyPasTest

Main Navigation
Question Browser: MRCP 1
Home
Subscriptions Question Browser Exam Builder Saved Exams
» MRCP 1 Session Progress
• Question Browser
Questions Correct 1
• Timed Test A 46-year-old woman is admitted to hospital with a left
basal, community-acquired pneumonia. She is on the Questions Incorrect 1
• Mock Exam
appropriate antibiotics. She is still pyrexial four days after Questions Total 2
• Past Papers admission and a chest X-ray confirms a left pleural
Questions Percentage 50 %
• Random Questions effusion. The house officer has performed a diagnostic tap.
Which of the following is an indication to insert a More
• My Performance
chest drain?
• Media Bank
• New Multimedia Pleural fluid protein level more than
50% of serum protein level
Online Extras
Pleural fluid LDH more than 60% of
Library serum LDH
Community Haemorrhagic pleural fluid
Help Pleural fluid pH < 7.2 Correct answer
PasTest Store Pleural fluid glucose < 1.6 mmol/l Your answer

My Account

Profile
This woman has a parapneumonic effusion. The
Newsletters
indications for chest tube insertion in patients with
My Career and Exams infected pleural effusions are: presence of organisms on
Order History a Gram stain of the pleural fluid, a frankly purulent Reference: Normal Values
pleural fluid, pleural pH < 7.2 in the setting of an
Learning Goals infected pleural effusion, loculated pleural effusions and Haematology
Question Filters poor clinical progress despite antibiotic treatment.
These indications for chest drain insertion are those Immunoglobulins
proposed by the BTS guidelines (Thorax 2003; 58;
Biochemistry
Security ii18);
http://thorax.bmj.com/cgi/content/full/58/suppl_2/ii18 Diabetes
Change Password
Sign Out Endocrinology
237
Blood gases

CSF

© 2011 PasTest Ltd | About Us | Contact Us | Help

file:///E|/Shakhawan/Respiratory%20S/3a.htm[3/13/2012 3:56:05 PM]


MyPasTest

Main Navigation
Question Browser: MRCP 1
Home
Subscriptions Question Browser Exam Builder Saved Exams
» MRCP 1 Session Progress
• Question Browser
Questions Correct 1
• Timed Test A 19-year-old army cadet is admitted to hospital with
cough, headaches and malaise. He has a temperature of 38 Questions Incorrect 1
• Mock Exam
°C. His blood count and renal and liver function are normal. Questions Total 2
• Past Papers Cold agglutinins are positive. A chest X-ray shows bibasal
Questions Percentage 50 %
• Random Questions shadowing. What is the most likely diagnosis?
• My Performance More
Legionella pneumonia
• Media Bank
Viral pneumonia
• New Multimedia
Q fever Reference: Normal Values
Online Extras
Klebsiella pneumonia Haematology
Library
Mycoplasma pneumonia
Community Immunoglobulins
Help 238 Biochemistry
PasTest Store Diabetes
Endocrinology
My Account
Blood gases
Profile
CSF
Newsletters
My Career and Exams
Order History
Learning Goals
Question Filters

Security

Change Password
Sign Out

© 2011 PasTest Ltd | About Us | Contact Us | Help

file:///E|/Shakhawan/Respiratory%20S/4.htm[3/13/2012 3:56:06 PM]


MyPasTest

Main Navigation
Question Browser: MRCP 1
Home
Subscriptions Question Browser Exam Builder Saved Exams
» MRCP 1 Session Progress
• Question Browser
Questions Correct 2
• Timed Test A 19-year-old army cadet is admitted to hospital with
cough, headaches and malaise. He has a temperature of 38 Questions Incorrect 1
• Mock Exam
°C. His blood count and renal and liver function are normal. Questions Total 3
• Past Papers Cold agglutinins are positive. A chest X-ray shows bibasal
Questions Percentage 66 %
• Random Questions shadowing. What is the most likely diagnosis?
• My Performance More
Legionella pneumonia
• Media Bank
Viral pneumonia
• New Multimedia
Q fever
Online Extras
Klebsiella pneumonia
Library
Mycoplasma pneumonia Your answer
Community
Help
PasTest Store
Mycoplasmal pneumonia

My Account Mycoplasma pneumonia is a common cause of


pneumonia, often occurring in young people. It is
Profile characterised by headaches, malaise and cough. The
chest X-ray findings might not correlate with the
Newsletters
patient’s condition. The white blood count can be
My Career and Exams normal and cold agglutinins occur in only half of
Order History patients. Reference: Normal Values
Learning Goals Extrapulmonary complications Haematology
Question Filters
Pericarditis Immunoglobulins
Myocarditis
Erythema multiforme Biochemistry
Security
Vomiting
Diabetes
Change Password Diarrhoea
Meningo-encephalitis Endocrinology
Sign Out
Blood gases
238
CSF

© 2011 PasTest Ltd | About Us | Contact Us | Help

file:///E|/Shakhawan/Respiratory%20S/4a.htm[3/13/2012 3:56:08 PM]


MyPasTest

Main Navigation
Question Browser: MRCP 1
Home
Subscriptions Question Browser Exam Builder Saved Exams
» MRCP 1 Session Progress
• Question Browser
Questions Correct 2
• Timed Test A 35-year-old woman complains of a painful rash on her
legs. She also gives a three-month history of dyspnoea on Questions Incorrect 1
• Mock Exam
exertion. Her doctor arranged for her to have a chest X-ray Questions Total 3
• Past Papers and this shows bilateral hilar prominence. Examination of
Questions Percentage 66 %
• Random Questions her legs reveals a purplish-red, nodular tender rash on her
shins. Apart from few bi-basal crepitations on auscultation, More
• My Performance
systemic examination is normal. A pulmonary function test
• Media Bank shows a mildly impaired DLCO.
• New Multimedia
Which one of the following is the best investigation to Reference: Normal Values
Online Extras confirm the diagnosis?
Haematology
Library
Sputum microscopy and culture
Community Immunoglobulins
High-resolution CT scan of the thorax
Help Biochemistry
Open lung biopsy
PasTest Store Diabetes
Kveim test
Serum ACE level Endocrinology
My Account
Blood gases
Profile 239
CSF
Newsletters
My Career and Exams
Order History
Learning Goals
Question Filters

Security

Change Password
Sign Out

© 2011 PasTest Ltd | About Us | Contact Us | Help

file:///E|/Shakhawan/Respiratory%20S/5.htm[3/13/2012 3:56:10 PM]


MyPasTest

Main Navigation
Question Browser: MRCP 1
Home
Subscriptions Question Browser Exam Builder Saved Exams
» MRCP 1 Session Progress
• Question Browser
Questions Correct 2
• Timed Test A 35-year-old woman complains of a painful rash on her
legs. She also gives a three-month history of dyspnoea on Questions Incorrect 2
• Mock Exam
exertion. Her doctor arranged for her to have a chest X-ray Questions Total 4
• Past Papers and this shows bilateral hilar prominence. Examination of
Questions Percentage 50 %
• Random Questions her legs reveals a purplish-red, nodular tender rash on her
shins. Apart from few bi-basal crepitations on auscultation, More
• My Performance
systemic examination is normal. A pulmonary function test
• Media Bank shows a mildly impaired DLCO.
• New Multimedia
Which one of the following is the best investigation to
Online Extras confirm the diagnosis?
Library
Sputum microscopy and culture
Community
High-resolution CT scan of the Correct answer
Help thorax
PasTest Store Open lung biopsy
Kveim test
My Account Serum ACE level Your answer
Profile
Newsletters
My Career and Exams
This woman has erythema nodosum and bilateral hilar Reference: Normal Values
Order History
lymphadenopathy. She has sarcoidosis. This has a good
Learning Goals prognosis. However, the diagnosis is clinical as she has Haematology
Question Filters dyspnoea and impaired diffusion in the lung of carbon
monoxide (DLCO), a high-resolution lung scan will Immunoglobulins
establish the diagnosis. Whilst Kveim test is the
definitive investigation it is rarely done now due to the Biochemistry
Security
convenience of high resolution CT as a viable Diabetes
Change Password alternative. Serum ACE, whilst is it likely to be raised, is
non-specific. Endocrinology
Sign Out
Blood gases
239 CSF

© 2011 PasTest Ltd | About Us | Contact Us | Help

file:///E|/Shakhawan/Respiratory%20S/5a.htm[3/13/2012 3:56:11 PM]


MyPasTest

Main Navigation
Question Browser: MRCP 1
Home
Subscriptions Question Browser Exam Builder Saved Exams
» MRCP 1 Session Progress
• Question Browser
Questions Correct 2
• Timed Test A 64-year-old woman is referred to the medical team from
the orthopaedic ward. She underwent a right total hip Questions Incorrect 2
• Mock Exam
replacement 6 days ago. She is known to suffer from mild Questions Total 4
• Past Papers chronic obstructive pulmonary disease and is on regular
Questions Percentage 50 %
• Random Questions inhaled steroids and a short-acting β 2 -agonist. She now
• My Performance complains of left-sided chest pain and is also dyspnoeic. More
Your clinical diagnosis is pulmonary embolism. Which one
• Media Bank of the following would not be a feature of pulmonary
• New Multimedia embolism in this patient?
Reference: Normal Values
Online Extras
Dyspnoea Haematology
Library
Tachypnoea
Community Immunoglobulins
New-onset atrial fibrillation
Help Biochemistry
Fever
PasTest Store Diabetes
Bradycardia
Endocrinology
My Account 240
Blood gases
Profile
CSF
Newsletters
My Career and Exams
Order History
Learning Goals
Question Filters

Security

Change Password
Sign Out

© 2011 PasTest Ltd | About Us | Contact Us | Help

file:///E|/Shakhawan/Respiratory%20S/6.htm[3/13/2012 3:56:13 PM]


MyPasTest

Main Navigation
Question Browser: MRCP 1
Home
Subscriptions Question Browser Exam Builder Saved Exams
» MRCP 1 Session Progress
• Question Browser
Questions Correct 3
• Timed Test A 64-year-old woman is referred to the medical team from
the orthopaedic ward. She underwent a right total hip Questions Incorrect 2
• Mock Exam
replacement 6 days ago. She is known to suffer from mild Questions Total 5
• Past Papers chronic obstructive pulmonary disease and is on regular
Questions Percentage 60 %
• Random Questions inhaled steroids and a short-acting β 2 -agonist. She now
• My Performance complains of left-sided chest pain and is also dyspnoeic. More
Your clinical diagnosis is pulmonary embolism. Which one
• Media Bank of the following would not be a feature of pulmonary
• New Multimedia embolism in this patient?
Online Extras
Dyspnoea
Library
Tachypnoea
Community
New-onset atrial fibrillation
Help
Fever
PasTest Store
Bradycardia Your answer

My Account

Profile Clinical features of pulmonary embolism


Newsletters
The clinical features of pulmonary embolism are:
My Career and Exams
Order History Tachypnoea (respiratory rate > 20/min) – the Reference: Normal Values
commonest feature, occurring in 85% of patients
Learning Goals Fever – a frequent finding, occurring in 34–50% Haematology
Question Filters of patients
Tachycardia – occurs in 30% of patients Immunoglobulins
Atrial flutter, atrial fibrillation and premature Biochemistry
Security beats can also occur
Diabetes
Change Password
Sign Out 240 Endocrinology

Blood gases

CSF

© 2011 PasTest Ltd | About Us | Contact Us | Help

file:///E|/Shakhawan/Respiratory%20S/6a.htm[3/13/2012 3:56:14 PM]


MyPasTest

Main Navigation
Question Browser: MRCP 1
Home
Subscriptions Question Browser Exam Builder Saved Exams
» MRCP 1 Session Progress
• Question Browser
Questions Correct 3
• Timed Test A 65-year-old woman has been diagnosed as having lung
cancer. Which of the following statements is most Questions Incorrect 2
• Mock Exam
appropriate? Questions Total 5
• Past Papers
Questions Percentage 60 %
• Random Questions Hypertrophic pulmonary osteoarthropathy (HPOA) is
• My Performance commonly seen patients with small-cell carcinoma More
• Media Bank Inappropriate secretion of antidiuretic hormone
(ISADH) is commonly seen in patients with squamous-
• New Multimedia cell carcinoma Reference: Normal Values
Online Extras Hypercalcaemia may occur without bone metastasis
Haematology
Library Paraneoplastic syndromes occur more commonly with
Community squamous-cell carcinomas Immunoglobulins
Help Hypercalcaemia associated with bone metastasis is Biochemistry
best treated with intravenous steroids
PasTest Store Diabetes
241
Endocrinology
My Account
Blood gases
Profile
CSF
Newsletters
My Career and Exams
Order History
Learning Goals
Question Filters

Security

Change Password
Sign Out

© 2011 PasTest Ltd | About Us | Contact Us | Help

file:///E|/Shakhawan/Respiratory%20S/7.htm[3/13/2012 3:56:15 PM]


MyPasTest

Main Navigation
Question Browser: MRCP 1
Home
Subscriptions Question Browser Exam Builder Saved Exams
» MRCP 1 Session Progress
• Question Browser
Questions Correct 4
• Timed Test A 65-year-old woman has been diagnosed as having lung
cancer. Which of the following statements is most Questions Incorrect 2
• Mock Exam
appropriate? Questions Total 6
• Past Papers
Questions Percentage 66 %
• Random Questions Hypertrophic pulmonary
• My Performance osteoarthropathy (HPOA) is commonly More
seen patients with small-cell carcinoma
• Media Bank
Inappropriate secretion of antidiuretic
• New Multimedia hormone (ISADH) is commonly seen in
Online Extras patients with squamous-cell carcinoma
Library Hypercalcaemia may occur without Your answer
bone metastasis
Community
Paraneoplastic syndromes occur more
Help commonly with squamous-cell
PasTest Store carcinomas
Hypercalcaemia associated with bone
metastasis is best treated with
My Account intravenous steroids
Profile
Newsletters
My Career and Exams
Order History Paraneoplastic syndromes are a variety of non- Reference: Normal Values
metastatic metabolic or neuromuscular manifestations
Learning Goals of lung cancer. They are commonly associated with Haematology
Question Filters small-cell carcinoma. HPOA is characterised by painful
symmetrical arthropathy involving the wrist, ankle and Immunoglobulins
knee joints. It is most commonly seen in patients with
Biochemistry
Security adenocarcinoma and is virtually unknown in those with
small-cell carcinoma. Diabetes
Change Password
Inappropriate secretion of ADH is most common in Endocrinology
Sign Out
patients with small-cell carcinoma. Hypercalcaemia may
occur as a paraneoplastic syndrome with squamous-cell Blood gases
carcinoma. This is due to the production of a
CSF
parathyroid hormone-related peptide by the tumour,
which increases bone resorption, and renal tubular
reabsorption of calcium. This is best treated with
intravenous fluids and bisphosphonates. Steroids are
mainly used for treating hypercalcaemia associated with
sarcoidosis.

241

© 2011 PasTest Ltd | About Us | Contact Us | Help

file:///E|/Shakhawan/Respiratory%20S/7a.htm[3/13/2012 3:56:17 PM]


MyPasTest

Main Navigation
Question Browser: MRCP 1
Home
Subscriptions Question Browser Exam Builder Saved Exams
» MRCP 1 Session Progress
• Question Browser
Questions Correct 4
• Timed Test You are asked to see a newly diagnosed asthmatic woman
who is eight weeks’ pregnant. She is not on any treatment Questions Incorrect 2
• Mock Exam
at the moment. Her PEFR diary shows wide diurnal Questions Total 6
• Past Papers variations and she also gives a past history of eczema.
Questions Percentage 66 %
• Random Questions Which of the following is true?
• My Performance More
Short-acting β2 -agonists are contraindicated during
• Media Bank
the first trimester
• New Multimedia
Inhaled steroids are associated with major congenital Reference: Normal Values
Online Extras deformities
Haematology
Library A leukotriene-receptor antagonist is the first-line
Community treatment Immunoglobulins
Help Low dose inhaled corticosteroids would be considered Biochemistry
acceptable
PasTest Store Diabetes
Steroid tablets are teratogenic
Endocrinology
My Account 242
Blood gases
Profile
CSF
Newsletters
My Career and Exams
Order History
Learning Goals
Question Filters

Security

Change Password
Sign Out

© 2011 PasTest Ltd | About Us | Contact Us | Help

file:///E|/Shakhawan/Respiratory%20S/8.htm[3/13/2012 3:56:19 PM]


MyPasTest

Main Navigation
Question Browser: MRCP 1
Home
Subscriptions Question Browser Exam Builder Saved Exams
» MRCP 1 Session Progress
• Question Browser
Questions Correct 5
• Timed Test You are asked to see a newly diagnosed asthmatic woman
who is eight weeks’ pregnant. She is not on any treatment Questions Incorrect 2
• Mock Exam
at the moment. Her PEFR diary shows wide diurnal Questions Total 7
• Past Papers variations and she also gives a past history of eczema.
Questions Percentage 71 %
• Random Questions Which of the following is true?
• My Performance More
Short-acting β2 -agonists are
• Media Bank
contraindicated during the first trimester
• New Multimedia
Inhaled steroids are associated with
Online Extras major congenital deformities
Library A leukotriene-receptor antagonist is
Community the first-line treatment

Help Low dose inhaled corticosteroids would Your answer


be considered acceptable
PasTest Store
Steroid tablets are teratogenic

My Account

Profile
Newsletters In general, the medicines used for asthma are safe
My Career and Exams during pregnancy.
Order History Leukotriene-receptor antagonists are not the first-line Reference: Normal Values
treatment during pregnancy. However, they may be
Learning Goals continued in women who have demonstrated Haematology
Question Filters significant improvement in asthma control with these
agents prior to pregnancy. Immunoglobulins
Biochemistry
Security
242 Diabetes
Change Password
Sign Out Endocrinology

Blood gases

CSF

© 2011 PasTest Ltd | About Us | Contact Us | Help

file:///E|/Shakhawan/Respiratory%20S/8a.htm[3/13/2012 3:56:20 PM]


MyPasTest

Main Navigation
Question Browser: MRCP 1
Home
Subscriptions Question Browser Exam Builder Saved Exams
» MRCP 1 Session Progress
• Question Browser
Questions Correct 5
• Timed Test A 65-year-old man with severe rheumatoid arthritis (RA) is
admitted with a right pleural effusion. He has been Questions Incorrect 2
• Mock Exam
complaining of dyspnoea on exertion for the last three Questions Total 7
• Past Papers months. He has never smoked and has not worked for over
Questions Percentage 71 %
• Random Questions 20 years when he was diagnosed to be suffering from
rheumatoid arthritis. Which of the following is true? More
• My Performance
• Media Bank Pleural effusions with rheumatoid arthritis occur in
• New Multimedia over 50% of patients
Reference: Normal Values
Online Extras A glucose level in pleural fluid of < 1.6 mmol/l is
characteristic of a rheumatoid pleural effusion Haematology
Library
Pleural effusions associated with RA have low levels
Community Immunoglobulins
of cholesterol
Help The most appropriate treatment is chemical Biochemistry
PasTest Store pleurodesis
Diabetes
Bilateral pleural effusions do not occur in RA
Endocrinology
My Account
243 Blood gases
Profile
CSF
Newsletters
My Career and Exams
Order History
Learning Goals
Question Filters

Security

Change Password
Sign Out

© 2011 PasTest Ltd | About Us | Contact Us | Help

file:///E|/Shakhawan/Respiratory%20S/9.htm[3/13/2012 3:56:21 PM]


MyPasTest

Main Navigation
Question Browser: MRCP 1
Home
Subscriptions Question Browser Exam Builder Saved Exams
» MRCP 1 Session Progress
• Question Browser
Questions Correct 5
• Timed Test A 65-year-old man with severe rheumatoid arthritis (RA) is
admitted with a right pleural effusion. He has been Questions Incorrect 3
• Mock Exam
complaining of dyspnoea on exertion for the last three Questions Total 8
• Past Papers months. He has never smoked and has not worked for over
Questions Percentage 62 %
• Random Questions 20 years when he was diagnosed to be suffering from
rheumatoid arthritis. Which of the following is true? More
• My Performance
• Media Bank Pleural effusions with rheumatoid
• New Multimedia arthritis occur in over 50% of
patients
Online Extras
A glucose level in pleural fluid of < Correct answer
Library 1.6 mmol/l is characteristic of a
Community rheumatoid pleural effusion
Help Pleural effusions associated with RA Your answer
have low levels of cholesterol
PasTest Store
The most appropriate treatment is
chemical pleurodesis
My Account Bilateral pleural effusions do not
Profile occur in RA

Newsletters
My Career and Exams
Order History Reference: Normal Values
Some 5% of patients with RA develop pleural effusions.
Learning Goals These mainly occur in older male patients with Haematology
Question Filters subcutaneous nodules. In 25% of patients it may be
bilateral. These effusions are characterised by a low Immunoglobulins
glucose (< 1.6 mmol/l), high LDH (> 700 IU/l), a low
Biochemistry
Security pH (< 7.2) and a high rheumatoid factor titre (>
1:320). Another feature is that they may have high Diabetes
Change Password cholesterol levels. The majority of these effusions
Sign Out resolve spontaneously within three months. Occasionally Endocrinology
they may persist and in such patients massive pleural
thickening develops. These patients may need Blood gases
decortication if they are symptomatic. CSF

243

© 2011 PasTest Ltd | About Us | Contact Us | Help

file:///E|/Shakhawan/Respiratory%20S/9a.htm[3/13/2012 3:56:23 PM]


MyPasTest

Main Navigation
Question Browser: MRCP 1
Home
Subscriptions Question Browser Exam Builder Saved Exams
» MRCP 1 Session Progress
• Question Browser
Questions Correct 5
• Timed Test A 45-year-old woman known to suffer from systemic lupus
erythematosus (SLE) is referred to the chest clinic with a Questions Incorrect 3
• Mock Exam
history of worsening dyspnoea. She is on a maintenance Questions Total 8
• Past Papers dose of 15 mg of prednisolone. She has never smoked, and
Questions Percentage 62 %
• Random Questions according to her she developed an unproductive cough and
dyspnoea three months ago. There is no history of fever, More
• My Performance
night sweats or weight loss. Her doctor has tried various
• Media Bank types of inhalers with no benefit. A PEFR diary shows no
• New Multimedia nocturnal dips. A chest X-ray shows a reticulonodular
pattern. Examination is unremarkable apart from the skin Reference: Normal Values
Online Extras changes of SLE. What is the most likely diagnosis?
Haematology
Library
Asthma Immunoglobulins
Community
Bronchiectasis
Help Biochemistry
Bronchiolitis obliterans
PasTest Store Diabetes
Tuberculosis
Fungal infection Endocrinology
My Account
Blood gases
Profile 244
CSF
Newsletters
My Career and Exams
Order History
Learning Goals
Question Filters

Security

Change Password
Sign Out

© 2011 PasTest Ltd | About Us | Contact Us | Help

file:///E|/Shakhawan/Respiratory%20S/10.htm[3/13/2012 3:56:24 PM]


MyPasTest

Main Navigation
Question Browser: MRCP 1
Home
Subscriptions Question Browser Exam Builder Saved Exams
» MRCP 1 Session Progress
• Question Browser
Questions Correct 6
• Timed Test A 45-year-old woman known to suffer from systemic lupus
erythematosus (SLE) is referred to the chest clinic with a Questions Incorrect 3
• Mock Exam
history of worsening dyspnoea. She is on a maintenance Questions Total 9
• Past Papers dose of 15 mg of prednisolone. She has never smoked, and
Questions Percentage 66 %
• Random Questions according to her she developed an unproductive cough and
dyspnoea three months ago. There is no history of fever, More
• My Performance
night sweats or weight loss. Her doctor has tried various
• Media Bank types of inhalers with no benefit. A PEFR diary shows no
• New Multimedia nocturnal dips. A chest X-ray shows a reticulonodular
pattern. Examination is unremarkable apart from the skin
Online Extras changes of SLE. What is the most likely diagnosis?
Library
Asthma
Community
Bronchiectasis
Help
Bronchiolitis obliterans Your answer
PasTest Store
Tuberculosis
Fungal infection
My Account

Profile
Newsletters
My Career and Exams ‘Bronchiolitis obliterans’ is the term used to describe
Order History fibrous scarring of the small airways. It is seen Reference: Normal Values
following: toxic-fume inhalation; mineral-dust
Learning Goals exposure; viral infection; mycoplasma and legionella Haematology
Question Filters infection; bone marrow, heart–lung and lung
transplantation; rheumatoid arthritis; SLE; and as a Immunoglobulins
side-effect of penicillamine. It presents as a dry cough
Biochemistry
Security and dyspnoea. Physical examination is unremarkable.
Expiratory wheeze may be audible. The chest X-ray Diabetes
Change Password findings can vary from normal to a reticular or
Sign Out reticulonodular pattern. The diagnosis can be confirmed Endocrinology
by lung biopsy. Patients rarely respond to steroids. The
prognosis is poor. Blood gases

CSF

244

© 2011 PasTest Ltd | About Us | Contact Us | Help

file:///E|/Shakhawan/Respiratory%20S/10a.htm[3/13/2012 3:56:26 PM]


MyPasTest

Main Navigation
Question Browser: MRCP 1
Home
Subscriptions Question Browser Exam Builder Saved Exams
» MRCP 1 Session Progress
• Question Browser
Questions Correct 6
• Timed Test A 44-year-old man is admitted with right-sided pneumonia.
According to him he has been unwell for three to four days Questions Incorrect 3
• Mock Exam
with malaise, fever, cough and muscular pain. He also has Questions Total 9
• Past Papers a rash on his abdomen and neck pain. He was previously fit
Questions Percentage 66 %
• Random Questions and has not travelled abroad. He is a plumber and also
keeps pigeons. According to his wife, two of his favourite More
• My Performance
pigeons died two weeks ago. Which organism would you
• Media Bank be suspicious of as being responsible for his
• New Multimedia pneumonia?
Reference: Normal Values
Online Extras Streptococcus pneumoniae
Haematology
Library Legionella pneumophila
Community Immunoglobulins
Coxiella burnetii
Help Chlamydia psittaci Biochemistry
PasTest Store Mycoplasma pneumoniae Diabetes

245 Endocrinology
My Account
Blood gases
Profile
CSF
Newsletters
My Career and Exams
Order History
Learning Goals
Question Filters

Security

Change Password
Sign Out

© 2011 PasTest Ltd | About Us | Contact Us | Help

file:///E|/Shakhawan/Respiratory%20S/11.htm[3/13/2012 3:56:27 PM]


MyPasTest

Main Navigation
Question Browser: MRCP 1
Home
Subscriptions Question Browser Exam Builder Saved Exams
» MRCP 1 Session Progress
• Question Browser
Questions Correct 7
• Timed Test A 44-year-old man is admitted with right-sided pneumonia.
According to him he has been unwell for three to four days Questions Incorrect 3
• Mock Exam
with malaise, fever, cough and muscular pain. He also has Questions Total 10
• Past Papers a rash on his abdomen and neck pain. He was previously fit
Questions Percentage 70 %
• Random Questions and has not travelled abroad. He is a plumber and also
keeps pigeons. According to his wife, two of his favourite More
• My Performance
pigeons died two weeks ago. Which organism would you
• Media Bank be suspicious of as being responsible for his
• New Multimedia pneumonia?

Online Extras Streptococcus pneumoniae


Library Legionella pneumophila
Community Coxiella burnetii
Help Chlamydia psittaci Your answer
PasTest Store Mycoplasma pneumoniae

My Account

Profile
Chlamydia psittaci (psittacosis) is characterised by
Newsletters
malaise, fever, myalgias and pneumonia. Exposure to
My Career and Exams an ill bird and a rash (Horder’s spots) are
Order History pathognomonic. In the context of his history, Chlamydia Reference: Normal Values
would be very high on the index of suspicion.
Learning Goals Erythromycin or tetracyclines are the drugs of choice. Haematology
Question Filters
Immunoglobulins
245 Biochemistry
Security
Diabetes
Change Password
Sign Out Endocrinology

Blood gases

CSF

© 2011 PasTest Ltd | About Us | Contact Us | Help

file:///E|/Shakhawan/Respiratory%20S/11a.htm[3/13/2012 3:56:29 PM]


MyPasTest

Main Navigation
Question Browser: MRCP 1
Home
Subscriptions Question Browser Exam Builder Saved Exams
» MRCP 1 Session Progress
• Question Browser
Questions Correct 7
• Timed Test A 47-year-old man presents to hospital with acute
breathlessness. His arterial blood gases show a p a (O2 ) of Questions Incorrect 3
• Mock Exam
8.5 kPa (65 mmHg) and a p a (CO 2 ) of 5.6 kPa (42.5 Questions Total 10
• Past Papers
mmHg) whilst breathing room air. Questions Percentage 70 %
• Random Questions
• My Performance Which of the following is the least likely explanation More
for this abnormality?
• Media Bank
• New Multimedia Ventilation/perfusion (VQ) mismatch Reference: Normal Values
Online Extras Right-to-left shunting
Haematology
Library Hypoventilation
Community Diffusion abnormality Immunoglobulins
Help Anaemia Biochemistry
PasTest Store Diabetes
341
Endocrinology
My Account
Blood gases
Profile
CSF
Newsletters
My Career and Exams
Order History
Learning Goals
Question Filters

Security

Change Password
Sign Out

© 2011 PasTest Ltd | About Us | Contact Us | Help

file:///E|/Shakhawan/Respiratory%20S/12.htm[3/13/2012 3:56:30 PM]


MyPasTest

Main Navigation
Question Browser: MRCP 1
Home
Subscriptions Question Browser Exam Builder Saved Exams
» MRCP 1 Session Progress
• Question Browser
Questions Correct 7
• Timed Test A 47-year-old man presents to hospital with acute
breathlessness. His arterial blood gases show a p a (O2 ) of Questions Incorrect 4
• Mock Exam
8.5 kPa (65 mmHg) and a p a (CO 2 ) of 5.6 kPa (42.5 Questions Total 11
• Past Papers
mmHg) whilst breathing room air. Questions Percentage 63 %
• Random Questions
• My Performance Which of the following is the least likely explanation More
for this abnormality?
• Media Bank
• New Multimedia Ventilation/perfusion (VQ) mismatch
Online Extras Right-to-left shunting
Library Hypoventilation Correct answer
Community Diffusion abnormality Your answer
Help Anaemia
PasTest Store

My Account
The Alveolar–arterial (A–a) oxygen gradient is one way
Profile to find the answer to this question, although the clinical
Newsletters history of breathlessness does make hypoventilation
seem an unlikely answer.
My Career and Exams
Order History p A(O2 ) = FiO 2 – (pa (CO 2 )/0.8) = 21 – (5.6/0.8) = 14 Reference: Normal Values
Learning Goals kPa Haematology
Question Filters
Therefore, A–a gradient = 14 – 8.5 = 5.5 kPa (42 Immunoglobulins
mmHg), the normal being approximately 0.5–2.0 kPa
Biochemistry
Security (< 15 mmHg). VQ mismatch, diffusion abnormalities
and right-to-left shunting all cause widening of the A–a Diabetes
Change Password gradient. Hypoventilation causes hypoxaemia with a
Sign Out normal A–a gradient and is therefore the most unlikely Endocrinology
explanation in this case.
Blood gases

CSF
341

© 2011 PasTest Ltd | About Us | Contact Us | Help

file:///E|/Shakhawan/Respiratory%20S/12a.htm[3/13/2012 3:56:32 PM]


MyPasTest

Main Navigation
Question Browser: MRCP 1
Home
Subscriptions Question Browser Exam Builder Saved Exams
» MRCP 1 Session Progress
• Question Browser
Questions Correct 7
• Timed Test A 53-year-old woman with end-stage renal failure develops
pulmonary tuberculosis. Which one of the following Questions Incorrect 4
• Mock Exam
drugs should be used in a reduced dose? Questions Total 11
• Past Papers
Questions Percentage 63 %
• Random Questions Rifampicin
• My Performance Isoniazid More
• Media Bank Pyrazinamide
• New Multimedia Ethambutol
Reference: Normal Values
Online Extras Pyridoxine
Haematology
Library
342
Community Immunoglobulins
Help Biochemistry
PasTest Store Diabetes
Endocrinology
My Account
Blood gases
Profile
CSF
Newsletters
My Career and Exams
Order History
Learning Goals
Question Filters

Security

Change Password
Sign Out

© 2011 PasTest Ltd | About Us | Contact Us | Help

file:///E|/Shakhawan/Respiratory%20S/13.htm[3/13/2012 3:56:33 PM]


MyPasTest

Main Navigation
Question Browser: MRCP 1
Home
Subscriptions Question Browser Exam Builder Saved Exams
» MRCP 1 Session Progress
• Question Browser
Questions Correct 8
• Timed Test A 53-year-old woman with end-stage renal failure develops
pulmonary tuberculosis. Which one of the following Questions Incorrect 4
• Mock Exam
drugs should be used in a reduced dose? Questions Total 12
• Past Papers
Questions Percentage 66 %
• Random Questions Rifampicin
• My Performance Isoniazid More
• Media Bank Pyrazinamide
• New Multimedia Ethambutol Your answer
Online Extras Pyridoxine
Library
Community
Help
PasTest Store Ethambutol is renally excreted and therefore dose
adjustment is necessary to minimise the risk of toxic
effects, ie optic neuropathy. Pyridoxine is a vitamin
My Account supplement given with isoniazid to minimise the risk of
peripheral neuropathy. The remaining drugs are mainly
Profile metabolised in the liver and may be given in normal
doses in renal failure.
Newsletters
My Career and Exams
Order History 342 Reference: Normal Values
Learning Goals Haematology
Question Filters
Immunoglobulins
Biochemistry
Security
Diabetes
Change Password
Sign Out Endocrinology

Blood gases

CSF

© 2011 PasTest Ltd | About Us | Contact Us | Help

file:///E|/Shakhawan/Respiratory%20S/13a.htm[3/13/2012 3:56:35 PM]


MyPasTest

Main Navigation
Question Browser: MRCP 1
Home
Subscriptions Question Browser Exam Builder Saved Exams
» MRCP 1 Session Progress
• Question Browser
Questions Correct 8
• Timed Test A 36-year-old woman with systemic sclerosis develops
breathlessness on exertion. Her pulmonary function tests Questions Incorrect 4
• Mock Exam
show normal spirometry but a decreased gas transfer factor Questions Total 12
• Past Papers (TLCO, transfer factor for carbon monoxide) and transfer
Questions Percentage 66 %
• Random Questions coefficient (KCO).
• My Performance More
Which of the following is the most likely explanation
• Media Bank for this abnormality?
• New Multimedia
Fibrosing alveolitis Reference: Normal Values
Online Extras
Pulmonary vascular disease Haematology
Library
Severe thoracic skin thickening
Community Immunoglobulins
Pleural involvement
Help Biochemistry
Respiratory muscle weakness
PasTest Store Diabetes
343
Endocrinology
My Account
Blood gases
Profile
CSF
Newsletters
My Career and Exams
Order History
Learning Goals
Question Filters

Security

Change Password
Sign Out

© 2011 PasTest Ltd | About Us | Contact Us | Help

file:///E|/Shakhawan/Respiratory%20S/14.htm[3/13/2012 3:56:36 PM]


MyPasTest

Main Navigation
Question Browser: MRCP 1
Home
Subscriptions Question Browser Exam Builder Saved Exams
» MRCP 1 Session Progress
• Question Browser
Questions Correct 8
• Timed Test A 36-year-old woman with systemic sclerosis develops
breathlessness on exertion. Her pulmonary function tests Questions Incorrect 5
• Mock Exam
show normal spirometry but a decreased gas transfer factor Questions Total 13
• Past Papers (TLCO, transfer factor for carbon monoxide) and transfer
Questions Percentage 61 %
• Random Questions coefficient (KCO).
• My Performance More
Which of the following is the most likely explanation
• Media Bank for this abnormality?
• New Multimedia
Fibrosing alveolitis Your answer
Online Extras
Pulmonary vascular disease Correct answer
Library
Severe thoracic skin thickening
Community
Pleural involvement
Help
Respiratory muscle weakness
PasTest Store

My Account

Profile Isolated decreases in gas transfer are typical of


pulmonary vascular disease, eg vasculitis, recurrent
Newsletters
pulmonary embolism (PE). In fibrosing alveolitis you
My Career and Exams would also expect to see decreased lung volumes with a
Order History restrictive ratio (> 80%) on spirometry. The other three Reference: Normal Values
answers would all give a picture of ‘extrapulmonary
Learning Goals restriction’ with a restrictive ratio, low TLCO but Haematology
Question Filters normal/high KCO (same cardiac output going through
smaller alveolar volume). Immunoglobulins
Biochemistry
Security
343 Diabetes
Change Password
Sign Out Endocrinology

Blood gases

CSF

© 2011 PasTest Ltd | About Us | Contact Us | Help

file:///E|/Shakhawan/Respiratory%20S/14a.htm[3/13/2012 3:56:37 PM]


MyPasTest

Main Navigation
Question Browser: MRCP 1
Home
Subscriptions Question Browser Exam Builder Saved Exams
» MRCP 1 Session Progress
• Question Browser
Questions Correct 8
• Timed Test A 60-year-old man is referred following a chest X-ray that
suggests interstitial lung disease. You proceed to Questions Incorrect 5
• Mock Exam
bronchoscopy with transbronchial lung biopsy to try and Questions Total 13
• Past Papers make a definitive histological diagnosis. Which of the
Questions Percentage 61 %
• Random Questions following is the least likely diagnosis to be confirmed
in this way? More
• My Performance
• Media Bank Cryptogenic fibrosing alveolitis (CFA)
• New Multimedia Sarcoidosis Reference: Normal Values
Online Extras Extrinsic allergic alveolitis
Haematology
Library Cryptogenic organising pneumonia (COP)
Community Lymphangitis carcinomatosa Immunoglobulins
Help Biochemistry
344
PasTest Store Diabetes
Endocrinology
My Account
Blood gases
Profile
CSF
Newsletters
My Career and Exams
Order History
Learning Goals
Question Filters

Security

Change Password
Sign Out

© 2011 PasTest Ltd | About Us | Contact Us | Help

file:///E|/Shakhawan/Respiratory%20S/15.htm[3/13/2012 3:56:39 PM]


MyPasTest

Main Navigation
Question Browser: MRCP 1
Home
Subscriptions Question Browser Exam Builder Saved Exams
» MRCP 1 Session Progress
• Question Browser
Questions Correct 8
• Timed Test A 60-year-old man is referred following a chest X-ray that
suggests interstitial lung disease. You proceed to Questions Incorrect 6
• Mock Exam
bronchoscopy with transbronchial lung biopsy to try and Questions Total 14
• Past Papers make a definitive histological diagnosis. Which of the
Questions Percentage 57 %
• Random Questions following is the least likely diagnosis to be confirmed
in this way? More
• My Performance
• Media Bank Cryptogenic fibrosing alveolitis Correct answer
• New Multimedia (CFA)
Online Extras Sarcoidosis
Library Extrinsic allergic alveolitis
Community Cryptogenic organising pneumonia Your answer
(COP)
Help
Lymphangitis carcinomatosa
PasTest Store

My Account

Profile CFA can be a patchy disease and transbronchial


bronchoscopy (TBBx) may simply reveal established
Newsletters
fibrosis with no particular aetiology suggested. If a
My Career and Exams biopsy is required in CFA then a surgical lung biopsy,
Order History guided by high-resolution computed tomography Reference: Normal Values
(HRCT) appearances, will give better specimens.
Learning Goals Sarcoidosis and EAA are both suggested by granulomas, Haematology
Question Filters lymphangitis by malignant cells and COP by the
presence of organising pneumonia. Immunoglobulins
Biochemistry
Security
344 Diabetes
Change Password
Sign Out Endocrinology

Blood gases

CSF

© 2011 PasTest Ltd | About Us | Contact Us | Help

file:///E|/Shakhawan/Respiratory%20S/15a.htm[3/13/2012 3:56:40 PM]


MyPasTest

Main Navigation
Question Browser: MRCP 1
Home
Subscriptions Question Browser Exam Builder Saved Exams
» MRCP 1 Session Progress
• Question Browser
Questions Correct 8
• Timed Test A 68-year-old man with bronchiectasis is found to have
acid-fast bacilli in his sputum. The microbiology report Questions Incorrect 6
• Mock Exam
suggests this may be an opportunistic or atypical Questions Total 14
• Past Papers mycobacterium. Which of the following is the least
Questions Percentage 57 %
• Random Questions likely infectious agent?
• My Performance More
Mycobacterium kansasii
• Media Bank
Mycobacterium malmoense
• New Multimedia
Mycobacterium xenopi Reference: Normal Values
Online Extras
Mycobacterium leprae Haematology
Library
Mycobacterium avium intracellulare
Community Immunoglobulins
Help 345 Biochemistry
PasTest Store Diabetes
Endocrinology
My Account
Blood gases
Profile
CSF
Newsletters
My Career and Exams
Order History
Learning Goals
Question Filters

Security

Change Password
Sign Out

© 2011 PasTest Ltd | About Us | Contact Us | Help

file:///E|/Shakhawan/Respiratory%20S/16.htm[3/13/2012 3:56:42 PM]


MyPasTest

Main Navigation
Question Browser: MRCP 1
Home
Subscriptions Question Browser Exam Builder Saved Exams
» MRCP 1 Session Progress
• Question Browser
Questions Correct 9
• Timed Test A 68-year-old man with bronchiectasis is found to have
acid-fast bacilli in his sputum. The microbiology report Questions Incorrect 6
• Mock Exam
suggests this may be an opportunistic or atypical Questions Total 15
• Past Papers mycobacterium. Which of the following is the least
Questions Percentage 60 %
• Random Questions likely infectious agent?
• My Performance More
Mycobacterium kansasii
• Media Bank
Mycobacterium malmoense
• New Multimedia
Mycobacterium xenopi
Online Extras
Mycobacterium leprae Your answer
Library
Mycobacterium avium intracellulare
Community
Help
PasTest Store
Mycobacterium leprae is the causative organism of
My Account leprosy, whereas the rest are all ‘opportunistic’
mycobacteria, i.e. they may colonise structurally
Profile abnormal lung such as seen in patients with cavitary
disease, bronchiectasis or COPD. Such patients may not
Newsletters
always require treatment. However, if treatment is
My Career and Exams required, then it is usually for longer than the standard
Order History 6 months needed to treat pulmonary TB – but (for the Reference: Normal Values
MRCP exam) you do not need to worry about complex
Learning Goals treatment regimens. Haematology
Question Filters
Immunoglobulins
345 Biochemistry
Security
Diabetes
Change Password
Sign Out Endocrinology

Blood gases

CSF

© 2011 PasTest Ltd | About Us | Contact Us | Help

file:///E|/Shakhawan/Respiratory%20S/16a.htm[3/13/2012 3:56:43 PM]


MyPasTest

Main Navigation
Question Browser: MRCP 1
Home
Subscriptions Question Browser Exam Builder Saved Exams
» MRCP 1 Session Progress
• Question Browser
Questions Correct 9
• Timed Test The anatomical dead space may be used to calculate
alveolar ventilation by subtracting it from the tidal volume Questions Incorrect 6
• Mock Exam
and multiplying the result by the respiratory rate. What Questions Total 15
• Past Papers would you expect the normal anatomical dead space
Questions Percentage 60 %
• Random Questions to be in a healthy adult male?
• My Performance More
50 ml
• Media Bank
150 ml
• New Multimedia
250 ml Reference: Normal Values
Online Extras
350 ml Haematology
Library
450 ml
Community Immunoglobulins
Help 346 Biochemistry
PasTest Store Diabetes
Endocrinology
My Account
Blood gases
Profile
CSF
Newsletters
My Career and Exams
Order History
Learning Goals
Question Filters

Security

Change Password
Sign Out

© 2011 PasTest Ltd | About Us | Contact Us | Help

file:///E|/Shakhawan/Respiratory%20S/17.htm[3/13/2012 3:56:45 PM]


MyPasTest

Main Navigation
Question Browser: MRCP 1
Home
Subscriptions Question Browser Exam Builder Saved Exams
» MRCP 1 Session Progress
• Question Browser
Questions Correct 10
• Timed Test The anatomical dead space may be used to calculate
alveolar ventilation by subtracting it from the tidal volume Questions Incorrect 6
• Mock Exam
and multiplying the result by the respiratory rate. What Questions Total 16
• Past Papers would you expect the normal anatomical dead space
Questions Percentage 62 %
• Random Questions to be in a healthy adult male?
• My Performance More
50 ml
• Media Bank
150 ml Your answer
• New Multimedia
250 ml
Online Extras
350 ml
Library
450 ml
Community
Help
PasTest Store
The normal anatomical dead space is approximately 150
My Account ml. If we take tidal volume to be about 500 ml and
respiratory rate to be about 15/minute, this gives a
Profile normal alveolar ventilation of (500 – 150) × 15 = 5250
ml/min. Diseases that cause an additional physiological
Newsletters
dead space where parts of lung do not take part in gas
My Career and Exams exchange, eg pneumonia, may increase this dead space.
Order History Reference: Normal Values
Learning Goals 346 Haematology
Question Filters
Immunoglobulins
Biochemistry
Security
Diabetes
Change Password
Sign Out Endocrinology

Blood gases

CSF

© 2011 PasTest Ltd | About Us | Contact Us | Help

file:///E|/Shakhawan/Respiratory%20S/17a.htm[3/13/2012 3:56:46 PM]


MyPasTest

Main Navigation
Question Browser: MRCP 1
Home
Subscriptions Question Browser Exam Builder Saved Exams
» MRCP 1 Session Progress
• Question Browser
Questions Correct 10
• Timed Test A 52-year-old woman with chronic obstructive pulmonary
disease is assessed for long-term oxygen therapy (LTOT). Questions Incorrect 6
• Mock Exam
She is found to be suitable for LTOT. What is the minimum Questions Total 16
• Past Papers number of hours per day that she should be using the
Questions Percentage 62 %
• Random Questions oxygen?
• My Performance More
3
• Media Bank
5
• New Multimedia
10 Reference: Normal Values
Online Extras
15 Haematology
Library
20
Community Immunoglobulins
Help 347 Biochemistry
PasTest Store Diabetes
Endocrinology
My Account
Blood gases
Profile
CSF
Newsletters
My Career and Exams
Order History
Learning Goals
Question Filters

Security

Change Password
Sign Out

© 2011 PasTest Ltd | About Us | Contact Us | Help

file:///E|/Shakhawan/Respiratory%20S/18.htm[3/13/2012 3:56:48 PM]


MyPasTest

Main Navigation
Question Browser: MRCP 1
Home
Subscriptions Question Browser Exam Builder Saved Exams
» MRCP 1 Session Progress
• Question Browser
Questions Correct 10
• Timed Test A 52-year-old woman with chronic obstructive pulmonary
disease is assessed for long-term oxygen therapy (LTOT). Questions Incorrect 7
• Mock Exam
She is found to be suitable for LTOT. What is the minimum Questions Total 17
• Past Papers number of hours per day that she should be using the
Questions Percentage 58 %
• Random Questions oxygen?
• My Performance More
3
• Media Bank
5
• New Multimedia
10 Your answer
Online Extras
15 Correct answer
Library
20
Community
Help
PasTest Store
Long-term oxygen therapy

My Account In the early 1980s two large studies (MRC and NOTT)
concluded that the minimum duration of long-term
Profile oxygen therapy (LTOT) should be 15 hours/day at a
flow rate that keeps the arterial Po 2 above 8.0 kPa (60
Newsletters
mmHg). At 3 years, survival was shown to be 50%
My Career and Exams better in the LTOT group compared with conventional
Order History treatment alone. Reference: Normal Values
Learning Goals Indications for LTOT Haematology
Question Filters
Two arterial blood gas measurements should be made Immunoglobulins
at least 3 weeks apart. Indications in for LTOT in
Biochemistry
Security patients with chronic obstructive pulmonary disease are:
Diabetes
Change Password Pao2 on air < 7.3 kPa with a normal/elevated
Sign Out Paco2 and an FEV1 < 1.5 l Endocrinology
Pa o 2 7.3–8.0 kPa with evidence of cor Blood gases
pulmonale, peripheral oedema or nocturnal
hypoxaemia CSF

347

© 2011 PasTest Ltd | About Us | Contact Us | Help

file:///E|/Shakhawan/Respiratory%20S/18a.htm[3/13/2012 3:56:49 PM]


MyPasTest

Main Navigation
Question Browser: MRCP 1
Home
Subscriptions Question Browser Exam Builder Saved Exams
» MRCP 1 Session Progress
• Question Browser
Questions Correct 10
• Timed Test A 62-year-old man presents with flushing and wheezing. A
mass is seen on his chest radiograph and you suspect that Questions Incorrect 7
• Mock Exam
he may have a carcinoid tumour. Questions Total 17
• Past Papers Which of the following features is LEAST expected in
Questions Percentage 58 %
• Random Questions a diagnosis of carcinoid?
• My Performance More
Histological appearance similar to that of a small-cell
• Media Bank carcinoma
• New Multimedia Highly vascular appearance Reference: Normal Values
Online Extras Normal bronchoscopic examination
Haematology
Library Less than 40% probability of developing carcinoid
Community syndrome Immunoglobulins
Help 90% 5–year survival post-surgery Biochemistry
PasTest Store 348 Diabetes
Endocrinology
My Account
Blood gases
Profile
CSF
Newsletters
My Career and Exams
Order History
Learning Goals
Question Filters

Security

Change Password
Sign Out

© 2011 PasTest Ltd | About Us | Contact Us | Help

file:///E|/Shakhawan/Respiratory%20S/19.htm[3/13/2012 3:56:51 PM]


MyPasTest

Main Navigation
Question Browser: MRCP 1
Home
Subscriptions Question Browser Exam Builder Saved Exams
» MRCP 1 Session Progress
• Question Browser
Questions Correct 10
• Timed Test A 62-year-old man presents with flushing and wheezing. A
mass is seen on his chest radiograph and you suspect that Questions Incorrect 8
• Mock Exam
he may have a carcinoid tumour. Questions Total 18
• Past Papers Which of the following features is LEAST expected in
Questions Percentage 55 %
• Random Questions a diagnosis of carcinoid?
• My Performance More
Histological appearance similar to
• Media Bank that of a small-cell carcinoma
• New Multimedia Highly vascular appearance
Online Extras Normal bronchoscopic examination Correct answer
Library Less than 40% probability of Your answer
Community developing carcinoid syndrome
Help 90% 5–year survival post-surgery
PasTest Store

My Account
Carcinoid tumours make up 1% of all lung tumours and
Profile 60% are visible in the bronchial tree. They originate
from the APUD cell line. Typically, they are very
Newsletters
vascular structures that on bronchoscopy appear as
My Career and Exams cherry-red balls that tend to bleed easily. Only a small
Order History percentage develops carcinoid syndrome (flushing, Reference: Normal Values
cramp, diarrhoea and wheezing). Histologically, they
Learning Goals may resemble small-cell carcinoma. With surgery the 5- Haematology
Question Filters year survival is 90% and 10-year survival is 75%.
Immunoglobulins
Biochemistry
Security 348
Diabetes
Change Password
Sign Out Endocrinology

Blood gases

CSF

© 2011 PasTest Ltd | About Us | Contact Us | Help

file:///E|/Shakhawan/Respiratory%20S/19a.htm[3/13/2012 3:56:52 PM]


MyPasTest

Main Navigation
Question Browser: MRCP 1
Home
Subscriptions Question Browser Exam Builder Saved Exams
» MRCP 1 Session Progress
• Question Browser
Questions Correct 10
• Timed Test A 58-year-old man with a heavy previous asbestos-
exposure history smokes 20 cigarettes per day and has Questions Incorrect 8
• Mock Exam
done since he was 17 years old. Questions Total 18
• Past Papers
What is his increased risk of developing bronchial Questions Percentage 55 %
• Random Questions
carcinoma compared to a lifelong non-smoker, never More
• My Performance exposed to asbestos?
• Media Bank
• New Multimedia 100 times
Reference: Normal Values
Online Extras 50 times
20 times Haematology
Library
Community 10 times Immunoglobulins
Help 2 times
Biochemistry
PasTest Store 349 Diabetes
Endocrinology
My Account
Blood gases
Profile
CSF
Newsletters
My Career and Exams
Order History
Learning Goals
Question Filters

Security

Change Password
Sign Out

© 2011 PasTest Ltd | About Us | Contact Us | Help

file:///E|/Shakhawan/Respiratory%20S/20.htm[3/13/2012 3:56:54 PM]


MyPasTest

Main Navigation
Question Browser: MRCP 1
Home
Subscriptions Question Browser Exam Builder Saved Exams
» MRCP 1 Session Progress
• Question Browser
Questions Correct 10
• Timed Test A 58-year-old man with a heavy previous asbestos-
exposure history smokes 20 cigarettes per day and has Questions Incorrect 9
• Mock Exam
done since he was 17 years old. Questions Total 19
• Past Papers
What is his increased risk of developing bronchial Questions Percentage 52 %
• Random Questions
carcinoma compared to a lifelong non-smoker, never More
• My Performance exposed to asbestos?
• Media Bank
• New Multimedia 100 times Your answer

Online Extras 50 times Correct answer

Library 20 times

Community 10 times

Help 2 times

PasTest Store

My Account
Asbestos exposure increases the risk of developing
Profile bronchial carcinoma 5 times, whereas smoking
increases the risk approximately 10–11 times. However,
Newsletters if a patient has both risk factors then the risk is
My Career and Exams multiplicative and is approximately 50 times greater
than a lifelong non-smoker never exposed to asbestos. Reference: Normal Values
Order History
Learning Goals Haematology
Question Filters 349
Immunoglobulins
Biochemistry
Security
Diabetes
Change Password
Sign Out Endocrinology

Blood gases

CSF

© 2011 PasTest Ltd | About Us | Contact Us | Help

file:///E|/Shakhawan/Respiratory%20S/20a.htm[3/13/2012 3:56:55 PM]


MyPasTest

Main Navigation
Question Browser: MRCP 1
Home
Subscriptions Question Browser Exam Builder Saved Exams
» MRCP 1 Session Progress
• Question Browser
Questions Correct 10
• Timed Test A 43-year-old woman is referred by her general
practitioner with a productive cough and inspiratory Questions Incorrect 9
• Mock Exam
crackles at the left base. Questions Total 19
• Past Papers
Which of the following is considered to be a core Questions Percentage 52 %
• Random Questions
adverse prognostic factor? More
• My Performance
• Media Bank Respiratory rate of 28/minute
• New Multimedia Blood pressure of 98/65 mmHg
Reference: Normal Values
Online Extras Serum urea concentration of 7.1 mmol/l
Haematology
Library Oxygen saturation of 92% on room air
Community Bilateral changes on chest radiograph Immunoglobulins
Help Biochemistry
350
PasTest Store Diabetes
Endocrinology
My Account
Blood gases
Profile
CSF
Newsletters
My Career and Exams
Order History
Learning Goals
Question Filters

Security

Change Password
Sign Out

© 2011 PasTest Ltd | About Us | Contact Us | Help

file:///E|/Shakhawan/Respiratory%20S/21.htm[3/13/2012 3:56:56 PM]


MyPasTest

Main Navigation
Question Browser: MRCP 1
Home
Subscriptions Question Browser Exam Builder Saved Exams
» MRCP 1 Session Progress
• Question Browser
Questions Correct 10
• Timed Test A 43-year-old woman is referred by her general
practitioner with a productive cough and inspiratory Questions Incorrect 10
• Mock Exam
crackles at the left base. Questions Total 20
• Past Papers
Which of the following is considered to be a core Questions Percentage 50 %
• Random Questions
adverse prognostic factor? More
• My Performance
• Media Bank Respiratory rate of 28/minute Your answer
• New Multimedia Blood pressure of 98/65 mmHg
Online Extras Serum urea concentration of 7.1 Correct answer
Library mmol/l

Community Oxygen saturation of 92% on room


air
Help
Bilateral changes on chest
PasTest Store radiograph

My Account

Profile
Core clinical adverse prognostic factors are given by the
Newsletters abbreviation ‘CURB’: Confusion (new onset) with a Mini-
My Career and Exams Mental Test score < 8; Urea > 7.0 mmol/l; Respiratory
rate > 30/minute; Blood pressure – systolic < 90 Reference: Normal Values
Order History
mmHg or diastolic < 60 mmHg. If any core clinical
Learning Goals factors are present then the patient is at increased risk Haematology
Question Filters of death and should not be sent home (British Thoracic
Society Guidelines, December 2001). Immunoglobulins
Biochemistry
Security
350 Diabetes
Change Password
Sign Out Endocrinology

Blood gases

CSF

© 2011 PasTest Ltd | About Us | Contact Us | Help

file:///E|/Shakhawan/Respiratory%20S/21a.htm[3/13/2012 3:56:58 PM]


MyPasTest

Main Navigation
Question Browser: MRCP 1
Home
Subscriptions Question Browser Exam Builder Saved Exams
» MRCP 1 Session Progress
• Question Browser
Questions Correct 10
• Timed Test A 56-year-old man has a chest X-ray performed as he has
become breathless on exertion and has inspiratory crackles. Questions Incorrect 10
• Mock Exam
The chest X-ray reveals upper lobe lung fibrosis. Questions Total 20
• Past Papers
Which of the following is the most likely explanation? Questions Percentage 50 %
• Random Questions
• My Performance More
Cryptogenic fibrosing alveolitis
• Media Bank
Langerhans' cell histiocytosis
• New Multimedia
Asbestosis Reference: Normal Values
Online Extras
Connective tissue fibrosing alveolitis
Haematology
Library
Drug-induced pulmonary fibrosis
Community Immunoglobulins
Help 351 Biochemistry
PasTest Store Diabetes
Endocrinology
My Account
Blood gases
Profile
CSF
Newsletters
My Career and Exams
Order History
Learning Goals
Question Filters

Security

Change Password
Sign Out

© 2011 PasTest Ltd | About Us | Contact Us | Help

file:///E|/Shakhawan/Respiratory%20S/22.htm[3/13/2012 3:56:59 PM]


MyPasTest

Main Navigation
Question Browser: MRCP 1
Home
Subscriptions Question Browser Exam Builder Saved Exams
» MRCP 1 Session Progress
• Question Browser
Questions Correct 10
• Timed Test A 56-year-old man has a chest X-ray performed as he has
become breathless on exertion and has inspiratory crackles. Questions Incorrect 11
• Mock Exam
The chest X-ray reveals upper lobe lung fibrosis. Questions Total 21
• Past Papers
Which of the following is the most likely explanation? Questions Percentage 47 %
• Random Questions
• My Performance More
Cryptogenic fibrosing alveolitis Your answer
• Media Bank
Langerhans' cell histiocytosis Correct answer
• New Multimedia
Asbestosis
Online Extras
Connective tissue fibrosing alveolitis
Library
Drug-induced pulmonary fibrosis
Community
Help
PasTest Store
Upper zone fibrosis typically occurs in TB, extrinsic
My Account allergic alveolitis, sarcoidosis, ankylosing spondylitis,
allergic bronchopulmonary aspergillosis (ABPA) and
Profile farmers lung, pneumoconiosis, histiocytosis and
silicosis. The rest usually result in lower zone fibrosis.
Newsletters However, many of these diseases can, as they progress,
My Career and Exams affect both the upper and lower zones.
Order History Reference: Normal Values
Learning Goals 351 Haematology
Question Filters
Immunoglobulins
Biochemistry
Security
Diabetes
Change Password
Sign Out Endocrinology

Blood gases

CSF

© 2011 PasTest Ltd | About Us | Contact Us | Help

file:///E|/Shakhawan/Respiratory%20S/22a.htm[3/13/2012 3:57:01 PM]


MyPasTest

Main Navigation
Question Browser: MRCP 1
Home
Subscriptions Question Browser Exam Builder Saved Exams
» MRCP 1 Session Progress
• Question Browser
Questions Correct 10
• Timed Test A 37-year-old man who speaks little English comes to the
TB clinic. His notes are missing but he is able to tell you Questions Incorrect 11
• Mock Exam
that he has been on treatment for almost 1 year. Questions Total 21
• Past Papers
What is the most likely reason he has been treated Questions Percentage 47 %
• Random Questions
for this length of time? More
• My Performance
• Media Bank Pulmonary TB
• New Multimedia Lymph-node TB
Reference: Normal Values
Online Extras TB meningitis
Haematology
Library Bony TB
Community Pleural TB Immunoglobulins
Help Biochemistry
352
PasTest Store Diabetes
Endocrinology
My Account
Blood gases
Profile
CSF
Newsletters
My Career and Exams
Order History
Learning Goals
Question Filters

Security

Change Password
Sign Out

© 2011 PasTest Ltd | About Us | Contact Us | Help

file:///E|/Shakhawan/Respiratory%20S/23.htm[3/13/2012 3:57:02 PM]


MyPasTest

Main Navigation
Question Browser: MRCP 1
Home
Subscriptions Question Browser Exam Builder Saved Exams
» MRCP 1 Session Progress
• Question Browser
Questions Correct 10
• Timed Test A 37-year-old man who speaks little English comes to the
TB clinic. His notes are missing but he is able to tell you Questions Incorrect 12
• Mock Exam
that he has been on treatment for almost 1 year. Questions Total 22
• Past Papers
What is the most likely reason he has been treated Questions Percentage 45 %
• Random Questions
for this length of time? More
• My Performance
• Media Bank Pulmonary TB
• New Multimedia Lymph-node TB
Online Extras TB meningitis Correct answer
Library Bony TB Your answer
Community Pleural TB
Help
PasTest Store

My Account In the UK, all fully sensitive TB is treated for 6 months


except for TB affecting the central nervous system,
Profile which is treated for 12 months. Multi-drug resistant TB
(MDRTB) will require longer courses of treatment,
Newsletters usually with a combination of less commonly used anti-
My Career and Exams TB drugs.
Order History Reference: Normal Values
Learning Goals 352 Haematology
Question Filters
Immunoglobulins
Biochemistry
Security
Diabetes
Change Password
Sign Out Endocrinology

Blood gases

CSF

© 2011 PasTest Ltd | About Us | Contact Us | Help

file:///E|/Shakhawan/Respiratory%20S/23a.htm[3/13/2012 3:57:04 PM]


MyPasTest

Main Navigation
Question Browser: MRCP 1
Home
Subscriptions Question Browser Exam Builder Saved Exams
» MRCP 1 Session Progress
• Question Browser
Questions Correct 0
• Timed Test A 33-year-old man is found to have strongly positive
aspergillus precipitins in his blood and complains of a cough Questions Incorrect 0
• Mock Exam
with intermittent bloody sputum. He has a business working Questions Total 0
• Past Papers as a builder, particularly involved in renovating farm houses
Questions Percentage 0%
• Random Questions and barns.
• My Performance More
What is the most likely diagnosis?
• Media Bank
• New Multimedia Allergic bronchopulmonary aspergillosis (ABPA)
Reference: Normal Values
Online Extras Colonising aspergillosis
Invasive aspergillosis Haematology
Library
Community Asperger’s syndrome Immunoglobulins
Help Type I hypersensitivity to Aspergillus fumigatus
Biochemistry
PasTest Store 353 Diabetes
Endocrinology
My Account
Blood gases
Profile
CSF
Newsletters
My Career and Exams
Order History
Learning Goals
Question Filters

Security

Change Password
Sign Out

© 2011 PasTest Ltd | About Us | Contact Us | Help

file:///E|/Shakhawan/Respiratory%20S/24.htm[3/13/2012 3:57:06 PM]


MyPasTest

Main Navigation
Question Browser: MRCP 1
Home
Subscriptions Question Browser Exam Builder Saved Exams
» MRCP 1 Session Progress
• Question Browser
Questions Correct 0
• Timed Test A 33-year-old man is found to have strongly positive
aspergillus precipitins in his blood and complains of a cough Questions Incorrect 1
• Mock Exam
with intermittent bloody sputum. He has a business working Questions Total 1
• Past Papers as a builder, particularly involved in renovating farm houses
Questions Percentage 0%
• Random Questions and barns.
• My Performance More
What is the most likely diagnosis?
• Media Bank
• New Multimedia Allergic bronchopulmonary Your answer
aspergillosis (ABPA)
Online Extras
Colonising aspergillosis Correct answer
Library
Invasive aspergillosis
Community
Asperger’s syndrome
Help
Type I hypersensitivity to
PasTest Store Aspergillus fumigatus

My Account

Profile
Aspergillus precipitins are IgG antibodies typically found
Newsletters in colonising aspergillosis (aspergilloma), which occurs
My Career and Exams in areas of damaged lungs, e.g. cavities. Type I
hypersensitivity to Aspergillus fumigatus occurs in Reference: Normal Values
Order History
ABPA. Invasive aspergillosis occurs in
Learning Goals immunocompromised patients. Asperger’s syndrome Haematology
Question Filters has nothing to do with aspergillus infection.
Immunoglobulins
Biochemistry
Security 353
Diabetes
Change Password
Sign Out Endocrinology

Blood gases

CSF

© 2011 PasTest Ltd | About Us | Contact Us | Help

file:///E|/Shakhawan/Respiratory%20S/24a.htm[3/13/2012 3:57:07 PM]


MyPasTest

Main Navigation
Question Browser: MRCP 1
Home
Subscriptions Question Browser Exam Builder Saved Exams
» MRCP 1 Session Progress
• Question Browser
Questions Correct 0
• Timed Test A 26-year-old woman arrives in the UK from Australia. A
few days later she presents to hospital with pleuritic chest Questions Incorrect 1
• Mock Exam
pain and breathlessness. She is not on the oral Questions Total 1
• Past Papers contraceptive pill and has no family or personal history of
Questions Percentage 0%
• Random Questions DVT/PE. A pulmonary embolus is confirmed radiologically
and she is commenced on warfarin. More
• My Performance
• Media Bank How long would you continue warfarin therapy in
these circumstances?
• New Multimedia
Reference: Normal Values
Online Extras 4–6 weeks
Haematology
Library 3 months
Community 6 months Immunoglobulins
Help 1 year Biochemistry
PasTest Store Lifelong Diabetes

354 Endocrinology
My Account
Blood gases
Profile
CSF
Newsletters
My Career and Exams
Order History
Learning Goals
Question Filters

Security

Change Password
Sign Out

© 2011 PasTest Ltd | About Us | Contact Us | Help

file:///E|/Shakhawan/Respiratory%20S/25.htm[3/13/2012 3:57:09 PM]


MyPasTest

Main Navigation
Question Browser: MRCP 1
Home
Subscriptions Question Browser Exam Builder Saved Exams
» MRCP 1 Session Progress
• Question Browser
Questions Correct 1
• Timed Test A 26-year-old woman arrives in the UK from Australia. A
few days later she presents to hospital with pleuritic chest Questions Incorrect 1
• Mock Exam
pain and breathlessness. She is not on the oral Questions Total 2
• Past Papers contraceptive pill and has no family or personal history of
Questions Percentage 50 %
• Random Questions DVT/PE. A pulmonary embolus is confirmed radiologically
and she is commenced on warfarin. More
• My Performance
• Media Bank How long would you continue warfarin therapy in
these circumstances?
• New Multimedia
Online Extras 4–6 weeks
Library 3 months Your answer
Community 6 months
Help 1 year
PasTest Store Lifelong

My Account

Profile
This young woman’s only risk factor given is the long-
Newsletters haul flight, which is only a temporary risk factor. The
My Career and Exams standard duration of anticoagulation is 4-6 weeks where
the DVT has occurred post surgery, or 3 months in Reference: Normal Values
Order History
other circumstances where there are no other risk
Learning Goals factors. Patients with long-term risk factors or recurrent Haematology
Question Filters events should be anti-coagulated for at least 6 months.
Immunoglobulins
Biochemistry
Security 354
Diabetes
Change Password
Sign Out Endocrinology

Blood gases

CSF

© 2011 PasTest Ltd | About Us | Contact Us | Help

file:///E|/Shakhawan/Respiratory%20S/25a.htm[3/13/2012 3:57:10 PM]


MyPasTest

Main Navigation
Question Browser: MRCP 1
Home
Subscriptions Question Browser Exam Builder Saved Exams
» MRCP 1 Session Progress
• Question Browser
Questions Correct 1
• Timed Test You are trying to introduce D-dimer testing into your
Accident & Emergency department to reduce the number of Questions Incorrect 1
• Mock Exam
patients who are admitted for suspected pulmonary Questions Total 2
• Past Papers embolus and heparinised unnecessarily.
Questions Percentage 50 %
• Random Questions
Which of the following is true regarding the use of D- More
• My Performance dimer measurement in the diagnosis of pulmonary
• Media Bank embolus?
• New Multimedia
It is a useful screening test for PE Reference: Normal Values
Online Extras
It is likely to be useful in a patient with pleuritic Haematology
Library chest pain, in the absence of breathlessness
Community It is not useful for confirming PE when the clinical Immunoglobulins
Help probability is high Biochemistry
PasTest Store A positive result is of more use clinically than a
negative result Diabetes

Should be performed in patients with a probable Endocrinology


My Account massive PE
Blood gases
Profile
355 CSF
Newsletters
My Career and Exams
Order History
Learning Goals
Question Filters

Security

Change Password
Sign Out

© 2011 PasTest Ltd | About Us | Contact Us | Help

file:///E|/Shakhawan/Respiratory%20S/26.htm[3/13/2012 3:57:11 PM]


MyPasTest

Main Navigation
Question Browser: MRCP 1
Home
Subscriptions Question Browser Exam Builder Saved Exams
» MRCP 1 Session Progress
• Question Browser
Questions Correct 1
• Timed Test You are trying to introduce D-dimer testing into your
Accident & Emergency department to reduce the number of Questions Incorrect 2
• Mock Exam
patients who are admitted for suspected pulmonary Questions Total 3
• Past Papers embolus and heparinised unnecessarily.
Questions Percentage 33 %
• Random Questions
Which of the following is true regarding the use of D- More
• My Performance dimer measurement in the diagnosis of pulmonary
• Media Bank embolus?
• New Multimedia
It is a useful screening test for PE Your answer
Online Extras
It is likely to be useful in a patient
Library with pleuritic chest pain, in the
Community absence of breathlessness
Help It is not useful for confirming PE Correct answer
when the clinical probability is high
PasTest Store
A positive result is of more use
clinically than a negative result
My Account Should be performed in patients
with a probable massive PE
Profile
Newsletters
My Career and Exams
Order History Reference: Normal Values
Most patients with PE are breathless and/or tachypnoeic
Learning Goals > 20/min. In the absence of these signs, pleuritic chest Haematology
Question Filters pain or haemoptysis is usually due to another cause. D-
Dimer measurement can be very useful if used wisely. Immunoglobulins
However, it should not be used as a screening test for
PE as D-dimers may be positive in hospitalised patients, Biochemistry
Security
obstetric patients and those with peripheral vascular Diabetes
Change Password disease, cancer, inflammatory conditions and with
increasing age. The D-Dimer test misses 10% of Endocrinology
Sign Out
patients with PE, while only 30% of patients with
positive D-Dimer findings have a confirmatory diagnosis Blood gases
of PE (i.e. the negative predictive value is greater than
CSF
the positive predictive value). D-Dimer measurements
should NOT be performed if: (1) an alternative diagnosis
is likely, (2) the clinical probability is high, or (3) there
is a probable massive PE.

355

© 2011 PasTest Ltd | About Us | Contact Us | Help

file:///E|/Shakhawan/Respiratory%20S/26a.htm[3/13/2012 3:57:13 PM]


MyPasTest

Main Navigation
Question Browser: MRCP 1
Home
Subscriptions Question Browser Exam Builder Saved Exams
» MRCP 1 Session Progress
• Question Browser
Questions Correct 1
• Timed Test A 41-year-old woman presents to the Accident &
Emergency department with a sudden onset of pleuritic Questions Incorrect 2
• Mock Exam
chest pain and breathlessness. A chest radiograph reveals a Questions Total 3
• Past Papers large right-sided pneumothorax. Pleural aspiration fails to
Questions Percentage 33 %
• Random Questions result in adequate re-expansion of the lung and you
therefore insert an intercostal tube connected to an More
• My Performance
underwater seal.
• Media Bank
After 24 hours of intercostal drainage the lung has
• New Multimedia
not re-expanded despite the drain still swinging with Reference: Normal Values
Online Extras respiration. What would you do next?
Haematology
Library
Wait another 24 hours
Community Immunoglobulins
Negative suction should be started at –1 to –2 cm
Help H 2O Biochemistry
PasTest Store Diabetes
High-volume/low-pressure suction should be used
Refer for immediate surgical intervention Endocrinology
My Account
Reposition chest drain Blood gases
Profile
356 CSF
Newsletters
My Career and Exams
Order History
Learning Goals
Question Filters

Security

Change Password
Sign Out

© 2011 PasTest Ltd | About Us | Contact Us | Help

file:///E|/Shakhawan/Respiratory%20S/27.htm[3/13/2012 3:57:14 PM]


MyPasTest

Main Navigation
Question Browser: MRCP 1
Home
Subscriptions Question Browser Exam Builder Saved Exams
» MRCP 1 Session Progress
• Question Browser
Questions Correct 1
• Timed Test A 41-year-old woman presents to the Accident &
Emergency department with a sudden onset of pleuritic Questions Incorrect 3
• Mock Exam
chest pain and breathlessness. A chest radiograph reveals a Questions Total 4
• Past Papers large right-sided pneumothorax. Pleural aspiration fails to
Questions Percentage 25 %
• Random Questions result in adequate re-expansion of the lung and you
therefore insert an intercostal tube connected to an More
• My Performance
underwater seal.
• Media Bank
After 24 hours of intercostal drainage the lung has
• New Multimedia
not re-expanded despite the drain still swinging with
Online Extras respiration. What would you do next?
Library
Wait another 24 hours Correct answer
Community
Negative suction should be started
Help at –1 to –2 cm H 2 O
PasTest Store
High-volume/low-pressure suction
should be used
My Account Refer for immediate surgical Your answer
intervention
Profile
Reposition chest drain
Newsletters
My Career and Exams
Order History Reference: Normal Values
Learning Goals If, at 48 hours, a pneumothorax fails to re-expand or Haematology
Question Filters there is a persistent air leak (bubbling present) then
you should refer the patient to a respiratory specialist Immunoglobulins
as negative suction may be required. Normal Biochemistry
Security intrapleural pressure is –3.4 cmH2 O during expiration,
rising to –8 cmH2 O during inspiration. This should be Diabetes
Change Password
started at –10 to –20 cmH2 O (–1 to –2 kPa = –7.5 to – Endocrinology
Sign Out 15 mmHg) using a high-volume/low-pressure suction
system. If high-volume/high-pressure suction is used Blood gases
then high-airflow suction may be generated, which can
lead to air stealing, hypoxaemia and/or the persistence CSF
of air leaks. If appropriate suction fails to result in
adequate re-expansion by 5–7 days in those without
pre-existing lung disease (earlier if lung disease) then
referral to a thoracic surgeon is indicated.

356

© 2011 PasTest Ltd | About Us | Contact Us | Help

file:///E|/Shakhawan/Respiratory%20S/27a.htm[3/13/2012 3:57:15 PM]


MyPasTest

Main Navigation
Question Browser: MRCP 1
Home
Subscriptions Question Browser Exam Builder Saved Exams
» MRCP 1 Session Progress
• Question Browser
Questions Correct 1
• Timed Test A 48-year-old man with hepatitis B is admitted via his GP
with a history of breathlessness. He has signs of chronic Questions Incorrect 3
• Mock Exam
liver disease and does not want to stay in hospital. His INR Questions Total 4
• Past Papers is 2.8 and his chest radiograph appears normal. The nurses
Questions Percentage 25 %
• Random Questions comment that his oxygen saturation drops from 98% to
91% on room air every time he gets up to leave. What is More
• My Performance
the most likely diagnosis?
• Media Bank
• New Multimedia Small pneumothorax
Reference: Normal Values
Online Extras Aspiration pneumonia
Pulmonary embolus Haematology
Library
Community Hepatopulmonary syndrome Immunoglobulins
Help Viral pneumonitis Biochemistry
PasTest Store 357 Diabetes
Endocrinology
My Account
Blood gases
Profile
CSF
Newsletters
My Career and Exams
Order History
Learning Goals
Question Filters

Security

Change Password
Sign Out

© 2011 PasTest Ltd | About Us | Contact Us | Help

file:///E|/Shakhawan/Respiratory%20S/28.htm[3/13/2012 3:57:17 PM]


MyPasTest

Main Navigation
Question Browser: MRCP 1
Home
Subscriptions Question Browser Exam Builder Saved Exams
» MRCP 1 Session Progress
• Question Browser
Questions Correct 2
• Timed Test A 48-year-old man with hepatitis B is admitted via his GP
with a history of breathlessness. He has signs of chronic Questions Incorrect 3
• Mock Exam
liver disease and does not want to stay in hospital. His INR Questions Total 5
• Past Papers is 2.8 and his chest radiograph appears normal. The nurses
Questions Percentage 40 %
• Random Questions comment that his oxygen saturation drops from 98% to
91% on room air every time he gets up to leave. What is More
• My Performance
the most likely diagnosis?
• Media Bank
• New Multimedia Small pneumothorax
Online Extras Aspiration pneumonia
Library Pulmonary embolus
Community Hepatopulmonary syndrome Your answer
Help Viral pneumonitis
PasTest Store

My Account
Hepatic disease can result in right-to-left shunting,
Profile secondary to intrapulmonary vasodilatation mainly in
the lower lobes. This is similar to pulmonary
Newsletters
arteriovenous malformations (PAVMs) in hereditary
My Career and Exams haemorrhagic telangiectasia, where the PAVMs are
Order History located mostly in the lower lobes. As a result, there is Reference: Normal Values
increased blood flow through the lower lobes when the
Learning Goals patient moves from the supine to the erect posture. The Haematology
Question Filters end result of this is that deoxygenated blood enters the
left side without going through the lungs, with Immunoglobulins
consequent desaturation in the erect posture. This
Biochemistry
Security phenomenon is called ‘orthodeoxia’.
Diabetes
Change Password
Sign Out 357 Endocrinology

Blood gases

CSF

© 2011 PasTest Ltd | About Us | Contact Us | Help

file:///E|/Shakhawan/Respiratory%20S/28a.htm[3/13/2012 3:57:18 PM]


MyPasTest

Main Navigation
Question Browser: MRCP 1
Home
Subscriptions Question Browser Exam Builder Saved Exams
» MRCP 1 Session Progress
• Question Browser
Questions Correct 2
• Timed Test A 72-year-old man with ischaemic heart disease is on a
variety of medication following a myocardial infarction 12 Questions Incorrect 3
• Mock Exam
months ago, including aspirin, atenolol, lisinopril, Questions Total 5
• Past Papers amiodarone and furosemide (frusemide). He is becoming
Questions Percentage 40 %
• Random Questions progressively breathless and his cardiologist sends him for
pulmonary function tests, which show a restrictive More
• My Performance
ventilatory defect with decreased gas transfer.
• Media Bank
Which one of his medications is most likely to be the
• New Multimedia
cause of these abnormalities? Reference: Normal Values
Online Extras
Aspirin Haematology
Library
Community Atenolol Immunoglobulins
Help Lisinopril Biochemistry
PasTest Store Amiodarone
Diabetes
Furosemide
Endocrinology
My Account 358
Blood gases
Profile
CSF
Newsletters
My Career and Exams
Order History
Learning Goals
Question Filters

Security

Change Password
Sign Out

© 2011 PasTest Ltd | About Us | Contact Us | Help

file:///E|/Shakhawan/Respiratory%20S/29.htm[3/13/2012 3:57:20 PM]


MyPasTest

Main Navigation
Question Browser: MRCP 1
Home
Subscriptions Question Browser Exam Builder Saved Exams
» MRCP 1 Session Progress
• Question Browser
Questions Correct 3
• Timed Test A 72-year-old man with ischaemic heart disease is on a
variety of medication following a myocardial infarction 12 Questions Incorrect 3
• Mock Exam
months ago, including aspirin, atenolol, lisinopril, Questions Total 6
• Past Papers amiodarone and furosemide (frusemide). He is becoming
Questions Percentage 50 %
• Random Questions progressively breathless and his cardiologist sends him for
pulmonary function tests, which show a restrictive More
• My Performance
ventilatory defect with decreased gas transfer.
• Media Bank
Which one of his medications is most likely to be the
• New Multimedia
cause of these abnormalities?
Online Extras
Library Aspirin
Community Atenolol
Help Lisinopril
PasTest Store Amiodarone Your answer
Furosemide

My Account

Profile
Newsletters Aspirin is associated with nasal polyps and asthma,
My Career and Exams atenolol with bronchoconstriction, lisinopril with cough
and furosemide very rarely with bronchoconstriction. Reference: Normal Values
Order History
Amiodarone is associated with pulmonary fibrosis.
Learning Goals Haematology
Question Filters
358 Immunoglobulins
Biochemistry
Security
Diabetes
Change Password
Sign Out Endocrinology

Blood gases

CSF

© 2011 PasTest Ltd | About Us | Contact Us | Help

file:///E|/Shakhawan/Respiratory%20S/29a.htm[3/13/2012 3:57:21 PM]


MyPasTest

Main Navigation
Question Browser: MRCP 1
Home
Subscriptions Question Browser Exam Builder Saved Exams
» MRCP 1 Session Progress
• Question Browser
Questions Correct 3
• Timed Test A 49-year-old woman with ulcerative colitis becomes
breathless and develops a dry cough. Her pulmonary Questions Incorrect 3
• Mock Exam
function tests are shown; Questions Total 6
• Past Papers
Questions Percentage 50 %
• Random Questions Value Predicted
• My Performance More
FEV1 1.6 2.6
• Media Bank
FVC 2.7 3.0
• New Multimedia
TLC 4.1 3.7 Reference: Normal Values
Online Extras
RV 1.3 0.8 Haematology
Library
TLCO 65% pred Immunoglobulins
Community
KCO 62% pred
Help Biochemistry
PasTest Store Which of the following pulmonary complications of Diabetes
ulcerative colitis is the most likely explanation?
Endocrinology
My Account
Bronchiolitis Blood gases
Profile Interstitial lung disease
CSF
Newsletters Pulmonary infiltrates with eosinophilia (PIE
My Career and Exams Syndrome)

Order History Bronchiectasis

Learning Goals Cryptogenic organising pneumonia (COP)

Question Filters 359

Security

Change Password
Sign Out

© 2011 PasTest Ltd | About Us | Contact Us | Help

file:///E|/Shakhawan/Respiratory%20S/30.htm[3/13/2012 3:57:23 PM]


MyPasTest

Main Navigation
Question Browser: MRCP 1
Home
Subscriptions Question Browser Exam Builder Saved Exams
» MRCP 1 Session Progress
• Question Browser
Questions Correct 3
• Timed Test A 49-year-old woman with ulcerative colitis becomes
breathless and develops a dry cough. Her pulmonary Questions Incorrect 4
• Mock Exam
function tests are shown; Questions Total 7
• Past Papers
Questions Percentage 42 %
• Random Questions Value Predicted
• My Performance More
FEV1 1.6 2.6
• Media Bank
FVC 2.7 3.0
• New Multimedia
TLC 4.1 3.7
Online Extras
RV 1.3 0.8
Library
TLCO 65% pred
Community
KCO 62% pred
Help
PasTest Store Which of the following pulmonary complications of
ulcerative colitis is the most likely explanation?
My Account
Bronchiolitis Correct answer
Profile Interstitial lung disease Your answer
Newsletters Pulmonary infiltrates with
My Career and Exams eosinophilia (PIE Syndrome)

Order History Bronchiectasis Reference: Normal Values


Learning Goals Cryptogenic organising pneumonia
(COP) Haematology
Question Filters
Immunoglobulins
Biochemistry
Security
Diabetes
Change Password All the above may be seen in the context of
inflammatory bowel disease but the low flow rates at Endocrinology
Sign Out
low lung volumes is suggestive of a small airway (ie
bronchiolar) problem. In combination with gas transfer Blood gases
the diagnosis is bronchiolitis. Expiratory thoracic HRCT
CSF
classically reveals a mosaic pattern compatible with gas
trapping.

359

© 2011 PasTest Ltd | About Us | Contact Us | Help

file:///E|/Shakhawan/Respiratory%20S/30a.htm[3/13/2012 3:57:24 PM]


MyPasTest

Main Navigation
Question Browser: MRCP 1
Home
Subscriptions Question Browser Exam Builder Saved Exams
» MRCP 1 Session Progress
• Question Browser
Questions Correct 3
• Timed Test An 18-year-old man presents with a gradual onset of
pallor, weakness, lethargy, dry cough and occasional Questions Incorrect 4
• Mock Exam
haemoptysis. There are no extrapulmonary features. His Questions Total 7
• Past Papers chest X-ray shows diffuse pulmonary infiltrates. Lung
Questions Percentage 42 %
• Random Questions biopsy shows no vasculitic changes and no evidence of
immunoglobulin or complement deposition. His gas transfer More
• My Performance
factor (TLCO) is found to be elevated.
• Media Bank
• New Multimedia What is the most likely diagnosis?
Reference: Normal Values
Online Extras Goodpasture’s syndrome
Haematology
Library Idiopathic pulmonary haemosiderosis
Community Immunoglobulins
Wegener’s granulomatosis
Help Beçhet's Disease Biochemistry
PasTest Store Systemic lupus erythematosus Diabetes

360 Endocrinology
My Account
Blood gases
Profile
CSF
Newsletters
My Career and Exams
Order History
Learning Goals
Question Filters

Security

Change Password
Sign Out

© 2011 PasTest Ltd | About Us | Contact Us | Help

file:///E|/Shakhawan/Respiratory%20S/31.htm[3/13/2012 3:57:26 PM]


MyPasTest

Main Navigation
Question Browser: MRCP 1
Home
Subscriptions Question Browser Exam Builder Saved Exams
» MRCP 1 Session Progress
• Question Browser
Questions Correct 4
• Timed Test An 18-year-old man presents with a gradual onset of
pallor, weakness, lethargy, dry cough and occasional Questions Incorrect 4
• Mock Exam
haemoptysis. There are no extrapulmonary features. His Questions Total 8
• Past Papers chest X-ray shows diffuse pulmonary infiltrates. Lung
Questions Percentage 50 %
• Random Questions biopsy shows no vasculitic changes and no evidence of
immunoglobulin or complement deposition. His gas transfer More
• My Performance
factor (TLCO) is found to be elevated.
• Media Bank
• New Multimedia What is the most likely diagnosis?

Online Extras Goodpasture’s syndrome


Library Idiopathic pulmonary haemosiderosis Your answer
Community Wegener’s granulomatosis
Help Beçhet's Disease
PasTest Store Systemic lupus erythematosus

My Account

Profile
IPH tends to occur in younger people and is
Newsletters
characterised by pallor, weakness, lethargy, dry cough
My Career and Exams and occasional haemoptysis. There are no
Order History extrapulmonary features. There are no abnormal Reference: Normal Values
immunological features, which differentiates it from
Learning Goals Goodpasture’s syndrome. Gas transfer is elevated as Haematology
Question Filters blood is already in the alveolar space.
Immunoglobulins
Biochemistry
Security 360
Diabetes
Change Password
Sign Out Endocrinology

Blood gases

CSF

© 2011 PasTest Ltd | About Us | Contact Us | Help

file:///E|/Shakhawan/Respiratory%20S/31a.htm[3/13/2012 3:57:27 PM]


MyPasTest

Main Navigation
Question Browser: MRCP 1
Home
Subscriptions Question Browser Exam Builder Saved Exams
» MRCP 1 Session Progress
• Question Browser
Questions Correct 4
• Timed Test Which of the following is a feature in acute
exacerbation of chronic bronchitis? Questions Incorrect 4
• Mock Exam
Questions Total 8
• Past Papers
Respiratory acidosis is associated with a lowering of Questions Percentage 50 %
• Random Questions bicarbonate levels
• My Performance Exacerbation is usually due to anaerobic infection More
• Media Bank Aminophylline/theophylline combinations form the
• New Multimedia first line of management
Reference: Normal Values
Online Extras An extensor–plantar response is common
Oxygen therapy should be continued until the Haematology
Library
symptoms subside
Community Immunoglobulins
Help 421 Biochemistry
PasTest Store Diabetes
Endocrinology
My Account
Blood gases
Profile
CSF
Newsletters
My Career and Exams
Order History
Learning Goals
Question Filters

Security

Change Password
Sign Out

© 2011 PasTest Ltd | About Us | Contact Us | Help

file:///E|/Shakhawan/Respiratory%20S/32.htm[3/13/2012 3:57:28 PM]


MyPasTest

Main Navigation
Question Browser: MRCP 1
Home
Subscriptions Question Browser Exam Builder Saved Exams
» MRCP 1 Session Progress
• Question Browser
Questions Correct 4
• Timed Test Which of the following is a feature in acute
exacerbation of chronic bronchitis? Questions Incorrect 5
• Mock Exam
Questions Total 9
• Past Papers
Respiratory acidosis is associated Questions Percentage 44 %
• Random Questions with a lowering of bicarbonate levels
• My Performance Exacerbation is usually due to Your answer More
• Media Bank anaerobic infection

• New Multimedia Aminophylline/theophylline


combinations form the first line of
Online Extras management
Library An extensor–plantar response is Correct answer
Community common
Help Oxygen therapy should be continued
until the symptoms subside
PasTest Store

My Account

Profile In respiratory acidosis, bicarbonate and hydrogen levels


are usually raised due to carbon dioxide retention and
Newsletters
the renal retention of bicarbonate. Aminophylline and
My Career and Exams theophylline are indicated in patients with reversible
Order History chronic obstructive pulmonary disease (COPD) only if Reference: Normal Values
nebulised bronchodilators and steroids are ineffective.
Learning Goals Oxygen therapy should aim for a p(O2 ) of 60 mmHg, Haematology
Question Filters particularly for ‘blue bloaters’ (with type-II respiratory
Immunoglobulins
failure). Any further increase will result in carbon
dioxide retention. Extensor–plantar responses are seen Biochemistry
Security in COPD due to carbon dioxide retention, which results
in carbon dioxide narcosis. Diabetes
Change Password
Sign Out Endocrinology
421 Blood gases

CSF

© 2011 PasTest Ltd | About Us | Contact Us | Help

file:///E|/Shakhawan/Respiratory%20S/32a.htm[3/13/2012 3:57:30 PM]


MyPasTest

Main Navigation
Question Browser: MRCP 1
Home
Subscriptions Question Browser Exam Builder Saved Exams
» MRCP 1 Session Progress
• Question Browser
Questions Correct 4
• Timed Test Which of the following relates to exacerbation of
chronic bronchitis in patients with COPD? Questions Incorrect 5
• Mock Exam
Questions Total 9
• Past Papers
Moraxella catarrhalis is commonly isolated on culture Questions Percentage 44 %
• Random Questions
Clinical symptoms are usually severe
• My Performance More
An elevated white cell count indicates exacerbation
• Media Bank
Trimethoprim–sulfamethoxazole combinations are
• New Multimedia effective in the treatment of M. catarrhalis infection
Reference: Normal Values
Online Extras Gram’s stain is inconclusive and blood cultures are
necessary Haematology
Library
Community Immunoglobulins
422
Help Biochemistry
PasTest Store Diabetes
Endocrinology
My Account
Blood gases
Profile
CSF
Newsletters
My Career and Exams
Order History
Learning Goals
Question Filters

Security

Change Password
Sign Out

© 2011 PasTest Ltd | About Us | Contact Us | Help

file:///E|/Shakhawan/Respiratory%20S/33.htm[3/13/2012 3:57:31 PM]


MyPasTest

Main Navigation
Question Browser: MRCP 1
Home
Subscriptions Question Browser Exam Builder Saved Exams
» MRCP 1 Session Progress
• Question Browser
Questions Correct 4
• Timed Test Which of the following relates to exacerbation of
Questions Incorrect 6
• Mock Exam chronic bronchitis in patients with COPD?
Questions Total 10
• Past Papers
Moraxella catarrhalis is commonly Correct answer Questions Percentage 40 %
• Random Questions isolated on culture
• My Performance Clinical symptoms are usually Your answer More
• Media Bank severe

• New Multimedia An elevated white cell count


indicates exacerbation
Online Extras
Trimethoprim–sulfamethoxazole
Library combinations are effective in the
Community treatment of M. catarrhalis infection
Help Gram’s stain is inconclusive and
blood cultures are necessary
PasTest Store

My Account

Profile Along with Haemophilus influenzae and Streptococcus


pneumoniae, Moraxella catarrhalis is a common cause
Newsletters
of exacerbation of chronic bronchitis and pneumonia in
My Career and Exams those patients with COPD. Symptoms of moraxella
Order History infection tend to be mild to moderate and a lack of Reference: Normal Values
leucocytosis is common. Resistance to both
Learning Goals trimethoprim–sulfamethoxazole combination and Haematology
Question Filters tetracycline has been reported. The most appropriate
choice would be a combination of ampicillin and Immunoglobulins
clavulinic acid, which suppresses M. catarrhalis β- Biochemistry
Security lactamases.
Diabetes
Change Password
Sign Out Endocrinology
422
Blood gases

CSF

© 2011 PasTest Ltd | About Us | Contact Us | Help

file:///E|/Shakhawan/Respiratory%20S/33a.htm[3/13/2012 3:57:33 PM]


MyPasTest

Main Navigation
Question Browser: MRCP 1
Home
Subscriptions Question Browser Exam Builder Saved Exams
» MRCP 1 Session Progress
• Question Browser
Questions Correct 4
• Timed Test Regarding lung development, which of the following
statements is the most appropriate? Questions Incorrect 6
• Mock Exam
Questions Total 10
• Past Papers
Continues until the age of 7 years Questions Percentage 40 %
• Random Questions
Fetal lung growth is more influenced by hormones
• My Performance than mechanical stimuli More
• Media Bank Polycystic kidneys may be associated with
• New Multimedia maldevelopment of the lungs
Reference: Normal Values
Online Extras Surfactant production is recognised by 20 weeks'
gestation Haematology
Library
Alveoli are seen by 24 weeks' gestation
Community Immunoglobulins
Help 423 Biochemistry
PasTest Store Diabetes
Endocrinology
My Account
Blood gases
Profile
CSF
Newsletters
My Career and Exams
Order History
Learning Goals
Question Filters

Security

Change Password
Sign Out

© 2011 PasTest Ltd | About Us | Contact Us | Help

file:///E|/Shakhawan/Respiratory%20S/34.htm[3/13/2012 3:57:34 PM]


MyPasTest

Main Navigation
Question Browser: MRCP 1
Home
Subscriptions Question Browser Exam Builder Saved Exams
» MRCP 1 Session Progress
• Question Browser
Questions Correct 4
• Timed Test Regarding lung development, which of the following
statements is the most appropriate? Questions Incorrect 7
• Mock Exam
Questions Total 11
• Past Papers
Continues until the age of 7 years Correct answer Questions Percentage 36 %
• Random Questions
Fetal lung growth is more influenced
• My Performance by hormones than mechanical stimuli More
• Media Bank Polycystic kidneys may be
• New Multimedia associated with maldevelopment of
the lungs
Online Extras
Surfactant production is recognised
Library by 20 weeks' gestation
Community Alveoli are seen by 24 weeks' Your answer
Help gestation
PasTest Store

My Account
Complete development of the alveoli and the pores of
Profile Kohn only occur around the age of 7 years. Distension
in the fluid-filled lungs, tonic negative pressure of
Newsletters
diaphragmatic tone and fetal breathing all cause lung
My Career and Exams growth. Absence of liquor, which is produced by the
Order History fetal kidneys, causes poor lung growth and a low Reference: Normal Values
phospholipid content. Renal agenesis is thus a cause of
Learning Goals hypoplastic lungs. Lamellar bodies in type-II Haematology
Question Filters pneumocytes that produce surfactant are seen by 22
weeks' gestation. These contain 80% Immunoglobulins
phosphatidylcholine. Complete alveolisation does not
Biochemistry
Security occur until 28 weeks' gestation.
Diabetes
Change Password
Sign Out 423 Endocrinology

Blood gases

CSF

© 2011 PasTest Ltd | About Us | Contact Us | Help

file:///E|/Shakhawan/Respiratory%20S/34a.htm[3/13/2012 3:57:36 PM]


MyPasTest

Main Navigation
Question Browser: MRCP 1
Home
Subscriptions Question Browser Exam Builder Saved Exams
» MRCP 1 Session Progress
• Question Browser
Questions Correct 4
• Timed Test Regarding the value of lung function tests
Questions Incorrect 7
• Mock Exam
The peak expiratory flow rate is most closely Questions Total 11
• Past Papers
correlated with height Questions Percentage 36 %
• Random Questions
In a restrictive disease, the flow-volume loop is
• My Performance different in shape to normal More
• Media Bank A mid-expiratory flow rate between 25% and 75% of
• New Multimedia expired vital capacity is not indicative of airway
obstruction Reference: Normal Values
Online Extras
They are not needed as a routine in the management Haematology
Library of asthma
Community They cannot differentiate between different causes of Immunoglobulins
Help wheeze Biochemistry
PasTest Store 424 Diabetes
Endocrinology
My Account
Blood gases
Profile
CSF
Newsletters
My Career and Exams
Order History
Learning Goals
Question Filters

Security

Change Password
Sign Out

© 2011 PasTest Ltd | About Us | Contact Us | Help

file:///E|/Shakhawan/Respiratory%20S/35.htm[3/13/2012 3:57:38 PM]


MyPasTest

Main Navigation
Question Browser: MRCP 1
Home
Subscriptions Question Browser Exam Builder Saved Exams
» MRCP 1 Session Progress
• Question Browser
Questions Correct 4
• Timed Test Regarding the value of lung function tests
Questions Incorrect 8
• Mock Exam
The peak expiratory flow rate is Correct answer Questions Total 12
• Past Papers
most closely correlated with height Questions Percentage 33 %
• Random Questions
In a restrictive disease, the flow-
• My Performance volume loop is different in shape to More
• Media Bank normal

• New Multimedia A mid-expiratory flow rate between Your answer


25% and 75% of expired vital
Online Extras capacity is not indicative of airway
Library obstruction
Community They are not needed as a routine in
the management of asthma
Help
They cannot differentiate between
PasTest Store different causes of wheeze

My Account

Profile
The most accurate correlation of the PEFR is with
Newsletters
height. In a restrictive pattern, the f–v loop is reduced
My Career and Exams in size but looks similar in shape to normal. A mid-
Order History expiratory flow rate (between 25% and 75% of the Reference: Normal Values
expired vital capacity) is a good measure of airways
Learning Goals obstruction. PEFR readings are an objective measure of Haematology
Question Filters airway obstruction and in any child able to do PEFR, it is
advisable to do them regularly to assess lung function. Immunoglobulins
Biochemistry
Security
424 Diabetes
Change Password
Sign Out Endocrinology

Blood gases

CSF

© 2011 PasTest Ltd | About Us | Contact Us | Help

file:///E|/Shakhawan/Respiratory%20S/35a.htm[3/13/2012 3:57:39 PM]


MyPasTest

Main Navigation
Question Browser: MRCP 1
Home
Subscriptions Question Browser Exam Builder Saved Exams
» MRCP 1 Session Progress
• Question Browser
Questions Correct 4
• Timed Test A 50-year-old lawyer attended a clinic complaining of a 15-
day history of dyspnoea and weight loss over the past 6 Questions Incorrect 8
• Mock Exam
months. He reports his sputum to be clear. On Questions Total 12
• Past Papers examination, a diagnosis of pleural effusion was made that
Questions Percentage 33 %
• Random Questions was confirmed on chest X-ray. A pleural tap revealed few
red cells and lymphocytes and a protein level of 40 g/l. More
• My Performance
• Media Bank What should be the next investigative step?
• New Multimedia
Bronchoscopy Reference: Normal Values
Online Extras
CT scan of the thorax Haematology
Library
Percutaneous pleural biopsy
Community Immunoglobulins
Sputum examination for tubercle bacilli
Help Biochemistry
Thoracoscopic pleural biopsy
PasTest Store Diabetes
425
Endocrinology
My Account
Blood gases
Profile
CSF
Newsletters
My Career and Exams
Order History
Learning Goals
Question Filters

Security

Change Password
Sign Out

© 2011 PasTest Ltd | About Us | Contact Us | Help

file:///E|/Shakhawan/Respiratory%20S/36.htm[3/13/2012 3:57:41 PM]


MyPasTest

Main Navigation
Question Browser: MRCP 1
Home
Subscriptions Question Browser Exam Builder Saved Exams
» MRCP 1 Session Progress
• Question Browser
Questions Correct 5
• Timed Test A 50-year-old lawyer attended a clinic complaining of a 15-
day history of dyspnoea and weight loss over the past 6 Questions Incorrect 8
• Mock Exam
months. He reports his sputum to be clear. On Questions Total 13
• Past Papers examination, a diagnosis of pleural effusion was made that
Questions Percentage 38 %
• Random Questions was confirmed on chest X-ray. A pleural tap revealed few
red cells and lymphocytes and a protein level of 40 g/l. More
• My Performance
• Media Bank What should be the next investigative step?
• New Multimedia
Bronchoscopy
Online Extras
CT scan of the thorax Your answer
Library
Percutaneous pleural biopsy
Community
Sputum examination for tubercle bacilli
Help
Thoracoscopic pleural biopsy
PasTest Store

My Account

Profile BTS guidelines from 2003 recommend contrast CT as


the investigation here, followed by thorascopic pleural
Newsletters biopsy if an exudate is suspected. A thoracoscopic
My Career and Exams pleural biopsy allows the physician to perform directed
pleural biopsies, remove the pleural fluid and carry out Reference: Normal Values
Order History
pleurodesis to prevent recurrence. Percutaneous pleural
Learning Goals biopsies generally yield poor samples. Haematology
Question Filters
Immunoglobulins
425
Biochemistry
Security
Diabetes
Change Password
Sign Out Endocrinology

Blood gases

CSF

© 2011 PasTest Ltd | About Us | Contact Us | Help

file:///E|/Shakhawan/Respiratory%20S/36a.htm[3/13/2012 3:57:42 PM]


MyPasTest

Main Navigation
Question Browser: MRCP 1
Home
Subscriptions Question Browser Exam Builder Saved Exams
» MRCP 1 Session Progress
• Question Browser
Questions Correct 5
• Timed Test A 70-year-old man with chronic bronchitis is admitted with
dyspnoea and peripheral cyanosis. On auscultation, there Questions Incorrect 8
• Mock Exam
are scattered rhonchi but no wheeze or evidence of Questions Total 13
• Past Papers consolidation. Arterial blood gases show: pH 7.38, Pao2 5.3
Questions Percentage 38 %
• Random Questions kPa (40 mmHg) and Paco2 6.0 kPa (45 mmHg).
• My Performance More
He is given 40% oxygen by face mask. Within 10 minutes
• Media Bank his cyanosis worsens and his respiratory rate falls. Repeat
• New Multimedia arterial blood gases show: pH 7.24, Pao2 9.6 kPa (72
Reference: Normal Values
Online Extras mmHg), Paco2 8.4 kPa (63 mmHg). What is the next step
in his management? Haematology
Library
Community Obtain a chest X-ray Immunoglobulins
Help Do a ventilation/perfusion scan Biochemistry
PasTest Store Decrease the fraction of inspired oxygen Diabetes
Initiate mechanical ventilation
Endocrinology
My Account Administer intravenous aminophylline
Blood gases
Profile 426 CSF
Newsletters
My Career and Exams
Order History
Learning Goals
Question Filters

Security

Change Password
Sign Out

© 2011 PasTest Ltd | About Us | Contact Us | Help

file:///E|/Shakhawan/Respiratory%20S/37.htm[3/13/2012 3:57:44 PM]


MyPasTest

Main Navigation
Question Browser: MRCP 1
Home
Subscriptions Question Browser Exam Builder Saved Exams
» MRCP 1 Session Progress
• Question Browser
Questions Correct 5
• Timed Test A 70-year-old man with chronic bronchitis is admitted with
dyspnoea and peripheral cyanosis. On auscultation, there Questions Incorrect 9
• Mock Exam
are scattered rhonchi but no wheeze or evidence of Questions Total 14
• Past Papers consolidation. Arterial blood gases show: pH 7.38, Pao2 5.3
Questions Percentage 35 %
• Random Questions kPa (40 mmHg) and Paco2 6.0 kPa (45 mmHg).
• My Performance More
He is given 40% oxygen by face mask. Within 10 minutes
• Media Bank his cyanosis worsens and his respiratory rate falls. Repeat
• New Multimedia arterial blood gases show: pH 7.24, Pao2 9.6 kPa (72
Online Extras mmHg), Paco2 8.4 kPa (63 mmHg). What is the next step
in his management?
Library
Community Obtain a chest X-ray
Help Do a ventilation/perfusion scan
PasTest Store Decrease the fraction of inspired Correct answer
oxygen

My Account Initiate mechanical ventilation Your answer


Administer intravenous
Profile aminophylline
Newsletters
My Career and Exams
Order History Reference: Normal Values
Acute respiratory failure in COPD
Learning Goals Haematology
Patients with advanced chronic obstructive pulmonary
Question Filters
disease (COPD) are at risk of developing acute Immunoglobulins
respiratory failure. Oxygen therapy is effective in
reversing the hypoxaemia of respiratory failure, but Biochemistry
Security
oxygen administration can lead to hypercapnia as these
Diabetes
Change Password patients have lost their sensitivity to hypercapnia –
when the hypoxaemia is corrected, they lose their Endocrinology
Sign Out stimulus to breathe and develop carbon dioxide
narcosis. Oxygen administration therefore has to be Blood gases
decreased prior to initiating mechanical ventilation.
CSF

426

© 2011 PasTest Ltd | About Us | Contact Us | Help

file:///E|/Shakhawan/Respiratory%20S/37a.htm[3/13/2012 3:57:45 PM]


MyPasTest

Main Navigation
Question Browser: MRCP 1
Home
Subscriptions Question Browser Exam Builder Saved Exams
» MRCP 1 Session Progress
• Question Browser
Questions Correct 5
• Timed Test A 25-year-old basketball player is brought to the accident
and emergency department with a history of sudden onset Questions Incorrect 9
• Mock Exam
of right-sided chest pain and breathlessness. On Questions Total 14
• Past Papers examination, tachycardia is noted. Decreased breath
Questions Percentage 35 %
• Random Questions sounds are heard on the right side. What is the probable
diagnosis? More
• My Performance
• Media Bank Rupture of subpleural tuberculous focus
• New Multimedia Rupture of a subphrenic abscess through the Reference: Normal Values
Online Extras diaphragm
Rupture of apical subpleural blebs Haematology
Library
Community Pulmonary embolism Immunoglobulins
Help Lobar pneumonia Biochemistry
PasTest Store 427 Diabetes
Endocrinology
My Account
Blood gases
Profile
CSF
Newsletters
My Career and Exams
Order History
Learning Goals
Question Filters

Security

Change Password
Sign Out

© 2011 PasTest Ltd | About Us | Contact Us | Help

file:///E|/Shakhawan/Respiratory%20S/38.htm[3/13/2012 3:57:46 PM]


MyPasTest

Main Navigation
Question Browser: MRCP 1
Home
Subscriptions Question Browser Exam Builder Saved Exams
» MRCP 1 Session Progress
• Question Browser
Questions Correct 6
• Timed Test A 25-year-old basketball player is brought to the accident
and emergency department with a history of sudden onset Questions Incorrect 9
• Mock Exam
of right-sided chest pain and breathlessness. On Questions Total 15
• Past Papers examination, tachycardia is noted. Decreased breath
Questions Percentage 40 %
• Random Questions sounds are heard on the right side. What is the probable
diagnosis? More
• My Performance
• Media Bank Rupture of subpleural tuberculous
• New Multimedia focus
Online Extras Rupture of a subphrenic abscess
through the diaphragm
Library
Rupture of apical subpleural blebs Your answer
Community
Pulmonary embolism
Help
Lobar pneumonia
PasTest Store

My Account

Profile Rupture of subpleural tuberculous focus is a rare cause,


particularly in the UK. Rupture of a subphrenic abscess
Newsletters
through the diaphragm may cause empyema, not
My Career and Exams pneumothorax. Primary spontaneous pneumothorax is
Order History due to rupture of apical pleural blebs. The incidence of Reference: Normal Values
pneumothorax is highest in males aged 15-30 years
Learning Goals where smoking, height and the presence of apical Haematology
Question Filters subpleural blebs appear to be the most important
aetiological factors. Height may be a significant factor in Immunoglobulins
this case as the patient is a basketball player. Lobar
Biochemistry
Security pneumonia usually presents with a history of cough,
fever and malaise. The majority of patients with Diabetes
Change Password pulmonary embolism present with pleurisy, shortness of
Sign Out breath and haemoptysis. 75% of pulmonary emboli Endocrinology
derive from deep vein thrombosis in the lower limb.
There does not appear to be any risk factor for venous Blood gases
thromboembolism in this case. CSF

427

© 2011 PasTest Ltd | About Us | Contact Us | Help

file:///E|/Shakhawan/Respiratory%20S/38a.htm[3/13/2012 3:57:48 PM]


MyPasTest

Main Navigation
Question Browser: MRCP 1
Home
Subscriptions Question Browser Exam Builder Saved Exams
» MRCP 1 Session Progress
• Question Browser
Questions Correct 6
• Timed Test Causes of a restrictive pattern on pulmonary function
testing include? Questions Incorrect 9
• Mock Exam
Questions Total 15
• Past Papers
Emphysema Questions Percentage 40 %
• Random Questions
Asthma
• My Performance More
Early cystic fibrosis
• Media Bank
Severe scoliosis
• New Multimedia
Bronchiectasis Reference: Normal Values
Online Extras
428 Haematology
Library
Community Immunoglobulins
Help Biochemistry
PasTest Store Diabetes
Endocrinology
My Account
Blood gases
Profile
CSF
Newsletters
My Career and Exams
Order History
Learning Goals
Question Filters

Security

Change Password
Sign Out

© 2011 PasTest Ltd | About Us | Contact Us | Help

file:///E|/Shakhawan/Respiratory%20S/39.htm[3/13/2012 3:57:49 PM]


MyPasTest

Main Navigation
Question Browser: MRCP 1
Home
Subscriptions Question Browser Exam Builder Saved Exams
» MRCP 1 Session Progress
• Question Browser
Questions Correct 7
• Timed Test Causes of a restrictive pattern on pulmonary function
testing include? Questions Incorrect 9
• Mock Exam
Questions Total 16
• Past Papers
Emphysema Questions Percentage 43 %
• Random Questions
Asthma
• My Performance More
Early cystic fibrosis
• Media Bank
Severe scoliosis Your answer
• New Multimedia
Bronchiectasis
Online Extras
Library
Community
Help Emphysema, asthma, cystic fibrosis and bronchiectasis
PasTest Store present an obstructive pattern with a low PEFR. Severe
scoliosis compromises respiratory muscle action and can
present as a restrictive pattern in pulmonary function
My Account tests.

Profile
Newsletters 428

My Career and Exams


Order History Reference: Normal Values
Learning Goals Haematology
Question Filters
Immunoglobulins
Biochemistry
Security
Diabetes
Change Password
Sign Out Endocrinology

Blood gases

CSF

© 2011 PasTest Ltd | About Us | Contact Us | Help

file:///E|/Shakhawan/Respiratory%20S/39a.htm[3/13/2012 3:57:50 PM]


MyPasTest

Main Navigation
Question Browser: MRCP 1
Home
Subscriptions Question Browser Exam Builder Saved Exams
» MRCP 1 Session Progress
• Question Browser
Questions Correct 7
• Timed Test A 25-year-old male is admitted with dyspnoea. Pulmonary
function tests reveal a reduced peak expiratory flow rate Questions Incorrect 9
• Mock Exam
55% below the normal range for his height and age. Questions Total 16
• Past Papers
What could be the probable diagnosis? Questions Percentage 43 %
• Random Questions
• My Performance More
Asthma
• Media Bank
Bronchial carcinoma
• New Multimedia
Kyphoscoliosis Reference: Normal Values
Online Extras
Bronchiectasis
Haematology
Library
Whooping cough
Community Immunoglobulins
Help 429 Biochemistry
PasTest Store Diabetes
Endocrinology
My Account
Blood gases
Profile
CSF
Newsletters
My Career and Exams
Order History
Learning Goals
Question Filters

Security

Change Password
Sign Out

© 2011 PasTest Ltd | About Us | Contact Us | Help

file:///E|/Shakhawan/Respiratory%20S/40.htm[3/13/2012 3:57:52 PM]


MyPasTest

Main Navigation
Question Browser: MRCP 1
Home
Subscriptions Question Browser Exam Builder Saved Exams
» MRCP 1 Session Progress
• Question Browser
Questions Correct 7
• Timed Test A 25-year-old male is admitted with dyspnoea. Pulmonary
function tests reveal a reduced peak expiratory flow rate Questions Incorrect 10
• Mock Exam
55% below the normal range for his height and age. Questions Total 17
• Past Papers
What could be the probable diagnosis? Questions Percentage 41 %
• Random Questions
• My Performance More
Asthma Correct answer
• Media Bank
Bronchial carcinoma
• New Multimedia
Kyphoscoliosis Your answer
Online Extras
Bronchiectasis
Library
Whooping cough
Community
Help
PasTest Store
The peak expiratory flow (PEF) is an indicator of large
My Account airways disease. It is measured by a maximal forced
expiration through a peak flow meter. It correlates well
Profile with the forced expiratory volume in one second (FEV 1 )
Newsletters and is used as an estimate of airway caliber. PEF can be
monitored by patients at home to assess asthma control
My Career and Exams and is of value in both the diagnosis and the
Order History management of asthma. Reference: Normal Values
Learning Goals Haematology
Question Filters 429
Immunoglobulins
Biochemistry
Security
Diabetes
Change Password
Sign Out Endocrinology

Blood gases

CSF

© 2011 PasTest Ltd | About Us | Contact Us | Help

file:///E|/Shakhawan/Respiratory%20S/40a.htm[3/13/2012 3:57:53 PM]


MyPasTest

Main Navigation
Question Browser: MRCP 1
Home
Subscriptions Question Browser Exam Builder Saved Exams
» MRCP 1 Session Progress
• Question Browser
Questions Correct 7
• Timed Test An 18-year-old girl presents with a four-day history of
cough, headache, fever and joint pains. Blood tests show Questions Incorrect 10
• Mock Exam
the presence of raised antibody titres and the presence of Questions Total 17
• Past Papers cold agglutinins. A diagnosis of Mycoplasma pneumoniae
Questions Percentage 41 %
• Random Questions infection is made. Which drug would you prescribe as
first line treatment for this patient? More
• My Performance
• Media Bank Tetracycline
• New Multimedia Rifampicin Reference: Normal Values
Online Extras Penicillin
Haematology
Library Clarithromycin
Community Co-trimoxazole Immunoglobulins
Help Biochemistry
430
PasTest Store Diabetes
Endocrinology
My Account
Blood gases
Profile
CSF
Newsletters
My Career and Exams
Order History
Learning Goals
Question Filters

Security

Change Password
Sign Out

© 2011 PasTest Ltd | About Us | Contact Us | Help

file:///E|/Shakhawan/Respiratory%20S/41.htm[3/13/2012 3:57:55 PM]


MyPasTest

Main Navigation
Question Browser: MRCP 1
Home
Subscriptions Question Browser Exam Builder Saved Exams
» MRCP 1 Session Progress
• Question Browser
Questions Correct 8
• Timed Test An 18-year-old girl presents with a four-day history of
cough, headache, fever and joint pains. Blood tests show Questions Incorrect 10
• Mock Exam
the presence of raised antibody titres and the presence of Questions Total 18
• Past Papers cold agglutinins. A diagnosis of Mycoplasma pneumoniae
Questions Percentage 44 %
• Random Questions infection is made. Which drug would you prescribe as
first line treatment for this patient? More
• My Performance
• Media Bank Tetracycline
• New Multimedia Rifampicin
Online Extras Penicillin
Library Clarithromycin Your answer
Community Co-trimoxazole
Help
PasTest Store

My Account Rifampicin therapy is given in legionella infection and in


severe cases of mycoplasma pneumoniae. Penicillin is
Profile commonly used in pneumococcal infection.
Clarithromycin is preferred for mycoplasma infections
Newsletters
while co-trimoxazole is given in pneumocystis infection.
My Career and Exams Tetracyline may be used for mycoplasma pneumonia,
Order History being as effective as macrolide antibiotics; however due Reference: Normal Values
to relative side effect profiles clarithromycin would be
Learning Goals the first choice here. Tetracycline is also used for Haematology
Question Filters Coxiella burnetti infection and psittacosis.
Immunoglobulins
Biochemistry
Security 430
Diabetes
Change Password
Sign Out Endocrinology

Blood gases

CSF

© 2011 PasTest Ltd | About Us | Contact Us | Help

file:///E|/Shakhawan/Respiratory%20S/41a.htm[3/13/2012 3:57:56 PM]


MyPasTest

Main Navigation
Question Browser: MRCP 1
Home
Subscriptions Question Browser Exam Builder Saved Exams
» MRCP 1 Session Progress
• Question Browser
Questions Correct 8
• Timed Test Male infertility in cystic fibrosis is mostly due to
which of the following? Questions Incorrect 10
• Mock Exam
Questions Total 18
• Past Papers
Impotence Questions Percentage 44 %
• Random Questions
Failure of development of the vas deferens
• My Performance More
Median survival age of 20 years
• Media Bank
Delayed puberty
• New Multimedia
Decreased spermatogenesis Reference: Normal Values
Online Extras
431 Haematology
Library
Community Immunoglobulins
Help Biochemistry
PasTest Store Diabetes
Endocrinology
My Account
Blood gases
Profile
CSF
Newsletters
My Career and Exams
Order History
Learning Goals
Question Filters

Security

Change Password
Sign Out

© 2011 PasTest Ltd | About Us | Contact Us | Help

file:///E|/Shakhawan/Respiratory%20S/42.htm[3/13/2012 3:57:58 PM]


MyPasTest

Main Navigation
Question Browser: MRCP 1
Home
Subscriptions Question Browser Exam Builder Saved Exams
» MRCP 1 Session Progress
• Question Browser
Questions Correct 9
• Timed Test Male infertility in cystic fibrosis is mostly due to
which of the following? Questions Incorrect 10
• Mock Exam
Questions Total 19
• Past Papers
Impotence Questions Percentage 47 %
• Random Questions
Failure of development of the vas Your answer
• My Performance deferens More
• Media Bank Median survival age of 20 years
• New Multimedia Delayed puberty
Online Extras Decreased spermatogenesis
Library
Community
Help
PasTest Store Most men with cystic fibrosis are infertile due to failure
of development of the vas deferens. Impotence and
decreased spermatogenesis are not seen. Delayed
My Account puberty occurs in cystic fibrosis but is not a cause of
infertility. The median survival is now predicted to be at
Profile least 40 years for children born in the 1990s.
Newsletters
My Career and Exams 431
Order History Reference: Normal Values
Learning Goals Haematology
Question Filters
Immunoglobulins
Biochemistry
Security
Diabetes
Change Password
Sign Out Endocrinology

Blood gases

CSF

© 2011 PasTest Ltd | About Us | Contact Us | Help

file:///E|/Shakhawan/Respiratory%20S/42a.htm[3/13/2012 3:57:59 PM]


MyPasTest

Main Navigation
Question Browser: MRCP 1
Home
Subscriptions Question Browser Exam Builder Saved Exams
» MRCP 1 Session Progress
• Question Browser
Questions Correct 9
• Timed Test A 56-year-old man with confirmed squamous cell
carcinoma of the right upper lobe of the lung has a normal Questions Incorrect 10
• Mock Exam
FEV1 and normal serum biochemistry. Which one of the Questions Total 19
• Past Papers
following investigations is most appropriate to assess Questions Percentage 47 %
• Random Questions operability?
• My Performance More
Bone scan
• Media Bank
Chest CT scan
• New Multimedia
Differential perfusion lung scan Reference: Normal Values
Online Extras
Measurement of total lung capacity Haematology
Library
Sputum cytology Immunoglobulins
Community
Help 432 Biochemistry
PasTest Store Diabetes
Endocrinology
My Account
Blood gases
Profile
CSF
Newsletters
My Career and Exams
Order History
Learning Goals
Question Filters

Security

Change Password
Sign Out

© 2011 PasTest Ltd | About Us | Contact Us | Help

file:///E|/Shakhawan/Respiratory%20S/43.htm[3/13/2012 3:58:00 PM]


MyPasTest

Main Navigation
Question Browser: MRCP 1
Home
Subscriptions Question Browser Exam Builder Saved Exams
» MRCP 1 Session Progress
• Question Browser
Questions Correct 10
• Timed Test A 56-year-old man with confirmed squamous cell
carcinoma of the right upper lobe of the lung has a normal Questions Incorrect 10
• Mock Exam
FEV1 and normal serum biochemistry. Which one of the Questions Total 20
• Past Papers
following investigations is most appropriate to assess Questions Percentage 50 %
• Random Questions operability?
• My Performance More
Bone scan
• Media Bank
Chest CT scan Your answer
• New Multimedia
Differential perfusion lung scan
Online Extras
Measurement of total lung capacity
Library
Sputum cytology
Community
Help
PasTest Store

A chest CT scan is the best method for staging


My Account squamous cell carcinoma of the lung. This would
indicate the extent of involvement and surgical
Profile approach. Five-year survival rates are > 75% in stage I
Newsletters disease (nodes 0, tumour confined within visceral
pleura) and 55% in stage II disease, which includes
My Career and Exams resection in patients with ipsilateral peribronchial or
Order History hilar node involvement. Sputum cytology is irrelevant, Reference: Normal Values
as the diagnosis has already been established.
Learning Goals Haematology
Measurement of total lung capacity and differential
Question Filters perfusion lung scans is not helpful in staging. A bone
Immunoglobulins
scan is not required as there is no clinical,
haematological or biochemical evidence of tumour Biochemistry
Security spread to bony sites.
Diabetes
Change Password
Sign Out Endocrinology
432
Blood gases

CSF

© 2011 PasTest Ltd | About Us | Contact Us | Help

file:///E|/Shakhawan/Respiratory%20S/43a.htm[3/13/2012 3:58:02 PM]


MyPasTest

Main Navigation
Question Browser: MRCP 1
Home
Subscriptions Question Browser Exam Builder Saved Exams
» MRCP 1 Session Progress
• Question Browser
Questions Correct 10
• Timed Test Which of the following conditions is most likely to be
associated with a decreased DLCO? Questions Incorrect 10
• Mock Exam
Questions Total 20
• Past Papers
Dehydration Questions Percentage 50 %
• Random Questions
Polycythaemia
• My Performance More
Pulmonary embolism
• Media Bank
Exercise
• New Multimedia
Acute poliomyelitis Reference: Normal Values
Online Extras
433 Haematology
Library
Community Immunoglobulins
Help Biochemistry
PasTest Store Diabetes
Endocrinology
My Account
Blood gases
Profile
CSF
Newsletters
My Career and Exams
Order History
Learning Goals
Question Filters

Security

Change Password
Sign Out

© 2011 PasTest Ltd | About Us | Contact Us | Help

file:///E|/Shakhawan/Respiratory%20S/44.htm[3/13/2012 3:58:03 PM]


MyPasTest

Main Navigation
Question Browser: MRCP 1
Home
Subscriptions Question Browser Exam Builder Saved Exams
» MRCP 1 Session Progress
• Question Browser
Questions Correct 11
• Timed Test Which of the following conditions is most likely to be
associated with a decreased DLCO? Questions Incorrect 10
• Mock Exam
Questions Total 21
• Past Papers
Dehydration Questions Percentage 52 %
• Random Questions
Polycythaemia
• My Performance More
Pulmonary embolism Your answer
• Media Bank
Exercise
• New Multimedia
Acute poliomyelitis
Online Extras
Library
Community
Help DLCO, the diffusion capacity in the lung for carbon
PasTest Store monoxide, is decreased in any condition that reduces
the effective alveolar surface area or affects the alveolar
membrane. Examples include pulmonary embolism and
My Account emphysema in which the alveolar surface area is
reduced.
Profile
Newsletters
433
My Career and Exams
Order History Reference: Normal Values
Learning Goals Haematology
Question Filters
Immunoglobulins
Biochemistry
Security
Diabetes
Change Password
Sign Out Endocrinology

Blood gases

CSF

© 2011 PasTest Ltd | About Us | Contact Us | Help

file:///E|/Shakhawan/Respiratory%20S/44a.htm[3/13/2012 3:58:04 PM]


MyPasTest

Main Navigation
Question Browser: MRCP 1
Home
Subscriptions Question Browser Exam Builder Saved Exams
» MRCP 1 Session Progress
• Question Browser
Questions Correct 11
• Timed Test Which of the following statements applies to the
epidemiology and characteristic presentation of Questions Incorrect 10
• Mock Exam
bronchial asthma? Questions Total 21
• Past Papers
Questions Percentage 52 %
• Random Questions The associated mortality rate has increased during
• My Performance the past three decades More
• Media Bank The incidence is higher in males than females

• New Multimedia Adults typically present with nocturnal cough as an


isolated symptom Reference: Normal Values
Online Extras
Bronchial hyper-reactivity is strongly associated with Haematology
Library atopy
Community Chronic asthma may decrease bone age by one year Immunoglobulins
Help in an 8-year-old child Biochemistry
PasTest Store 434 Diabetes
Endocrinology
My Account
Blood gases
Profile
CSF
Newsletters
My Career and Exams
Order History
Learning Goals
Question Filters

Security

Change Password
Sign Out

© 2011 PasTest Ltd | About Us | Contact Us | Help

file:///E|/Shakhawan/Respiratory%20S/45.htm[3/13/2012 3:58:06 PM]


MyPasTest

Main Navigation
Question Browser: MRCP 1
Home
Subscriptions Question Browser Exam Builder Saved Exams
» MRCP 1 Session Progress
• Question Browser
Questions Correct 11
• Timed Test Which of the following statements applies to the
epidemiology and characteristic presentation of Questions Incorrect 11
• Mock Exam
bronchial asthma? Questions Total 22
• Past Papers
Questions Percentage 50 %
• Random Questions The associated mortality rate has
• My Performance increased during the past three More
decades
• Media Bank
The incidence is higher in males
• New Multimedia than females
Online Extras Adults typically present with Your answer
Library nocturnal cough as an isolated
symptom
Community
Bronchial hyper-reactivity is strongly Correct answer
Help associated with atopy
PasTest Store Chronic asthma may decrease bone
age by one year in an 8-year-old
child
My Account

Profile
Newsletters
My Career and Exams The mortality rate from asthma has not changed in the
Order History UK during the last 5 years, even though there is now Reference: Normal Values
better recognition and better treatment of asthma.
Learning Goals Nocturnal cough may be the only symptom in children. Haematology
Question Filters It is common for asthma to begin in childhood, and
generally occurs in atopic individuals who readily form Immunoglobulins
IgE antibodies to commonly encountered antigens.
Biochemistry
Security
Diabetes
Change Password 434
Sign Out Endocrinology

Blood gases

CSF

© 2011 PasTest Ltd | About Us | Contact Us | Help

file:///E|/Shakhawan/Respiratory%20S/45a.htm[3/13/2012 3:58:07 PM]


MyPasTest

Main Navigation
Question Browser: MRCP 1
Home
Subscriptions Question Browser Exam Builder Saved Exams
» MRCP 1 Session Progress
• Question Browser
Questions Correct 11
• Timed Test Which of the following statements applies to the peak
expiratory flow rate? Questions Incorrect 11
• Mock Exam
Questions Total 22
• Past Papers
It is a less sensitive parameter for assessing Questions Percentage 50 %
• Random Questions improvement to therapy in patients with acute
• My Performance bronchial asthma More
• Media Bank It measures small-airway obstruction

• New Multimedia It is more related to age than height


Reference: Normal Values
Online Extras Less than 50% of normal is an indication for
aminophylline therapy as the next step in those with Haematology
Library acute asthma
Community It is usually effort-dependent, even in people who Immunoglobulins
Help have practiced the test Biochemistry
PasTest Store 435 Diabetes
Endocrinology
My Account
Blood gases
Profile
CSF
Newsletters
My Career and Exams
Order History
Learning Goals
Question Filters

Security

Change Password
Sign Out

© 2011 PasTest Ltd | About Us | Contact Us | Help

file:///E|/Shakhawan/Respiratory%20S/46.htm[3/13/2012 3:58:09 PM]


MyPasTest

Main Navigation
Question Browser: MRCP 1
Home
Subscriptions Question Browser Exam Builder Saved Exams
» MRCP 1 Session Progress
• Question Browser
Questions Correct 11
• Timed Test Which of the following statements applies to the peak
expiratory flow rate? Questions Incorrect 12
• Mock Exam
Questions Total 23
• Past Papers
It is a less sensitive parameter for Questions Percentage 47 %
• Random Questions assessing improvement to therapy in
• My Performance patients with acute bronchial asthma More
• Media Bank It measures small-airway Correct answer
obstruction
• New Multimedia
It is more related to age than height
Online Extras
Less than 50% of normal is an
Library indication for aminophylline therapy
Community as the next step in those with acute
asthma
Help
It is usually effort-dependent, even Your answer
PasTest Store in people who have practiced the test

My Account

Profile
PEFR is a sensitive measurement of small-airway
Newsletters
obstruction, although the maximal mid-expiratory flow
My Career and Exams rate is more changed in mild disease. The ease of
Order History performing the PEFR at the bedside has made it more Reference: Normal Values
relevant in clinical practice. It is more related to height
Learning Goals than age, and is effort-independent if the proper Haematology
Question Filters technique is used.
Immunoglobulins
Biochemistry
Security 435
Diabetes
Change Password
Sign Out Endocrinology

Blood gases

CSF

© 2011 PasTest Ltd | About Us | Contact Us | Help

file:///E|/Shakhawan/Respiratory%20S/46a.htm[3/13/2012 3:58:10 PM]


MyPasTest

Main Navigation
Question Browser: MRCP 1
Home
Subscriptions Question Browser Exam Builder Saved Exams
» MRCP 1 Session Progress
• Question Browser
Questions Correct 11
• Timed Test Which of the following does NOT increase the risk of
death in patients with severe pneumonia? Questions Incorrect 12
• Mock Exam
Questions Total 23
• Past Papers
Diastolic blood pressure < 60 mmHg Questions Percentage 47 %
• Random Questions
Age 49 years
• My Performance More
Urea > 7 mmol/l
• Media Bank
WBC count < 4 × 10 9 /l
• New Multimedia
Atrial fibrillation Reference: Normal Values
Online Extras
Haematology
Library 436
Community Immunoglobulins
Help Biochemistry
PasTest Store Diabetes
Endocrinology
My Account
Blood gases
Profile
CSF
Newsletters
My Career and Exams
Order History
Learning Goals
Question Filters

Security

Change Password
Sign Out

© 2011 PasTest Ltd | About Us | Contact Us | Help

file:///E|/Shakhawan/Respiratory%20S/47.htm[3/13/2012 3:58:12 PM]


MyPasTest

Main Navigation
Question Browser: MRCP 1
Home
Subscriptions Question Browser Exam Builder Saved Exams
» MRCP 1 Session Progress
• Question Browser
Questions Correct 12
• Timed Test Which of the following does NOT increase the risk of
death in patients with severe pneumonia? Questions Incorrect 12
• Mock Exam
Questions Total 24
• Past Papers
Diastolic blood pressure < 60 mmHg Questions Percentage 50 %
• Random Questions
Age 49 years Your answer
• My Performance More
Urea > 7 mmol/l
• Media Bank
WBC count < 4 × 10 9 /l
• New Multimedia
Atrial fibrillation
Online Extras
Library
Community
Help Patients over 60 years of age with severe pneumonia
PasTest Store have an increased of risk of dying. The other factors
that increase the risk of death are tachypnoea
(respiratory rate > 30/min), underlying disease,
My Account confusion, multi-lobular involvement, albumin < 35 g/l,
hypoxia p a (O2 ) < 8 kPa, leucocytosis > 20 × 10 9 /l and
Profile
bacteraemia. Leukopaenia is also associated with a
Newsletters higher rate of mortality in pneumonia.
My Career and Exams
http://preview.tinyurl.com/lcgp76
Order History Reference: Normal Values
Learning Goals Haematology
Question Filters 436
Immunoglobulins
Biochemistry
Security
Diabetes
Change Password
Sign Out Endocrinology

Blood gases

CSF

© 2011 PasTest Ltd | About Us | Contact Us | Help

file:///E|/Shakhawan/Respiratory%20S/47a.htm[3/13/2012 3:58:13 PM]


MyPasTest

Main Navigation
Question Browser: MRCP 1
Home
Subscriptions Question Browser Exam Builder Saved Exams
» MRCP 1 Session Progress
• Question Browser
Questions Correct 12
• Timed Test A 26-year female iv drug user presents with a productive
cough and fever of 2–3 days’ duration She had flu last Questions Incorrect 12
• Mock Exam
week. Other than a leucocytosis and high CRP, her blood Questions Total 24
• Past Papers results are normal. A chest X-ray shows bilateral cavitating
Questions Percentage 50 %
• Random Questions pneumonia. What is the most probable cause of her
pneumonia? More
• My Performance
• Media Bank Pneumococcal pneumonia
• New Multimedia Staphylococcal pneumonia Reference: Normal Values
Online Extras Pneumocystis jiroveci pneumonia
Haematology
Library Klebsiella pneumonia
Community Fungal pneumonia Immunoglobulins
Help Biochemistry
437
PasTest Store Diabetes
Endocrinology
My Account
Blood gases
Profile
CSF
Newsletters
My Career and Exams
Order History
Learning Goals
Question Filters

Security

Change Password
Sign Out

© 2011 PasTest Ltd | About Us | Contact Us | Help

file:///E|/Shakhawan/Respiratory%20S/48.htm[3/13/2012 3:58:14 PM]


MyPasTest

Main Navigation
Question Browser: MRCP 1
Home
Subscriptions Question Browser Exam Builder Saved Exams
» MRCP 1 Session Progress
• Question Browser
Questions Correct 13
• Timed Test A 26-year female iv drug user presents with a productive
cough and fever of 2–3 days’ duration She had flu last Questions Incorrect 12
• Mock Exam
week. Other than a leucocytosis and high CRP, her blood Questions Total 25
• Past Papers results are normal. A chest X-ray shows bilateral cavitating
Questions Percentage 52 %
• Random Questions pneumonia. What is the most probable cause of her
pneumonia? More
• My Performance
• Media Bank Pneumococcal pneumonia
• New Multimedia Staphylococcal pneumonia Your answer
Online Extras Pneumocystis jiroveci pneumonia
Library Klebsiella pneumonia
Community Fungal pneumonia
Help
PasTest Store

My Account In general, staphylococcal pneumonia follows a viral


infection – usually with flu-like symptoms. This type of
Profile pneumonia is frequently seen in iv drug abusers and
patients with central line. It is also common in those
Newsletters
patients with an underlying disease, eg leukaemia,
My Career and Exams lymphoma or cystic fibrosis. X-rays show bilateral
Order History cavitating bronchopneumonia. Pneumothorax, effusion Reference: Normal Values
and empyema are frequent. Intravenous antibiotics
Learning Goals should be administered properly. The drug of choice is Haematology
Question Filters flucloxacillin. Klebsiella spp. also cause cavitating
pneumonia, particularly of the upper lobes, and most Immunoglobulins
commonly in the elderly.
Biochemistry
Security
Diabetes
Change Password 437
Sign Out Endocrinology

Blood gases

CSF

© 2011 PasTest Ltd | About Us | Contact Us | Help

file:///E|/Shakhawan/Respiratory%20S/48a.htm[3/13/2012 3:58:16 PM]


MyPasTest

Main Navigation
Question Browser: MRCP 1
Home
Subscriptions Question Browser Exam Builder Saved Exams
» MRCP 1 Session Progress
• Question Browser
Questions Correct 13
• Timed Test A 62-year-old smoker with a 2-day history of cough and
fever is admitted with a diagnosis of left-sided pneumonia Questions Incorrect 12
• Mock Exam
and left-sided pleural effusion up to the sixth intercostal Questions Total 25
• Past Papers space. Pleural fluid is aspirated and sent for tests. Which
Questions Percentage 52 %
• Random Questions of the following is an indication for inserting a chest
drain? More
• My Performance
• Media Bank Blood-stained pleural fluid
• New Multimedia Pleural fluid pH < 7.2 Reference: Normal Values
Online Extras Serous pleural fluid
Haematology
Library Pleural fluid lactate dehydrogenase > 200 IU/l
Community Pleural fluid glucose > 2 mmol/l Immunoglobulins
Help Biochemistry
438
PasTest Store Diabetes
Endocrinology
My Account
Blood gases
Profile
CSF
Newsletters
My Career and Exams
Order History
Learning Goals
Question Filters

Security

Change Password
Sign Out

© 2011 PasTest Ltd | About Us | Contact Us | Help

file:///E|/Shakhawan/Respiratory%20S/49.htm[3/13/2012 3:58:17 PM]


MyPasTest

Main Navigation
Question Browser: MRCP 1
Home
Subscriptions Question Browser Exam Builder Saved Exams
» MRCP 1 Session Progress
• Question Browser
Questions Correct 14
• Timed Test A 62-year-old smoker with a 2-day history of cough and
fever is admitted with a diagnosis of left-sided pneumonia Questions Incorrect 12
• Mock Exam
and left-sided pleural effusion up to the sixth intercostal Questions Total 26
• Past Papers space. Pleural fluid is aspirated and sent for tests. Which
Questions Percentage 53 %
• Random Questions of the following is an indication for inserting a chest
drain? More
• My Performance
• Media Bank Blood-stained pleural fluid
• New Multimedia Pleural fluid pH < 7.2 Your answer
Online Extras Serous pleural fluid
Library Pleural fluid lactate dehydrogenase >
Community 200 IU/l
Help Pleural fluid glucose > 2 mmol/l
PasTest Store

My Account
Pleural effusions should be drained when the fluid is
Profile infected, as they should when the pH is < 7.2. Pleural
fluid should also be drained when a Gram stain shows
Newsletters
the presence of organisms, or when the fluid is frankly
My Career and Exams purulent and clinical improvement is slow despite
Order History antibiotic therapy. Reference: Normal Values
Learning Goals Haematology
Question Filters 438
Immunoglobulins
Biochemistry
Security
Diabetes
Change Password
Sign Out Endocrinology

Blood gases

CSF

© 2011 PasTest Ltd | About Us | Contact Us | Help

file:///E|/Shakhawan/Respiratory%20S/49a.htm[3/13/2012 3:58:18 PM]


MyPasTest

Main Navigation
Question Browser: MRCP 1
Home
Subscriptions Question Browser Exam Builder Saved Exams
» MRCP 1 Session Progress
• Question Browser
Questions Correct 14
• Timed Test A 75-year-old man with a BMI of 31 presents with a
history of worsening breathlessness and cough of about 1- Questions Incorrect 12
• Mock Exam
week duration. He is normally short of breath and has had Questions Total 26
• Past Papers to give up walking to the shops which are only 300m away.
Questions Percentage 53 %
• Random Questions He has a chronic cough which is worse in the morning. He
is a smoker and still smoked 20 cigarettes per day until he More
• My Performance
became ill this time. His arterial blood gases in room air are
• Media Bank as follows: pH 7.24, p a (O2 ) 6.2 kPa, p a (CO 2 ) 9.8 kPa,
• New Multimedia bicarbonate 33 mmol/l.
Reference: Normal Values
Online Extras Which of the following is the most likely diagnosis?
Haematology
Library
Pulmonary embolus Immunoglobulins
Community
Acute asthma
Help Biochemistry
Obstructive sleep apnoea (OSA)
PasTest Store Diabetes
Acute exacerbation of chronic obstructive pulmonary
disease (COPD) Endocrinology
My Account
Pulmonary oedema Blood gases
Profile
439 CSF
Newsletters
My Career and Exams
Order History
Learning Goals
Question Filters

Security

Change Password
Sign Out

© 2011 PasTest Ltd | About Us | Contact Us | Help

file:///E|/Shakhawan/Respiratory%20S/50.htm[3/13/2012 3:58:20 PM]


MyPasTest

Main Navigation
Question Browser: MRCP 1
Home
Subscriptions Question Browser Exam Builder Saved Exams
» MRCP 1 Session Progress
• Question Browser
Questions Correct 15
• Timed Test A 72-year-old woman is admitted with a sudden-onset,
left-sided pleuritic chest pain with shortness of breath. She Questions Incorrect 12
• Mock Exam
is being treated for asthma, which has been well controlled Questions Total 27
• Past Papers
on a low dose of inhaled corticosteroids and long-acting β- Questions Percentage 55 %
• Random Questions agonist. She underwent left hemiarthroplasty 12 days ago,
• My Performance and was discharged as she was doing well. Her chest is More
clear on auscultation. She is tachycardic (132 beats/min)
• Media Bank and an ECG shows sinus tachycardia. Her peak expiratory
• New Multimedia flow (PEF) rate is 300 l/min (best 400 l/min). Arterial blood
gases are as follows: pH 7.34, p a (O2 ) 7.6 kPa, p a (CO 2 ) 3.5 Reference: Normal Values
Online Extras
kPa. She is started on oxygen. A chest radiograph is Haematology
Library normal.
Community Immunoglobulins
What would be the most appropriate immediate
Help action taken by you as a medical SHO? Biochemistry
PasTest Store Diabetes
Start nebulised bronchodilators and monitor PEF rate
Request D-dimers urgently Endocrinology
My Account
Start low molecular weight heparin suspecting PE, Blood gases
Profile and request a V/Q scan
CSF
Newsletters Start low molecular weight heparin suspecting PE,
My Career and Exams and request CT pulmonary angiography

Order History Request a chest radiograph in expiration

Learning Goals 440


Question Filters

Security

Change Password
Sign Out

© 2011 PasTest Ltd | About Us | Contact Us | Help

file:///E|/Shakhawan/Respiratory%20S/51.htm[3/13/2012 3:58:21 PM]


MyPasTest

Main Navigation
Question Browser: MRCP 1
Home
Subscriptions Question Browser Exam Builder Saved Exams
» MRCP 1 Session Progress
• Question Browser
Questions Correct 16
• Timed Test A 72-year-old woman is admitted with a sudden-onset,
left-sided pleuritic chest pain with shortness of breath. She Questions Incorrect 12
• Mock Exam
is being treated for asthma, which has been well controlled Questions Total 28
• Past Papers
on a low dose of inhaled corticosteroids and long-acting β- Questions Percentage 57 %
• Random Questions agonist. She underwent left hemiarthroplasty 12 days ago,
• My Performance and was discharged as she was doing well. Her chest is More
clear on auscultation. She is tachycardic (132 beats/min)
• Media Bank and an ECG shows sinus tachycardia. Her peak expiratory
• New Multimedia flow (PEF) rate is 300 l/min (best 400 l/min). Arterial blood
gases are as follows: pH 7.34, p a (O2 ) 7.6 kPa, p a (CO 2 ) 3.5
Online Extras
kPa. She is started on oxygen. A chest radiograph is
Library normal.
Community
What would be the most appropriate immediate
Help action taken by you as a medical SHO?
PasTest Store
Start nebulised bronchodilators and
monitor PEF rate
My Account Request D-dimers urgently
Profile Start low molecular weight heparin
Newsletters suspecting PE, and request a V/Q scan

My Career and Exams Start low molecular weight heparin Your answer
suspecting PE, and request CT
Order History pulmonary angiography Reference: Normal Values
Learning Goals Request a chest radiograph in Haematology
Question Filters expiration
Immunoglobulins
Biochemistry
Security
Diabetes
Change Password Her PEF rate is only mildly reduced (75% best). It is
Sign Out unlikely that this patient’s symptoms are due to an Endocrinology
exacerbation of her asthma. A small pneumothorax, not Blood gases
apparent on the inspiratory chest radiograph, is also
unlikely since it would not cause marked hypoxia. The CSF
symptoms and findings point towards a pulmonary
embolism (PE), for which the clinical probability is high.
D-Dimers should not be measured, since the result
would not alter the need for definitive investigation:
because she has had a recent operation it would be
high anyway. D-Dimers should only be measured when
the probability of PE is low and further investigations
would not be pursued.
A CT pulmonary angiogram would be the imaging
procedure of choice in this case, after starting low
molecular weight heparin, most Emergency departments
have these much more rapidly available than V/Q scan
and the diagnosis can often be made the same evening
as admission.

440

© 2011 PasTest Ltd | About Us | Contact Us | Help

file:///E|/Shakhawan/Respiratory%20S/51a.htm[3/13/2012 3:58:23 PM]


MyPasTest

Main Navigation
Question Browser: MRCP 1
Home
Subscriptions Question Browser Exam Builder Saved Exams
» MRCP 1 Session Progress
• Question Browser
Questions Correct 16
• Timed Test A 35-year-old woman who was previously fit and well
presents with breathlessness that has been getting worse Questions Incorrect 12
• Mock Exam
over 3 or 4 months. Her sister died a few years ago with a Questions Total 28
• Past Papers lung disease. On examination, her jugular venous pressure
Questions Percentage 57 %
• Random Questions is raised and she has a palpable heave at the left sternal
edge. Her BMI is 23, BP is 135/72 mmHg, and her pulse is More
• My Performance
80/min and regular. What would be your provisional
• Media Bank diagnosis?
• New Multimedia
Familial primary pulmonary hypertension Reference: Normal Values
Online Extras
Tricuspid regurgitation Haematology
Library
Chronic pulmonary thromboembolism
Community Immunoglobulins
Constrictive pericarditis
Help Biochemistry
Pulmonary venous hypertension
PasTest Store Diabetes
441
Endocrinology
My Account
Blood gases
Profile
CSF
Newsletters
My Career and Exams
Order History
Learning Goals
Question Filters

Security

Change Password
Sign Out

© 2011 PasTest Ltd | About Us | Contact Us | Help

file:///E|/Shakhawan/Respiratory%20S/52.htm[3/13/2012 3:58:24 PM]


MyPasTest

Main Navigation
Question Browser: MRCP 1
Home
Subscriptions Question Browser Exam Builder Saved Exams
» MRCP 1 Session Progress
• Question Browser
Questions Correct 17
• Timed Test A 35-year-old woman who was previously fit and well
presents with breathlessness that has been getting worse Questions Incorrect 12
• Mock Exam
over 3 or 4 months. Her sister died a few years ago with a Questions Total 29
• Past Papers lung disease. On examination, her jugular venous pressure
Questions Percentage 58 %
• Random Questions is raised and she has a palpable heave at the left sternal
edge. Her BMI is 23, BP is 135/72 mmHg, and her pulse is More
• My Performance
80/min and regular. What would be your provisional
• Media Bank diagnosis?
• New Multimedia
Familial primary pulmonary Your answer
Online Extras hypertension
Library Tricuspid regurgitation
Community Chronic pulmonary thromboembolism
Help Constrictive pericarditis
PasTest Store Pulmonary venous hypertension

My Account

Profile
Primary pulmonary hypertension presents with
Newsletters
breathlessness, fatigue, angina (due to right ventricular
My Career and Exams ischaemia) or presyncope/syncope. About 6% of all
Order History patients with primary pulmonary hypertension have a Reference: Normal Values
family history of the condition, which exhibits an
Learning Goals autosomal-dominant pattern of inheritance with Haematology
Question Filters incomplete penetrance. Physical signs include elevated
JVP, left parasternal heave, pansystolic murmur Immunoglobulins
(tricuspid regurgitation) and right ventricular S4,
Biochemistry
Security peripheral oedema. An important differential diagnosis
of primary pulmonary hypertension is chronic Diabetes
Change Password pulmonary thromboembolism.
Sign Out Endocrinology

Blood gases
441
CSF

© 2011 PasTest Ltd | About Us | Contact Us | Help

file:///E|/Shakhawan/Respiratory%20S/52a.htm[3/13/2012 3:58:26 PM]


MyPasTest

Main Navigation
Question Browser: MRCP 1
Home
Subscriptions Question Browser Exam Builder Saved Exams
» MRCP 1 Session Progress
• Question Browser
Questions Correct 17
• Timed Test A 72-year-old woman is admitted with an infective
exacerbation of chronic obstructive pulmonary disease Questions Incorrect 12
• Mock Exam
(COPD). On admission her blood gases taken while Questions Total 29
• Past Papers breathing 28% oxygen are as follows: pH 7.31, p(O2 ) 7.9
Questions Percentage 58 %
• Random Questions kPa, p(CO 2 ) 7.5 kPa. Which of the following best
• My Performance describes the blood gas picture? More
• Media Bank
Compensated type-1 respiratory failure
• New Multimedia
Compensated type-2 respiratory failure Reference: Normal Values
Online Extras
Decompensated type-1 respiratory failure Haematology
Library
Decompensated type-2 respiratory failure
Community Immunoglobulins
Partially compensated respiratory alkalosis
Help Biochemistry
PasTest Store 442
Diabetes
Endocrinology
My Account
Blood gases
Profile
CSF
Newsletters
My Career and Exams
Order History
Learning Goals
Question Filters

Security

Change Password
Sign Out

© 2011 PasTest Ltd | About Us | Contact Us | Help

file:///E|/Shakhawan/Respiratory%20S/53.htm[3/13/2012 3:58:27 PM]


MyPasTest

Main Navigation
Question Browser: MRCP 1
Home
Subscriptions Question Browser Exam Builder Saved Exams
» MRCP 1 Session Progress
• Question Browser
Questions Correct 18
• Timed Test A 72-year-old woman is admitted with an infective
exacerbation of chronic obstructive pulmonary disease Questions Incorrect 12
• Mock Exam
(COPD). On admission her blood gases taken while Questions Total 30
• Past Papers breathing 28% oxygen are as follows: pH 7.31, p(O2 ) 7.9
Questions Percentage 60 %
• Random Questions kPa, p(CO 2 ) 7.5 kPa. Which of the following best
• My Performance describes the blood gas picture? More
• Media Bank
Compensated type-1 respiratory failure
• New Multimedia
Compensated type-2 respiratory failure
Online Extras
Decompensated type-1 respiratory
Library failure
Community Decompensated type-2 respiratory Your answer
Help failure
PasTest Store Partially compensated respiratory
alkalosis

My Account

Profile
Newsletters The patient’s blood gases show that she is hypoxic on
oxygen, the p a (CO 2 ) is elevated (p a (CO 2 ) > 6 kPa) and
My Career and Exams
the pH is low. These confirm a respiratory acidosis.
Order History There is hypoxia combined with CO2 retention. Reference: Normal Values
Learning Goals Therefore, the blood gas picture is of decompensated Haematology
Question Filters type-2 (respiratory) failure. The prognosis is poor
unless treated appropriately. Immunoglobulins

Treatment to be considered in this case is non-invasive, Biochemistry


Security
positive-pressure ventilation (NIPPV). Data from good-
Diabetes
Change Password quality randomised controlled trials permit NPPV to be
recommended as the first-line intervention, coupled Endocrinology
Sign Out with the usual medical care, in all suitable patients with
respiratory failure secondary to an acute exacerbation of Blood gases
COPD. A trial of NPPV should be considered early in the
course of respiratory failure, and before severe acidosis CSF
ensures, as a means of avoiding endotracheal
intubation, reducing mortality and treatment failure.

442

© 2011 PasTest Ltd | About Us | Contact Us | Help

file:///E|/Shakhawan/Respiratory%20S/53a.htm[3/13/2012 3:58:28 PM]


MyPasTest

Main Navigation
Question Browser: MRCP 1
Home
Subscriptions Question Browser Exam Builder Saved Exams
» MRCP 1 Session Progress
• Question Browser
Questions Correct 18
• Timed Test An anxious, 25-year-old saleswoman presented with mild
shortness of breath on exertion, which had come on Questions Incorrect 12
• Mock Exam
gradually over several months. The symptom was Questions Total 30
• Past Papers intermittent and seemed to get worse in the evening. She
Questions Percentage 60 %
• Random Questions has also been on treatment for depression over the last 2
months. On examination she has minimal weakness of More
• My Performance
shoulder abductors and slight weakness of eye closure
• Media Bank bilaterally. Deep tendon reflexes are present and
• New Multimedia symmetrical throughout and plantars responses are flexor.
You now have the results of the investigations: FBC, U&E, Reference: Normal Values
Online Extras LFT, normal; chest X-ray and lung function tests, normal;
ECG, normal. Haematology
Library
Community Immunoglobulins
What is the likely diagnosis?
Help Biochemistry
Angina
PasTest Store Diabetes
Eaton–Lambert syndrome
Myasthenia gravis Endocrinology
My Account
Somatisation disorder Blood gases
Profile
TIA CSF
Newsletters
My Career and Exams 443
Order History
Learning Goals
Question Filters

Security

Change Password
Sign Out

© 2011 PasTest Ltd | About Us | Contact Us | Help

file:///E|/Shakhawan/Respiratory%20S/54.htm[3/13/2012 3:58:30 PM]


MyPasTest

Main Navigation
Question Browser: MRCP 1
Home
Subscriptions Question Browser Exam Builder Saved Exams
» MRCP 1 Session Progress
• Question Browser
Questions Correct 18
• Timed Test An anxious, 25-year-old saleswoman presented with mild
shortness of breath on exertion, which had come on Questions Incorrect 13
• Mock Exam
gradually over several months. The symptom was Questions Total 31
• Past Papers intermittent and seemed to get worse in the evening. She
Questions Percentage 58 %
• Random Questions has also been on treatment for depression over the last 2
months. On examination she has minimal weakness of More
• My Performance
shoulder abductors and slight weakness of eye closure
• Media Bank bilaterally. Deep tendon reflexes are present and
• New Multimedia symmetrical throughout and plantars responses are flexor.
You now have the results of the investigations: FBC, U&E,
Online Extras LFT, normal; chest X-ray and lung function tests, normal;
Library ECG, normal.
Community What is the likely diagnosis?
Help
Angina
PasTest Store
Eaton–Lambert syndrome
Myasthenia gravis Correct answer
My Account
Somatisation disorder Your answer
Profile
TIA
Newsletters
My Career and Exams
Order History Reference: Normal Values
Learning Goals This is a case of myasthenia gravis. Muscle weakness Haematology
Question Filters may not be apparent on a single examination, so the
examination should be repeated. Most affected is the Immunoglobulins
ocular and shoulder-girdle muscle. Respiratory and
proximal lower limb muscles may be involved early in Biochemistry
Security
the disease. Breathlessness may develop early and Diabetes
Change Password cause sudden death. Swallowing problems, slurred
speech and difficulty in chewing may be caused by Endocrinology
Sign Out
bulbar involvement. Asymmetrical involvement of the
external ocular muscle may mimic cranial nerve palsy. Blood gases
Pupillary reflex is normal. Mild ptosis and weak facial
CSF
muscles make patients appear depressed. Thymic
enlargement may be seen in just 15% of patients.
Myasthenia gravis is autoimmune in origin.

443

© 2011 PasTest Ltd | About Us | Contact Us | Help

file:///E|/Shakhawan/Respiratory%20S/54a.htm[3/13/2012 3:58:31 PM]


MyPasTest

Main Navigation
Question Browser: MRCP 1
Home
Subscriptions Question Browser Exam Builder Saved Exams
» MRCP 1 Session Progress
• Question Browser
Questions Correct 18
• Timed Test A 25-year-old male patient with cystic fibrosis is to
undergo lung transplantation. Which of the following is Questions Incorrect 13
• Mock Exam
NOT true about the lung transplant in this patient? Questions Total 31
• Past Papers
Questions Percentage 58 %
• Random Questions Donor-selection parameters include age under 40
• My Performance years More
• Media Bank Donor’s chest measurements should be slightly
smaller than those of the recipient
• New Multimedia
It is essential to match for the ABO blood group Reference: Normal Values
Online Extras
Matching for the rhesus blood group compatibility is Haematology
Library not essential
Community A single lung transplant is preferred to double lung Immunoglobulins
Help transplant Biochemistry
PasTest Store 444 Diabetes
Endocrinology
My Account
Blood gases
Profile
CSF
Newsletters
My Career and Exams
Order History
Learning Goals
Question Filters

Security

Change Password
Sign Out

© 2011 PasTest Ltd | About Us | Contact Us | Help

file:///E|/Shakhawan/Respiratory%20S/55.htm[3/13/2012 3:58:33 PM]


MyPasTest

Main Navigation
Question Browser: MRCP 1
Home
Subscriptions Question Browser Exam Builder Saved Exams
» MRCP 1 Session Progress
• Question Browser
Questions Correct 18
• Timed Test A 25-year-old male patient with cystic fibrosis is to
undergo lung transplantation. Which of the following is Questions Incorrect 14
• Mock Exam
NOT true about the lung transplant in this patient? Questions Total 32
• Past Papers
Questions Percentage 56 %
• Random Questions Donor-selection parameters include
• My Performance age under 40 years More
• Media Bank Donor’s chest measurements should
be slightly smaller than those of the
• New Multimedia recipient
Online Extras It is essential to match for the ABO Your answer
Library blood group
Community Matching for the rhesus blood group
compatibility is not essential
Help
A single lung transplant is preferred Correct answer
PasTest Store to double lung transplant

My Account

Profile
Indications for the treatment are patient under 60 years
Newsletters
with life expectancy of less than 18 months. No
My Career and Exams underlying cancer or serious systemic disease should be
Order History present. Donor is matched for the ABO grouping and Reference: Normal Values
the rhesus blood group compatibility is not essential.
Learning Goals Donor should have good cardiac and lung function and Haematology
Question Filters should be under the age of 40. The chest of the donor
should be slightly less than the recipient. Double lung Immunoglobulins
transplant is usually performed because of the risk of
Biochemistry
Security chronic infection in the remaining lung.
Diabetes
Change Password
Sign Out 444 Endocrinology

Blood gases

CSF

© 2011 PasTest Ltd | About Us | Contact Us | Help

file:///E|/Shakhawan/Respiratory%20S/55a.htm[3/13/2012 3:58:34 PM]


MyPasTest

Main Navigation
Question Browser: MRCP 1
Home
Subscriptions Question Browser Exam Builder Saved Exams
» MRCP 1 Session Progress
• Question Browser
Questions Correct 18
• Timed Test A 76-year-old woman, who is generally fit, self-caring and
independent, is admitted with an acute COPD exacerbation. Questions Incorrect 14
• Mock Exam
She has been put on bronchodilator therapy. One hour after Questions Total 32
• Past Papers admission she remains distressed with a respiratory rate of
Questions Percentage 56 %
• Random Questions 30/minute and is peripherally cyanosed. Repeated arterial
blood gases show a severe respiratory acidosis. The report More
• My Performance
of her latest arterial blood gas is: pH 7.252, p a (O2 ) 6.5,
• Media Bank p a (CO 2 ) 9.8. She is on 28% O 2 . She is becoming
• New Multimedia exhausted and her oxygen saturation is 76%. The on-call
Reference: Normal Values
Online Extras intensivist says there are no intensive-care beds available
for mechanical ventilation. Which of the following Haematology
Library management options is correct for a patient in such a
Community situation? Immunoglobulins
Help Biochemistry
Continue with bronchodilator therapy and arrange
PasTest Store repeat arterial gases in an hour’s time Diabetes
Arrange for non-invasive, positive-pressure
ventilation Endocrinology
My Account
Transfer her to another hospital with a mechanical Blood gases
Profile ventilation facility
CSF
Newsletters Give an intravenous infusion of aminophylline
My Career and Exams Intubate the patient at the bedside and manually
Order History ventilate
Learning Goals 445
Question Filters

Security

Change Password
Sign Out

© 2011 PasTest Ltd | About Us | Contact Us | Help

file:///E|/Shakhawan/Respiratory%20S/56.htm[3/13/2012 3:58:36 PM]


MyPasTest

Main Navigation
Question Browser: MRCP 1
Home
Subscriptions Question Browser Exam Builder Saved Exams
» MRCP 1 Session Progress
• Question Browser
Questions Correct 18
• Timed Test A 76-year-old woman, who is generally fit, self-caring and
independent, is admitted with an acute COPD exacerbation. Questions Incorrect 15
• Mock Exam
She has been put on bronchodilator therapy. One hour after Questions Total 33
• Past Papers admission she remains distressed with a respiratory rate of
Questions Percentage 54 %
• Random Questions 30/minute and is peripherally cyanosed. Repeated arterial
blood gases show a severe respiratory acidosis. The report More
• My Performance
of her latest arterial blood gas is: pH 7.252, p a (O2 ) 6.5,
• Media Bank p a (CO 2 ) 9.8. She is on 28% O 2 . She is becoming
• New Multimedia exhausted and her oxygen saturation is 76%. The on-call
Online Extras intensivist says there are no intensive-care beds available
for mechanical ventilation. Which of the following
Library management options is correct for a patient in such a
Community situation?
Help
Continue with bronchodilator
PasTest Store therapy and arrange repeat arterial
gases in an hour’s time
Arrange for non-invasive, positive- Correct answer
My Account
pressure ventilation
Profile Transfer her to another hospital Your answer
Newsletters with a mechanical ventilation facility
My Career and Exams Give an intravenous infusion of
aminophylline
Order History Reference: Normal Values
Intubate the patient at the bedside
Learning Goals and manually ventilate Haematology
Question Filters
Immunoglobulins
Biochemistry
Security
The patient is definitely retaining CO2 , which needs to Diabetes
Change Password
be taken care of by support. Increasing the inhaled O 2 Endocrinology
Sign Out
would not be at all helpful for the patient. Enhancing
ventilation by unloading fatigued ventilatory muscles is Blood gases
an important goal in treating such exacerbations of
CSF
acute chronic obstructive pulmonary disease (COPD).
Non-invasive, positive-pressure ventilation (NIPPV)
should be considered when there is a need for
ventilatory assistance as indicated by worsening
dyspnoea, acute respiratory acidosis, CO2 retention and
worsening oxygenation. NIPPV has been shown to
reduce intubation rates, lower hospital mortality rates
and lead to shorter hospital stays.

445

© 2011 PasTest Ltd | About Us | Contact Us | Help

file:///E|/Shakhawan/Respiratory%20S/56a.htm[3/13/2012 3:58:37 PM]


MyPasTest

Main Navigation
Question Browser: MRCP 1
Home
Subscriptions Question Browser Exam Builder Saved Exams
» MRCP 1 Session Progress
• Question Browser
Questions Correct 18
• Timed Test Which of the following is not a common symptom of
lung cancer on presentation? Questions Incorrect 15
• Mock Exam
Questions Total 33
• Past Papers
Cough Questions Percentage 54 %
• Random Questions
Chest pain
• My Performance More
Cough and chest pain
• Media Bank
Coughing blood
• New Multimedia
Shortness of breath Reference: Normal Values
Online Extras
446 Haematology
Library
Community Immunoglobulins
Help Biochemistry
PasTest Store Diabetes
Endocrinology
My Account
Blood gases
Profile
CSF
Newsletters
My Career and Exams
Order History
Learning Goals
Question Filters

Security

Change Password
Sign Out

© 2011 PasTest Ltd | About Us | Contact Us | Help

file:///E|/Shakhawan/Respiratory%20S/57.htm[3/13/2012 3:58:39 PM]


MyPasTest

Main Navigation
Question Browser: MRCP 1
Home
Subscriptions Question Browser Exam Builder Saved Exams
» MRCP 1 Session Progress
• Question Browser
Questions Correct 19
• Timed Test Which of the following is not a common symptom of
lung cancer on presentation? Questions Incorrect 15
• Mock Exam
Questions Total 34
• Past Papers
Cough Questions Percentage 55 %
• Random Questions
Chest pain
• My Performance More
Cough and chest pain
• Media Bank
Coughing blood
• New Multimedia
Shortness of breath Your answer
Online Extras
Library
Community
Help Cough is the commonest symptom (about 41% of
PasTest Store patients) at presentation, followed by chest pain in 22%
of patients who have later been diagnosed to have lung
carcinoma. About 15% of patients present with both
My Account cough and chest pain. Only 7% of patients present with
the symptom of coughing up blood. In less than 5% of
Profile cases they present with other symptoms: shortness of
breath, hoarseness, weight loss, malaise and distant
Newsletters
spread.
My Career and Exams
Order History Reference: Normal Values
446
Learning Goals Haematology
Question Filters
Immunoglobulins
Biochemistry
Security
Diabetes
Change Password
Sign Out Endocrinology

Blood gases

CSF

© 2011 PasTest Ltd | About Us | Contact Us | Help

file:///E|/Shakhawan/Respiratory%20S/57a.htm[3/13/2012 3:58:40 PM]


MyPasTest

Main Navigation
Question Browser: MRCP 1
Home
Subscriptions Question Browser Exam Builder Saved Exams
» MRCP 1 Session Progress
• Question Browser
Questions Correct 0
• Timed Test A 48-year-old mechanic presented to the clinic complaining
of increasing shortness of breath over the last 3 months, Questions Incorrect 0
• Mock Exam
but with no symptoms of cough. There was no recent Questions Total 0
• Past Papers history of chest trauma. He smokes 20–25 cigarettes per
Questions Percentage 0%
• Random Questions day and two cans of beer every day. His brother recently
died of stomach cancer and he was quite concerned about More
• My Performance
his problem. Clinically, he had a large pleural effusion on
• Media Bank his left side, which was confirmed on X-ray. On aspiration,
• New Multimedia a white fluid came out and there was no clear supernatant
after centrifugation. Triglycerides were detected. Reference: Normal Values
Online Extras
What is the probable cause of the effusion? Haematology
Library
Community Immunoglobulins
Lymphoma
Help Tuberculosis Biochemistry
PasTest Store Metastatic stomach carcinoma Diabetes
Lymphangioleomyomatosis Endocrinology
My Account Yellow-nail syndrome
Blood gases
Profile
447 CSF
Newsletters
My Career and Exams
Order History
Learning Goals
Question Filters

Security

Change Password
Sign Out

© 2011 PasTest Ltd | About Us | Contact Us | Help

file:///E|/Shakhawan/Respiratory%20S/58.htm[3/13/2012 3:58:41 PM]


MyPasTest

Main Navigation
Question Browser: MRCP 1
Home
Subscriptions Question Browser Exam Builder Saved Exams
» MRCP 1 Session Progress
• Question Browser
Questions Correct 0
• Timed Test A 48-year-old mechanic presented to the clinic complaining
of increasing shortness of breath over the last 3 months, Questions Incorrect 1
• Mock Exam
but with no symptoms of cough. There was no recent Questions Total 1
• Past Papers history of chest trauma. He smokes 20–25 cigarettes per
Questions Percentage 0%
• Random Questions day and two cans of beer every day. His brother recently
died of stomach cancer and he was quite concerned about More
• My Performance
his problem. Clinically, he had a large pleural effusion on
• Media Bank his left side, which was confirmed on X-ray. On aspiration,
• New Multimedia a white fluid came out and there was no clear supernatant
after centrifugation. Triglycerides were detected.
Online Extras
Library What is the probable cause of the effusion?

Community Lymphoma Correct answer


Help Tuberculosis
PasTest Store Metastatic stomach carcinoma
Lymphangioleomyomatosis Your answer
My Account Yellow-nail syndrome
Profile
Newsletters
My Career and Exams
This effusion is chylous fluid, classically described as a Reference: Normal Values
Order History
milky, opalescent fluid that contains fat globules. It is
Learning Goals differentiated from empyema because the latter yields a Haematology
Question Filters clear supernatant on centrifugation. Pseudochyle is a
fluid that has a high lipid content, as cholesterol, and is Immunoglobulins
typically seen in patients with tuberculosis. The
commonest cause of a chylothorax is trauma or a Biochemistry
Security
malignancy. The malignancy is usually a lymphoma. Diabetes
Change Password Although metastatic lung deposits may cause
chylothorax, it is not so common. Other causes of Endocrinology
Sign Out
chylothorax are a congenital absence of thoracic ducts,
filariasis, tuberculous mediastinal lymphadenitis, Blood gases
lymphangioleiomyomatosis, yellow-nail syndrome and
CSF
left subclavian venous thrombosis.

447

© 2011 PasTest Ltd | About Us | Contact Us | Help

file:///E|/Shakhawan/Respiratory%20S/58a.htm[3/13/2012 3:58:43 PM]


MyPasTest

Main Navigation
Question Browser: MRCP 1
Home
Subscriptions Question Browser Exam Builder Saved Exams
» MRCP 1 Session Progress
• Question Browser
Questions Correct 0
• Timed Test A 42-year-old salesman was admitted with a diagnosis of
pneumonia. He is allergic to erythromycin. X-ray shows a Questions Incorrect 1
• Mock Exam
hazy opacity in the right lower and mid-zone. Blood Questions Total 1
• Past Papers investigations show hyponatraemia and slight rise in the
Questions Percentage 0%
• Random Questions level of liver aminotransferase. After starting the medication
on the fifth day he became acutely jaundiced and his liver More
• My Performance
aminotransferase level became very high. He also
• Media Bank complained of discoloration of his urine, which did not show
• New Multimedia haematuria on dipstick testing. Which of the following
drugs probably caused the jaundice? Reference: Normal Values
Online Extras
Ciprofloxacin Haematology
Library
Community Clarithromycin Immunoglobulins
Help Amoxicillin Biochemistry
PasTest Store Rifampicin
Diabetes
Flucloxacillin
Endocrinology
My Account
448 Blood gases
Profile
CSF
Newsletters
My Career and Exams
Order History
Learning Goals
Question Filters

Security

Change Password
Sign Out

© 2011 PasTest Ltd | About Us | Contact Us | Help

file:///E|/Shakhawan/Respiratory%20S/59.htm[3/13/2012 3:58:44 PM]


MyPasTest

Main Navigation
Question Browser: MRCP 1
Home
Subscriptions Question Browser Exam Builder Saved Exams
» MRCP 1 Session Progress
• Question Browser
Questions Correct 1
• Timed Test A 42-year-old salesman was admitted with a diagnosis of
pneumonia. He is allergic to erythromycin. X-ray shows a Questions Incorrect 1
• Mock Exam
hazy opacity in the right lower and mid-zone. Blood Questions Total 2
• Past Papers investigations show hyponatraemia and slight rise in the
Questions Percentage 50 %
• Random Questions level of liver aminotransferase. After starting the medication
on the fifth day he became acutely jaundiced and his liver More
• My Performance
aminotransferase level became very high. He also
• Media Bank complained of discoloration of his urine, which did not show
• New Multimedia haematuria on dipstick testing. Which of the following
drugs probably caused the jaundice?
Online Extras
Library Ciprofloxacin
Community Clarithromycin
Help Amoxicillin
PasTest Store Rifampicin Your answer
Flucloxacillin
My Account

Profile
Newsletters
This patient has legionella pneumonia and therefore
My Career and Exams rifampicin was prescribed. However, rifampicin, as it is
Order History a hepatic enzyme inducer, can lead to acute jaundice Reference: Normal Values
with a rise in the liver aminotransferase level. It also
Learning Goals causes red or orange discoloration of the urine and Haematology
Question Filters other body fluids. Patient should be warned about this
side-effect. Rifampicin is generally not used as a Immunoglobulins
monotherapy to treat legionella pneumonia. As this
Biochemistry
Security patient is allergic to the macrolides, ciprofloxacin is the
other drug of choice. Monotherapy with rifampicin is Diabetes
Change Password associated with the development of resistance.
Sign Out Endocrinology

Blood gases
448
CSF

© 2011 PasTest Ltd | About Us | Contact Us | Help

file:///E|/Shakhawan/Respiratory%20S/59a.htm[3/13/2012 3:58:46 PM]


MyPasTest

Main Navigation
Question Browser: MRCP 1
Home
Subscriptions Question Browser Exam Builder Saved Exams
» MRCP 1 Session Progress
• Question Browser
Questions Correct 1
• Timed Test A breathless 70-year-old smoker presents with the
following lung function tests: Questions Incorrect 1
• Mock Exam
Questions Total 2
• Past Papers
Forced expiratory volume in 1 second Questions Percentage 50 %
• Random Questions (FEV 1 ) 1.5 l (60%)
• My Performance More
Forced vital capacity (FVC) 1.8 l (55%)
• Media Bank
FEV1 /FVC ratio 84%
• New Multimedia
66% Reference: Normal Values
Online Extras Total lung capacity (TLC) predicted
Haematology
Library 57%
Community Residual volume (RV) predicted Immunoglobulins
Help 55% Biochemistry
PasTest Store Carbon monoxide transfer factor (Tlco) predicted
Diabetes
60%
Transfer coefficient (Kco) predicted Endocrinology
My Account
Blood gases
What is the most likely diagnosis in this case?
Profile
CSF
Newsletters Emphysema
My Career and Exams Anaemia
Order History Fibrosing alveolitis
Learning Goals Asthma
Question Filters Obesity

449
Security

Change Password
Sign Out

© 2011 PasTest Ltd | About Us | Contact Us | Help

file:///E|/Shakhawan/Respiratory%20S/60.htm[3/13/2012 3:58:47 PM]


MyPasTest

Main Navigation
Question Browser: MRCP 1
Home
Subscriptions Question Browser Exam Builder Saved Exams
» MRCP 1 Session Progress
• Question Browser
Questions Correct 1
• Timed Test A breathless 70-year-old smoker presents with the
following lung function tests: Questions Incorrect 2
• Mock Exam
Questions Total 3
• Past Papers
Forced expiratory volume in 1 second Questions Percentage 33 %
• Random Questions (FEV 1 ) 1.5 l (60%)
• My Performance More
Forced vital capacity (FVC) 1.8 l (55%)
• Media Bank
FEV1 /FVC ratio 84%
• New Multimedia
66%
Online Extras Total lung capacity (TLC) predicted
Library 57%
Community Residual volume (RV) predicted
Help 55%
PasTest Store Carbon monoxide transfer factor (Tlco) predicted
60%
Transfer coefficient (Kco) predicted
My Account
What is the most likely diagnosis in this case?
Profile
Newsletters Emphysema Your answer
My Career and Exams Anaemia
Order History Fibrosing alveolitis Correct answer Reference: Normal Values
Learning Goals Asthma Haematology
Question Filters Obesity Immunoglobulins
Biochemistry
Security
Diabetes
Change Password Restrictive lung function tests
Sign Out Endocrinology
The lung function tests show a significant restrictive
defect. Only fibrosing alveolitis or obesity can fit this Blood gases
picture. But given the decrease in Kco (ie after
correcting for alveolar volumes), the most likely answer CSF
is fibrosis because obesity alone would not affect the
Kco. In the case of obesity, the gas exchange after
correcting for the alveolar volume would in fact be high.

449

© 2011 PasTest Ltd | About Us | Contact Us | Help

file:///E|/Shakhawan/Respiratory%20S/60a.htm[3/13/2012 3:58:48 PM]


MyPasTest

Main Navigation
Question Browser: MRCP 1
Home
Subscriptions Question Browser Exam Builder Saved Exams
» MRCP 1 Session Progress
• Question Browser
Questions Correct 1
• Timed Test A 50-year-old smoker was diagnosed with a non-small-cell
Questions Incorrect 2
• Mock Exam carcinoma. Investigation revealed the presence of a 4 cm ×
3 cm × 2 cm tumour on the left side in the lower lobe of his Questions Total 3
• Past Papers lung that has invaded the visceral pleura. The ipsilateral Questions Percentage 33 %
• Random Questions hilar lymph node is also involved, but there is no metastatic
involvement of any distal organ. What is the stage of More
• My Performance
disease in this patient?
• Media Bank
• New Multimedia T 4 N0 M 0
Reference: Normal Values
Online Extras T 2 N0 M 0
Haematology
Library T 2 N1 M 0
Community Immunoglobulins
T 3 N1 M 0
Help Biochemistry
T 4 N1 M 0
PasTest Store Diabetes
450
Endocrinology
My Account
Blood gases
Profile
CSF
Newsletters
My Career and Exams
Order History
Learning Goals
Question Filters

Security

Change Password
Sign Out

© 2011 PasTest Ltd | About Us | Contact Us | Help

file:///E|/Shakhawan/Respiratory%20S/61.htm[3/13/2012 3:58:50 PM]


MyPasTest

Main Navigation
Question Browser: MRCP 1
Home
Subscriptions Question Browser Exam Builder Saved Exams
» MRCP 1 Session Progress
• Question Browser
Questions Correct 1
• Timed Test A 50-year-old smoker was diagnosed with a non-small-cell
carcinoma. Investigation revealed the presence of a 4 cm × Questions Incorrect 3
• Mock Exam
3 cm × 2 cm tumour on the left side in the lower lobe of his Questions Total 4
• Past Papers lung that has invaded the visceral pleura. The ipsilateral
Questions Percentage 25 %
• Random Questions hilar lymph node is also involved, but there is no metastatic
involvement of any distal organ. What is the stage of More
• My Performance
disease in this patient?
• Media Bank
• New Multimedia T 4 N0 M 0
Online Extras T 2 N0 M 0
Library T 2 N1 M 0 Correct answer
Community T 3 N1 M 0
Help
T 4 N1 M 0 Your answer
PasTest Store

My Account

Profile It is important to remember the criteria for staging


Newsletters carcinoma of the lung. As the tumour is more than 3 cm
in the greatest dimension and has only invaded the
My Career and Exams visceral pleura, it is designated T 2 . Metastasis to the
Order History ipsilateral peribronchial and/or ipsilateral hilar lymph Reference: Normal Values
nodes makes the nodal stage N1 . Absence of distal
Learning Goals Haematology
metastasis is called M 0 . It would be useful to go
Question Filters
through the staging in detail for other possible Immunoglobulins
combinations.
Biochemistry
Security
Diabetes
Change Password 450
Sign Out Endocrinology

Blood gases

CSF

© 2011 PasTest Ltd | About Us | Contact Us | Help

file:///E|/Shakhawan/Respiratory%20S/61a.htm[3/13/2012 3:58:51 PM]


MyPasTest

Main Navigation
Question Browser: MRCP 1
Home
Subscriptions Question Browser Exam Builder Saved Exams
» MRCP 1 Session Progress
• Question Browser
Questions Correct 1
• Timed Test A 46-year-male accompanied by his partner came to the
clinic. She complained that he has become lethargic, Questions Incorrect 3
• Mock Exam
increasingly sleepy in the daytime, has a headache in the Questions Total 4
• Past Papers morning and some degree of impairment of intellectual
Questions Percentage 25 %
• Random Questions ability. He has a BMI of 34, smokes 20 cigarettes per day
and about 30 units of alcohol per week. No significant past More
• My Performance
history and drug history is present. Other than a slightly
• Media Bank lower air entry in both lungs, there was no other significant
• New Multimedia finding on examination. Chest X-ray showed emphysema.
Arterial blood gas results were: pH 7.41, p a (O2 ) 9.8, Reference: Normal Values
Online Extras
p a (CO 2 ) 5.8. Which investigation would you next
Haematology
Library perform to obtain the correct diagnosis?
Community Immunoglobulins
Lung function test
Help Biochemistry
Thyroid function test
PasTest Store Diabetes
Polysomnographic studies
Echocardiography Endocrinology
My Account
CT scan brain Blood gases
Profile
452 CSF
Newsletters
My Career and Exams
Order History
Learning Goals
Question Filters

Security

Change Password
Sign Out

© 2011 PasTest Ltd | About Us | Contact Us | Help

file:///E|/Shakhawan/Respiratory%20S/62.htm[3/13/2012 3:58:54 PM]


MyPasTest

Main Navigation
Question Browser: MRCP 1
Home
Subscriptions Question Browser Exam Builder Saved Exams
» MRCP 1 Session Progress
• Question Browser
Questions Correct 1
• Timed Test A 46-year-male accompanied by his partner came to the
clinic. She complained that he has become lethargic, Questions Incorrect 4
• Mock Exam
increasingly sleepy in the daytime, has a headache in the Questions Total 5
• Past Papers morning and some degree of impairment of intellectual
Questions Percentage 20 %
• Random Questions ability. He has a BMI of 34, smokes 20 cigarettes per day
and about 30 units of alcohol per week. No significant past More
• My Performance
history and drug history is present. Other than a slightly
• Media Bank lower air entry in both lungs, there was no other significant
• New Multimedia finding on examination. Chest X-ray showed emphysema.
Arterial blood gas results were: pH 7.41, p a (O2 ) 9.8,
Online Extras
p a (CO 2 ) 5.8. Which investigation would you next
Library perform to obtain the correct diagnosis?
Community
Lung function test Your answer
Help
Thyroid function test
PasTest Store
Polysomnographic studies Correct answer
Echocardiography
My Account
CT scan brain
Profile
Newsletters
My Career and Exams
Order History This patient has obstructive sleep apnoea syndrome, a Reference: Normal Values
Learning Goals condition that most often occurs in overweight middle-
aged man and affects 1–2% of the population. Haematology
Question Filters Respiratory muscle activity is reduced during sleep, Immunoglobulins
especially during REM sleep when the diaphragm is the
only active muscle. Apnoea occurs when the sucked- Biochemistry
Security back tongue closes the airway at the back of the throat.
When awake, this tendency is overcome by the action Diabetes
Change Password
of the opening muscles of the higher airway; however,
Sign Out Endocrinology
these become hypotonic during sleep. Partial narrowing
causes snoring, while critical narrowing leads to sleep Blood gases
apnoea. This results in hypoxia and an increased
respiratory-effort stimulus to overcome the obstruction. CSF
The combination of the hypoxia and increased effort
awakens the patient, but these awakenings are so brief
that the patient remains unaware of them. Ultimately,
the condition results in sleep deprivation, especially
REM sleep, with consequent daytime sleepiness and
impaired intellectual performance. Important
contributory factors are obesity, a small pharyngeal
opening and coexisting COPD. In many cases, the
diagnosis can be made by simple finger oximetry and
from an associated good history of the sleep–snore
cycle. However, full polysomnographic studies are the
most conclusive. The diagnosis is confirmed if there are
more than 15 apnoea or hypopnoea episodes in any one
hour of sleep. Management consists of correcting
treatable factors with nasal, continuous, positive airway
pressure (CPAP) delivered by a mask during sleep.

452

file:///E|/Shakhawan/Respiratory%20S/62a.htm[3/13/2012 3:58:55 PM]


MyPasTest

Main Navigation
Question Browser: MRCP 1
Home
Subscriptions Question Browser Exam Builder Saved Exams
» MRCP 1 Session Progress
• Question Browser
Questions Correct 1
• Timed Test In which of the following emergency medical
presentations is non-invasive ventilation an Questions Incorrect 4
• Mock Exam
established treatment? Questions Total 5
• Past Papers
Questions Percentage 20 %
• Random Questions Tension pneumothorax
• My Performance Acute asthma More
• Media Bank Acute exacerbation of COPD with Type 2 respiratory
• New Multimedia failure
Reference: Normal Values
Online Extras ARDS
Pulmonary oedema with hypertension Haematology
Library
Community Immunoglobulins
453
Help Biochemistry
PasTest Store Diabetes
Endocrinology
My Account
Blood gases
Profile
CSF
Newsletters
My Career and Exams
Order History
Learning Goals
Question Filters

Security

Change Password
Sign Out

© 2011 PasTest Ltd | About Us | Contact Us | Help

file:///E|/Shakhawan/Respiratory%20S/63.htm[3/13/2012 3:58:57 PM]


MyPasTest

Main Navigation
Question Browser: MRCP 1
Home
Subscriptions Question Browser Exam Builder Saved Exams
» MRCP 1 Session Progress
• Question Browser
Questions Correct 2
• Timed Test In which of the following emergency medical
presentations is non-invasive ventilation an Questions Incorrect 4
• Mock Exam
established treatment? Questions Total 6
• Past Papers
Questions Percentage 33 %
• Random Questions Tension pneumothorax
• My Performance Acute asthma More
• Media Bank Acute exacerbation of COPD with Type Your answer
• New Multimedia 2 respiratory failure

Online Extras ARDS

Library Pulmonary oedema with hypertension

Community
Help
PasTest Store
Non-invasive ventilation (NIV) is currently being
evaluated in an number of emergency situations. The
My Account best evidence relates to exacerbations of chronic
obstructive pulmonary disease (COPD). In particular,
Profile this type of therapy is effective in patients with
decompensated type-2 respiratory failure. Physiological
Newsletters
responses (heart and respiratory rate, and arterial blood
My Career and Exams gases) improve more quickly with NIV in these patients
Order History compared to standard treatment. Intubation is also less Reference: Normal Values
frequently required. NIV is part of a range of therapies
Learning Goals for pulmonary oedema, but more usually where Haematology
Question Filters pulmonary oedema co-exists with hypertension,
measures to control BP are normally attempted first. Immunoglobulins
Biochemistry
Security
453 Diabetes
Change Password
Sign Out Endocrinology

Blood gases

CSF

© 2011 PasTest Ltd | About Us | Contact Us | Help

file:///E|/Shakhawan/Respiratory%20S/63a.htm[3/13/2012 3:58:58 PM]


MyPasTest

Main Navigation
Question Browser: MRCP 1
Home
Subscriptions Question Browser Exam Builder Saved Exams
» MRCP 1 Session Progress
• Question Browser
Questions Correct 2
• Timed Test A 49-year-old homosexual accountant came to the clinic
with increased breathlessness. He had began to become Questions Incorrect 4
• Mock Exam
wheezy after a tooth extraction procedure 5 months ago Questions Total 6
• Past Papers and also had an associated troublesome cough. He used to
Questions Percentage 33 %
• Random Questions smoke 15 cigarettes per day but gave up smoking about 2
months ago. Salbutamol and beclometasone inhalers poorly More
• My Performance
controlled his symptoms. Recently he had been unwell: he
• Media Bank had had a fever and had lost about 3.2 kg (7 lb.) in weight.
• New Multimedia There was no history of recent foreign travel and no
significant past illness. On examination, he had a Reference: Normal Values
Online Extras temperature of 37.2 °C and occasional rhonchi on both
Haematology
Library sides. Tests showed: Hb 14.6 g/dl, WBC 10.2 × 10 9 /l
Community (neutrophils 53%, lymphocytes 30%, raised eosinophils Immunoglobulins
noted) , ESR 110 mm in first hour; normal U&E; normal
Help urine dipstick. Chest X-ray showed extensive symmetrical Biochemistry
PasTest Store homogenous shadowing affecting all the peripheral lung
Diabetes
field. A skin test for inhaled antigens, including Aspergillus
fumigatus , was negative. His serum IgE was normal. A Endocrinology
My Account serological screen for parasitic infection was negative.
Pulmonary function was within normal limits. Oxygen Blood gases
Profile saturations are 97% and there is no desaturation on
exercise. CSF
Newsletters
My Career and Exams What is the probable diagnosis?
Order History
Pneumocystis pneumonia
Learning Goals
Loffler’s syndrome
Question Filters
Asthma
Cryptogenic pulmonary eosinophilia
Security
Wegener's granulomatosis
Change Password
454
Sign Out

© 2011 PasTest Ltd | About Us | Contact Us | Help

file:///E|/Shakhawan/Respiratory%20S/64.htm[3/13/2012 3:59:00 PM]


MyPasTest

Main Navigation
Question Browser: MRCP 1
Home
Subscriptions Question Browser Exam Builder Saved Exams
» MRCP 1 Session Progress
• Question Browser
Questions Correct 3
• Timed Test A 49-year-old homosexual accountant came to the clinic
with increased breathlessness. He had began to become Questions Incorrect 4
• Mock Exam
wheezy after a tooth extraction procedure 5 months ago Questions Total 7
• Past Papers and also had an associated troublesome cough. He used to
Questions Percentage 42 %
• Random Questions smoke 15 cigarettes per day but gave up smoking about 2
months ago. Salbutamol and beclometasone inhalers poorly More
• My Performance
controlled his symptoms. Recently he had been unwell: he
• Media Bank had had a fever and had lost about 3.2 kg (7 lb.) in weight.
• New Multimedia There was no history of recent foreign travel and no
significant past illness. On examination, he had a
Online Extras temperature of 37.2 °C and occasional rhonchi on both
Library sides. Tests showed: Hb 14.6 g/dl, WBC 10.2 × 10 9 /l
Community (neutrophils 53%, lymphocytes 30%, raised eosinophils
noted) , ESR 110 mm in first hour; normal U&E; normal
Help urine dipstick. Chest X-ray showed extensive symmetrical
PasTest Store homogenous shadowing affecting all the peripheral lung
field. A skin test for inhaled antigens, including Aspergillus
fumigatus , was negative. His serum IgE was normal. A
My Account serological screen for parasitic infection was negative.
Pulmonary function was within normal limits. Oxygen
Profile saturations are 97% and there is no desaturation on
Newsletters exercise.

My Career and Exams What is the probable diagnosis?


Order History Reference: Normal Values
Pneumocystis pneumonia
Learning Goals Haematology
Loffler’s syndrome
Question Filters
Asthma Immunoglobulins
Cryptogenic pulmonary eosinophilia Your answer Biochemistry
Security
Wegener's granulomatosis Diabetes
Change Password
Sign Out Endocrinology

Blood gases
This patient has eosinophilia and associated definite CSF
pulmonary signs. There is no indication of drug
involvement or malignancy. Nothing was mentioned
about atopy or extrapulmonary involvement in the
scenario. The IgE level is normal unlike fungal and
parasitic hypersensitivity. Patients may have systemic
features of malaise, weight loss, fever and a raised
ESR, and about half of them have features of asthma.
When the disease is self-limiting and lasts less than a
month it is Loeffler’s syndrome. The disease responds
to steroid treatment, which needs to be continued for
about one year. There is no relation to the patient’s
sexual orientation and Pneumocystis jiroveci secondary
to HIV is unlikely.

454

© 2011 PasTest Ltd | About Us | Contact Us | Help

file:///E|/Shakhawan/Respiratory%20S/64a.htm[3/13/2012 3:59:01 PM]


MyPasTest

Main Navigation
Question Browser: MRCP 1
Home
Subscriptions Question Browser Exam Builder Saved Exams
» MRCP 1 Session Progress
• Question Browser
Questions Correct 3
• Timed Test A 60-year-old hairdresser complains that, after an attack of
flu last year, she has been more breathless than usual Questions Incorrect 4
• Mock Exam
when taking her evening walk, and has also felt short of Questions Total 7
• Past Papers breath when climbing the stairs. She has become
Questions Percentage 42 %
• Random Questions concerned that she has a cardiac problem.
She has lost about 6.4 kg (14 lbs) during the last year. She More
• My Performance
has smoked 20 cigarettes per day for 45 years but does not
• Media Bank drink alcohol. Other than an occasional paracetamol for her
• New Multimedia headache, she is on no regular medication.
On examination she was apyrexic and had bilateral Reference: Normal Values
Online Extras clubbing. No lymphadenopathy was seen. Her JVP was not
raised and heart sounds were normal. Bibasal inspiratory Haematology
Library
crepts were audible. No pedal oedema was seen. Bilateral Immunoglobulins
Community
reticular shadowing, mostly on the bases, was seen on
Help chest X-ray. Biochemistry
PasTest Store Routine bloods were normal, except for an ESR of 35 mm in
the first hour. Her HRCT showed probable fibrosis, most Diabetes
marked in the basal region.
Endocrinology
My Account Respiratory function test showed restrictive ventilatory
defect. Bronchoalveolar lavage showed an increased Blood gases
Profile number of cells – neutrophils and macrophages – but no
malignant cells. An open-lung biopsy showed an exudate of CSF
Newsletters
intra-alveolar macrophages with patchy interstitial fibrosis.
My Career and Exams
What is the likely diagnosis?
Order History
Learning Goals Idiopathic pulmonary fibrosis
Question Filters Extrinsic alveolitis
Lymphangitis carcinomatosis
Security Chronic left heart failure

Change Password Sarcoidosis

Sign Out 455

© 2011 PasTest Ltd | About Us | Contact Us | Help

file:///E|/Shakhawan/Respiratory%20S/65.htm[3/13/2012 3:59:02 PM]


MyPasTest

Main Navigation
Question Browser: MRCP 1
Home
Subscriptions Question Browser Exam Builder Saved Exams
» MRCP 1 Session Progress
• Question Browser
Questions Correct 4
• Timed Test A 60-year-old hairdresser complains that, after an attack of
Questions Incorrect 4
• Mock Exam flu last year, she has been more breathless than usual
when taking her evening walk, and has also felt short of Questions Total 8
• Past Papers breath when climbing the stairs. She has become Questions Percentage 50 %
• Random Questions concerned that she has a cardiac problem.
She has lost about 6.4 kg (14 lbs) during the last year. She More
• My Performance
has smoked 20 cigarettes per day for 45 years but does not
• Media Bank drink alcohol. Other than an occasional paracetamol for her
• New Multimedia headache, she is on no regular medication.
On examination she was apyrexic and had bilateral
Online Extras clubbing. No lymphadenopathy was seen. Her JVP was not
Library raised and heart sounds were normal. Bibasal inspiratory
crepts were audible. No pedal oedema was seen. Bilateral
Community
reticular shadowing, mostly on the bases, was seen on
Help chest X-ray.
PasTest Store Routine bloods were normal, except for an ESR of 35 mm in
the first hour. Her HRCT showed probable fibrosis, most
marked in the basal region.
My Account Respiratory function test showed restrictive ventilatory
defect. Bronchoalveolar lavage showed an increased
Profile number of cells – neutrophils and macrophages – but no
malignant cells. An open-lung biopsy showed an exudate of
Newsletters
intra-alveolar macrophages with patchy interstitial fibrosis.
My Career and Exams
What is the likely diagnosis?
Order History Reference: Normal Values
Learning Goals Idiopathic pulmonary fibrosis Your answer Haematology
Question Filters Extrinsic alveolitis
Immunoglobulins
Lymphangitis carcinomatosis
Biochemistry
Security Chronic left heart failure
Sarcoidosis Diabetes
Change Password
Sign Out Endocrinology

Blood gases

CSF
Idiopathic pulmonary fibrosis (IPF), synonymous with
cryptogenic fibrosing alveolitis (CFA), is a progressive
and usually fatal disease of unknown cause
characterised by sequential acute lung injury with
subsequent scarring and end-stage lung disease. It may
occur in any decade of life but is most commonly seen
between the ages of 50 and 60 years; it is slightly
more frequent in males than females. A history of
progressive breathlessness on exertion in the absence
of wheeze is typical. A dry cough may be present, but
sputum production unusual until the later stages of the
disease. Haemoptysis is uncommon, but should suggest
the development of lung malignancy that occurs with a
7- to 14-fold relative risk in patients with cryptogenic
fibrosing alveolitis. Chest pain is uncommon.
Constitutional symptoms such as weight loss and
lethargy are recognised. Lung function tests show a
restrictive picture. Recent re-classification of the group
of idiopathic interstitial pneumonias have allowed
characterisation of seven different histological patterns
based on lung biopsy analysis; the pattern in IPF is that
of usual interstitial pneumonia (UIP). Patients with
desquamative histology have a favourable response to
treatment with prednisolone. All other causes should be
excluded before diagnosing cryptogenic fibrosing

file:///E|/Shakhawan/Respiratory%20S/65a.htm[3/13/2012 3:59:04 PM]


MyPasTest

alveolitis. Median survival time is approximately 5


years. Azathioprine or cyclophosphamide may be added
to the treatment and a single lung transplant may be
offered.

455

© 2011 PasTest Ltd | About Us | Contact Us | Help

file:///E|/Shakhawan/Respiratory%20S/65a.htm[3/13/2012 3:59:04 PM]


MyPasTest

Main Navigation
Question Browser: MRCP 1
Home
Subscriptions Question Browser Exam Builder Saved Exams
» MRCP 1 Session Progress
• Question Browser
Questions Correct 4
• Timed Test Which of the following statements applies to patients
with cystic fibrosis (CF)? Questions Incorrect 4
• Mock Exam
Questions Total 8
• Past Papers
Neonatal screening for CF results in a better survival Questions Percentage 50 %
• Random Questions rate and reduced decline in lung function
• My Performance Allergic bronchopulmonary aspergillosis is a More
• Media Bank recognised complication, found in 15% of adult CF
patients
• New Multimedia
The controlled intake of high-calorie food is the first Reference: Normal Values
Online Extras line of management for patients with CF-related
diabetes Haematology
Library
Community Reduced body mass index is an absolute Immunoglobulins
contraindication for heart–lung transplantation
Help Biochemistry
A correlation exists between genotype and the
PasTest Store severity of the disease Diabetes

456 Endocrinology
My Account
Blood gases
Profile
CSF
Newsletters
My Career and Exams
Order History
Learning Goals
Question Filters

Security

Change Password
Sign Out

© 2011 PasTest Ltd | About Us | Contact Us | Help

file:///E|/Shakhawan/Respiratory%20S/66.htm[3/13/2012 3:59:05 PM]


MyPasTest

Main Navigation
Question Browser: MRCP 1
Home
Subscriptions Question Browser Exam Builder Saved Exams
» MRCP 1 Session Progress
• Question Browser
Questions Correct 4
• Timed Test Which of the following statements applies to patients
with cystic fibrosis (CF)? Questions Incorrect 5
• Mock Exam
Questions Total 9
• Past Papers
Neonatal screening for CF results in Your answer Questions Percentage 44 %
• Random Questions a better survival rate and reduced
• My Performance decline in lung function More
• Media Bank Allergic bronchopulmonary Correct answer
aspergillosis is a recognised
• New Multimedia complication, found in 15% of adult
Online Extras CF patients
Library The controlled intake of high-calorie
food is the first line of management
Community for patients with CF-related diabetes
Help Reduced body mass index is an
PasTest Store absolute contraindication for heart–
lung transplantation
A correlation exists between
My Account genotype and the severity of the
Profile disease

Newsletters
My Career and Exams
Order History Reference: Normal Values
The commonest mutation in patients with cystic fibrosis
Learning Goals (CF) is the Delta-F508 mutation. There is no correlation Haematology
Question Filters between genotype, the range of manifestations, the age
of symptoms and the survival rate in CF. Immunoglobulins
Neonatal screening allows the early implementation of
Biochemistry
Security prophylactic therapeutic measures. Prospective and
retrospective studies have demonstrated that neonates Diabetes
Change Password detected by screening have a better nutritional status
Sign Out than those who were discovered by symptoms. The age Endocrinology
of acquisition of Pseudomonas aeruginosa and the
decline in lung function were found to be comparable in Blood gases
the two groups. The increased age of survival for CF CSF
patients is associated with increased complications,
such as diabetes mellitus, acquisition of multiple
bacteria in their respiratory flora, osteoporosis and liver
disease. Diabetes is now seen in almost 10% of patients
over the age of 20 years, the main manifestations being
weight loss, repeated respiratory infections and decline
in lung function. The treatment of choice is
subcutaneous insulin therapy. Calorie intake should not
be restricted in CF patients who are prone to
malnutrition due to their pancreatic insufficiency. The
increasing incidence of allergic bronchopulmonary
aspergillosis (ABPA) is a recognised phenomenon. ABPA
is encountered in 1 in 6 adult CF patients.
Manifestations of ABPA include asthma symptoms,
flitting opacities on the chest radiograph, increased
eosinophil count, and hyperreactivity to the skin-prick
test and increased specific plasma IgE. The main
treatment is high-dose corticosteroids initially, with a
smaller maintenance dose. The duration of treatment
ranges from few months to few years. Anti-fungal
agents are of no proven benefit. Heart–lung
transplantation is offered to patients who exhibit a rapid
decline in lung function despite optimal treatment, and
to patients with respiratory failure. Moderately reduced

file:///E|/Shakhawan/Respiratory%20S/66a.htm[3/13/2012 3:59:07 PM]


MyPasTest

body mass index (BMI) is an indication for


transplantation. A severe reduction in BMI is a relative
contraindication to transplantation, this is because it is
associated with a reduced survival but not an absolute
one.

456

© 2011 PasTest Ltd | About Us | Contact Us | Help

file:///E|/Shakhawan/Respiratory%20S/66a.htm[3/13/2012 3:59:07 PM]


MyPasTest

Main Navigation
Question Browser: MRCP 1
Home
Subscriptions Question Browser Exam Builder Saved Exams
» MRCP 1 Session Progress
• Question Browser
Questions Correct 4
• Timed Test A 45-year-old man, presenting with weight loss and cough,
is found to have small cell lung cancer. Questions Incorrect 5
• Mock Exam
Questions Total 9
• Past Papers Which of the following best describes the additional
biocemical and clinical features that may occur in this Questions Percentage 44 %
• Random Questions
condition? More
• My Performance
• Media Bank Cushing's syndrome, if present, is characterised by
• New Multimedia buffalo hump, striae and central obesity
Reference: Normal Values
Online Extras Hypercalcaemia is commonly seen
Hypertrophic pulmonary osteoarthropathy is a very Haematology
Library
rare feature
Community Immunoglobulins
Acidois is commonly seen
Help Biochemistry
Syndrome of inappropriate antidiuretic hormone
PasTest Store secretion (SIADH) occurs in >60% of cases Diabetes

457 Endocrinology
My Account
Blood gases
Profile
CSF
Newsletters
My Career and Exams
Order History
Learning Goals
Question Filters

Security

Change Password
Sign Out

© 2011 PasTest Ltd | About Us | Contact Us | Help

file:///E|/Shakhawan/Respiratory%20S/67.htm[3/13/2012 3:59:08 PM]


MyPasTest

Main Navigation
Question Browser: MRCP 1
Home
Subscriptions Question Browser Exam Builder Saved Exams
» MRCP 1 Session Progress
• Question Browser
Questions Correct 4
• Timed Test A 45-year-old man, presenting with weight loss and cough,
is found to have small cell lung cancer. Questions Incorrect 6
• Mock Exam
Questions Total 10
• Past Papers Which of the following best describes the additional
biocemical and clinical features that may occur in this Questions Percentage 40 %
• Random Questions
condition? More
• My Performance
• Media Bank Cushing's syndrome, if present, is Your answer
• New Multimedia characterised by buffalo hump, striae
and central obesity
Online Extras
Hypercalcaemia is commonly seen
Library
Hypertrophic pulmonary Correct answer
Community osteoarthropathy is a very rare
Help feature
PasTest Store Acidois is commonly seen
Syndrome of inappropriate
antidiuretic hormone secretion
My Account (SIADH) occurs in >60% of cases
Profile
Newsletters
My Career and Exams
Small-cell carcinoma is uncommonly associated with Reference: Normal Values
Order History
hypercalcaemia and the syndrome of inappropriate
Learning Goals antidiuretic hormone secretion (SIADH) (5-10% of Haematology
Question Filters cases). Due to the short natural history, Cushing’s
syndrome in small-cell carcinoma does not manifest Immunoglobulins
classically by buffalo hump, striae or central obesity. Its
presence is suspected by arterial hypertension, Biochemistry
Security
hyperglycaemia, hypokalaemia, alkalosis and muscle Diabetes
Change Password weakness. Chemotherapy is the treatment of choice.
Hypertrophic pulmonary osteoarthropathy is very rare in Endocrinology
Sign Out
small-cell carcinoma of the lung.
Blood gases

CSF
457

© 2011 PasTest Ltd | About Us | Contact Us | Help

file:///E|/Shakhawan/Respiratory%20S/67a.htm[3/13/2012 3:59:10 PM]


MyPasTest

Main Navigation
Question Browser: MRCP 1
Home
Subscriptions Question Browser Exam Builder Saved Exams
» MRCP 1 Session Progress
• Question Browser
Questions Correct 4
• Timed Test A 26-year-old office secretary who smokes 10–15
cigarettes per day presented in the clinic after a couple of Questions Incorrect 6
• Mock Exam
episodes of haemoptysis. She also said that she had felt Questions Total 10
• Past Papers tired recently and gave a history of treatment for an upper
Questions Percentage 40 %
• Random Questions respiratory tract infection a couple of months ago.
According to her she never fully recovered from the episode More
• My Performance
and had been persistently coughing ever since. On
• Media Bank examination she looked pale, had minimal pedal oedema
• New Multimedia and diffused crepitations on chest auscultation. Her urine
was positive for protein and blood. FBC showed anaemia, Reference: Normal Values
Online Extras ANCA proved positive and the chest X-ray showed blotchy
shadows over the lung field. Haematology
Library
Community Immunoglobulins
What is your probable diagnosis?
Help Biochemistry
Pulmonary tuberculosis
PasTest Store Diabetes
Bronchogenic carcinoma
Sarcoidosis Endocrinology
My Account
Goodpasture’s syndrome Blood gases
Profile
Wegener’s granulomatosis CSF
Newsletters
My Career and Exams 458
Order History
Learning Goals
Question Filters

Security

Change Password
Sign Out

© 2011 PasTest Ltd | About Us | Contact Us | Help

file:///E|/Shakhawan/Respiratory%20S/68.htm[3/13/2012 3:59:11 PM]


MyPasTest

Main Navigation
Question Browser: MRCP 1
Home
Subscriptions Question Browser Exam Builder Saved Exams
» MRCP 1 Session Progress
• Question Browser
Questions Correct 5
• Timed Test A 26-year-old office secretary who smokes 10–15
cigarettes per day presented in the clinic after a couple of Questions Incorrect 6
• Mock Exam
episodes of haemoptysis. She also said that she had felt Questions Total 11
• Past Papers tired recently and gave a history of treatment for an upper
Questions Percentage 45 %
• Random Questions respiratory tract infection a couple of months ago.
According to her she never fully recovered from the episode More
• My Performance
and had been persistently coughing ever since. On
• Media Bank examination she looked pale, had minimal pedal oedema
• New Multimedia and diffused crepitations on chest auscultation. Her urine
was positive for protein and blood. FBC showed anaemia,
Online Extras ANCA proved positive and the chest X-ray showed blotchy
Library shadows over the lung field.
Community What is your probable diagnosis?
Help
Pulmonary tuberculosis
PasTest Store
Bronchogenic carcinoma
Sarcoidosis
My Account
Goodpasture’s syndrome Your answer
Profile
Wegener’s granulomatosis
Newsletters
My Career and Exams
Order History Reference: Normal Values
Learning Goals This is a case of Goodpasture’s syndrome, which usually Haematology
Question Filters occurs in people over the age of 16 years. It starts with
an upper respiratory infection followed by cough, Immunoglobulins
intermittent haemoptysis and tiredness. Later, anaemia
develops and a massive episode of haemoptysis may Biochemistry
Security
occur. The typical chest X-ray picture is due to Diabetes
Change Password intrapulmonary haemorrhage. These features are
followed, in weeks or months, by the development of Endocrinology
Sign Out
glomerulonephritis. The basic cause of the disease is a
type-II cytotoxic reaction against the basement Blood gases
membrane of both the kidneys and lungs. Antineutrophil
CSF
cytoplasmic antibody (ANCA) may be positive.
Glomerulonephritis may present as asymptomatic
proteinuria and/or microscopic haematuria. This is later
followed by development of the acute nephritic
syndrome, nephrotic syndrome and chronic renal
failure. Wegener’s may present with predominant
respiratory symptoms, often with symptoms of
granuloma formation, e.g. nosebleeds for a number of
months before diagnosis. Alternatively patients may
present with renal disease, those presenting with
respiratory symptoms are said to have around an 80%
chance of eventual renal dysfunction.

458

© 2011 PasTest Ltd | About Us | Contact Us | Help

file:///E|/Shakhawan/Respiratory%20S/68a.htm[3/13/2012 3:59:12 PM]


MyPasTest

Main Navigation
Question Browser: MRCP 1
Home
Subscriptions Question Browser Exam Builder Saved Exams
» MRCP 1 Session Progress
• Question Browser
Questions Correct 5
• Timed Test An anxious 22-year-old woman presented with mild
shortness of breath on exertion that had come on gradually Questions Incorrect 6
• Mock Exam
over months. Her symptoms are intermittent, but worse in Questions Total 11
• Past Papers the evening, and her speech becomes slurred during the
Questions Percentage 45 %
• Random Questions episodes. She has recently started on treatment for
anxiety. On examination she looked depressed but there More
• My Performance
were no other positive clinical findings. Other than an ESR
• Media Bank of 26, her routine blood results were normal. Chest X-ray,
• New Multimedia lung function tests and ECG were all normal. What is the
most likely diagnosis? Reference: Normal Values
Online Extras
Unstable angina Haematology
Library
Community Eaton–Lambert syndrome Immunoglobulins
Help Myasthenia gravis Biochemistry
PasTest Store Somatisation disorder
Diabetes
TIA
Endocrinology
My Account
459 Blood gases
Profile
CSF
Newsletters
My Career and Exams
Order History
Learning Goals
Question Filters

Security

Change Password
Sign Out

© 2011 PasTest Ltd | About Us | Contact Us | Help

file:///E|/Shakhawan/Respiratory%20S/69.htm[3/13/2012 3:59:14 PM]


MyPasTest

Main Navigation
Question Browser: MRCP 1
Home
Subscriptions Question Browser Exam Builder Saved Exams
» MRCP 1 Session Progress
• Question Browser
Questions Correct 6
• Timed Test An anxious 22-year-old woman presented with mild
shortness of breath on exertion that had come on gradually Questions Incorrect 6
• Mock Exam
over months. Her symptoms are intermittent, but worse in Questions Total 12
• Past Papers the evening, and her speech becomes slurred during the
Questions Percentage 50 %
• Random Questions episodes. She has recently started on treatment for
anxiety. On examination she looked depressed but there More
• My Performance
were no other positive clinical findings. Other than an ESR
• Media Bank of 26, her routine blood results were normal. Chest X-ray,
• New Multimedia lung function tests and ECG were all normal. What is the
most likely diagnosis?
Online Extras
Library Unstable angina
Community Eaton–Lambert syndrome
Help Myasthenia gravis Your answer
PasTest Store Somatisation disorder
TIA
My Account

Profile
Newsletters
This is a case of myasthenia gravis. On examination,
My Career and Exams the muscle weakness is not apparent unless it is
Order History examined repeatedly. Most often, the affected muscles Reference: Normal Values
are the ocular and shoulder-girdle muscles. The muscles
Learning Goals of respiration can be involved early and breathlessness Haematology
Question Filters may also develop early. This may result in sudden
death. Bulbar involvement causes problems in Immunoglobulins
swallowing, slurred speech and difficulty in chewing.
Biochemistry
Security Mild, facial muscle weakness gives the patient a
depressed look. Thymic involvement is seen in about Diabetes
Change Password 15% of cases. The disease is autoimmune in origin.
Sign Out Endocrinology

Blood gases
459
CSF

© 2011 PasTest Ltd | About Us | Contact Us | Help

file:///E|/Shakhawan/Respiratory%20S/69a.htm[3/13/2012 3:59:16 PM]


MyPasTest

Main Navigation
Question Browser: MRCP 1
Home
Subscriptions Question Browser Exam Builder Saved Exams
» MRCP 1 Session Progress
• Question Browser
Questions Correct 6
• Timed Test A 69-year-old woman with rheumatoid arthritis is referred
with a history of recurrent chest infections, intermittent Questions Incorrect 6
• Mock Exam
wheeze and production of half an eggcup-full of phlegm Questions Total 12
• Past Papers daily, on occasions with a streak of blood. She is a retired
Questions Percentage 50 %
• Random Questions secretary and has never smoked. On examination she had
coarse crepitations on both the bases. What is the most More
• My Performance
likely diagnosis?
• Media Bank
• New Multimedia Chronic bronchitis
Reference: Normal Values
Online Extras Diffuse interstitial lung fibrosis
Carcinoma of the lung Haematology
Library
Community Tuberculosis Immunoglobulins
Help Bronchiectasis Biochemistry
PasTest Store 460 Diabetes
Endocrinology
My Account
Blood gases
Profile
CSF
Newsletters
My Career and Exams
Order History
Learning Goals
Question Filters

Security

Change Password
Sign Out

© 2011 PasTest Ltd | About Us | Contact Us | Help

file:///E|/Shakhawan/Respiratory%20S/70.htm[3/13/2012 3:59:17 PM]


MyPasTest

Main Navigation
Question Browser: MRCP 1
Home
Subscriptions Question Browser Exam Builder Saved Exams
» MRCP 1 Session Progress
• Question Browser
Questions Correct 7
• Timed Test A 69-year-old woman with rheumatoid arthritis is referred
with a history of recurrent chest infections, intermittent Questions Incorrect 6
• Mock Exam
wheeze and production of half an eggcup-full of phlegm Questions Total 13
• Past Papers daily, on occasions with a streak of blood. She is a retired
Questions Percentage 53 %
• Random Questions secretary and has never smoked. On examination she had
coarse crepitations on both the bases. What is the most More
• My Performance
likely diagnosis?
• Media Bank
• New Multimedia Chronic bronchitis
Online Extras Diffuse interstitial lung fibrosis
Library Carcinoma of the lung
Community Tuberculosis
Help Bronchiectasis Your answer
PasTest Store

My Account
Bronchiectasis is associated with rheumatoid arthritis,
Profile occurring in 3–4% of patients with this condition. As
with all other causes of bronchiectasis, it presents with
Newsletters
recurrent chest infections and excessive phlegm.
My Career and Exams Recurrent haemoptysis is a common feature. Chronic
Order History obstructive pulmonary disease (COPD) and lung cancer Reference: Normal Values
are unlikely in a non-smoker. Tuberculosis should be
Learning Goals excluded as it can cause bronchiectasis and Haematology
Question Filters haemoptysis. Although interstitial fibrosis occurs in up
to 20% of patients with rheumatoid arthritis, it is not Immunoglobulins
associated with sputum production or haemoptysis
Biochemistry
Security (unless there is an associated cancer).
This woman should undergo pulmonary function tests to Diabetes
Change Password assess the presence of airway obstruction, which is
Sign Out associated with bronchiectasis. A high-resolution Endocrinology
computed tomography (HRCT) scan of the lungs is the
investigation of choice to confirm the presence of Blood gases
bronchiectasis. CSF

460

© 2011 PasTest Ltd | About Us | Contact Us | Help

file:///E|/Shakhawan/Respiratory%20S/70a.htm[3/13/2012 3:59:18 PM]


MyPasTest

Main Navigation
Question Browser: MRCP 1
Home
Subscriptions Question Browser Exam Builder Saved Exams
» MRCP 1 Session Progress
• Question Browser
Questions Correct 7
• Timed Test A 27-year-old woman presented in A&E with a 2-week
history of haemoptysis and a 3-day history of red-coloured Questions Incorrect 6
• Mock Exam
urine. She had a similar problem 2 years ago but recovered Questions Total 13
• Past Papers fully. She looked pale and was tachycardic, her pulse was
Questions Percentage 53 %
• Random Questions 128/min and regular and her BP was 150/96 mmHg.
Bilateral crackles were present on auscultation. Laboratory More
• My Performance
testing gave the following results:
• Media Bank
• New Multimedia Hb, 7.5 g/dl; WBC, 13 × 10 6 /l; Platelets, 480 × 10 6 /l;
Reference: Normal Values
Online Extras Clotting, normal; Na, 140 mmol/l; K, 6.7 mmol/l; Urea,
46 mmol/l; Creatinine, 650 µmol/l. Haematology
Library
Community Urine showed blood and protein. The chest X-ray showed Immunoglobulins
patchy interstitial shadowing in both lung fields.
Help Biochemistry
PasTest Store What is the probable diagnosis? Diabetes
Goodpasture’s syndrome Endocrinology
My Account
Microscopic polyangiitis Blood gases
Profile Wegener’s granulomatosis
CSF
Newsletters PAN
My Career and Exams Tuberculosis
Order History
461
Learning Goals
Question Filters

Security

Change Password
Sign Out

© 2011 PasTest Ltd | About Us | Contact Us | Help

file:///E|/Shakhawan/Respiratory%20S/71.htm[3/13/2012 3:59:20 PM]


MyPasTest

Main Navigation
Question Browser: MRCP 1
Home
Subscriptions Question Browser Exam Builder Saved Exams
» MRCP 1 Session Progress
• Question Browser
Questions Correct 7
• Timed Test A 27-year-old woman presented in A&E with a 2-week
history of haemoptysis and a 3-day history of red-coloured Questions Incorrect 7
• Mock Exam
urine. She had a similar problem 2 years ago but recovered Questions Total 14
• Past Papers fully. She looked pale and was tachycardic, her pulse was
Questions Percentage 50 %
• Random Questions 128/min and regular and her BP was 150/96 mmHg.
Bilateral crackles were present on auscultation. Laboratory More
• My Performance
testing gave the following results:
• Media Bank
• New Multimedia Hb, 7.5 g/dl; WBC, 13 × 10 6 /l; Platelets, 480 × 10 6 /l;
Online Extras Clotting, normal; Na, 140 mmol/l; K, 6.7 mmol/l; Urea,
46 mmol/l; Creatinine, 650 µmol/l.
Library
Community Urine showed blood and protein. The chest X-ray showed
patchy interstitial shadowing in both lung fields.
Help
PasTest Store What is the probable diagnosis?

Goodpasture’s syndrome
My Account
Microscopic polyangiitis Correct answer
Profile Wegener’s granulomatosis Your answer
Newsletters PAN
My Career and Exams Tuberculosis
Order History Reference: Normal Values
Learning Goals Haematology
Question Filters
Immunoglobulins
The patient is suffering a pulmonary haemorrhage, as
shown on the chest X-ray. Associated renal involvement Biochemistry
Security is present. The changes can be caused by microscopic
polyangiitis, Weg ener’s or Goodpasture’s syndrome. Diabetes
Change Password
The larger vessel vasculitides do not present in this
Sign Out Endocrinology
way. Microscopic polyangiitistends to recur more often
than the other two, making this the probable diagnosis. Blood gases

CSF
461

© 2011 PasTest Ltd | About Us | Contact Us | Help

file:///E|/Shakhawan/Respiratory%20S/71a.htm[3/13/2012 3:59:21 PM]


MyPasTest

Main Navigation
Question Browser: MRCP 1
Home
Subscriptions Question Browser Exam Builder Saved Exams
» MRCP 1 Session Progress
• Question Browser
Questions Correct 7
• Timed Test A 56-year-old retired man who smoked was diagnosed with
smear-positive mycobacterium tuberculosis. He used to Questions Incorrect 7
• Mock Exam
work as a stonemason. Questions Total 14
• Past Papers
If there is an increased occupational risk of getting Questions Percentage 50 %
• Random Questions
mycobacterium tuberculosis infection, which one of More
• My Performance the following is the most likely cause?
• Media Bank
• New Multimedia Pneumoconiosis
Reference: Normal Values
Online Extras Asbestosis
Silicosis Haematology
Library
Community Lead poisoning Immunoglobulins
Help Berylliosis
Biochemistry
PasTest Store 462 Diabetes
Endocrinology
My Account
Blood gases
Profile
CSF
Newsletters
My Career and Exams
Order History
Learning Goals
Question Filters

Security

Change Password
Sign Out

© 2011 PasTest Ltd | About Us | Contact Us | Help

file:///E|/Shakhawan/Respiratory%20S/72.htm[3/13/2012 3:59:23 PM]


MyPasTest

Main Navigation
Question Browser: MRCP 1
Home
Subscriptions Question Browser Exam Builder Saved Exams
» MRCP 1 Session Progress
• Question Browser
Questions Correct 8
• Timed Test A 56-year-old retired man who smoked was diagnosed with
smear-positive mycobacterium tuberculosis. He used to Questions Incorrect 7
• Mock Exam
work as a stonemason. Questions Total 15
• Past Papers
If there is an increased occupational risk of getting Questions Percentage 53 %
• Random Questions
mycobacterium tuberculosis infection, which one of More
• My Performance the following is the most likely cause?
• Media Bank
• New Multimedia Pneumoconiosis

Online Extras Asbestosis

Library Silicosis Your answer

Community Lead poisoning

Help Berylliosis

PasTest Store

My Account
Stonemasons, fettlers and slate-miners/-workers are at
Profile risk of developing silicosis. Silica acts as a toxin to
macrophages, and so impairing their function. This
Newsletters results in an increased risk of contracting a
My Career and Exams mycobacterium tuberculosis infection .
Order History Reference: Normal Values
Learning Goals 462 Haematology
Question Filters
Immunoglobulins
Biochemistry
Security
Diabetes
Change Password
Sign Out Endocrinology

Blood gases

CSF

© 2011 PasTest Ltd | About Us | Contact Us | Help

file:///E|/Shakhawan/Respiratory%20S/72a.htm[3/13/2012 3:59:24 PM]


MyPasTest

Main Navigation
Question Browser: MRCP 1
Home
Subscriptions Question Browser Exam Builder Saved Exams
» MRCP 1 Session Progress
• Question Browser
Questions Correct 0
• Timed Test In which of the following respiratory disease is
clubbing not a feature? Questions Incorrect 0
• Mock Exam
Questions Total 0
• Past Papers
Asbestosis Questions Percentage 0%
• Random Questions
Fibrosing alveolitis
• My Performance More
Lung abscess
• Media Bank
Chronic bronchitis
• New Multimedia
Bronchial carcinoma Reference: Normal Values
Online Extras
463 Haematology
Library
Community Immunoglobulins
Help Biochemistry
PasTest Store Diabetes
Endocrinology
My Account
Blood gases
Profile
CSF
Newsletters
My Career and Exams
Order History
Learning Goals
Question Filters

Security

Change Password
Sign Out

© 2011 PasTest Ltd | About Us | Contact Us | Help

file:///E|/Shakhawan/Respiratory%20S/73.htm[3/13/2012 3:59:26 PM]


MyPasTest

Main Navigation
Question Browser: MRCP 1
Home
Subscriptions Question Browser Exam Builder Saved Exams
» MRCP 1 Session Progress
• Question Browser
Questions Correct 0
• Timed Test In which of the following respiratory disease is
clubbing not a feature? Questions Incorrect 1
• Mock Exam
Questions Total 1
• Past Papers
Asbestosis Your answer Questions Percentage 0%
• Random Questions
Fibrosing alveolitis
• My Performance More
Lung abscess
• Media Bank
Chronic bronchitis Correct answer
• New Multimedia
Bronchial carcinoma
Online Extras
Library
Community
Help Loss of the natural angle between the nail and the
PasTest Store nailbed is seen in clubbing. An increased curvature of
the nail develops later. Clubbing is seen in suppurative
diseases such as long-standing bronchiectasis, acute
My Account lung abscesses and empyema. However, clubbing is not
seen in uncomplicated bronchitis. Clubbing is also found
Profile in fibrosing alveolitis and asbestosis, in malignant
disease, particularly carcinoma of the bronchus and in
Newsletters
pleural malignancy. Finger clubbing may also be
My Career and Exams associated with hypertrophic pulmonary
Order History osteoarthropathy, a painful osteitis of the distal ends of Reference: Normal Values
the long bones of the lower arms and legs. Malignancy
Learning Goals is associated in 95% of these cases. Haematology
Question Filters
Immunoglobulins
463 Biochemistry
Security
Diabetes
Change Password
Sign Out Endocrinology

Blood gases

CSF

© 2011 PasTest Ltd | About Us | Contact Us | Help

file:///E|/Shakhawan/Respiratory%20S/73a.htm[3/13/2012 3:59:27 PM]


MyPasTest

Main Navigation
Question Browser: MRCP 1
Home
Subscriptions Question Browser Exam Builder Saved Exams
» MRCP 1 Session Progress
• Question Browser
Questions Correct 0
• Timed Test A 22-year-old man presented with shortness of breath, an
increase in sputum production and a temperature of 37.9 Questions Incorrect 0
• Mock Exam
°C. He suffers from recurrent chest infections three or four Questions Total 0
• Past Papers times a year. This young man regularly produces copious
Questions Percentage 0%
• Random Questions amounts of sputum, more so in the morning. A slow,
progressive clumsiness and instability set in during early More
• My Performance
childhood and he has been wheelchair-bound since the age
• Media Bank of 12 years. He grew up in foster care but found out that
• New Multimedia his uncle died young, wheelchair-bound. There are coarse
crackles bilaterally and a chest X-ray shows signs of Reference: Normal Values
Online Extras bronchiectasis. What is the cause of this?
Haematology
Library
Cystic fibrosis Immunoglobulins
Community
Hypogammaglobulinaemia in ataxia telangiectasia
Help Biochemistry
Hereditary spinocerebellar degeneration
PasTest Store Diabetes
Williams–Campbell syndrome
Mounier–Kuhn syndrome Endocrinology
My Account
Blood gases
Profile 464
CSF
Newsletters
My Career and Exams
Order History
Learning Goals
Question Filters

Security

Change Password
Sign Out

© 2011 PasTest Ltd | About Us | Contact Us | Help

file:///E|/Shakhawan/Respiratory%20S/74.htm[3/13/2012 3:59:29 PM]


MyPasTest

Main Navigation
Question Browser: MRCP 1
Home
Subscriptions Question Browser Exam Builder Saved Exams
» MRCP 1 Session Progress
• Question Browser
Questions Correct 0
• Timed Test A 22-year-old man presented with shortness of breath, an
increase in sputum production and a temperature of 37.9 Questions Incorrect 1
• Mock Exam
°C. He suffers from recurrent chest infections three or four Questions Total 1
• Past Papers times a year. This young man regularly produces copious
Questions Percentage 0%
• Random Questions amounts of sputum, more so in the morning. A slow,
progressive clumsiness and instability set in during early More
• My Performance
childhood and he has been wheelchair-bound since the age
• Media Bank of 12 years. He grew up in foster care but found out that
• New Multimedia his uncle died young, wheelchair-bound. There are coarse
crackles bilaterally and a chest X-ray shows signs of
Online Extras bronchiectasis. What is the cause of this?
Library
Cystic fibrosis Your answer
Community
Hypogammaglobulinaemia in ataxia Correct answer
Help
telangiectasia
PasTest Store
Hereditary spinocerebellar
degeneration
My Account Williams–Campbell syndrome

Profile Mounier–Kuhn syndrome

Newsletters
My Career and Exams
Order History Reference: Normal Values
Patients are typically progressively ataxic from an early
Learning Goals age, with oculomotor apraxia and chorioathetosis Haematology
Question Filters developing later. It is inherited as an autosomal-
recessive. The clinical diagnosis is based on Immunoglobulins
identification of the symptoms and multiple
Biochemistry
Security telangiectasia seen particularly in the conjunctiva. a-
Fetoprotein levels may be raised. Cell defects and Diabetes
Change Password humoral immunity with IgA and IgE deficiencies and
Sign Out defective DNA repair results in recurrent infections and Endocrinology
tumour susceptibility. Death in the late teens or
twenties from bronchiectasis is typical. Blood gases

CSF
In the Williams–Campbell syndrome there is a deficiency
of the bronchial cartilage. The Mounier–Kuhn syndrome
or tracheobronchomegaly is the ‘adult equivalent’ of the
congenital deficiency of bronchial cartilage.

464

© 2011 PasTest Ltd | About Us | Contact Us | Help

file:///E|/Shakhawan/Respiratory%20S/74a.htm[3/13/2012 3:59:30 PM]


MyPasTest

Main Navigation
Question Browser: MRCP 1
Home
Subscriptions Question Browser Exam Builder Saved Exams
» MRCP 1 Session Progress
• Question Browser
Questions Correct 0
• Timed Test A 46-year-old cardiologist attended a local conference last
weekend and fell ill with a fever of up to 40 °C that lasted Questions Incorrect 0
• Mock Exam
for 2 days. He had associated shortness of breath and dry Questions Total 0
• Past Papers cough. In addition, he had loose motions for a day. His
Questions Percentage 0%
• Random Questions blood results showed deranged LFTs and hyponatraemia.
• My Performance His WBC count was 10.2 × 10 9 /l. Bibasal consolidation was More
seen on his X-ray.
• Media Bank
• New Multimedia What would be the most effective treatment for his
condition? Reference: Normal Values
Online Extras
Haematology
Library Amoxicillin
Community Cefuroxime Immunoglobulins
Help Clarithromycin Biochemistry
PasTest Store Flucloxacillin Diabetes
Ciprofloxacin
Endocrinology
My Account
465 Blood gases
Profile
CSF
Newsletters
My Career and Exams
Order History
Learning Goals
Question Filters

Security

Change Password
Sign Out

© 2011 PasTest Ltd | About Us | Contact Us | Help

file:///E|/Shakhawan/Respiratory%20S/75.htm[3/13/2012 3:59:31 PM]


MyPasTest

Main Navigation
Question Browser: MRCP 1
Home
Subscriptions Question Browser Exam Builder Saved Exams
» MRCP 1 Session Progress
• Question Browser
Questions Correct 1
• Timed Test A 46-year-old cardiologist attended a local conference last
weekend and fell ill with a fever of up to 40 °C that lasted Questions Incorrect 0
• Mock Exam
for 2 days. He had associated shortness of breath and dry Questions Total 1
• Past Papers cough. In addition, he had loose motions for a day. His
Questions Percentage 100 %
• Random Questions blood results showed deranged LFTs and hyponatraemia.
• My Performance His WBC count was 10.2 × 10 9 /l. Bibasal consolidation was More
seen on his X-ray.
• Media Bank
• New Multimedia What would be the most effective treatment for his
condition?
Online Extras
Library Amoxicillin
Community Cefuroxime
Help Clarithromycin Your answer
PasTest Store Flucloxacillin
Ciprofloxacin
My Account

Profile
Newsletters
This is a case of legionella pneumonia and the preferred
My Career and Exams treatment is with the macrolide, clarithromycin.
Order History Ciprofloxacin can also be effective and rifampicin can be Reference: Normal Values
used. Legionella outbreaks are seen in previously fit
Learning Goals Haematology
individuals staying in hotels or institutions where the
Question Filters shower facilities and/or the cooling system are
Immunoglobulins
contaminated with the organism. The incubation period
is 2–10 days. Flu-like symptoms, fever, malaise and Biochemistry
Security myalgia typically precede a dry cough and dyspnoea.
Extrapulmonary symptoms include anorexia, diarrhoea Diabetes
Change Password and vomiting, hepatitis, renal failure, confusion and
Sign Out coma. A chest X-ray shows bibasal consolidation, Endocrinology
sometimes with a small pleural effusion. Blood results Blood gases
show an elevated white count, hyponatraemia and
deranged LFTs. Urinalysis may show haematuria. CSF
Diagnosis is by the urinary antigen test, which is highly
specific. It is important to remember that the organism
does not show up on Gram-staining.

465

© 2011 PasTest Ltd | About Us | Contact Us | Help

file:///E|/Shakhawan/Respiratory%20S/75a.htm[3/13/2012 3:59:33 PM]


MyPasTest

Main Navigation
Question Browser: MRCP 1
Home
Subscriptions Question Browser Exam Builder Saved Exams
» MRCP 1 Session Progress
• Question Browser
Questions Correct 1
• Timed Test A 29-year-old office secretary has been suffering from
intermittent pain and tenderness affecting her elbows, wrist Questions Incorrect 0
• Mock Exam
and ankles for last 2 years. Symptomatic relief had being Questions Total 1
• Past Papers obtained from NSAIDs. For last 3 months she has been
Questions Percentage 100 %
• Random Questions increasingly unwell, and with night sweats, fever and a
weight loss of about 6.4 kg (14 lbs). She also developed a More
• My Performance
non-productive cough and left-sided pleuritic chest pain.
• Media Bank She smoked 30 cigarettes per day. On examination she had
• New Multimedia temperature of 37.8 °C. The syno vium was palpable over
her wrist joints. On respiratory examination her left lower Reference: Normal Values
Online Extras zone was dull to percussion with decreased breath sounds.
A chest X-ray confirmed a left-sided pleural effusion in Haematology
Library
addition to some fibrotic patches on both the upper zones. Immunoglobulins
Community
Aspiration showed a straw-coloured fluid with a protein
Help concentration of 46 g/l and a glucose concentration of 1.6 Biochemistry
PasTest Store mmol/l. The fluid contained many lymphocytes but no
malignant cells. A culture was sterile on the fifth day. Diabetes

What is the probable cause of the pleural effusion? Endocrinology


My Account
Blood gases
Profile Sarcoidosis
Rheumatoid pleural effusion CSF
Newsletters
My Career and Exams Tuberculosis
Order History Pleural effusion secondary to lung malignancy
Learning Goals Lymphoma
Question Filters
466

Security

Change Password
Sign Out

© 2011 PasTest Ltd | About Us | Contact Us | Help

file:///E|/Shakhawan/Respiratory%20S/76.htm[3/13/2012 3:59:34 PM]


MyPasTest

Main Navigation
Question Browser: MRCP 1
Home
Subscriptions Question Browser Exam Builder Saved Exams
» MRCP 1 Session Progress
• Question Browser
Questions Correct 1
• Timed Test A 29-year-old office secretary has been suffering from
intermittent pain and tenderness affecting her elbows, wrist Questions Incorrect 1
• Mock Exam
and ankles for last 2 years. Symptomatic relief had being Questions Total 2
• Past Papers obtained from NSAIDs. For last 3 months she has been
Questions Percentage 50 %
• Random Questions increasingly unwell, and with night sweats, fever and a
weight loss of about 6.4 kg (14 lbs). She also developed a More
• My Performance
non-productive cough and left-sided pleuritic chest pain.
• Media Bank She smoked 30 cigarettes per day. On examination she had
• New Multimedia temperature of 37.8 °C. The syno vium was palpable over
her wrist joints. On respiratory examination her left lower
Online Extras zone was dull to percussion with decreased breath sounds.
Library A chest X-ray confirmed a left-sided pleural effusion in
addition to some fibrotic patches on both the upper zones.
Community
Aspiration showed a straw-coloured fluid with a protein
Help concentration of 46 g/l and a glucose concentration of 1.6
PasTest Store mmol/l. The fluid contained many lymphocytes but no
malignant cells. A culture was sterile on the fifth day.

What is the probable cause of the pleural effusion?


My Account

Profile Sarcoidosis
Newsletters Rheumatoid pleural effusion Your answer
My Career and Exams Tuberculosis Correct answer
Order History Pleural effusion secondary to lung Reference: Normal Values
malignancy
Learning Goals Haematology
Lymphoma
Question Filters
Immunoglobulins
Biochemistry
Security
Diabetes
Change Password Two main points for the diagnosis are the unilateral
pleural effusion and upper zone shadowing. Rheumatoid Endocrinology
Sign Out
effusions are unusual in the young and are small and
asymptomatic. This woman may have rheumatoid Blood gases
disease but this is not the cause of her effusion. It is
CSF
now well recognised that tuberculosis may be
associated with an inflammatory polyarthritis which may
follow a similar pattern to rheumatoid arthritis seen
here. Lymphocyte predominance suggests lymphoma,
carcinoma or tuberculosis. A low glucose concentration
favours an infection, malignancy or a rheumatoid
condition. But the fibrotic shadow of the upper zone
clinches the diagnosis in this case and is typical of post-
primary tuberculosis. Sarcoidosis shows upper zone
fibrosis with pleural effusion at end-stage only, and it is
rare. No other feature favours the diagnosis.

466

© 2011 PasTest Ltd | About Us | Contact Us | Help

file:///E|/Shakhawan/Respiratory%20S/76a.htm[3/13/2012 3:59:36 PM]


MyPasTest

Main Navigation
Question Browser: MRCP 1
Home
Subscriptions Question Browser Exam Builder Saved Exams
» MRCP 1 Session Progress
• Question Browser
Questions Correct 1
• Timed Test A 65-year-old man known to have COPD presented with
progressive respiratory failure. He was treated in ITU with Questions Incorrect 1
• Mock Exam
mechanical ventilation and improved. After extubation he Questions Total 2
• Past Papers was transferred to the ward. On the second day on the
Questions Percentage 50 %
• Random Questions ward, his temperature spiked and he developed a
productive cough with a yellow-greenish sputum. Blood More
• My Performance
results showed leucocytosis. A chest X-ray revealed a
• Media Bank right-sided middle and lower lobe pneumonia.
• New Multimedia
What is the most probable cause of his pneumonia? Reference: Normal Values
Online Extras
Pneumococcal pneumonia Haematology
Library
Community Aspiration pneumonia Immunoglobulins
Help Pseudomonas pneumonia Biochemistry
PasTest Store Staphylococcal pneumonia
Diabetes
Haemophilus pneumonia
Endocrinology
My Account
467 Blood gases
Profile
CSF
Newsletters
My Career and Exams
Order History
Learning Goals
Question Filters

Security

Change Password
Sign Out

© 2011 PasTest Ltd | About Us | Contact Us | Help

file:///E|/Shakhawan/Respiratory%20S/77.htm[3/13/2012 3:59:37 PM]


MyPasTest

Main Navigation
Question Browser: MRCP 1
Home
Subscriptions Question Browser Exam Builder Saved Exams
» MRCP 1 Session Progress
• Question Browser
Questions Correct 2
• Timed Test A 65-year-old man known to have COPD presented with
progressive respiratory failure. He was treated in ITU with Questions Incorrect 1
• Mock Exam
mechanical ventilation and improved. After extubation he Questions Total 3
• Past Papers was transferred to the ward. On the second day on the
Questions Percentage 66 %
• Random Questions ward, his temperature spiked and he developed a
productive cough with a yellow-greenish sputum. Blood More
• My Performance
results showed leucocytosis. A chest X-ray revealed a
• Media Bank right-sided middle and lower lobe pneumonia.
• New Multimedia
What is the most probable cause of his pneumonia?
Online Extras
Library Pneumococcal pneumonia
Community Aspiration pneumonia
Help Pseudomonas pneumonia Your answer
PasTest Store Staphylococcal pneumonia
Haemophilus pneumonia
My Account

Profile
Newsletters
Pseudomonas is a common pathogen in patients with
My Career and Exams bronchiectasis and cystic fibrosis. It also causes
Order History hospital-acquired infections, particularly on the ITU or Reference: Normal Values
after surgery. Nosocomial or hospital-acquired
Learning Goals infections should be suspected in patients with an onset Haematology
Question Filters of symptoms at least 48 hours after admission to the
hospital. The sputum colour also gives a clue to the Immunoglobulins
most likely diagnosis. Treatment is with anti-
Biochemistry
Security pseudomonal penicillin, ceftazidime, meropenem or
ciprofloxacin. Diabetes
Change Password
Sign Out Endocrinology
467
Blood gases

CSF

© 2011 PasTest Ltd | About Us | Contact Us | Help

file:///E|/Shakhawan/Respiratory%20S/77a.htm[3/13/2012 3:59:38 PM]


MyPasTest

Main Navigation
Question Browser: MRCP 1
Home
Subscriptions Question Browser Exam Builder Saved Exams
» MRCP 1 Session Progress
• Question Browser
Questions Correct 2
• Timed Test A 48-year-old woman is admitted with a couple of days’
history of fever with rigors and breathlessness. On Questions Incorrect 1
• Mock Exam
examination she looks extremely unwell and is confused, Questions Total 3
• Past Papers cyanosed, has a respiratory rate of 36/min and a systolic
Questions Percentage 66 %
• Random Questions blood pressure of 86 mmHg. There is dullness on
percussion and bronchial breathing at her right base. The More
• My Performance
chest radiograph reveals consolidation.
• Media Bank
Which of the following would be the most appropriate
• New Multimedia
antibiotic regimen to use? Reference: Normal Values
Online Extras
Oral amoxicillin Haematology
Library
Community Oral amoxicillin and oral clarithromycin Immunoglobulins
Help Intravenous cefotaxime and IV clarithromycin Biochemistry
PasTest Store Intravenous ceftazidime and intravenous vancomycin
Diabetes
Intravenous amoxicillin and intravenous
clarithromycin Endocrinology
My Account
Blood gases
Profile 468
CSF
Newsletters
My Career and Exams
Order History
Learning Goals
Question Filters

Security

Change Password
Sign Out

© 2011 PasTest Ltd | About Us | Contact Us | Help

file:///E|/Shakhawan/Respiratory%20S/78.htm[3/13/2012 3:59:40 PM]


MyPasTest

Main Navigation
Question Browser: MRCP 1
Home
Subscriptions Question Browser Exam Builder Saved Exams
» MRCP 1 Session Progress
• Question Browser
Questions Correct 2
• Timed Test A 48-year-old woman is admitted with a couple of days’ history of fever with rigors and
breathlessness. On examination she looks extremely unwell and is confused, cyanosed, has a Questions Incorrect 2
• Mock Exam
respiratory rate of 36/min and a systolic blood pressure of 86 mmHg. There is dullness on Questions Total 4
• Past Papers percussion and bronchial breathing at her right base. The chest radiograph reveals consolidation.
Questions Percentage 50 %
• Random Questions
Which of the following would be the most appropriate antibiotic regimen to use? More
• My Performance
• Media Bank Oral amoxicillin Your answer
• New Multimedia Oral amoxicillin and oral clarithromycin
Online Extras Intravenous cefotaxime and IV clarithromycin Correct answer
Library Intravenous ceftazidime and intravenous vancomycin
Community Intravenous amoxicillin and intravenous clarithromycin
Help
PasTest Store

My Account This woman has severe pneumonia as defined by the British Thoracic Society guidelines. It
requires any two of the following features: confusion, urea > 7 mmol/l, respiratory rate >
Profile 30/min and hypotension (SBP < 90 mmHg, DBP < 60 mmHg). Appropriate treatment (as
recommended by the British Thoracic Society) is with intravenous antimicrobials: co-amoxiclav
Newsletters 1.2 g three times daily or cefuroxime 1.5 g three times daily or cefotaxime 1 g three times daily
My Career and Exams or ceftriaxone 2 g once daily plus erythromycin 500 mg four times daily or clarithromycin 500
mg twice daily. Reference: Normal Values
Order History
Learning Goals See guidelines (2001) British Thoracic Society on the management of pneumonia; also 2004 Haematology
update.
Question Filters
Immunoglobulins
http://www.brit-
thoracic.org.uk/Portals/0/Clinical%20Information/Pneumonia/Guidelines/MACAP2001gline.pdf Biochemistry
Security
http://www.brit- Diabetes
Change Password
thoracic.org.uk/Portals/0/Clinical%20Information/Pneumonia/Guidelines/MACAPrevisedApr04.pdf
Sign Out Endocrinology

Blood gases
468
CSF

© 2011 PasTest Ltd | About Us | Contact Us | Help

file:///E|/Shakhawan/Respiratory%20S/78a.htm[3/13/2012 3:59:41 PM]


MyPasTest

Main Navigation
Question Browser: MRCP 1
Home
Subscriptions Question Browser Exam Builder Saved Exams
» MRCP 1 Session Progress
• Question Browser
Questions Correct 2
• Timed Test A 46-year-old meat-factory worker is found to have Q
fever pneumonia. Questions Incorrect 2
• Mock Exam
Questions Total 4
• Past Papers Which of the following statements is correct?
Questions Percentage 50 %
• Random Questions
• My Performance He requires high-dose penicillin for his treatment More
• Media Bank His occupation is not important for the diagnosis

• New Multimedia There is no long-term sequel of the disease


Reference: Normal Values
Online Extras The organism responsible is Coxiella pneumoniae
The organism is usually inhaled from infected dust Haematology
Library
Community Immunoglobulins
469
Help Biochemistry
PasTest Store Diabetes
Endocrinology
My Account
Blood gases
Profile
CSF
Newsletters
My Career and Exams
Order History
Learning Goals
Question Filters

Security

Change Password
Sign Out

© 2011 PasTest Ltd | About Us | Contact Us | Help

file:///E|/Shakhawan/Respiratory%20S/79.htm[3/13/2012 3:59:43 PM]


MyPasTest

Main Navigation
Question Browser: MRCP 1
Home
Subscriptions Question Browser Exam Builder Saved Exams
» MRCP 1 Session Progress
• Question Browser
Questions Correct 2
• Timed Test A 46-year-old meat-factory worker is found to have Q
fever pneumonia. Questions Incorrect 3
• Mock Exam
Questions Total 5
• Past Papers Which of the following statements is correct?
Questions Percentage 40 %
• Random Questions
• My Performance He requires high-dose penicillin for More
his treatment
• Media Bank
His occupation is not important for
• New Multimedia the diagnosis
Online Extras There is no long-term sequel of the
Library disease
Community The organism responsible is Coxiella Your answer
pneumoniae
Help
The organism is usually inhaled Correct answer
PasTest Store from infected dust

My Account

Profile
Q fever is due to Coxiella burnetii and is acquired
Newsletters
through contact with animals. The organism is very
My Career and Exams resistant to drying and is inhaled from infected dust. It
Order History is not notifiable, but can occur in outbreaks in farming Reference: Normal Values
communities and in abattoirs. A chest X-ray may show
Learning Goals multi-lobar consolidation. Treatment is with prolonged Haematology
Question Filters courses of tetracyclines. Rarely, infection can be
persistent leading to chronic symptoms including Immunoglobulins
fatigue, malaise and sweats. In cases of chronic
Biochemistry
Security disease, culture-negative endocarditis should be
suspected. Lengthy antibiotic courses can be curative. Diabetes
Change Password
Sign Out Endocrinology
469
Blood gases

CSF

© 2011 PasTest Ltd | About Us | Contact Us | Help

file:///E|/Shakhawan/Respiratory%20S/79a.htm[3/13/2012 3:59:44 PM]


MyPasTest

Main Navigation
Question Browser: MRCP 1
Home
Subscriptions Question Browser Exam Builder Saved Exams
» MRCP 1 Session Progress
• Question Browser
Questions Correct 2
• Timed Test As the medical SHO on call you are summoned to A&E to
see a 25-year-old man whose condition has suddenly Questions Incorrect 3
• Mock Exam
deteriorated. He arrived about 45 minutes earlier with a 2- Questions Total 5
• Past Papers hour history of central pleuritic-type chest pain and
Questions Percentage 40 %
• Random Questions breathlessness. He collapsed while awaiting radiography.
He is now agitated and cyanosed. His pulse is 128/min and More
• My Performance
BP 76/40 mmHg. Oxygen saturation is reading 76% with
• Media Bank the patient breathing high-flow oxygen via a re-breathing
• New Multimedia mask. On respiratory examination you hear reduced breath
sounds in the right lung field with deviation of the trachea Reference: Normal Values
Online Extras towards the left. On percussion it is resonant bilaterally.
What immediate course of action should you take? Haematology
Library
Community Immunoglobulins
Contact the ITU for urgent mechanical ventilation
Help Biochemistry
Insert a large-bore needle into the left, second
PasTest Store intercostal space Diabetes
Insert a large-bore needle into the right, second
intercostal space Endocrinology
My Account
Check his arterial blood gases and start nebulisation Blood gases
Profile with salbutamol
CSF
Newsletters Ask for an urgent portable chest radiograph
My Career and Exams
470
Order History
Learning Goals
Question Filters

Security

Change Password
Sign Out

© 2011 PasTest Ltd | About Us | Contact Us | Help

file:///E|/Shakhawan/Respiratory%20S/80.htm[3/13/2012 3:59:46 PM]


MyPasTest

Main Navigation
Question Browser: MRCP 1
Home
Subscriptions Question Browser Exam Builder Saved Exams
» MRCP 1 Session Progress
• Question Browser
Questions Correct 3
• Timed Test As the medical SHO on call you are summoned to A&E to
see a 25-year-old man whose condition has suddenly Questions Incorrect 3
• Mock Exam
deteriorated. He arrived about 45 minutes earlier with a 2- Questions Total 6
• Past Papers hour history of central pleuritic-type chest pain and
Questions Percentage 50 %
• Random Questions breathlessness. He collapsed while awaiting radiography.
He is now agitated and cyanosed. His pulse is 128/min and More
• My Performance
BP 76/40 mmHg. Oxygen saturation is reading 76% with
• Media Bank the patient breathing high-flow oxygen via a re-breathing
• New Multimedia mask. On respiratory examination you hear reduced breath
sounds in the right lung field with deviation of the trachea
Online Extras towards the left. On percussion it is resonant bilaterally.
Library What immediate course of action should you take?
Community
Contact the ITU for urgent mechanical
Help ventilation
PasTest Store Insert a large-bore needle into the left,
second intercostal space
Insert a large-bore needle into the Your answer
My Account
right, second intercostal space
Profile Check his arterial blood gases and
Newsletters start nebulisation with salbutamol
My Career and Exams Ask for an urgent portable chest
radiograph
Order History Reference: Normal Values
Learning Goals Haematology
Question Filters
Immunoglobulins
The history might suggest a number of diagnoses,
Biochemistry
Security including cardiac tamponade, massive pulmonary
embolism, haemothorax or aortic dissection other than Diabetes
Change Password a pneumothorax. But the respiratory examination
Sign Out findings indicate that he almost certainly has sustained Endocrinology
a spontaneous pneumothorax that has now developed
into a tension pneumothorax. This is a Blood gases
periarrest/emergency situation. There is no time to CSF
arrange for a portable chest radiograph. Insertion of a
large-bore needle should be attempted to reduce the
pressure in the right hemithorax. If the diagnosis was
correct, it would be accompanied by a loud ‘hiss’.
Mechanical ventilation would not be required once the
lung has re-inflated.

470

© 2011 PasTest Ltd | About Us | Contact Us | Help

file:///E|/Shakhawan/Respiratory%20S/80a.htm[3/13/2012 3:59:47 PM]


MyPasTest

Main Navigation
Question Browser: MRCP 1
Home
Subscriptions Question Browser Exam Builder Saved Exams
» MRCP 1 Session Progress
• Question Browser
Questions Correct 3
• Timed Test An 82-year-old man living alone in a bungalow came to the
clinic complaining of feeling generally unwell for about the Questions Incorrect 3
• Mock Exam
last 3–4 months and of losing about 9.5 kg (21 lbs) in Questions Total 6
• Past Papers weight during this period. On further enquiry he said he
Questions Percentage 50 %
• Random Questions had been having night sweats for the last month. He also
has a past history of angina and arthritis and was on More
• My Performance
medication. On examination he did not look well. He was
• Media Bank pyrexic and without lymphadenopathy. Bibasal crepitus on
• New Multimedia the lower zone was heard on chest auscultation. He had
hepatosplenomegaly and clubbing. Investigations showed Reference: Normal Values
Online Extras
WBC 12.3 × 10 6 /l (neutrophils 52%, lymphocytes 39%), Haematology
Library Hb 9.1 g/dl, with all other routine investigations being
Community normal. A chest X-ray showed 1–2 mm diameter miliary Immunoglobulins
shadows all over the lung field. The Mantoux test was
Help negative. No bacteria grew in a sputum culture. Biochemistry
PasTest Store Diabetes
What is the probable cause of the illness and the X-
ray finding? Endocrinology
My Account
Sarcoidosis Blood gases
Profile
Mycoplasma pneumonia CSF
Newsletters
Staphylococcal pneumonia
My Career and Exams
Miliary tuberculosis
Order History
Bacterial endocarditis
Learning Goals
Question Filters 471

Security

Change Password
Sign Out

© 2011 PasTest Ltd | About Us | Contact Us | Help

file:///E|/Shakhawan/Respiratory%20S/81.htm[3/13/2012 3:59:49 PM]


MyPasTest

Main Navigation
Question Browser: MRCP 1
Home
Subscriptions Question Browser Exam Builder Saved Exams
» MRCP 1 Session Progress
• Question Browser
Questions Correct 4
• Timed Test An 82-year-old man living alone in a bungalow came to the
clinic complaining of feeling generally unwell for about the Questions Incorrect 3
• Mock Exam
last 3–4 months and of losing about 9.5 kg (21 lbs) in Questions Total 7
• Past Papers weight during this period. On further enquiry he said he
Questions Percentage 57 %
• Random Questions had been having night sweats for the last month. He also
has a past history of angina and arthritis and was on More
• My Performance
medication. On examination he did not look well. He was
• Media Bank pyrexic and without lymphadenopathy. Bibasal crepitus on
• New Multimedia the lower zone was heard on chest auscultation. He had
hepatosplenomegaly and clubbing. Investigations showed
Online Extras
WBC 12.3 × 10 6 /l (neutrophils 52%, lymphocytes 39%),
Library Hb 9.1 g/dl, with all other routine investigations being
Community normal. A chest X-ray showed 1–2 mm diameter miliary
shadows all over the lung field. The Mantoux test was
Help negative. No bacteria grew in a sputum culture.
PasTest Store
What is the probable cause of the illness and the X-
ray finding?
My Account
Sarcoidosis
Profile
Mycoplasma pneumonia
Newsletters
Staphylococcal pneumonia
My Career and Exams
Miliary tuberculosis Your answer
Order History Reference: Normal Values
Bacterial endocarditis
Learning Goals Haematology
Question Filters
Immunoglobulins
Biochemistry
Security This is a case of miliary tuberculosis, which is caused by
a diffuse disseminated spread of tubercle bacilli via the Diabetes
Change Password bloodstream. In older patients it is difficult to diagnose.
Sign Out It presents with a gradual onset of vague ill health, loss Endocrinology
of weight and then fever. It can also present as
Blood gases
tubercular meningitis. Hepatosplenomegaly is seen in
advanced disease. Choroidal tubercles are seen in the CSF
eyes. Initially, a chest X-ray may be normal. Later,
chest X-ray reveals the presence of small 1–2 mm
lesions. CT scanning may show a lung parenchymal
abnormality at an early stage. The Mantoux test may be
negative in up to half of the patients with severe
disease. A transbronchial biopsy is positive at an early
stage. Biopsy of liver and bone marrow may be
required. If untreated miliary tuberculosis is universally
fatal.

471

© 2011 PasTest Ltd | About Us | Contact Us | Help

file:///E|/Shakhawan/Respiratory%20S/81a.htm[3/13/2012 3:59:50 PM]


MyPasTest

Main Navigation
Question Browser: MRCP 1
Home
Subscriptions Question Browser Exam Builder Saved Exams
» MRCP 1 Session Progress
• Question Browser
Questions Correct 4
• Timed Test Which one of the following statement is true about
the FEF25%–75% (forced expiratory flow rate Questions Incorrect 3
• Mock Exam
between 25% and 75% of the forced vital capacity) Questions Total 7
• Past Papers
in pulmonary function tests? Questions Percentage 57 %
• Random Questions
• My Performance It reflects the status of the small airways More
• Media Bank It is not impaired in bronchiolitis obliterans
• New Multimedia It is effort-dependent
Reference: Normal Values
Online Extras It is not affected in smokers
It is useful to identify tracheal obstruction Haematology
Library
Community Immunoglobulins
472
Help Biochemistry
PasTest Store Diabetes
Endocrinology
My Account
Blood gases
Profile
CSF
Newsletters
My Career and Exams
Order History
Learning Goals
Question Filters

Security

Change Password
Sign Out

© 2011 PasTest Ltd | About Us | Contact Us | Help

file:///E|/Shakhawan/Respiratory%20S/82.htm[3/13/2012 3:59:51 PM]


MyPasTest

Main Navigation
Question Browser: MRCP 1
Home
Subscriptions Question Browser Exam Builder Saved Exams
» MRCP 1 Session Progress
• Question Browser
Questions Correct 4
• Timed Test Which one of the following statement is true about
the FEF25%–75% (forced expiratory flow rate Questions Incorrect 4
• Mock Exam
between 25% and 75% of the forced vital capacity) Questions Total 8
• Past Papers
in pulmonary function tests? Questions Percentage 50 %
• Random Questions
• My Performance It reflects the status of the small Correct answer More
airways
• Media Bank
It is not impaired in bronchiolitis
• New Multimedia obliterans
Online Extras It is effort-dependent Your answer
Library It is not affected in smokers
Community It is useful to identify tracheal
Help obstruction
PasTest Store

My Account
The FEF 25%–75% primarily reflects the status of the
Profile small airways. It is more sensitive than the FEV1 for
Newsletters identifying early airway obstruction. It is effort-
My Career and Exams INdependent. It is impaired in bronchiolitis obliterans,
smokers and rejection reactions in bone marrow, lung
Order History and heart transplants. Reference: Normal Values
Learning Goals Haematology
Question Filters
472 Immunoglobulins
Biochemistry
Security
Diabetes
Change Password
Sign Out Endocrinology

Blood gases

CSF

© 2011 PasTest Ltd | About Us | Contact Us | Help

file:///E|/Shakhawan/Respiratory%20S/82a.htm[3/13/2012 3:59:53 PM]


MyPasTest

Main Navigation
Question Browser: MRCP 1
Home
Subscriptions Question Browser Exam Builder Saved Exams
» MRCP 1 Session Progress
• Question Browser
Questions Correct 4
• Timed Test A 65-year-old man came to clinic with a history of proximal
muscle weakness. He has had a cough for 8 weeks. There Questions Incorrect 4
• Mock Exam
is associated pain in the small joints of the hands, and has Questions Total 8
• Past Papers small haemorrhages in the nail folds. He is apyrexic and on
Questions Percentage 50 %
• Random Questions examination there is no lymphadenopathy or clubbing.
Bibasal crackles can be heard and his chest X-ray reveals More
• My Performance
diffuse reticular infiltrates. Lung function tests show a
• Media Bank restrictive pattern. What is the underlying cause of his
• New Multimedia interstitial lung disease?
Reference: Normal Values
Online Extras Cryptogenic fibrosing alveolitis
Haematology
Library SLE
Community Immunoglobulins
Ankylosing spondylitis
Help Polymyositis Biochemistry
PasTest Store Rheumatoid arthritis Diabetes

473 Endocrinology
My Account
Blood gases
Profile
CSF
Newsletters
My Career and Exams
Order History
Learning Goals
Question Filters

Security

Change Password
Sign Out

© 2011 PasTest Ltd | About Us | Contact Us | Help

file:///E|/Shakhawan/Respiratory%20S/83.htm[3/13/2012 3:59:54 PM]


MyPasTest

Main Navigation
Question Browser: MRCP 1
Home
Subscriptions Question Browser Exam Builder Saved Exams
» MRCP 1 Session Progress
• Question Browser
Questions Correct 5
• Timed Test A 65-year-old man came to clinic with a history of proximal
muscle weakness. He has had a cough for 8 weeks. There Questions Incorrect 4
• Mock Exam
is associated pain in the small joints of the hands, and has Questions Total 9
• Past Papers small haemorrhages in the nail folds. He is apyrexic and on
Questions Percentage 55 %
• Random Questions examination there is no lymphadenopathy or clubbing.
Bibasal crackles can be heard and his chest X-ray reveals More
• My Performance
diffuse reticular infiltrates. Lung function tests show a
• Media Bank restrictive pattern. What is the underlying cause of his
• New Multimedia interstitial lung disease?

Online Extras Cryptogenic fibrosing alveolitis


Library SLE
Community Ankylosing spondylitis
Help Polymyositis Your answer
PasTest Store Rheumatoid arthritis

My Account

Profile
Polymyositis and dermatomyositis are inflammatory
Newsletters
conditions involving the muscle and skin, respectively.
My Career and Exams Patients often complain of proximal muscle weakness.
Order History Pain in the small joints of the fingers is associated with Reference: Normal Values
the condition. The patients may present with ragged
Learning Goals cuticles and haemorrhages at the finger-nail folds. Haematology
Question Filters Interstitial lung disease can occur in these patients.
Underlying malignancy of the lungs, ovaries, breasts or Immunoglobulins
stomach is present in 5–8% of cases.
Biochemistry
Security
Diabetes
Change Password 473
Sign Out Endocrinology

Blood gases

CSF

© 2011 PasTest Ltd | About Us | Contact Us | Help

file:///E|/Shakhawan/Respiratory%20S/83a.htm[3/13/2012 3:59:56 PM]


MyPasTest

Main Navigation
Question Browser: MRCP 1
Home
Subscriptions Question Browser Exam Builder Saved Exams
» MRCP 1 Session Progress
• Question Browser
Questions Correct 5
• Timed Test At the time of discharge of a 75-year-old non-smoker, with
Questions Incorrect 4
• Mock Exam known COPD, it was decided that, according to the criteria,
he should be having long-term oxygen therapy in home. Questions Total 9
• Past Papers
Questions Percentage 55 %
• Random Questions What is not considered as a lone criterium for LTOT
among the following options? More
• My Performance
• Media Bank Arterial blood gas showing p a (O2 ) 7.2 kPa
• New Multimedia Cor pulmonale Reference: Normal Values
Online Extras FEV1 < 1.5 litres despite maximal treatment
Haematology
Library
FVC < 2 litres despite maximal treatment
Community Immunoglobulins
Arterial blood gas showing p a (O2 ) 7.8 kPa with
Help pulmonary hypertension Biochemistry
PasTest Store Diabetes
474
Endocrinology
My Account
Blood gases
Profile
CSF
Newsletters
My Career and Exams
Order History
Learning Goals
Question Filters

Security

Change Password
Sign Out

© 2011 PasTest Ltd | About Us | Contact Us | Help

file:///E|/Shakhawan/Respiratory%20S/84.htm[3/13/2012 3:59:57 PM]


MyPasTest

Main Navigation
Question Browser: MRCP 1
Home
Subscriptions Question Browser Exam Builder Saved Exams
» MRCP 1 Session Progress
• Question Browser
Questions Correct 6
• Timed Test At the time of discharge of a 75-year-old non-smoker, with
known COPD, it was decided that, according to the criteria, Questions Incorrect 4
• Mock Exam
he should be having long-term oxygen therapy in home. Questions Total 10
• Past Papers
What is not considered as a lone criterium for LTOT Questions Percentage 60 %
• Random Questions
among the following options? More
• My Performance
• Media Bank Arterial blood gas showing p a (O2 ) 7.2
• New Multimedia kPa
Online Extras Cor pulmonale Your answer
Library FEV1 < 1.5 litres despite maximal
Community treatment

Help FVC < 2 litres despite maximal


treatment
PasTest Store
Arterial blood gas showing p a (O2 ) 7.8
kPa with pulmonary hypertension
My Account

Profile
Newsletters
My Career and Exams Long-term oxygen therapy (LTOT): The MRC trial
showed that if p a (O2 ) was maintained > 8.0 kPa for >
Order History Reference: Normal Values
15 hours a day the 3-year survival improved by 50%.
Learning Goals UK DoH guidelines suggest that LTOT should be Haematology
provided for those patients who are clinically stable and
Question Filters
non-smokers with p a (O2 ) < 7.3 kPa, FEV1 < 1.5 litres, Immunoglobulins
FVC < 2 litres despite maximal treatment. These values
Biochemistry
Security should be stable on two occasions more than 3 weeks
apart. Diabetes
Change Password
It is also necessary to provide LTOT if the patient has a Endocrinology
Sign Out
p a (O2 ) of 7.3–8.0 kPa and pulmonary hypertension
showing RVH and loud S2 and features of cor Blood gases
pulmonale. Cor pulmonale in the absence of hypoxia CSF
may have a primary cardiological underlying cause and
therefore may not require home oxygen therapy.

474

© 2011 PasTest Ltd | About Us | Contact Us | Help

file:///E|/Shakhawan/Respiratory%20S/84a.htm[3/13/2012 3:59:59 PM]


MyPasTest

Main Navigation
Question Browser: MRCP 1
Home
Subscriptions Question Browser Exam Builder Saved Exams
» MRCP 1 Session Progress
• Question Browser
Questions Correct 6
• Timed Test A 67-year-old woman has been diagnosed as suffering
from bronchiectasis on a high-resolution computed Questions Incorrect 4
• Mock Exam
tomography (HRCT) scan of the lung. Which one of the Questions Total 10
• Past Papers following statements is NOT true?
Questions Percentage 60 %
• Random Questions
• My Performance She is at risk of developing a pneumothorax More
• Media Bank She is at risk of developing a brain abscess

• New Multimedia Massive haemoptysis is the commonest cause of


death in her age group Reference: Normal Values
Online Extras
Recurrent chest infections are likely at her age Haematology
Library
Her immunoglobulin levels should be checked
Community Immunoglobulins
Help 475 Biochemistry
PasTest Store Diabetes
Endocrinology
My Account
Blood gases
Profile
CSF
Newsletters
My Career and Exams
Order History
Learning Goals
Question Filters

Security

Change Password
Sign Out

© 2011 PasTest Ltd | About Us | Contact Us | Help

file:///E|/Shakhawan/Respiratory%20S/85.htm[3/13/2012 4:00:00 PM]


MyPasTest

Main Navigation
Question Browser: MRCP 1
Home
Subscriptions Question Browser Exam Builder Saved Exams
» MRCP 1 Session Progress
• Question Browser
Questions Correct 7
• Timed Test A 67-year-old woman has been diagnosed as suffering
from bronchiectasis on a high-resolution computed Questions Incorrect 4
• Mock Exam
tomography (HRCT) scan of the lung. Which one of the Questions Total 11
• Past Papers following statements is NOT true?
Questions Percentage 63 %
• Random Questions
• My Performance She is at risk of developing a More
pneumothorax
• Media Bank
She is at risk of developing a brain
• New Multimedia abscess
Online Extras Massive haemoptysis is the commonest Your answer
Library cause of death in her age group
Community Recurrent chest infections are likely at
her age
Help
Her immunoglobulin levels should be
PasTest Store checked

My Account

Profile
Congenital and acquired immunodeficiencies are well-
Newsletters
known causes of bronchiectasis. To exclude them, the
My Career and Exams serum immunoglobulins should be checked. In
Order History particular, the IgG subclass (IgG1, -2, -3 and -4). Reference: Normal Values
Patients with an immunoglobulin deficiency should be
Learning Goals treated with regular immunoglobulin infusions. Haematology
Question Filters Pneumothorax, recurrent chest infections and brain
abscesses are well-known complications of Immunoglobulins
bronchiectasis. Haemoptysis is a common symptom but
Biochemistry
Security is rarely a cause of death.
Diabetes
Change Password
Sign Out 475 Endocrinology

Blood gases

CSF

© 2011 PasTest Ltd | About Us | Contact Us | Help

file:///E|/Shakhawan/Respiratory%20S/85a.htm[3/13/2012 4:00:01 PM]


MyPasTest

Main Navigation
Question Browser: MRCP 1
Home
Subscriptions Question Browser Exam Builder Saved Exams
» MRCP 1 Session Progress
• Question Browser
Questions Correct 7
• Timed Test A 32-year-old Black woman presents with a 3-month
history of a non-productive cough, dyspnoea and pleuritic Questions Incorrect 4
• Mock Exam
chest pain, especially with climbing stairs. She reports Questions Total 11
• Past Papers intermittent fevers of up to 39 °C and a 3.5-kg weight loss.
Questions Percentage 63 %
• Random Questions She complains of wrist and ankle pain that has interfered
with her work. She smokes two packets of cigarettes per More
• My Performance
day. Her full blood count is normal and serum ANA is
• Media Bank negative. On examination there are red nodules over her
• New Multimedia lower legs. What is the most likely diagnosis?
Reference: Normal Values
Online Extras Goodpasture's syndrome
Haematology
Library Adenocarcinoma of the lung
Community Immunoglobulins
Systemic lupus erythematosus
Help Sarcoidosis Biochemistry
PasTest Store Histoplasmosis Diabetes

476 Endocrinology
My Account
Blood gases
Profile
CSF
Newsletters
My Career and Exams
Order History
Learning Goals
Question Filters

Security

Change Password
Sign Out

© 2011 PasTest Ltd | About Us | Contact Us | Help

file:///E|/Shakhawan/Respiratory%20S/86.htm[3/13/2012 4:00:03 PM]


MyPasTest

Main Navigation
Question Browser: MRCP 1
Home
Subscriptions Question Browser Exam Builder Saved Exams
» MRCP 1 Session Progress
• Question Browser
Questions Correct 8
• Timed Test A 32-year-old Black woman presents with a 3-month
history of a non-productive cough, dyspnoea and pleuritic Questions Incorrect 4
• Mock Exam
chest pain, especially with climbing stairs. She reports Questions Total 12
• Past Papers intermittent fevers of up to 39 °C and a 3.5-kg weight loss.
Questions Percentage 66 %
• Random Questions She complains of wrist and ankle pain that has interfered
with her work. She smokes two packets of cigarettes per More
• My Performance
day. Her full blood count is normal and serum ANA is
• Media Bank negative. On examination there are red nodules over her
• New Multimedia lower legs. What is the most likely diagnosis?

Online Extras Goodpasture's syndrome


Library Adenocarcinoma of the lung
Community Systemic lupus erythematosus
Help Sarcoidosis Your answer
PasTest Store Histoplasmosis

My Account

Profile
Sarcoidosis is most common in Black women, but the
Newsletters
aetiology of sarcoidosis is unknown. It is characterised
My Career and Exams by the presence of non-caseating granulomas in at least
Order History two organs. Respiratory involvement has been Reference: Normal Values
described in more than 90% of patients. Lung
Learning Goals involvement includes both the lymph nodes and the Haematology
Question Filters lung parenchyma. Clinical manifestations include dry
cough, dyspnoea and arthralgias, with systemic Immunoglobulins
symptoms including fever, weight loss and fatigue.
Biochemistry
Security Erythema nodosum is also frequently seen in patients
with sarcoidosis. Diabetes
Change Password
Sign Out Endocrinology
476
Blood gases

CSF

© 2011 PasTest Ltd | About Us | Contact Us | Help

file:///E|/Shakhawan/Respiratory%20S/86a.htm[3/13/2012 4:00:04 PM]


MyPasTest

Main Navigation
Question Browser: MRCP 1
Home
Subscriptions Question Browser Exam Builder Saved Exams
» MRCP 1 Session Progress
• Question Browser
Questions Correct 8
• Timed Test A 24-year-old thin man complains of constant daytime
sleepiness. He mentions involuntary naps, often in the Questions Incorrect 4
• Mock Exam
middle of activity, which occur suddenly and without Questions Total 12
• Past Papers warning. He also caused an accident when he fell asleep
Questions Percentage 66 %
• Random Questions while driving home from work. The patient works as an
office manager and has no history of exposure to More
• My Performance
chemicals. Which of the following treatments would be
• Media Bank indicated?
• New Multimedia
Nortriptyline Reference: Normal Values
Online Extras
Fluoxetine Haematology
Library
Diazepam
Community Immunoglobulins
Modafinil
Help Biochemistry
Continuous positive airway pressure-breathing device
PasTest Store Diabetes
477
Endocrinology
My Account
Blood gases
Profile
CSF
Newsletters
My Career and Exams
Order History
Learning Goals
Question Filters

Security

Change Password
Sign Out

© 2011 PasTest Ltd | About Us | Contact Us | Help

file:///E|/Shakhawan/Respiratory%20S/87.htm[3/13/2012 4:00:06 PM]


MyPasTest

Main Navigation
Question Browser: MRCP 1
Home
Subscriptions Question Browser Exam Builder Saved Exams
» MRCP 1 Session Progress
• Question Browser
Questions Correct 8
• Timed Test A 24-year-old thin man complains of constant daytime
sleepiness. He mentions involuntary naps, often in the Questions Incorrect 5
• Mock Exam
middle of activity, which occur suddenly and without Questions Total 13
• Past Papers warning. He also caused an accident when he fell asleep
Questions Percentage 61 %
• Random Questions while driving home from work. The patient works as an
office manager and has no history of exposure to More
• My Performance
chemicals. Which of the following treatments would be
• Media Bank indicated?
• New Multimedia
Nortriptyline
Online Extras
Fluoxetine
Library
Diazepam
Community
Modafinil Correct answer
Help
Continuous positive airway Your answer
PasTest Store pressure-breathing device

My Account

Profile
This patient has narcolepsy: a sleep disorder causing
Newsletters
hypersomnia, which usually starts in adolescence or
My Career and Exams young adulthood. Treatment involves the use of central
Order History nervous system stimulants such as modafinil to allow Reference: Normal Values
daytime functioning.
Learning Goals Haematology
Question Filters Continuous positive airway pressure-breathing devices
are used in the treatment of sleep apnoea. A typical Immunoglobulins
patient with sleep apnoea is usually older, obese and
there will be a long history of gradually worsening Biochemistry
Security
snoring with apnoeas, possibly witnessed by the spouse, Diabetes
Change Password who will probably have moved out of the bedroom
because of the noise. There is usually a history of fairly Endocrinology
Sign Out
high alcohol intake and smoking.
Blood gases

CSF
477

© 2011 PasTest Ltd | About Us | Contact Us | Help

file:///E|/Shakhawan/Respiratory%20S/87a.htm[3/13/2012 4:00:07 PM]


MyPasTest

Main Navigation
Question Browser: MRCP 1
Home
Subscriptions Question Browser Exam Builder Saved Exams
» MRCP 1 Session Progress
• Question Browser
Questions Correct 8
• Timed Test A patient with small-cell lung cancer has a serum sodium
concentration of 121 mmol/l. Which of the following is Questions Incorrect 5
• Mock Exam
the most likely cause? Questions Total 13
• Past Papers
Questions Percentage 61 %
• Random Questions Sodium-restricted diet
• My Performance Sodium-reduced water drinking More
• Media Bank SIADH
• New Multimedia Liver metastases
Reference: Normal Values
Online Extras Bone metastases
Haematology
Library
478
Community Immunoglobulins
Help Biochemistry
PasTest Store Diabetes
Endocrinology
My Account
Blood gases
Profile
CSF
Newsletters
My Career and Exams
Order History
Learning Goals
Question Filters

Security

Change Password
Sign Out

© 2011 PasTest Ltd | About Us | Contact Us | Help

file:///E|/Shakhawan/Respiratory%20S/88.htm[3/13/2012 4:00:08 PM]


MyPasTest

Main Navigation
Question Browser: MRCP 1
Home
Subscriptions Question Browser Exam Builder Saved Exams
» MRCP 1 Session Progress
• Question Browser
Questions Correct 9
• Timed Test A patient with small-cell lung cancer has a serum sodium
concentration of 121 mmol/l. Which of the following is Questions Incorrect 5
• Mock Exam
the most likely cause? Questions Total 14
• Past Papers
Questions Percentage 64 %
• Random Questions Sodium-restricted diet
• My Performance Sodium-reduced water drinking More
• Media Bank SIADH Your answer
• New Multimedia Liver metastases
Online Extras Bone metastases
Library
Community
Help
PasTest Store The continued secretion of vasopressin (antidiuretic
hormone, ADH) in an amount in excess of the body’s
needs, leads to overhydration in both the intracellular
My Account and extracellular compartments – the so-called
‘syndrome of inappropriate antidiuretic hormone
Profile secretion’ (SIADH). The cerebral oedema resulting from
water intoxication causes drowsiness, lethargy,
Newsletters
irritability, mental confusion and disorientation, with
My Career and Exams seizures and coma being the most profound features.
Order History Peripheral oedema is remarkably rare. The patient is Reference: Normal Values
usually asymptomatic until the sodium concentration
Learning Goals falls below 120 mmol/l and the hyponatraemia is Haematology
Question Filters dilutional in type with a low serum osmolality. Urine
osmolality usually exceeds 300 mOsmol/kg. The Immunoglobulins
commonest cancer causing this syndrome is small-cell
Biochemistry
Security cancer, where it is clinically obvious in 10% of cases,
with subclinical involvement detectable by a water- Diabetes
Change Password loading test in more than 50%. Restriction of fluid to a
Sign Out daily intake of 700–1000 ml may redress the Endocrinology
hyponatraemia, but demethylchlortetracycline
(demeclocycline) 600–1200 mg daily is often highly Blood gases
effective, making water restriction unnecessary.Infusion CSF
of hypertonic saline is hazardous, often precipitating
cardiac failure or cerebral oedema.

478

© 2011 PasTest Ltd | About Us | Contact Us | Help

file:///E|/Shakhawan/Respiratory%20S/88a.htm[3/13/2012 4:00:10 PM]


MyPasTest

Main Navigation
Question Browser: MRCP 1
Home
Subscriptions Question Browser Exam Builder Saved Exams
» MRCP 1 Session Progress
• Question Browser
Questions Correct 9
• Timed Test Which of the following is the best agent for treating
chlamydia pneumonia in a woman who is 25 weeks Questions Incorrect 5
• Mock Exam
pregnant? Questions Total 14
• Past Papers
Questions Percentage 64 %
• Random Questions Erythromycin
• My Performance Piperacillin More
• Media Bank Clindamycin
• New Multimedia Ampicillin
Reference: Normal Values
Online Extras Imipenem
Haematology
Library
479
Community Immunoglobulins
Help Biochemistry
PasTest Store Diabetes
Endocrinology
My Account
Blood gases
Profile
CSF
Newsletters
My Career and Exams
Order History
Learning Goals
Question Filters

Security

Change Password
Sign Out

© 2011 PasTest Ltd | About Us | Contact Us | Help

file:///E|/Shakhawan/Respiratory%20S/89.htm[3/13/2012 4:00:11 PM]


MyPasTest

Main Navigation
Question Browser: MRCP 1
Home
Subscriptions Question Browser Exam Builder Saved Exams
» MRCP 1 Session Progress
• Question Browser
Questions Correct 10
• Timed Test Which of the following is the best agent for treating
chlamydia pneumonia in a woman who is 25 weeks Questions Incorrect 5
• Mock Exam
pregnant? Questions Total 15
• Past Papers
Questions Percentage 66 %
• Random Questions Erythromycin Your answer
• My Performance Piperacillin More
• Media Bank Clindamycin
• New Multimedia Ampicillin
Online Extras Imipenem
Library
Community
Help
PasTest Store Macrolide antibiotics (e.g. erythromycin) are the
treatment of choice for chlamydia and other atypical
pneumonias. The most frequent side-effects are
My Account nausea, vomiting and diarrhoea. It should be kept in
mind that treatment is likely to be most effective when
Profile given over a long rather than a short time, suboptimal
doses are avoided and compliance is strict. In this case
Newsletters
macrolides would have the best evidence of safety in
My Career and Exams pregnancy and efficacy, although most evidence to
Order History support safe use is in the third trimester. Reference: Normal Values
Learning Goals Haematology
Question Filters 479
Immunoglobulins
Biochemistry
Security
Diabetes
Change Password
Sign Out Endocrinology

Blood gases

CSF

© 2011 PasTest Ltd | About Us | Contact Us | Help

file:///E|/Shakhawan/Respiratory%20S/89a.htm[3/13/2012 4:00:13 PM]


MyPasTest

Main Navigation
Question Browser: MRCP 1
Home
Subscriptions Question Browser Exam Builder Saved Exams
» MRCP 1 Session Progress
• Question Browser
Questions Correct 10
• Timed Test Which pulmonary function test may be altered to a
similar degree in both restrictive lung disease and Questions Incorrect 5
• Mock Exam
obstructive lung disease? Questions Total 15
• Past Papers
Questions Percentage 66 %
• Random Questions Residual volume
• My Performance Tidal volume More
• Media Bank Total lung capacity
• New Multimedia Forced expiratory volume in 1 second/forced vital
Reference: Normal Values
Online Extras capacity (FEV 1 /FVC) ratio
Forced expiratory volume in 1 second (FEV 1 ) Haematology
Library
Community Immunoglobulins
480
Help Biochemistry
PasTest Store Diabetes
Endocrinology
My Account
Blood gases
Profile
CSF
Newsletters
My Career and Exams
Order History
Learning Goals
Question Filters

Security

Change Password
Sign Out

© 2011 PasTest Ltd | About Us | Contact Us | Help

file:///E|/Shakhawan/Respiratory%20S/90.htm[3/13/2012 4:00:14 PM]


MyPasTest

Main Navigation
Question Browser: MRCP 1
Home
Subscriptions Question Browser Exam Builder Saved Exams
» MRCP 1 Session Progress
• Question Browser
Questions Correct 10
• Timed Test Which pulmonary function test may be altered to a
similar degree in both restrictive lung disease and Questions Incorrect 6
• Mock Exam
obstructive lung disease? Questions Total 16
• Past Papers
Questions Percentage 62 %
• Random Questions Residual volume
• My Performance Tidal volume Correct answer More
• Media Bank Total lung capacity
• New Multimedia Forced expiratory volume in 1 Your answer
Online Extras second/forced vital capacity
(FEV 1 /FVC) ratio
Library
Forced expiratory volume in 1
Community second (FEV 1 )
Help
PasTest Store

My Account Tidal volume = the amount of gas inspired or expired


with each breath.
Profile
Newsletters Residual volume = the volume of air remaining in the
lung after a maximal expiration (cannot be measured
My Career and Exams with spirometry).
Order History Reference: Normal Values
Total lung capacity = the volume of air remaining in the
Learning Goals lung after a maximal inspiration. Haematology
Question Filters
Residual volume is increased in obstructive lung disease Immunoglobulins
due to air trapping. The FEV1 /FVC ratio is < 70% in
Biochemistry
Security obstructive lung disease and > 70% in restrictive lung
disease. Diabetes
Change Password
Sign Out Endocrinology
480 Blood gases

CSF

© 2011 PasTest Ltd | About Us | Contact Us | Help

file:///E|/Shakhawan/Respiratory%20S/90a.htm[3/13/2012 4:00:16 PM]


MyPasTest

Main Navigation
Question Browser: MRCP 1
Home
Subscriptions Question Browser Exam Builder Saved Exams
» MRCP 1 Session Progress
• Question Browser
Questions Correct 10
• Timed Test A 26-year-old man presents with fever, headache and a
moderate productive cough. The chest X-ray shows Questions Incorrect 6
• Mock Exam
increased interstitial markings. Laboratory examination Questions Total 16
• Past Papers shows an elevated LDH and anaemia with the presence of
Questions Percentage 62 %
• Random Questions cold agglutinins. What is the most likely diagnosis?
• My Performance More
Non-Hodgkin’s lymphoma
• Media Bank
Pneumocystis jiroveci pneumonia
• New Multimedia
Mycoplasma pneumonia Reference: Normal Values
Online Extras
Exogene allergic alveolitis Haematology
Library
Chlamydia pneumonia
Community Immunoglobulins
Help 481 Biochemistry
PasTest Store Diabetes
Endocrinology
My Account
Blood gases
Profile
CSF
Newsletters
My Career and Exams
Order History
Learning Goals
Question Filters

Security

Change Password
Sign Out

© 2011 PasTest Ltd | About Us | Contact Us | Help

file:///E|/Shakhawan/Respiratory%20S/91.htm[3/13/2012 4:00:17 PM]


MyPasTest

Main Navigation
Question Browser: MRCP 1
Home
Subscriptions Question Browser Exam Builder Saved Exams
» MRCP 1 Session Progress
• Question Browser
Questions Correct 11
• Timed Test A 26-year-old man presents with fever, headache and a
moderate productive cough. The chest X-ray shows Questions Incorrect 6
• Mock Exam
increased interstitial markings. Laboratory examination Questions Total 17
• Past Papers shows an elevated LDH and anaemia with the presence of
Questions Percentage 64 %
• Random Questions cold agglutinins. What is the most likely diagnosis?
• My Performance More
Non-Hodgkin’s lymphoma
• Media Bank
Pneumocystis jiroveci pneumonia
• New Multimedia
Mycoplasma pneumonia Your answer
Online Extras
Exogene allergic alveolitis
Library
Chlamydia pneumonia
Community
Help
PasTest Store
Acute, cold autoimmune haemolytic anaemia is
My Account commonly seen in adolescents and young adults
following infection with Mycoplasma pneumoniae.
Profile Haemolysis occurs approximately 1–2 weeks following
infection and is most commonly associated with a rise in
Newsletters
polyclonal anti-I IgM antibodies with Mycoplasma
My Career and Exams pneumonia. The typical patient is usually a young adult
Order History who experiences a respiratory tract infection Reference: Normal Values
accompanied by headache, myalgia, cough and fever,
Learning Goals and with a chest X-ray that shows bronchopneumonia. Haematology
Question Filters The cough is often non-productive, but when sputum is
obtained it is mucoid, shows predominantly Immunoglobulins
mononuclear cells and no dominant organism. A
Biochemistry
Security characteristic feature is the relatively high frequency of
extrapulmonary complications such as rash, neurological Diabetes
Change Password syndromes (aseptic meningitis, encephalitis,
Sign Out neuropathies), myocarditis, pericarditis and haemolytic Endocrinology
anaemia. The diagnosis should be suspected in those
patients with a relatively mild form of pneumonia, Blood gases
particularly in previously healthy young adults. With CSF
regard to treatment, the pathogen lacks a cell wall and
hence is not susceptible to penicillin, cephalosporins or
other cell wall-active antibiotics. The usual therapeutic
agents are macrolides such as erythromycin,
clarithromycin, azithromycin or doxycycline.

481

© 2011 PasTest Ltd | About Us | Contact Us | Help

file:///E|/Shakhawan/Respiratory%20S/91a.htm[3/13/2012 4:00:18 PM]


MyPasTest

Main Navigation
Question Browser: MRCP 1
Home
Subscriptions Question Browser Exam Builder Saved Exams
» MRCP 1 Session Progress
• Question Browser
Questions Correct 11
• Timed Test A 24-year-old man with HIV and a CD4+ lymphocyte count
Questions Incorrect 6
• Mock Exam of 150 μ/l has been complaining of gradually worsening
dyspnoea associated with a non-productive cough and Questions Total 17
• Past Papers
fever for the last 2 weeks. A chest X-ray shows bilateral Questions Percentage 64 %
• Random Questions diffuse ground-glass opacities.
• My Performance More
What is the diagnosis?
• Media Bank
• New Multimedia Tuberculosis
Reference: Normal Values
Online Extras Legionellosis
Pneumocystis jiroveci pneumonia Haematology
Library
Community Infectious mononucleosis Immunoglobulins
Help Toxoplasmosis Biochemistry
PasTest Store Diabetes
482
Endocrinology
My Account
Blood gases
Profile
CSF
Newsletters
My Career and Exams
Order History
Learning Goals
Question Filters

Security

Change Password
Sign Out

© 2011 PasTest Ltd | About Us | Contact Us | Help

file:///E|/Shakhawan/Respiratory%20S/92.htm[3/13/2012 4:00:20 PM]


MyPasTest

Main Navigation
Question Browser: MRCP 1
Home
Subscriptions Question Browser Exam Builder Saved Exams
» MRCP 1 Session Progress
• Question Browser
Questions Correct 12
• Timed Test A 24-year-old man with HIV and a CD4+ lymphocyte count
Questions Incorrect 6
• Mock Exam of 150 μ/l has been complaining of gradually worsening
dyspnoea associated with a non-productive cough and Questions Total 18
• Past Papers
fever for the last 2 weeks. A chest X-ray shows bilateral Questions Percentage 66 %
• Random Questions diffuse ground-glass opacities.
• My Performance More
What is the diagnosis?
• Media Bank
• New Multimedia Tuberculosis
Online Extras Legionellosis
Library Pneumocystis jiroveci pneumonia Your answer
Community Infectious mononucleosis
Help Toxoplasmosis
PasTest Store

My Account
Pneumocystis jiroveci pneumonia (previously known as
Profile Pneumocystis carinii pneumonia) typically presents with
Newsletters gradually increasing dyspnoea and cough over weeks,
but sometimes it presents as an acute illness with rapid
My Career and Exams deterioration over a few days. The chest X-ray usually
Order History shows diffuse ground-glass opacities, which strongly Reference: Normal Values
suggests the diagnosis; sometimes, however, it shows
Learning Goals nodular opacities, lobar consolidation or a normal film. Haematology
Question Filters Cystic abnormalities and spontaneous pneumothoraces
in patients with known or suspected HIV infection are Immunoglobulins
usually caused by Pneumocystis jiroveci pneumonia. Biochemistry
Security Pneumocystis jiroveci pneumonia is unlikely in a patient
who had a CD4+ cell count above 200 cells/µl in the Diabetes
Change Password preceding 2 months in the absence of other HIV-
Sign Out associated symptoms. Approximately 90% of patients Endocrinology
with Pneumocystis jiroveci pneumonia have an elevated
Blood gases
serum lactic dehydrogenase, but this may occur with
other pulmonary diseases. In recognition of its genetic CSF
and functional distinctness, the organism that causes
human PCP was renamed Pneumocystis jiroveci (Frenkel
1999). Changing the organism's name does not
preclude the use of the acronym PCP because it can be
read "Pneumocystis pneumonia."

482

© 2011 PasTest Ltd | About Us | Contact Us | Help

file:///E|/Shakhawan/Respiratory%20S/92a.htm[3/13/2012 4:00:21 PM]


MyPasTest

Main Navigation
Question Browser: MRCP 1
Home
Subscriptions Question Browser Exam Builder Saved Exams
» MRCP 1 Session Progress
• Question Browser
Questions Correct 12
• Timed Test A patient with tuberculosis was initially treated with
streptomycin, later this was changed to a combination of Questions Incorrect 6
• Mock Exam
isoniazid, rifampicin, pyrazinamide and ethambutol. He now Questions Total 18
• Past Papers complains of impaired visual acuity and loss of red/green
Questions Percentage 66 %
• Random Questions colour discrimination. Which drug is responsible?
• My Performance More
Streptomycin
• Media Bank
Ethambutol
• New Multimedia
Rifampicin Reference: Normal Values
Online Extras
Pyrazinamide Haematology
Library
Isoniazid
Community Immunoglobulins
Help 483 Biochemistry
PasTest Store Diabetes
Endocrinology
My Account
Blood gases
Profile
CSF
Newsletters
My Career and Exams
Order History
Learning Goals
Question Filters

Security

Change Password
Sign Out

© 2011 PasTest Ltd | About Us | Contact Us | Help

file:///E|/Shakhawan/Respiratory%20S/93.htm[3/13/2012 4:00:23 PM]


MyPasTest

Main Navigation
Question Browser: MRCP 1
Home
Subscriptions Question Browser Exam Builder Saved Exams
» MRCP 1 Session Progress
• Question Browser
Questions Correct 13
• Timed Test A patient with tuberculosis was initially treated with
streptomycin, later this was changed to a combination of Questions Incorrect 6
• Mock Exam
isoniazid, rifampicin, pyrazinamide and ethambutol. He now Questions Total 19
• Past Papers complains of impaired visual acuity and loss of red/green
Questions Percentage 68 %
• Random Questions colour discrimination. Which drug is responsible?
• My Performance More
Streptomycin
• Media Bank
Ethambutol Your answer
• New Multimedia
Rifampicin
Online Extras
Pyrazinamide
Library
Isoniazid
Community
Help
PasTest Store
The side-effects of ethambutol are largely confined to
My Account visual disturbances in the form of loss of acuity, colour
blindness or restriction of visual fields. These toxic
Profile effects are more common where an excessive dosage is
used or if the patient’s renal function is impaired. The
Newsletters
earliest features of ocular toxicity are subjective and
My Career and Exams patients should be advised to discontinue therapy
Order History immediately if they develop deterioration in vision and Reference: Normal Values
to seek further advice promptly. Early discontinuation of
Learning Goals the drug is almost always followed by recovery of Haematology
Question Filters eyesight. Patients who cannot understand warnings
about visual side-effects should, if possible, be given an Immunoglobulins
alternative drug. In particular, ethambutol should be
Biochemistry
Security used with caution in children until they are at least 5
years old and capable of reporting symptomatic visual Diabetes
Change Password changes accurately.
Sign Out Endocrinology

Blood gases
483
CSF

© 2011 PasTest Ltd | About Us | Contact Us | Help

file:///E|/Shakhawan/Respiratory%20S/93a.htm[3/13/2012 4:00:24 PM]


MyPasTest

Main Navigation
Question Browser: MRCP 1
Home
Subscriptions Question Browser Exam Builder Saved Exams
» MRCP 1 Session Progress
• Question Browser
Questions Correct 13
• Timed Test A 60-year-old man presents to the clinic complaining of a
6-month history of dyspnoea on exertion and a non- Questions Incorrect 6
• Mock Exam
productive cough. On examination there is clubbing, and Questions Total 19
• Past Papers crepitations are heard at the lung bases. Lung function
Questions Percentage 68 %
• Random Questions tests show a reduced vital capacity and an increased
FEV1 /FVC ratio. What is the most likely diagnosis? More
• My Performance
• Media Bank Idiopathic pulmonary fibrosis
• New Multimedia Carcinoma of the lung Reference: Normal Values
Online Extras COPD
Haematology
Library Tuberculosis
Community Immunoglobulins
Bronchiectasis
Help Biochemistry
PasTest Store 484
Diabetes
Endocrinology
My Account
Blood gases
Profile
CSF
Newsletters
My Career and Exams
Order History
Learning Goals
Question Filters

Security

Change Password
Sign Out

© 2011 PasTest Ltd | About Us | Contact Us | Help

file:///E|/Shakhawan/Respiratory%20S/94.htm[3/13/2012 4:00:25 PM]


MyPasTest

Main Navigation
Question Browser: MRCP 1
Home
Subscriptions Question Browser Exam Builder Saved Exams
» MRCP 1 Session Progress
• Question Browser
Questions Correct 14
• Timed Test A 60-year-old man presents to the clinic complaining of a
6-month history of dyspnoea on exertion and a non- Questions Incorrect 6
• Mock Exam
productive cough. On examination there is clubbing, and Questions Total 20
• Past Papers crepitations are heard at the lung bases. Lung function
Questions Percentage 70 %
• Random Questions tests show a reduced vital capacity and an increased
FEV1 /FVC ratio. What is the most likely diagnosis? More
• My Performance
• Media Bank Idiopathic pulmonary fibrosis Your answer
• New Multimedia Carcinoma of the lung
Online Extras COPD
Library Tuberculosis
Community Bronchiectasis
Help
PasTest Store

My Account Idiopathic fibrosing alveolitis (IPF), synonymous with


cryptogenic fibrosing alveolitis (CFA), is a progressive
Profile and usually fatal disease of unknown cause
Newsletters characterised by sequential acute lung injury with
subsequent scarring adn end-stage lung disease. It may
My Career and Exams occur in any decade of life but is most commonly seen
Order History between the ages of 50 and 60 years; it is slightly Reference: Normal Values
more frequent in males than females. A history of
Learning Goals Haematology
progressive breathlessness on exertion in the absence
Question Filters of wheeze is typical. A dry cough may be present, but
Immunoglobulins
sputum production is unusual until the later stages of
the disease. Haemoptysis is uncommon, but should Biochemistry
Security suggest the development of lung malignancy that occurs
with a 7- to 14-fold relative risk in patients with Diabetes
Change Password cryptogenic fibrosing alveolitis. Chest pain is
Sign Out uncommon. Constitutional symptoms such as weight Endocrinology
loss and lethargy are recognised. Lung function tests Blood gases
show a restrictive picture. Recent re-classification of the
group of idiopathic interstitial pneumonias have allowed CSF
characterisation of seven different histological patterns
based on lung biopsy analysis; the pattern in IPF is that
of usual interstitial pneumonia (UIP).

484

© 2011 PasTest Ltd | About Us | Contact Us | Help

file:///E|/Shakhawan/Respiratory%20S/94a.htm[3/13/2012 4:00:27 PM]


MyPasTest

Main Navigation
Question Browser: MRCP 1
Home
Subscriptions Question Browser Exam Builder Saved Exams
» MRCP 1 Session Progress
• Question Browser
Questions Correct 14
• Timed Test A 60-year-old smoker is being evaluated for a coronary
bypass graft. Which is the best preoperative screen of Questions Incorrect 6
• Mock Exam
pulmonary function for this patient? Questions Total 20
• Past Papers
Questions Percentage 70 %
• Random Questions Arterial pH
• My Performance Arterial p(CO 2 ) More
• Media Bank Arterial p(O2 )
• New Multimedia
Forced expiratory volume in 1 second/forced vital Reference: Normal Values
Online Extras capacity (FEV 1 /FVC) ratio
Haematology
Library Oxygen saturation
Community Immunoglobulins
485
Help Biochemistry
PasTest Store Diabetes
Endocrinology
My Account
Blood gases
Profile
CSF
Newsletters
My Career and Exams
Order History
Learning Goals
Question Filters

Security

Change Password
Sign Out

© 2011 PasTest Ltd | About Us | Contact Us | Help

file:///E|/Shakhawan/Respiratory%20S/95.htm[3/13/2012 4:00:28 PM]


MyPasTest

Main Navigation
Question Browser: MRCP 1
Home
Subscriptions Question Browser Exam Builder Saved Exams
» MRCP 1 Session Progress
• Question Browser
Questions Correct 14
• Timed Test A 60-year-old smoker is being evaluated for a coronary
bypass graft. Which is the best preoperative screen of Questions Incorrect 7
• Mock Exam
pulmonary function for this patient? Questions Total 21
• Past Papers
Questions Percentage 66 %
• Random Questions Arterial pH
• My Performance Arterial p(CO 2 ) Your answer More
• Media Bank Arterial p(O2 )
• New Multimedia
Forced expiratory volume in 1 Correct answer
Online Extras second/forced vital capacity
Library (FEV 1 /FVC) ratio

Community Oxygen saturation


Help
PasTest Store

Pulmonary function tests provide the best preoperative


My Account screen, because they reflect dynamic measurements of
Profile pulmonary function in the patient. The arterial blood gas
measurements reflect acid–base disorders, but they do
Newsletters not predict pulmonary complications in the
My Career and Exams postoperative state.
Order History Reference: Normal Values
Learning Goals 485 Haematology
Question Filters
Immunoglobulins
Biochemistry
Security
Diabetes
Change Password
Sign Out Endocrinology

Blood gases

CSF

© 2011 PasTest Ltd | About Us | Contact Us | Help

file:///E|/Shakhawan/Respiratory%20S/95a.htm[3/13/2012 4:00:30 PM]


MyPasTest

Main Navigation
Question Browser: MRCP 1
Home
Subscriptions Question Browser Exam Builder Saved Exams
» MRCP 1 Session Progress
• Question Browser
Questions Correct 14
• Timed Test A 49-year-old woman has been admitted with haemoptysis
and epistaxis, the chest X-ray shows multiple rounded Questions Incorrect 7
• Mock Exam
lesions with alveolar shadowing. Serum is c-ANCA positive. Questions Total 21
• Past Papers What is the most likely diagnosis?
Questions Percentage 66 %
• Random Questions
• My Performance Tuberculosis More
• Media Bank Carcinoma of the lung

• New Multimedia Echinococcosis


Reference: Normal Values
Online Extras Wegener’s granulomatosis
Systemic lupus erythematosus Haematology
Library
Community Immunoglobulins
486
Help Biochemistry
PasTest Store Diabetes
Endocrinology
My Account
Blood gases
Profile
CSF
Newsletters
My Career and Exams
Order History
Learning Goals
Question Filters

Security

Change Password
Sign Out

© 2011 PasTest Ltd | About Us | Contact Us | Help

file:///E|/Shakhawan/Respiratory%20S/96.htm[3/13/2012 4:00:31 PM]


MyPasTest

Main Navigation
Question Browser: MRCP 1
Home
Subscriptions Question Browser Exam Builder Saved Exams
» MRCP 1 Session Progress
• Question Browser
Questions Correct 15
• Timed Test A 49-year-old woman has been admitted with haemoptysis
and epistaxis, the chest X-ray shows multiple rounded Questions Incorrect 7
• Mock Exam
lesions with alveolar shadowing. Serum is c-ANCA positive. Questions Total 22
• Past Papers What is the most likely diagnosis?
Questions Percentage 68 %
• Random Questions
• My Performance Tuberculosis More
• Media Bank Carcinoma of the lung

• New Multimedia Echinococcosis

Online Extras Wegener’s granulomatosis Your answer

Library Systemic lupus erythematosus

Community
Help
PasTest Store
Almost all the patients so diagnosed have evidence of
granulomatous lung disease at presentation, which is
My Account often accompanied by alveolar capillaritis. The bronchi
can also be affected and bronchial stenoses occur as
Profile late manifestations. Symptoms include cough,
dyspnoea, haemoptysis and chest pain, which can be
Newsletters
pleuritic. Signs on chest examination depend on the
My Career and Exams nature of the pulmonary lesions and include fine
Order History crepitations and bronchial breathing or, less commonly, Reference: Normal Values
pleural rubs and signs of pleural effusions. Pulmonary
Learning Goals granulomas are usually diagnosed from chest X-ray and Haematology
Question Filters CT scans. They may appear as single or multiple
rounded lesions, which can cavitate. Bronchoscopy often Immunoglobulins
reveals granulomatous inflammation and the diagnosis
Biochemistry
Security can sometimes be made from bronchial biopsies.
Diabetes
Change Password
Sign Out 486 Endocrinology

Blood gases

CSF

© 2011 PasTest Ltd | About Us | Contact Us | Help

file:///E|/Shakhawan/Respiratory%20S/96a.htm[3/13/2012 4:00:33 PM]


MyPasTest

Main Navigation
Question Browser: MRCP 1
Home
Subscriptions Question Browser Exam Builder Saved Exams
» MRCP 1 Session Progress
• Question Browser
Questions Correct 15
• Timed Test Which drug therapy is indicated for a young adult
with mild intermittent asthma (no night-time Questions Incorrect 7
• Mock Exam
symptoms, no trigger) as required medication? Questions Total 22
• Past Papers
Questions Percentage 68 %
• Random Questions Budesonide inhaler
• My Performance Salbutamol inhaler More
• Media Bank Salmeterol inhaler
• New Multimedia Oral cromoglycate
Reference: Normal Values
Online Extras Oral leukotriene-receptor antagonists
Haematology
Library
487
Community Immunoglobulins
Help Biochemistry
PasTest Store Diabetes
Endocrinology
My Account
Blood gases
Profile
CSF
Newsletters
My Career and Exams
Order History
Learning Goals
Question Filters

Security

Change Password
Sign Out

© 2011 PasTest Ltd | About Us | Contact Us | Help

file:///E|/Shakhawan/Respiratory%20S/97.htm[3/13/2012 4:00:34 PM]


MyPasTest

Main Navigation
Question Browser: MRCP 1
Home
Subscriptions Question Browser Exam Builder Saved Exams
» MRCP 1 Session Progress
• Question Browser
Questions Correct 16
• Timed Test Which drug therapy is indicated for a young adult
with mild intermittent asthma (no night-time Questions Incorrect 7
• Mock Exam
symptoms, no trigger) as required medication? Questions Total 23
• Past Papers
Questions Percentage 69 %
• Random Questions Budesonide inhaler
• My Performance Salbutamol inhaler Your answer More
• Media Bank Salmeterol inhaler
• New Multimedia Oral cromoglycate
Online Extras Oral leukotriene-receptor antagonists
Library
Community
Help
PasTest Store Occasional-relief bronchodilators are indicated in step 1
in the treatment of chronic asthma, if relief is only
needed once daily, if there are no night-time symptoms
My Account or no impaired lung function. Regular inhaled
corticosteroids are added in step 2. Regular inhaled
Profile long-acting β2 -agonists (salmeterol) are added in step
Newsletters 3. Oral leukotriene-receptor antagonists can also be
added in step 3 if the asthma is still not controlled.
My Career and Exams
Order History Reference: Normal Values
Learning Goals 487
Haematology
Question Filters
Immunoglobulins
Biochemistry
Security
Diabetes
Change Password
Sign Out Endocrinology

Blood gases

CSF

© 2011 PasTest Ltd | About Us | Contact Us | Help

file:///E|/Shakhawan/Respiratory%20S/97a.htm[3/13/2012 4:00:35 PM]


MyPasTest

Main Navigation
Question Browser: MRCP 1
Home
Subscriptions Question Browser Exam Builder Saved Exams
» MRCP 1 Session Progress
• Question Browser
Questions Correct 16
• Timed Test A young adult was referred because of cough and
shortness of breath. An extrinsic allergic alveolitis was Questions Incorrect 7
• Mock Exam
diagnosed. Beside reduction of exposure to the Questions Total 23
• Past Papers allergen, which other therapy is most likely to be
Questions Percentage 69 %
• Random Questions successful?
• My Performance More
Antibiotics
• Media Bank
Non-steroidal anti-inflammatory drugs
• New Multimedia
Immunoglobulins Reference: Normal Values
Online Extras
Corticosteroids Haematology
Library
Desensitisation
Community Immunoglobulins
Help 488 Biochemistry
PasTest Store Diabetes
Endocrinology
My Account
Blood gases
Profile
CSF
Newsletters
My Career and Exams
Order History
Learning Goals
Question Filters

Security

Change Password
Sign Out

© 2011 PasTest Ltd | About Us | Contact Us | Help

file:///E|/Shakhawan/Respiratory%20S/98.htm[3/13/2012 4:00:37 PM]


MyPasTest

Main Navigation
Question Browser: MRCP 1
Home
Subscriptions Question Browser Exam Builder Saved Exams
» MRCP 1 Session Progress
• Question Browser
Questions Correct 17
• Timed Test A young adult was referred because of cough and
shortness of breath. An extrinsic allergic alveolitis was Questions Incorrect 7
• Mock Exam
diagnosed. Beside reduction of exposure to the Questions Total 24
• Past Papers allergen, which other therapy is most likely to be
Questions Percentage 70 %
• Random Questions successful?
• My Performance More
Antibiotics
• Media Bank
Non-steroidal anti-inflammatory drugs
• New Multimedia
Immunoglobulins
Online Extras
Corticosteroids Your answer
Library
Desensitisation
Community
Help
PasTest Store
Management of the patient centres on reducing any
My Account further exposure to a minimum, but first the diagnosis
must be secure. Desensitisation has no beneficial effect.
Profile Ideally, affected individuals change their relevant
working, domestic and recreational environment
Newsletters
completely, but this may mean a profound loss of
My Career and Exams income or great expense, and is often unrealistic. Nor is
Order History it fully justified on purely medical grounds since Reference: Normal Values
continued exposure does not inevitably lead to
Learning Goals progressive disease. Affected individuals who continue Haematology
Question Filters to work in the occupation responsible for their disease
can often reduce their exposure substantially by Immunoglobulins
changing the pattern of their particular duties. An
Biochemistry
Security alternative is to use industrial respirators, which filter
out 98–99% of respirable dust from the ambient air. Diabetes
Change Password These are especially valuable when exposures are
Sign Out intermittent and short, but they may be uncomfortably Endocrinology
hot when worn for long periods or when there is heavy
work, and so compliance with their use may be poor. Blood gases
Whatever course is followed, continuing exposure CSF
should be accompanied by regular medical surveillance.
If there is no progression, it is reasonable for some
exposure to continue. When there is progressive
disease, exposure should cease. This may involve a loss
of earnings, and may entitle the affected worker to
compensation. Occasionally, individuals with progressive
disease may refuse to change their occupation or
hobby, and their physician must weigh the possible
advantages of long-term corticosteroid therapy against
the risks.

488

© 2011 PasTest Ltd | About Us | Contact Us | Help

file:///E|/Shakhawan/Respiratory%20S/98a.htm[3/13/2012 4:00:38 PM]


MyPasTest

Main Navigation
Question Browser: MRCP 1
Home
Subscriptions Question Browser Exam Builder Saved Exams
» MRCP 1 Session Progress
• Question Browser
Questions Correct 17
• Timed Test A 24-year-old medical student (height 165 cm, weight 78
kg) has been complaining of a few months’ history of Questions Incorrect 7
• Mock Exam
shortness of breath on exertion and of coughing up blood Questions Total 24
• Past Papers once. She is a few days away from her final examination
Questions Percentage 70 %
• Random Questions and smokes 20 cigarettes per day. She takes no medication
except for the oral contraceptive pill. Only past medical More
• My Performance
history of note is a DVT after a long flight from Australia.
• Media Bank
What is the most likely diagnosis?
• New Multimedia
Reference: Normal Values
Online Extras Hyperventilation syndrome due to stress
Haematology
Library Tuberculosis
Community Recurrent pulmonary embolism Immunoglobulins
Help Sarcoidosis Biochemistry
PasTest Store Goodpasture’s syndrome Diabetes

489 Endocrinology
My Account
Blood gases
Profile
CSF
Newsletters
My Career and Exams
Order History
Learning Goals
Question Filters

Security

Change Password
Sign Out

© 2011 PasTest Ltd | About Us | Contact Us | Help

file:///E|/Shakhawan/Respiratory%20S/99.htm[3/13/2012 4:00:40 PM]


MyPasTest

Main Navigation
Question Browser: MRCP 1
Home
Subscriptions Question Browser Exam Builder Saved Exams
» MRCP 1 Session Progress
• Question Browser
Questions Correct 18
• Timed Test A 24-year-old medical student (height 165 cm, weight 78
kg) has been complaining of a few months’ history of Questions Incorrect 7
• Mock Exam
shortness of breath on exertion and of coughing up blood Questions Total 25
• Past Papers once. She is a few days away from her final examination
Questions Percentage 72 %
• Random Questions and smokes 20 cigarettes per day. She takes no medication
except for the oral contraceptive pill. Only past medical More
• My Performance
history of note is a DVT after a long flight from Australia.
• Media Bank
What is the most likely diagnosis?
• New Multimedia
Online Extras Hyperventilation syndrome due to
Library stress
Community Tuberculosis
Help Recurrent pulmonary embolism Your answer
PasTest Store Sarcoidosis
Goodpasture’s syndrome

My Account

Profile
Newsletters Pulmonary embolism can present in diverse ways. The
My Career and Exams syndrome of pleuritic pain or haemoptysis, in the
absence of circulatory collapse, is the most frequent Reference: Normal Values
Order History
mode of presentation of acute pulmonary embolism. It
Learning Goals occurred in 60% of patients recruited in a collaborative Haematology
Question Filters investigation, the Prospective Investigation of
Pulmonary Embolism Diagnosis (PIOPED). A syndrome Immunoglobulins
of dyspnoea in the absence of haemoptysis or pleuritic
pain or circulatory collapse occurred in 25%. Circulatory Biochemistry
Security
collapse (systolic blood pressure less than 80 mmHg or Diabetes
Change Password loss of consciousness) was an uncommon mode of
presentation, occurring in 15%. Obesity and a high Endocrinology
Sign Out
oestrogen content in oral contraceptives have been
linked to thromboembolic events. Most patients with Blood gases
pulmonary embolism had smoked at one time or
CSF
continued to smoke at the time of their pulmonary
embolism.

489

© 2011 PasTest Ltd | About Us | Contact Us | Help

file:///E|/Shakhawan/Respiratory%20S/99a.htm[3/13/2012 4:00:41 PM]


MyPasTest

Main Navigation
Question Browser: MRCP 1
Home
Subscriptions Question Browser Exam Builder Saved Exams
» MRCP 1 Session Progress
• Question Browser
Questions Correct 18
• Timed Test Which form of lung disease develops typically in
people with a 1 -antitrypsin deficiency? Questions Incorrect 7
• Mock Exam
Questions Total 25
• Past Papers
Atelectasis Questions Percentage 72 %
• Random Questions
Pneumonitis
• My Performance More
Emphysema
• Media Bank
Interstitial fibrosis
• New Multimedia
Bronchiectasis Reference: Normal Values
Online Extras
490 Haematology
Library
Community Immunoglobulins
Help Biochemistry
PasTest Store Diabetes
Endocrinology
My Account
Blood gases
Profile
CSF
Newsletters
My Career and Exams
Order History
Learning Goals
Question Filters

Security

Change Password
Sign Out

© 2011 PasTest Ltd | About Us | Contact Us | Help

file:///E|/Shakhawan/Respiratory%20S/100.htm[3/13/2012 4:00:43 PM]


MyPasTest

Main Navigation
Question Browser: MRCP 1
Home
Subscriptions Question Browser Exam Builder Saved Exams
» MRCP 1 Session Progress
• Question Browser
Questions Correct 19
• Timed Test Which form of lung disease develops typically in
people with a 1 -antitrypsin deficiency? Questions Incorrect 7
• Mock Exam
Questions Total 26
• Past Papers
Atelectasis Questions Percentage 73 %
• Random Questions
Pneumonitis
• My Performance More
Emphysema Your answer
• Media Bank
Interstitial fibrosis
• New Multimedia
Bronchiectasis
Online Extras
Library
Community
Help The association of a 1 -antitrypsin with the development
PasTest Store of premature emphysema has led to the wider
conclusion that emphysema results from an imbalance
between proteases and antiproteases within the lung.
My Account The elastase and a 1 -antitrypsin balance clearly
Profile illustrates the processes involved in the development of
emphysema and the interplay between the
Newsletters environmental and genetic factors that determine its
My Career and Exams onset. Patients usually present with increasing
dyspnoea and weight loss, with cor pulmonale and
Order History polycythaemia occurring late in the course of the Reference: Normal Values
Learning Goals disease. Chest X-rays typically show bilateral basal Haematology
emphysema with paucity and pruning of the basal
Question Filters
pulmonary vessels. Immunoglobulins
Biochemistry
Security
490 Diabetes
Change Password
Sign Out Endocrinology

Blood gases

CSF

© 2011 PasTest Ltd | About Us | Contact Us | Help

file:///E|/Shakhawan/Respiratory%20S/100a.htm[3/13/2012 4:00:44 PM]


MyPasTest

Main Navigation
Question Browser: MRCP 1
Home
Subscriptions Question Browser Exam Builder Saved Exams
» MRCP 1 Session Progress
• Question Browser
Questions Correct 19
• Timed Test A 78 year old patient with COPD was admitted to a medical
ward and received appropriate pharmacological treatment Questions Incorrect 7
• Mock Exam
according to BTS guidelines (systemic steroids, nebulised Questions Total 26
• Past Papers bronchodilators, controlled oxygen therapy). He is severely
Questions Percentage 73 %
• Random Questions disabled and would not be considered for the intensive care
unit and he has previously failed a trial of non-invasive More
• My Performance
ventilation due to inability to tolerate the mask. The night
• Media Bank following his admission his condition worsened. Arterial
• New Multimedia blood gases were measured and demonstrated: pH 7.29,
p a (O2 ) 5.85 kPa, p a (CO 2 ) 9.33 kPa on 28% oxygen Reference: Normal Values
Online Extras
delivered by Venturi mask.
Haematology
Library
What is your next management decision based on Immunoglobulins
Community
published studies?
Help Biochemistry
PasTest Store Start aminophylline infusion
Diabetes
Start doxapram infusion
Endocrinology
My Account Increase oxygen
Give iv antibiotics Blood gases
Profile
Start salbutamol infusion CSF
Newsletters
My Career and Exams 491
Order History
Learning Goals
Question Filters

Security

Change Password
Sign Out

© 2011 PasTest Ltd | About Us | Contact Us | Help

file:///E|/Shakhawan/Respiratory%20S/101.htm[3/13/2012 4:00:46 PM]


MyPasTest

Main Navigation
Question Browser: MRCP 1
Home
Subscriptions Question Browser Exam Builder Saved Exams
» MRCP 1 Session Progress
• Question Browser
Questions Correct 19
• Timed Test A 78 year old patient with COPD was admitted to a medical
ward and received appropriate pharmacological treatment Questions Incorrect 8
• Mock Exam
according to BTS guidelines (systemic steroids, nebulised Questions Total 27
• Past Papers bronchodilators, controlled oxygen therapy). He is severely
Questions Percentage 70 %
• Random Questions disabled and would not be considered for the intensive care
unit and he has previously failed a trial of non-invasive More
• My Performance
ventilation due to inability to tolerate the mask. The night
• Media Bank following his admission his condition worsened. Arterial
• New Multimedia blood gases were measured and demonstrated: pH 7.29,
p a (O2 ) 5.85 kPa, p a (CO 2 ) 9.33 kPa on 28% oxygen
Online Extras
delivered by Venturi mask.
Library
What is your next management decision based on
Community
published studies?
Help
PasTest Store Start aminophylline infusion Your answer
Start doxapram infusion Correct answer

My Account Increase oxygen


Give iv antibiotics
Profile
Start salbutamol infusion
Newsletters
My Career and Exams
Order History Reference: Normal Values
Learning Goals Clinical trials, summarised in a Cochrane review, have Haematology
Question Filters demonstrated a beneficial effect of doxapram over
placebo in the management of acute respiratory failure. Immunoglobulins
In practice, it is not well tolerated by many patients and Biochemistry
Security a therapeutic tolerance tends to develop rendering the
drug less effective after more prolonged use. Its precise Diabetes
Change Password position within the ventilatory support protocol is not
Sign Out well defined, but doxapram can be useful in patients as Endocrinology
a short-term treatment whilst other, more effective,
Blood gases
support is instituted or a decision is made not to
proceed with mechanical support. Aminophylline CSF
infusion has been shown to be beneficial in acute
asthma but not in COPD, although it is still a
consideration in NICE guidelines. Most of the COPD
studies are in mild to moderate COPD exacerbations,
there is little or no published evidence in severe COPD.

491

© 2011 PasTest Ltd | About Us | Contact Us | Help

file:///E|/Shakhawan/Respiratory%20S/101a.htm[3/13/2012 4:00:47 PM]


MyPasTest

Main Navigation
Question Browser: MRCP 1
Home
Subscriptions Question Browser Exam Builder Saved Exams
» MRCP 1 Session Progress
• Question Browser
Questions Correct 19
• Timed Test A 60-year-old patient was referred with a 1-year history of
persistent cough productive of mucopurulent sputum Questions Incorrect 8
• Mock Exam
throughout the year. He has been treated by his GP for Questions Total 27
• Past Papers recurrent chest infections. What is the most likely
Questions Percentage 70 %
• Random Questions diagnosis?
• My Performance More
Fibrosing alveolitis
• Media Bank
Carcinoma of the lung
• New Multimedia
Sarcoidosis Reference: Normal Values
Online Extras
Bronchiectasis Haematology
Library
Allergic asthma
Community Immunoglobulins
Help 492 Biochemistry
PasTest Store Diabetes
Endocrinology
My Account
Blood gases
Profile
CSF
Newsletters
My Career and Exams
Order History
Learning Goals
Question Filters

Security

Change Password
Sign Out

© 2011 PasTest Ltd | About Us | Contact Us | Help

file:///E|/Shakhawan/Respiratory%20S/102.htm[3/13/2012 4:00:49 PM]


MyPasTest

Main Navigation
Question Browser: MRCP 1
Home
Subscriptions Question Browser Exam Builder Saved Exams
» MRCP 1 Session Progress
• Question Browser
Questions Correct 20
• Timed Test A 60-year-old patient was referred with a 1-year history of
persistent cough productive of mucopurulent sputum Questions Incorrect 8
• Mock Exam
throughout the year. He has been treated by his GP for Questions Total 28
• Past Papers recurrent chest infections. What is the most likely
Questions Percentage 71 %
• Random Questions diagnosis?
• My Performance More
Fibrosing alveolitis
• Media Bank
Carcinoma of the lung
• New Multimedia
Sarcoidosis
Online Extras
Bronchiectasis Your answer
Library
Allergic asthma
Community
Help
PasTest Store
Bronchiectasis should be suspected when there is a
My Account history of persistent cough productive of mucopurulent
or purulent sputum throughout the year. Patients have
Profile frequently been treated for recurrent chest infections
and labelled as ‘bronchitic’, often despite the absence of
Newsletters
a history of smoking. Patients may produce mucoid
My Career and Exams sputum early in their disease, developing purulent
Order History sputum when they suffer an exacerbation associated Reference: Normal Values
with a viral upper respiratory tract infection. Such
Learning Goals exacerbations may be associated with pleuritic chest Haematology
Question Filters pain, haemoptysis, fever and sometimes wheeze. Those
presenting as adults often recall a ‘chesty cough’ or Immunoglobulins
‘wheezy bronchitis’ associated with upper respiratory
Biochemistry
Security tract infections in childhood, followed by complete
resolution of symptoms in their teens and early adult life Diabetes
Change Password before the symptoms return after a viral trigger. Upper
Sign Out respiratory tract symptoms such as postnasal drip are Endocrinology
common, and in about 30% of cases there is a history
of chronic sinusitis. Patients with bronchiectasis also Blood gases
suffer from undue tiredness, which many find more CSF
troublesome than the productive cough.

492

© 2011 PasTest Ltd | About Us | Contact Us | Help

file:///E|/Shakhawan/Respiratory%20S/102a.htm[3/13/2012 4:00:50 PM]


MyPasTest

Main Navigation
Question Browser: MRCP 1
Home
Subscriptions Question Browser Exam Builder Saved Exams
» MRCP 1 Session Progress
• Question Browser
Questions Correct 20
• Timed Test A 20-year-old woman complains of a sudden onset of
dyspnoea associated with pleuritic chest pain. Which Questions Incorrect 8
• Mock Exam
assessment is the most accurate to confirm your Questions Total 28
• Past Papers diagnosis of pulmonary embolism?
Questions Percentage 71 %
• Random Questions
• My Performance D-Dimer More
• Media Bank Echocardiography

• New Multimedia Ventilation/perfusion scan


Reference: Normal Values
Online Extras Contrast-enhanced spiral computed tomography
Pulmonary angiography Haematology
Library
Community Immunoglobulins
493
Help Biochemistry
PasTest Store Diabetes
Endocrinology
My Account
Blood gases
Profile
CSF
Newsletters
My Career and Exams
Order History
Learning Goals
Question Filters

Security

Change Password
Sign Out

© 2011 PasTest Ltd | About Us | Contact Us | Help

file:///E|/Shakhawan/Respiratory%20S/103.htm[3/13/2012 4:00:51 PM]


MyPasTest

Main Navigation
Question Browser: MRCP 1
Home
Subscriptions Question Browser Exam Builder Saved Exams
» MRCP 1 Session Progress
• Question Browser
Questions Correct 20
• Timed Test A 20-year-old woman complains of a sudden onset of
dyspnoea associated with pleuritic chest pain. Which Questions Incorrect 9
• Mock Exam
assessment is the most accurate to confirm your Questions Total 29
• Past Papers diagnosis of pulmonary embolism?
Questions Percentage 68 %
• Random Questions
• My Performance D-Dimer More
• Media Bank Echocardiography

• New Multimedia Ventilation/perfusion scan Your answer

Online Extras Contrast-enhanced spiral computed


tomography
Library
Pulmonary angiography Correct answer
Community
Help
PasTest Store
Although pulmonary angiography is associated with
My Account serious complications in about 1% of patients, it
remains the diagnostic reference test for pulmonary
Profile embolism.
Newsletters A negative D-dimer test is useful for excluding
My Career and Exams pulmonary embolism in patients who are clinically
thought to be at low risk, but a ‘positive’ result does not Reference: Normal Values
Order History
establish the diagnosis. Echocardiography may show
Learning Goals right ventricular dilatation and evidence of pulmonary Haematology
Question Filters hypertension, which, in the proper clinical setting, may
strengthen the clinical impression that a pulmonary Immunoglobulins
embolism has occurred. The Prospective Investigation
of Pulmonary Embolism Diagnosis (PIOPED) study Biochemistry
Security
emphasised the poor predictive value of scans reported Diabetes
Change Password as intermediate probability, a common occurrence in
routine clinical practice. Endocrinology
Sign Out
Blood gases
493 CSF

© 2011 PasTest Ltd | About Us | Contact Us | Help

file:///E|/Shakhawan/Respiratory%20S/103a.htm[3/13/2012 4:00:53 PM]


MyPasTest

Main Navigation
Question Browser: MRCP 1
Home
Subscriptions Question Browser Exam Builder Saved Exams
» MRCP 1 Session Progress
• Question Browser
Questions Correct 0
• Timed Test A 58-year-old woman has been admitted with pulmonary
embolism. After 7 days she develops an arterial thrombosis Questions Incorrect 0
• Mock Exam
in her left leg, the thrombocyte count is 40 × 10 9 /l. Which Questions Total 0
• Past Papers
drug is most likely to be responsible? Questions Percentage 0%
• Random Questions
• My Performance Intravenous heparin for acute treatment More
• Media Bank Warfarin for continuous out-patient treatment
• New Multimedia Temazepam for night-time sleep
Reference: Normal Values
Online Extras Tramadol for pain control
Bisacodyl for her constipation Haematology
Library
Community Immunoglobulins
494
Help Biochemistry
PasTest Store Diabetes
Endocrinology
My Account
Blood gases
Profile
CSF
Newsletters
My Career and Exams
Order History
Learning Goals
Question Filters

Security

Change Password
Sign Out

© 2011 PasTest Ltd | About Us | Contact Us | Help

file:///E|/Shakhawan/Respiratory%20S/104.htm[3/13/2012 4:00:54 PM]


MyPasTest

Main Navigation
Question Browser: MRCP 1
Home
Subscriptions Question Browser Exam Builder Saved Exams
» MRCP 1 Session Progress
• Question Browser
Questions Correct 1
• Timed Test A 58-year-old woman has been admitted with pulmonary
Questions Incorrect 0
• Mock Exam embolism. After 7 days she develops an arterial thrombosis
in her left leg, the thrombocyte count is 40 × 10 9 /l. Which Questions Total 1
• Past Papers
drug is most likely to be responsible? Questions Percentage 100 %
• Random Questions
• My Performance Intravenous heparin for acute Your answer More
treatment
• Media Bank
Warfarin for continuous out-patient
• New Multimedia
treatment
Online Extras
Temazepam for night-time sleep
Library
Tramadol for pain control
Community
Bisacodyl for her constipation
Help
PasTest Store

My Account Heparin-induced thrombocytopenia (HIT) is caused by


IgG antibodies that recognise multi-molecular
Profile complexes of platelet factor 4 and heparin. Typically,
Newsletters the fall in platelet count begins 5–10 days after starting
heparin; however, in patients who received heparin
My Career and Exams within the past 100 days, the platelet count can fall
Order History abruptly upon resuming heparin therapy, probably Reference: Normal Values
because of residual circulating HIT antibodies. HIT
Learning Goals Haematology
occurs in as many as 5% of certain high-risk
Question Filters populations. Most patients with HIT develop venous or
Immunoglobulins
arterial thrombosis, most commonly a deep-vein
thrombosis, pulmonary embolism, major limb artery Biochemistry
Security thrombosis, stroke or myocardial infarction. Acute or
chronic adrenal failure from bilateral adrenal Diabetes
Change Password haemorrhagic necrosis has been described. The
Sign Out thrombocytopenia is typically moderate in severity Endocrinology
(median platelet count nadir, 60 × 10 9 /l), but in only Blood gases
10% of patients does the platelet count fall to less than
CSF
20 × 10 9 /l. In at least 10% of patients, the platelet
count never drops below 150 × 10 9 /l.

494

© 2011 PasTest Ltd | About Us | Contact Us | Help

file:///E|/Shakhawan/Respiratory%20S/104a.htm[3/13/2012 4:00:56 PM]


MyPasTest

Main Navigation
Question Browser: MRCP 1
Home
Subscriptions Question Browser Exam Builder Saved Exams
» MRCP 1 Session Progress
• Question Browser
Questions Correct 1
• Timed Test A 50-year-old patient presents with blood eosinophilia in
Questions Incorrect 0
• Mock Exam association with a radiographic pulmonary infiltrate. A
bronchoscopy shows an excess of eosinophils in Questions Total 1
• Past Papers bronchoalveolar lavage fluid in the absence of pathogenic Questions Percentage 100 %
• Random Questions micro-organisms. The diagnosis of eosinophilic pneumonia
is made. What is the best treatment apart from More
• My Performance
removing the causal factors?
• Media Bank
• New Multimedia Clarithromycin
Reference: Normal Values
Online Extras Systemic steroids
Leukotriene-receptor antagonists Haematology
Library
Community Inhaled β2 -agonists Immunoglobulins
Help Nebulised β2 -agonists Biochemistry
PasTest Store Diabetes
495
Endocrinology
My Account
Blood gases
Profile
CSF
Newsletters
My Career and Exams
Order History
Learning Goals
Question Filters

Security

Change Password
Sign Out

© 2011 PasTest Ltd | About Us | Contact Us | Help

file:///E|/Shakhawan/Respiratory%20S/105.htm[3/13/2012 4:00:57 PM]


MyPasTest

Main Navigation
Question Browser: MRCP 1
Home
Subscriptions Question Browser Exam Builder Saved Exams
» MRCP 1 Session Progress
• Question Browser
Questions Correct 2
• Timed Test A 50-year-old patient presents with blood eosinophilia in
association with a radiographic pulmonary infiltrate. A Questions Incorrect 0
• Mock Exam
bronchoscopy shows an excess of eosinophils in Questions Total 2
• Past Papers bronchoalveolar lavage fluid in the absence of pathogenic
Questions Percentage 100 %
• Random Questions micro-organisms. The diagnosis of eosinophilic pneumonia
is made. What is the best treatment apart from More
• My Performance
removing the causal factors?
• Media Bank
• New Multimedia Clarithromycin
Online Extras Systemic steroids Your answer
Library Leukotriene-receptor antagonists
Community Inhaled β2 -agonists
Help Nebulised β2 -agonists
PasTest Store

My Account

Profile Eosinophilic pneumonia often responds well to


corticosteroid medication, though treatment may need
Newsletters to be prolonged (6 months or more) in patients with the
My Career and Exams chronic forms of the disorder. The importance of
identifying whether it is associated with the causal
Order History Reference: Normal Values
factors listed below lies with the additional need to also
Learning Goals treat these. Otherwise eosinophilic pneumonia may not Haematology
respond adequately to steroid therapy and the
Question Filters
associated diseases may produce other Immunoglobulins
manifestations.Ask yourself whether there are any
causal factors: Biochemistry
Security
Diabetes
Change Password Is there parasitic infestation?
Sign Out Endocrinology
Have any drugs been administered? Blood gases

Is there asthma? CSF

Is there evidence of allergy to parasites or


drugs?

Is there evidence of allergic bronchopulmonary


mycosis (particularly aspergillosis)?

Is there evidence of vasculitis?

Is there evidence of the hypereosinophilic


syndrome?

Is there evidence of other disorders known to


be associated with eosinophilic pneumonia?

495

file:///E|/Shakhawan/Respiratory%20S/105a.htm[3/13/2012 4:00:59 PM]


MyPasTest

Main Navigation
Question Browser: MRCP 1
Home
Subscriptions Question Browser Exam Builder Saved Exams
» MRCP 1 Session Progress
• Question Browser
Questions Correct 2
• Timed Test A 62-year-old housewife has been complaining of a 2-
month history of lethargy associated with shortness of Questions Incorrect 0
• Mock Exam
breath. She has never smoked and takes no medication. Questions Total 2
• Past Papers The chest X-ray shows multiple round lesions increasing in
Questions Percentage 100 %
• Random Questions size and numbers at the base. There is no hilar
lymphadenopathy. What is the most likely diagnosis? More
• My Performance
• Media Bank Tuberculosis
• New Multimedia Pulmonary metastases Reference: Normal Values
Online Extras Silicosis
Haematology
Library Rib fractures
Community Lung abscesses Immunoglobulins
Help Biochemistry
496
PasTest Store Diabetes
Endocrinology
My Account
Blood gases
Profile
CSF
Newsletters
My Career and Exams
Order History
Learning Goals
Question Filters

Security

Change Password
Sign Out

© 2011 PasTest Ltd | About Us | Contact Us | Help

file:///E|/Shakhawan/Respiratory%20S/106.htm[3/13/2012 4:01:00 PM]


MyPasTest

Main Navigation
Question Browser: MRCP 1
Home
Subscriptions Question Browser Exam Builder Saved Exams
» MRCP 1 Session Progress
• Question Browser
Questions Correct 2
• Timed Test A 62-year-old housewife has been complaining of a 2-
month history of lethargy associated with shortness of Questions Incorrect 1
• Mock Exam
breath. She has never smoked and takes no medication. Questions Total 3
• Past Papers The chest X-ray shows multiple round lesions increasing in
Questions Percentage 66 %
• Random Questions size and numbers at the base. There is no hilar
lymphadenopathy. What is the most likely diagnosis? More
• My Performance
• Media Bank Tuberculosis Your answer
• New Multimedia Pulmonary metastases Correct answer
Online Extras Silicosis
Library Rib fractures
Community Lung abscesses
Help
PasTest Store

My Account Multiple metastases range enormously in size and


number, from ‘cannon balls’ to miliary shadowing, and
Profile may be accompanied by hilar lymphadenopathy or
pleural effusion. Breast, colon, renal and lung primaries
Newsletters
are probably the commonest underlying tumours, but
My Career and Exams other tumours are amenable to chemotherapy, such as
Order History testicular cancer and choriocarcinoma. Diagnosis may Reference: Normal Values
be achieved by cytology or histology on various samples
Learning Goals from the pleura or lung, and can occasionally be made Haematology
Question Filters from cytology on expectoration or induced sputum.
Lymphangitis carcinomatosa is most commonly due to Immunoglobulins
breast and primary lung tumours (usually
Biochemistry
Security adenocarcinomas). Patients can be asymptomatic when
the disease is first suspected on the basis of a chest X- Diabetes
Change Password ray showing diffusely increased interstitial markings
Sign Out accompanied by Kerley B lines, hilar lymphadenopathy Endocrinology
or pleural effusion. Diagnosis may be established by
cytology from sputum or pleural fluid, but often requires Blood gases
bronchoscopic or transbronchial lung biopsy. CSF

496

© 2011 PasTest Ltd | About Us | Contact Us | Help

file:///E|/Shakhawan/Respiratory%20S/106a.htm[3/13/2012 4:01:02 PM]


MyPasTest

Main Navigation
Question Browser: MRCP 1
Home
Subscriptions Question Browser Exam Builder Saved Exams
» MRCP 1 Session Progress
• Question Browser
Questions Correct 2
• Timed Test A 67-year-old retired man has been complaining of a 6-
month history of increasing shortness of breath, dull right- Questions Incorrect 1
• Mock Exam
sided chest pain, loss of appetite and night sweats. He Questions Total 3
• Past Papers worked all his life in the ship-building industry where he
Questions Percentage 66 %
• Random Questions was exposed to asbestos. He has never smoked in his life.
On examination he is slightly cyanosed but there is no More
• My Performance
clubbing. Examination of the lungs revealed dullness to
• Media Bank percussion and reduced air entry at the right base. What is
• New Multimedia the most likely diagnosis?
Reference: Normal Values
Online Extras COPD
Haematology
Library Bronchial carcinoma
Community Immunoglobulins
Recurrent pulmonary embolism
Help Malignant mesothelioma Biochemistry
PasTest Store Tuberculosis Diabetes

498 Endocrinology
My Account
Blood gases
Profile
CSF
Newsletters
My Career and Exams
Order History
Learning Goals
Question Filters

Security

Change Password
Sign Out

© 2011 PasTest Ltd | About Us | Contact Us | Help

file:///E|/Shakhawan/Respiratory%20S/107.htm[3/13/2012 4:01:03 PM]


MyPasTest

Main Navigation
Question Browser: MRCP 1
Home
Subscriptions Question Browser Exam Builder Saved Exams
» MRCP 1 Session Progress
• Question Browser
Questions Correct 3
• Timed Test A 67-year-old retired man has been complaining of a 6-
month history of increasing shortness of breath, dull right- Questions Incorrect 1
• Mock Exam
sided chest pain, loss of appetite and night sweats. He Questions Total 4
• Past Papers worked all his life in the ship-building industry where he
Questions Percentage 75 %
• Random Questions was exposed to asbestos. He has never smoked in his life.
On examination he is slightly cyanosed but there is no More
• My Performance
clubbing. Examination of the lungs revealed dullness to
• Media Bank percussion and reduced air entry at the right base. What is
• New Multimedia the most likely diagnosis?

Online Extras COPD


Library Bronchial carcinoma
Community Recurrent pulmonary embolism
Help Malignant mesothelioma Your answer
PasTest Store Tuberculosis

My Account

Profile
Malignant mesothelioma arising from the pleura was
Newsletters
first recognised in the 1950s, and during the 1960s
My Career and Exams much evidence accumulated indicating a strong link
Order History between the condition and exposure to asbestos. The Reference: Normal Values
risk is a function of the concentration of fibres and
Learning Goals duration of exposure. Exposure is greatest in those Haematology
Question Filters involved in mining or quarrying the material, and in
those who handle the raw fibres. Many workers Immunoglobulins
engaged in the ship-building industry during the 1940s
Biochemistry
Security and 1950s were exposed to asbestos, and more
recently it has been in widespread use in the building Diabetes
Change Password industry. It is rare for mesotheliomas to develop within
Sign Out 20 years of exposure and most patients were initially Endocrinology
exposed 30 or more years before clinical presentation.
In the United Kingdom, although the risk was first Blood gases
recognised in the 1960s, it was not until the mid-1970s CSF
that there was a significant reduction in exposure to
asbestos. The age of presentation is usually between 50
and 70, although the incidence is increasing in older
patients. There is a male predominance, reflecting the
greater likelihood of previous occupational exposure,
which should be sought with a careful lifetime
occupational history. Symptoms due to local disease are
mainly those of pain and breathlessness. Pain may be
pleuritic in nature, but is often a dull ache due to direct
involvement of the chest wall. Shortness of breath is
usually associated with a pleural effusion, although as
the tumour progresses it gradually encases the lung.
Systemic symptoms include tiredness, anorexia, weight
loss and fever, and occasionally drenching sweats.
Finger-clubbing has been recorded but is rare. Physical
findings in the chest are those of a pleural effusion, but
with advanced disease there is progressive reduction in
chest wall movement. Direct extension through the
chest wall can result in a palpable mass, and this may
develop at the site of previous biopsy.

498

file:///E|/Shakhawan/Respiratory%20S/107a.htm[3/13/2012 4:01:05 PM]


MyPasTest

Main Navigation
Question Browser: MRCP 1
Home
Subscriptions Question Browser Exam Builder Saved Exams
» MRCP 1 Session Progress
• Question Browser
Questions Correct 3
• Timed Test A 32-year-old patient with asthma has been stable with
inhaled salbutamol when required. Recently she had to use Questions Incorrect 1
• Mock Exam
her inhaler more frequently and also at night. What is the Questions Total 4
• Past Papers next step in her therapy?
Questions Percentage 75 %
• Random Questions
• My Performance Inhaled β2 -agonist at maximum dose More
• Media Bank Regular inhaled budesonide, inhaled salbutamol when
required
• New Multimedia
Addition of oral corticosteroids Reference: Normal Values
Online Extras
Addition of oral leukotriene-receptor antagonist Haematology
Library
Addition of oral theophylline Immunoglobulins
Community
Help 499 Biochemistry
PasTest Store Diabetes
Endocrinology
My Account
Blood gases
Profile
CSF
Newsletters
My Career and Exams
Order History
Learning Goals
Question Filters

Security

Change Password
Sign Out

© 2011 PasTest Ltd | About Us | Contact Us | Help

file:///E|/Shakhawan/Respiratory%20S/108.htm[3/13/2012 4:01:07 PM]


MyPasTest

Main Navigation
Question Browser: MRCP 1
Home
Subscriptions Question Browser Exam Builder Saved Exams
» MRCP 1 Session Progress
• Question Browser
Questions Correct 4
• Timed Test A 32-year-old patient with asthma has been stable with
inhaled salbutamol when required. Recently she had to use Questions Incorrect 1
• Mock Exam
her inhaler more frequently and also at night. What is the Questions Total 5
• Past Papers next step in her therapy?
Questions Percentage 80 %
• Random Questions
• My Performance Inhaled β2 -agonist at maximum dose More
• Media Bank Regular inhaled budesonide, inhaled Your answer
salbutamol when required
• New Multimedia
Addition of oral corticosteroids
Online Extras
Addition of oral leukotriene-receptor
Library
antagonist
Community
Addition of oral theophylline
Help
PasTest Store

My Account This patient needs step 2 in the management of chronic


asthma because she needs her β2 -agonist inhaler more
Profile
than once a day and also complains of night-time
Newsletters symptoms. Step 2 therapy consists of a regular
My Career and Exams standard-dose inhaled corticosteroid and an additional
inhaled short-acting β2 -agonist as required. Oral Reference: Normal Values
Order History
leukotriene-receptor antagonists and theophylline are
Learning Goals indicated in step 3 if the asthma is still not controlled. Haematology
Question Filters Oral corticosteroids should be added in step 5.
Immunoglobulins
Biochemistry
Security 499
Diabetes
Change Password
Sign Out Endocrinology

Blood gases

CSF

© 2011 PasTest Ltd | About Us | Contact Us | Help

file:///E|/Shakhawan/Respiratory%20S/108a.htm[3/13/2012 4:01:08 PM]


MyPasTest

Main Navigation
Question Browser: MRCP 1
Home
Subscriptions Question Browser Exam Builder Saved Exams
» MRCP 1 Session Progress
• Question Browser
Questions Correct 4
• Timed Test Carcinoma of the lung is diagnosed in a 66-year-old
smoker. Questions Incorrect 1
• Mock Exam
Questions Total 5
• Past Papers Which of the following techniques would be most
reliable in staging of intrathoracic lymph node Questions Percentage 80 %
• Random Questions
involvement? More
• My Performance
• Media Bank Pulmonary angiography
• New Multimedia Magnetic resonance imaging (MRI)
Reference: Normal Values
Online Extras Bronchoscopy
Haematology
Library Computed tomographic (CT) scanning
Community VQ scan Immunoglobulins
Help Biochemistry
500
PasTest Store Diabetes
Endocrinology
My Account
Blood gases
Profile
CSF
Newsletters
My Career and Exams
Order History
Learning Goals
Question Filters

Security

Change Password
Sign Out

© 2011 PasTest Ltd | About Us | Contact Us | Help

file:///E|/Shakhawan/Respiratory%20S/109.htm[3/13/2012 4:01:09 PM]


MyPasTest

Main Navigation
Question Browser: MRCP 1
Home
Subscriptions Question Browser Exam Builder Saved Exams
» MRCP 1 Session Progress
• Question Browser
Questions Correct 4
• Timed Test Carcinoma of the lung is diagnosed in a 66-year-old
smoker. Questions Incorrect 2
• Mock Exam
Questions Total 6
• Past Papers Which of the following techniques would be most
reliable in staging of intrathoracic lymph node Questions Percentage 66 %
• Random Questions
involvement? More
• My Performance
• Media Bank Pulmonary angiography
• New Multimedia Magnetic resonance imaging (MRI) Your answer
Online Extras Bronchoscopy
Library Computed tomographic (CT) Correct answer
Community scanning

Help VQ scan

PasTest Store

My Account
Computed tomographic (CT) scanning is recommended
Profile as a staging procedure for patients who are candidates
for potentially curative surgery to identify enlarged
Newsletters mediastinal lymph nodes in order to determine the best
My Career and Exams approach for biopsy or aspiration cytology of the
appropriate lymph node. Where available, PET scanning Reference: Normal Values
Order History
may be superior but there are a limited number of
Learning Goals scanners in the UK.CT scanning may be performed with Haematology
Question Filters or without contrast enhancement, depending on local
protocols and the ability to see the relevant mediastinal Immunoglobulins
structures without the need for contrast. Contrast
enhancement makes it easier for inexperienced Biochemistry
Security
observers to interpret the CT findings. Spiral CT Diabetes
Change Password scanning allows optimal contrast enhancement and
should be used wherever practical. Overall, preoperative Endocrinology
Sign Out
CT staging has been shown to overstage or understage
when compared with operative findings in 40% of Blood gases
patients. This means that patients should not be denied
CSF
surgery on equivocal CT findings, and histological or
cytological confirmation of intrathoracic lymph node
involvement is required for patients in whom the only
contraindication to potentially curative surgery is lymph
node enlargement.

500

© 2011 PasTest Ltd | About Us | Contact Us | Help

file:///E|/Shakhawan/Respiratory%20S/109a.htm[3/13/2012 4:01:11 PM]


MyPasTest

Main Navigation
Question Browser: MRCP 1
Home
Subscriptions Question Browser Exam Builder Saved Exams
» MRCP 1 Session Progress
• Question Browser
Questions Correct 4
• Timed Test A 24-year-old patient presents with anorexia, fever and hot
flushes. The chest X-ray shows a 4-cm, large, left upper Questions Incorrect 2
• Mock Exam
lobe cavity. Active tuberculosis is suspected. Questions Total 6
• Past Papers
Questions Percentage 66 %
• Random Questions What is the next appropriate step to confirm the
diagnosis? More
• My Performance
• Media Bank Computed tomographic (CT) scanning
• New Multimedia Mantoux test Reference: Normal Values
Online Extras Blood cultures
Haematology
Library Sputum sample
Community Serum inflammatory markers Immunoglobulins
Help Biochemistry
501
PasTest Store Diabetes
Endocrinology
My Account
Blood gases
Profile
CSF
Newsletters
My Career and Exams
Order History
Learning Goals
Question Filters

Security

Change Password
Sign Out

© 2011 PasTest Ltd | About Us | Contact Us | Help

file:///E|/Shakhawan/Respiratory%20S/110.htm[3/13/2012 4:01:12 PM]


MyPasTest

Main Navigation
Question Browser: MRCP 1
Home
Subscriptions Question Browser Exam Builder Saved Exams
» MRCP 1 Session Progress
• Question Browser
Questions Correct 5
• Timed Test A 24-year-old patient presents with anorexia, fever and hot
flushes. The chest X-ray shows a 4-cm, large, left upper Questions Incorrect 2
• Mock Exam
lobe cavity. Active tuberculosis is suspected. Questions Total 7
• Past Papers
Questions Percentage 71 %
• Random Questions What is the next appropriate step to confirm the
diagnosis? More
• My Performance
• Media Bank Computed tomographic (CT) scanning
• New Multimedia Mantoux test
Online Extras Blood cultures
Library Sputum sample Your answer
Community Serum inflammatory markers
Help
PasTest Store

My Account The laboratory diagnosis of pulmonary tuberculosis


relies on examination and culture of sputum or other
Profile respiratory tract specimens. The definitive diagnosis
requires the growth of Mycobacterium tuberculosis from
Newsletters
respiratory secretions, while a probable diagnosis can
My Career and Exams be based on typical clinical and chest X-ray findings
Order History with either sputum positive for acid-fast bacilli or other Reference: Normal Values
specimens, or typical histopathological findings on
Learning Goals biopsy material. The specificity of these latter Haematology
Question Filters approaches depends on the prevalence of disease due
to non-tuberculosis mycobacteria in the population. Immunoglobulins
Biochemistry
Security
501 Diabetes
Change Password
Sign Out Endocrinology

Blood gases

CSF

© 2011 PasTest Ltd | About Us | Contact Us | Help

file:///E|/Shakhawan/Respiratory%20S/110a.htm[3/13/2012 4:01:15 PM]


MyPasTest

Main Navigation
Question Browser: MRCP 1
Home
Subscriptions Question Browser Exam Builder Saved Exams
» MRCP 1 Session Progress
• Question Browser
Questions Correct 5
• Timed Test A 49-year-old woman has been admitted with haemoptysis
and epistaxis, the chest X-ray shows multiple rounded Questions Incorrect 2
• Mock Exam
lesions with alveolar shadowing. Laboratory parameters Questions Total 7
• Past Papers show a leucocytosis without eosinophilia but with
Questions Percentage 71 %
• Random Questions microhaematuria, proteinuria as well as antineutrophil
cytoplasmic antibodies (c-ANCA). Which drug treatment More
• My Performance
is the most appropriate?
• Media Bank
• New Multimedia Erythromycin
Reference: Normal Values
Online Extras Ampicillin
Cyclophosphamide in combination with corticosteroids Haematology
Library
Community Ciclosporin Immunoglobulins
Help Aciclovir Biochemistry
PasTest Store 502 Diabetes
Endocrinology
My Account
Blood gases
Profile
CSF
Newsletters
My Career and Exams
Order History
Learning Goals
Question Filters

Security

Change Password
Sign Out

© 2011 PasTest Ltd | About Us | Contact Us | Help

file:///E|/Shakhawan/Respiratory%20S/111.htm[3/13/2012 4:01:16 PM]


MyPasTest

Main Navigation
Question Browser: MRCP 1
Home
Subscriptions Question Browser Exam Builder Saved Exams
» MRCP 1 Session Progress
• Question Browser
Questions Correct 6
• Timed Test A 49-year-old woman has been admitted with haemoptysis
and epistaxis, the chest X-ray shows multiple rounded Questions Incorrect 2
• Mock Exam
lesions with alveolar shadowing. Laboratory parameters Questions Total 8
• Past Papers show a leucocytosis without eosinophilia but with
Questions Percentage 75 %
• Random Questions microhaematuria, proteinuria as well as antineutrophil
cytoplasmic antibodies (c-ANCA). Which drug treatment More
• My Performance
is the most appropriate?
• Media Bank
• New Multimedia Erythromycin
Online Extras Ampicillin
Library Cyclophosphamide in combination with Your answer
corticosteroids
Community
Ciclosporin
Help
Aciclovir
PasTest Store

My Account

Profile The combination of prednisolone and cyclophosphamide


is now established as the standard induction therapy for
Newsletters
patients with generalised Wegener’s granulomatosis or
My Career and Exams microscopic polyangiitis. There is consensus on how
Order History corticosteroids should be used, but less so for Reference: Normal Values
cyclophosphamide. Prednisolone is given in doses of
Learning Goals around 1 mg/kg per day initially, after which the dose is Haematology
Question Filters reduced rapidly, typically at weekly intervals. Controlled
trials show that the addition of pulses of Immunoglobulins
methylprednisolone is unlikely to confer additional
Biochemistry
Security benefit. Traditionally, patients received daily oral
cyclophosphamide (2 mg/kg per day), but latterly Diabetes
Change Password intravenous boluses have proved increasingly popular,
Sign Out given in doses of 0.5–0.75 g/m 2 body surface area at Endocrinology
intervals of 2 weeks (at least for short periods) to 2 Blood gases
months.
CSF

502

© 2011 PasTest Ltd | About Us | Contact Us | Help

file:///E|/Shakhawan/Respiratory%20S/111a.htm[3/13/2012 4:01:17 PM]


MyPasTest

Main Navigation
Question Browser: MRCP 1
Home
Subscriptions Question Browser Exam Builder Saved Exams
» MRCP 1 Session Progress
• Question Browser
Questions Correct 6
• Timed Test A 65-year-old man complains of lethargy, fever, dry cough,
headache, chest pain and increasing shortness of breath. Questions Incorrect 2
• Mock Exam
He returned from a cruise 2 days ago. His chest X-ray Questions Total 8
• Past Papers shows bilateral infiltrates, the p(O2 ) is 8.35 kPa. What is
Questions Percentage 75 %
• Random Questions the most likely diagnosis?
• My Performance More
Tuberculosis
• Media Bank
Pulmonary embolism
• New Multimedia
Small-cell carcinoma of the lung Reference: Normal Values
Online Extras
Sarcoidosis Haematology
Library
Legionella pneumonia Immunoglobulins
Community
Help 503 Biochemistry
PasTest Store Diabetes
Endocrinology
My Account
Blood gases
Profile
CSF
Newsletters
My Career and Exams
Order History
Learning Goals
Question Filters

Security

Change Password
Sign Out

© 2011 PasTest Ltd | About Us | Contact Us | Help

file:///E|/Shakhawan/Respiratory%20S/112.htm[3/13/2012 4:01:19 PM]


MyPasTest

Main Navigation
Question Browser: MRCP 1
Home
Subscriptions Question Browser Exam Builder Saved Exams
» MRCP 1 Session Progress
• Question Browser
Questions Correct 6
• Timed Test A 65-year-old man complains of lethargy, fever, dry cough,
headache, chest pain and increasing shortness of breath. Questions Incorrect 3
• Mock Exam
He returned from a cruise 2 days ago. His chest X-ray Questions Total 9
• Past Papers shows bilateral infiltrates, the p(O2 ) is 8.35 kPa. What is
Questions Percentage 66 %
• Random Questions the most likely diagnosis?
• My Performance More
Tuberculosis
• Media Bank
Pulmonary embolism Your answer
• New Multimedia
Small-cell carcinoma of the lung
Online Extras
Sarcoidosis
Library
Legionella pneumonia Correct answer
Community
Help
PasTest Store

Legionella infection is the cause of around 2–5% of


My Account cases of community-acquired pneumonia admitted to
hospital, although there is wider geographical and
Profile seasonal variation. Infection tends to lead to moderate
Newsletters or severe infection rather than mild illness, and most
patients require hospital admission within 5–7 days of
My Career and Exams the start of symptoms. The incubation period is usually
Order History 2–10 days, with a mean of 7 days; males are two to Reference: Normal Values
three times more frequently affected than females.
Learning Goals Haematology
Infection at the extremes of age is rare and the highest
Question Filters incidence is in 40- to 70-year-old people, with a mean
Immunoglobulins
age of 53 years. People particularly at risk include
cigarette smokers, alcoholics, diabetics and those with Biochemistry
Security a chronic illness or who are receiving corticosteroids or
immunosuppressive therapy. Consequently, the type of Diabetes
Change Password patient who requires admission to hospital is
Sign Out particularly at risk from a nosocomial source. Typically, Endocrinology
the illness starts fairly abruptly with high fever, shivers, Blood gases
bad headache and muscle pains. Upper respiratory tract
symptoms, herpes labialis and skin rashes are CSF
uncommon. The cough is usually dry initially, but
dyspnoea is common and the illness often progresses
quickly. Sometimes there may be a history of a recent
hotel holiday abroad or a stay in hospital, which can
alert the clinician to the possible diagnosis. The patient
commonly looks toxic and ill, with a high fever over 39
°C. Confusion and delirium or diarrhoea can dominate
the clinical picture, masking the true diagnosis of
pneumonia. Focal neurological signs, particularly of a
cerebellar type, are well described. Amnesia on
recovery is common.

503

© 2011 PasTest Ltd | About Us | Contact Us | Help

file:///E|/Shakhawan/Respiratory%20S/112a.htm[3/13/2012 4:01:20 PM]


MyPasTest

Main Navigation
Question Browser: MRCP 1
Home
Subscriptions Question Browser Exam Builder Saved Exams
» MRCP 1 Session Progress
• Question Browser
Questions Correct 6
• Timed Test A 65-year-old man complains of lethargy, fever, dry cough,
headache, chest pain and increasing shortness of breath. Questions Incorrect 3
• Mock Exam
He returned from a cruise 2 days ago. His chest X-ray Questions Total 9
• Past Papers shows bilateral infiltrates, the p(O2 ) is 8.35 kPa. What is
Questions Percentage 66 %
• Random Questions the most appropriate therapy?
• My Performance More
Intravenous corticosteroids
• Media Bank
Isoniazid
• New Multimedia
Ampicillin Reference: Normal Values
Online Extras
Amphotericin B Haematology
Library
Erythromycin Immunoglobulins
Community
Help 504 Biochemistry
PasTest Store Diabetes
Endocrinology
My Account
Blood gases
Profile
CSF
Newsletters
My Career and Exams
Order History
Learning Goals
Question Filters

Security

Change Password
Sign Out

© 2011 PasTest Ltd | About Us | Contact Us | Help

file:///E|/Shakhawan/Respiratory%20S/113.htm[3/13/2012 4:01:22 PM]


MyPasTest

Main Navigation
Question Browser: MRCP 1
Home
Subscriptions Question Browser Exam Builder Saved Exams
» MRCP 1 Session Progress
• Question Browser
Questions Correct 7
• Timed Test A 65-year-old man complains of lethargy, fever, dry cough,
headache, chest pain and increasing shortness of breath. Questions Incorrect 3
• Mock Exam
He returned from a cruise 2 days ago. His chest X-ray Questions Total 10
• Past Papers shows bilateral infiltrates, the p(O2 ) is 8.35 kPa. What is
Questions Percentage 70 %
• Random Questions the most appropriate therapy?
• My Performance More
Intravenous corticosteroids
• Media Bank
Isoniazid
• New Multimedia
Ampicillin
Online Extras
Amphotericin B
Library
Erythromycin Your answer
Community
Help
PasTest Store

This patient has Legionella pneumonia. The most


My Account relevant factor is the ability of the antibiotic to
penetrate intracellularly into alveolar macrophages
Profile where the legionella organism hides and divides. A
Newsletters macrolide, such as erythromycin or clarithromycin, is at
present recommended as the drug of first choice, in
My Career and Exams dosages of 500–1000 mg every 6 h for erythromycin
Order History and 500 mg twice daily for clarithromycin, being given Reference: Normal Values
intravenously if required. In vitro and animal
Learning Goals Haematology
experiments and clinical experience support the efficacy
Question Filters of rifampicin and fluoroquinolones. Rifampicin is often
Immunoglobulins
recommended as additional therapy to erythromycin, in
a dose of 600 mg once or twice daily in patients with Biochemistry
Security severe infection or who are deteriorating. General
supportive measures are particularly important, with Diabetes
Change Password attention to adequate hydration and correction of
Sign Out hypoxaemia with the early use of assisted ventilation for Endocrinology
advancing respiratory failure. Blood gases
Prevention: The most important principle to follow is to CSF
avoid holding water at temperatures between 20 and
45 °C, which is the range in which legionella
multiplication occurs. Other preventive measures should
be taken, ie: minimise colonisation, growth and release
of legionellae into the atmosphere; physically or
chemically treat water to kill the bacteria; protect
maintenance personnel who work on contaminated
systems.

504

© 2011 PasTest Ltd | About Us | Contact Us | Help

file:///E|/Shakhawan/Respiratory%20S/113a.htm[3/13/2012 4:01:23 PM]


MyPasTest

Main Navigation
Question Browser: MRCP 1
Home
Subscriptions Question Browser Exam Builder Saved Exams
» MRCP 1 Session Progress
• Question Browser
Questions Correct 7
• Timed Test A 55-year-old patient has been complaining of a 4-week
history of shortness of breath and cough with occasional Questions Incorrect 3
• Mock Exam
bloody phlegm. He has been smoking 10 cigarettes per day Questions Total 10
• Past Papers for 25 years. The clinical examination, chest X-ray and
Questions Percentage 70 %
• Random Questions laboratory parameters including arterial blood gases are
unremarkable. More
• My Performance
• Media Bank Which of the following investigations is most likely to
give the correct diagnosis?
• New Multimedia
Reference: Normal Values
Online Extras Sputum cytology
Haematology
Library Spirometry
Community CT thorax Immunoglobulins
Help Follow-up chest X-ray in 6 weeks Biochemistry
PasTest Store Ventilation/perfusion scan of the lung Diabetes

505 Endocrinology
My Account
Blood gases
Profile
CSF
Newsletters
My Career and Exams
Order History
Learning Goals
Question Filters

Security

Change Password
Sign Out

© 2011 PasTest Ltd | About Us | Contact Us | Help

file:///E|/Shakhawan/Respiratory%20S/114.htm[3/13/2012 4:01:25 PM]


MyPasTest

Main Navigation
Question Browser: MRCP 1
Home
Subscriptions Question Browser Exam Builder Saved Exams
» MRCP 1 Session Progress
• Question Browser
Questions Correct 8
• Timed Test A 55-year-old patient has been complaining of a 4-week
history of shortness of breath and cough with occasional Questions Incorrect 3
• Mock Exam
bloody phlegm. He has been smoking 10 cigarettes per day Questions Total 11
• Past Papers for 25 years. The clinical examination, chest X-ray and
Questions Percentage 72 %
• Random Questions laboratory parameters including arterial blood gases are
unremarkable. More
• My Performance
• Media Bank Which of the following investigations is most likely to
give the correct diagnosis?
• New Multimedia
Online Extras Sputum cytology
Library Spirometry
Community CT thorax Your answer
Help Follow-up chest X-ray in 6 weeks
PasTest Store Ventilation/perfusion scan of the lung

My Account

Profile
The finding of a normal chest X-ray does not exclude
Newsletters bronchial carcinoma, as patients presenting with
My Career and Exams haemoptysis and a normal chest X-ray are sometimes
found to have a central tumour on bronchoscopy. A Reference: Normal Values
Order History
computed tomographic (CT) scan is usually done first in
Learning Goals case the invasive investigation can be avoided. Haematology
Question Filters Spirometry and V/Q scanning have no diagnostic value.
Immunoglobulins
Biochemistry
Security 505
Diabetes
Change Password
Sign Out Endocrinology

Blood gases

CSF

© 2011 PasTest Ltd | About Us | Contact Us | Help

file:///E|/Shakhawan/Respiratory%20S/114a.htm[3/13/2012 4:01:26 PM]


MyPasTest

Main Navigation
Question Browser: MRCP 1
Home
Subscriptions Question Browser Exam Builder Saved Exams
» MRCP 1 Session Progress
• Question Browser
Questions Correct 8
• Timed Test A 30-year-old asylum seeker has been complaining of
cough, fever and weight loss. The chest X-ray shows a Questions Incorrect 3
• Mock Exam
large, upper lobe lesion, the sputum shows acid-fast bacilli Questions Total 11
• Past Papers that are confirmed as Mycobacterium tuberculosis by
Questions Percentage 72 %
• Random Questions polymerase chain reaction (PCR). Drug therapy with
isoniazid, rifampicin, ethambutol and pyrazinamide has More
• My Performance
been started under directly observed therapy (DOT). During
• Media Bank the next 4 weeks the disease is still progressing. What is
• New Multimedia the most likely reason?
Reference: Normal Values
Online Extras Infection with multi-resistant tuberculosis
Haematology
Library Infection with atypical mycobacteriae
Community Immunoglobulins
Underlying bacterial pneumonia
Help Carcinoma of the lung Biochemistry
PasTest Store Aspiration pneumonia Diabetes

506 Endocrinology
My Account
Blood gases
Profile
CSF
Newsletters
My Career and Exams
Order History
Learning Goals
Question Filters

Security

Change Password
Sign Out

© 2011 PasTest Ltd | About Us | Contact Us | Help

file:///E|/Shakhawan/Respiratory%20S/115.htm[3/13/2012 4:01:27 PM]


MyPasTest

Main Navigation
Question Browser: MRCP 1
Home
Subscriptions Question Browser Exam Builder Saved Exams
» MRCP 1 Session Progress
• Question Browser
Questions Correct 9
• Timed Test A 30-year-old asylum seeker has been complaining of
Questions Incorrect 3
• Mock Exam cough, fever and weight loss. The chest X-ray shows a
large, upper lobe lesion, the sputum shows acid-fast bacilli Questions Total 12
• Past Papers that are confirmed as Mycobacterium tuberculosis by Questions Percentage 75 %
• Random Questions polymerase chain reaction (PCR). Drug therapy with
isoniazid, rifampicin, ethambutol and pyrazinamide has More
• My Performance
been started under directly observed therapy (DOT). During
• Media Bank the next 4 weeks the disease is still progressing. What is
• New Multimedia the most likely reason?

Online Extras Infection with multi-resistant Your answer


Library tuberculosis
Community Infection with atypical mycobacteriae
Help Underlying bacterial pneumonia
PasTest Store Carcinoma of the lung
Aspiration pneumonia
My Account

Profile
Newsletters
Multidrug-resistant tuberculosis is defined as resistance
My Career and Exams to rifampicin and isoniazid with or without resistance to
Order History other anti-TB drugs. The treatment of patients with Reference: Normal Values
drug-resistant tuberculosis should only be carried out
Learning Goals by specialist physicians with appropriate experience in Haematology
Question Filters the management of such cases. Initial drug resistance is
uncommon (< 2%) in previously untreated White Immunoglobulins
patients born in the UK. Higher levels of resistance
Biochemistry
Security occur in ethnic minority groups, particularly those of the
Indian subcontinent and Black-African ethnic origin, Diabetes
Change Password with isoniazid resistance occurring in 4–6% of such
Sign Out patients. HIV-positivity, independent of ethnic group, is Endocrinology
also a marker for increased drug resistance: a positive
HIV result increases at least fourfold the chances of Blood gases
single- or multiple-drug resistance compared with an CSF
HIV-negative individual.

Treatment is complex, time-consuming and demanding


for both the patient and the physician. Such treatment
should only be carried out by physicians with substantial
experience in managing complex resistant cases, and
only in hospitals with appropriate isolation facilities. This
may require transfer of the patient to an appropriate
unit. Treatment of such patients has to be planned on
an individual basis and needs to include reserve drugs.
Such treatment must be closely monitored because of
increased toxicity, but, more importantly, full
compliance is essential to prevent the emergence of
further drug resistance. Therefore all such treatment
must be directly observed throughout, both on an in-
patient and an out-patient basis. Treatment should start
with five or more drugs to which the organism is, or is
likely to be, susceptible and continue until sputum
cultures become negative. Drug treatment then has to
be continued with at least three drugs to which the
organism is susceptible on in vitro testing for a
minimum of a further 9 months and perhaps up to or
beyond 24 months, depending on the in vitro drug-
resistance profile, the available drugs and the patient’s
HIV status. Consideration may also have to be given to

file:///E|/Shakhawan/Respiratory%20S/115a.htm[3/13/2012 4:01:29 PM]


MyPasTest

resection of pulmonary lesions under drug cover.

506

© 2011 PasTest Ltd | About Us | Contact Us | Help

file:///E|/Shakhawan/Respiratory%20S/115a.htm[3/13/2012 4:01:29 PM]


MyPasTest

Main Navigation
Question Browser: MRCP 1
Home
Subscriptions Question Browser Exam Builder Saved Exams
» MRCP 1 Session Progress
• Question Browser
Questions Correct 9
• Timed Test After a tennis match, a 20-year-old thin woman complains
of left-sided chest pain that radiates into her abdomen. The Questions Incorrect 3
• Mock Exam
physical examination reveals reduced air entry at the right Questions Total 12
• Past Papers base of the lung with hyper-resonant percussion. The
Questions Percentage 75 %
• Random Questions abdominal examination shows generalised tenderness. A
few minutes later she develops cyanosis. What is the More
• My Performance
diagnosis?
• Media Bank
• New Multimedia Myocardial infarction
Reference: Normal Values
Online Extras Tension pneumothorax
Ectopic pregnancy Haematology
Library
Community Pulmonary embolism Immunoglobulins
Help Acute pancreatitis Biochemistry
PasTest Store 507 Diabetes
Endocrinology
My Account
Blood gases
Profile
CSF
Newsletters
My Career and Exams
Order History
Learning Goals
Question Filters

Security

Change Password
Sign Out

© 2011 PasTest Ltd | About Us | Contact Us | Help

file:///E|/Shakhawan/Respiratory%20S/116.htm[3/13/2012 4:01:31 PM]


MyPasTest

Main Navigation
Question Browser: MRCP 1
Home
Subscriptions Question Browser Exam Builder Saved Exams
» MRCP 1 Session Progress
• Question Browser
Questions Correct 10
• Timed Test After a tennis match, a 20-year-old thin woman complains
of left-sided chest pain that radiates into her abdomen. The Questions Incorrect 3
• Mock Exam
physical examination reveals reduced air entry at the right Questions Total 13
• Past Papers base of the lung with hyper-resonant percussion. The
Questions Percentage 76 %
• Random Questions abdominal examination shows generalised tenderness. A
few minutes later she develops cyanosis. What is the More
• My Performance
diagnosis?
• Media Bank
• New Multimedia Myocardial infarction
Online Extras Tension pneumothorax Your answer
Library Ectopic pregnancy
Community Pulmonary embolism
Help Acute pancreatitis
PasTest Store

My Account
If a tension pneumothorax is present, a cannula of
Profile adequate length should be promptly inserted into the
second intercostal space in the mid-clavicular line and
Newsletters
left in place until a functioning intercostal tube can be
My Career and Exams positioned. Tension pneumothorax occurs when the
Order History intrapleural pressure exceeds the atmospheric pressure Reference: Normal Values
throughout inspiration as well as expiration. It is
Learning Goals thought to result from the operation of a one-way valve Haematology
Question Filters system, drawing air into the pleural space during
inspiration and not allowing it out during expiration. The Immunoglobulins
development of tension pneumothorax is often, but not
Biochemistry
Security always, heralded by a sudden deterioration in the
cardiopulmonary status of the patient related to Diabetes
Change Password impaired venous return, reduced cardiac output and
Sign Out hypoxaemia. The development of tension in a Endocrinology
pneumothorax is not dependent on the size of the
pneumothorax, and the clinical scenario of tension Blood gases
pneumothorax may correlate poorly with chest X-ray CSF
findings. The clinical status is striking. The patient
rapidly becomes distressed with rapid laboured
respiration, cyanosis, sweating and tachycardia. It
should be particularly suspected in those on mechanical
ventilators or nasal non-invasive ventilation who
suddenly deteriorate or develop EMD arrest, and is
frequently missed in the ICU setting. If a tension
pneumothorax occurs, the patient should be given high-
concentration oxygen and a cannula should be
introduced into the pleural space, usually in the second
anterior intercostal space mid-clavicular line. Air should
be removed until the patient is no longer compromised
and then an intercostal tube should be inserted into the
pleural space as previously described. Advanced
Trauma Life Support guidelines recommend the use of a
3–6 cm long cannula to perform needle thoracocentesis
for life-threatening tension pneumothorax. However, in
57% of patients with tension pneumothorax the
thickness of the chest wall has been found to be greater
than 3 cm. It is therefore recommended that a cannula
length of at least 4.5 cm should be used in needle
thoracocentesis of tension pneumothoraces. The
cannula should be left in place until bubbling is
confirmed in the underwater-seal system to confirm

file:///E|/Shakhawan/Respiratory%20S/116a.htm[3/13/2012 4:01:32 PM]


MyPasTest

proper function of the intercostal tube.

507

© 2011 PasTest Ltd | About Us | Contact Us | Help

file:///E|/Shakhawan/Respiratory%20S/116a.htm[3/13/2012 4:01:32 PM]


MyPasTest

Main Navigation
Question Browser: MRCP 1
Home
Subscriptions Question Browser Exam Builder Saved Exams
» MRCP 1 Session Progress
• Question Browser
Questions Correct 10
• Timed Test A patient presents with symptoms suggesting
bronchiectasis and abdominal distension, bloating and foul- Questions Incorrect 3
• Mock Exam
smelling faeces. What is the most likely diagnosis? Questions Total 13
• Past Papers
Questions Percentage 76 %
• Random Questions Carcinoma of the lung
• My Performance Wegener’s granulomatosis More
• Media Bank Pneumococcus pneumonia
• New Multimedia Goodpasture’s syndrome
Reference: Normal Values
Online Extras Cystic fibrosis
Haematology
Library
508
Community Immunoglobulins
Help Biochemistry
PasTest Store Diabetes
Endocrinology
My Account
Blood gases
Profile
CSF
Newsletters
My Career and Exams
Order History
Learning Goals
Question Filters

Security

Change Password
Sign Out

© 2011 PasTest Ltd | About Us | Contact Us | Help

file:///E|/Shakhawan/Respiratory%20S/117.htm[3/13/2012 4:01:33 PM]


MyPasTest

Main Navigation
Question Browser: MRCP 1
Home
Subscriptions Question Browser Exam Builder Saved Exams
» MRCP 1 Session Progress
• Question Browser
Questions Correct 11
• Timed Test A patient presents with symptoms suggesting
bronchiectasis and abdominal distension, bloating and foul- Questions Incorrect 3
• Mock Exam
smelling faeces. What is the most likely diagnosis? Questions Total 14
• Past Papers
Questions Percentage 78 %
• Random Questions Carcinoma of the lung
• My Performance Wegener’s granulomatosis More
• Media Bank Pneumococcus pneumonia
• New Multimedia Goodpasture’s syndrome
Online Extras Cystic fibrosis Your answer
Library
Community
Help
PasTest Store The United States Cystic Fibrosis Foundation Registry
data show that as many as 10% of cystic fibrosis
patients are not diagnosed until adult life. The main
My Account presentation is with respiratory problems, usually
recurrent lower respiratory infections with chronic
Profile sputum production. Some patients are given a prior
diagnosis of bronchiectasis, atypical asthma, nasal
Newsletters
polyposis or allergic bronchopulmonary aspergillosis. A
My Career and Exams new diagnosis of cystic fibrosis has been described even
Order History in adults in their seventh decade. Depletion of sodium, Reference: Normal Values
chloride and potassium due to excessive sweating, and
Learning Goals secondary renal chloride retention, may result in Haematology
Question Filters presentation with dehydration and heat exhaustion in an
otherwise apparently completely fit adult. Pancreatic Immunoglobulins
insufficiency can lead to steatorrhoea. The vast
Biochemistry
Security majority of patients with cystic fibrosis can be
diagnosed by a sweat test. Diabetes
Change Password
Sign Out Endocrinology
508
Blood gases

CSF

© 2011 PasTest Ltd | About Us | Contact Us | Help

file:///E|/Shakhawan/Respiratory%20S/117a.htm[3/13/2012 4:01:35 PM]


MyPasTest

Main Navigation
Question Browser: MRCP 1
Home
Subscriptions Question Browser Exam Builder Saved Exams
» MRCP 1 Session Progress
• Question Browser
Questions Correct 11
• Timed Test A 36-year-old lorry driver who smokes heavily has been
complaining of a 2-day history of cough associated with Questions Incorrect 3
• Mock Exam
fever. He also complains of right-sided chest pain on Questions Total 14
• Past Papers inspiration. On examination he is slightly cyanosed, has a
Questions Percentage 78 %
• Random Questions temperature of 38 °C, a respiratory rate of 38/min, a BP of
100/70 mm/Hg and a pulse rate of 130/min. He has basal More
• My Performance
crepitations and dullness to percussion at the right lung
• Media Bank base. What is the most likely diagnosis?
• New Multimedia
Atelectasis due to carcinoma of the lung Reference: Normal Values
Online Extras
Bronchopneumonia Haematology
Library
Mesothelioma
Community Immunoglobulins
Tuberculosis
Help Biochemistry
Pneumothorax
PasTest Store Diabetes
509
Endocrinology
My Account
Blood gases
Profile
CSF
Newsletters
My Career and Exams
Order History
Learning Goals
Question Filters

Security

Change Password
Sign Out

© 2011 PasTest Ltd | About Us | Contact Us | Help

file:///E|/Shakhawan/Respiratory%20S/118.htm[3/13/2012 4:01:36 PM]


MyPasTest

Main Navigation
Question Browser: MRCP 1
Home
Subscriptions Question Browser Exam Builder Saved Exams
» MRCP 1 Session Progress
• Question Browser
Questions Correct 12
• Timed Test A 36-year-old lorry driver who smokes heavily has been
complaining of a 2-day history of cough associated with Questions Incorrect 3
• Mock Exam
fever. He also complains of right-sided chest pain on Questions Total 15
• Past Papers inspiration. On examination he is slightly cyanosed, has a
Questions Percentage 80 %
• Random Questions temperature of 38 °C, a respiratory rate of 38/min, a BP of
100/70 mm/Hg and a pulse rate of 130/min. He has basal More
• My Performance
crepitations and dullness to percussion at the right lung
• Media Bank base. What is the most likely diagnosis?
• New Multimedia
Atelectasis due to carcinoma of the
Online Extras lung
Library Bronchopneumonia Your answer
Community Mesothelioma
Help Tuberculosis
PasTest Store Pneumothorax

My Account

Profile
The classic presentation of pneumonia is of a cough and
Newsletters
fever with the variable presence of sputum production,
My Career and Exams dyspnoea and pleurisy. Most patients have
Order History constitutional symptoms such as malaise, fatigue and Reference: Normal Values
asthenia, and many also have gastrointestinal
Learning Goals symptoms. Examination of the lung might reveal Haematology
Question Filters decreased vesicular breath sounds, localised foci of
crepitations, dullness to percussion and sometimes Immunoglobulins
bronchial wheeze. The chest X-ray is a pivotal test for
Biochemistry
Security the confirmation of pneumonia.
Diabetes
Change Password
Sign Out 509 Endocrinology

Blood gases

CSF

© 2011 PasTest Ltd | About Us | Contact Us | Help

file:///E|/Shakhawan/Respiratory%20S/118a.htm[3/13/2012 4:01:38 PM]


MyPasTest

Main Navigation
Question Browser: MRCP 1
Home
Subscriptions Question Browser Exam Builder Saved Exams
» MRCP 1 Session Progress
• Question Browser
Questions Correct 12
• Timed Test Which lung disease is associated with the clinical
observations ‘pink puffer’ and ‘blue bloater’? Questions Incorrect 3
• Mock Exam
Questions Total 15
• Past Papers
Cystic fibrosis Questions Percentage 80 %
• Random Questions
Pulmonary fibrosis
• My Performance More
COPD
• Media Bank
Small-cell lung cancer
• New Multimedia
Tuberculosis Reference: Normal Values
Online Extras
510 Haematology
Library
Community Immunoglobulins
Help Biochemistry
PasTest Store Diabetes
Endocrinology
My Account
Blood gases
Profile
CSF
Newsletters
My Career and Exams
Order History
Learning Goals
Question Filters

Security

Change Password
Sign Out

© 2011 PasTest Ltd | About Us | Contact Us | Help

file:///E|/Shakhawan/Respiratory%20S/119.htm[3/13/2012 4:01:39 PM]


MyPasTest

Main Navigation
Question Browser: MRCP 1
Home
Subscriptions Question Browser Exam Builder Saved Exams
» MRCP 1 Session Progress
• Question Browser
Questions Correct 13
• Timed Test Which lung disease is associated with the clinical
observations ‘pink puffer’ and ‘blue bloater’? Questions Incorrect 3
• Mock Exam
Questions Total 16
• Past Papers
Cystic fibrosis Questions Percentage 81 %
• Random Questions
Pulmonary fibrosis
• My Performance More
COPD Your answer
• Media Bank
Small-cell lung cancer
• New Multimedia
Tuberculosis
Online Extras
Library
Community
Help Pink puffers have a good respiratory drive.
PasTest Store
Features include: purse-lip breathing with intense
dyspnoea, patient is often thin and elderly, little
My Account sputum produced, oedema and overt heart failure are
rare complications.
Profile
Investigations: blood gases are near normal until pre-
Newsletters terminally there is very severe airways obstruction,
My Career and Exams total lung capacity is increased, there is a reduction in
transfer factor.
Order History Reference: Normal Values
Learning Goals Blue bloaters have a poor respiratory drive. Haematology
Question Filters
Features include: quite mild dyspnoea, patient often Immunoglobulins
obese, large volumes of sputum produced, infective
Biochemistry
Security exacerbations, patient often oedematous, may
develop cor pulmonale. Diabetes
Change Password
Investigations: blood gases – hypercapnia,
Sign Out Endocrinology
hypoxaemia, elevated plasma bicarbonate, severe
nocturnal hypoxaemia, airways obstruction may only Blood gases
be moderate, transfer factor approximately normal.
CSF

510

© 2011 PasTest Ltd | About Us | Contact Us | Help

file:///E|/Shakhawan/Respiratory%20S/119a.htm[3/13/2012 4:01:41 PM]


MyPasTest

Main Navigation
Question Browser: MRCP 1
Home
Subscriptions Question Browser Exam Builder Saved Exams
» MRCP 1 Session Progress
• Question Browser
Questions Correct 13
• Timed Test A 67-year-old patient with lung cancer complains of
difficulty breathing, coughing and swelling of his face, neck, Questions Incorrect 3
• Mock Exam
upper body and arms. Superior vena cava syndrome is Questions Total 16
• Past Papers diagnosed.
Questions Percentage 81 %
• Random Questions
Which of the following treatments is most likely to be More
• My Performance successful in giving early relief of symptoms?
• Media Bank
• New Multimedia Corticosteroids
Reference: Normal Values
Online Extras Radiotherapy
Surgery Haematology
Library
Community Chemotherapy Immunoglobulins
Help Antihypertensive drugs
Biochemistry
PasTest Store 511 Diabetes
Endocrinology
My Account
Blood gases
Profile
CSF
Newsletters
My Career and Exams
Order History
Learning Goals
Question Filters

Security

Change Password
Sign Out

© 2011 PasTest Ltd | About Us | Contact Us | Help

file:///E|/Shakhawan/Respiratory%20S/120.htm[3/13/2012 4:01:42 PM]


MyPasTest

Main Navigation
Question Browser: MRCP 1
Home
Subscriptions Question Browser Exam Builder Saved Exams
» MRCP 1 Session Progress
• Question Browser
Questions Correct 13
• Timed Test A 67-year-old patient with lung cancer complains of
difficulty breathing, coughing and swelling of his face, neck, Questions Incorrect 4
• Mock Exam
upper body and arms. Superior vena cava syndrome is Questions Total 17
• Past Papers diagnosed.
Questions Percentage 76 %
• Random Questions
Which of the following treatments is most likely to be More
• My Performance successful in giving early relief of symptoms?
• Media Bank
• New Multimedia Corticosteroids Your answer

Online Extras Radiotherapy Correct answer

Library Surgery

Community Chemotherapy

Help Antihypertensive drugs

PasTest Store

My Account
Superior vena cava syndrome (SVCS) is a collection of
Profile symptoms caused by the partial blockage of the vein
that carries blood from the head, neck, chest and arms
Newsletters to the heart. Symptoms that may indicate this
My Career and Exams syndrome include difficulty breathing, coughing and
swelling of the face, neck, upper body and arms. In rare Reference: Normal Values
Order History
instances, patients may complain of hoarseness, chest
Learning Goals pain, difficulty swallowing and coughing up blood. Haematology
Question Filters Physical signs of SVCS include swelling of the neck or
chest veins, collection of fluid in the face or arms and Immunoglobulins
rapid breathing. The syndrome of superior vena cava
obstruction is relieved in about 80% of sufferers, but Biochemistry
Security
usually requires a more conventional course of 5–10 Diabetes
Change Password fractions of radiotherapy. Pain from bone secondaries
can be relieved in more than 50% of sufferers by a Endocrinology
Sign Out
single fraction of 8 Gy, often given at the same time as
a clinic visit. Brain metastases generally respond poorly Blood gases
to radiotherapy. A 48-hour trial of dexamethasone, 4
CSF
mg orally four times daily, is recommended as initial
management in the case of brain metastases.

511

© 2011 PasTest Ltd | About Us | Contact Us | Help

file:///E|/Shakhawan/Respiratory%20S/120a.htm[3/13/2012 4:01:43 PM]


MyPasTest

Main Navigation
Question Browser: MRCP 1
Home
Subscriptions Question Browser Exam Builder Saved Exams
» MRCP 1 Session Progress
• Question Browser
Questions Correct 13
• Timed Test A 58-year-old smoker with chronic bronchitis was treated
with antibiotics for a right upper lobe bronchopneumonia by Questions Incorrect 4
• Mock Exam
his GP. After 6 weeks he was readmitted to hospital. The Questions Total 17
• Past Papers chest X-ray shows signs of a pneumonia in the same place.
Questions Percentage 76 %
• Random Questions What is the most likely reason?
• My Performance More
Candida pneumonia
• Media Bank
Immunodeficiency
• New Multimedia
Bronchial carcinoma with post-stenotic pneumonia Reference: Normal Values
Online Extras
Sarcoidosis Haematology
Library
Tuberculosis
Community Immunoglobulins
Help 512 Biochemistry
PasTest Store Diabetes
Endocrinology
My Account
Blood gases
Profile
CSF
Newsletters
My Career and Exams
Order History
Learning Goals
Question Filters

Security

Change Password
Sign Out

© 2011 PasTest Ltd | About Us | Contact Us | Help

file:///E|/Shakhawan/Respiratory%20S/121.htm[3/13/2012 4:01:45 PM]


MyPasTest

Main Navigation
Question Browser: MRCP 1
Home
Subscriptions Question Browser Exam Builder Saved Exams
» MRCP 1 Session Progress
• Question Browser
Questions Correct 14
• Timed Test A 58-year-old smoker with chronic bronchitis was treated
with antibiotics for a right upper lobe bronchopneumonia by Questions Incorrect 4
• Mock Exam
his GP. After 6 weeks he was readmitted to hospital. The Questions Total 18
• Past Papers chest X-ray shows signs of a pneumonia in the same place.
Questions Percentage 77 %
• Random Questions What is the most likely reason?
• My Performance More
Candida pneumonia
• Media Bank
Immunodeficiency
• New Multimedia
Bronchial carcinoma with post-stenotic Your answer
Online Extras pneumonia
Library Sarcoidosis
Community Tuberculosis
Help
PasTest Store

My Account Non-resolving pneumonia is an indication of


bronchogenic carcinoma. An ill-defined homogeneous or
Profile patchy consolidation in a segmental or non-segmental
distribution may be an indication of bronchogenic
Newsletters
carcinoma. Patients with these findings are often
My Career and Exams initially treated for pneumonia; the lack of response to
Order History antibiotic therapy suggests the diagnosis of a Reference: Normal Values
malignancy. The opacity may contain air bronchograms
Learning Goals and air alveolograms. This presentation is often seen Haematology
Question Filters with adenocarcinoma and bronchoalveolar carcinoma.
Bronchial stenosis and post-stenotic changes are Immunoglobulins
commonly seen because most non–small-cell
Biochemistry
Security carcinomas demonstrate intraluminal growth. Narrowing
of the main bronchi or a complete cut-off can be Diabetes
Change Password identified on chest X-rays. An endobronchial lesion
Sign Out commonly leads to partial or complete atelectasis and is Endocrinology
the commonest sign of bronchogenic carcinoma.
Complete endobronchial obstruction can sometimes Blood gases
produce distal mucoid impaction, which may be visible CSF
on plain chest X-rays as a tubular or branching opacity.

512

© 2011 PasTest Ltd | About Us | Contact Us | Help

file:///E|/Shakhawan/Respiratory%20S/121a.htm[3/13/2012 4:01:46 PM]


MyPasTest

Main Navigation
Question Browser: MRCP 1
Home
Subscriptions Question Browser Exam Builder Saved Exams
» MRCP 1 Session Progress
• Question Browser
Questions Correct 14
• Timed Test A 43-year-old patient with rheumatoid arthritis has been
referred to you because of increasing shortness of breath Questions Incorrect 4
• Mock Exam
and dry cough. Which medication is most likely to be Questions Total 18
• Past Papers responsible for her symptoms?
Questions Percentage 77 %
• Random Questions
• My Performance Gold More
• Media Bank Oral corticosteroids

• New Multimedia Methotrexate


Reference: Normal Values
Online Extras Sulfasalazine
Ibuprofen Haematology
Library
Community Immunoglobulins
513
Help Biochemistry
PasTest Store Diabetes
Endocrinology
My Account
Blood gases
Profile
CSF
Newsletters
My Career and Exams
Order History
Learning Goals
Question Filters

Security

Change Password
Sign Out

© 2011 PasTest Ltd | About Us | Contact Us | Help

file:///E|/Shakhawan/Respiratory%20S/122.htm[3/13/2012 4:01:48 PM]


MyPasTest

Main Navigation
Question Browser: MRCP 1
Home
Subscriptions Question Browser Exam Builder Saved Exams
» MRCP 1 Session Progress
• Question Browser
Questions Correct 15
• Timed Test A 43-year-old patient with rheumatoid arthritis has been
referred to you because of increasing shortness of breath Questions Incorrect 4
• Mock Exam
and dry cough. Which medication is most likely to be Questions Total 19
• Past Papers responsible for her symptoms?
Questions Percentage 78 %
• Random Questions
• My Performance Gold More
• Media Bank Oral corticosteroids

• New Multimedia Methotrexate Your answer

Online Extras Sulfasalazine

Library Ibuprofen

Community
Help
PasTest Store
Methotrexate is an effective and moderately toxic
disease-modifying anti-rheumatoid drug (DMARD).
My Account Pulmonary complications, in the form of pneumonitis
(inflammation of the lung), are rare idiosyncratic
Profile reactions and are potentially lethal. The classical
presentation is with rapid onset dyspnoea (shortness of
Newsletters
breath) which may result in death after a few days.
My Career and Exams Therefore, patients should be advised to stop
Order History methotrexate if they experience dyspnoea or cough and Reference: Normal Values
to seek immediate medical attention. Penicillamine,
Learning Goals another DMARD, may also cause respiratory Haematology
Question Filters complications, mainly bronchiolitis obliterans. A review
of the literature suggests than auranofin (oral gold), Immunoglobulins
hydroxychloroquine and chloroquine are less toxic but
Biochemistry
Security also less efficacious than sulphasalazine, methotrexate,
parenteral gold, cyclosporin and penicillamine. At the Diabetes
Change Password other end of the spectrum, agents such as chlorambucil
Sign Out and cyclophosphamide are the most toxic and are best Endocrinology
reserved to treat patients with life-threatening
manifestations such as vasculitis. Like methotrexate, Blood gases
both sulphasalazine and gold may produce CSF
haematological abnormalities and monitoring of
haematological parameters is required.

513

© 2011 PasTest Ltd | About Us | Contact Us | Help

file:///E|/Shakhawan/Respiratory%20S/122a.htm[3/13/2012 4:01:50 PM]


MyPasTest

Main Navigation
Question Browser: MRCP 1
Home
Subscriptions Question Browser Exam Builder Saved Exams
» MRCP 1 Session Progress
• Question Browser
Questions Correct 15
• Timed Test Which of the following interventions is most likely to
Questions Incorrect 4
• Mock Exam slow deterioration in the FEV 1 and improve the
prognosis of a cigarette smoker with spirometric Questions Total 19
• Past Papers
evidence of moderate COPD? Questions Percentage 78 %
• Random Questions
• My Performance Tobacco-smoking cessation More
• Media Bank Frequent use of antibiotics for acute bacterial
exacerbations of chronic bronchitis
• New Multimedia
Daily use of salmeterol to improve FEV1 Reference: Normal Values
Online Extras
Long-term oxygen therapy at 2 l/min Haematology
Library
Community Daily use of ipratropium bromide to improve FEV1 Immunoglobulins
Help Biochemistry
514
PasTest Store Diabetes
Endocrinology
My Account
Blood gases
Profile
CSF
Newsletters
My Career and Exams
Order History
Learning Goals
Question Filters

Security

Change Password
Sign Out

© 2011 PasTest Ltd | About Us | Contact Us | Help

file:///E|/Shakhawan/Respiratory%20S/123.htm[3/13/2012 4:01:51 PM]


MyPasTest

Main Navigation
Question Browser: MRCP 1
Home
Subscriptions Question Browser Exam Builder Saved Exams
» MRCP 1 Session Progress
• Question Browser
Questions Correct 16
• Timed Test Which of the following interventions is most likely to
Questions Incorrect 4
• Mock Exam slow deterioration in the FEV 1 and improve the
prognosis of a cigarette smoker with spirometric Questions Total 20
• Past Papers
evidence of moderate COPD? Questions Percentage 80 %
• Random Questions
• My Performance Tobacco-smoking cessation Your answer More
• Media Bank Frequent use of antibiotics for acute
bacterial exacerbations of chronic
• New Multimedia bronchitis
Online Extras Daily use of salmeterol to improve
Library FEV1
Community Long-term oxygen therapy at 2 l/min
Help Daily use of ipratropium bromide to
PasTest Store improve FEV1

My Account

Profile
There is no treatment that has been shown to
Newsletters significantly slow or alter the progression of COPD
My Career and Exams except smoking cessation. The single best thing patients
with COPD can do for themselves is to stop smoking. All
Order History other treatments are largely for symptom relief. Reference: Normal Values
Learning Goals Haematology
Chronic obstructive pulmonary disease is a common
Question Filters chronic respiratory illness. It is steadily progressive and Immunoglobulins
carries a large burden of mortality and morbidity. It
causes about 30,500 deaths per year in the UK. The Biochemistry
Security morbidity burden this raises for the NHS is huge. Of all
emergency medical admissions, 25% are due to acute Diabetes
Change Password
respiratory infections and COPD exacerbations comprise Endocrinology
Sign Out over half this total. The cost in hospital care for these
patients is about £500 million. Housing changes and Blood gases
alterations add to the social costs of this disease. COPD
is caused by tobacco smoking, heavy exposure to CSF
occupational dusts and chemicals, air pollution,
cannabis smoking (which is now being recognised) and
a 1 -antitrypsin deficiency.

514

© 2011 PasTest Ltd | About Us | Contact Us | Help

file:///E|/Shakhawan/Respiratory%20S/123a.htm[3/13/2012 4:01:52 PM]


MyPasTest

Main Navigation
Question Browser: MRCP 1
Home
Subscriptions Question Browser Exam Builder Saved Exams
» MRCP 1 Session Progress
• Question Browser
Questions Correct 16
• Timed Test A 60-year-old man presents with a history of cough and
weight loss. He has smoked 40 cigarettes per day since he Questions Incorrect 4
• Mock Exam
was 17 years old. He describes recent darkening of his Questions Total 20
• Past Papers skin, and the chest X-ray reveals a suspicious mass for
Questions Percentage 80 %
• Random Questions lung cancer at the left hilum. What is the most likely
histology? More
• My Performance
• Media Bank Adenocarcinoma
• New Multimedia Small-cell carcinoma Reference: Normal Values
Online Extras Squamous-cell carcinoma
Haematology
Library Mesothelioma
Community Large-cell carcinoma Immunoglobulins
Help Biochemistry
515
PasTest Store Diabetes
Endocrinology
My Account
Blood gases
Profile
CSF
Newsletters
My Career and Exams
Order History
Learning Goals
Question Filters

Security

Change Password
Sign Out

© 2011 PasTest Ltd | About Us | Contact Us | Help

file:///E|/Shakhawan/Respiratory%20S/124.htm[3/13/2012 4:01:54 PM]


MyPasTest

Main Navigation
Question Browser: MRCP 1
Home
Subscriptions Question Browser Exam Builder Saved Exams
» MRCP 1 Session Progress
• Question Browser
Questions Correct 17
• Timed Test A 60-year-old man presents with a history of cough and
weight loss. He has smoked 40 cigarettes per day since he Questions Incorrect 4
• Mock Exam
was 17 years old. He describes recent darkening of his Questions Total 21
• Past Papers skin, and the chest X-ray reveals a suspicious mass for
Questions Percentage 80 %
• Random Questions lung cancer at the left hilum. What is the most likely
histology? More
• My Performance
• Media Bank Adenocarcinoma
• New Multimedia Small-cell carcinoma Your answer
Online Extras Squamous-cell carcinoma
Library Mesothelioma
Community Large-cell carcinoma
Help
PasTest Store

My Account Endocrine symptoms are seen in patients with lung


cancer due to the syndrome of ectopic hormone
Profile secretion. Small-cell lung cancer and bronchial carcinoid
tumours are both associated with ectopic ACTH
Newsletters
secretion, whereas squamous-cell carcinomas are often
My Career and Exams associated with hypercalcaemia due to parathyroid
Order History hormone-related peptide secretion. In small cell lung Reference: Normal Values
cancer, around 5% of cases are thought to manifest
Learning Goals ectopic ACTH secretion. Adrenal metastatic spread Haematology
Question Filters should be excluded with appropriate imaging. Treatment
of the clinical or biochemical abnormalities associated Immunoglobulins
with endocrinopathies of non-endocrine origin is best
Biochemistry
Security directed at the primary disorder. In neoplastic disease,
this may involve surgical excision, radiotherapy or Diabetes
Change Password chemotherapy. If the cancer can’t be targeted,
Sign Out symptomatic treatment should be applied. Endocrinology

Blood gases
515 CSF

© 2011 PasTest Ltd | About Us | Contact Us | Help

file:///E|/Shakhawan/Respiratory%20S/124a.htm[3/13/2012 4:01:55 PM]


MyPasTest

Main Navigation
Question Browser: MRCP 1
Home
Subscriptions Question Browser Exam Builder Saved Exams
» MRCP 1 Session Progress
• Question Browser
Questions Correct 17
• Timed Test A 65-year-old man who has a 40-pack year history of
smoking is referred to the clinic with haemoptysis. He has Questions Incorrect 4
• Mock Exam
no other symptoms. His chest radiograph shows an ill- Questions Total 21
• Past Papers defined 2-cm opacity at the periphery of his left lower lobe.
Questions Percentage 80 %
• Random Questions After checking his routine blood tests and spirometry
in clinic, which test would you arrange next? More
• My Performance
• Media Bank Bronchoscopy
• New Multimedia CT scan of the chest Reference: Normal Values
Online Extras PET scan
Haematology
Library Repeat chest radiograph in 1 month from now
Community Sputum cytology Immunoglobulins
Help Biochemistry
948
PasTest Store Diabetes
Endocrinology
My Account
Blood gases
Profile
CSF
Newsletters
My Career and Exams
Order History
Learning Goals
Question Filters

Security

Change Password
Sign Out

© 2011 PasTest Ltd | About Us | Contact Us | Help

file:///E|/Shakhawan/Respiratory%20S/125.htm[3/13/2012 4:01:57 PM]


MyPasTest

Main Navigation
Question Browser: MRCP 1
Home
Subscriptions Question Browser Exam Builder Saved Exams
» MRCP 1 Session Progress
• Question Browser
Questions Correct 17
• Timed Test A 65-year-old man who has a 40-pack year history of
smoking is referred to the clinic with haemoptysis. He has Questions Incorrect 5
• Mock Exam
no other symptoms. His chest radiograph shows an ill- Questions Total 22
• Past Papers defined 2-cm opacity at the periphery of his left lower lobe.
Questions Percentage 77 %
• Random Questions After checking his routine blood tests and spirometry
in clinic, which test would you arrange next? More
• My Performance
• Media Bank Bronchoscopy
• New Multimedia CT scan of the chest Correct answer
Online Extras PET scan Your answer
Library Repeat chest radiograph in 1 month
Community from now
Help Sputum cytology
PasTest Store

My Account
Lung cancer is the most likely diagnosis in this man. A
Profile CT scan is the best test to perform initially to locate,
characterise and stage the lesion if it is cancer.
Newsletters
Histological confirmation would then be sought, with
My Career and Exams either a bronchoscopy if the lesion is proximal or a CT
Order History or ultrasound-guided biopsy if it is peripheral. A Reference: Normal Values
positron-emission tomography (PET) scan would
Learning Goals determine whether there are distant metastases and is Haematology
Question Filters performed after the CT. Sputum cytology can be used
to provide a histological diagnosis, but a tissue biopsy is Immunoglobulins
favoured by the pathologists. Pneumonia can cause
Biochemistry
Security haemoptysis and chest radiograph abnormalities, but it
can also be associated with a proximal carcinoma. You Diabetes
Change Password should have a low threshold for performing a CT scan
Sign Out initially in a high-risk patient. Endocrinology

Blood gases
948 CSF

© 2011 PasTest Ltd | About Us | Contact Us | Help

file:///E|/Shakhawan/Respiratory%20S/125a.htm[3/13/2012 4:01:58 PM]


MyPasTest

Main Navigation
Question Browser: MRCP 1
Home
Subscriptions Question Browser Exam Builder Saved Exams
» MRCP 1 Session Progress
• Question Browser
Questions Correct 17
• Timed Test A 23-year-old woman with a lifelong history of atopy, hay
Questions Incorrect 5
• Mock Exam fever and mild asthma attends her GP. Over the last 3
months she has been waking in the early morning coughing Questions Total 22
• Past Papers and wheezing, and it is slowly getting worse. She is taking Questions Percentage 77 %
• Random Questions inhaled salbutamol seven times a day and is also using
• My Performance inhaled steroids 400 μg twice a day. Her predicted peak More
flow rate is 500 l/min and it is now 350 l/min. What would
• Media Bank you advise her GP to do?
• New Multimedia
Call an ambulance and admit her to hospital Reference: Normal Values
Online Extras
Start an oral leukotriene-receptor antagonist, such as Haematology
Library
montelukast
Community Immunoglobulins
Start her on oral theophylline
Help Biochemistry
Start an inhaled long-acting b 2 -agonist
PasTest Store Diabetes
Lend her a nebuliser to use at home
Endocrinology
My Account 949
Blood gases
Profile
CSF
Newsletters
My Career and Exams
Order History
Learning Goals
Question Filters

Security

Change Password
Sign Out

© 2011 PasTest Ltd | About Us | Contact Us | Help

file:///E|/Shakhawan/Respiratory%20S/126.htm[3/13/2012 4:01:59 PM]


MyPasTest

Main Navigation
Question Browser: MRCP 1
Home
Subscriptions Question Browser Exam Builder Saved Exams
» MRCP 1 Session Progress
• Question Browser
Questions Correct 18
• Timed Test A 23-year-old woman with a lifelong history of atopy, hay
fever and mild asthma attends her GP. Over the last 3 Questions Incorrect 5
• Mock Exam
months she has been waking in the early morning coughing Questions Total 23
• Past Papers and wheezing, and it is slowly getting worse. She is taking
Questions Percentage 78 %
• Random Questions inhaled salbutamol seven times a day and is also using
• My Performance inhaled steroids 400 μg twice a day. Her predicted peak More
flow rate is 500 l/min and it is now 350 l/min. What would
• Media Bank you advise her GP to do?
• New Multimedia
Call an ambulance and admit her to
Online Extras
hospital
Library
Start an oral leukotriene-receptor
Community antagonist, such as montelukast
Help Start her on oral theophylline
PasTest Store Start an inhaled long-acting b 2 - Your answer
agonist
My Account Lend her a nebuliser to use at home

Profile
Newsletters
My Career and Exams
This is a woman with features of inadequately controlled
Order History asthma. She has no features of an acute or life- Reference: Normal Values
Learning Goals threatening attack, so does not require hospital
admission. She is already taking a moderate dose of Haematology
Question Filters inhaled steroids, but needs add-on therapy. The British Immunoglobulins
Guidelines on the Management of Asthma (see Thorax
2003, 58(Suppl. 1)) recommend commencing a long- Biochemistry
Security
acting β 2 -agonist, such as salmeterol, if the patient is
Diabetes
Change Password regularly taking an inhaled steroid. If they do not
improve with this, or if additional therapy is still Endocrinology
Sign Out
required, oral theophylline or a leukotriene-receptor
antagonist can then be tried. A nebuliser is not Blood gases
recommended in place of increasing her other therapies.
CSF

949

© 2011 PasTest Ltd | About Us | Contact Us | Help

file:///E|/Shakhawan/Respiratory%20S/126a.htm[3/13/2012 4:02:01 PM]


MyPasTest

Main Navigation
Question Browser: MRCP 1
Home
Subscriptions Question Browser Exam Builder Saved Exams
» MRCP 1 Session Progress
• Question Browser
Questions Correct 18
• Timed Test A 65-year-old man, who has smoked 20 cigarettes a day
for 50 years and who has worked in the shipyard industry, Questions Incorrect 5
• Mock Exam
presents to clinic with chest pain and increasing shortness Questions Total 23
• Past Papers of breath. Clinical examination and chest radiograph show
Questions Percentage 78 %
• Random Questions he has a right pleural effusion. What investigation would
you perform next? More
• My Performance
• Media Bank Pleural aspiration
• New Multimedia Abrams’ pleural biopsy Reference: Normal Values
Online Extras Bronchoscopy
Haematology
Library Thoracoscopy
Community CT scan Immunoglobulins
Help Biochemistry
950
PasTest Store Diabetes
Endocrinology
My Account
Blood gases
Profile
CSF
Newsletters
My Career and Exams
Order History
Learning Goals
Question Filters

Security

Change Password
Sign Out

© 2011 PasTest Ltd | About Us | Contact Us | Help

file:///E|/Shakhawan/Respiratory%20S/127.htm[3/13/2012 4:02:03 PM]


MyPasTest

Main Navigation
Question Browser: MRCP 1
Home
Subscriptions Question Browser Exam Builder Saved Exams
» MRCP 1 Session Progress
• Question Browser
Questions Correct 18
• Timed Test A 65-year-old man, who has smoked 20 cigarettes a day
for 50 years and who has worked in the shipyard industry, Questions Incorrect 6
• Mock Exam
presents to clinic with chest pain and increasing shortness Questions Total 24
• Past Papers of breath. Clinical examination and chest radiograph show
Questions Percentage 75 %
• Random Questions he has a right pleural effusion. What investigation would
you perform next? More
• My Performance
• Media Bank Pleural aspiration Correct answer
• New Multimedia Abrams’ pleural biopsy
Online Extras Bronchoscopy
Library Thoracoscopy
Community CT scan Your answer
Help
PasTest Store

My Account Pleural aspiration is a useful preliminary test, which can


be performed in clinic. Pleural fluid should be sent for
Profile cytology, protein, LDH, pH and culture. The likely
causes for the pleural effusion in this man are
Newsletters
metastatic lung cancer or mesothelioma. Pleural
My Career and Exams aspiration may be diagnostic in the case of lung cancer,
Order History although not in mesothelioma. The next step would be Reference: Normal Values
to perform a contrast CT scan. In lung cancer, this may
Learning Goals show a chest primary and, depending on its location, a Haematology
Question Filters bronchoscopy or CT-guided biopsy may be performed.
In cases of mesothelioma, CT might show pleural Immunoglobulins
thickening and nodularity, which would be amenable to
Biochemistry
Security either CT-guided biopsy or thoracoscopy. CT pleural
biopsy has been shown to have a diagnostic advantage Diabetes
Change Password over blind Abrams’ biopsy in cytologically negative
Sign Out malignant pleural effusions with pleural thickening, with Endocrinology
a sensitivity of 87% vs 47% (see Maskell et al 2003.
Lancet,361,1326–30). Blood gases

CSF

950

© 2011 PasTest Ltd | About Us | Contact Us | Help

file:///E|/Shakhawan/Respiratory%20S/127a.htm[3/13/2012 4:02:04 PM]


MyPasTest

Main Navigation
Question Browser: MRCP 1
Home
Subscriptions Question Browser Exam Builder Saved Exams
» MRCP 1 Session Progress
• Question Browser
Questions Correct 18
• Timed Test A 78-year-old man, who worked as a plumber, presents
with a unilateral pleural effusion. He has felt unwell for Questions Incorrect 6
• Mock Exam
some time and spends much of the day sitting in a chair. Questions Total 24
• Past Papers Pleural biopsies taken at thoracoscopy have shown
Questions Percentage 75 %
• Random Questions malignant mesothelioma and extensive tumour on all
pleural surfaces was noted. Having explained the More
• My Performance
diagnosis to him, which would be the most
• Media Bank appropriate treatment to consider next?
• New Multimedia
Radical surgery alone Reference: Normal Values
Online Extras
Radical surgery combined with chemotherapy and Haematology
Library radiotherapy
Community Immunoglobulins
Chemotherapy alone
Help Radiotherapy to the hemithorax Biochemistry
PasTest Store Radiotherapy to the thoracoscopy tract site Diabetes

951 Endocrinology
My Account
Blood gases
Profile
CSF
Newsletters
My Career and Exams
Order History
Learning Goals
Question Filters

Security

Change Password
Sign Out

© 2011 PasTest Ltd | About Us | Contact Us | Help

file:///E|/Shakhawan/Respiratory%20S/128.htm[3/13/2012 4:02:05 PM]


MyPasTest

Main Navigation
Question Browser: MRCP 1
Home
Subscriptions Question Browser Exam Builder Saved Exams
» MRCP 1 Session Progress
• Question Browser
Questions Correct 18
• Timed Test A 78-year-old man, who worked as a plumber, presents
with a unilateral pleural effusion. He has felt unwell for Questions Incorrect 7
• Mock Exam
some time and spends much of the day sitting in a chair. Questions Total 25
• Past Papers Pleural biopsies taken at thoracoscopy have shown
Questions Percentage 72 %
• Random Questions malignant mesothelioma and extensive tumour on all
pleural surfaces was noted. Having explained the More
• My Performance
diagnosis to him, which would be the most
• Media Bank appropriate treatment to consider next?
• New Multimedia
Radical surgery alone
Online Extras
Radical surgery combined with Your answer
Library chemotherapy and radiotherapy
Community Chemotherapy alone
Help Radiotherapy to the hemithorax
PasTest Store Radiotherapy to the thoracoscopy Correct answer
tract site
My Account

Profile
Newsletters
Radical surgery (extraparietal pneumonectomy) involves
My Career and Exams removing the lung, parietal pleura, diaphragm and
Order History pericardium. It is suitable only for fit patients with a low Reference: Normal Values
tumour volume. Used alone, it does not improve
Learning Goals survival, but studies are underway using it as part of Haematology
Question Filters trimodality treatment with chemotherapy and
radiotherapy. These suggest it may improve survival, Immunoglobulins
although it is not curative. It has a perioperative
Biochemistry
Security morbidity of up to 50% and mortality of up to 8%.
Chemotherapy, as yet, has not been found to have any Diabetes
Change Password impact on survival, although 30% of patients have an
Sign Out objective tumour response. Radiotherapy to the entire Endocrinology
pleural surface is technically difficult and associated
with a high risk of radiation pneumonitis, hepatitis and Blood gases
pericarditis. Results are disappointing. Radiotherapy to CSF
chest instrumentation-tract sites is effective at
preventing tumour seeding and growth (see Boutin et al
1995. Chest,108(3),754-8) and is recommended after
pleural procedures. This is routine practice, along with
providing symptomatic treatment.

951

© 2011 PasTest Ltd | About Us | Contact Us | Help

file:///E|/Shakhawan/Respiratory%20S/128a.htm[3/13/2012 4:02:07 PM]


MyPasTest

Main Navigation
Question Browser: MRCP 1
Home
Subscriptions Question Browser Exam Builder Saved Exams
» MRCP 1 Session Progress
• Question Browser
Questions Correct 18
• Timed Test A 70-year-old woman with known chronic obstructive
pulmonary disease (COPD) is admitted to the respiratory Questions Incorrect 7
• Mock Exam
ward. She is breathless, coughing up green sputum and has Questions Total 25
• Past Papers an audible wheeze. She is given 24% oxygen via a Venturi
Questions Percentage 72 %
• Random Questions mask, nebulised salbutamol and ipratropium and oral
prednisolone. After one hour, she is still dyspnoeic, tells you More
• My Performance
she feels no better and you note that her blood gases have
• Media Bank deteriorated. Her pH is 7.30, p(O2 ) 6 kPa, p(CO 2 ) 10 kPa,
• New Multimedia bicarbonate 22 mmol/l, base excess –4 mmol/l.
Reference: Normal Values
Online Extras What management would you recommend next?
Haematology
Library
Increase the oxygen to 28% Immunoglobulins
Community
Commence intravenous aminophylline
Help Biochemistry
Commence intravenous doxapram
PasTest Store Diabetes
Commence non-invasive ventilation (NIV)
Refer her to the intensive care unit for intubation and Endocrinology
My Account
ventilation Blood gases
Profile
952 CSF
Newsletters
My Career and Exams
Order History
Learning Goals
Question Filters

Security

Change Password
Sign Out

© 2011 PasTest Ltd | About Us | Contact Us | Help

file:///E|/Shakhawan/Respiratory%20S/129.htm[3/13/2012 4:02:08 PM]


MyPasTest

Main Navigation
Question Browser: MRCP 1
Home
Subscriptions Question Browser Exam Builder Saved Exams
» MRCP 1 Session Progress
• Question Browser
Questions Correct 19
• Timed Test A 70-year-old woman with known chronic obstructive
pulmonary disease (COPD) is admitted to the respiratory Questions Incorrect 7
• Mock Exam
ward. She is breathless, coughing up green sputum and has Questions Total 26
• Past Papers an audible wheeze. She is given 24% oxygen via a Venturi
Questions Percentage 73 %
• Random Questions mask, nebulised salbutamol and ipratropium and oral
prednisolone. After one hour, she is still dyspnoeic, tells you More
• My Performance
she feels no better and you note that her blood gases have
• Media Bank deteriorated. Her pH is 7.30, p(O2 ) 6 kPa, p(CO 2 ) 10 kPa,
• New Multimedia bicarbonate 22 mmol/l, base excess –4 mmol/l.
Online Extras What management would you recommend next?
Library
Increase the oxygen to 28%
Community
Commence intravenous aminophylline
Help
Commence intravenous doxapram
PasTest Store
Commence non-invasive ventilation Your answer
(NIV)
My Account
Refer her to the intensive care unit for
Profile intubation and ventilation

Newsletters
My Career and Exams
Order History Reference: Normal Values
This woman has a respiratory acidosis from the carbon
Learning Goals dioxide retention and is also hypoxic. Increasing her Haematology
Question Filters inspired oxygen concentration will lead to further CO2
Immunoglobulins
retention and worsen the acidosis. The role of
aminophylline is unclear. It can be used with nebulised Biochemistry
Security bronchodilators, but care should be taken regarding
drug interactions and drug levels. Doxapram is a Diabetes
Change Password respiratory stimulant, which can be effective at
Sign Out improving ventilation. Its use has been superseded by Endocrinology
NIV, but it might be useful in situations where NIV is Blood gases
unavailable. NIV has been found to decrease mortality
and decrease the need for intubation in exacerbations of CSF
COPD (see Plant et al 2000. Lancet,355,1931–5). It is
appropriate for conscious patients with a pH < 7.35 who
have not improved on controlled oxygen and standard
medical therapy. Mechanical ventilation may be
necessary if they are unconscious or their pH is below
7.25.

952

© 2011 PasTest Ltd | About Us | Contact Us | Help

file:///E|/Shakhawan/Respiratory%20S/129a.htm[3/13/2012 4:02:10 PM]


MyPasTest

Main Navigation
Question Browser: MRCP 1
Home
Subscriptions Question Browser Exam Builder Saved Exams
» MRCP 1 Session Progress
• Question Browser
Questions Correct 19
• Timed Test A fit 50-year-old man with a 20-pack year smoking history
presents with haemoptysis and a 3-cm right upper lobe Questions Incorrect 7
• Mock Exam
mass on his chest radiograph. Bronchoscopy has shown this Questions Total 26
• Past Papers is a squamous-cell carcinoma; CT has confirmed the mass
Questions Percentage 73 %
• Random Questions and has also shown enlarged 1-cm short-axis hilar and
paratracheal nodes. He is discussed at the lung-cancer More
• My Performance
multidisciplinary team meeting. What are they likely to
• Media Bank think is the most appropriate next course of
• New Multimedia management?
Reference: Normal Values
Online Extras Refer for PET scan
Haematology
Library Refer for surgical resection of the mass
Community Immunoglobulins
Refer for radiotherapy to the mass
Help Refer for chemotherapy Biochemistry
PasTest Store Refer to the palliative care team Diabetes

953 Endocrinology
My Account
Blood gases
Profile
CSF
Newsletters
My Career and Exams
Order History
Learning Goals
Question Filters

Security

Change Password
Sign Out

© 2011 PasTest Ltd | About Us | Contact Us | Help

file:///E|/Shakhawan/Respiratory%20S/130.htm[3/13/2012 4:02:11 PM]


MyPasTest

Main Navigation
Question Browser: MRCP 1
Home
Subscriptions Question Browser Exam Builder Saved Exams
» MRCP 1 Session Progress
• Question Browser
Questions Correct 0
• Timed Test A fit 50-year-old man with a 20-pack year smoking history
presents with haemoptysis and a 3-cm right upper lobe Questions Incorrect 1
• Mock Exam
mass on his chest radiograph. Bronchoscopy has shown this Questions Total 1
• Past Papers is a squamous-cell carcinoma; CT has confirmed the mass
Questions Percentage 0%
• Random Questions and has also shown enlarged 1-cm short-axis hilar and
paratracheal nodes. He is discussed at the lung-cancer More
• My Performance
multidisciplinary team meeting. What are they likely to
• Media Bank think is the most appropriate next course of
• New Multimedia management?

Online Extras Refer for PET scan Correct answer


Library Refer for surgical resection of the
Community mass
Help Refer for radiotherapy to the mass
PasTest Store Refer for chemotherapy
Refer to the palliative care team Your answer
My Account

Profile
Newsletters
This is a man with potentially operable non-small-cell
My Career and Exams lung cancer. He has enlarged hilar nodes, but these
Order History could be removed with pneumonectomy, but he also has Reference: Normal Values
enlarged mediastinal nodes, which if they were
Learning Goals cancerous, would make the cancer resection non- Haematology
Question Filters curative. However CT is not great at assessing whether
enlarged nodes are inflammatory or malignant. He Immunoglobulins
therefore needs further assessment of his mediastinal
Biochemistry
Security nodes prior to surgery, and this can be done with
mediastinoscopy or a positron-emission tomography Diabetes
Change Password (PET) scan. In a PET scan, radiolabelled glucose is
Sign Out injected peripherally and is taken up by metabolically Endocrinology
active tissues, such as the brain and any cancer. It
would show metastases from the primary cancer. If his Blood gases
cancer is inoperable, palliative radiotherapy can be CSF
given to the mass, to try and control the haemoptysis.
Chemotherapy could also be considered. Palliative care
team referral would be appropriate if he does not want
interventions or has troublesome symptoms.

953

© 2011 PasTest Ltd | About Us | Contact Us | Help

file:///E|/Shakhawan/Respiratory%20S/130a.htm[3/13/2012 4:02:12 PM]


MyPasTest

Main Navigation
Question Browser: MRCP 1
Home
Subscriptions Question Browser Exam Builder Saved Exams
» MRCP 1 Session Progress
• Question Browser
Questions Correct 0
• Timed Test A 30-year-old lorry driver comes to clinic. He is overweight
with a body mass index of 40 and has been having trouble Questions Incorrect 1
• Mock Exam
staying awake at the wheel. His wife says he has always Questions Total 1
• Past Papers snored and she has often heard him stop breathing. His
Questions Percentage 0%
• Random Questions baseline oxygen saturation in clinic is 97%. His overnight
sleep study shows a 4% oxygen dip rate of 20/hour, and More
• My Performance
his Epworth Sleepiness Score is 19.
• Media Bank
• New Multimedia What would you recommend as the most appropriate
management for him? Reference: Normal Values
Online Extras
Weight loss Haematology
Library
Community Mandibular advancement device Immunoglobulins
Help Start modafanil – a wake-promoting drug Biochemistry
PasTest Store Start CPAP (continuous positive airways pressure
Diabetes
ventilation)
Start BiPAP (bilevel positive airways pressure Endocrinology
My Account ventilation)
Blood gases
Profile
954 CSF
Newsletters
My Career and Exams
Order History
Learning Goals
Question Filters

Security

Change Password
Sign Out

© 2011 PasTest Ltd | About Us | Contact Us | Help

file:///E|/Shakhawan/Respiratory%20S/131.htm[3/13/2012 4:02:14 PM]


MyPasTest

Main Navigation
Question Browser: MRCP 1
Home
Subscriptions Question Browser Exam Builder Saved Exams
» MRCP 1 Session Progress
• Question Browser
Questions Correct 1
• Timed Test A 30-year-old lorry driver comes to clinic. He is overweight
with a body mass index of 40 and has been having trouble Questions Incorrect 1
• Mock Exam
staying awake at the wheel. His wife says he has always Questions Total 2
• Past Papers snored and she has often heard him stop breathing. His
Questions Percentage 50 %
• Random Questions baseline oxygen saturation in clinic is 97%. His overnight
sleep study shows a 4% oxygen dip rate of 20/hour, and More
• My Performance
his Epworth Sleepiness Score is 19.
• Media Bank
• New Multimedia What would you recommend as the most appropriate
management for him?
Online Extras
Library Weight loss
Community Mandibular advancement device
Help Start modafanil – a wake-promoting
drug
PasTest Store
Start CPAP (continuous positive Your answer
airways pressure ventilation)
My Account Start BiPAP (bilevel positive airways
Profile pressure ventilation)

Newsletters
My Career and Exams
Order History Reference: Normal Values
This man has significant obstructive sleep apnoea
Learning Goals (OSA), which is making him sleepy (Epworth Sleepiness Haematology
Question Filters Score of 10 or above = sleepy). As he is a lorry driver,
he cannot drive heavy goods vehicles until this is Immunoglobulins
treated. The quickest and most effective treatment for
Biochemistry
Security severe OSA is CPAP. Continuous positive pressure
airways ventilation acts as a splint to keep the airway Diabetes
Change Password open, leading to a normal night’s sleep. BiPAP is used if
Sign Out people are in respiratory failure: although this is rare Endocrinology
with OSA alone, it can occur if they have an associated
problem, such as obesity hypoventilation or COPD. Blood gases
Weight loss is no doubt the ultimate treatment for OSA, CSF
but in practice it is difficult and slow. Mandibular
advancement devices are effective at treating milder
OSA – they hold the lower jaw in a fixed position during
the night. Modafanil is a drug that is licensed for
excessive daytime sleepiness in people with OSA treated
with CPAP, as well as narcolepsy.

954

© 2011 PasTest Ltd | About Us | Contact Us | Help

file:///E|/Shakhawan/Respiratory%20S/131a.htm[3/13/2012 4:02:15 PM]


MyPasTest

Main Navigation
Question Browser: MRCP 1
Home
Subscriptions Question Browser Exam Builder Saved Exams
» MRCP 1
• Question Browser
• Timed Test A 50-year-old woman comes to Session Progress
• Mock Exam clinic complaining of
breathlessness on exertion, which Questions Correct 1
• Past Papers has been worsening for 1 year.
• Random Questions She has never smoked. Chest x- Questions Incorrect 1
ray shows mild hyperinflation but Questions Total 2
• My Performance is otherwise unremarkable. Her
• Media Bank lung function tests are shown Questions Percentage 50 %
below: More
• New Multimedia
Online Extras Predicted
Library FEV1 (l) 1.1 1.7–3.0 Reference: Normal Values
Community
FVC (l) 2.7 2.0–3.5
Help Haematology
FRC (l) 4.2 1.7–3.4
PasTest Store Immunoglobulins
TLC (l) 6.5 3.6–5.6
Biochemistry
TLCO
My Account (mmol/min 5.86 Diabetes
per kPa) (77%) 5.7–9.5
Profile Endocrinology
Newsletters KCO
(mmol/min Blood gases
My Career and Exams per kPa 1.19
per l) (72%) 1.66 CSF
Order History
Learning Goals
What is the most likely
Question Filters diagnosis?

Asthma
Security
COPD
Change Password Pulmonary fibrosis
Sign Out Bronchiectasis
Pulmonary haemorrhage

955

© 2011 PasTest Ltd | About Us | Contact Us | Help

file:///E|/Shakhawan/Respiratory%20S/132.htm[3/13/2012 4:02:17 PM]


MyPasTest

Main Navigation
Question Browser: MRCP 1
Home
Subscriptions Question Browser Exam Builder Saved Exams
» MRCP 1
• Question Browser
• Timed Test A 50-year-old woman comes to Session Progress
• Mock Exam clinic complaining of breathlessness
on exertion, which has been Questions Correct 1
• Past Papers worsening for 1 year. She has never
• Random Questions smoked. Chest x-ray shows mild Questions Incorrect 2
hyperinflation but is otherwise Questions Total 3
• My Performance unremarkable. Her lung function
• Media Bank tests are shown below: Questions Percentage 33 %

• New Multimedia More


Predicted
Online Extras
FEV1 (l) 1.1 1.7–3.0
Library
Community FVC (l) 2.7 2.0–3.5

Help FRC (l) 4.2 1.7–3.4

PasTest Store TLC (l) 6.5 3.6–5.6


TLCO
(mmol/min 5.86
My Account per kPa) (77%) 5.7–9.5
Profile KCO
Newsletters (mmol/min 1.19
per kPa per l) (72%) 1.66
My Career and Exams
Order History What is the most likely
diagnosis?
Learning Goals
Question Filters Asthma Correct answer Reference: Normal Values
COPD Haematology
Security Pulmonary Your answer
fibrosis Immunoglobulins
Change Password
Biochemistry
Sign Out Bronchiectasis
Diabetes
Pulmonary
haemorrhage Endocrinology

Blood gases
CSF

This woman has obstructive


spirometry, with raised lung
volumes. This is suggestive of
asthma or chronic obstructive
pulmonary disease (COPD). As
she has never smoked, she is
more likely to have asthma.
Non-smokers can develop
COPD, but they tend to have a
heavy passive smoke exposure
and be older. Bronchiectasis can
give a mild obstructive defect,
but would not tend to give such
marked hyperinflation.
Pulmonary fibrosis would give a
restrictive defect. Pulmonary
haemorrhage would give a
raised TLCO and KCO (transfer
factor for carbon monoxide).

955

© 2011 PasTest Ltd | About Us | Contact Us | Help

file:///E|/Shakhawan/Respiratory%20S/132a.htm[3/13/2012 4:02:18 PM]


MyPasTest

Main Navigation
Question Browser: MRCP 1
Home
Subscriptions Question Browser Exam Builder Saved Exams
» MRCP 1 Session Progress
• Question Browser
Questions Correct 1
• Timed Test A 50-year-old man attends clinic with haemoptysis and a
1-week history of dyspnoea. Examination is unremarkable, Questions Incorrect 2
• Mock Exam
but his chest radiograph shows bilateral fluffy shadows. Questions Total 3
• Past Papers Urine dipstick is positive for blood and protein. What would
Questions Percentage 33 %
• Random Questions you do next?
• My Performance More
Send a serum ANCA and arrange to see him in clinic
• Media Bank in 1 week
• New Multimedia Perform pulmonary function tests and see him in 1 Reference: Normal Values
Online Extras week for repeat tests
Arrange an out-patient renal ultrasound Haematology
Library
Community Admit him to the ward, check his renal function and Immunoglobulins
send blood for autoantibodies
Help Biochemistry
Admit him to the ward and start iv cyclophosphamide
PasTest Store Diabetes
956
Endocrinology
My Account
Blood gases
Profile
CSF
Newsletters
My Career and Exams
Order History
Learning Goals
Question Filters

Security

Change Password
Sign Out

© 2011 PasTest Ltd | About Us | Contact Us | Help

file:///E|/Shakhawan/Respiratory%20S/133.htm[3/13/2012 4:02:20 PM]


MyPasTest

Main Navigation
Question Browser: MRCP 1
Home
Subscriptions Question Browser Exam Builder Saved Exams
» MRCP 1 Session Progress
• Question Browser
Questions Correct 1
• Timed Test A 50-year-old man attends clinic with haemoptysis and a
1-week history of dyspnoea. Examination is unremarkable, Questions Incorrect 3
• Mock Exam
but his chest radiograph shows bilateral fluffy shadows. Questions Total 4
• Past Papers Urine dipstick is positive for blood and protein. What would
Questions Percentage 25 %
• Random Questions you do next?
• My Performance More
Send a serum ANCA and arrange to Your answer
• Media Bank see him in clinic in 1 week
• New Multimedia Perform pulmonary function tests
Online Extras and see him in 1 week for repeat
tests
Library
Arrange an out-patient renal
Community ultrasound
Help Admit him to the ward, check his Correct answer
PasTest Store renal function and send blood for
autoantibodies
Admit him to the ward and start iv
My Account cyclophosphamide
Profile
Newsletters
My Career and Exams
Order History The fluffy shadows on the chest X-ray could represent Reference: Normal Values
pulmonary oedema, interstitial lung disease, vasculitic
Learning Goals lung disease or pulmonary haemorrhage. CT may help Haematology
Question Filters to differentiate between these possibilities. Combined
with the positive urine dipstick, this man could have an Immunoglobulins
active vasculitis with pulmonary haemorrhage and renal
Biochemistry
Security disease. He could have Wegener’s granulomatosis,
Goodpasture’s syndrome or microscopic polyangiitis. He Diabetes
Change Password needs urgent admission and assessment. He may have
Sign Out renal failure, he may develop respiratory failure and he Endocrinology
may need to start urgent immunosuppression if a
diagnosis of vasculitis is made. This diagnosis is best Blood gases
made by biopsy: either of the kidneys, or an open lung CSF
biopsy, or of any skin lesion/nose lesion if present.
Ideally, he should have a tissue diagnosis before
starting immunosuppressive treatment.

956

© 2011 PasTest Ltd | About Us | Contact Us | Help

file:///E|/Shakhawan/Respiratory%20S/133a.htm[3/13/2012 4:02:21 PM]


MyPasTest

Main Navigation
Question Browser: MRCP 1
Home
Subscriptions Question Browser Exam Builder Saved Exams
» MRCP 1 Session Progress
• Question Browser
Questions Correct 1
• Timed Test A 50-year-old woman is admitted with a dry cough,
shortness of breath and a 2-week history of intermittent Questions Incorrect 3
• Mock Exam
fevers. She had had flu-like symptoms at the beginning of Questions Total 4
• Past Papers her illness. On examination she has right-sided crepitations
Questions Percentage 25 %
• Random Questions and a chest X-ray shows patchy shadowing at her right
lower lobe, with an air bronchogram. Her white cell count More
• My Performance
and CRP are raised. She is started on antibiotics for
• Media Bank community-acquired pneumonia and after 2 days of
• New Multimedia clinically improving, she is discharged. You see her in clinic
3 months later, when she tells you she is no better. Her Reference: Normal Values
Online Extras chest X-ray shows left upper lobe consolidation.
Haematology
Library
What is the most likely cause of this? Immunoglobulins
Community
Help Recurrent bacterial pneumonia Biochemistry
PasTest Store Eosinophilic pneumonia Diabetes
Cryptogenic organising pneumonia
Endocrinology
My Account Lymphangioleiomyomatosis
Blood gases
Profile Pulmonary alveolar proteinosis
CSF
Newsletters 957
My Career and Exams
Order History
Learning Goals
Question Filters

Security

Change Password
Sign Out

© 2011 PasTest Ltd | About Us | Contact Us | Help

file:///E|/Shakhawan/Respiratory%20S/134.htm[3/13/2012 4:02:23 PM]


MyPasTest

Main Navigation
Question Browser: MRCP 1
Home
Subscriptions Question Browser Exam Builder Saved Exams
» MRCP 1 Session Progress
• Question Browser
Questions Correct 2
• Timed Test A 50-year-old woman is admitted with a dry cough,
shortness of breath and a 2-week history of intermittent Questions Incorrect 3
• Mock Exam
fevers. She had had flu-like symptoms at the beginning of Questions Total 5
• Past Papers her illness. On examination she has right-sided crepitations
Questions Percentage 40 %
• Random Questions and a chest X-ray shows patchy shadowing at her right
lower lobe, with an air bronchogram. Her white cell count More
• My Performance
and CRP are raised. She is started on antibiotics for
• Media Bank community-acquired pneumonia and after 2 days of
• New Multimedia clinically improving, she is discharged. You see her in clinic
3 months later, when she tells you she is no better. Her
Online Extras chest X-ray shows left upper lobe consolidation.
Library
What is the most likely cause of this?
Community
Help Recurrent bacterial pneumonia
PasTest Store Eosinophilic pneumonia
Cryptogenic organising pneumonia Your answer
My Account Lymphangioleiomyomatosis

Profile Pulmonary alveolar proteinosis

Newsletters
My Career and Exams
Order History Reference: Normal Values
Cryptogenic organising pneumonia is a non-specific
Learning Goals Haematology
inflammatory pulmonary process, with buds of
Question Filters granulation tissue forming in the distal air spaces.
Immunoglobulins
Organising pneumonia may result from a number of
causes, including connective tissue disease, infection Biochemistry
Security and drugs, but if there is no obvious cause it is called
‘cryptogenic’. It causes non-specific symptoms of Diabetes
Change Password fever, dry cough, malaise, anorexia and weight loss.
Sign Out It is associated with raised a WCC and C-reactive Endocrinology
protein (CRP) levels. The chest X-ray can show Blood gases
consolidation, nodules or thickened septal lines. The
consolidation typically occurs in different places at CSF
different times. CT findings are characteristic with
multiple patchy alveolar opacities, which often
spontaneously migrate. The diagnosis may be made
from CT alone, or from transbronchial or open lung
biopsy. Treatment is with steroids. Relapse is common
with further consolidation and may need treatment
with increased steroid doses.
Recurrent lobar bacterial infection is unusual in an
immunocompetent adult, and eosinophilic pneumonia
gives flitting peripheral chest X-ray shadows.
Lymphangioleiomyomatosis is a rare disease affecting
women of child-bearing age, who have abnormal
proliferation of atypical smooth muscle cells
throughout their lungs and airways. CT shows multiple
small cysts. Pulmonary alveolar proteinosis is a rare
defect where the alveoli become filled with
proteinaceous material that cannot be cleared. CT
shows airspace shadowing.

957

file:///E|/Shakhawan/Respiratory%20S/134a.htm[3/13/2012 4:02:24 PM]


MyPasTest

Main Navigation
Question Browser: MRCP 1
Home
Subscriptions Question Browser Exam Builder Saved Exams
» MRCP 1 Session Progress
• Question Browser
Questions Correct 2
• Timed Test A 22-year-old student is admitted to the emergency
department, having been found by her flatmates to be Questions Incorrect 3
• Mock Exam
unsteady on her feet and having problems with her Questions Total 5
• Past Papers memory. They tell you they have been using old-fashioned
Questions Percentage 40 %
• Random Questions gas fires as it has been so cold and you are wondering
whether she could have carbon monoxide poisoning. More
• My Performance
• Media Bank Which test will be the most helpful in determining
this?
• New Multimedia
Reference: Normal Values
Online Extras Clinical examination
Haematology
Library Pulse oximetry
Community Arterial blood gas analysis Immunoglobulins
Help Exhaled breath test Biochemistry
PasTest Store Chest radiograph Diabetes

958 Endocrinology
My Account
Blood gases
Profile
CSF
Newsletters
My Career and Exams
Order History
Learning Goals
Question Filters

Security

Change Password
Sign Out

© 2011 PasTest Ltd | About Us | Contact Us | Help

file:///E|/Shakhawan/Respiratory%20S/135.htm[3/13/2012 4:02:26 PM]


MyPasTest

Main Navigation
Question Browser: MRCP 1
Home
Subscriptions Question Browser Exam Builder Saved Exams
» MRCP 1 Session Progress
• Question Browser
Questions Correct 2
• Timed Test A 22-year-old student is admitted to the emergency
department, having been found by her flatmates to be Questions Incorrect 4
• Mock Exam
unsteady on her feet and having problems with her Questions Total 6
• Past Papers memory. They tell you they have been using old-fashioned
Questions Percentage 33 %
• Random Questions gas fires as it has been so cold and you are wondering
whether she could have carbon monoxide poisoning. More
• My Performance
• Media Bank Which test will be the most helpful in determining
this?
• New Multimedia
Online Extras Clinical examination
Library Pulse oximetry Your answer
Community Arterial blood gas analysis Correct answer
Help Exhaled breath test
PasTest Store Chest radiograph

My Account

Profile
Breath carbon monoxide levels can be measured using
Newsletters devices intended for smoking cessation work. They can
My Career and Exams also be measured on a co-oximeter, measuring
carboxyhaemoglobin levels. A&E Departments now also Reference: Normal Values
Order History
carry a blood gas machine which is able to read CO on a
Learning Goals blood sample. Clinical examination is unhelpful, as Haematology
Question Filters patients are not cyanosed, but a cherry-red colour.
Pulse oximetry appears normal, due to Immunoglobulins
carboxyhaemoglobin having similar absorption spectra
to oxyhaemoglobin. Arterial oxygen levels may be Biochemistry
Security
normal, as may a chest radiograph. Treatment for Diabetes
Change Password carbon monoxide poisoning is to give high levels of
inspired oxygen, by ventilation (or hyperbaric means if Endocrinology
Sign Out
necessary), to displace the carbon monoxide and
increase the amount of dissolved oxygen. Blood gases

CSF
958

© 2011 PasTest Ltd | About Us | Contact Us | Help

file:///E|/Shakhawan/Respiratory%20S/135a.htm[3/13/2012 4:02:27 PM]


MyPasTest

Main Navigation
Question Browser: MRCP 1
Home
Subscriptions Question Browser Exam Builder Saved Exams
» MRCP 1 Session Progress
• Question Browser
Questions Correct 2
• Timed Test You are fast-bleeped to see a 50-year-old woman on the
medical ward. She was admitted earlier that day with a Questions Incorrect 4
• Mock Exam
swollen lower leg following her return from a holiday in Questions Total 6
• Past Papers Australia. Her d-dimers were raised and she was started on
Questions Percentage 33 %
• Random Questions subcutaneous unfractionated heparin. She is now short of
breath, pale, clammy, tachycardic and hypotensive. Her More
• My Performance
ECG shows sinus tachycardia. The house officer has given
• Media Bank her iv fluids, but her blood pressure is continuing to fall
• New Multimedia (70/50 mmHg).
Reference: Normal Values
Online Extras What would you consider doing next?
Haematology
Library
Intravenous heparin
Community Immunoglobulins
Further iv fluids
Help Biochemistry
Thrombolysis
PasTest Store Diabetes
Echocardiogram
V/Q scan Endocrinology
My Account
Blood gases
Profile 959
CSF
Newsletters
My Career and Exams
Order History
Learning Goals
Question Filters

Security

Change Password
Sign Out

© 2011 PasTest Ltd | About Us | Contact Us | Help

file:///E|/Shakhawan/Respiratory%20S/136.htm[3/13/2012 4:02:28 PM]


MyPasTest

Main Navigation
Question Browser: MRCP 1
Home
Subscriptions Question Browser Exam Builder Saved Exams
» MRCP 1 Session Progress
• Question Browser
Questions Correct 3
• Timed Test You are fast-bleeped to see a 50-year-old woman on the
medical ward. She was admitted earlier that day with a Questions Incorrect 4
• Mock Exam
swollen lower leg following her return from a holiday in Questions Total 7
• Past Papers Australia. Her d-dimers were raised and she was started on
Questions Percentage 42 %
• Random Questions subcutaneous unfractionated heparin. She is now short of
breath, pale, clammy, tachycardic and hypotensive. Her More
• My Performance
ECG shows sinus tachycardia. The house officer has given
• Media Bank her iv fluids, but her blood pressure is continuing to fall
• New Multimedia (70/50 mmHg).

Online Extras What would you consider doing next?


Library
Intravenous heparin
Community
Further iv fluids
Help
Thrombolysis Your answer
PasTest Store
Echocardiogram
V/Q scan
My Account

Profile
Newsletters
My Career and Exams This is a woman who has a high clinical probability of
having had a massive pulmonary embolus (PE). She has Reference: Normal Values
Order History
respiratory and cardiovascular compromise from this PE
Learning Goals and is likely to die without treatment. Investigation with Haematology
Question Filters either echocardiogram showing pulmonary hypertension
or CT showing a clot(s), would be reasonable to confirm Immunoglobulins
the diagnosis, but this woman is rapidly deteriorating.
She is not well enough for a V/Q scan. She has had iv Biochemistry
Security
fluids and unfractionated heparin already, with no Diabetes
Change Password effect. Thrombolysis can be used for acute pulmonary
emboli causing circulatory collapse and there is Endocrinology
Sign Out
emerging evidence to support this treatment. The
risk/benefit analysis must be considered, but in a Blood gases
patient who is at imminent risk of arrest/death, give
CSF
alteplase 100 mg over 1.5 hours peripherally. (see BTS
guidelines for the management of suspected acute
pulmonary embolism. Thorax 2003,58(6),470).

959

© 2011 PasTest Ltd | About Us | Contact Us | Help

file:///E|/Shakhawan/Respiratory%20S/136a.htm[3/13/2012 4:02:30 PM]


MyPasTest

Main Navigation
Question Browser: MRCP 1
Home
Subscriptions Question Browser Exam Builder Saved Exams
» MRCP 1 Session Progress
• Question Browser
Questions Correct 3
• Timed Test Which of the following is the best predictor for obstructive
sleep apnoea? Questions Incorrect 4
• Mock Exam
Questions Total 7
• Past Papers
Neck size Questions Percentage 42 %
• Random Questions
Chest size
• My Performance More
Abdominal girth
• Media Bank
Waist to hip ratio
• New Multimedia
Uvulopalatal distance Reference: Normal Values
Online Extras
960 Haematology
Library
Community Immunoglobulins
Help Biochemistry
PasTest Store Diabetes
Endocrinology
My Account
Blood gases
Profile
CSF
Newsletters
My Career and Exams
Order History
Learning Goals
Question Filters

Security

Change Password
Sign Out

© 2011 PasTest Ltd | About Us | Contact Us | Help

file:///E|/Shakhawan/Respiratory%20S/137.htm[3/13/2012 4:02:31 PM]


MyPasTest

Main Navigation
Question Browser: MRCP 1
Home
Subscriptions Question Browser Exam Builder Saved Exams
» MRCP 1 Session Progress
• Question Browser
Questions Correct 3
• Timed Test Which of the following is the best predictor for obstructive
sleep apnoea? Questions Incorrect 5
• Mock Exam
Questions Total 8
• Past Papers
Neck size Correct answer Questions Percentage 37 %
• Random Questions
Chest size
• My Performance More
Abdominal girth
• Media Bank
Waist to hip ratio
• New Multimedia
Uvulopalatal distance Your answer
Online Extras
Library
Community
Help Neck size is the best predictor of OSA, with > 43 cm
PasTest Store (>17 inches) being associated with an increased risk.
The mass loading from the obese or muscular neck
overwhelms the residual dilator muscle action of the
My Account pharynx when the patient is asleep, causing airway
obstruction and subsequent apnoea. Patients may also
Profile have a small or set back mandible, which predisposes
them to OSA as well. Typically, patients also have upper
Newsletters
body obesity, which is the typical male fat pattern.
My Career and Exams
Order History Reference: Normal Values
960
Learning Goals Haematology
Question Filters
Immunoglobulins
Biochemistry
Security
Diabetes
Change Password
Sign Out Endocrinology

Blood gases

CSF

© 2011 PasTest Ltd | About Us | Contact Us | Help

file:///E|/Shakhawan/Respiratory%20S/137a.htm[3/13/2012 4:02:33 PM]


MyPasTest

Main Navigation
Question Browser: MRCP 1
Home
Subscriptions Question Browser Exam Builder Saved Exams
» MRCP 1 Session Progress
• Question Browser
Questions Correct 3
• Timed Test You are asked to review a patient with known asthma on
the haematology ward. He is neutropenic from Questions Incorrect 5
• Mock Exam
chemotherapy for Hodgkin’s lymphoma. He has a cough Questions Total 8
• Past Papers and a low-grade fever, sparse crepitations on chest
Questions Percentage 37 %
• Random Questions examination and his chest X-ray shows diffuse pulmonary
shadowing. He has been on broad-spectrum antibiotics for More
• My Performance
1 week with no improvement. His sputum has shown a few
• Media Bank hyphae, but is culture-negative. Blood cultures have been
• New Multimedia negative. Aspergillus precipitins are negative, as is an
aspergillus skin-prick test. Reference: Normal Values
Online Extras
What is the diagnosis? Haematology
Library
Community Immunoglobulins
Allergic bronchopulmonary aspergillosis
Help Biochemistry
Invasive aspergillosis
PasTest Store Diabetes
Aspergilloma
Pneumocystis pneumonia Endocrinology
My Account Systemic candidosis
Blood gases
Profile
961 CSF
Newsletters
My Career and Exams
Order History
Learning Goals
Question Filters

Security

Change Password
Sign Out

© 2011 PasTest Ltd | About Us | Contact Us | Help

file:///E|/Shakhawan/Respiratory%20S/138.htm[3/13/2012 4:02:35 PM]


MyPasTest

Main Navigation
Question Browser: MRCP 1
Home
Subscriptions Question Browser Exam Builder Saved Exams
» MRCP 1 Session Progress
• Question Browser
Questions Correct 3
• Timed Test You are asked to review a patient with known asthma on
the haematology ward. He is neutropenic from Questions Incorrect 6
• Mock Exam
chemotherapy for Hodgkin’s lymphoma. He has a cough Questions Total 9
• Past Papers and a low-grade fever, sparse crepitations on chest
Questions Percentage 33 %
• Random Questions examination and his chest X-ray shows diffuse pulmonary
shadowing. He has been on broad-spectrum antibiotics for More
• My Performance
1 week with no improvement. His sputum has shown a few
• Media Bank hyphae, but is culture-negative. Blood cultures have been
• New Multimedia negative. Aspergillus precipitins are negative, as is an
aspergillus skin-prick test.
Online Extras
Library What is the diagnosis?
Community
Allergic bronchopulmonary
Help aspergillosis
PasTest Store Invasive aspergillosis Correct answer
Aspergilloma Your answer
My Account Pneumocystis pneumonia

Profile Systemic candidosis

Newsletters
My Career and Exams
Order History Reference: Normal Values
This man has invasive aspergillosis, due to his
Learning Goals immunosuppression. He has fungal hyphae in his Haematology
Question Filters sputum and a corresponding clinical and radiological
picture. He is unable to mount an immune response and Immunoglobulins
so precipitin and skin-prick tests are negative.
Biochemistry
Security Aspergilloma is a fungal ball in an area of previously
damaged lung tissue, such as old TB. It causes high Diabetes
Change Password levels of aspergillus antibody so precipitins are high,
Sign Out although the skin-prick test is negative as there is no Endocrinology
IgE-mediated allergy to aspergillus. Allergic
bronchopulmonary aspergillosis has high IgE and Blood gases
antibodies to aspergillus, so the skin-prick is positive CSF
and precipitins are high. Pneumocystis pneumonia (PCP)
does not cause hyphae in the sputum. Candida can be
identified and cultured from the sputum. It rarely
causes objective evidence of lung invasion and is not
recognised as having specific radiological features.

961

© 2011 PasTest Ltd | About Us | Contact Us | Help

file:///E|/Shakhawan/Respiratory%20S/138a.htm[3/13/2012 4:02:37 PM]


MyPasTest

Main Navigation
Question Browser: MRCP 1
Home
Subscriptions Question Browser Exam Builder Saved Exams
» MRCP 1 Session Progress
• Question Browser
Questions Correct 3
• Timed Test You see a 50-year-old man in the general medical clinic.
He was previously a keen sportsman, but over the past two Questions Incorrect 6
• Mock Exam
months or so has begun to feel breathless whenever he lies Questions Total 9
• Past Papers down flat or when he goes swimming. He has also noticed
Questions Percentage 33 %
• Random Questions some weakness of grip in both hands. On examination he
has mild weakness and wasting of his intrinsic hand More
• My Performance
muscles. Power in his legs is also minimally decreased, with
• Media Bank reflexes being present and symmetric.
• New Multimedia
What is the most likely cause of this presentation? Reference: Normal Values
Online Extras
Periodic paralysis Haematology
Library
Community Polymyositis Immunoglobulins
Help Chronic inflammatory demyelinating polyneuropathy Biochemistry
PasTest Store Motor neurone disease
Diabetes
Spinal muscular atrophy
Endocrinology
My Account 962
Blood gases
Profile
CSF
Newsletters
My Career and Exams
Order History
Learning Goals
Question Filters

Security

Change Password
Sign Out

© 2011 PasTest Ltd | About Us | Contact Us | Help

file:///E|/Shakhawan/Respiratory%20S/139.htm[3/13/2012 4:02:38 PM]


MyPasTest

Main Navigation
Question Browser: MRCP 1
Home
Subscriptions Question Browser Exam Builder Saved Exams
» MRCP 1 Session Progress
• Question Browser
Questions Correct 4
• Timed Test You see a 50-year-old man in the general medical clinic.
He was previously a keen sportsman, but over the past two Questions Incorrect 6
• Mock Exam
months or so has begun to feel breathless whenever he lies Questions Total 10
• Past Papers down flat or when he goes swimming. He has also noticed
Questions Percentage 40 %
• Random Questions some weakness of grip in both hands. On examination he
has mild weakness and wasting of his intrinsic hand More
• My Performance
muscles. Power in his legs is also minimally decreased, with
• Media Bank reflexes being present and symmetric.
• New Multimedia
What is the most likely cause of this presentation?
Online Extras
Library Periodic paralysis
Community Polymyositis
Help Chronic inflammatory demyelinating
polyneuropathy
PasTest Store
Motor neurone disease Your answer
Spinal muscular atrophy
My Account

Profile
Newsletters
My Career and Exams The most likely diagnosis here is motor neurone disease
(MND) in its commonest form of amyotrophic lateral Reference: Normal Values
Order History
sclerosis, causing limb weakness and wasting.
Learning Goals Fasciculations are often present but their absence does Haematology
Question Filters not exclude the diagnostic possibility. Involvement of
upper and lower motor neurones is typical but the Immunoglobulins
clinical signs evolve in the course of the illness and the
absence of "hard" upper motor neurone signs here does Biochemistry
Security
not at all exclude the diagnosis. In this case there is Diabetes
Change Password also likely bilateral diaphragmatic weakness causing
orthopnoea and exertional dyspnoea. Respiratory Endocrinology
Sign Out
involvement is thought to be present in up to 50% of
MND cases at presentation though symptoms may be Blood gases
subtle. Respiratory failure is the commonest cause of
CSF
death in this condition.

Bilateral diaphragmatic weakness can cause severe


dyspnoea, particularly during exertion, or on lying,
when the raised abdominal pressure causes lung
compression. Being in water also causes breathlessness
as the downward movement of the diaphragm due to
gravity is prevented by the raised abdominal pressure.
It can be confused with cardiac failure or nocturnal
asthma. Causes of bilateral diaphragm weakness include
trauma, cervical cord lesions, motor neurone disease
(MND), polio, peripheral neuropathy, acute idiopathic
polyneuropathy, myopathies and muscular dystrophies.
The patient should be advised to sleep with the bed
propped up, and saturations should be checked
regularly to ensure he is not slipping into respiratory
failure. If this were the case, nocturnal non-invasive
ventilation would be required. Guillain–Barré syndrome
does not cause this picture, but herpes zoster can cause
unilateral diaphragmatic weakness, which rarely causes
symptoms unless there is coexisting pulmonary disease.

Of the other incorrect answer options presented,


polymyositis and spino-muscular atrophy could also
cause respiratory muscle weakness but in the context of

file:///E|/Shakhawan/Respiratory%20S/139a.htm[3/13/2012 4:02:39 PM]


MyPasTest

different clinical pictures (polymyositis - painful tender


proximal muscles; spinal muscular atrophy, if late-onset
rather than the infantile form, tends to present in adults
in their 20's and 30's). Chronic inflammatory
demyelinating polyneuropathy produces a chronic
relapsing sensorimotor neuropathy (and is aetiologically
related to Guillain Barre syndrome). Periodic paralysis
produces paroxystic muscle weakness usually provoked
by metabolic change (e.g. exercise, stress, certain
meals).

962

© 2011 PasTest Ltd | About Us | Contact Us | Help

file:///E|/Shakhawan/Respiratory%20S/139a.htm[3/13/2012 4:02:39 PM]


MyPasTest

Main Navigation
Question Browser: MRCP 1
Home
Subscriptions Question Browser Exam Builder Saved Exams
» MRCP 1 Session Progress
• Question Browser
Questions Correct 4
• Timed Test A 30-year-old man from Somalia attends your clinic with a
productive cough. Sputum is smear-positive for Questions Incorrect 6
• Mock Exam
tuberculosis. What does this mean? Questions Total 10
• Past Papers
Questions Percentage 40 %
• Random Questions He needs treatment for tuberculosis and his close
• My Performance contacts need screening, but he is not infectious to More
casual contacts
• Media Bank
He needs treatment for tuberculosis, his close
• New Multimedia contacts need screening and he needs to be isolated Reference: Normal Values
Online Extras from casual contacts
He needs treatment for tuberculosis, but he is not Haematology
Library
infectious to close or casual contacts
Community Immunoglobulins
He has multidrug-resistant tuberculosis
Help Biochemistry
He has HIV-associated tuberculosis
PasTest Store Diabetes
963
Endocrinology
My Account
Blood gases
Profile
CSF
Newsletters
My Career and Exams
Order History
Learning Goals
Question Filters

Security

Change Password
Sign Out

© 2011 PasTest Ltd | About Us | Contact Us | Help

file:///E|/Shakhawan/Respiratory%20S/140.htm[3/13/2012 4:02:41 PM]


MyPasTest

Main Navigation
Question Browser: MRCP 1
Home
Subscriptions Question Browser Exam Builder Saved Exams
» MRCP 1 Session Progress
• Question Browser
Questions Correct 1
• Timed Test A 30-year-old man from Somalia attends your clinic with a
productive cough. Sputum is smear-positive for Questions Incorrect 0
• Mock Exam
tuberculosis. What does this mean? Questions Total 1
• Past Papers
Questions Percentage 100 %
• Random Questions He needs treatment for tuberculosis
• My Performance and his close contacts need screening, More
but he is not infectious to casual
• Media Bank contacts
• New Multimedia He needs treatment for tuberculosis, Your answer
Online Extras his close contacts need screening and
he needs to be isolated from casual
Library contacts
Community He needs treatment for tuberculosis,
Help but he is not infectious to close or
casual contacts
PasTest Store
He has multidrug-resistant
tuberculosis
My Account He has HIV-associated tuberculosis
Profile
Newsletters
My Career and Exams
Order History Smear-positive tuberculosis means the patient is highly Reference: Normal Values
infectious to both close contacts (more than 8 hours
Learning Goals together per day) and casual contacts, such as other Haematology
Question Filters patients on the ward and healthcare workers. He
therefore needs isolating in a negative-pressure room Immunoglobulins
and contacts should wear particulate masks until he has
Biochemistry
Security received antituberculous therapy for 2 weeks. The
sputum may remain positive after this time, but the Diabetes
Change Password organisms will be dead. Culture-positive tuberculosis
Sign Out means the immediate smear is negative, but prolonged Endocrinology
culture has shown tuberculosis. Smear-positive TB does
not necessarily indicate drug resistance or associated Blood gases
HIV infection. CSF

963

© 2011 PasTest Ltd | About Us | Contact Us | Help

file:///E|/Shakhawan/Respiratory%20S/140a.htm[3/13/2012 4:02:42 PM]


MyPasTest

Main Navigation
Question Browser: MRCP 1
Home
Subscriptions Question Browser Exam Builder Saved Exams
» MRCP 1 Session Progress
• Question Browser
Questions Correct 1
• Timed Test A 40-year-old nurse, who is a UK-born Caucasian, comes
to clinic. She has been coughing for the last 3 months, has Questions Incorrect 0
• Mock Exam
a cavity on her chest radiograph and Mycobacterium Questions Total 1
• Past Papers tuberculosis has been grown from her sputum. HIV testing
Questions Percentage 100 %
• Random Questions is negative. Antibiotic sensitivities are awaited. Which of
the following antibiotic regimes would be optimal? More
• My Performance
• Media Bank Rifampicin, isoniazid
• New Multimedia Rifampicin, isoniazid, pyrazinamide Reference: Normal Values
Online Extras Rifampicin, isoniazid, pyrazinamide and ethambutol
Haematology
Library Rifampicin, isoniazid, pyrazinamide, ethambutol and
Community ciprofloxacin Immunoglobulins
Help Rifampicin, isoniazid, pyrazinamide, ethambutol, Biochemistry
ciprofloxacin, streptomycin
PasTest Store Diabetes
964
Endocrinology
My Account
Blood gases
Profile
CSF
Newsletters
My Career and Exams
Order History
Learning Goals
Question Filters

Security

Change Password
Sign Out

© 2011 PasTest Ltd | About Us | Contact Us | Help

file:///E|/Shakhawan/Respiratory%20S/141.htm[3/13/2012 4:02:44 PM]


MyPasTest

Main Navigation
Question Browser: MRCP 1
Home
Subscriptions Question Browser Exam Builder Saved Exams
» MRCP 1 Session Progress
• Question Browser
Questions Correct 2
• Timed Test A 40-year-old nurse, who is a UK-born Caucasian, comes
to clinic. She has been coughing for the last 3 months, has Questions Incorrect 0
• Mock Exam
a cavity on her chest radiograph and Mycobacterium Questions Total 2
• Past Papers tuberculosis has been grown from her sputum. HIV testing
Questions Percentage 100 %
• Random Questions is negative. Antibiotic sensitivities are awaited. Which of
the following antibiotic regimes would be optimal? More
• My Performance
• Media Bank Rifampicin, isoniazid
• New Multimedia Rifampicin, isoniazid, pyrazinamide
Online Extras Rifampicin, isoniazid, pyrazinamide and Your answer
Library ethambutol
Community Rifampicin, isoniazid, pyrazinamide,
ethambutol and ciprofloxacin
Help
Rifampicin, isoniazid, pyrazinamide,
PasTest Store ethambutol, ciprofloxacin, streptomycin

My Account

Profile
For the initial empiric treatment of tuberculosis (TB),
Newsletters
patients are started on a 4-drug regimen; isoniazid,
My Career and Exams rifampicin, pyrazinamide, and either ethambutol or
Order History streptomycin. Once the isolate is known to be fully Reference: Normal Values
susceptible, ethambutol (or streptomycin if used as a
Learning Goals fourth drug) can be discontinued. After 2 months of Haematology
Question Filters therapy (for a fully susceptible isolate), pyrazinamide
can be stopped. Isoniazid plus rifampicin are continued Immunoglobulins
as daily or intermittent therapy for 4 or more months.
Biochemistry
Security Therapy must be extended if the patient has cavitary
disease or remains culture positive after 2 months of Diabetes
Change Password treatment.
Sign Out Endocrinology
Treatment regimes are different in the case of HIV
related TB, where immunocompromise and also the Blood gases
issue of drug interactions becomes important; this
CSF
requires close specialist management. Extra-pulmonary
TB (e.g. bone, CNS) requires a longer duration of
treatment, usually 9-12 months. The emerging problem
of drug-resistant TB also requires individual
management by a specialist team, as patterns of drug
resistance vary.

964

© 2011 PasTest Ltd | About Us | Contact Us | Help

file:///E|/Shakhawan/Respiratory%20S/141a.htm[3/13/2012 4:02:45 PM]


MyPasTest

Main Navigation
Question Browser: MRCP 1
Home
Subscriptions Question Browser Exam Builder Saved Exams
» MRCP 1 Session Progress
• Question Browser
Questions Correct 2
• Timed Test A 30-year-old man from the Russia is seen in the
emergency department. He was diagnosed with pulmonary Questions Incorrect 0
• Mock Exam
tuberculosis 4 months ago in Russia and is taking rifampicin Questions Total 2
• Past Papers and isoniazid. He comes because of a productive cough,
Questions Percentage 100 %
• Random Questions fevers, weight loss and malaise.
• My Performance More
What would you like to do next?
• Media Bank
• New Multimedia Send a sputum sample and arrange to see him in
out-patients Reference: Normal Values
Online Extras
Admit him to hospital, send a sputum sample and Haematology
Library start him on amoxicillin
Community Admit him to hospital, send a sputum sample and Immunoglobulins
Help start him on pyrazinamide Biochemistry
PasTest Store Admit him to hospital, send a sputum sample and
start him on amoxicillin and pyrazinamide Diabetes

Admit him to hospital, send a sputum sample and Endocrinology


My Account add pyrazinamide, amikacin and ethambutol
Blood gases
Profile
965 CSF
Newsletters
My Career and Exams
Order History
Learning Goals
Question Filters

Security

Change Password
Sign Out

© 2011 PasTest Ltd | About Us | Contact Us | Help

file:///E|/Shakhawan/Respiratory%20S/142.htm[3/13/2012 4:02:47 PM]


MyPasTest

Main Navigation
Question Browser: MRCP 1
Home
Subscriptions Question Browser Exam Builder Saved Exams
» MRCP 1 Session Progress
• Question Browser
Questions Correct 3
• Timed Test A 30-year-old man from the Russia is seen in the
emergency department. He was diagnosed with pulmonary Questions Incorrect 0
• Mock Exam
tuberculosis 4 months ago in Russia and is taking rifampicin Questions Total 3
• Past Papers and isoniazid. He comes because of a productive cough,
Questions Percentage 100 %
• Random Questions fevers, weight loss and malaise.
• My Performance More
What would you like to do next?
• Media Bank
• New Multimedia Send a sputum sample and arrange to
see him in out-patients
Online Extras
Admit him to hospital, send a sputum
Library sample and start him on amoxicillin
Community Admit him to hospital, send a sputum
Help sample and start him on pyrazinamide
PasTest Store Admit him to hospital, send a sputum
sample and start him on amoxicillin and
pyrazinamide
My Account Admit him to hospital, send a sputum Your answer
sample and add pyrazinamide, amikacin
Profile
and ethambutol
Newsletters
My Career and Exams
Order History Reference: Normal Values
Learning Goals The concern with this man is one of multidrug-resistant Haematology
Question Filters tuberculosis. He is failing on his current regime and has
clinical features of active tuberculosis. Treatment failure Immunoglobulins
is usually due to poor compliance, which can lead to the
development of drug resistance. Other risk factors for Biochemistry
Security
resistant disease are: previous anti-TB treatment, HIV Diabetes
Change Password infection and contact with drug-resistant TB. Although
he may have a superimposed simple bacterial infection, Endocrinology
Sign Out
this should not deter from investigating and treating
him for TB, as most of the antibiotics used for TB will Blood gases
cover the usual bacterial chest pathogens. A single drug
CSF
should never be added to a failing TB regime. Add two
or three, ideally ones to which the organism is known to
be sensitive and which the patient has not previously
taken. Send sputum for culture and polymerase chain
reaction (PCR) testing before starting further treatment.
If he has confirmed multidrug-resistant TB, ensure he is
on five or more drugs to which the organism is likely to
be susceptible.

965

© 2011 PasTest Ltd | About Us | Contact Us | Help

file:///E|/Shakhawan/Respiratory%20S/142a.htm[3/13/2012 4:02:48 PM]


MyPasTest

Main Navigation
Question Browser: MRCP 1
Home
Subscriptions Question Browser Exam Builder Saved Exams
» MRCP 1 Session Progress
• Question Browser
Questions Correct 3
• Timed Test A 55-year-old woman attends the chest clinic complaining
of a dry cough she has had for 6 months. It is worse when Questions Incorrect 0
• Mock Exam
she has been walking and when she wakes up in the Questions Total 3
• Past Papers mornings. Examination and chest X-ray are both normal, as
Questions Percentage 100 %
• Random Questions are her pulmonary function tests.
• My Performance More
Which of the following would be most helpful in making a
• Media Bank diagnosis?
• New Multimedia
CT scan of her chest Reference: Normal Values
Online Extras
Serial peak flows Haematology
Library
ENT examination with direct laryngoscopy
Community Immunoglobulins
Trial of high-dose inhaled steroids
Help Biochemistry
Ambulatory oesophageal pH monitoring
PasTest Store Diabetes
967
Endocrinology
My Account
Blood gases
Profile
CSF
Newsletters
My Career and Exams
Order History
Learning Goals
Question Filters

Security

Change Password
Sign Out

© 2011 PasTest Ltd | About Us | Contact Us | Help

file:///E|/Shakhawan/Respiratory%20S/143.htm[3/13/2012 4:02:50 PM]


MyPasTest

Main Navigation
Question Browser: MRCP 1
Home
Subscriptions Question Browser Exam Builder Saved Exams
» MRCP 1 Session Progress
• Question Browser
Questions Correct 3
• Timed Test A 55-year-old woman attends the chest clinic complaining
of a dry cough she has had for 6 months. It is worse when Questions Incorrect 1
• Mock Exam
she has been walking and when she wakes up in the Questions Total 4
• Past Papers mornings. Examination and chest X-ray are both normal, as
Questions Percentage 75 %
• Random Questions are her pulmonary function tests.
• My Performance More
Which of the following would be most helpful in making a
• Media Bank diagnosis?
• New Multimedia
CT scan of her chest
Online Extras
Serial peak flows Your answer
Library
ENT examination with direct
Community laryngoscopy
Help Trial of high-dose inhaled steroids Correct answer
PasTest Store Ambulatory oesophageal pH
monitoring

My Account

Profile
Newsletters Cough-variant asthma represents one end of the
My Career and Exams asthma spectrum, with airway inflammation but minimal
bronchoconstriction. Hence peak flow measurement may Reference: Normal Values
Order History
not be as useful in establishing the diagnosis and
Learning Goals potential benefit of steroids as a trial of steroids itself. Haematology
Question Filters There may not be a typical asthma history; the cough is
typically worse in the mornings, in the cold air and after Immunoglobulins
exercise. Inhaled steroids are beneficial, but
bronchodilators may have little effect. Metacholine Biochemistry
Security
challenge testing may be negative. Spirometry may be Diabetes
Change Password normal with no evidence of bronchodilatation and peak
flows are often stable. Treatment is with high-dose Endocrinology
Sign Out
inhaled steroids for at least 2 months, or a short course
of oral steroids. Response to steroids is helpful when Blood gases
making the diagnosis of asthma. CT scan is not usually
CSF
required unless there is a suspicion of interstitial lung
disease or cancer, or when all other investigations and
treatments have failed. ENT examination may be helpful
if there is a suspicion of a postnasal drip, which along
with reflux and asthma are the three commonest causes
of chronic cough. pH monitoring is performed in cases
of suspected reflux that have not responded to
empirical treatment with a proton-pump inhibitor, or if
diagnostic uncertainty exists.

967

© 2011 PasTest Ltd | About Us | Contact Us | Help

file:///E|/Shakhawan/Respiratory%20S/143a.htm[3/13/2012 4:02:51 PM]


MyPasTest

Main Navigation
Question Browser: MRCP 1
Home
Subscriptions Question Browser Exam Builder Saved Exams
» MRCP 1 Session Progress
• Question Browser
Questions Correct 3
• Timed Test A 60-year-old woman attends the clinic complaining of
shortness of breath over the preceding 2 months. She has Questions Incorrect 1
• Mock Exam
also had problems with nasal irritation, discharge and sinus Questions Total 4
• Past Papers pain. She is known to have asthma, which has recently
Questions Percentage 75 %
• Random Questions been poorly controlled, despite inhaled steroids. Her full
blood count has shown an eosinophilia of 13% and her More
• My Performance
chest X-ray shows peripheral pulmonary shadows.
• Media Bank
What is the most likely diagnosis?
• New Multimedia
Reference: Normal Values
Online Extras Severe asthma
Haematology
Library Allergic bronchopulmonary aspergillosis (ABPA)
Community Churg–Strauss syndrome Immunoglobulins
Help Wegener’s granulomatosis Biochemistry
PasTest Store Cryptogenic organising pneumonia (COP) Diabetes

968 Endocrinology
My Account
Blood gases
Profile
CSF
Newsletters
My Career and Exams
Order History
Learning Goals
Question Filters

Security

Change Password
Sign Out

© 2011 PasTest Ltd | About Us | Contact Us | Help

file:///E|/Shakhawan/Respiratory%20S/144.htm[3/13/2012 4:02:53 PM]


MyPasTest

Main Navigation
Question Browser: MRCP 1
Home
Subscriptions Question Browser Exam Builder Saved Exams
» MRCP 1 Session Progress
• Question Browser
Questions Correct 4
• Timed Test A 60-year-old woman attends the clinic complaining of
shortness of breath over the preceding 2 months. She has Questions Incorrect 1
• Mock Exam
also had problems with nasal irritation, discharge and sinus Questions Total 5
• Past Papers pain. She is known to have asthma, which has recently
Questions Percentage 80 %
• Random Questions been poorly controlled, despite inhaled steroids. Her full
blood count has shown an eosinophilia of 13% and her More
• My Performance
chest X-ray shows peripheral pulmonary shadows.
• Media Bank
What is the most likely diagnosis?
• New Multimedia
Online Extras Severe asthma
Library Allergic bronchopulmonary aspergillosis
Community (ABPA)
Help Churg–Strauss syndrome Your answer
PasTest Store Wegener’s granulomatosis
Cryptogenic organising pneumonia
(COP)
My Account

Profile
Newsletters
My Career and Exams This woman’s eosinophil count is high, higher than it
would be with asthma alone. Eosinophil levels in the Reference: Normal Values
Order History
blood are tightly regulated. Eosinophils accumulate in
Learning Goals allergic or hypersensitivity disease, including asthma Haematology
Question Filters with associated eczema, as well as allergic
bronchopulmonary aspergillosis, Churg–Strauss Immunoglobulins
syndrome, Loeffler’s syndrome, tropical pulmonary
eosinophilia, chronic pulmonary eosinophilia, Biochemistry
Security
hypereosinophilic syndrome and acute eosinophilic Diabetes
Change Password pneumonia. Churg–Strauss syndrome is an eosinophilic
granulomatous inflammation of the respiratory tract Endocrinology
Sign Out
with small- and medium-vessel necrotising vasculitis. It
is diagnosed on finding four out of the following: Blood gases
asthma, blood eosinophilia > 10%, vasculitic
CSF
neuropathy, pulmonary infiltrates, sinus disease or
extravascular eosinophils on biopsy findings. It is
diagnosed clinically, although a biopsy should be sought
for pathological confirmation.

Wegener’s granulomatosis is not typically associated


with an eosinophilia, neither is COP. ABPA is a condition
where people with asthma have a vigorous IgE
response to aspergillus, with associated eosinophilia, a
positive skin-prick test to aspergillus and flitting
consolidation on the chest X-ray. There is no associated
sinus disease.

968

© 2011 PasTest Ltd | About Us | Contact Us | Help

file:///E|/Shakhawan/Respiratory%20S/144a.htm[3/13/2012 4:02:54 PM]


MyPasTest

Main Navigation
Question Browser: MRCP 1
Home
Subscriptions Question Browser Exam Builder Saved Exams
» MRCP 1 Session Progress
• Question Browser
Questions Correct 4
• Timed Test A 40-year-old woman with severe rheumatoid arthritis is
referred to you from the rheumatologists. She has been Questions Incorrect 1
• Mock Exam
short of breath over the last 2 months and her chest X-ray Questions Total 5
• Past Papers shows some widespread patchy shadowing. She has
Questions Percentage 80 %
• Random Questions bilateral inspiratory crepitations and a low grade fever of
• My Performance 37.8 o C. She is taking methotrexate for her arthritis and has More
been for 4 years. You perform a CT scan, which shows
• Media Bank multifocal consolidation.
• New Multimedia
What is the most likely diagnosis? Reference: Normal Values
Online Extras
Haematology
Library Pneumocystis pneumonia
Community Methotrexate pneumonitis Immunoglobulins
Help Tuberculosis Biochemistry
PasTest Store Rheumatoid lung fibrosis Diabetes
Organising pneumonia
Endocrinology
My Account
969 Blood gases
Profile
CSF
Newsletters
My Career and Exams
Order History
Learning Goals
Question Filters

Security

Change Password
Sign Out

© 2011 PasTest Ltd | About Us | Contact Us | Help

file:///E|/Shakhawan/Respiratory%20S/145.htm[3/13/2012 4:02:56 PM]


MyPasTest

Main Navigation
Question Browser: MRCP 1
Home
Subscriptions Question Browser Exam Builder Saved Exams
» MRCP 1 Session Progress
• Question Browser
Questions Correct 5
• Timed Test A 40-year-old woman with severe rheumatoid arthritis is
referred to you from the rheumatologists. She has been Questions Incorrect 1
• Mock Exam
short of breath over the last 2 months and her chest X-ray Questions Total 6
• Past Papers shows some widespread patchy shadowing. She has
Questions Percentage 83 %
• Random Questions bilateral inspiratory crepitations and a low grade fever of
• My Performance 37.8 o C. She is taking methotrexate for her arthritis and has More
been for 4 years. You perform a CT scan, which shows
• Media Bank multifocal consolidation.
• New Multimedia
What is the most likely diagnosis?
Online Extras
Library Pneumocystis pneumonia
Community Methotrexate pneumonitis
Help Tuberculosis
PasTest Store Rheumatoid lung fibrosis
Organising pneumonia Your answer
My Account

Profile
Newsletters
Patients with rheumatological diseases can develop
My Career and Exams pulmonary complications from a number of different
Order History pathologies. Rheumatoid arthritis can affect the lung. Reference: Normal Values
Fibrosis is found in up to 60% of patients with
Learning Goals rheumatoid, but chest X-ray changes in only 5%. Haematology
Question Filters Symptomatic disease is unusual, although it presents
with a dry cough and dyspnoea and may have fine Immunoglobulins
crackles on auscultation. Organising pneumonia occurs Biochemistry
Security in rheumatoid arthritis, with fever, dyspnoea and
multifocal consolidation. This responds dramatically to Diabetes
Change Password steroids. Opportunistic pulmonary infection from
Sign Out immunosuppressive drugs can occur, but neither Endocrinology
pneumocystis pneumonia (PCP) nor tuberculosis would
Blood gases
give these CT appearances. Methotrexate-induced
pneumonitis usually occurs within 4 months of starting CSF
the drug. CT shows pulmonary infiltrates.

969

© 2011 PasTest Ltd | About Us | Contact Us | Help

file:///E|/Shakhawan/Respiratory%20S/145a.htm[3/13/2012 4:02:57 PM]


MyPasTest

Main Navigation
Question Browser: MRCP 1
Home
Subscriptions Question Browser Exam Builder Saved Exams
» MRCP 1 Session Progress
• Question Browser
Questions Correct 5
• Timed Test You are asked to look at a Heaf test performed 1 week ago
on a man. His wife is on the ward with pulmonary Questions Incorrect 1
• Mock Exam
tuberculosis. He is asymptomatic and has a normal chest Questions Total 6
• Past Papers radiograph. He has had a previous BCG vaccination. The
Questions Percentage 83 %
• Random Questions dots are joining up to make a faint red ring. What does
this mean? More
• My Performance
• Media Bank The man has tuberculosis
• New Multimedia The man has previously had a BCG vaccination Reference: Normal Values
Online Extras The man does not have tuberculosis and has not had
a BCG vaccination Haematology
Library
Community This man has had a BCG vaccination and has Immunoglobulins
tuberculosis
Help Biochemistry
The man has HIV, but does not have tuberculosis
PasTest Store Diabetes
970
Endocrinology
My Account
Blood gases
Profile
CSF
Newsletters
My Career and Exams
Order History
Learning Goals
Question Filters

Security

Change Password
Sign Out

© 2011 PasTest Ltd | About Us | Contact Us | Help

file:///E|/Shakhawan/Respiratory%20S/146.htm[3/13/2012 4:02:58 PM]


MyPasTest

Main Navigation
Question Browser: MRCP 1
Home
Subscriptions Question Browser Exam Builder Saved Exams
» MRCP 1 Session Progress
• Question Browser
Questions Correct 6
• Timed Test You are asked to look at a Heaf test performed 1 week ago
on a man. His wife is on the ward with pulmonary Questions Incorrect 1
• Mock Exam
tuberculosis. He is asymptomatic and has a normal chest Questions Total 7
• Past Papers radiograph. He has had a previous BCG vaccination. The
Questions Percentage 85 %
• Random Questions dots are joining up to make a faint red ring. What does
this mean? More
• My Performance
• Media Bank The man has tuberculosis
• New Multimedia The man has previously had a BCG Your answer
Online Extras vaccination
Library The man does not have tuberculosis
and has not had a BCG vaccination
Community
This man has had a BCG vaccination
Help and has tuberculosis
PasTest Store The man has HIV, but does not have
tuberculosis
My Account

Profile
Newsletters
Heaf tests are graded as follows:
My Career and Exams
Order History 0 no reaction Reference: Normal Values
Learning Goals 1 4–6 small dots Haematology
Question Filters 2 dots coalesce, normal skin in centre
Immunoglobulins
3 dots coalesce, central skin filled in
Biochemistry
Security solid induration > 10 mm with or without
4 vesiculation or ulceration Diabetes
Change Password
Sign Out Grade 2 is a normal response in the presence of a Endocrinology
previous BCG vaccination. Without prior BCG Blood gases
vaccination in an HIV-negative patient, useful results
are 0, indicating no TB or 3–4 indicating active disease. CSF

970

© 2011 PasTest Ltd | About Us | Contact Us | Help

file:///E|/Shakhawan/Respiratory%20S/146a.htm[3/13/2012 4:03:00 PM]


MyPasTest

Main Navigation
Question Browser: MRCP 1
Home
Subscriptions Question Browser Exam Builder Saved Exams
» MRCP 1 Session Progress
• Question Browser
Questions Correct 6
• Timed Test You are called to see a 50-year-old woman who is having
difficulty breathing following a laparoscopic Questions Incorrect 1
• Mock Exam
cholecystectomy. She is making a lot of noisy inspiratory Questions Total 7
• Past Papers effort with stridor. You notice she is taking warfarin long
Questions Percentage 85 %
• Random Questions term for thromboembolic disease, salbutamol and inhaled
steroids for asthma and penicillamine for severe rheumatoid More
• My Performance
arthritis.
• Media Bank
Which test might be the most helpful in diagnosing
• New Multimedia
her current problem? Reference: Normal Values
Online Extras
Peak flow Haematology
Library
Community Spirometry with transfer factor measurement Immunoglobulins
Help Spirometry with flow volume loops Biochemistry
PasTest Store Chest X-ray
Diabetes
CT chest
Endocrinology
My Account 971
Blood gases
Profile
CSF
Newsletters
My Career and Exams
Order History
Learning Goals
Question Filters

Security

Change Password
Sign Out

© 2011 PasTest Ltd | About Us | Contact Us | Help

file:///E|/Shakhawan/Respiratory%20S/147.htm[3/13/2012 4:03:01 PM]


MyPasTest

Main Navigation
Question Browser: MRCP 1
Home
Subscriptions Question Browser Exam Builder Saved Exams
» MRCP 1 Session Progress
• Question Browser
Questions Correct 7
• Timed Test You are called to see a 50-year-old woman who is having
difficulty breathing following a laparoscopic Questions Incorrect 1
• Mock Exam
cholecystectomy. She is making a lot of noisy inspiratory Questions Total 8
• Past Papers effort with stridor. You notice she is taking warfarin long
Questions Percentage 87 %
• Random Questions term for thromboembolic disease, salbutamol and inhaled
steroids for asthma and penicillamine for severe rheumatoid More
• My Performance
arthritis.
• Media Bank
Which test might be the most helpful in diagnosing
• New Multimedia
her current problem?
Online Extras
Library Peak flow
Community Spirometry with transfer factor
measurement
Help
Spirometry with flow volume loops Your answer
PasTest Store
Chest X-ray
CT chest
My Account

Profile
Newsletters
My Career and Exams This woman has stridor due to cricoarytenoid arthritis.
This is seen in studies in up to 75% of patients with Reference: Normal Values
Order History
rheumatoid arthritis. It can cause sore throat, hoarse
Learning Goals voice and stridor, but is often asymptomatic. However, Haematology
Question Filters symptoms may rapidly worsen in the postoperative
period. It is unrelated to any lung fibrosis. Flow volume Immunoglobulins
loop can be abnormal, as can direct laryngoscopy and
HRCT (high-resolution CT scan) of the larynx. It may Biochemistry
Security
need urgent tracheostomy and steroids, both oral and Diabetes
Change Password joint injection. The presence of stridor makes the
possibility of an acute exacerbation of asthma or a new Endocrinology
Sign Out
pulmonary embolus less likely.
Blood gases

CSF
971

© 2011 PasTest Ltd | About Us | Contact Us | Help

file:///E|/Shakhawan/Respiratory%20S/147a.htm[3/13/2012 4:03:03 PM]


MyPasTest

Main Navigation
Question Browser: MRCP 1
Home
Subscriptions Question Browser Exam Builder Saved Exams
» MRCP 1 Session Progress
• Question Browser
Questions Correct 7
• Timed Test You have a Caucasian patient with cystic fibrosis on the
ward. His 20-year-old sister does not have cystic fibrosis, Questions Incorrect 1
• Mock Exam
but comes to ask you about the chances of her having Questions Total 8
• Past Papers children with cystic fibrosis. They have the same parents.
Questions Percentage 87 %
• Random Questions Which of the following statements is correct?
• My Performance More
She has a 1 in 4 chance of being a carrier of the
• Media Bank cystic fibrosis gene
• New Multimedia She is not a carrier of the cystic fibrosis gene Reference: Normal Values
Online Extras If she does not have the DF508 deletion she is not a
cystic fibrosis carrier Haematology
Library
Community Genetic testing is not necessary to assign carrier Immunoglobulins
status
Help Biochemistry
Her Caucasian partner has a 1 in 25 chance of being
PasTest Store a cystic fibrosis carrier Diabetes

973 Endocrinology
My Account
Blood gases
Profile
CSF
Newsletters
My Career and Exams
Order History
Learning Goals
Question Filters

Security

Change Password
Sign Out

© 2011 PasTest Ltd | About Us | Contact Us | Help

file:///E|/Shakhawan/Respiratory%20S/148.htm[3/13/2012 4:03:04 PM]


MyPasTest

Main Navigation
Question Browser: MRCP 1
Home
Subscriptions Question Browser Exam Builder Saved Exams
» MRCP 1 Session Progress
• Question Browser
Questions Correct 7
• Timed Test You have a Caucasian patient with cystic fibrosis on the
ward. His 20-year-old sister does not have cystic fibrosis, Questions Incorrect 2
• Mock Exam
but comes to ask you about the chances of her having Questions Total 9
• Past Papers children with cystic fibrosis. They have the same parents.
Questions Percentage 77 %
• Random Questions Which of the following statements is correct?
• My Performance More
She has a 1 in 4 chance of being a
• Media Bank carrier of the cystic fibrosis gene
• New Multimedia She is not a carrier of the cystic
Online Extras fibrosis gene
Library If she does not have the DF508 Your answer
deletion she is not a cystic fibrosis
Community carrier
Help Genetic testing is not necessary to
PasTest Store assign carrier status
Her Caucasian partner has a 1 in 25 Correct answer
chance of being a cystic fibrosis
My Account carrier
Profile
Newsletters
My Career and Exams
Order History The carrier frequency for cystic fibrosis (CF) in Reference: Normal Values
Caucasians is 1 in 25. The CF gene is inherited
Learning Goals autosomal recessively. Therefore a sibling of an affected Haematology
Question Filters person has a 3 in 4 chance of being a CF carrier or
having the disease. In the case of this patient's sister, Immunoglobulins
she knows she doesn't have CF, so her chance of being
Biochemistry
Security a carrier for the CF gene is 2/3. CF accounts for 1 in
2500 live births. CF is much rarer in patients of Afro– Diabetes
Change Password Caribbean and Asian origin. There are more than 1000
Sign Out different mutations recognised in the gene for the cystic Endocrinology
fibrosis transmembrane conductance regulator. The
most common is DF508, which accounts for around Blood gases
67% of CF alleles in the UK and northern Europe: 85% CSF
of CF alleles can be screened for genetically. Genetic
testing would be necessary to assign carrier status,
although the diagnosis of cystic fibrosis could be made
clinically with sweat testing on someone who is
homozygous.

973

© 2011 PasTest Ltd | About Us | Contact Us | Help

file:///E|/Shakhawan/Respiratory%20S/148a.htm[3/13/2012 4:03:06 PM]


MyPasTest

Main Navigation
Question Browser: MRCP 1
Home
Subscriptions Question Browser Exam Builder Saved Exams
» MRCP 1 Session Progress
• Question Browser
Questions Correct 7
• Timed Test A 70-year-old man with chronic obstructive pulmonary
disease and bronchiectasis had a sputum sample sent from Questions Incorrect 2
• Mock Exam
clinic because he had been more breathless for 6 months Questions Total 9
• Past Papers and was coughing purulent sputum. The result has shown
Questions Percentage 77 %
• Random Questions the presence of Mycobacterium malmoense. What does this
mean? More
• My Performance
• Media Bank He should have further sputum samples sent for
• New Multimedia culture
Reference: Normal Values
Online Extras He needs admitting to hospital and isolating in a side
room Haematology
Library
Public health authorities should be informed to begin
Community Immunoglobulins
contact tracing
Help He should be started on triple-drug antituberculous Biochemistry
PasTest Store therapy
Diabetes
He should be started on quadruple-drug
antituberculous therapy Endocrinology
My Account
Blood gases
Profile 974
CSF
Newsletters
My Career and Exams
Order History
Learning Goals
Question Filters

Security

Change Password
Sign Out

© 2011 PasTest Ltd | About Us | Contact Us | Help

file:///E|/Shakhawan/Respiratory%20S/149.htm[3/13/2012 4:03:07 PM]


MyPasTest

Main Navigation
Question Browser: MRCP 1
Home
Subscriptions Question Browser Exam Builder Saved Exams
» MRCP 1 Session Progress
• Question Browser
Questions Correct 7
• Timed Test A 70-year-old man with chronic obstructive pulmonary
disease and bronchiectasis had a sputum sample sent from Questions Incorrect 3
• Mock Exam
clinic because he had been more breathless for 6 months Questions Total 10
• Past Papers and was coughing purulent sputum. The result has shown
Questions Percentage 70 %
• Random Questions the presence of Mycobacterium malmoense. What does this
mean? More
• My Performance
• Media Bank He should have further sputum Correct answer
• New Multimedia samples sent for culture
Online Extras He needs admitting to hospital and
isolating in a side room
Library
Public health authorities should be Your answer
Community informed to begin contact tracing
Help He should be started on triple-drug
PasTest Store antituberculous therapy
He should be started on quadruple-
drug antituberculous therapy
My Account

Profile
Newsletters
My Career and Exams Mycobacterium malmoense is a non-tuberculous
Order History mycobacterium and an environmental low-grade Reference: Normal Values
pathogen in humans. This organism most commonly
Learning Goals causes pulmonary infection in middle-aged and older Haematology
Question Filters adults with pre-existing lung disease or
immunodeficiency. M. malmoense can also cause local Immunoglobulins
invasion from a skin lesion. It causes non-specific
Biochemistry
Security symptoms, such as malaise and weight loss, or chest
disease not progressing as expected. Chest X-ray Diabetes
Change Password appearances can be indistinguishable from those seen
Sign Out in patients with Mycobacterium tuberculosis infection, Endocrinology
with upper zone fibrosis and cavitation. A single isolate
from a non-sterile site may not be significant and may Blood gases
represent contamination. More than two isolates from a CSF
non-sterile site are required to establish disease. There
is a low-risk of cross-infection, so there is no need to
isolate patients or notify public health authorities. There
are BTS guidelines on the treatment of each of the
different non-tuberculous mycobacteria, but this is with
either two or three antituberculous drugs (see
Subcommittee of the Joint Tuberculosis Committee of
the British Thoracic Society 2000. Management of
opportunistic mycobacterial infections.
Thorax,55(3),210–18).

974

© 2011 PasTest Ltd | About Us | Contact Us | Help

file:///E|/Shakhawan/Respiratory%20S/149a.htm[3/13/2012 4:03:09 PM]


MyPasTest

Main Navigation
Question Browser: MRCP 1
Home
Subscriptions Question Browser Exam Builder Saved Exams
» MRCP 1 Session Progress
• Question Browser
Questions Correct 7
• Timed Test A 23-year-old woman who is 34 weeks’ pregnant is
admitted to the emergency department with shortness of Questions Incorrect 3
• Mock Exam
breath, cough and wheeze. She is known to have moderate Questions Total 10
• Past Papers
asthma and normally takes fluticasone 750 μg salmeterol Questions Percentage 70 %
• Random Questions twice a day and salbutamol as needed. Her peak flow is
• My Performance usually 450. She has been deteriorating over the past 3 More
days despite increasing her fluticasone and starting 40 mg
• Media Bank oral prednisolone daily. She is now unable to speak in
• New Multimedia sentences, her peak flow is less than 150 l/min and her
pulse rate is 130/minute. Saturations are 96% and you Reference: Normal Values
Online Extras
hear a widespread wheeze on examination. Her p(O2 ) is 15
Haematology
Library kPa and p(CO 2 ) 4.5 kPa. She has been given high-flow
Community oxygen, repeated nebulised salbutamol and Atrovent. Immunoglobulins
Help Biochemistry
Which of the following would you consider to be
PasTest Store appropriate next? Diabetes

Liase with the obstetricians regarding emergency Endocrinology


My Account section
Blood gases
Profile Intravenous aminophylline
CSF
Newsletters Intravenous hydrocortisone
My Career and Exams Non-invasive ventilation
Order History ITU review with reference to possible intubation and
ventilation
Learning Goals
Question Filters 975

Security

Change Password
Sign Out

© 2011 PasTest Ltd | About Us | Contact Us | Help

file:///E|/Shakhawan/Respiratory%20S/150.htm[3/13/2012 4:03:10 PM]


MyPasTest

Main Navigation
Question Browser: MRCP 1
Home
Subscriptions Question Browser Exam Builder Saved Exams
» MRCP 1 Session Progress
• Question Browser
Questions Correct 7
• Timed Test A 23-year-old woman who is 34 weeks’ pregnant is
admitted to the emergency department with shortness of Questions Incorrect 4
• Mock Exam
breath, cough and wheeze. She is known to have moderate Questions Total 11
• Past Papers
asthma and normally takes fluticasone 750 μg salmeterol Questions Percentage 63 %
• Random Questions twice a day and salbutamol as needed. Her peak flow is
• My Performance usually 450. She has been deteriorating over the past 3 More
days despite increasing her fluticasone and starting 40 mg
• Media Bank oral prednisolone daily. She is now unable to speak in
• New Multimedia sentences, her peak flow is less than 150 l/min and her
pulse rate is 130/minute. Saturations are 96% and you
Online Extras
hear a widespread wheeze on examination. Her p(O2 ) is 15
Library kPa and p(CO 2 ) 4.5 kPa. She has been given high-flow
Community oxygen, repeated nebulised salbutamol and Atrovent.
Help
Which of the following would you consider to be
PasTest Store appropriate next?

Liase with the obstetricians


My Account regarding emergency section
Profile Intravenous aminophylline Your answer
Newsletters Intravenous hydrocortisone
My Career and Exams Non-invasive ventilation
Order History ITU review with reference to Correct answer Reference: Normal Values
possible intubation and ventilation
Learning Goals Haematology
Question Filters
Immunoglobulins
Biochemistry
Security Drug therapy for the pregnant woman should be
delivered as for the non-pregnant patient. Saturations Diabetes
Change Password
should be maintained above 95% with oxygen and close Endocrinology
Sign Out monitoring essential. Whilst intravenous aminophylline
used to be recommended, evidence no longer supports Blood gases
its use. She has had adequate doses of oral steroids, so
does not need iv hydrocortisone. It is of paramount CSF
importance to stabilise her and an emergency section
would not be in her best interests. She has features of
acute severe asthma, and intubation and ventilation
should be considered at this point. There is minimal
evidence at present to support the use of non-invasive
ventilation in acute asthma (see British Guideline on the
Management of Asthma 2003. Thorax,58,Suppl. 1).

975

© 2011 PasTest Ltd | About Us | Contact Us | Help

file:///E|/Shakhawan/Respiratory%20S/150a.htm[3/13/2012 4:03:12 PM]


MyPasTest

Main Navigation
Question Browser: MRCP 1
Home
Subscriptions Question Browser Exam Builder Saved Exams
» MRCP 1 Session Progress
• Question Browser
Questions Correct 7
• Timed Test A 29-year-old woman with asthma is seen on the ward
round. She was admitted the previous day with anaphylaxis Questions Incorrect 4
• Mock Exam
due to latex exposure. She asks you about immunotherapy Questions Total 11
• Past Papers for anaphylaxis. What do you tell her?
Questions Percentage 63 %
• Random Questions
• My Performance This is a well-established treatment for anaphylaxis More
• Media Bank This is a universally safe treatment for anaphylaxis

• New Multimedia This will give her protection against anaphylactic


episodes for about 15 years Reference: Normal Values
Online Extras
This is useful if she cannot avoid latex exposure Haematology
Library
It takes 2 months to complete the treatment course
Community Immunoglobulins
Help 976 Biochemistry
PasTest Store Diabetes
Endocrinology
My Account
Blood gases
Profile
CSF
Newsletters
My Career and Exams
Order History
Learning Goals
Question Filters

Security

Change Password
Sign Out

© 2011 PasTest Ltd | About Us | Contact Us | Help

file:///E|/Shakhawan/Respiratory%20S/151.htm[3/13/2012 4:03:13 PM]


MyPasTest

Main Navigation
Question Browser: MRCP 1
Home
Subscriptions Question Browser Exam Builder Saved Exams
» MRCP 1 Session Progress
• Question Browser
Questions Correct 8
• Timed Test A 29-year-old woman with asthma is seen on the ward
round. She was admitted the previous day with anaphylaxis Questions Incorrect 4
• Mock Exam
due to latex exposure. She asks you about immunotherapy Questions Total 12
• Past Papers for anaphylaxis. What do you tell her?
Questions Percentage 66 %
• Random Questions
• My Performance This is a well-established treatment for More
anaphylaxis
• Media Bank
This is a universally safe treatment for
• New Multimedia anaphylaxis
Online Extras This will give her protection against
Library anaphylactic episodes for about 15
years
Community
This is useful if she cannot avoid latex Your answer
Help exposure
PasTest Store It takes 2 months to complete the
treatment course
My Account

Profile
Newsletters
Allergen immunotherapy aims to desensitise a person to
My Career and Exams the allergen they are allergic to. It is useful in those
Order History people who cannot avoid exposure to the allergen. Reference: Normal Values
Small amounts of the allergen are injected weekly, with
Learning Goals slowly increasing dose strengths until the maximum Haematology
Question Filters dose of the allergen is administered. This can take up
to 12 months. It can protect against anaphylaxis for 3– Immunoglobulins
5 years, but ‘top-up’ doses are necessary. Allergen
Biochemistry
Security administration can cause anaphylaxis and deaths have
been reported. It is only performed in some centres and Diabetes
Change Password experience is increasing. Centres may not treat people
Sign Out with chronic asthma, due to the risks of Endocrinology
bronchoconstriction.
Blood gases

CSF
976

© 2011 PasTest Ltd | About Us | Contact Us | Help

file:///E|/Shakhawan/Respiratory%20S/151a.htm[3/13/2012 4:03:15 PM]


MyPasTest

Main Navigation
Question Browser: MRCP 1
Home
Subscriptions Question Browser Exam Builder Saved Exams
» MRCP 1 Session Progress
• Question Browser
Questions Correct 8
• Timed Test You see a 70-year-old woman in clinic with COPD. She is
breathless when walking around her house and garden and Questions Incorrect 4
• Mock Exam
is a current smoker of 10 cigarettes per day. She has an Questions Total 12
• Past Papers FEV1 of 1.2 litres (40% predicted) and an FVC of 2.0 litres
Questions Percentage 66 %
• Random Questions (50% predicted). She had minimal bronchodilator
• My Performance reversibility following nebulised salbutamol. Her oxygen More
saturations are 93% on air and she takes salbutamol only
• Media Bank as needed. What would be the next treatment option
• New Multimedia for her?
Reference: Normal Values
Online Extras
Anticholinergic inhaler Haematology
Library
Inhaled steroids
Community Immunoglobulins
Oral leukotriene-receptor antagonist
Help Biochemistry
Oral theophylline
PasTest Store Diabetes
Long-term domiciliary oxygen
Endocrinology
My Account 977
Blood gases
Profile
CSF
Newsletters
My Career and Exams
Order History
Learning Goals
Question Filters

Security

Change Password
Sign Out

© 2011 PasTest Ltd | About Us | Contact Us | Help

file:///E|/Shakhawan/Respiratory%20S/152.htm[3/13/2012 4:03:16 PM]


MyPasTest

Main Navigation
Question Browser: MRCP 1
Home
Subscriptions Question Browser Exam Builder Saved Exams
» MRCP 1 Session Progress
• Question Browser
Questions Correct 9
• Timed Test You see a 70-year-old woman in clinic with COPD. She is
breathless when walking around her house and garden and Questions Incorrect 4
• Mock Exam
is a current smoker of 10 cigarettes per day. She has an Questions Total 13
• Past Papers FEV1 of 1.2 litres (40% predicted) and an FVC of 2.0 litres
Questions Percentage 69 %
• Random Questions (50% predicted). She had minimal bronchodilator
• My Performance reversibility following nebulised salbutamol. Her oxygen More
saturations are 93% on air and she takes salbutamol only
• Media Bank as needed. What would be the next treatment option
• New Multimedia for her?
Online Extras
Anticholinergic inhaler Your answer
Library
Inhaled steroids
Community
Oral leukotriene-receptor antagonist
Help
Oral theophylline
PasTest Store
Long-term domiciliary oxygen

My Account

Profile
Newsletters This woman has moderate COPD based on her
spirometry and is now symptomatic. Clearly she needs
My Career and Exams to stop smoking, but the next treatment would be an
Order History anticholinergic inhaler. After this, she could have a Reference: Normal Values
steroid trial to assess whether she would benefit from
Learning Goals Haematology
long-term inhaled steroids, which she may do if she has
Question Filters a > 15% improvement in her FEV1 , or has frequent
Immunoglobulins
exacerbations, or were to develop severe COPD. Oral
leukotriene-receptor antagonists are not used in the Biochemistry
Security management of COPD. Oral theophylline is used, but
after adequate inhaled therapy is established. Oral Diabetes
Change Password
theophyllines are favoured less in recent COPD
Sign Out Endocrinology
guidelines. She does not fulfil the requirements for long-
term oxygen therapy, as she not only smokes but her Blood gases
oxygen saturations are satisfactory.
CSF

977

© 2011 PasTest Ltd | About Us | Contact Us | Help

file:///E|/Shakhawan/Respiratory%20S/152a.htm[3/13/2012 4:03:18 PM]


MyPasTest

Main Navigation
Question Browser: MRCP 1
Home
Subscriptions Question Browser Exam Builder Saved Exams
» MRCP 1 Session Progress
• Question Browser
Questions Correct 9
• Timed Test A 70-year-old man attends clinic. He is an ex-smoker of 50
pack years. He has COPD, with an FEV1 of 40% predicted Questions Incorrect 4
• Mock Exam
and minimal bronchodilator reversibility. He is breathless Questions Total 13
• Past Papers
after walking 500 metres and was keen to be referred to Questions Percentage 69 %
• Random Questions discuss the possibility of pulmonary rehabilitation. What do
• My Performance you tell him? More
• Media Bank His exercise tolerance is too poor to be considered
• New Multimedia for the rehabilitation programme
Reference: Normal Values
Online Extras Rehabilitation will make little difference to the length
of any future hospital stays Haematology
Library
Following the rehabilitation programme, his lung Immunoglobulins
Community
function should improve
Help Biochemistry
Following the rehabilitation programme, his walking
PasTest Store distance should improve Diabetes
If his exercise tolerance did improve following
Endocrinology
My Account pulmonary rehabilitation, this would be a long-lasting
improvement Blood gases
Profile
978 CSF
Newsletters
My Career and Exams
Order History
Learning Goals
Question Filters

Security

Change Password
Sign Out

© 2011 PasTest Ltd | About Us | Contact Us | Help

file:///E|/Shakhawan/Respiratory%20S/153.htm[3/13/2012 4:03:19 PM]


MyPasTest

Main Navigation
Question Browser: MRCP 1
Home
Subscriptions Question Browser Exam Builder Saved Exams
» MRCP 1 Session Progress
• Question Browser
Questions Correct 10
• Timed Test A 70-year-old man attends clinic. He is an ex-smoker of 50
pack years. He has COPD, with an FEV1 of 40% predicted Questions Incorrect 4
• Mock Exam
and minimal bronchodilator reversibility. He is breathless Questions Total 14
• Past Papers
after walking 500 metres and was keen to be referred to Questions Percentage 71 %
• Random Questions discuss the possibility of pulmonary rehabilitation. What do
• My Performance you tell him? More
• Media Bank His exercise tolerance is too poor to be
• New Multimedia considered for the rehabilitation
programme
Online Extras
Rehabilitation will make little difference
Library
to the length of any future hospital
Community stays
Help Following the rehabilitation
PasTest Store programme, his lung function should
improve
Following the rehabilitation Your answer
My Account programme, his walking distance should
improve
Profile
If his exercise tolerance did improve
Newsletters following pulmonary rehabilitation, this
My Career and Exams would be a long-lasting improvement
Order History Reference: Normal Values
Learning Goals Haematology
Question Filters
Immunoglobulins
There have been many randomised controlled trials to
establish the effects of pulmonary rehabilitation. This is Biochemistry
Security a programme of aerobic lower extremity training
combined with education, which has been found to Diabetes
Change Password provide significant improvements in functional exercise,
Sign Out although little change in lung function testing. Patients Endocrinology
who have completed pulmonary rehabilitation have no Blood gases
fewer hospital admissions because of chest problems,
but their hospital stays are likely to be shorter. Decline CSF
in exercise tolerance and heath status occurs between 6
and 12 months after the completion of a course. The
effect of sustained improvement with ongoing
rehabilitation has yet to be evaluated.

978

© 2011 PasTest Ltd | About Us | Contact Us | Help

file:///E|/Shakhawan/Respiratory%20S/153a.htm[3/13/2012 4:03:21 PM]


MyPasTest

Main Navigation
Question Browser: MRCP 1
Home
Subscriptions Question Browser Exam Builder Saved Exams
» MRCP 1 Session Progress
• Question Browser
Questions Correct 10
• Timed Test A 60-year-old man who has a 30-pack year smoking
history comes to clinic with worsening shortness of breath Questions Incorrect 4
• Mock Exam
over the last 6 months. He works as a baker and keeps Questions Total 14
• Past Papers racing pigeons. On examination, he is clubbed, has
Questions Percentage 71 %
• Random Questions saturations of 91% on air and has widespread fine
inspiratory crepitations. His chest radiograph shows More
• My Performance
reticulonodular shadowing and his CT scan confirms
• Media Bank reticulation, mainly subpleural and some honeycombing.
• New Multimedia What is the diagnosis?
Reference: Normal Values
Online Extras Pulmonary sarcoidosis
Haematology
Library Usual interstitial pneumonitis
Community Immunoglobulins
Hypersensitivity pneumonitis
Help Occupational asthma Biochemistry
PasTest Store Langerhans’ cell histiocytosis Diabetes

979 Endocrinology
My Account
Blood gases
Profile
CSF
Newsletters
My Career and Exams
Order History
Learning Goals
Question Filters

Security

Change Password
Sign Out

© 2011 PasTest Ltd | About Us | Contact Us | Help

file:///E|/Shakhawan/Respiratory%20S/154.htm[3/13/2012 4:03:23 PM]


MyPasTest

Main Navigation
Question Browser: MRCP 1
Home
Subscriptions Question Browser Exam Builder Saved Exams
» MRCP 1 Session Progress
• Question Browser
Questions Correct 1
• Timed Test A 60-year-old man who has a 30-pack year smoking
history comes to clinic with worsening shortness of breath Questions Incorrect 0
• Mock Exam
over the last 6 months. He works as a baker and keeps Questions Total 1
• Past Papers racing pigeons. On examination, he is clubbed, has
Questions Percentage 100 %
• Random Questions saturations of 91% on air and has widespread fine
inspiratory crepitations. His chest radiograph shows More
• My Performance
reticulonodular shadowing and his CT scan confirms
• Media Bank reticulation, mainly subpleural and some honeycombing.
• New Multimedia What is the diagnosis?

Online Extras Pulmonary sarcoidosis


Library Usual interstitial pneumonitis Your answer
Community Hypersensitivity pneumonitis
Help Occupational asthma
PasTest Store Langerhans’ cell histiocytosis

My Account

Profile
Dyspnoea, clubbing and inspiratory crepitations are the
Newsletters
classic features of usual interstitial pneumonitis. Chest
My Career and Exams X-ray will show reticulation, which is classically
Order History subpleural in distribution on CT. Pulmonary sarcoidosis Reference: Normal Values
does not give clubbing and crepitations would be
Learning Goals associated with end-stage fibrotic disease. The CT Haematology
Question Filters appearances would be those of nodularity, including
fissural nodularity. The CT in hypersensitivity Immunoglobulins
pneumonitis shows: ground-glass shadowing, with
Biochemistry
Security reticular and nodular patterns; in occupational asthma,
possibly non-specific features of air trapping; and in Diabetes
Change Password Langerhans’cell histiocytosis it shows nodules and cystic
Sign Out lesions. Endocrinology

Blood gases
979 CSF

© 2011 PasTest Ltd | About Us | Contact Us | Help

file:///E|/Shakhawan/Respiratory%20S/154a.htm[3/13/2012 4:03:24 PM]


MyPasTest

Main Navigation
Question Browser: MRCP 1
Home
Subscriptions Question Browser Exam Builder Saved Exams
» MRCP 1 Session Progress
• Question Browser
Questions Correct 0
• Timed Test A 40-year-old Afro–Caribbean woman presents to clinic
following an insurance medical examination, where she was Questions Incorrect 0
• Mock Exam
found to have bilateral hilar lymphadenopathy on her chest Questions Total 0
• Past Papers radiograph. Her CT scan confirms hilar lymphadenopathy
Questions Percentage 0%
• Random Questions and comments on nodularity in the lung fields.
• My Performance More
What would you do next?
• Media Bank
• New Multimedia Arrange a trans-bronchial, lymph node needle
aspiration Reference: Normal Values
Online Extras
Arrange a lung biopsy Haematology
Library
Commence oral prednisolone
Community Immunoglobulins
Send sputum samples to microbiology
Help Biochemistry
Observe and repeat her chest X-ray in 3 months
PasTest Store Diabetes
980
Endocrinology
My Account
Blood gases
Profile
CSF
Newsletters
My Career and Exams
Order History
Learning Goals
Question Filters

Security

Change Password
Sign Out

© 2011 PasTest Ltd | About Us | Contact Us | Help

file:///E|/Shakhawan/Respiratory%20S/155.htm[3/13/2012 4:03:26 PM]


MyPasTest

Main Navigation
Question Browser: MRCP 1
Home
Subscriptions Question Browser Exam Builder Saved Exams
» MRCP 1 Session Progress
• Question Browser
Questions Correct 1
• Timed Test A 40-year-old Afro–Caribbean woman presents to clinic
following an insurance medical examination, where she was Questions Incorrect 0
• Mock Exam
found to have bilateral hilar lymphadenopathy on her chest Questions Total 1
• Past Papers radiograph. Her CT scan confirms hilar lymphadenopathy
Questions Percentage 100 %
• Random Questions and comments on nodularity in the lung fields.
• My Performance More
What would you do next?
• Media Bank
• New Multimedia Arrange a trans-bronchial, lymph node
needle aspiration
Online Extras
Arrange a lung biopsy
Library
Commence oral prednisolone
Community
Send sputum samples to microbiology
Help
Observe and repeat her chest X-ray in Your answer
PasTest Store 3 months

My Account

Profile
This woman has typical features of pulmonary and
Newsletters lymph node sarcoid. If the history and radiology is
My Career and Exams typical, the diagnosis of sarcoidosis can be confidently
made and there may not be a need to proceed to tissue Reference: Normal Values
Order History
biopsy. If there were atypical features and tissue biopsy
Learning Goals was required, either trans-bronchial or open lung biopsy Haematology
Question Filters is preferable. Trans-bronchial, lymph node aspiration is
useful for assessing metastatic spread to lymph nodes, Immunoglobulins
but is not a good test for non-malignant conditions. In
the absence of symptoms, she does not need steroid Biochemistry
Security
treatment. She does not need sputum sending for Diabetes
Change Password culture as there is no suspicion of tuberculosis. She can
be followed up and observed radiologically at regular Endocrinology
Sign Out
intervals. Some 50% of patients with stage 2
sarcoidosis, ie hilar lymphadenopathy and parenchymal Blood gases
infiltrate, recover spontaneously in 2 years.
CSF

980

© 2011 PasTest Ltd | About Us | Contact Us | Help

file:///E|/Shakhawan/Respiratory%20S/155a.htm[3/13/2012 4:03:27 PM]


MyPasTest

Main Navigation
Question Browser: MRCP 1
Home
Subscriptions Question Browser Exam Builder Saved Exams
» MRCP 1 Session Progress
• Question Browser
Questions Correct 1
• Timed Test A 30-year-old woman who is a non-smoker comes to clinic
with a 1-year history of a dry cough and most recently Questions Incorrect 0
• Mock Exam
haemoptysis. Examination is normal, but her chest Questions Total 1
• Past Papers radiograph shows a smooth lesion at the origin of her right
Questions Percentage 100 %
• Random Questions lower lobe bronchus. CT scan confirms this and is otherwise
normal. You perform the bronchoscopy and see a smooth More
• My Performance
red tumour. What would you do next?
• Media Bank
• New Multimedia Biopsy the tumour
Reference: Normal Values
Online Extras Discuss her with the thoracic surgeons with a view to
biopsy Haematology
Library
Discuss her with the thoracic surgeons with a view to
Community Immunoglobulins
tumour resection
Help Discuss her with the oncologists with a view to Biochemistry
PasTest Store chemotherapy
Diabetes
Repeat the chest X-ray in 3 months
Endocrinology
My Account
981 Blood gases
Profile
CSF
Newsletters
My Career and Exams
Order History
Learning Goals
Question Filters

Security

Change Password
Sign Out

© 2011 PasTest Ltd | About Us | Contact Us | Help

file:///E|/Shakhawan/Respiratory%20S/156.htm[3/13/2012 4:03:28 PM]


MyPasTest

Main Navigation
Question Browser: MRCP 1
Home
Subscriptions Question Browser Exam Builder Saved Exams
» MRCP 1 Session Progress
• Question Browser
Questions Correct 1
• Timed Test A 30-year-old woman who is a non-smoker comes to clinic
with a 1-year history of a dry cough and most recently Questions Incorrect 1
• Mock Exam
haemoptysis. Examination is normal, but her chest Questions Total 2
• Past Papers radiograph shows a smooth lesion at the origin of her right
Questions Percentage 50 %
• Random Questions lower lobe bronchus. CT scan confirms this and is otherwise
normal. You perform the bronchoscopy and see a smooth More
• My Performance
red tumour. What would you do next?
• Media Bank
• New Multimedia Biopsy the tumour Your answer
Online Extras Discuss her with the thoracic
surgeons with a view to biopsy
Library
Discuss her with the thoracic Correct answer
Community surgeons with a view to tumour
Help resection
PasTest Store Discuss her with the oncologists
with a view to chemotherapy
Repeat the chest X-ray in 3 months
My Account

Profile
Newsletters
My Career and Exams This woman has a pulmonary carcinoid tumour, which is
Order History an uncommon neuroendocrine primary lung tumour, Reference: Normal Values
accounting for 1–2% of all lung tumours. They are slow
Learning Goals growing and benign, although there are more Haematology
Question Filters aggressive subtypes which can metastasise. Carcinoid
syndrome usually occurs in patients with metastases. Immunoglobulins
Carcinoids are typically located endobronchially, so can
Biochemistry
Security cause wheeze and cough. They can bleed profusely on
biopsy, so this is not advised. A person with an isolated Diabetes
Change Password pulmonary carcinoid should be referred for tumour
Sign Out resection, and histology may not be necessary prior to Endocrinology
surgery if the clinical picture is typical. Chemotherapy
can be used for metastatic carcinoids. Radiotherapy is Blood gases
not used. CSF

981

© 2011 PasTest Ltd | About Us | Contact Us | Help

file:///E|/Shakhawan/Respiratory%20S/156a.htm[3/13/2012 4:03:30 PM]


MyPasTest

Main Navigation
Question Browser: MRCP 1
Home
Subscriptions Question Browser Exam Builder Saved Exams
» MRCP 1 Session Progress
• Question Browser
Questions Correct 1
• Timed Test A 40-year-old man presents to the emergency department
with difficulty breathing and swallowing. Examination is Questions Incorrect 1
• Mock Exam
normal. Chest X-ray shows an enlarged upper Questions Total 2
• Past Papers mediastinum, so you arrange a CT scan, which is performed
Questions Percentage 50 %
• Random Questions the next day. This shows an enlarged thymus gland.
• My Performance More
What would you do next?
• Media Bank
• New Multimedia Arrange a fine-needle aspirate of the thymus
Reference: Normal Values
Online Extras Arrange a biopsy of the thymus
Refer to the surgeons for thymus excision Haematology
Library
Community Refer to the oncologists for radiotherapy Immunoglobulins
Help Await the results of antibody testing prior to making
Biochemistry
a decision
PasTest Store Diabetes
982
Endocrinology
My Account
Blood gases
Profile
CSF
Newsletters
My Career and Exams
Order History
Learning Goals
Question Filters

Security

Change Password
Sign Out

© 2011 PasTest Ltd | About Us | Contact Us | Help

file:///E|/Shakhawan/Respiratory%20S/157.htm[3/13/2012 4:03:31 PM]


MyPasTest

Main Navigation
Question Browser: MRCP 1
Home
Subscriptions Question Browser Exam Builder Saved Exams
» MRCP 1 Session Progress
• Question Browser
Questions Correct 2
• Timed Test A 40-year-old man presents to the emergency department
with difficulty breathing and swallowing. Examination is Questions Incorrect 1
• Mock Exam
normal. Chest X-ray shows an enlarged upper Questions Total 3
• Past Papers mediastinum, so you arrange a CT scan, which is performed
Questions Percentage 66 %
• Random Questions the next day. This shows an enlarged thymus gland.
• My Performance More
What would you do next?
• Media Bank
• New Multimedia Arrange a fine-needle aspirate of the
thymus
Online Extras
Arrange a biopsy of the thymus
Library
Refer to the surgeons for thymus Your answer
Community excision
Help Refer to the oncologists for
PasTest Store radiotherapy
Await the results of antibody testing
prior to making a decision
My Account

Profile
Newsletters
My Career and Exams This man has a thymoma, which is a tumour of
epithelial origin arising in the thymus. Between 30 and Reference: Normal Values
Order History
40% of patients with a thymoma have myasthenia
Learning Goals gravis, and will have positive antiacetylcholine-receptor Haematology
Question Filters antibodies. The myasthenia often does not improve
after the thymus is removed. Awaiting the results of Immunoglobulins
antibody testing does not alter the management, which
is thymectomy (20% of patients with myasthenia gravis Biochemistry
Security
have a thymoma). Thymomas contained within the Diabetes
Change Password thymic capsule tend to be benign, but those that have
extended beyond it are generally malignant. Biopsy or Endocrinology
Sign Out
FNA can breech the capsule and hence increase the risk
of thymoma tumour seeding and should be avoided. Blood gases
Postoperative radiotherapy is indicated for those
CSF
malignant or incompletely excised thymomas.

982

© 2011 PasTest Ltd | About Us | Contact Us | Help

file:///E|/Shakhawan/Respiratory%20S/157a.htm[3/13/2012 4:03:33 PM]


MyPasTest

Main Navigation
Question Browser: MRCP 1
Home
Subscriptions Question Browser Exam Builder Saved Exams
» MRCP 1 Session Progress
• Question Browser
Questions Correct 2
• Timed Test What would be the optimal management for a 70-year-old
man with moderate COPD who has attended the emergency Questions Incorrect 1
• Mock Exam
department with increasing dyspnoea and has been found Questions Total 3
• Past Papers to have a 3-cm pneumothorax?
Questions Percentage 66 %
• Random Questions
• My Performance Conservative management, with observation and More
repeat chest X-ray after 4 hours
• Media Bank
Conservative management, with observation and
• New Multimedia repeat chest X-ray after 12 hours Reference: Normal Values
Online Extras Needle aspiration
Haematology
Library Chest drain insertion if needle aspiration fails
Community Chest drain insertion initially Immunoglobulins
Help Biochemistry
983
PasTest Store Diabetes
Endocrinology
My Account
Blood gases
Profile
CSF
Newsletters
My Career and Exams
Order History
Learning Goals
Question Filters

Security

Change Password
Sign Out

© 2011 PasTest Ltd | About Us | Contact Us | Help

file:///E|/Shakhawan/Respiratory%20S/158.htm[3/13/2012 4:03:34 PM]


MyPasTest

Main Navigation
Question Browser: MRCP 1
Home
Subscriptions Question Browser Exam Builder Saved Exams
» MRCP 1 Session Progress
• Question Browser
Questions Correct 3
• Timed Test What would be the optimal management for a 70-year-old
man with moderate COPD who has attended the emergency Questions Incorrect 1
• Mock Exam
department with increasing dyspnoea and has been found Questions Total 4
• Past Papers to have a 3-cm pneumothorax?
Questions Percentage 75 %
• Random Questions
• My Performance Conservative management, with More
observation and repeat chest X-ray after
• Media Bank 4 hours
• New Multimedia Conservative management, with
Online Extras observation and repeat chest X-ray after
12 hours
Library
Needle aspiration
Community
Chest drain insertion if needle
Help aspiration fails
PasTest Store Chest drain insertion initially Your answer

My Account

Profile
This is a man with a pneumothorax secondary to his
Newsletters
COPD. He is symptomatic from it, and the British
My Career and Exams Thoracic Society Guidelines suggest he should not
Order History undergo needle aspiration but proceed to chest drain Reference: Normal Values
insertion initially. Patients with a primary pneumothorax
Learning Goals may need no intervention if they are asymptomatic, as Haematology
Question Filters the pneumothorax may resolve, or they may benefit
from needle aspiration. Only if this fails and they are Immunoglobulins
symptomatic do they need a chest drain. It may be
Biochemistry
Security reasonable to consider medical pleurodesis while the
drain is in situ for this man with COPD, if he were unfit Diabetes
Change Password for future surgery, to try and prevent pneumothorax
Sign Out recurrence in the future. (BTS Guidelines for the Endocrinology
Management of Pleural Disease 2003. Thorax,58, Suppl.
II). Blood gases

CSF

983

© 2011 PasTest Ltd | About Us | Contact Us | Help

file:///E|/Shakhawan/Respiratory%20S/158a.htm[3/13/2012 4:03:36 PM]


MyPasTest

Main Navigation
Question Browser: MRCP 1
Home
Subscriptions Question Browser Exam Builder Saved Exams
» MRCP 1 Session Progress
• Question Browser
Questions Correct 3
• Timed Test The nurses on the ward ask you to look at a chest drain
that has been inserted in a patient for a pneumothorax. Questions Incorrect 1
• Mock Exam
They are concerned about whether it is still working. When Questions Total 4
• Past Papers the patient coughs, nothing happens. When he breathes in
Questions Percentage 75 %
• Random Questions and out, the fluid in the tube moves up and down. What
does this mean? More
• My Performance
• Media Bank Air and fluid are draining from the pleural space
• New Multimedia Fluid alone is draining from the pleural space Reference: Normal Values
Online Extras Air is no longer draining from the pleural space as
the drain is blocked Haematology
Library
Community Air is no longer draining from the pleural space, but Immunoglobulins
the drain is still working
Help Biochemistry
He needs to commence suction to the drain
PasTest Store Diabetes
984
Endocrinology
My Account
Blood gases
Profile
CSF
Newsletters
My Career and Exams
Order History
Learning Goals
Question Filters

Security

Change Password
Sign Out

© 2011 PasTest Ltd | About Us | Contact Us | Help

file:///E|/Shakhawan/Respiratory%20S/159.htm[3/13/2012 4:03:37 PM]


MyPasTest

Main Navigation
Question Browser: MRCP 1
Home
Subscriptions Question Browser Exam Builder Saved Exams
» MRCP 1 Session Progress
• Question Browser
Questions Correct 4
• Timed Test The nurses on the ward ask you to look at a chest drain
that has been inserted in a patient for a pneumothorax. Questions Incorrect 1
• Mock Exam
They are concerned about whether it is still working. When Questions Total 5
• Past Papers the patient coughs, nothing happens. When he breathes in
Questions Percentage 80 %
• Random Questions and out, the fluid in the tube moves up and down. What
does this mean? More
• My Performance
• Media Bank Air and fluid are draining from the
• New Multimedia pleural space
Online Extras Fluid alone is draining from the pleural
space
Library
Air is no longer draining from the
Community pleural space as the drain is blocked
Help Air is no longer draining from the Your answer
PasTest Store pleural space, but the drain is still
working
He needs to commence suction to the
My Account drain
Profile
Newsletters
My Career and Exams
Order History The fluid level rising and falling in the drain ‘swinging’ Reference: Normal Values
shows it is still in contact with the pleural space and the
Learning Goals fluid level is moving with respiration. Air is not bubbling Haematology
Question Filters out of the drain when the patient coughs, as the air has
stopped draining from the pleural space and the lung Immunoglobulins
has re-inflated. If a drain does not bubble or swing,
Biochemistry
Security then it is blocked or kinked and is not working. With a
simple pneumothorax, there would be minimal fluid Diabetes
Change Password drainage from the chest. Suction is necessary if the
Sign Out drain is still bubbling, but the lung has not fully re- Endocrinology
inflated on the chest X-ray.
Blood gases

CSF
984

© 2011 PasTest Ltd | About Us | Contact Us | Help

file:///E|/Shakhawan/Respiratory%20S/159a.htm[3/13/2012 4:03:39 PM]


MyPasTest

Main Navigation
Question Browser: MRCP 1
Home
Subscriptions Question Browser Exam Builder Saved Exams
» MRCP 1 Session Progress
• Question Browser
Questions Correct 4
• Timed Test A 40-year-old woman comes to clinic with a 1-year history
of breathlessness and chest pain on exertion. She has a Questions Incorrect 1
• Mock Exam
loud P2 on listening to her heart and ECG shows right axis Questions Total 5
• Past Papers deviation and right ventricular hypertrophy. Her pulmonary
Questions Percentage 80 %
• Random Questions function tests are normal, although blood gas shows
hypoxia. Echocardiogram shows a resting pulmonary artery More
• My Performance
pressure of 25 mmHg. CT scan shows enlarged pulmonary
• Media Bank arteries, but no clot. What features in her history may
• New Multimedia help with the diagnosis?
Reference: Normal Values
Online Extras Previous breast cancer
Haematology
Library Previous heroin use
Community Immunoglobulins
Previous dexfenfluramine use
Help Previous sildenafil use Biochemistry
PasTest Store Previous bosentan use Diabetes

985 Endocrinology
My Account
Blood gases
Profile
CSF
Newsletters
My Career and Exams
Order History
Learning Goals
Question Filters

Security

Change Password
Sign Out

© 2011 PasTest Ltd | About Us | Contact Us | Help

file:///E|/Shakhawan/Respiratory%20S/160.htm[3/13/2012 4:03:40 PM]


MyPasTest

Main Navigation
Question Browser: MRCP 1
Home
Subscriptions Question Browser Exam Builder Saved Exams
» MRCP 1 Session Progress
• Question Browser
Questions Correct 5
• Timed Test A 40-year-old woman comes to clinic with a 1-year history
of breathlessness and chest pain on exertion. She has a Questions Incorrect 1
• Mock Exam
loud P2 on listening to her heart and ECG shows right axis Questions Total 6
• Past Papers deviation and right ventricular hypertrophy. Her pulmonary
Questions Percentage 83 %
• Random Questions function tests are normal, although blood gas shows
hypoxia. Echocardiogram shows a resting pulmonary artery More
• My Performance
pressure of 25 mmHg. CT scan shows enlarged pulmonary
• Media Bank arteries, but no clot. What features in her history may
• New Multimedia help with the diagnosis?

Online Extras Previous breast cancer


Library Previous heroin use
Community Previous dexfenfluramine use Your answer
Help Previous sildenafil use
PasTest Store Previous bosentan use

My Account

Profile
This woman has pulmonary hypertension, with no
Newsletters
obvious cause and certainly no evidence of chronic
My Career and Exams thromboembolic disease. Primary pulmonary
Order History hypertension (PPH) is a rare disease of uncertain Reference: Normal Values
aetiology. It has however been associated with
Learning Goals anorectic drug use (fenfluramine, dexfenfluramine), Haematology
Question Filters amfetamine use and possibly cocaine use, but not
heroin. Sildenafil (Viagra) has been used in trials as a Immunoglobulins
treatment for pulmonary hypertension, while bosentan
Biochemistry
Security is an endothelin-receptor antagonist used in the
treatment of pulmonary hypertension. Breast cancer is Diabetes
Change Password unrelated to pulmonary hypertension.
Sign Out Endocrinology

Blood gases
985
CSF

© 2011 PasTest Ltd | About Us | Contact Us | Help

file:///E|/Shakhawan/Respiratory%20S/160a.htm[3/13/2012 4:03:42 PM]


MyPasTest

Main Navigation
Question Browser: MRCP 1
Home
Subscriptions Question Browser Exam Builder Saved Exams
» MRCP 1 Session Progress
• Question Browser
Questions Correct 5
• Timed Test A 35-year-old woman with recently diagnosed primary
pulmonary hypertension asks you some questions regarding Questions Incorrect 1
• Mock Exam
treatment options. She is awaiting transfer to a specialist Questions Total 6
• Past Papers centre for right heart catheterisation. Which of the
Questions Percentage 83 %
• Random Questions following are true?
• My Performance More
She will benefit from taking long-term
• Media Bank anticoagulation with warfarin
• New Multimedia She will benefit from taking verapamil Reference: Normal Values
Online Extras She will benefit from taking lisinopril
Haematology
Library She will benefit from taking the oral contraceptive pill
Community She will be able to have children, as long as she is Immunoglobulins
Help carefully monitored Biochemistry
PasTest Store 986 Diabetes
Endocrinology
My Account
Blood gases
Profile
CSF
Newsletters
My Career and Exams
Order History
Learning Goals
Question Filters

Security

Change Password
Sign Out

© 2011 PasTest Ltd | About Us | Contact Us | Help

file:///E|/Shakhawan/Respiratory%20S/161.htm[3/13/2012 4:03:43 PM]


MyPasTest

Main Navigation
Question Browser: MRCP 1
Home
Subscriptions Question Browser Exam Builder Saved Exams
» MRCP 1 Session Progress
• Question Browser
Questions Correct 5
• Timed Test A 35-year-old woman with recently diagnosed primary
pulmonary hypertension asks you some questions regarding Questions Incorrect 2
• Mock Exam
treatment options. She is awaiting transfer to a specialist Questions Total 7
• Past Papers centre for right heart catheterisation. Which of the
Questions Percentage 71 %
• Random Questions following are true?
• My Performance More
She will benefit from taking long- Correct answer
• Media Bank term anticoagulation with warfarin
• New Multimedia She will benefit from taking Your answer
Online Extras verapamil
Library She will benefit from taking lisinopril
Community She will benefit from taking the oral
contraceptive pill
Help
She will be able to have children, as
PasTest Store long as she is carefully monitored

My Account

Profile
All patients with primary pulmonary hypertension (PPH),
Newsletters
that is pulmonary hypertension of unknown aetiology,
My Career and Exams are at risk of thromboembolic disease. Several
Order History uncontrolled studies have suggested a survival benefit Reference: Normal Values
from anticoagulation, although no randomised controlled
Learning Goals trials exist. Vasodilator studies are performed in patients Haematology
Question Filters with PPH to assess vasodilator response. However,
verapamil is not used as it has negatively ionotropic Immunoglobulins
effects. Angiotensin-converting enzyme (ACE) inhibitors
Biochemistry
Security have no useful benefit in PPH. Pregnancy is poorly
tolerated in patients with PPH and oral contraceptives Diabetes
Change Password increase the risk of venous thromboembolism, so are
Sign Out not advised. A number of endothelin receptor Endocrinology
antagonists have recently become available and are
likely to become a mainstay of therapy for PPH. Blood gases

CSF

986

© 2011 PasTest Ltd | About Us | Contact Us | Help

file:///E|/Shakhawan/Respiratory%20S/161a.htm[3/13/2012 4:03:44 PM]


MyPasTest

Main Navigation
Question Browser: MRCP 1
Home
Subscriptions Question Browser Exam Builder Saved Exams
» MRCP 1 Session Progress
• Question Browser
Questions Correct 5
• Timed Test A 50-year-old man who has a history of iv drug use is
admitted with a productive cough, fevers and rigors. Questions Incorrect 2
• Mock Exam
Examination and chest X-ray show a small right-sided Questions Total 7
• Past Papers effusion and right lower lobe consolidation. Pleural
Questions Percentage 71 %
• Random Questions aspiration of the fluid shows it to be a clear straw colour,
with a protein level of 35 g/l and a pH of 7.12. It has been More
• My Performance
sent for culture, along with blood cultures.
• Media Bank
Which would be the most appropriate course of
• New Multimedia
management? Reference: Normal Values
Online Extras
Start iv cefuroxime and oral metronidazole and Haematology
Library
reassess the size of the effusion in 3 days
Community Immunoglobulins
Start iv benzylpenicillin and oral clarithromycin and
Help insert a chest drain into the effusion Biochemistry
PasTest Store Start oral amoxicillin and oral metronidazole and Diabetes
repeat a pleural tap the following day
Endocrinology
My Account Start iv cefuroxime and oral metronidazole and
arrange a medical thoracoscopy Blood gases
Profile
Start oral amoxicillin and oral metronidazole and refer CSF
Newsletters to the thoracic surgeons for debridement
My Career and Exams
987
Order History
Learning Goals
Question Filters

Security

Change Password
Sign Out

© 2011 PasTest Ltd | About Us | Contact Us | Help

file:///E|/Shakhawan/Respiratory%20S/162.htm[3/13/2012 4:03:46 PM]


MyPasTest

Main Navigation
Question Browser: MRCP 1
Home
Subscriptions Question Browser Exam Builder Saved Exams
» MRCP 1 Session Progress
• Question Browser
Questions Correct 6
• Timed Test A 50-year-old man who has a history of iv drug use is
admitted with a productive cough, fevers and rigors. Questions Incorrect 2
• Mock Exam
Examination and chest X-ray show a small right-sided Questions Total 8
• Past Papers effusion and right lower lobe consolidation. Pleural
Questions Percentage 75 %
• Random Questions aspiration of the fluid shows it to be a clear straw colour,
with a protein level of 35 g/l and a pH of 7.12. It has been More
• My Performance
sent for culture, along with blood cultures.
• Media Bank
Which would be the most appropriate course of
• New Multimedia
management?
Online Extras
Library Start iv cefuroxime and oral
metronidazole and reassess the size of
Community the effusion in 3 days
Help Start iv benzylpenicillin and oral Your answer
PasTest Store clarithromycin and insert a chest drain
into the effusion
Start oral amoxicillin and oral
My Account metronidazole and repeat a pleural tap
Profile the following day

Newsletters Start iv cefuroxime and oral


metronidazole and arrange a medical
My Career and Exams thoracoscopy
Order History Start oral amoxicillin and oral Reference: Normal Values
Learning Goals metronidazole and refer to the thoracic
Haematology
surgeons for debridement
Question Filters
Immunoglobulins
Biochemistry
Security
Diabetes
Change Password This man has pneumonia and an empyema. He has an
exudative acidic effusion with a pH of below 7.2. This Endocrinology
Sign Out
therefore needs drainage as well as antibiotic
treatment, with iv benzylpenicillin and clarithromycin Blood gases
being reasonable choices. If the effusion was
CSF
parapneumonic and not an empyema, ie the pH > 7.2
and culture-negative, treatment with antibiotics and
reassessment of the effusion would be a reasonable
course of management. However, if the patient’s fever,
white cell count or inflammatory markers fail to settle
this course should be employed together with repeated
aspiration to ensure empyema has not subsequently
developed. Empyemas, which fail to resolve with
medical management alone, may require surgical
intervention with debridement. Medical thoracoscopy is
not indicated in this setting. (See BTS Guidelines for the
Management of Pleural Disease 2003. Thorax,58, Suppl.
II.)

987

© 2011 PasTest Ltd | About Us | Contact Us | Help

file:///E|/Shakhawan/Respiratory%20S/162a.htm[3/13/2012 4:03:47 PM]


MyPasTest

Main Navigation
Question Browser: MRCP 1
Home
Subscriptions Question Browser Exam Builder Saved Exams
» MRCP 1 Session Progress
• Question Browser
Questions Correct 6
• Timed Test In a cyanosed patient which one of the following
statements is accurate? Questions Incorrect 2
• Mock Exam
Questions Total 8
• Past Papers
The PaO2 is not normally above 50 mmHg (7 Questions Percentage 75 %
• Random Questions kpascal)
• My Performance More
In methaemoglobinaemia the PaO2 is never above 50
• Media Bank mmHg (7 kpascal)
• New Multimedia The expected reduced haemoglobin level is around 2
Reference: Normal Values
Online Extras g/l
The blue tinge of the skin and mucous membrane is Haematology
Library
due to CO2 retention
Community Immunoglobulins
O 2 therapy should be avoided as it may worsen
Help Biochemistry
hypercapnia
PasTest Store Diabetes
1671
Endocrinology
My Account
Blood gases
Profile
CSF
Newsletters
My Career and Exams
Order History
Learning Goals
Question Filters

Security

Change Password
Sign Out

© 2011 PasTest Ltd | About Us | Contact Us | Help

file:///E|/Shakhawan/Respiratory%20S/163.htm[3/13/2012 4:03:49 PM]


MyPasTest

Main Navigation
Question Browser: MRCP 1
Home
Subscriptions Question Browser Exam Builder Saved Exams
» MRCP 1 Session Progress
• Question Browser
Questions Correct 6
• Timed Test In a cyanosed patient which one of the following
statements is accurate? Questions Incorrect 3
• Mock Exam
Questions Total 9
• Past Papers
The PaO2 is not normally above 50 Correct answer Questions Percentage 66 %
• Random Questions mmHg (7 kpascal)
• My Performance More
In methaemoglobinaemia the PaO2
• Media Bank is never above 50 mmHg (7 kpascal)
• New Multimedia The expected reduced haemoglobin
Online Extras level is around 2 g/l

Library The blue tinge of the skin and Your answer


mucous membrane is due to CO2
Community retention
Help O 2 therapy should be avoided as it
PasTest Store may worsen hypercapnia

My Account

Profile
Persistent cyanosis without hypoxia (normal PaO2 )
Newsletters
suggests a diagnosis of methaemoglobinaemia, or
My Career and Exams sulfhaemoglobinaemia. In a cyanosed patient the
Order History amount of reduced haemoglobin in the blood is at least Reference: Normal Values
5 g/dl or more. The blue colour of the skin and mucous
Learning Goals membrane is due to hypoxia and not hypercapnia. Haematology
Question Filters Hypoxia should be corrected by oxygen therapy.
Immunoglobulins
Biochemistry
Security 1671
Diabetes
Change Password
Sign Out Endocrinology

Blood gases

CSF

© 2011 PasTest Ltd | About Us | Contact Us | Help

file:///E|/Shakhawan/Respiratory%20S/163a.htm[3/13/2012 4:03:50 PM]


MyPasTest

Main Navigation
Question Browser: MRCP 1
Home
Subscriptions Question Browser Exam Builder Saved Exams
» MRCP 1 Session Progress
• Question Browser
Questions Correct 6
• Timed Test Which one of the following clinical findings is MOST
suggestive of pulmonary embolism (PE)? Questions Incorrect 3
• Mock Exam
Questions Total 9
• Past Papers
Spiking temperature of 39ºC lasting more than one Questions Percentage 66 %
• Random Questions week
• My Performance Haemoptysis of more than 5 ml with negative chest More
• Media Bank X-ray

• New Multimedia Chest pain worse on deep breathing and respiratory


rate of 26/min Reference: Normal Values
Online Extras
Recurrent chest pain in the same location Haematology
Library
Chest pain on lying flat
Community Immunoglobulins
Help 1672 Biochemistry
PasTest Store Diabetes
Endocrinology
My Account
Blood gases
Profile
CSF
Newsletters
My Career and Exams
Order History
Learning Goals
Question Filters

Security

Change Password
Sign Out

© 2011 PasTest Ltd | About Us | Contact Us | Help

file:///E|/Shakhawan/Respiratory%20S/164.htm[3/13/2012 4:03:52 PM]


MyPasTest

Main Navigation
Question Browser: MRCP 1
Home
Subscriptions Question Browser Exam Builder Saved Exams
» MRCP 1 Session Progress
• Question Browser
Questions Correct 7
• Timed Test Which one of the following clinical findings is MOST
suggestive of pulmonary embolism (PE)? Questions Incorrect 3
• Mock Exam
Questions Total 10
• Past Papers
Spiking temperature of 39ºC lasting Questions Percentage 70 %
• Random Questions more than one week
• My Performance Haemoptysis of more than 5 ml with More
• Media Bank negative chest X-ray

• New Multimedia Chest pain worse on deep breathing Your answer


and respiratory rate of 26/min
Online Extras
Recurrent chest pain in the same
Library location
Community Chest pain on lying flat
Help
PasTest Store

My Account The clinical features of pulmonary embolism (PE) can be


diverse and confusing and range from no symptoms to
Profile sudden death. Sudden shortness of breath, pleuritic
chest pain with haemoptysis and tachypnoea are the
Newsletters
commonest features. However, a diagnosis that rests on
My Career and Exams clinical grounds alone is often incorrect. Several
Order History features point away from the diagnosis of PE and Reference: Normal Values
include positive features of chest infections such as
Learning Goals swinging temperature (>39ºC) for more than a few Haematology
Question Filters days associated with cough and purulent sputum.
Recurrent chest pain in the same location is unusual for Immunoglobulins
PE and it indicates an underlying diseased lung such as
Biochemistry
Security bronchiectasis with recurrent chest infection.
Haemoptysis in PE is described as blood tinged, blood Diabetes
Change Password streaked or pure blood. It is rarely more than 5 ml or
Sign Out massive and seldom lasts more than a few days. A Endocrinology
normal chest radiograph is uncommon in acute PE, but
the usual finding is not specific. Chest pain on lying flat Blood gases
is characteristic of pericarditis. CSF

1672

© 2011 PasTest Ltd | About Us | Contact Us | Help

file:///E|/Shakhawan/Respiratory%20S/164a.htm[3/13/2012 4:03:53 PM]


MyPasTest

Main Navigation
Question Browser: MRCP 1
Home
Subscriptions Question Browser Exam Builder Saved Exams
» MRCP 1 Session Progress
• Question Browser
Questions Correct 7
• Timed Test Which one of the following conditions is MOST likely
to be associated with obstructive spirometry and Questions Incorrect 3
• Mock Exam
normal TLCO? Questions Total 10
• Past Papers
Questions Percentage 70 %
• Random Questions Asthma
• My Performance Emphysema More
• Media Bank Sarcoidosis
• New Multimedia Pulmonary hypertension
Reference: Normal Values
Online Extras Fibrosing alveolitis
Haematology
Library
1673
Community Immunoglobulins
Help Biochemistry
PasTest Store Diabetes
Endocrinology
My Account
Blood gases
Profile
CSF
Newsletters
My Career and Exams
Order History
Learning Goals
Question Filters

Security

Change Password
Sign Out

© 2011 PasTest Ltd | About Us | Contact Us | Help

file:///E|/Shakhawan/Respiratory%20S/165.htm[3/13/2012 4:03:55 PM]


MyPasTest

Main Navigation
Question Browser: MRCP 1
Home
Subscriptions Question Browser Exam Builder Saved Exams
» MRCP 1 Session Progress
• Question Browser
Questions Correct 8
• Timed Test Which one of the following conditions is MOST likely
to be associated with obstructive spirometry and Questions Incorrect 3
• Mock Exam
normal TLCO? Questions Total 11
• Past Papers
Questions Percentage 72 %
• Random Questions Asthma Your answer
• My Performance Emphysema More
• Media Bank Sarcoidosis
• New Multimedia Pulmonary hypertension
Online Extras Fibrosing alveolitis
Library
Community
Help
PasTest Store The diffusion of CO from the alveoli to the pulmonary
blood is governed by the integrity of the alveolar
membrane, the capillary blood volume, or both (air-
My Account blood barrier). A reduction in the diffusion capacity of
CO is encountered in conditions affecting the capillary
Profile bed size such as pulmonary emboli and pulmonary
vasculitis, or conditions that cause changes in the
Newsletters
characteristics of the alveolar membrane which include
My Career and Exams diseases in which some form of intra-alveolar filling
Order History process has occurred and the air to blood diffusion Reference: Normal Values
pathway is actually lengthened (pneumonia, pulmonary
Learning Goals oedema, alveolar proteinosis). Similarly, TLCO is Haematology
Question Filters reduced in patients with infiltrative disorders of the lung
that affects both the capillary bed size and the alveolar Immunoglobulins
membrane integrity such as sarcoidosis, interstitial lung
Biochemistry
Security diseases, or collagen vascular diseases. Removal or
destruction of lung tissue, such as surgery or Diabetes
Change Password emphysema decreases both membrane and blood
Sign Out volume components and produces low TLCO. An Endocrinology
increase in TLCO results occasionally from an increase
in capillary blood volume secondary to haemodynamic Blood gases
changes in pulmonary circulation; an increase in CSF
pulmonary arterial or left atrial pressures, as in
congestive heart failure, or an increase in pulmonary
blood flow, as in arterial septal defect. The TLCO is
sometimes increased in patients with bronchial asthma
during an attack, but the cause of this change is not
known. Alveolar haemorrhage from any cause can result
in a false increase of TLCO despite the presence of an
underlying diffusion defect.

1673

© 2011 PasTest Ltd | About Us | Contact Us | Help

file:///E|/Shakhawan/Respiratory%20S/165a.htm[3/13/2012 4:03:56 PM]


MyPasTest

Main Navigation
Question Browser: MRCP 1
Home
Subscriptions Question Browser Exam Builder Saved Exams
» MRCP 1 Session Progress
• Question Browser
Questions Correct 8
• Timed Test Which one of the following features is rarely
encountered in patients with sleep apnoea Questions Incorrect 3
• Mock Exam
syndrome? Questions Total 11
• Past Papers
Questions Percentage 72 %
• Random Questions Female gender
• My Performance Day time sleepiness More
• Media Bank Hypertension
• New Multimedia Large neck size
Reference: Normal Values
Online Extras Snoring
Haematology
Library
1674
Community Immunoglobulins
Help Biochemistry
PasTest Store Diabetes
Endocrinology
My Account
Blood gases
Profile
CSF
Newsletters
My Career and Exams
Order History
Learning Goals
Question Filters

Security

Change Password
Sign Out

© 2011 PasTest Ltd | About Us | Contact Us | Help

file:///E|/Shakhawan/Respiratory%20S/166.htm[3/13/2012 4:03:57 PM]


MyPasTest

Main Navigation
Question Browser: MRCP 1
Home
Subscriptions Question Browser Exam Builder Saved Exams
» MRCP 1 Session Progress
• Question Browser
Questions Correct 8
• Timed Test Which one of the following features is rarely
encountered in patients with sleep apnoea Questions Incorrect 4
• Mock Exam
syndrome? Questions Total 12
• Past Papers
Questions Percentage 66 %
• Random Questions Female gender Correct answer
• My Performance Day time sleepiness More
• Media Bank Hypertension Your answer
• New Multimedia Large neck size
Online Extras Snoring
Library
Community
Help
PasTest Store The cardinal manifestation of obstructive sleep apnoea
syndrome (SAS) is snoring during sleep and severe
daytime sleepiness. Other symptoms include night
My Account sweats, personality change, morning confusion and
headache. Patients are described as middle aged obese
Profile males with wide neck and structurally abnormal
airways. There is increasing prevalence of systemic
Newsletters
hypertension. There may be increased risk for
My Career and Exams pulmonary hypertension and cardiac arrhythmias.
Order History Reference: Normal Values
Learning Goals 1674 Haematology
Question Filters
Immunoglobulins
Biochemistry
Security
Diabetes
Change Password
Sign Out Endocrinology

Blood gases

CSF

© 2011 PasTest Ltd | About Us | Contact Us | Help

file:///E|/Shakhawan/Respiratory%20S/166a.htm[3/13/2012 4:03:59 PM]


MyPasTest

Main Navigation
Question Browser: MRCP 1
Home
Subscriptions Question Browser Exam Builder Saved Exams
» MRCP 1 Session Progress
• Question Browser
Questions Correct 8
• Timed Test Which one of the following features is MOST accurate
regarding Pneumocystis jiroveci pneumonia (PCP)? Questions Incorrect 4
• Mock Exam
Questions Total 12
• Past Papers
Occurs exclusively in AIDS Questions Percentage 66 %
• Random Questions
Pleural effusion is frequently bilateral
• My Performance More
Auscultation of the lungs usually reveals no
• Media Bank abnormality
• New Multimedia Blood culture is positive in one-third of cases
Reference: Normal Values
Online Extras Metronidazole is the treatment of choice
Haematology
Library
1675
Community Immunoglobulins
Help Biochemistry
PasTest Store Diabetes
Endocrinology
My Account
Blood gases
Profile
CSF
Newsletters
My Career and Exams
Order History
Learning Goals
Question Filters

Security

Change Password
Sign Out

© 2011 PasTest Ltd | About Us | Contact Us | Help

file:///E|/Shakhawan/Respiratory%20S/167.htm[3/13/2012 4:04:00 PM]


MyPasTest

Main Navigation
Question Browser: MRCP 1
Home
Subscriptions Question Browser Exam Builder Saved Exams
» MRCP 1 Session Progress
• Question Browser
Questions Correct 9
• Timed Test Which one of the following features is MOST accurate
regarding Pneumocystis jiroveci pneumonia (PCP)? Questions Incorrect 4
• Mock Exam
Questions Total 13
• Past Papers
Occurs exclusively in AIDS Questions Percentage 69 %
• Random Questions
Pleural effusion is frequently bilateral
• My Performance More
Auscultation of the lungs usually Your answer
• Media Bank reveals no abnormality
• New Multimedia Blood culture is positive in one-third of
Online Extras cases

Library Metronidazole is the treatment of


choice
Community
Help
PasTest Store
Pneumocystis jiroveci pneumonia (PCP) is a pulmonary
My Account disease characterised by dyspnoea, tachypnoea and
hypoxaemia that occur in patients deficient in
Profile immunoglobulin G and M, and patients deficient in
cellular mediated immunity. The vast majority of adult
Newsletters
patients have AIDS, however, it can also occur in those
My Career and Exams who received chemotherapy for haematological
Order History malignant disease or organ transplant. It can also occur Reference: Normal Values
in malnourished or premature infants. On examination
Learning Goals patients usually show signs of respiratory distress Haematology
Question Filters (tachypnoea, dyspnoea). Auscultation of the lung
usually reveals no abnormalities. The trophozoite does Immunoglobulins
not enter the blood, the organism is identified in
Biochemistry
Security pulmonary secretions obtained by bronchopulmonary
lavage or lung biopsy and stained by methenamine Diabetes
Change Password silver or Giemsa stain. Pentamidine isethionate or
Sign Out cotrimoxazole is the recommended treatment Endocrinology
(metronidazole is not effective in the treatment of PCP).
Blood gases

CSF
1675

© 2011 PasTest Ltd | About Us | Contact Us | Help

file:///E|/Shakhawan/Respiratory%20S/167a.htm[3/13/2012 4:04:02 PM]


MyPasTest

Main Navigation
Question Browser: MRCP 1
Home
Subscriptions Question Browser Exam Builder Saved Exams
» MRCP 1 Session Progress
• Question Browser
Questions Correct 9
• Timed Test Which one of the following features is MOST
characteristic of cystic fibrosis? Questions Incorrect 4
• Mock Exam
Questions Total 13
• Past Papers
Inherited as autosomal dominant Questions Percentage 69 %
• Random Questions
Pancreatic insufficiency is almost always present in
• My Performance adult patients More
• Media Bank In patients with recurrent chest infections
• New Multimedia Burkholderia cepaciais the most frequent organism
isolated from sputum Reference: Normal Values
Online Extras
Family members who carry the gene are at risk of Haematology
Library developing mild recurrent bronchitis
Community Patients typically have reduced levels of sodium and Immunoglobulins
Help chloride in the sweat Biochemistry
PasTest Store 1676 Diabetes
Endocrinology
My Account
Blood gases
Profile
CSF
Newsletters
My Career and Exams
Order History
Learning Goals
Question Filters

Security

Change Password
Sign Out

© 2011 PasTest Ltd | About Us | Contact Us | Help

file:///E|/Shakhawan/Respiratory%20S/168.htm[3/13/2012 4:04:03 PM]


MyPasTest

Main Navigation
Question Browser: MRCP 1
Home
Subscriptions Question Browser Exam Builder Saved Exams
» MRCP 1 Session Progress
• Question Browser
Questions Correct 10
• Timed Test Which one of the following features is MOST
characteristic of cystic fibrosis? Questions Incorrect 4
• Mock Exam
Questions Total 14
• Past Papers
Inherited as autosomal dominant Questions Percentage 71 %
• Random Questions
Pancreatic insufficiency is almost Your answer
• My Performance always present in adult patients More
• Media Bank In patients with recurrent chest
• New Multimedia infections Burkholderia cepaciais the
most frequent organism isolated from
Online Extras sputum
Library Family members who carry the gene
Community are at risk of developing mild recurrent
bronchitis
Help
Patients typically have reduced levels
PasTest Store of sodium and chloride in the sweat

My Account

Profile
Cystic fibrosis (CF) is an autosomal recessive disease
Newsletters
affecting the function of glandular tissue, characterised
My Career and Exams by elevated levels of sodium and chloride in the sweat.
Order History It is caused by abnormal viscid secretions from mucous Reference: Normal Values
glands leading to chronic pulmonary disease and
Learning Goals pancreatic insufficiency, which will be evident in more Haematology
Question Filters than 95% of adult cases. Recurrent chest infections are
usually caused by Pseudomonas aeruginosa and Immunoglobulins
Staphylococcus aureus. Burkholderia cepaciaoccurs in
Biochemistry
Security 5–10% of cases. The carrier rate is 5% in the
Caucasian population, heterozygotes are clinically Diabetes
Change Password normal.
Sign Out Endocrinology

Blood gases
1676
CSF

© 2011 PasTest Ltd | About Us | Contact Us | Help

file:///E|/Shakhawan/Respiratory%20S/168a.htm[3/13/2012 4:04:05 PM]


MyPasTest

Main Navigation
Question Browser: MRCP 1
Home
Subscriptions Question Browser Exam Builder Saved Exams
» MRCP 1 Session Progress
• Question Browser
Questions Correct 10
• Timed Test A 28-year-old black nurse develops painful nodules on the
shin of both legs. She has low grade fever and has lost 5 Questions Incorrect 4
• Mock Exam
kg in the two months prior to her presentation. Her chest Questions Total 14
• Past Papers X-ray shows bilateral hilar lymphadenopathy. The MOST
Questions Percentage 71 %
• Random Questions likely outcome of this patient’s illness is?
• My Performance More
Complete remission after appropriate course of
• Media Bank steroid and cytotoxic drugs
• New Multimedia Complete remission without any specific treatment Reference: Normal Values
Online Extras Complete initial remission soon interrupted by
increasing relapses Haematology
Library
Community Diffuse reticulo-nodular changes in the lung and Immunoglobulins
progressive shortness of breath
Help Biochemistry
Generalised lymphadenopathy and progressive
PasTest Store wasting in 5–10 years Diabetes

1677 Endocrinology
My Account
Blood gases
Profile
CSF
Newsletters
My Career and Exams
Order History
Learning Goals
Question Filters

Security

Change Password
Sign Out

© 2011 PasTest Ltd | About Us | Contact Us | Help

file:///E|/Shakhawan/Respiratory%20S/169.htm[3/13/2012 4:04:06 PM]


MyPasTest

Main Navigation
Question Browser: MRCP 1
Home
Subscriptions Question Browser Exam Builder Saved Exams
» MRCP 1 Session Progress
• Question Browser
Questions Correct 10
• Timed Test A 28-year-old black nurse develops painful nodules on the
shin of both legs. She has low grade fever and has lost 5 Questions Incorrect 5
• Mock Exam
kg in the two months prior to her presentation. Her chest Questions Total 15
• Past Papers X-ray shows bilateral hilar lymphadenopathy. The MOST
Questions Percentage 66 %
• Random Questions likely outcome of this patient’s illness is?
• My Performance More
Complete remission after
• Media Bank appropriate course of steroid and
• New Multimedia cytotoxic drugs
Online Extras Complete remission without any Correct answer
specific treatment
Library
Complete initial remission soon Your answer
Community interrupted by increasing relapses
Help Diffuse reticulo-nodular changes in
PasTest Store the lung and progressive shortness of
breath
Generalised lymphadenopathy and
My Account progressive wasting in 5–10 years
Profile
Newsletters
My Career and Exams
Order History Acute sarcoidosis includes the complex of erythema Reference: Normal Values
nodosum and X-ray findings of bilateral hilar
Learning Goals adenopathy, often accompanied by joint symptoms, Haematology
Question Filters including arthritis at the ankles, knees, wrists, or
elbows. Spontaneous remission occurs in nearly two- Immunoglobulins
thirds of patients with acute sarcoidosis, while 10–30%
Biochemistry
Security develop a chronic course. Remissions often occur within
the first six months after diagnosis. NSAIDs are very Diabetes
Change Password useful for musculoskeletal symptom control.
Sign Out Endocrinology

Blood gases
1677
CSF

© 2011 PasTest Ltd | About Us | Contact Us | Help

file:///E|/Shakhawan/Respiratory%20S/169a.htm[3/13/2012 4:04:08 PM]


MyPasTest

Main Navigation
Question Browser: MRCP 1
Home
Subscriptions Question Browser Exam Builder Saved Exams
» MRCP 1 Session Progress
• Question Browser
Questions Correct 10
• Timed Test Which one of the following features is MOST
characteristic of small cell bronchial carcinoma? Questions Incorrect 5
• Mock Exam
Questions Total 15
• Past Papers
History of prior asbestos exposure is usually obtained Questions Percentage 66 %
• Random Questions
Hyponatraemia
• My Performance More
Known as small cell because the cancer cell origin is
• Media Bank from small lymphocytes
• New Multimedia It has a relatively better prognosis when compared
Reference: Normal Values
Online Extras with other bronchial cancers
Surgery is often the only defined treatment Haematology
Library
Community Immunoglobulins
1678
Help Biochemistry
PasTest Store Diabetes
Endocrinology
My Account
Blood gases
Profile
CSF
Newsletters
My Career and Exams
Order History
Learning Goals
Question Filters

Security

Change Password
Sign Out

© 2011 PasTest Ltd | About Us | Contact Us | Help

file:///E|/Shakhawan/Respiratory%20S/170.htm[3/13/2012 4:04:09 PM]


MyPasTest

Main Navigation
Question Browser: MRCP 1
Home
Subscriptions Question Browser Exam Builder Saved Exams
» MRCP 1 Session Progress
• Question Browser
Questions Correct 11
• Timed Test Which one of the following features is MOST
characteristic of small cell bronchial carcinoma? Questions Incorrect 5
• Mock Exam
Questions Total 16
• Past Papers
History of prior asbestos exposure is Questions Percentage 68 %
• Random Questions usually obtained
• My Performance Hyponatraemia Your answer More
• Media Bank Known as small cell because the cancer
• New Multimedia cell origin is from small lymphocytes

Online Extras It has a relatively better prognosis


when compared with other bronchial
Library cancers
Community Surgery is often the only defined
Help treatment
PasTest Store

My Account
Small cell (oat cell) bronchial carcinoma is frequently
Profile associated with ectopic hormone production. The
syndrome of inappropriate anti-diuretic hormone
Newsletters
secretion (SIADH) causes hyponatraemia. By the time
My Career and Exams the diagnosis has been made the tumour is usually
Order History disseminated, so that surgery is seldom considered. Reference: Normal Values
Unlike mesothelioma a history of asbestos exposure is
Learning Goals seldom obtained. The prognosis is very poor and Haematology
Question Filters survival beyond two years is exceptional.
Immunoglobulins
Biochemistry
Security 1678
Diabetes
Change Password
Sign Out Endocrinology

Blood gases

CSF

© 2011 PasTest Ltd | About Us | Contact Us | Help

file:///E|/Shakhawan/Respiratory%20S/170a.htm[3/13/2012 4:04:10 PM]


MyPasTest

Main Navigation
Question Browser: MRCP 1
Home
Subscriptions Question Browser Exam Builder Saved Exams
» MRCP 1 Session Progress
• Question Browser
Questions Correct 11
• Timed Test Which one of the following conditions is rarely
associated with pulmonary infiltrates and peripheral Questions Incorrect 5
• Mock Exam
eosinophilia? Questions Total 16
• Past Papers
Questions Percentage 68 %
• Random Questions Allergic bronchopulmonary aspergillosis
• My Performance Löeffler’s syndrome More
• Media Bank Churg-Strauss syndrome
• New Multimedia Sulphonamide therapy
Reference: Normal Values
Online Extras Fibrosing alveolitis
Haematology
Library
1679
Community Immunoglobulins
Help Biochemistry
PasTest Store Diabetes
Endocrinology
My Account
Blood gases
Profile
CSF
Newsletters
My Career and Exams
Order History
Learning Goals
Question Filters

Security

Change Password
Sign Out

© 2011 PasTest Ltd | About Us | Contact Us | Help

file:///E|/Shakhawan/Respiratory%20S/171.htm[3/13/2012 4:04:12 PM]


MyPasTest

Main Navigation
Question Browser: MRCP 1
Home
Subscriptions Question Browser Exam Builder Saved Exams
» MRCP 1 Session Progress
• Question Browser
Questions Correct 11
• Timed Test Which one of the following conditions is rarely
associated with pulmonary infiltrates and peripheral Questions Incorrect 6
• Mock Exam
eosinophilia? Questions Total 17
• Past Papers
Questions Percentage 64 %
• Random Questions Allergic bronchopulmonary
• My Performance aspergillosis More
• Media Bank Löeffler’s syndrome

• New Multimedia Churg-Strauss syndrome

Online Extras Sulphonamide therapy Your answer

Library Fibrosing alveolitis Correct answer

Community
Help
PasTest Store
Eosinophilic lung diseases are a heterogeneous group of
disorders which are characterised by the presence of
My Account pulmonary symptoms or an abnormal chest X-ray
accompanied by inflammatory cell infiltrate in the
Profile airways and/or lung parenchyma which contain a large
number of eosinophils. Many of these disorders are
Newsletters
associated with peripheral eosinophilia. The following
My Career and Exams list is just an example:
Order History Reference: Normal Values
Drugs and toxins (nitrofurantoin, L-tryptophan,
Learning Goals sulphonamides) Haematology
Question Filters
Helminthic infection (Löeffler’s syndrome, larva Immunoglobulins
migran)
Biochemistry
Security Acute and chronic eosinophilic pneumonia (primary)
Diabetes
Change Password Churg-Strauss syndrome
Sign Out Allergic bronchopulmonary aspergillosis Endocrinology

Hypereosinophilic syndrome Blood gases

CSF

1679

© 2011 PasTest Ltd | About Us | Contact Us | Help

file:///E|/Shakhawan/Respiratory%20S/171a.htm[3/13/2012 4:04:13 PM]


MyPasTest

Main Navigation
Question Browser: MRCP 1
Home
Subscriptions Question Browser Exam Builder Saved Exams
» MRCP 1 Session Progress
• Question Browser
Questions Correct 11
• Timed Test Which one of the following statements with regard to
sarcoidosis is true? Questions Incorrect 6
• Mock Exam
Questions Total 17
• Past Papers
Parenchymal lung disease is often accompanied with Questions Percentage 64 %
• Random Questions pleural effusion
• My Performance Clubbing of the fingers is an early feature More
• Media Bank Jaundice and portal hypertension are the
• New Multimedia predominant features of hepatic sarcoidosis
Reference: Normal Values
Online Extras A positive tuberculin test in a patient with chronic
sarcoidosis is suggestive of concomitant tuberculosis Haematology
Library
Hypercalcaemia when it manifests is usually resistant
Community to steroid therapy Immunoglobulins
Help Biochemistry
1680
PasTest Store Diabetes
Endocrinology
My Account
Blood gases
Profile
CSF
Newsletters
My Career and Exams
Order History
Learning Goals
Question Filters

Security

Change Password
Sign Out

© 2011 PasTest Ltd | About Us | Contact Us | Help

file:///E|/Shakhawan/Respiratory%20S/172.htm[3/13/2012 4:04:15 PM]


MyPasTest

Main Navigation
Question Browser: MRCP 1
Home
Subscriptions Question Browser Exam Builder Saved Exams
» MRCP 1 Session Progress
• Question Browser
Questions Correct 11
• Timed Test Which one of the following statements with regard to
sarcoidosis is true? Questions Incorrect 7
• Mock Exam
Questions Total 18
• Past Papers
Parenchymal lung disease is often Your answer Questions Percentage 61 %
• Random Questions accompanied with pleural effusion
• My Performance Clubbing of the fingers is an early More
• Media Bank feature

• New Multimedia Jaundice and portal hypertension


are the predominant features of
Online Extras hepatic sarcoidosis
Library A positive tuberculin test in a Correct answer
Community patient with chronic sarcoidosis is
suggestive of concomitant
Help tuberculosis
PasTest Store Hypercalcaemia when it manifests is
usually resistant to steroid therapy
My Account

Profile
Newsletters
Sarcoidosis is a systemic disorder of unknown cause
My Career and Exams that is characterised by its pathological hallmark, the
Order History noncaseating granuloma primarily affecting the Reference: Normal Values
respiratory tract, skin, eye, heart, kidneys and liver.
Learning Goals Pleural disease is relatively infrequent, with effusions Haematology
Question Filters occurring in fewer than 5% of patients. Clubbing of the
fingers is not a recognised feature of sarcoidosis. Immunoglobulins
Although liver biopsy reveals granulomatous
Biochemistry
Security involvement in 40–70% of patients, clinically significant
hepatic disease is rare. A tuberculin test is usually Diabetes
Change Password negative in chronic sarcoidosis, however, most
Sign Out sarcoidosis patients who develop tuberculosis become Endocrinology
tuberculin-positive. Hypercalcaemia, a potentially
important complication of sarcoidosis, occurs in fewer Blood gases
than 10% of patients and is thought to be due to CSF
elevated levels of 1,25-dihydroxyvitamin D (calcitriol),
which is produced by macrophages within the
granulomas. High-dose glucocorticoids are very helpful
in vitamin D intoxication, granulomatous diseases such
as sarcoidosis, and haematologic malignancies known or
likely to be glucocorticoid-responsive.

1680

© 2011 PasTest Ltd | About Us | Contact Us | Help

file:///E|/Shakhawan/Respiratory%20S/172a.htm[3/13/2012 4:04:16 PM]


MyPasTest

Main Navigation
Question Browser: MRCP 1
Home
Subscriptions Question Browser Exam Builder Saved Exams
» MRCP 1 Session Progress
• Question Browser
Questions Correct 11
• Timed Test A 21-year-old medical student presents with confusion and
dyspnoea 24 hours after fracturing his left femur in a ski Questions Incorrect 7
• Mock Exam
competition. Which one of the following skin lesions is Questions Total 18
• Past Papers expected on physical examination?
Questions Percentage 61 %
• Random Questions
• My Performance Multiple vesicular lesions on the upper back More
• Media Bank Target lesions on the chest

• New Multimedia Tender red nodules on the shin


Reference: Normal Values
Online Extras Multiple petechiae in both axillae
Palpable purpura on the buttock Haematology
Library
Community Immunoglobulins
1681
Help Biochemistry
PasTest Store Diabetes
Endocrinology
My Account
Blood gases
Profile
CSF
Newsletters
My Career and Exams
Order History
Learning Goals
Question Filters

Security

Change Password
Sign Out

© 2011 PasTest Ltd | About Us | Contact Us | Help

file:///E|/Shakhawan/Respiratory%20S/173.htm[3/13/2012 4:04:18 PM]


MyPasTest

Main Navigation
Question Browser: MRCP 1
Home
Subscriptions Question Browser Exam Builder Saved Exams
» MRCP 1 Session Progress
• Question Browser
Questions Correct 11
• Timed Test A 21-year-old medical student presents with confusion and
dyspnoea 24 hours after fracturing his left femur in a ski Questions Incorrect 8
• Mock Exam
competition. Which one of the following skin lesions is Questions Total 19
• Past Papers expected on physical examination?
Questions Percentage 57 %
• Random Questions
• My Performance Multiple vesicular lesions on the More
upper back
• Media Bank
Target lesions on the chest Your answer
• New Multimedia
Tender red nodules on the shin
Online Extras
Multiple petechiae in both axillae Correct answer
Library
Palpable purpura on the buttock
Community
Help
PasTest Store
The appearance of showers of petechiae over the axillae
or upper half of the body is characteristic of fat
My Account embolism syndrome occurring in a patient with recent
traumatic fracture.
Profile
Newsletters 1681
My Career and Exams
Order History Reference: Normal Values
Learning Goals Haematology
Question Filters
Immunoglobulins
Biochemistry
Security
Diabetes
Change Password
Sign Out Endocrinology

Blood gases

CSF

© 2011 PasTest Ltd | About Us | Contact Us | Help

file:///E|/Shakhawan/Respiratory%20S/173a.htm[3/13/2012 4:04:19 PM]


MyPasTest

Main Navigation
Question Browser: MRCP 1
Home
Subscriptions Question Browser Exam Builder Saved Exams
» MRCP 1 Session Progress
• Question Browser
Questions Correct 11
• Timed Test Most cases of community acquired pneumonia are
caused by which one of the following? Questions Incorrect 8
• Mock Exam
Questions Total 19
• Past Papers
Streptococcus pneumoniae Questions Percentage 57 %
• Random Questions
Mycoplasma pneumoniae
• My Performance More
Staphylococcus aureus
• Media Bank
Haemophilus influenzae
• New Multimedia
Viral pneumonia Reference: Normal Values
Online Extras
1682 Haematology
Library
Community Immunoglobulins
Help Biochemistry
PasTest Store Diabetes
Endocrinology
My Account
Blood gases
Profile
CSF
Newsletters
My Career and Exams
Order History
Learning Goals
Question Filters

Security

Change Password
Sign Out

© 2011 PasTest Ltd | About Us | Contact Us | Help

file:///E|/Shakhawan/Respiratory%20S/174.htm[3/13/2012 4:04:20 PM]


MyPasTest

Main Navigation
Question Browser: MRCP 1
Home
Subscriptions Question Browser Exam Builder Saved Exams
» MRCP 1 Session Progress
• Question Browser
Questions Correct 12
• Timed Test Most cases of community acquired pneumonia are
caused by which one of the following? Questions Incorrect 8
• Mock Exam
Questions Total 20
• Past Papers
Streptococcus pneumoniae Your answer Questions Percentage 60 %
• Random Questions
Mycoplasma pneumoniae
• My Performance More
Staphylococcus aureus
• Media Bank
Haemophilus influenzae
• New Multimedia
Viral pneumonia
Online Extras
Library
Community
Help Community acquired pneumonia is contracted in the
PasTest Store community rather than in hospital. In the northern
hemisphere, community acquired pneumonia affects
approximately 12/1000 people per year, particularly
My Account during winter and at the extremes of age (incidence: <
1 year old 30–50/1000 per year; 71–85 years 50/1000
Profile per year). Over 100 microorganisms have been
implicated, but most cases are caused by Streptococcus
Newsletters
pneumoniae. Smoking is probably an important risk
My Career and Exams factor.
Order History Reference: Normal Values
Learning Goals 1682 Haematology
Question Filters
Immunoglobulins
Biochemistry
Security
Diabetes
Change Password
Sign Out Endocrinology

Blood gases

CSF

© 2011 PasTest Ltd | About Us | Contact Us | Help

file:///E|/Shakhawan/Respiratory%20S/174a.htm[3/13/2012 4:04:22 PM]


MyPasTest

Main Navigation
Question Browser: MRCP 1
Home
Subscriptions Question Browser Exam Builder Saved Exams
» MRCP 1 Session Progress
• Question Browser
Questions Correct 12
• Timed Test The main limiting feature of spiral computed
tomographic scanning for pulmonary embolism is? Questions Incorrect 8
• Mock Exam
Questions Total 20
• Past Papers
High level of artefacts due to unavoidable chest Questions Percentage 60 %
• Random Questions movement during respiration
• My Performance Low sensitivity for detecting pulmonary emboli in More
• Media Bank main pulmonary arteries

• New Multimedia Technical difficulty in passing a catheter into the


pulmonary artery Reference: Normal Values
Online Extras
Long scanning time Haematology
Library
Low sensitivity for detecting pulmonary emboli in
Community subsegmental pulmonary arteries Immunoglobulins
Help Biochemistry
1683
PasTest Store Diabetes
Endocrinology
My Account
Blood gases
Profile
CSF
Newsletters
My Career and Exams
Order History
Learning Goals
Question Filters

Security

Change Password
Sign Out

© 2011 PasTest Ltd | About Us | Contact Us | Help

file:///E|/Shakhawan/Respiratory%20S/175.htm[3/13/2012 4:04:24 PM]


MyPasTest

Main Navigation
Question Browser: MRCP 1
Home
Subscriptions Question Browser Exam Builder Saved Exams
» MRCP 1 Session Progress
• Question Browser
Questions Correct 13
• Timed Test The main limiting feature of spiral computed
tomographic scanning for pulmonary embolism is? Questions Incorrect 8
• Mock Exam
Questions Total 21
• Past Papers
High level of artefacts due to Questions Percentage 61 %
• Random Questions unavoidable chest movement during
• My Performance respiration More
• Media Bank Low sensitivity for detecting pulmonary
emboli in main pulmonary arteries
• New Multimedia
Technical difficulty in passing a
Online Extras catheter into the pulmonary artery
Library Long scanning time
Community Low sensitivity for detecting pulmonary Your answer
Help emboli in subsegmental pulmonary
arteries
PasTest Store

My Account

Profile The development of fast scanning techniques with


helical (spiral) CT scanners has facilitated the use of this
Newsletters
tool in the diagnosis of acute and chronic PE. Spiral CT
My Career and Exams scanning allows imaging of the entire chest with use of
Order History intravenous contrast enhancement during a single Reference: Normal Values
breath-hold. The majority of studies performed to date
Learning Goals have shown CT angiography to be an accurate non- Haematology
Question Filters invasive tool in the diagnosis of PE at the main, lobar
and segmental pulmonary artery levels. However, CT Immunoglobulins
angiography is less accurate in imaging peripheral
Biochemistry
Security emboli in the sub-segmental arteries. The sensitivity
and specificity is generally regarded as being Diabetes
Change Password comparable to that of standard pulmonary angiography.
Sign Out Technical factors may cause approximately 5–10% of Endocrinology
CT angiography to be non-diagnostic. However, this
latter figure matches that of standard pulmonary Blood gases
angiography. CSF

1683

© 2011 PasTest Ltd | About Us | Contact Us | Help

file:///E|/Shakhawan/Respiratory%20S/175a.htm[3/13/2012 4:04:25 PM]


MyPasTest

Main Navigation
Question Browser: MRCP 1
Home
Subscriptions Question Browser Exam Builder Saved Exams
» MRCP 1 Session Progress
• Question Browser
Questions Correct 13
• Timed Test A 50-year-old hospital porter is an inpatient on the surgical
ward after a routine cholecystectomy. He normally smokes Questions Incorrect 8
• Mock Exam
30 cigarettes a day. Four days after the operation he Questions Total 21
• Past Papers begins to spike fevers and expectorate green phlegm. A
Questions Percentage 61 %
• Random Questions CXR shows lobar consolidation in his right lung. He has O 2
• My Performance saturations of 85% on air. The surgical consultant asks you More
to assess him and start him on some antibiotics.
• Media Bank
• New Multimedia Which one of the following treatments would you
choose? Reference: Normal Values
Online Extras
Haematology
Library Penicillin + Macrolide
Community Cephalosporin alone Immunoglobulins
Help Quinolone alone Biochemistry
PasTest Store Cephalosporin + aminoglycoside Diabetes
Penicillin + Flucloxacillin + Macrolide
Endocrinology
My Account
1684 Blood gases
Profile
CSF
Newsletters
My Career and Exams
Order History
Learning Goals
Question Filters

Security

Change Password
Sign Out

© 2011 PasTest Ltd | About Us | Contact Us | Help

file:///E|/Shakhawan/Respiratory%20S/176.htm[3/13/2012 4:04:26 PM]


MyPasTest

Main Navigation
Question Browser: MRCP 1
Home
Subscriptions Question Browser Exam Builder Saved Exams
» MRCP 1 Session Progress
• Question Browser
Questions Correct 13
• Timed Test A 50-year-old hospital porter is an inpatient on the surgical ward
after a routine cholecystectomy. He normally smokes 30 cigarettes Questions Incorrect 9
• Mock Exam
a day. Four days after the operation he begins to spike fevers and Questions Total 22
• Past Papers expectorate green phlegm. A CXR shows lobar consolidation in his
Questions Percentage 59 %
• Random Questions right lung. He has O 2 saturations of 85% on air. The surgical
• My Performance consultant asks you to assess him and start him on some More
antibiotics.
• Media Bank
• New Multimedia Which one of the following treatments would you choose?
Online Extras Penicillin + Macrolide
Library Cephalosporin alone
Community Quinolone alone
Help Cephalosporin + aminoglycoside Correct answer
PasTest Store Penicillin + Flucloxacillin + Macrolide Your answer

My Account

Profile
Newsletters This patient has a hospital acquired pneumonia, the third most
common hospital acquired infection after UTIs and wound
My Career and Exams infections. As they occur in hospital the pathogens involved are
Order History very different from those that cause community acquired Reference: Normal Values
pneumonia. Gram-negative organisms are far more common
Learning Goals due to: Haematology
Question Filters
Immunoglobulins
A: Colonisation of the oropharynx by Gram-negative bacilli is
very common in hospitalised patients – who have often been Biochemistry
Security on broad spectrum antibiotics already
Diabetes
Change Password B: Increased risk of micro-aspiration of nasopharyngeal
secretions Endocrinology
Sign Out
C: Patients in hospital often have decreased immune systems Blood gases
Thus antibiotics that will cover such organisms should be
instituted, most commonly as a combination therapy. A third CSF
generation cephalosporin with an aminoglycoside is the
present BTS recommendation.

It is acknowledged that local hospital prescribing policies vary


according to resistance patterns. In addition cephalosporins as a
group are less used than they were in the past, due to
associated risk of Clostridium difficile colitis.

However, recent guidelines (published 2008) exist to support


the use of cephalosporins in the clinical context described here:
see Report of the Working Party on Hospital-Acquired
Pneumonia of the British Society for Antimicrobial
Chemotherapy

http://jac.oxfordjournals.org/cgi/content/full/62/1/5#DKN162C6

1684

© 2011 PasTest Ltd | About Us | Contact Us | Help

file:///E|/Shakhawan/Respiratory%20S/176a.htm[3/13/2012 4:04:28 PM]


MyPasTest

Main Navigation
Question Browser: MRCP 1
Home
Subscriptions Question Browser Exam Builder Saved Exams
» MRCP 1 Session Progress
• Question Browser
Questions Correct 13
• Timed Test A sixty-year-old lady is seen every six months in the Chest
Questions Incorrect 9
• Mock Exam Clinic for follow-up for her cryptogenic fibrosing alveolitis.
Despite several prolonged courses of corticosteroids her Questions Total 22
• Past Papers lung function continues to deteriorate. You organise a Questions Percentage 59 %
• Random Questions repeat set of PFTs before starting her on a course of
cyclophosphamide. More
• My Performance
• Media Bank Which one of the results below would best fit this
• New Multimedia patient? (Options expressed as % predicted values)
Reference: Normal Values
Online Extras FEV1 60 FVC 65 ratio 90% Va 60 KCO 60
Haematology
Library
FEV1 60 FVC 65 ratio 90% Va 85 KCO 130
Community Immunoglobulins
FEV1 65 FVC 85 ratio 70% Va 60 KCO 60
Help Biochemistry
FEV1 65 FVC 85 ratio 70% Va 85 KCO 130
PasTest Store Diabetes
FEV1 60 FVC 65 ratio 90% Va 100 KCO 50
Endocrinology
My Account
1685 Blood gases
Profile
CSF
Newsletters
My Career and Exams
Order History
Learning Goals
Question Filters

Security

Change Password
Sign Out

© 2011 PasTest Ltd | About Us | Contact Us | Help

file:///E|/Shakhawan/Respiratory%20S/177.htm[3/13/2012 4:04:29 PM]


MyPasTest

Main Navigation
Question Browser: MRCP 1
Home
Subscriptions Question Browser Exam Builder Saved Exams
» MRCP 1 Session Progress
• Question Browser
Questions Correct 13
• Timed Test A sixty-year-old lady is seen every six months in the Chest
Clinic for follow-up for her cryptogenic fibrosing alveolitis. Questions Incorrect 10
• Mock Exam
Despite several prolonged courses of corticosteroids her Questions Total 23
• Past Papers lung function continues to deteriorate. You organise a
Questions Percentage 56 %
• Random Questions repeat set of PFTs before starting her on a course of
cyclophosphamide. More
• My Performance
• Media Bank Which one of the results below would best fit this
• New Multimedia patient? (Options expressed as % predicted values)

Online Extras FEV1 60 FVC 65 ratio 90% Va 60 Correct answer


Library KCO 60
Community FEV1 60 FVC 65 ratio 90% Va 85
Help KCO 130
PasTest Store FEV1 65 FVC 85 ratio 70% Va 60
KCO 60

My Account FEV1 65 FVC 85 ratio 70% Va 85 Your answer


KCO 130
Profile
FEV1 60 FVC 65 ratio 90% Va 100
Newsletters KCO 50
My Career and Exams
Order History Reference: Normal Values
Learning Goals Haematology
Question Filters CFA typically has a restrictive lung function pattern. Immunoglobulins
Due to the underlying parenchymal fibrosis these
patient’s lungs have very poor compliance, though Biochemistry
Security high elastic recoil. Static lung volumes therefore a
very low. Both FEV1 and FVC are reduced below Diabetes
Change Password
predicted values, but because of this high elastic Endocrinology
Sign Out recoil most forced expiratory volume will be expelled
in the first second compared to full forced expiration – Blood gases
this leading to a relatively high FER ( FEV1 /FVC ratio).
CSF
Thus using this you can discount items 3 and 4.
The DLCO, which is simply the product of Va x KCO,
is used to monitor disease progression and response
to treatment in fibrosing lung disease. In lung fibrosis
the DLCO is low, typically as a product of both low Va
and KCO, as compared to the occurrence in lung
haemorrhage where a low DLCO is a product of a very
low Va but a high KCO vs the pattern in pulmonary
vasculitis with a low KCO but often normal Va.

1685

© 2011 PasTest Ltd | About Us | Contact Us | Help

file:///E|/Shakhawan/Respiratory%20S/177a.htm[3/13/2012 4:04:31 PM]


MyPasTest

Main Navigation
Question Browser: MRCP 1
Home
Subscriptions Question Browser Exam Builder Saved Exams
» MRCP 1 Session Progress
• Question Browser
Questions Correct 13
• Timed Test A 50-year-old sales representative with a body mass index
(BMI) of 34 is referred to the Sleep Clinic because he keeps Questions Incorrect 10
• Mock Exam
falling asleep at the wheel, having had three car crashes. Questions Total 23
• Past Papers His wife complains that he keeps her awake all night
Questions Percentage 56 %
• Random Questions snoring. A sleep study confirms moderate sleep apnoea. He
has been warned not to return to driving until reviewed in More
• My Performance
6 months time. Which one of the treatments below
• Media Bank would be the most suitable first line therapy?
• New Multimedia
Long-term oxygen therapy Reference: Normal Values
Online Extras
Mandibular advancement splinting Haematology
Library
Pharyngeal wall surgery
Community Immunoglobulins
Tracheostomy
Help Biochemistry
Weight loss
PasTest Store Diabetes
1686
Endocrinology
My Account
Blood gases
Profile
CSF
Newsletters
My Career and Exams
Order History
Learning Goals
Question Filters

Security

Change Password
Sign Out

© 2011 PasTest Ltd | About Us | Contact Us | Help

file:///E|/Shakhawan/Respiratory%20S/178.htm[3/13/2012 4:04:32 PM]


MyPasTest

Main Navigation
Question Browser: MRCP 1
Home
Subscriptions Question Browser Exam Builder Saved Exams
» MRCP 1 Session Progress
• Question Browser
Questions Correct 13
• Timed Test A 50-year-old sales representative with a body mass index
(BMI) of 34 is referred to the Sleep Clinic because he keeps Questions Incorrect 11
• Mock Exam
falling asleep at the wheel, having had three car crashes. Questions Total 24
• Past Papers His wife complains that he keeps her awake all night
Questions Percentage 54 %
• Random Questions snoring. A sleep study confirms moderate sleep apnoea. He
has been warned not to return to driving until reviewed in More
• My Performance
6 months time. Which one of the treatments below
• Media Bank would be the most suitable first line therapy?
• New Multimedia
Long-term oxygen therapy
Online Extras
Mandibular advancement splinting Your answer
Library
Pharyngeal wall surgery
Community
Tracheostomy
Help
Weight loss Correct answer
PasTest Store

My Account

Profile Obstructive sleep apnoea is caused by loss of upper


airway/pharyngeal muscle tone during REM sleep which
Newsletters
leads to airway obstruction and consequent apnoeic
My Career and Exams episodes. It effects 1–2% of middle aged men. Good
Order History first line treatments in most patients are simple Reference: Normal Values
measures, such as weight loss and alcohol avoidance,
Learning Goals given that he has been warned not to drive, it seems Haematology
Question Filters reasonable to give him a chance to lose weight. Surgery
is really a last ditch attempt to solve the problem. Many Immunoglobulins
trials have looked at the effectiveness of both
Biochemistry
Security mandibular advancement splints (a tailor made mouth
piece which helps to keep the jaw forward and aids Diabetes
Change Password upper airway muscle tone when asleep) and CPAP,
Sign Out whilst CPAP is effective, the evidence for mandibular Endocrinology
advancement splints is more controversial. LTOT is
really only an adjunct in patients who have other co- Blood gases
existent lung pathologies. CSF

1686

© 2011 PasTest Ltd | About Us | Contact Us | Help

file:///E|/Shakhawan/Respiratory%20S/178a.htm[3/13/2012 4:04:34 PM]


MyPasTest

Main Navigation
Question Browser: MRCP 1
Home
Subscriptions Question Browser Exam Builder Saved Exams
» MRCP 1 Session Progress
• Question Browser
Questions Correct 13
• Timed Test A 74-year-old man, who is a retired builder of railway
carriages, presents to his GP. Over the past year or so he Questions Incorrect 11
• Mock Exam
has noticed gradually increasing shortness of breath on Questions Total 24
• Past Papers exertion. He has a frequent dry and non-productive cough.
Questions Percentage 54 %
• Random Questions On examination the GP notices that he has digital clubbing,
peripheral oedema and a raised JVP, with fine end- More
• My Performance
inspiratory crackles heard at the bases on auscultation.
• Media Bank Chest X-ray reveals irregular shadows in the lower lung
• New Multimedia zones and thickened pleural plaques affecting the lower
zones. What diagnosis would fit best with this clinical Reference: Normal Values
Online Extras history and these findings on examination and
investigation? Haematology
Library
Community Immunoglobulins
Asthma
Help Biochemistry
Bronchial carcinoma
PasTest Store Diabetes
Cryptogenic fibrosing alveolitis
Asbestosis Endocrinology
My Account
Silicosis
Blood gases
Profile
1890 CSF
Newsletters
My Career and Exams
Order History
Learning Goals
Question Filters

Security

Change Password
Sign Out

© 2011 PasTest Ltd | About Us | Contact Us | Help

file:///E|/Shakhawan/Respiratory%20S/179.htm[3/13/2012 4:04:35 PM]


MyPasTest

Main Navigation
Question Browser: MRCP 1
Home
Subscriptions Question Browser Exam Builder Saved Exams
» MRCP 1 Session Progress
• Question Browser
Questions Correct 14
• Timed Test A 74-year-old man, who is a retired builder of railway
carriages, presents to his GP. Over the past year or so he Questions Incorrect 11
• Mock Exam
has noticed gradually increasing shortness of breath on Questions Total 25
• Past Papers exertion. He has a frequent dry and non-productive cough.
Questions Percentage 56 %
• Random Questions On examination the GP notices that he has digital clubbing,
peripheral oedema and a raised JVP, with fine end- More
• My Performance
inspiratory crackles heard at the bases on auscultation.
• Media Bank Chest X-ray reveals irregular shadows in the lower lung
• New Multimedia zones and thickened pleural plaques affecting the lower
zones. What diagnosis would fit best with this clinical
Online Extras history and these findings on examination and
Library investigation?
Community
Asthma
Help
Bronchial carcinoma
PasTest Store
Cryptogenic fibrosing alveolitis
Asbestosis Your answer
My Account Silicosis
Profile
Newsletters
My Career and Exams
Order History The clue lies in this man’s occupation since asbestos Reference: Normal Values
fibres were often used in the building of railway
Learning Goals carriages. There is a long gap between exposure to the Haematology
Question Filters fibres and asbestosis of some 20–30 years, therefore
many cases are still coming to light, and quite some Immunoglobulins
time after the exposure to asbestos was curtailed.
Biochemistry
Security Currently, around 5–10 new cases per 100,000 of the
population occur per year. Without the exposure to Diabetes
Change Password asbestos option C would have been a reasonable
Sign Out alternative answer. Diagnosis is confirmed on exposure Endocrinology
history, typical imaging studies and isolation of fibres.
Arterial blood gas measurement in these patients Blood gases
usually demonstrates hypoxia, while pulmonary function CSF
tests demonstrate decreased vital capacity, decreased
total lung capacity and decreased gas transfer. Death
may occur due to cor pulmonale or asbestos exposure-
related cancer. such as mesothelioma or bronchial
carcinoma. Therapy is with oxygen by prescription and
effective management of acute infections, but decline is
usually inexorable.

1890

© 2011 PasTest Ltd | About Us | Contact Us | Help

file:///E|/Shakhawan/Respiratory%20S/179a.htm[3/13/2012 4:04:37 PM]


MyPasTest

Main Navigation
Question Browser: MRCP 1
Home
Subscriptions Question Browser Exam Builder Saved Exams
» MRCP 1 Session Progress
• Question Browser
Questions Correct 14
• Timed Test A 64-year-old demolition worker presents to his GP with a
severe dull right-sided chest pain. He first noticed it some Questions Incorrect 11
• Mock Exam
months ago and he is now having particular problems with Questions Total 25
• Past Papers shortness of breath. Apparently he was given an asthma
Questions Percentage 56 %
• Random Questions inhaler for shortness of breath around 2 years earlier. On
examination he appears to have a right-sided pleural More
• My Performance
effusion and is clubbed. He admits to being exposed to
• Media Bank asbestos. On further questioning he says his brother, who
• New Multimedia worked with him, died of ‘some sort of lung cancer’ around
2 years ago. X-ray confirms the right pleural effusion with Reference: Normal Values
Online Extras evidence of pleural plaques elsewhere.
Haematology
Library
What diagnosis would fit best with this clinical
Community Immunoglobulins
picture?
Help Biochemistry
Mesothelioma
PasTest Store Diabetes
Bronchial carcinoma
Tuberculosis Endocrinology
My Account
Asthma Blood gases
Profile
Rheumatoid arthritis CSF
Newsletters
My Career and Exams 1891
Order History
Learning Goals
Question Filters

Security

Change Password
Sign Out

© 2011 PasTest Ltd | About Us | Contact Us | Help

file:///E|/Shakhawan/Respiratory%20S/180.htm[3/13/2012 4:04:39 PM]


MyPasTest

Main Navigation
Question Browser: MRCP 1
Home
Subscriptions Question Browser Exam Builder Saved Exams
» MRCP 1 Session Progress
• Question Browser
Questions Correct 15
• Timed Test A 64-year-old demolition worker presents to his GP with a
severe dull right-sided chest pain. He first noticed it some Questions Incorrect 11
• Mock Exam
months ago and he is now having particular problems with Questions Total 26
• Past Papers shortness of breath. Apparently he was given an asthma
Questions Percentage 57 %
• Random Questions inhaler for shortness of breath around 2 years earlier. On
examination he appears to have a right-sided pleural More
• My Performance
effusion and is clubbed. He admits to being exposed to
• Media Bank asbestos. On further questioning he says his brother, who
• New Multimedia worked with him, died of ‘some sort of lung cancer’ around
2 years ago. X-ray confirms the right pleural effusion with
Online Extras evidence of pleural plaques elsewhere.
Library
What diagnosis would fit best with this clinical
Community picture?
Help
Mesothelioma Your answer
PasTest Store
Bronchial carcinoma
Tuberculosis
My Account
Asthma
Profile
Rheumatoid arthritis
Newsletters
My Career and Exams
Order History Reference: Normal Values
Learning Goals The clue here again is the occupation, and the fact that Haematology
Question Filters a co-worker had already died of a cancer related to the
chest. A restrictive lung defect, evidence of asbestosis Immunoglobulins
and now a painful pleural effusion would fit best with
mesothelioma. Diagnosis is confirmed via pleural Biochemistry
Security
biopsy, median survival after diagnosis is 2 years. Diabetes
Change Password However, the cancer is poorly responsive to any form of
intervention and the 2-year figure hides the fact that a Endocrinology
Sign Out
significant number of patients survive far less than this
2-year median figure. Blood gases

CSF
1891

© 2011 PasTest Ltd | About Us | Contact Us | Help

file:///E|/Shakhawan/Respiratory%20S/180a.htm[3/13/2012 4:04:40 PM]


MyPasTest

Main Navigation
Question Browser: MRCP 1
Home
Subscriptions Question Browser Exam Builder Saved Exams
» MRCP 1 Session Progress
• Question Browser
Questions Correct 0
• Timed Test A 35-year-old woman who has been treated for asthma
presents for review. She helps with mucking out at a horse- Questions Incorrect 0
• Mock Exam
riding stable over the weekends. She had been well until Questions Total 0
• Past Papers the past year or so, bit has since suffered recurrent chest
Questions Percentage 0%
• Random Questions infections, a dry cough and shortness of breath. There is no
haemoptysis. Full blood count reveals a raised eosinophil More
• My Performance
count. Chest X-ray reveals evidence of pulmonary
• Media Bank infiltration and some lobar consolidation and cavitation.
• New Multimedia
What is the diagnosis that fits best with this clinical Reference: Normal Values
Online Extras picture?
Haematology
Library
Aspergilloma
Community Immunoglobulins
Allergic bronchopulmonary aspergillosis (ABPA)
Help Biochemistry
Asthma
PasTest Store Diabetes
Tuberculosis
Bronchiectasis Endocrinology
My Account
Blood gases
Profile 1892
CSF
Newsletters
My Career and Exams
Order History
Learning Goals
Question Filters

Security

Change Password
Sign Out

© 2011 PasTest Ltd | About Us | Contact Us | Help

file:///E|/Shakhawan/Respiratory%20S/181.htm[3/13/2012 4:04:42 PM]


MyPasTest

Main Navigation
Question Browser: MRCP 1
Home
Subscriptions Question Browser Exam Builder Saved Exams
» MRCP 1 Session Progress
• Question Browser
Questions Correct 1
• Timed Test A 35-year-old woman who has been treated for asthma
presents for review. She helps with mucking out at a horse- Questions Incorrect 0
• Mock Exam
riding stable over the weekends. She had been well until Questions Total 1
• Past Papers the past year or so, bit has since suffered recurrent chest
Questions Percentage 100 %
• Random Questions infections, a dry cough and shortness of breath. There is no
haemoptysis. Full blood count reveals a raised eosinophil More
• My Performance
count. Chest X-ray reveals evidence of pulmonary
• Media Bank infiltration and some lobar consolidation and cavitation.
• New Multimedia
What is the diagnosis that fits best with this clinical
Online Extras picture?
Library
Aspergilloma
Community
Allergic bronchopulmonary aspergillosis Your answer
Help (ABPA)
PasTest Store Asthma
Tuberculosis
My Account Bronchiectasis
Profile
Newsletters
My Career and Exams
This woman helps out at a stables and is likely to have Reference: Normal Values
Order History
been exposed to Aspergillus fumigatus. Diffuse X-ray
Learning Goals changes, an asthmatic-type presentation originally, now Haematology
Question Filters bordering on bronchiectasis with frequent infections and
peripheral blood eosinophilia point towards ABPA. Immunoglobulins
Aspergilloma tends to become invasive, beginning in an
old cavity, such as one left by TB infection, and you Biochemistry
Security
would not expect such severe X-ray changes in Diabetes
Change Password someone with asthma. Management involves oral
prednisolone initially, and then later bronchodilators. Endocrinology
Sign Out
The X-ray changes that appear long-term are in
response to a type-2 hypersensitivity reaction. Blood gases

CSF
1892

© 2011 PasTest Ltd | About Us | Contact Us | Help

file:///E|/Shakhawan/Respiratory%20S/181a.htm[3/13/2012 4:04:43 PM]


MyPasTest

Main Navigation
Question Browser: MRCP 1
Home
Subscriptions Question Browser Exam Builder Saved Exams
» MRCP 1 Session Progress
• Question Browser
Questions Correct 1
• Timed Test A 78-year-old woman with a previous history of
tuberculosis presents to her GP. She has had three Questions Incorrect 0
• Mock Exam
episodes of haemoptysis in the past few months. As a Questions Total 1
• Past Papers previous smoker she is worried that she may have lung
Questions Percentage 100 %
• Random Questions cancer. Chest X-ray reveals an old tuberculosis cavity
containing a mass surrounded by a crescent of air. What is More
• My Performance
the diagnosis that fits best with this clinical picture?
• Media Bank
• New Multimedia Bronchial carcinoma
Reference: Normal Values
Online Extras Wegener’s granulomatosis
Aspergilloma Haematology
Library
Community Asthma Immunoglobulins
Help Reactivated tuberculosis Biochemistry
PasTest Store 1893 Diabetes
Endocrinology
My Account
Blood gases
Profile
CSF
Newsletters
My Career and Exams
Order History
Learning Goals
Question Filters

Security

Change Password
Sign Out

© 2011 PasTest Ltd | About Us | Contact Us | Help

file:///E|/Shakhawan/Respiratory%20S/182.htm[3/13/2012 4:04:44 PM]


MyPasTest

Main Navigation
Question Browser: MRCP 1
Home
Subscriptions Question Browser Exam Builder Saved Exams
» MRCP 1 Session Progress
• Question Browser
Questions Correct 2
• Timed Test A 78-year-old woman with a previous history of
tuberculosis presents to her GP. She has had three Questions Incorrect 0
• Mock Exam
episodes of haemoptysis in the past few months. As a Questions Total 2
• Past Papers previous smoker she is worried that she may have lung
Questions Percentage 100 %
• Random Questions cancer. Chest X-ray reveals an old tuberculosis cavity
containing a mass surrounded by a crescent of air. What is More
• My Performance
the diagnosis that fits best with this clinical picture?
• Media Bank
• New Multimedia Bronchial carcinoma
Online Extras Wegener’s granulomatosis
Library Aspergilloma Your answer
Community Asthma
Help Reactivated tuberculosis
PasTest Store

My Account
This woman fits the clinical picture of having developed
Profile an aspergilloma in a pre-existing lung cavity due to old
tuberculosis. Diagnosis may be confirmed by sputum
Newsletters
culture or by serum precipitating antibody. Treatment of
My Career and Exams aspergilloma is problematic. Up to 10% of aspergillomas
Order History may resolve spontaneously, other treatment possibilities Reference: Normal Values
involve surgical excision or long-term treatment with
Learning Goals oral itraconazole. Surgery should be considered as a Haematology
Question Filters first-choice option where erosion into a major vessel
and massive haemoptysis is a possibility. Immunoglobulins
Biochemistry
Security
1893 Diabetes
Change Password
Sign Out Endocrinology

Blood gases

CSF

© 2011 PasTest Ltd | About Us | Contact Us | Help

file:///E|/Shakhawan/Respiratory%20S/182a.htm[3/13/2012 4:04:46 PM]


MyPasTest

Main Navigation
Question Browser: MRCP 1
Home
Subscriptions Question Browser Exam Builder Saved Exams
» MRCP 1 Session Progress
• Question Browser
Questions Correct 2
• Timed Test A 30-year-old woman with a history of asthma presents for
Questions Incorrect 0
• Mock Exam review. She has been taking 400 μg bd of beclometasone
and salbutamol prn, which she is using with increasing Questions Total 2
• Past Papers
frequency. What is the best-fit next change to her therapy? Questions Percentage 100 %
• Random Questions
• My Performance Increase her regular repeat prescriptions for More
salbutamol
• Media Bank
Increase her inhaled beclometasone dose
• New Multimedia
Changer her inhaled steroid to fluticasone Reference: Normal Values
Online Extras
Add in a long-acting inhaled β2 -agonist Haematology
Library
Community Do nothing Immunoglobulins
Help 1894 Biochemistry
PasTest Store Diabetes
Endocrinology
My Account
Blood gases
Profile
CSF
Newsletters
My Career and Exams
Order History
Learning Goals
Question Filters

Security

Change Password
Sign Out

© 2011 PasTest Ltd | About Us | Contact Us | Help

file:///E|/Shakhawan/Respiratory%20S/183.htm[3/13/2012 4:04:47 PM]


MyPasTest

Main Navigation
Question Browser: MRCP 1
Home
Subscriptions Question Browser Exam Builder Saved Exams
» MRCP 1 Session Progress
• Question Browser
Questions Correct 3
• Timed Test A 30-year-old woman with a history of asthma presents for
Questions Incorrect 0
• Mock Exam review. She has been taking 400 μg bd of beclometasone
and salbutamol prn, which she is using with increasing Questions Total 3
• Past Papers
frequency. What is the best-fit next change to her therapy? Questions Percentage 100 %
• Random Questions
• My Performance Increase her regular repeat More
prescriptions for salbutamol
• Media Bank
Increase her inhaled beclometasone
• New Multimedia dose
Online Extras Changer her inhaled steroid to
Library fluticasone
Community Add in a long-acting inhaled β2 - Your answer
Help agonist
PasTest Store Do nothing

My Account

Profile
The current BTS/SIGN (British Thoracic Society/Scottish
Newsletters Intercollegiate Guidelines Network) guidelines support
My Career and Exams the addition of a long-acting β2 -agonist to therapy
Order History rather than increasing the inhaled steroid dose. This has Reference: Normal Values
been proved in studies to achieve better symptom
Learning Goals control than increasing the inhaled steroid dose. Fixed- Haematology
Question Filters dose combination products (Seretide and Symbicort)
containing both inhaled steroid and long-acting β2 - Immunoglobulins
agonist are available and may offer a convenient dosing Biochemistry
Security option for some patients.
Diabetes
Change Password
Sign Out Endocrinology
1894
Blood gases

CSF

© 2011 PasTest Ltd | About Us | Contact Us | Help

file:///E|/Shakhawan/Respiratory%20S/183a.htm[3/13/2012 4:04:49 PM]


MyPasTest

Main Navigation
Question Browser: MRCP 1
Home
Subscriptions Question Browser Exam Builder Saved Exams
» MRCP 1 Session Progress
• Question Browser
Questions Correct 3
• Timed Test A 72-year-old woman who smokes 5–10 cigarettes per day
and has a past history of whooping cough presents with Questions Incorrect 0
• Mock Exam
chronic cough and recurrent chest infections for review. She Questions Total 3
• Past Papers admits to producing frequent amounts of purulent sputum
Questions Percentage 100 %
• Random Questions and of intermittently suffering night sweats for a number of
months. There have also been occasional episodes of More
• My Performance
haemoptysis. Chest X-ray reveals hyperinflation, crowded
• Media Bank lung markings and small cyst-like spaces at the lung bases.
• New Multimedia What is the most likely underlying pathology?
Reference: Normal Values
Online Extras Tuberculosis
Haematology
Library Asthma
Community Immunoglobulins
Bronchiectasis
Help Chronic lung abscess Biochemistry
PasTest Store Bronchial carcinoma Diabetes

1895 Endocrinology
My Account
Blood gases
Profile
CSF
Newsletters
My Career and Exams
Order History
Learning Goals
Question Filters

Security

Change Password
Sign Out

© 2011 PasTest Ltd | About Us | Contact Us | Help

file:///E|/Shakhawan/Respiratory%20S/184.htm[3/13/2012 4:04:50 PM]


MyPasTest

Main Navigation
Question Browser: MRCP 1
Home
Subscriptions Question Browser Exam Builder Saved Exams
» MRCP 1 Session Progress
• Question Browser
Questions Correct 4
• Timed Test A 72-year-old woman who smokes 5–10 cigarettes per day
and has a past history of whooping cough presents with Questions Incorrect 0
• Mock Exam
chronic cough and recurrent chest infections for review. She Questions Total 4
• Past Papers admits to producing frequent amounts of purulent sputum
Questions Percentage 100 %
• Random Questions and of intermittently suffering night sweats for a number of
months. There have also been occasional episodes of More
• My Performance
haemoptysis. Chest X-ray reveals hyperinflation, crowded
• Media Bank lung markings and small cyst-like spaces at the lung bases.
• New Multimedia What is the most likely underlying pathology?

Online Extras Tuberculosis


Library Asthma
Community Bronchiectasis Your answer
Help Chronic lung abscess
PasTest Store Bronchial carcinoma

My Account

Profile
The past history of whooping cough and smoking,
Newsletters
coupled with the X-ray changes are very suggestive of
My Career and Exams bronchiectasis. Non-pharmacological management
Order History involves self-physiotherapy and adequate hydration. Reference: Normal Values
Chronic-intermittent, oral antibiotic therapy is used by
Learning Goals some physicians, although it may encourage multi-drug Haematology
Question Filters resistance and its use is not endorsed by all. Influenza
and pneumococcal vaccinations are strongly Immunoglobulins
recommended. Surgical referral may be recommended
Biochemistry
Security for patients with chronic severe localised infection that
fails to resolve after iv antibiotic therapy. Diabetes
Change Password
Sign Out Endocrinology
1895
Blood gases

CSF

© 2011 PasTest Ltd | About Us | Contact Us | Help

file:///E|/Shakhawan/Respiratory%20S/184a.htm[3/13/2012 4:04:51 PM]


MyPasTest

Main Navigation
Question Browser: MRCP 1
Home
Subscriptions Question Browser Exam Builder Saved Exams
» MRCP 1 Session Progress
• Question Browser
Questions Correct 4
• Timed Test A 21-year-old trainee nursery nurse presents to A&E
acutely unwell. She has a cough and has become Questions Incorrect 0
• Mock Exam
progressively worse, now with severe shortness of breath. Questions Total 4
• Past Papers Her flatmate reports that she has become unsteady on her
Questions Percentage 100 %
• Random Questions feet over the past few days. There is no important past
history of note, she smokes 20 cigarettes per day. There is More
• My Performance
a striking rash over her scalp, face and trunk with a mixed
• Media Bank crop of vesicles and pustules suggestive of chickenpox. On
• New Multimedia examination she has bilateral cerebellar ataxia.
Reference: Normal Values
Online Extras Which diagnosis best fits with this clinical picture?
Haematology
Library
Acute varicella zoster virus infection with CNS and
Community Immunoglobulins
lung involvement
Help Bacterial pneumonia Biochemistry
PasTest Store Herpes simplex encephalitis Diabetes
Generalised viral infection of unknown origin Endocrinology
My Account Likely immunocompromised patient
Blood gases
Profile
1896 CSF
Newsletters
My Career and Exams
Order History
Learning Goals
Question Filters

Security

Change Password
Sign Out

© 2011 PasTest Ltd | About Us | Contact Us | Help

file:///E|/Shakhawan/Respiratory%20S/185.htm[3/13/2012 4:04:53 PM]


MyPasTest

Main Navigation
Question Browser: MRCP 1
Home
Subscriptions Question Browser Exam Builder Saved Exams
» MRCP 1 Session Progress
• Question Browser
Questions Correct 5
• Timed Test A 21-year-old trainee nursery nurse presents to A&E
acutely unwell. She has a cough and has become Questions Incorrect 0
• Mock Exam
progressively worse, now with severe shortness of breath. Questions Total 5
• Past Papers Her flatmate reports that she has become unsteady on her
Questions Percentage 100 %
• Random Questions feet over the past few days. There is no important past
history of note, she smokes 20 cigarettes per day. There is More
• My Performance
a striking rash over her scalp, face and trunk with a mixed
• Media Bank crop of vesicles and pustules suggestive of chickenpox. On
• New Multimedia examination she has bilateral cerebellar ataxia.

Online Extras Which diagnosis best fits with this clinical picture?
Library
Acute varicella zoster virus infection Your answer
Community with CNS and lung involvement
Help Bacterial pneumonia
PasTest Store Herpes simplex encephalitis
Generalised viral infection of unknown
My Account origin
Likely immunocompromised patient
Profile
Newsletters
My Career and Exams
Order History Reference: Normal Values
Although uncommon, lung involvement with acute
Learning Goals chickenpox infection is seen. The pneumonitis generally Haematology
Question Filters begins some 1–6 days after the appearance of the skin
eruption and is more common in young adults, with Immunoglobulins
cigarette smokers being particularly at risk. This woman
was not exposed to chickenpox as a child, and exposure Biochemistry
Security
has now occurred due to her work as a nursery nurse. Diabetes
Change Password The chest radiograph normally shows diffuse lung
changes throughout both lung fields, but symptoms are Endocrinology
Sign Out
commonly worse than the X-ray picture. CNS
involvement is rarer, occurring in around 1 in 1000 Blood gases
cases, and presenting with acute truncal ataxia.
CSF
Treatment is with intravenous aciclovir, with support
from the high-dependency/intensive care unit as
needed.

1896

© 2011 PasTest Ltd | About Us | Contact Us | Help

file:///E|/Shakhawan/Respiratory%20S/185a.htm[3/13/2012 4:04:55 PM]


MyPasTest

Main Navigation
Question Browser: MRCP 1
Home
Subscriptions Question Browser Exam Builder Saved Exams
» MRCP 1 Session Progress
• Question Browser
Questions Correct 5
• Timed Test A 72-year-old man is admitted via the on-call team with an
Questions Incorrect 0
• Mock Exam acute respiratory tract infection. He has a 40 pack-year
smoking history and continues to smoke 15 cigarettes per Questions Total 5
• Past Papers day. After recovery from his initial infection, lung function Questions Percentage 100 %
• Random Questions testing indicates that his FEV1 (forced expiratory volume in
• My Performance 1 second) is less than 50% of predicted, and chronic More
• Media Bank obstructive pulmonary disease (COPD) is diagnosed. This is
his third exacerbation this year so far, which have
• New Multimedia necessitated hospital admission. What is the best Reference: Normal Values
Online Extras pharmacological intervention for him?
Haematology
Library Salbutamol prn for symptom relief
Community Immunoglobulins
Low-dose beclometasone inhaled-steroid therapy as
Help prophylaxis Biochemistry
PasTest Store High-dose inhaled steroid therapy as prophylaxis Diabetes
Inhaled anticholinergics for symptom relief
Endocrinology
My Account Combination therapy with a high-dose inhaled steroid
and a long-acting b 2 -agonist Blood gases
Profile
CSF
Newsletters 1897
My Career and Exams
Order History
Learning Goals
Question Filters

Security

Change Password
Sign Out

© 2011 PasTest Ltd | About Us | Contact Us | Help

file:///E|/Shakhawan/Respiratory%20S/186.htm[3/13/2012 4:04:56 PM]


MyPasTest

Main Navigation
Question Browser: MRCP 1
Home
Subscriptions Question Browser Exam Builder Saved Exams
» MRCP 1 Session Progress
• Question Browser
Questions Correct 5
• Timed Test A 72-year-old man is admitted via the on-call team with an
acute respiratory tract infection. He has a 40 pack-year Questions Incorrect 1
• Mock Exam
smoking history and continues to smoke 15 cigarettes per Questions Total 6
• Past Papers day. After recovery from his initial infection, lung function
Questions Percentage 83 %
• Random Questions testing indicates that his FEV1 (forced expiratory volume in
• My Performance 1 second) is less than 50% of predicted, and chronic More
• Media Bank obstructive pulmonary disease (COPD) is diagnosed. This is
his third exacerbation this year so far, which have
• New Multimedia necessitated hospital admission. What is the best
Online Extras pharmacological intervention for him?
Library Salbutamol prn for symptom relief
Community Low-dose beclometasone inhaled-
Help steroid therapy as prophylaxis
PasTest Store High-dose inhaled steroid therapy
as prophylaxis
Inhaled anticholinergics for Your answer
My Account
symptom relief
Profile Combination therapy with a high- Correct answer
Newsletters dose inhaled steroid and a long-
acting b 2 -agonist
My Career and Exams
Order History Reference: Normal Values
Learning Goals Haematology
Question Filters
Recent evidence has shown that, for patients who suffer Immunoglobulins
recurrent severe exacerbations of COPD and who have
an FEV1 less than 50% predicted, combination therapy Biochemistry
Security
with high-dose inhaled steroids and a long-acting β2 - Diabetes
Change Password agonist may reduce severe exacerbations. A long-term
Sign Out study (TORCH) has reported which narrowly failed it’s Endocrinology
primary endpoint of mortality (p=0.052), although Blood gases
fewer exacerbations and better quality of life was seen.
Of course, this combination should be used in CSF
conjunction with smoking cessation where possible, and
long-term home oxygen therapy if the criteria for its
prescription are met. COPD carries a worse long-term
survival than some cancers. Once respiratory failure
criteria have been met, the 5-year survival rate is only
around 25%. There is no value in using low-dose
inhaled steroid therapy in the management of COPD.

1897

© 2011 PasTest Ltd | About Us | Contact Us | Help

file:///E|/Shakhawan/Respiratory%20S/186a.htm[3/13/2012 4:04:57 PM]


MyPasTest

Main Navigation
Question Browser: MRCP 1
Home
Subscriptions Question Browser Exam Builder Saved Exams
» MRCP 1 Session Progress
• Question Browser
Questions Correct 5
• Timed Test A 75-year-old woman visits your COPD clinic for review.
Her blood gases were checked on her last visit 2 months Questions Incorrect 1
• Mock Exam
ago when she was relatively well, and you check them Questions Total 6
• Past Papers again today. Her p a (O2 ) on air on both occasions was only
Questions Percentage 83 %
• Random Questions 6.8 kPa. There is no CO2 retention on 28% O 2 . To her
• My Performance credit, she did succeed in stopping smoking 6 months ago. More
She is maintained on combination inhaled steroid and long-
• Media Bank
acting β2 -agonist therapy.
• New Multimedia
Reference: Normal Values
Online Extras What is the best next management step?
Haematology
Library
Continue her current treatment and review in 4
Community months time Immunoglobulins
Help Offer her oxygen cylinders for use as required Biochemistry
PasTest Store Suggest she uses an oxygen concentrator for at least Diabetes
19 h per day
Endocrinology
My Account Give her rotational antibiotics to prevent an
exacerbation Blood gases
Profile Add in an anticholinergic to her therapy
CSF
Newsletters
1898
My Career and Exams
Order History
Learning Goals
Question Filters

Security

Change Password
Sign Out

© 2011 PasTest Ltd | About Us | Contact Us | Help

file:///E|/Shakhawan/Respiratory%20S/187.htm[3/13/2012 4:04:59 PM]


MyPasTest

Main Navigation
Question Browser: MRCP 1
Home
Subscriptions Question Browser Exam Builder Saved Exams
» MRCP 1 Session Progress
• Question Browser
Questions Correct 5
• Timed Test A 75-year-old woman visits your COPD clinic for review.
Her blood gases were checked on her last visit 2 months Questions Incorrect 2
• Mock Exam
ago when she was relatively well, and you check them Questions Total 7
• Past Papers again today. Her p a (O2 ) on air on both occasions was only
Questions Percentage 71 %
• Random Questions 6.8 kPa. There is no CO2 retention on 28% O 2 . To her
• My Performance credit, she did succeed in stopping smoking 6 months ago. More
She is maintained on combination inhaled steroid and long-
• Media Bank
acting β2 -agonist therapy.
• New Multimedia
Online Extras What is the best next management step?
Library
Continue her current treatment and
Community review in 4 months time
Help Offer her oxygen cylinders for use
PasTest Store as required
Suggest she uses an oxygen Correct answer
concentrator for at least 19 h per day
My Account
Give her rotational antibiotics to
Profile prevent an exacerbation
Newsletters Add in an anticholinergic to her Your answer
therapy
My Career and Exams
Order History Reference: Normal Values
Learning Goals Haematology
Question Filters
Studies have shown that at least 15 h of oxygen Immunoglobulins
therapy per day is required to reduce the pulmonary
hypertension associated with COPD and to treat the Biochemistry
Security
underlying pathology of future right heart failure. A
Change Password reduction in the mortality rate is only seen with more
than 19 h of use per day. This cannot be realistically
Sign Out achieved via the use of cylinders, and patients should
be given a concentrator and masks to deliver the
oxygen at a concentration of 28%. In predominantly
emphysematous disease, some benefit has been
achieved via lung reduction surgery in selected patients,
but the cohort of suitable patients is unfortunately very
small.

1898

file:///E|/Shakhawan/Respiratory%20S/187a.htm[3/13/2012 4:05:00 PM]


MyPasTest

Main Navigation
Question Browser: MRCP 1
Home
Subscriptions Question Browser Exam Builder Saved Exams
» MRCP 1 Session Progress
• Question Browser
Questions Correct 5
• Timed Test A 33-year-old man presents with increasing symptoms of
severe breathlessness on exercise. Up until the last few Questions Incorrect 2
• Mock Exam
months he had been holding down a job as a successful Questions Total 7
• Past Papers salesman. There is a history of smoking 8–10 cigarettes per
Questions Percentage 71 %
• Random Questions day. His father died at a young age (below 50 years) of
severe chest disease. Routine blood tests reveal that this More
• My Performance
patient is mildly jaundiced with a bilirubin of 90 µmol/l, his
• Media Bank AST and ALT are also outside the normal range. Chest X-
• New Multimedia ray reveals basal emphysema.
Reference: Normal Values
Online Extras Which diagnosis best fits this clinical picture?
Haematology
Library
COPD secondary to excessive smoking
Community Immunoglobulins
Cirrhosis
Help Biochemistry
Stress
PasTest Store Diabetes
α-1-Antitrypsin deficiency
Gilbert's syndrome Endocrinology
My Account
Blood gases
Profile 1899
CSF
Newsletters
My Career and Exams
Order History
Learning Goals
Question Filters

Security

Change Password
Sign Out

© 2011 PasTest Ltd | About Us | Contact Us | Help

file:///E|/Shakhawan/Respiratory%20S/188.htm[3/13/2012 4:05:02 PM]


MyPasTest

Main Navigation
Question Browser: MRCP 1
Home
Subscriptions Question Browser Exam Builder Saved Exams
» MRCP 1 Session Progress
• Question Browser
Questions Correct 6
• Timed Test A 33-year-old man presents with increasing symptoms of
severe breathlessness on exercise. Up until the last few Questions Incorrect 2
• Mock Exam
months he had been holding down a job as a successful Questions Total 8
• Past Papers salesman. There is a history of smoking 8–10 cigarettes per
Questions Percentage 75 %
• Random Questions day. His father died at a young age (below 50 years) of
severe chest disease. Routine blood tests reveal that this More
• My Performance
patient is mildly jaundiced with a bilirubin of 90 µmol/l, his
• Media Bank AST and ALT are also outside the normal range. Chest X-
• New Multimedia ray reveals basal emphysema.

Online Extras Which diagnosis best fits this clinical picture?


Library
COPD secondary to excessive smoking
Community
Cirrhosis
Help
Stress
PasTest Store
α-1-Antitrypsin deficiency Your answer
Gilbert's syndrome
My Account

Profile
Newsletters
My Career and Exams This man, who is under 40 years of age, presents with
Order History breathlessness and with radiographic evidence of Reference: Normal Values
emphysema. Cigarette smoking acts synergistically to
Learning Goals promote the development of emphysema. Hereditary a- Haematology
Question Filters 1-antitrypsin deficiency accounts for around 2% of
cases of emphysema. Around 75% of patients with a-1- Immunoglobulins
antitrypsin deficiency develop chest pathology, around Biochemistry
Security 15% of patients have associated cirrhosis – the likely
diagnosis in this case. Patients with hepatic Diabetes
Change Password decompensation may be offered liver transplantation,
Sign Out those with chest pathology are strongly advised to stop Endocrinology
smoking. Around 1 in 10 northern Europeans carry a
Blood gases
gene mutation for this deficiency, heterozygotes may be
at increased risk of lung disease if they smoke, CSF
homozygotes are pre-disposed to presentation with
early emphysema.

1899

© 2011 PasTest Ltd | About Us | Contact Us | Help

file:///E|/Shakhawan/Respiratory%20S/188a.htm[3/13/2012 4:05:03 PM]


MyPasTest

Main Navigation
Question Browser: MRCP 1
Home
Subscriptions Question Browser Exam Builder Saved Exams
» MRCP 1 Session Progress
• Question Browser
Questions Correct 6
• Timed Test A 45-year-old woman visits the surgery with her 15-year-
old son who has recently been diagnosed with asthma. She Questions Incorrect 2
• Mock Exam
has studied the pathology of asthma and has a number of Questions Total 8
• Past Papers questions as to potential causative factors. Which of the
Questions Percentage 75 %
• Random Questions following responses best describes the pathology of
asthma? More
• My Performance
• Media Bank Asthma predominantly occurs due to airway
• New Multimedia hyporesponsiveness
Reference: Normal Values
Online Extras Asthma predominantly occurs due to airway
hyperresponsiveness Haematology
Library
Asthma predominantly occurs due to airflow
Community Immunoglobulins
limitation
Help Asthma predominantly occurs due to airway Biochemistry
PasTest Store inflammation
Diabetes
Asthma occurs due to a combination of airway
hyperresponsiveness, airflow limitation and airway Endocrinology
My Account
inflammation
Blood gases
Profile
1900 CSF
Newsletters
My Career and Exams
Order History
Learning Goals
Question Filters

Security

Change Password
Sign Out

© 2011 PasTest Ltd | About Us | Contact Us | Help

file:///E|/Shakhawan/Respiratory%20S/189.htm[3/13/2012 4:05:05 PM]


MyPasTest

Main Navigation
Question Browser: MRCP 1
Home
Subscriptions Question Browser Exam Builder Saved Exams
» MRCP 1 Session Progress
• Question Browser
Questions Correct 7
• Timed Test A 45-year-old woman visits the surgery with her 15-year-
old son who has recently been diagnosed with asthma. She Questions Incorrect 2
• Mock Exam
has studied the pathology of asthma and has a number of Questions Total 9
• Past Papers questions as to potential causative factors. Which of the
Questions Percentage 77 %
• Random Questions following responses best describes the pathology of
asthma? More
• My Performance
• Media Bank Asthma predominantly occurs due to
• New Multimedia airway hyporesponsiveness
Online Extras Asthma predominantly occurs due to
airway hyperresponsiveness
Library
Asthma predominantly occurs due to
Community airflow limitation
Help Asthma predominantly occurs due to
PasTest Store airway inflammation
Asthma occurs due to a combination of Your answer
airway hyperresponsiveness, airflow
My Account
limitation and airway inflammation
Profile
Newsletters
My Career and Exams
Order History Asthma is characterised by airflow limitation that is Reference: Normal Values
usually reversible spontaneously or with treatment,
Learning Goals although later on in the disease there may be an Haematology
Question Filters irreversible component to airflow limitation. There is
also airway hyperresponsiveness to a wide range of Immunoglobulins
external stimuli. A predominantly eosinophilic pattern of
Biochemistry
Security inflammation occurs, with associated plasma exudates,
oedema, mucus-plug formation, bronchial smooth Diabetes
Change Password muscle hypertrophy and long-term epithelial damage.
Sign Out In many Western countries the prevalence of asthma is Endocrinology
increasing, particularly during the second decade of life,
where it may affect up to 10–15% of the population. Blood gases

CSF

1900

© 2011 PasTest Ltd | About Us | Contact Us | Help

file:///E|/Shakhawan/Respiratory%20S/189a.htm[3/13/2012 4:05:06 PM]


MyPasTest

Main Navigation
Question Browser: MRCP 1
Home
Subscriptions Question Browser Exam Builder Saved Exams
» MRCP 1 Session Progress
• Question Browser
Questions Correct 7
• Timed Test A 32-year-old contract spray painter presents to the
respiratory clinic for review. His asthma is becoming Questions Incorrect 2
• Mock Exam
increasingly difficult to control and he now requires Questions Total 9
• Past Papers
fluticasone 500 μg/day and salmeterol 100 μg/day just to Questions Percentage 77 %
• Random Questions perform reasonable activities of daily living. He reports that
• My Performance the only time he has felt well in recent months is when he More
spent 3 weeks on holiday at his mother’s house at the
• Media Bank seaside. Chest X-ray reveals mild hyperinflation, and lung
• New Multimedia function reveals an obstructive defect. What is the
diagnosis that best fits with his symptoms? Reference: Normal Values
Online Extras
Haematology
Library Simple asthma
Community Occupational asthma Immunoglobulins
Help Bronchiectasis Biochemistry
PasTest Store a 1 -Antitrypsin deficiency Diabetes
Pulmonary fibrosis Endocrinology
My Account
1901 Blood gases
Profile
CSF
Newsletters
My Career and Exams
Order History
Learning Goals
Question Filters

Security

Change Password
Sign Out

© 2011 PasTest Ltd | About Us | Contact Us | Help

file:///E|/Shakhawan/Respiratory%20S/190.htm[3/13/2012 4:05:08 PM]


MyPasTest

Main Navigation
Question Browser: MRCP 1
Home
Subscriptions Question Browser Exam Builder Saved Exams
» MRCP 1 Session Progress
• Question Browser
Questions Correct 8
• Timed Test A 32-year-old contract spray painter presents to the
respiratory clinic for review. His asthma is becoming Questions Incorrect 2
• Mock Exam
increasingly difficult to control and he now requires Questions Total 10
• Past Papers
fluticasone 500 μg/day and salmeterol 100 μg/day just to Questions Percentage 80 %
• Random Questions perform reasonable activities of daily living. He reports that
• My Performance the only time he has felt well in recent months is when he More
spent 3 weeks on holiday at his mother’s house at the
• Media Bank seaside. Chest X-ray reveals mild hyperinflation, and lung
• New Multimedia function reveals an obstructive defect. What is the
diagnosis that best fits with his symptoms?
Online Extras
Library Simple asthma
Community Occupational asthma Your answer
Help Bronchiectasis
PasTest Store a 1 -Antitrypsin deficiency
Pulmonary fibrosis
My Account

Profile
Newsletters
He works as a paint sprayer, and asthma is known to
My Career and Exams
be associated with isocyanates that may be a
Order History component of industrial paints or lacquers. Other Reference: Normal Values
Learning Goals industries associated with occupational asthma include
welding, those who look after laboratory animals, Haematology
Question Filters farmers, millers and those who handle grain, workers Immunoglobulins
manufacturing biological washing powders, those
involved in metal refining and workers involved in the Biochemistry
Security industrial coatings’ business. The clue to his condition is
that he improved during his prolonged holiday. Workers Diabetes
Change Password
who keep a peak-flow diary usually show marked
Sign Out Endocrinology
deterioration in their peak flow associated with the
working week. Proper assessment of industrial Blood gases
processes and safety equipment should take place to
minimise any risk. CSF

1901

© 2011 PasTest Ltd | About Us | Contact Us | Help

file:///E|/Shakhawan/Respiratory%20S/190a.htm[3/13/2012 4:05:09 PM]


MyPasTest

Main Navigation
Question Browser: MRCP 1
Home
Subscriptions Question Browser Exam Builder Saved Exams
» MRCP 1 Session Progress
• Question Browser
Questions Correct 8
• Timed Test Leukotriene-receptor antagonists (LTRAs) may be used as
add-on therapy in the treatment of asthma. Which of the Questions Incorrect 2
• Mock Exam
following statements best describes how and when they Questions Total 10
• Past Papers should be used?
Questions Percentage 80 %
• Random Questions
• My Performance LTRAs should be used as initial therapy in adults More
• Media Bank They are an ideal add-in for those patients using
excessive prn salbutamol
• New Multimedia
LTRAs are usually not of therapeutic value in patients Reference: Normal Values
Online Extras with aspirin-intolerant asthma and should be avoided
in this group Haematology
Library
Community LTRAs should be add-in therapy when patients are Immunoglobulins
poorly controlled on dual therapy with high-dose
Help Biochemistry
inhaled corticosteroids and a long-acting β2 -agonist
PasTest Store Diabetes
LTRAs may elicit a delayed response in terms of
symptom improvement, and it is necessary to Endocrinology
My Account continue treatment for at least 6 months
Blood gases
Profile 1902
CSF
Newsletters
My Career and Exams
Order History
Learning Goals
Question Filters

Security

Change Password
Sign Out

© 2011 PasTest Ltd | About Us | Contact Us | Help

file:///E|/Shakhawan/Respiratory%20S/191.htm[3/13/2012 4:05:11 PM]


MyPasTest

Main Navigation
Question Browser: MRCP 1
Home
Subscriptions Question Browser Exam Builder Saved Exams
» MRCP 1 Session Progress
• Question Browser
Questions Correct 9
• Timed Test Leukotriene-receptor antagonists (LTRAs) may be used as
Questions Incorrect 2
• Mock Exam add-on therapy in the treatment of asthma. Which of the
following statements best describes how and when they Questions Total 11
• Past Papers should be used? Questions Percentage 81 %
• Random Questions
• My Performance LTRAs should be used as initial therapy More
in adults
• Media Bank
They are an ideal add-in for those
• New Multimedia patients using excessive prn salbutamol
Online Extras LTRAs are usually not of therapeutic
Library value in patients with aspirin-intolerant
asthma and should be avoided in this
Community group
Help LTRAs should be add-in therapy when Your answer
PasTest Store patients are poorly controlled on dual
therapy with high-dose inhaled
corticosteroids and a long-acting β2 -
My Account agonist
Profile LTRAs may elicit a delayed response in
terms of symptom improvement, and it
Newsletters
is necessary to continue treatment for
My Career and Exams at least 6 months
Order History Reference: Normal Values
Learning Goals Haematology
Question Filters
Immunoglobulins
LTRAs such as montelukast are given orally and are
effective in a subpopulation of asthma patients. A 4- Biochemistry
Security week treatment trial to determine symptomatic
response to the therapeutic class is normally Diabetes
Change Password
recommended. Patients who are aspirin-intolerant
Sign Out Endocrinology
appear to derive particular benefit from this therapy.
LTRAs act by inhibiting the cysteinyl leukotriene Blood gases
receptor and partially inhibiting the inflammatory
cascade associated with asthma. They should be CSF
considered as add-in therapy for those patients who are
poorly controlled on high-dose inhaled steroid and
inhaled long-acting β2 -agonists.

1902

© 2011 PasTest Ltd | About Us | Contact Us | Help

file:///E|/Shakhawan/Respiratory%20S/191a.htm[3/13/2012 4:05:12 PM]


MyPasTest

Main Navigation
Question Browser: MRCP 1
Home
Subscriptions Question Browser Exam Builder Saved Exams
» MRCP 1 Session Progress
• Question Browser
Questions Correct 9
• Timed Test A 23-year-old woman presents to A&E with acute asthma.
Questions Incorrect 2
• Mock Exam She has had a cough and progressively worsening
symptoms over the past 3 days. This will be her fourth Questions Total 11
• Past Papers hospital admission in recent years and she has had two Questions Percentage 81 %
• Random Questions previous stays on ITU. On admission to the department you
note her to be drowsy and that she has a bradycardia with More
• My Performance
poor air entry.
• Media Bank
Which of the following biochemical features on 40%
• New Multimedia
oxygen would best fit with a near fatal attack? Reference: Normal Values
Online Extras
p a (CO 2 ) 4 kPa, p a (O2 ) 10 kPa, pH 7.38 Haematology
Library
Community p a (CO 2 ) 7 kPa, p a (O2 ) 7 kPa, pH 7.18 Immunoglobulins
Help p a (CO 2 ) 5.6 kPa p a (O2 ) 8.5kPa, pH 7.42 Biochemistry
PasTest Store p a (CO 2 ) 4.5 kPa, p a (O2 ) 12 kPa, pH 7.40 Diabetes
p a (CO 2 ) 4 kPa, p a (O2 ) 11 kPa pH 7.37 Endocrinology
My Account
Blood gases
Profile 1903
CSF
Newsletters
My Career and Exams
Order History
Learning Goals
Question Filters

Security

Change Password
Sign Out

© 2011 PasTest Ltd | About Us | Contact Us | Help

file:///E|/Shakhawan/Respiratory%20S/192.htm[3/13/2012 4:05:14 PM]


MyPasTest

Main Navigation
Question Browser: MRCP 1
Home
Subscriptions Question Browser Exam Builder Saved Exams
» MRCP 1 Session Progress
• Question Browser
Questions Correct 10
• Timed Test A 23-year-old woman presents to A&E with acute asthma.
She has had a cough and progressively worsening Questions Incorrect 2
• Mock Exam
symptoms over the past 3 days. This will be her fourth Questions Total 12
• Past Papers hospital admission in recent years and she has had two
Questions Percentage 83 %
• Random Questions previous stays on ITU. On admission to the department you
note her to be drowsy and that she has a bradycardia with More
• My Performance
poor air entry.
• Media Bank
Which of the following biochemical features on 40%
• New Multimedia
oxygen would best fit with a near fatal attack?
Online Extras
Library p a (CO 2 ) 4 kPa, p a (O2 ) 10 kPa, pH
7.38
Community
p a (CO 2 ) 7 kPa, p a (O2 ) 7 kPa, pH 7.18 Your answer
Help
PasTest Store p a (CO 2 ) 5.6 kPa p a (O2 ) 8.5kPa, pH
7.42
p a (CO 2 ) 4.5 kPa, p a (O2 ) 12 kPa, pH
My Account
7.40
Profile p a (CO 2 ) 4 kPa, p a (O2 ) 11 kPa pH 7.37
Newsletters
My Career and Exams
Order History Reference: Normal Values
Learning Goals This patient is hypercapnic and hypoxic, with an Haematology
Question Filters acidosis, and is at severe risk. Continuous nebuliser
therapy with salbutamol, intravenous hydrocortisone Immunoglobulins
and urgent ITU assessment are essential. Evidence from
Biochemistry
Security a Cochrane analysis also indicates that iv magnesium
may be of value as acute therapy. It is essential that Diabetes
Change Password such patients are kept in hospital for at least 5 days, as
Sign Out sudden death due to asthma in this patient group is Endocrinology
documented to occur some 2–5 days after this type of
acute admission. Blood gases

CSF

1903

© 2011 PasTest Ltd | About Us | Contact Us | Help

file:///E|/Shakhawan/Respiratory%20S/192a.htm[3/13/2012 4:05:15 PM]


MyPasTest

Main Navigation
Question Browser: MRCP 1
Home
Subscriptions Question Browser Exam Builder Saved Exams
» MRCP 1 Session Progress
• Question Browser
Questions Correct 10
• Timed Test A 37-year-old woman visits your clinic with her 12-year-
old daughter who is affected by cystic fibrosis. She now has Questions Incorrect 2
• Mock Exam
a new partner and wants to have a second child with him. Questions Total 12
• Past Papers There are a number of questions she has about the
Questions Percentage 83 %
• Random Questions inheritance pattern of cystic fibrosis. Which of the following
statements best explains the inheritance pattern? More
• My Performance
• Media Bank Cystic fibrosis has an autosomal-dominant
• New Multimedia inheritance pattern
Reference: Normal Values
Online Extras Cystic fibrosis has a sex-linked dominant inheritance
pattern Haematology
Library
Cystic fibrosis has a sex-linked recessive inheritance
Community Immunoglobulins
pattern
Help Cystic fibrosis occurs mostly due to new mutations Biochemistry
PasTest Store Cystic fibrosis has an autosomal-recessive Diabetes
inheritance pattern
Endocrinology
My Account
1904 Blood gases
Profile
CSF
Newsletters
My Career and Exams
Order History
Learning Goals
Question Filters

Security

Change Password
Sign Out

© 2011 PasTest Ltd | About Us | Contact Us | Help

file:///E|/Shakhawan/Respiratory%20S/193.htm[3/13/2012 4:05:17 PM]


MyPasTest

Main Navigation
Question Browser: MRCP 1
Home
Subscriptions Question Browser Exam Builder Saved Exams
» MRCP 1 Session Progress
• Question Browser
Questions Correct 11
• Timed Test A 37-year-old woman visits your clinic with her 12-year-
old daughter who is affected by cystic fibrosis. She now has Questions Incorrect 2
• Mock Exam
a new partner and wants to have a second child with him. Questions Total 13
• Past Papers There are a number of questions she has about the
Questions Percentage 84 %
• Random Questions inheritance pattern of cystic fibrosis. Which of the following
statements best explains the inheritance pattern? More
• My Performance
• Media Bank Cystic fibrosis has an autosomal-
• New Multimedia dominant inheritance pattern
Online Extras Cystic fibrosis has a sex-linked
dominant inheritance pattern
Library
Cystic fibrosis has a sex-linked
Community recessive inheritance pattern
Help Cystic fibrosis occurs mostly due to
PasTest Store new mutations
Cystic fibrosis has an autosomal- Your answer
recessive inheritance pattern
My Account

Profile
Newsletters
My Career and Exams Cystic fibrosis is the most common fatal inherited
Order History condition affecting Caucasians. It occurs with a carrier Reference: Normal Values
frequency of 1 in 22, and has an autosomal-recessive
Learning Goals pattern of inheritance. The gene mutation occurs on the Haematology
Question Filters long arm of chromosome 7 and the commonest
abnormality is a deletion at position 508 in the amino- Immunoglobulins
acid sequence. This mutation leads to a mutation in the
Biochemistry
Security cystic fibrosis transmembrane conductance regulator
(CFTR), a chloride channel. In turn, this mutation leads Diabetes
Change Password to abnormally thick and viscous secretions, and later
Sign Out problems with recurrent respiratory tract infections and Endocrinology
pancreatic dysfunction.
Blood gases

CSF
1904

© 2011 PasTest Ltd | About Us | Contact Us | Help

file:///E|/Shakhawan/Respiratory%20S/193a.htm[3/13/2012 4:05:18 PM]


MyPasTest

Main Navigation
Question Browser: MRCP 1
Home
Subscriptions Question Browser Exam Builder Saved Exams
» MRCP 1 Session Progress
• Question Browser
Questions Correct 11
• Timed Test A 39-year-old woman attends her GP complaining of a
chronic cough. Her only past history of note includes Questions Incorrect 2
• Mock Exam
trandolapril for hypertension, which is now well controlled. Questions Total 13
• Past Papers She is a non-smoker. Chest X-ray was unremarkable. The
Questions Percentage 84 %
• Random Questions cough is becoming socially embarrassing and she is seeking
help and reassurance. What is the most likely cause? More
• My Performance
• Media Bank Her trandolapril therapy
• New Multimedia Anxiety Reference: Normal Values
Online Extras Asthma
Haematology
Library Occupational asthma
Community Chronic sinusitis Immunoglobulins
Help Biochemistry
1905
PasTest Store Diabetes
Endocrinology
My Account
Blood gases
Profile
CSF
Newsletters
My Career and Exams
Order History
Learning Goals
Question Filters

Security

Change Password
Sign Out

© 2011 PasTest Ltd | About Us | Contact Us | Help

file:///E|/Shakhawan/Respiratory%20S/194.htm[3/13/2012 4:05:20 PM]


MyPasTest

Main Navigation
Question Browser: MRCP 1
Home
Subscriptions Question Browser Exam Builder Saved Exams
» MRCP 1 Session Progress
• Question Browser
Questions Correct 12
• Timed Test A 39-year-old woman attends her GP complaining of a
chronic cough. Her only past history of note includes Questions Incorrect 2
• Mock Exam
trandolapril for hypertension, which is now well controlled. Questions Total 14
• Past Papers She is a non-smoker. Chest X-ray was unremarkable. The
Questions Percentage 85 %
• Random Questions cough is becoming socially embarrassing and she is seeking
help and reassurance. What is the most likely cause? More
• My Performance
• Media Bank Her trandolapril therapy Your answer
• New Multimedia Anxiety
Online Extras Asthma
Library Occupational asthma
Community Chronic sinusitis
Help
PasTest Store

My Account ACE inhibitor-related cough occurs in up to 15% of


patients taking the therapy: probably due to changes in
Profile prostaglandin synthesis associated with ACE inhibitor
use. Standard investigations for chronic cough where
Newsletters
the chest X-ray is unremarkable include sinus CT and
My Career and Exams ENT examination, lung function and cough provocation
Order History testing, oesophageal pH testing, fibre-optic Reference: Normal Values
bronchoscopy and, finally, psychological assessment
Learning Goals could be considered. Haematology
Question Filters
Immunoglobulins
1905 Biochemistry
Security
Diabetes
Change Password
Sign Out Endocrinology

Blood gases

CSF

© 2011 PasTest Ltd | About Us | Contact Us | Help

file:///E|/Shakhawan/Respiratory%20S/194a.htm[3/13/2012 4:05:21 PM]


MyPasTest

Main Navigation
Question Browser: MRCP 1
Home
Subscriptions Question Browser Exam Builder Saved Exams
» MRCP 1 Session Progress
• Question Browser
Questions Correct 12
• Timed Test A 45-year-old oil executive returns from Indonesia for
investigation after an extended period of illness. During his Questions Incorrect 2
• Mock Exam
stay he has developed asthma, associated with progressive Questions Total 14
• Past Papers malaise and weight loss. He is now lethargic and has lost
Questions Percentage 85 %
• Random Questions 6.4 kg (1 stone) in the past 3 months. His chest X-ray is
abnormal with basilar reticulonodular and alveolar More
• My Performance
infiltrates. Blood testing reveals a marked peripheral blood
• Media Bank eosinophilia. The filarial complement-fixation test is
• New Multimedia positive. What is the most likely cause of this clinical?
Reference: Normal Values
Online Extras Tuberculosis
Haematology
Library Invasive aspergillosis
Community Immunoglobulins
Infection with Wuchereria bancrofti
Help Brucellosis Biochemistry
PasTest Store Sarcoidosis Diabetes

1906 Endocrinology
My Account
Blood gases
Profile
CSF
Newsletters
My Career and Exams
Order History
Learning Goals
Question Filters

Security

Change Password
Sign Out

© 2011 PasTest Ltd | About Us | Contact Us | Help

file:///E|/Shakhawan/Respiratory%20S/195.htm[3/13/2012 4:05:22 PM]


MyPasTest

Main Navigation
Question Browser: MRCP 1
Home
Subscriptions Question Browser Exam Builder Saved Exams
» MRCP 1 Session Progress
• Question Browser
Questions Correct 13
• Timed Test A 45-year-old oil executive returns from Indonesia for
investigation after an extended period of illness. During his Questions Incorrect 2
• Mock Exam
stay he has developed asthma, associated with progressive Questions Total 15
• Past Papers malaise and weight loss. He is now lethargic and has lost
Questions Percentage 86 %
• Random Questions 6.4 kg (1 stone) in the past 3 months. His chest X-ray is
abnormal with basilar reticulonodular and alveolar More
• My Performance
infiltrates. Blood testing reveals a marked peripheral blood
• Media Bank eosinophilia. The filarial complement-fixation test is
• New Multimedia positive. What is the most likely cause of this clinical?

Online Extras Tuberculosis


Library Invasive aspergillosis
Community Infection with Wuchereria bancrofti Your answer
Help Brucellosis
PasTest Store Sarcoidosis

My Account

Profile
This is tropical pulmonary eosinophilia associated with
Newsletters
infection from W. bancrofti (a condition associated with
My Career and Exams microfilaria). Treatment of choice is a 10–14-day course
Order History of diethylcarbamazine to eradicate the infection. The Reference: Normal Values
condition is seen in the Asian subcontinent and presents
Learning Goals with cough, wheeze, fever, lassitude and weight loss. Haematology
Question Filters
Immunoglobulins
1906 Biochemistry
Security
Diabetes
Change Password
Sign Out Endocrinology

Blood gases

CSF

© 2011 PasTest Ltd | About Us | Contact Us | Help

file:///E|/Shakhawan/Respiratory%20S/195a.htm[3/13/2012 4:05:24 PM]


MyPasTest

Main Navigation
Question Browser: MRCP 1
Home
Subscriptions Question Browser Exam Builder Saved Exams
» MRCP 1 Session Progress
• Question Browser
Questions Correct 13
• Timed Test A 23-year-old smoker of 30 cigarettes per day presents
with rapidly increasing shortness of breath, cough and Questions Incorrect 2
• Mock Exam
haemoptysis over the past month following a chest Questions Total 15
• Past Papers infection. His chest X-ray demonstrates diffuse alveolar
Questions Percentage 86 %
• Random Questions infiltrates. Blood testing indicates raised urea and
creatinine levels, with positive anti-GBM antibodies. Urine More
• My Performance
analysis reveals haematuria and proteinuria. Pulmonary
• Media Bank transfer factor is elevated. What diagnosis best fits this
• New Multimedia clinical picture?
Reference: Normal Values
Online Extras Wegener’s granulomatosis
Haematology
Library Goodpasture’s syndrome
Community Immunoglobulins
Drug-induced renopulmonary disease
Help Systemic lupus erythematosus Biochemistry
PasTest Store Idiopathic glomerulonephritis Diabetes

1907 Endocrinology
My Account
Blood gases
Profile
CSF
Newsletters
My Career and Exams
Order History
Learning Goals
Question Filters

Security

Change Password
Sign Out

© 2011 PasTest Ltd | About Us | Contact Us | Help

file:///E|/Shakhawan/Respiratory%20S/196.htm[3/13/2012 4:05:25 PM]


MyPasTest

Main Navigation
Question Browser: MRCP 1
Home
Subscriptions Question Browser Exam Builder Saved Exams
» MRCP 1 Session Progress
• Question Browser
Questions Correct 14
• Timed Test A 23-year-old smoker of 30 cigarettes per day presents
with rapidly increasing shortness of breath, cough and Questions Incorrect 2
• Mock Exam
haemoptysis over the past month following a chest Questions Total 16
• Past Papers infection. His chest X-ray demonstrates diffuse alveolar
Questions Percentage 87 %
• Random Questions infiltrates. Blood testing indicates raised urea and
creatinine levels, with positive anti-GBM antibodies. Urine More
• My Performance
analysis reveals haematuria and proteinuria. Pulmonary
• Media Bank transfer factor is elevated. What diagnosis best fits this
• New Multimedia clinical picture?

Online Extras Wegener’s granulomatosis


Library Goodpasture’s syndrome Your answer
Community Drug-induced renopulmonary disease
Help Systemic lupus erythematosus
PasTest Store Idiopathic glomerulonephritis

My Account

Profile
The picture of renal and pulmonary symptoms with
Newsletters
evidence of glomerulonephritis and intrapulmonary
My Career and Exams haemorrhage is particularly suggestive of Goodpasture’s
Order History syndrome, especially with the positive anti-GBM Reference: Normal Values
antibodies. The condition has a male:female ratio of
Learning Goals 6:1, accounts for 5% of cases of rapidly progressive Haematology
Question Filters glomerulonephritis and 80% of patients are HLA-DR2
positive. Management is with prednisolone and Immunoglobulins
cyclophosphamide, dialysis support where necessary
Biochemistry
Security and plasma-exchange therapy in centres where it is
offered. Diabetes
Change Password
Sign Out Endocrinology
1907
Blood gases

CSF

© 2011 PasTest Ltd | About Us | Contact Us | Help

file:///E|/Shakhawan/Respiratory%20S/196a.htm[3/13/2012 4:05:27 PM]


MyPasTest

Main Navigation
Question Browser: MRCP 1
Home
Subscriptions Question Browser Exam Builder Saved Exams
» MRCP 1 Session Progress
• Question Browser
Questions Correct 14
• Timed Test A 35-year-old woman visits her GP complaining of a dry
non-productive cough and lethargy. There has been slow Questions Incorrect 2
• Mock Exam
weight loss over the past few months and there is a Questions Total 16
• Past Papers nodular rash on her shins. Blood testing reveals mild
Questions Percentage 87 %
• Random Questions hypercalcaemia, a non-specific increase in
immunoglobulins, normochromic normocytic anaemia and a More
• My Performance
raised serum ACE level. A chest X-ray reveals bilateral hilar
• Media Bank lymphadenopathy. Tuberculin skin testing is negative. What
• New Multimedia diagnosis fits best with this clinical picture?
Reference: Normal Values
Online Extras Sarcoidosis
Haematology
Library Tuberculosis
Community Immunoglobulins
Lymphoma
Help Lymphangitis carcinomatosa Biochemistry
PasTest Store Cryptogenic fibrosing alveolitis Diabetes

1908 Endocrinology
My Account
Blood gases
Profile
CSF
Newsletters
My Career and Exams
Order History
Learning Goals
Question Filters

Security

Change Password
Sign Out

© 2011 PasTest Ltd | About Us | Contact Us | Help

file:///E|/Shakhawan/Respiratory%20S/197.htm[3/13/2012 4:05:28 PM]


MyPasTest

Main Navigation
Question Browser: MRCP 1
Home
Subscriptions Question Browser Exam Builder Saved Exams
» MRCP 1 Session Progress
• Question Browser
Questions Correct 15
• Timed Test A 35-year-old woman visits her GP complaining of a dry
non-productive cough and lethargy. There has been slow Questions Incorrect 2
• Mock Exam
weight loss over the past few months and there is a Questions Total 17
• Past Papers nodular rash on her shins. Blood testing reveals mild
Questions Percentage 88 %
• Random Questions hypercalcaemia, a non-specific increase in
immunoglobulins, normochromic normocytic anaemia and a More
• My Performance
raised serum ACE level. A chest X-ray reveals bilateral hilar
• Media Bank lymphadenopathy. Tuberculin skin testing is negative. What
• New Multimedia diagnosis fits best with this clinical picture?

Online Extras Sarcoidosis Your answer


Library Tuberculosis
Community Lymphoma
Help Lymphangitis carcinomatosa
PasTest Store Cryptogenic fibrosing alveolitis

My Account

Profile
Sarcoid is most common in young women and the UK
Newsletters
prevalence is approximately 19 per 100,000 of the
My Career and Exams population. It is a multisystem granulomatous disorder:
Order History 50% of sufferers present with respiratory symptoms, or Reference: Normal Values
abnormalities on chest X-ray; 10% of cases are
Learning Goals associated with erythema nodosum; 25% with uveitis; Haematology
Question Filters and 10% of established cases are associated with
hypercalcaemia. A definitive diagnosis is usually Immunoglobulins
obtained via transbronchial biopsy in over 90% of
Biochemistry
Security cases. The disease severity varies amongst racial
groups, and is more severe in Black populations. Diabetes
Change Password Treatment with corticosteroids is usual for severe
Sign Out disease or persistent changes on chest X-ray, but Endocrinology
mortality in the UK from sarcoidosis is probably less
than 5%. Blood gases

CSF

1908

© 2011 PasTest Ltd | About Us | Contact Us | Help

file:///E|/Shakhawan/Respiratory%20S/197a.htm[3/13/2012 4:05:30 PM]


MyPasTest

Main Navigation
Question Browser: MRCP 1
Home
Subscriptions Question Browser Exam Builder Saved Exams
» MRCP 1 Session Progress
• Question Browser
Questions Correct 15
• Timed Test A 42-year-old has had Raynaud’s phenomenon for many
years, but now presents to her GP with non-specific Questions Incorrect 2
• Mock Exam
symptoms of shortness of breath and a dry non-productive Questions Total 17
• Past Papers cough, as well as heartburn and a flitting arthralgia
Questions Percentage 88 %
• Random Questions affecting a number of small joints. On examination the GP
notices that her face appears a little waxy and More
• My Performance
expressionless and that her hands are cold with nodular
• Media Bank thickening of the skin over the fingers. There is
• New Multimedia telangiectasia. The most striking abnormality on respiratory
examination are fine end-expiratory crackles on chest Reference: Normal Values
Online Extras auscultation. Chest X-ray reveals a fibrotic lung picture.
Antinuclear antibodies are positive. What diagnosis fits best Haematology
Library
with this clinical picture? Immunoglobulins
Community
Help Amyloidosis Biochemistry
PasTest Store Cryptogenic fibrosing alveolitis Diabetes
Systemic lupus erythematosus
Endocrinology
My Account Progressive systemic sclerosis
Blood gases
Profile Primary pulmonary hypertension
CSF
Newsletters 1909
My Career and Exams
Order History
Learning Goals
Question Filters

Security

Change Password
Sign Out

© 2011 PasTest Ltd | About Us | Contact Us | Help

file:///E|/Shakhawan/Respiratory%20S/198.htm[3/13/2012 4:05:31 PM]


MyPasTest

Main Navigation
Question Browser: MRCP 1
Home
Subscriptions Question Browser Exam Builder Saved Exams
» MRCP 1 Session Progress
• Question Browser
Questions Correct 16
• Timed Test A 42-year-old has had Raynaud’s phenomenon for many
years, but now presents to her GP with non-specific Questions Incorrect 2
• Mock Exam
symptoms of shortness of breath and a dry non-productive Questions Total 18
• Past Papers cough, as well as heartburn and a flitting arthralgia
Questions Percentage 88 %
• Random Questions affecting a number of small joints. On examination the GP
notices that her face appears a little waxy and More
• My Performance
expressionless and that her hands are cold with nodular
• Media Bank thickening of the skin over the fingers. There is
• New Multimedia telangiectasia. The most striking abnormality on respiratory
examination are fine end-expiratory crackles on chest
Online Extras auscultation. Chest X-ray reveals a fibrotic lung picture.
Library Antinuclear antibodies are positive. What diagnosis fits best
with this clinical picture?
Community
Help Amyloidosis
PasTest Store Cryptogenic fibrosing alveolitis
Systemic lupus erythematosus
My Account Progressive systemic sclerosis Your answer

Profile Primary pulmonary hypertension

Newsletters
My Career and Exams
Order History Reference: Normal Values
The picture of Raynaud’s disease, peripheral calcinosis,
Learning Goals oesophageal symptoms, sclerodactyly, telangiectasia Haematology
Question Filters and evidence of pulmonary fibrosis is highly suggestive
of progressive systemic sclerosis, indeed of the CREST Immunoglobulins
syndrome. Positive antinuclear antibodies support the
Biochemistry
Security diagnosis. Some 30% of patients are also rheumatoid
factor-positive. At present, there are no proven disease- Diabetes
Change Password modifying agents and patients suffer from progressive
Sign Out symptoms including worsening pulmonary hypertension: Endocrinology
however, penicillamine does show at least some
promise in this area. Calcium antagonists may prove Blood gases
useful in the treatment of Raynaud’s, and there are a CSF
number of therapies that can be employed for
symptomatic reflux disease.

1909

© 2011 PasTest Ltd | About Us | Contact Us | Help

file:///E|/Shakhawan/Respiratory%20S/198a.htm[3/13/2012 4:05:33 PM]


MyPasTest

Main Navigation
Question Browser: MRCP 1
Home
Subscriptions Question Browser Exam Builder Saved Exams
» MRCP 1 Session Progress
• Question Browser
Questions Correct 16
• Timed Test A 61-year-old man with a 40 pack-year smoking history
presents with chronic cough, haemoptysis and weight loss. Questions Incorrect 2
• Mock Exam
Unfortunately his chest X-ray reveals a large mass at the Questions Total 18
• Past Papers left hilum, suggestive of a bronchial carcinoma. A raised
Questions Percentage 88 %
• Random Questions calcium is noted on routine blood work-up.
• My Performance More
What type of carcinoma would best fit this clinical
• Media Bank picture?
• New Multimedia
Adenocarcinoma of the bronchus Reference: Normal Values
Online Extras
Squamous cell-carcinoma of the bronchus Haematology
Library
Small-cell bronchial carcinoma
Community Immunoglobulins
Large-cell bronchial carcinoma
Help Biochemistry
Bronchioalveolar-cell carcinoma
PasTest Store Diabetes
1931
Endocrinology
My Account
Blood gases
Profile
CSF
Newsletters
My Career and Exams
Order History
Learning Goals
Question Filters

Security

Change Password
Sign Out

© 2011 PasTest Ltd | About Us | Contact Us | Help

file:///E|/Shakhawan/Respiratory%20S/199.htm[3/13/2012 4:05:34 PM]


MyPasTest

Main Navigation
Question Browser: MRCP 1
Home
Subscriptions Question Browser Exam Builder Saved Exams
» MRCP 1 Session Progress
• Question Browser
Questions Correct 17
• Timed Test A 61-year-old man with a 40 pack-year smoking history
presents with chronic cough, haemoptysis and weight loss. Questions Incorrect 2
• Mock Exam
Unfortunately his chest X-ray reveals a large mass at the Questions Total 19
• Past Papers left hilum, suggestive of a bronchial carcinoma. A raised
Questions Percentage 89 %
• Random Questions calcium is noted on routine blood work-up.
• My Performance More
What type of carcinoma would best fit this clinical
• Media Bank picture?
• New Multimedia
Adenocarcinoma of the bronchus
Online Extras
Squamous cell-carcinoma of the Your answer
Library bronchus
Community Small-cell bronchial carcinoma
Help Large-cell bronchial carcinoma
PasTest Store Bronchioalveolar-cell carcinoma

My Account

Profile
Squamous-cell and small-cell bronchial carcinomas tend
Newsletters to occupy a central location. Squamous-cell carcinomas
My Career and Exams are more frequently associated with hypercalcaemia, so
the correct answer in this case is option B. Reference: Normal Values
Order History
Learning Goals The other options tend to occur more frequently in the Haematology
periphery. In terms of frequency, adenocarcinomas and
Question Filters
squamous-cell carcinomas are seen most frequently, Immunoglobulins
having an incidence of around 30% each, small-cell
carcinomas around 20%, large cell carcinomas 15-20% Biochemistry
Security
and bronchioalveolar-cell carcinomas around 5% of the
Diabetes
Change Password total. Bronchioalveolar-cell carcinoma has no correlation
with cigarette smoking. Endocrinology
Sign Out
Blood gases
1931
CSF

© 2011 PasTest Ltd | About Us | Contact Us | Help

file:///E|/Shakhawan/Respiratory%20S/199a.htm[3/13/2012 4:05:35 PM]


MyPasTest

Main Navigation
Question Browser: MRCP 1
Home
Subscriptions Question Browser Exam Builder Saved Exams
» MRCP 1 Session Progress
• Question Browser
Questions Correct 17
• Timed Test A 73-year-old woman presents with weight loss and a
chronic cough. Her husband has noticed that her pupil is Questions Incorrect 2
• Mock Exam
constricted and her right eyelid is drooping. She has had Questions Total 19
• Past Papers pain in her right shoulder for some months, which her GP
Questions Percentage 89 %
• Random Questions has described as 'probable rheumatism'. Unfortunately her
chest x-ray reveals a mass in the right lung apex with More
• My Performance
possible lymphadenopathy at the right hilum.
• Media Bank
What is the most likely diagnosis in this case?
• New Multimedia
Reference: Normal Values
Online Extras Eaton-Lambert Syndrome
Haematology
Library Horner’s syndrome caused by Pancoast’s tumour
Community Small-cell carcinoma Immunoglobulins
Help Tuberculosis Biochemistry
PasTest Store Aspergilloma Diabetes

1932 Endocrinology
My Account
Blood gases
Profile
CSF
Newsletters
My Career and Exams
Order History
Learning Goals
Question Filters

Security

Change Password
Sign Out

© 2011 PasTest Ltd | About Us | Contact Us | Help

file:///E|/Shakhawan/Respiratory%20S/200.htm[3/13/2012 4:05:37 PM]


MyPasTest

Main Navigation
Question Browser: MRCP 1
Home
Subscriptions Question Browser Exam Builder Saved Exams
» MRCP 1 Session Progress
• Question Browser
Questions Correct 18
• Timed Test A 73-year-old woman presents with weight loss and a
chronic cough. Her husband has noticed that her pupil is Questions Incorrect 2
• Mock Exam
constricted and her right eyelid is drooping. She has had Questions Total 20
• Past Papers pain in her right shoulder for some months, which her GP
Questions Percentage 90 %
• Random Questions has described as 'probable rheumatism'. Unfortunately her
chest x-ray reveals a mass in the right lung apex with More
• My Performance
possible lymphadenopathy at the right hilum.
• Media Bank
What is the most likely diagnosis in this case?
• New Multimedia
Online Extras Eaton-Lambert Syndrome
Library Horner’s syndrome caused by Your answer
Community Pancoast’s tumour
Help Small-cell carcinoma
PasTest Store Tuberculosis
Aspergilloma

My Account

Profile
Newsletters Eaton-Lambert syndrome is a paraneoplastic syndrome
My Career and Exams associated with proximal myopathy, related to a
deficient action of cholinergic neurones. Horner’s Reference: Normal Values
Order History
syndrome is caused by an apical lung tumour, which
Learning Goals leads to spinal cord damage between spinal levels C8 Haematology
Question Filters and T1. Symptoms are pupil constriction, ptosis and
facial anhydrosis. Pancoast’s tumours are mostly Immunoglobulins
squamous or adenocarcinomas, only 3-5% are said to
be due to small cell tumours, this is because small-cell Biochemistry
Security
carcinoma of the bronchus is more likely to occur Diabetes
Change Password centrally and so wouldn’t be expected to present like
this. It is associated with SIADH in 5-10% of cases. Endocrinology
Sign Out
Blood gases
1932 CSF

© 2011 PasTest Ltd | About Us | Contact Us | Help

file:///E|/Shakhawan/Respiratory%20S/200a.htm[3/13/2012 4:05:38 PM]


MyPasTest

Main Navigation
Question Browser: MRCP 1
Home
Subscriptions Question Browser Exam Builder Saved Exams
» MRCP 1 Session Progress
• Question Browser
Questions Correct 0
• Timed Test A 68-year-old man presents with sudden-onset, left-sided
weakness. He has a past history of smoking but nothing Questions Incorrect 0
• Mock Exam
else. There is a suggestion of papilloedema on fundoscopy. Questions Total 0
• Past Papers His chest X-ray on admission to A&E reveals a right middle
Questions Percentage 0%
• Random Questions and lower lobe consolidation, with evidence of a mass at
the right hilum. Blood testing reveals a normochromic More
• My Performance
normocytic anaemia with a calcium concentration of 2.9
• Media Bank mmol/l and a mildly raised urea and creatinine
• New Multimedia concentrations.
Reference: Normal Values
Online Extras What is the best next stage in your management
plan? Haematology
Library
Community Immunoglobulins
Urgent non-contrast CT brain scan
Help Urgent contrast CT brain scan Biochemistry
PasTest Store CT brain scan after interval of 3-5 days Diabetes
Arrange urgent treatment of his hypercalcaemia Endocrinology
My Account Admit for bronchoscopy
Blood gases
Profile
1933 CSF
Newsletters
My Career and Exams
Order History
Learning Goals
Question Filters

Security

Change Password
Sign Out

© 2011 PasTest Ltd | About Us | Contact Us | Help

file:///E|/Shakhawan/Respiratory%20S/201.htm[3/13/2012 4:05:40 PM]


MyPasTest

Main Navigation
Question Browser: MRCP 1
Home
Subscriptions Question Browser Exam Builder Saved Exams
» MRCP 1 Session Progress
• Question Browser
Questions Correct 0
• Timed Test A 68-year-old man presents with sudden-onset, left-sided
weakness. He has a past history of smoking but nothing Questions Incorrect 1
• Mock Exam
else. There is a suggestion of papilloedema on fundoscopy. Questions Total 1
• Past Papers His chest X-ray on admission to A&E reveals a right middle
Questions Percentage 0%
• Random Questions and lower lobe consolidation, with evidence of a mass at
the right hilum. Blood testing reveals a normochromic More
• My Performance
normocytic anaemia with a calcium concentration of 2.9
• Media Bank mmol/l and a mildly raised urea and creatinine
• New Multimedia concentrations.

Online Extras What is the best next stage in your management


Library plan?

Community Urgent non-contrast CT brain scan


Help Urgent contrast CT brain scan Correct answer
PasTest Store CT brain scan after interval of 3-5
days
My Account Arrange urgent treatment of his
hypercalcaemia
Profile
Admit for bronchoscopy Your answer
Newsletters
My Career and Exams
Order History Reference: Normal Values
Learning Goals There is a suggestion that this man has a bronchial Haematology
Question Filters carcinoma in a central location, he is a smoker and the
possibility of small-cell carcinoma with brain metastases Immunoglobulins
is raised. He needs an urgent contrast brain CT scan to
discover the cause of his neurological symptoms. If his Biochemistry
Security
sudden weakness is due to brain metastases then he Diabetes
Change Password would be started on urgent dexamethasone and should
be assessed for cranial irradiation. Prognosis of small- Endocrinology
Sign Out
cell carcinoma of the bronchus is appalling, with a
median survival of 9 months for patients with extensive Blood gases
disease.
CSF

1933

© 2011 PasTest Ltd | About Us | Contact Us | Help

file:///E|/Shakhawan/Respiratory%20S/201a.htm[3/13/2012 4:05:41 PM]


MyPasTest

Main Navigation
Question Browser: MRCP 1
Home
Subscriptions Question Browser Exam Builder Saved Exams
» MRCP 1 Session Progress
• Question Browser
Questions Correct 0
• Timed Test A 53-year-old parrot expert presents for review. For some
months he has suffered increasing tiredness, high fevers at Questions Incorrect 1
• Mock Exam
night, cough and muscle ache. Chest X-ray reveals a Questions Total 1
• Past Papers diffuse pneumonic picture. His white blood cell count is
Questions Percentage 0%
• Random Questions normal, transaminases are just above the normal range.
Two sets of blood cultures prove negative. More
• My Performance
• Media Bank What diagnosis fits best with this clinical picture?
• New Multimedia
Streptococcal Pneumonia Reference: Normal Values
Online Extras
Tuberculosis Haematology
Library
Chlamydia psittaci pneumonia
Community Immunoglobulins
Chlamydia pneumoniae pneumonia
Help Biochemistry
Mycoplasma pneumonia
PasTest Store Diabetes
1934
Endocrinology
My Account
Blood gases
Profile
CSF
Newsletters
My Career and Exams
Order History
Learning Goals
Question Filters

Security

Change Password
Sign Out

© 2011 PasTest Ltd | About Us | Contact Us | Help

file:///E|/Shakhawan/Respiratory%20S/202.htm[3/13/2012 4:05:43 PM]


MyPasTest

Main Navigation
Question Browser: MRCP 1
Home
Subscriptions Question Browser Exam Builder Saved Exams
» MRCP 1 Session Progress
• Question Browser
Questions Correct 1
• Timed Test A 53-year-old parrot expert presents for review. For some
months he has suffered increasing tiredness, high fevers at Questions Incorrect 1
• Mock Exam
night, cough and muscle ache. Chest X-ray reveals a Questions Total 2
• Past Papers diffuse pneumonic picture. His white blood cell count is
Questions Percentage 50 %
• Random Questions normal, transaminases are just above the normal range.
Two sets of blood cultures prove negative. More
• My Performance
• Media Bank What diagnosis fits best with this clinical picture?
• New Multimedia
Streptococcal Pneumonia
Online Extras
Tuberculosis
Library
Chlamydia psittaci pneumonia Your answer
Community
Chlamydia pneumoniae pneumonia
Help
Mycoplasma pneumonia
PasTest Store

My Account

Profile Chlamydia psittacipneumonia was originally described in


relation to those who kept the psittacine family of birds
Newsletters (e.g. parrots). The peak age at presentation is 30-60
My Career and Exams years and it has an equal sex distribution. Mortality is
low (0.7%). The incubation period is 1-2 weeks, but the Reference: Normal Values
Order History
illness may follow a protracted course over a number of
Learning Goals months. Symptoms are as described, and liver function Haematology
Question Filters tests may show mild abnormalities in up to 50%. The
chest X-ray may show segmental or diffuse Immunoglobulins
consolidation. Diagnosis is confirmed by enzyme
immunoassay, treatment is with macrolide or Biochemistry
Security
tetracycline antibiotics. Diabetes
Change Password
Sign Out Endocrinology
1934
Blood gases

CSF

© 2011 PasTest Ltd | About Us | Contact Us | Help

file:///E|/Shakhawan/Respiratory%20S/202a.htm[3/13/2012 4:05:44 PM]


MyPasTest

Main Navigation
Question Browser: MRCP 1
Home
Subscriptions Question Browser Exam Builder Saved Exams
» MRCP 1 Session Progress
• Question Browser
Questions Correct 1
• Timed Test An 81-year-old woman consulted her GP because of
influenza 2 weeks previously. For the past few days she Questions Incorrect 1
• Mock Exam
has had increasing cough, associated with purulent sputum Questions Total 2
• Past Papers and haemoptysis. Her daughter visits and notices that she
Questions Percentage 50 %
• Random Questions has become acutely confused and arranges an ambulance.
On arrival to A&E she is agitated with a respiratory rate of More
• My Performance
35/min. Blood gasses reveal that she is hypoxic, her white
• Media Bank blood cell count is 20 × 10 9 /l, predominantly neutrophils.
• New Multimedia Her creatinine concentration is 250mmol/l. Chest X-ray
reveals patchy areas of consolidation, with necrosis and Reference: Normal Values
Online Extras
empyema formation. Haematology
Library
Community What diagnosis best fits this clinical picture? Immunoglobulins
Help Streptococcus pneumoniae pneumonia Biochemistry
PasTest Store Staphlococcus aureus pneumonia Diabetes
Haemophilus influenzae pneumonia Endocrinology
My Account Mycoplasma pneumonia
Blood gases
Profile Chlamydia psittaci pneumonia
CSF
Newsletters
1935
My Career and Exams
Order History
Learning Goals
Question Filters

Security

Change Password
Sign Out

© 2011 PasTest Ltd | About Us | Contact Us | Help

file:///E|/Shakhawan/Respiratory%20S/203.htm[3/13/2012 4:05:46 PM]


MyPasTest

Main Navigation
Question Browser: MRCP 1
Home
Subscriptions Question Browser Exam Builder Saved Exams
» MRCP 1 Session Progress
• Question Browser
Questions Correct 2
• Timed Test An 81-year-old woman consulted her GP because of
influenza 2 weeks previously. For the past few days she Questions Incorrect 1
• Mock Exam
has had increasing cough, associated with purulent sputum Questions Total 3
• Past Papers and haemoptysis. Her daughter visits and notices that she
Questions Percentage 66 %
• Random Questions has become acutely confused and arranges an ambulance.
On arrival to A&E she is agitated with a respiratory rate of More
• My Performance
35/min. Blood gasses reveal that she is hypoxic, her white
• Media Bank blood cell count is 20 × 10 9 /l, predominantly neutrophils.
• New Multimedia Her creatinine concentration is 250mmol/l. Chest X-ray
reveals patchy areas of consolidation, with necrosis and
Online Extras
empyema formation.
Library
Community What diagnosis best fits this clinical picture?

Help Streptococcus pneumoniae pneumonia


PasTest Store Staphlococcus aureus pneumonia Your answer
Haemophilus influenzae pneumonia
My Account Mycoplasma pneumonia
Profile Chlamydia psittaci pneumonia
Newsletters
My Career and Exams
Order History Reference: Normal Values
In general, staphylococcal pneumonia only occurs after
Learning Goals Haematology
a preceding viral illness. Patchy areas of consolidation
Question Filters occur that eventually break down in abscess formation.
Immunoglobulins
Pneumothorax, pleural effusion and empyemas are a
frequent occurrence in staphylococcal pneumonia. Biochemistry
Security Clinical features of severe pneumonia include a
respiratory rate > 30/min, diastolic blood pressure < 60 Diabetes
Change Password mmHg, confusion, involvement of more than one lobe,
Sign Out low albumin concentration, white cell count > 20 × Endocrinology
10 9 /l or less than 1 × 10 9 /l and renal dysfunction. Blood gases
Fulminant staphylococcal pneumonia, as in this case,
carries a high mortality. Standard therapy would be iv CSF
cefuroxime and clarithromycin, with added flucloxacillin
for suspected Staphylococcus aureus pneumonia.

1935

© 2011 PasTest Ltd | About Us | Contact Us | Help

file:///E|/Shakhawan/Respiratory%20S/203a.htm[3/13/2012 4:05:47 PM]


MyPasTest

Main Navigation
Question Browser: MRCP 1
Home
Subscriptions Question Browser Exam Builder Saved Exams
» MRCP 1 Session Progress
• Question Browser
Questions Correct 2
• Timed Test An 18-year-old student presents to A&E suffering from
severe pharyngitis and earache, there is also lethargy and Questions Incorrect 1
• Mock Exam
muscle ache. He also complains of wheezing, but there is Questions Total 3
• Past Papers no previous history of asthma. On examination he has a
Questions Percentage 66 %
• Random Questions non-exudative pharyngitis and bullous myringitis, wheeze
on auscultation but no evidence of consolidation. There is More
• My Performance
erythema nodosum. The white blood cell count is 14 ×
• Media Bank 10 9 /l , but there is no specific indication of bacterial
• New Multimedia infection. Cold agglutinins are detected. A chest X-ray
reveals dramatic upper-lobe consolidation quite out of Reference: Normal Values
Online Extras
character with the clinical examination. What diagnosis best Haematology
Library fits this clinical picture?
Community Immunoglobulins
Streptococcus pneumoniae pneumonia
Help Biochemistry
Haemophilus influenzae pneumonia
PasTest Store Diabetes
Tuberculosis
Mycoplasma pneumonia Endocrinology
My Account
Chlamydia pneumonia Blood gases
Profile
1936 CSF
Newsletters
My Career and Exams
Order History
Learning Goals
Question Filters

Security

Change Password
Sign Out

© 2011 PasTest Ltd | About Us | Contact Us | Help

file:///E|/Shakhawan/Respiratory%20S/204.htm[3/13/2012 4:05:49 PM]


MyPasTest

Main Navigation
Question Browser: MRCP 1
Home
Subscriptions Question Browser Exam Builder Saved Exams
» MRCP 1 Session Progress
• Question Browser
Questions Correct 3
• Timed Test An 18-year-old student presents to A&E suffering from
severe pharyngitis and earache, there is also lethargy and Questions Incorrect 1
• Mock Exam
muscle ache. He also complains of wheezing, but there is Questions Total 4
• Past Papers no previous history of asthma. On examination he has a
Questions Percentage 75 %
• Random Questions non-exudative pharyngitis and bullous myringitis, wheeze
on auscultation but no evidence of consolidation. There is More
• My Performance
erythema nodosum. The white blood cell count is 14 ×
• Media Bank 10 9 /l , but there is no specific indication of bacterial
• New Multimedia infection. Cold agglutinins are detected. A chest X-ray
reveals dramatic upper-lobe consolidation quite out of
Online Extras
character with the clinical examination. What diagnosis best
Library fits this clinical picture?
Community
Streptococcus pneumoniae pneumonia
Help
Haemophilus influenzae pneumonia
PasTest Store
Tuberculosis
Mycoplasma pneumonia Your answer
My Account
Chlamydia pneumonia
Profile
Newsletters
My Career and Exams
Order History Mycoplasma occurs in epidemics occurring every 4 to 5 Reference: Normal Values
years, more commonly amongst close-knit populations
Learning Goals Haematology
like those in schools and colleges. Peak age of
Question Filters presentation is 5-20 years, and mycoplasma is said to
Immunoglobulins
account for 7% of community-acquired pneumonia.
Cold agglutinins are associated with up to 50% of Biochemistry
Security cases, muscle tenderness is said to occur in up to 50%
of cases and, often, X-ray appearances are much worse Diabetes
Change Password than is suggested by the clinical examination of the
Sign Out respiratory system. Treatment is a 2-week course of Endocrinology
erythromycin or clarithromycin. X-ray resolution is Blood gases
complete by week 8 in around 90% of patients.
CSF

1936

© 2011 PasTest Ltd | About Us | Contact Us | Help

file:///E|/Shakhawan/Respiratory%20S/204a.htm[3/13/2012 4:05:50 PM]


MyPasTest

Main Navigation
Question Browser: MRCP 1
Home
Subscriptions Question Browser Exam Builder Saved Exams
» MRCP 1 Session Progress
• Question Browser
Questions Correct 3
• Timed Test A 54-year-old overweight man, landed at Heathrow airport
after a long-haul flight from Australia and collapsed while Questions Incorrect 1
• Mock Exam
waiting for his luggage. After fluid resuscitation his systolic Questions Total 4
• Past Papers blood pressure is only 80 mmHg on arrival at A&E, his
Questions Percentage 75 %
• Random Questions p(O2 ) is 6.3 kPa on a re-breather mask and his p(CO 2 ) is
• My Performance 3.1 kPa. His chest X-ray reveals possible slight elevation of More
the right diaphragm. ECG reveals tachycardia. Apparently,
• Media Bank he had had a cough and was laid up for a few days before
• New Multimedia travelling. Which diagnosis best fits this clinical picture?
Reference: Normal Values
Online Extras
Myocardial infarction Haematology
Library
Acute pneumonia
Community Immunoglobulins
Acute exacerbation of asthma
Help Biochemistry
Acute exacerbation of COPD
PasTest Store Diabetes
Pulmonary embolism
Endocrinology
My Account 1938
Blood gases
Profile
CSF
Newsletters
My Career and Exams
Order History
Learning Goals
Question Filters

Security

Change Password
Sign Out

© 2011 PasTest Ltd | About Us | Contact Us | Help

file:///E|/Shakhawan/Respiratory%20S/205.htm[3/13/2012 4:05:51 PM]


MyPasTest

Main Navigation
Question Browser: MRCP 1
Home
Subscriptions Question Browser Exam Builder Saved Exams
» MRCP 1 Session Progress
• Question Browser
Questions Correct 4
• Timed Test A 54-year-old overweight man, landed at Heathrow airport
after a long-haul flight from Australia and collapsed while Questions Incorrect 1
• Mock Exam
waiting for his luggage. After fluid resuscitation his systolic Questions Total 5
• Past Papers blood pressure is only 80 mmHg on arrival at A&E, his
Questions Percentage 80 %
• Random Questions p(O2 ) is 6.3 kPa on a re-breather mask and his p(CO 2 ) is
• My Performance 3.1 kPa. His chest X-ray reveals possible slight elevation of More
the right diaphragm. ECG reveals tachycardia. Apparently,
• Media Bank he had had a cough and was laid up for a few days before
• New Multimedia travelling. Which diagnosis best fits this clinical picture?
Online Extras
Myocardial infarction
Library
Acute pneumonia
Community
Acute exacerbation of asthma
Help
Acute exacerbation of COPD
PasTest Store
Pulmonary embolism Your answer

My Account

Profile
Newsletters This man has circulatory decompensation and profound
hypoxia, suggestive of a massive pulmonary embolus.
My Career and Exams Although his history of respiratory tract infection a few
Order History days before travelling might lead you to consider Reference: Normal Values
pneumonia, pulmonary embolus has to be excluded
Learning Goals Haematology
here. Although not mentioned in the scenario, the ECG
Question Filters might show the S1Q3T3 pattern in this case. He has
Immunoglobulins
almost certainly acquired his pulmonary embolus
secondary to a deep vein thrombosis as a result of his Biochemistry
Security inactivity during the flight. Acute management includes
heparin by continuous infusion, with consideration of Diabetes
Change Password thrombolysis using streptokinase if he fails to stabilise
Sign Out with fluid replacement. For a first pulmonary embolus, Endocrinology
where there is a clear precipitating factor as in this Blood gases
case, warfarin treatment would usually be continued for
6 months. CSF

1938

© 2011 PasTest Ltd | About Us | Contact Us | Help

file:///E|/Shakhawan/Respiratory%20S/205a.htm[3/13/2012 4:05:53 PM]


MyPasTest

Main Navigation
Question Browser: MRCP 1
Home
Subscriptions Question Browser Exam Builder Saved Exams
» MRCP 1 Session Progress
• Question Browser
Questions Correct 0
• Timed Test A 36-year-old primary schoolteacher from the East End of
London presents with increasing shortness of breath Questions Incorrect 0
• Mock Exam
accompanied by sudden-onset, right-sided pleuritic chest Questions Total 0
• Past Papers pain. She gives a history of influenza for a few days before
Questions Percentage 0%
• Random Questions this acute presentation and also says she suffered a
pulmonary embolus 2 years ago while taking the More
• My Performance
contraceptive pill and describes her pain as identical to that
• Media Bank occasion. On further questioning it transpires that her
• New Multimedia mother had suffered from recurrent deep vein thrombosis.
Arterial blood gases reveal a p(O 2 ) of 7.2 kPa on a re- Reference: Normal Values
Online Extras
breather mask, with a p(CO 2 ) of 3.2 kPa. Her chest X-ray
Haematology
Library reveals a wedge-shaped area of consolidation affecting her
Community right middle and lower lobes. The white blood cell count is Immunoglobulins
normal. Which diagnosis fits best with this clinical picture?
Help Biochemistry
PasTest Store Recurrent pulmonary embolism Diabetes
Staphlyoccal pneumonia
Endocrinology
My Account Pneumothorax
Blood gases
Profile Tuberculosis
Bronchial carcinoma CSF
Newsletters
My Career and Exams 1939
Order History
Learning Goals
Question Filters

Security

Change Password
Sign Out

© 2011 PasTest Ltd | About Us | Contact Us | Help

file:///E|/Shakhawan/Respiratory%20S/206.htm[3/13/2012 4:05:54 PM]


MyPasTest

Main Navigation
Question Browser: MRCP 1
Home
Subscriptions Question Browser Exam Builder Saved Exams
» MRCP 1 Session Progress
• Question Browser
Questions Correct 1
• Timed Test A 36-year-old primary schoolteacher from the East End of
London presents with increasing shortness of breath Questions Incorrect 0
• Mock Exam
accompanied by sudden-onset, right-sided pleuritic chest Questions Total 1
• Past Papers pain. She gives a history of influenza for a few days before
Questions Percentage 100 %
• Random Questions this acute presentation and also says she suffered a
pulmonary embolus 2 years ago while taking the More
• My Performance
contraceptive pill and describes her pain as identical to that
• Media Bank occasion. On further questioning it transpires that her
• New Multimedia mother had suffered from recurrent deep vein thrombosis.
Arterial blood gases reveal a p(O 2 ) of 7.2 kPa on a re-
Online Extras
breather mask, with a p(CO 2 ) of 3.2 kPa. Her chest X-ray
Library reveals a wedge-shaped area of consolidation affecting her
Community right middle and lower lobes. The white blood cell count is
normal. Which diagnosis fits best with this clinical picture?
Help
PasTest Store Recurrent pulmonary embolism Your answer
Staphlyoccal pneumonia
My Account Pneumothorax

Profile Tuberculosis

Newsletters Bronchial carcinoma

My Career and Exams


Order History Reference: Normal Values
Learning Goals Haematology
The suspicion, given that this lady has had a previous
Question Filters pulmonary embolism while taking the oral contraceptive Immunoglobulins
pill, is that she has an inherited disorder of clotting.
Although option B might be a plausible alternative, with Biochemistry
Security the history of previous pulmonary embolism, a second
clot needs excluding. It is crucial during the initial work Diabetes
Change Password
up to take extra samples for clotting studies before
Sign Out Endocrinology
commencing heparin treatment. Conditions that may
predispose to pulmonary embolism include antithrombin Blood gases
III deficiency, protein C deficiency and the factor V
Leiden mutation. As she has suffered a further CSF
pulmonary embolism while off the contraceptive pill,
lifelong warfarin treatment should be considered. Her
job and its location raises the possibility of TB, but this
would only be an outside chance.

1939

© 2011 PasTest Ltd | About Us | Contact Us | Help

file:///E|/Shakhawan/Respiratory%20S/206a.htm[3/13/2012 4:05:56 PM]


MyPasTest

Main Navigation
Question Browser: MRCP 1
Home
Subscriptions Question Browser Exam Builder Saved Exams
» MRCP 1 Session Progress
• Question Browser
Questions Correct 1
• Timed Test A 24-year-old immigrant from Eastern Europe presents for
review. He has suffered several months of fevers, night Questions Incorrect 0
• Mock Exam
sweats and weight loss and has a chronic cough, Questions Total 1
• Past Papers occasionally associated with haemoptysis. Chest X-ray
Questions Percentage 100 %
• Random Questions reveals a calcified lesion within the periphery of his left
lung, with enlarged calcified left hilar lymph nodes. There is More
• My Performance
a normochromic normocytic anaemia, and his white blood
• Media Bank cell count is just below the normal range. Acid-fast bacilli
• New Multimedia (AFB) are found in one out of six sputum samples. Sputum
is sent for extended culture. Which diagnosis fits best with Reference: Normal Values
Online Extras this clinical picture?
Haematology
Library
Bronchial carcinoma Immunoglobulins
Community
Pneumonia
Help Biochemistry
Old tuberculosis
PasTest Store Diabetes
Idiopathic pulmonary nodule
Active pulmonary tuberculosis Endocrinology
My Account
Blood gases
Profile 1940
CSF
Newsletters
My Career and Exams
Order History
Learning Goals
Question Filters

Security

Change Password
Sign Out

© 2011 PasTest Ltd | About Us | Contact Us | Help

file:///E|/Shakhawan/Respiratory%20S/207.htm[3/13/2012 4:05:58 PM]


MyPasTest

Main Navigation
Question Browser: MRCP 1
Home
Subscriptions Question Browser Exam Builder Saved Exams
» MRCP 1 Session Progress
• Question Browser
Questions Correct 2
• Timed Test A 24-year-old immigrant from Eastern Europe presents for
review. He has suffered several months of fevers, night Questions Incorrect 0
• Mock Exam
sweats and weight loss and has a chronic cough, Questions Total 2
• Past Papers occasionally associated with haemoptysis. Chest X-ray
Questions Percentage 100 %
• Random Questions reveals a calcified lesion within the periphery of his left
lung, with enlarged calcified left hilar lymph nodes. There is More
• My Performance
a normochromic normocytic anaemia, and his white blood
• Media Bank cell count is just below the normal range. Acid-fast bacilli
• New Multimedia (AFB) are found in one out of six sputum samples. Sputum
is sent for extended culture. Which diagnosis fits best with
Online Extras this clinical picture?
Library
Bronchial carcinoma
Community
Pneumonia
Help
Old tuberculosis
PasTest Store
Idiopathic pulmonary nodule
Active pulmonary tuberculosis Your answer
My Account

Profile
Newsletters
My Career and Exams This man has symptoms of TB, an abnormal chest X-ray
Order History consistent with primary infection and AFB on sputum Reference: Normal Values
examination. Treatment is with multi antibiotic therapy
Learning Goals using pyrazinamide, rifampicin, ethambutol and Haematology
Question Filters isoniazid. Drug-resistant TB is becoming more common,
and is associated with prior therapy, acquisition of TB in Immunoglobulins
the developing world, homelessness, HIV infection, iv
Biochemistry
Security drug abuse and infection acquired in institutions, eg
prison. Liver function-test (LFT) monitoring while on Diabetes
Change Password therapy to exclude drug-induced hepatitis is essential.
Sign Out Endocrinology

Blood gases
1940
CSF

© 2011 PasTest Ltd | About Us | Contact Us | Help

file:///E|/Shakhawan/Respiratory%20S/207a.htm[3/13/2012 4:05:59 PM]


MyPasTest

Main Navigation
Question Browser: MRCP 1
Home
Subscriptions Question Browser Exam Builder Saved Exams
» MRCP 1 Session Progress
• Question Browser
Questions Correct 2
• Timed Test A 33-year-old HIV-positive man presents for review. He is
poorly compliant with antiretroviral therapy and his recent Questions Incorrect 0
• Mock Exam
CD4 count is only 90/ml blood. He complains of a gradual- Questions Total 2
• Past Papers onset headache, fever, malaise, night sweats and a cough
Questions Percentage 100 %
• Random Questions associated with haemoptysis. He is emaciated. On
examination there is widespread lymphadenopathy, there More
• My Performance
are crackles and wheeze on auscultation of his chest, and
• Media Bank tenderness over the liver edge. Blood testing reveals a
• New Multimedia normochromic normocytic anaemia, he has a low white
count, urea and creatinine levels are raised and liver Reference: Normal Values
Online Extras function tests are abnormal. Sputum samples reveal acid-
and alcohol-fast bacilli (AFB). Chest X-ray reveals calcified Haematology
Library
lymph nodes, cavitation and areas of lung fibrosis and hilar Immunoglobulins
Community
retraction. Which diagnosis fits best with this clinical
Help picture? Biochemistry
PasTest Store Diabetes
Primary pulmonary tuberculosis
Miliary tuberculosis Endocrinology
My Account
Bacterial pneumonia Blood gases
Profile Pulmonary fibrosis
CSF
Newsletters Bronchial carcinoma
My Career and Exams
1941
Order History
Learning Goals
Question Filters

Security

Change Password
Sign Out

© 2011 PasTest Ltd | About Us | Contact Us | Help

file:///E|/Shakhawan/Respiratory%20S/208.htm[3/13/2012 4:06:01 PM]


MyPasTest

Main Navigation
Question Browser: MRCP 1
Home
Subscriptions Question Browser Exam Builder Saved Exams
» MRCP 1 Session Progress
• Question Browser
Questions Correct 2
• Timed Test A 33-year-old HIV-positive man presents for review. He is
poorly compliant with antiretroviral therapy and his recent Questions Incorrect 1
• Mock Exam
CD4 count is only 90/ml blood. He complains of a gradual- Questions Total 3
• Past Papers onset headache, fever, malaise, night sweats and a cough
Questions Percentage 66 %
• Random Questions associated with haemoptysis. He is emaciated. On
examination there is widespread lymphadenopathy, there More
• My Performance
are crackles and wheeze on auscultation of his chest, and
• Media Bank tenderness over the liver edge. Blood testing reveals a
• New Multimedia normochromic normocytic anaemia, he has a low white
count, urea and creatinine levels are raised and liver
Online Extras function tests are abnormal. Sputum samples reveal acid-
Library and alcohol-fast bacilli (AFB). Chest X-ray reveals calcified
lymph nodes, cavitation and areas of lung fibrosis and hilar
Community
retraction. Which diagnosis fits best with this clinical
Help picture?
PasTest Store
Primary pulmonary tuberculosis Your answer
Miliary tuberculosis Correct answer
My Account
Bacterial pneumonia
Profile Pulmonary fibrosis
Newsletters Bronchial carcinoma
My Career and Exams
Order History Reference: Normal Values
Learning Goals Haematology
Question Filters There is evidence of severe tuberculosis with
extrapulmonary spread. TB meningitis presents with a Immunoglobulins
gradual-onset headache and malaise, if left
Biochemistry
Security unrecognised it can progress to sudden-onset stupor or
coma and there may be associated VIth nerve palsy. Diabetes
Change Password The abnormal liver function tests suggest TB infection of
Sign Out the liver; other affected sites may be the skeletal and Endocrinology
genitourinary systems. Treatment involves at least 6
months’ triple antibiotic therapy with rifampicin, Blood gases
isoniazid and ethambutol. CSF

1941

© 2011 PasTest Ltd | About Us | Contact Us | Help

file:///E|/Shakhawan/Respiratory%20S/208a.htm[3/13/2012 4:06:02 PM]


MyPasTest

Main Navigation
Question Browser: MRCP 1
Home
Subscriptions Question Browser Exam Builder Saved Exams
» MRCP 1 Session Progress
• Question Browser
Questions Correct 2
• Timed Test A 30-year-old woman presents with shortness of breath.
This began gradually, around 2.5 years ago, but now she is Questions Incorrect 1
• Mock Exam
breathless on climbing a flight of stairs. There is no past Questions Total 3
• Past Papers history of note. On examination the JVP is raised, carotid
Questions Percentage 66 %
• Random Questions pulse volume is reduced and there is evidence of right
ventricular hypertrophy. There are right-sided murmurs on More
• My Performance
cardiac auscultation. Her chest X-ray shows pulmonary
• Media Bank artery enlargement, ECG shows right axis deviation and
• New Multimedia right ventricular hypertrophy. Arterial blood gases reveal
hypoxia and hypercapnia, a lung perfusion scan is normal. Reference: Normal Values
Online Extras Cardiac catheterisation reveals that right-sided pressures
are markedly raised. Haematology
Library
Community Immunoglobulins
Which diagnosis best fits with this clinical picture?
Help Biochemistry
Chronic thromboembolic disease
PasTest Store Diabetes
Right ventricular failure
Primary pulmonary hypertension (PPH) Endocrinology
My Account
Cryptogenic fibrosing alveolitis Blood gases
Profile
Asthma CSF
Newsletters
My Career and Exams 1942
Order History
Learning Goals
Question Filters

Security

Change Password
Sign Out

© 2011 PasTest Ltd | About Us | Contact Us | Help

file:///E|/Shakhawan/Respiratory%20S/209.htm[3/13/2012 4:06:03 PM]


MyPasTest

Main Navigation
Question Browser: MRCP 1
Home
Subscriptions Question Browser Exam Builder Saved Exams
» MRCP 1 Session Progress
• Question Browser
Questions Correct 3
• Timed Test A 30-year-old woman presents with shortness of breath.
This began gradually, around 2.5 years ago, but now she is Questions Incorrect 1
• Mock Exam
breathless on climbing a flight of stairs. There is no past Questions Total 4
• Past Papers history of note. On examination the JVP is raised, carotid
Questions Percentage 75 %
• Random Questions pulse volume is reduced and there is evidence of right
ventricular hypertrophy. There are right-sided murmurs on More
• My Performance
cardiac auscultation. Her chest X-ray shows pulmonary
• Media Bank artery enlargement, ECG shows right axis deviation and
• New Multimedia right ventricular hypertrophy. Arterial blood gases reveal
hypoxia and hypercapnia, a lung perfusion scan is normal.
Online Extras Cardiac catheterisation reveals that right-sided pressures
Library are markedly raised.
Community Which diagnosis best fits with this clinical picture?
Help
Chronic thromboembolic disease
PasTest Store
Right ventricular failure
Primary pulmonary hypertension (PPH) Your answer
My Account
Cryptogenic fibrosing alveolitis
Profile
Asthma
Newsletters
My Career and Exams
Order History Reference: Normal Values
Learning Goals The perfusion scan is not suggestive of chronic Haematology
Question Filters thromboembolic disease, which makes primary
pulmonary hypertension the most likely diagnosis. Immunoglobulins
Plexogenic pulmonary arteriopathy is found in 30–60%
of sufferers and characterised by medial hypertrophy Biochemistry
Security
and concentric laminar intimal fibrosis. The gene for Diabetes
Change Password PPH has now been mapped to chromosome 2, and
mutations in the bone morphogenic protein receptor Endocrinology
Sign Out
have been identified in some patients. Intravenous
epoprostenol (prostacyclin) and oxygen therapy has Blood gases
been shown to improve quality of life in sufferers.
CSF
Recently, phosphodiesterase (PDE)-5 inhibitors have
also been shown to have some therapeutic advantage.
Unfortunately, several studies report a mean survival of
only 2.5 years from diagnosis, with right ventricular
failure and sudden death the main causes of mortality.

1942

© 2011 PasTest Ltd | About Us | Contact Us | Help

file:///E|/Shakhawan/Respiratory%20S/209a.htm[3/13/2012 4:06:05 PM]


MyPasTest

Main Navigation
Question Browser: MRCP 1
Home
Subscriptions Question Browser Exam Builder Saved Exams
» MRCP 1 Session Progress
• Question Browser
Questions Correct 3
• Timed Test A 58-year-old memorial stonemason presents to the chest
clinic. Over the past few years he has noted a gradual Questions Incorrect 1
• Mock Exam
increase in shortness of breath, with cough and occasional Questions Total 4
• Past Papers wheeze. He is a non-smoker and has no other past history
Questions Percentage 75 %
• Random Questions of note. His chest X-ray is abnormal with small rounded
opacities and irregular upper zone fibrosis. There is hilar More
• My Performance
lymphadenopathy with ‘eggshell’ calcification. Pulmonary
• Media Bank function testing reveals a restrictive picture and there is
• New Multimedia mild hypoxia.
Reference: Normal Values
Online Extras Which diagnosis best fits with this clinical picture?
Haematology
Library
Silicosis
Community Immunoglobulins
Asthma
Help Biochemistry
Idiopathic pulmonary fibrosis
PasTest Store Diabetes
Tuberculosis
Byssinosis Endocrinology
My Account
Blood gases
Profile 1943
CSF
Newsletters
My Career and Exams
Order History
Learning Goals
Question Filters

Security

Change Password
Sign Out

© 2011 PasTest Ltd | About Us | Contact Us | Help

file:///E|/Shakhawan/Respiratory%20S/210.htm[3/13/2012 4:06:06 PM]


MyPasTest

Main Navigation
Question Browser: MRCP 1
Home
Subscriptions Question Browser Exam Builder Saved Exams
» MRCP 1 Session Progress
• Question Browser
Questions Correct 4
• Timed Test A 58-year-old memorial stonemason presents to the chest
clinic. Over the past few years he has noted a gradual Questions Incorrect 1
• Mock Exam
increase in shortness of breath, with cough and occasional Questions Total 5
• Past Papers wheeze. He is a non-smoker and has no other past history
Questions Percentage 80 %
• Random Questions of note. His chest X-ray is abnormal with small rounded
opacities and irregular upper zone fibrosis. There is hilar More
• My Performance
lymphadenopathy with ‘eggshell’ calcification. Pulmonary
• Media Bank function testing reveals a restrictive picture and there is
• New Multimedia mild hypoxia.

Online Extras Which diagnosis best fits with this clinical picture?
Library
Silicosis Your answer
Community
Asthma
Help
Idiopathic pulmonary fibrosis
PasTest Store
Tuberculosis
Byssinosis
My Account

Profile
Newsletters
My Career and Exams This man’s job is likely to have exposed him to silica
dust, and unfortunately he is suffering from silicosis. Reference: Normal Values
Order History
Silicosis is characterised by shortness of breath, wheeze
Learning Goals and cough with lung function tests and chest X-ray Haematology
Question Filters suggestive of irregular fibrosis. Pathogenesis is related
to activation of alveolar macrophages via ingestion of Immunoglobulins
silica dust, releasing a number of proteolytic enzymes
and inflammatory cytokines. Chronic silicosis may not Biochemistry
Security
progress, but accelerated forms and continued exposure Diabetes
Change Password are associated with respiratory failure over a few years.
Management involves removal of exposure, screening Endocrinology
Sign Out
for and treatment of tuberculosis if found in association
and supportive measures such as bronchodilators and Blood gases
oxygen therapy.
CSF

1943

© 2011 PasTest Ltd | About Us | Contact Us | Help

file:///E|/Shakhawan/Respiratory%20S/210a.htm[3/13/2012 4:06:08 PM]


MyPasTest

Main Navigation
Question Browser: MRCP 1
Home
Subscriptions Question Browser Exam Builder Saved Exams
» MRCP 1 Session Progress
• Question Browser
Questions Correct 4
• Timed Test A morbidly obese 36 year-old man presents for review. His
main reason for attendance is that his wife is concerned Questions Incorrect 1
• Mock Exam
about his loud snoring and the fact that he stops breathing Questions Total 5
• Past Papers during the night for periods of up to 8–10 seconds,
Questions Percentage 80 %
• Random Questions followed by coughing, snoring and arousal. Recently he has
become hypertensive and is also on treatment for More
• My Performance
impotence. His 24-h urinary free cortisol level is normal.
• Media Bank Which diagnosis best fits this picture?
• New Multimedia
Cushing’s disease Reference: Normal Values
Online Extras
Obstructive sleep apnoea Haematology
Library
Simple snoring
Community Immunoglobulins
Simple obesity
Help Biochemistry
Essential hypertension
PasTest Store Diabetes
1944
Endocrinology
My Account
Blood gases
Profile
CSF
Newsletters
My Career and Exams
Order History
Learning Goals
Question Filters

Security

Change Password
Sign Out

© 2011 PasTest Ltd | About Us | Contact Us | Help

file:///E|/Shakhawan/Respiratory%20S/211.htm[3/13/2012 4:06:09 PM]


MyPasTest

Main Navigation
Question Browser: MRCP 1
Home
Subscriptions Question Browser Exam Builder Saved Exams
» MRCP 1 Session Progress
• Question Browser
Questions Correct 5
• Timed Test A morbidly obese 36 year-old man presents for review. His
main reason for attendance is that his wife is concerned Questions Incorrect 1
• Mock Exam
about his loud snoring and the fact that he stops breathing Questions Total 6
• Past Papers during the night for periods of up to 8–10 seconds,
Questions Percentage 83 %
• Random Questions followed by coughing, snoring and arousal. Recently he has
become hypertensive and is also on treatment for More
• My Performance
impotence. His 24-h urinary free cortisol level is normal.
• Media Bank Which diagnosis best fits this picture?
• New Multimedia
Cushing’s disease
Online Extras
Obstructive sleep apnoea Your answer
Library
Simple snoring
Community
Simple obesity
Help
Essential hypertension
PasTest Store

My Account

Profile Symptoms of sleep apnoea include memory impairment,


daytime somnolence, grossly disrupted sleep pattern,
Newsletters
decreased libido and systemic hypertension. Further
My Career and Exams investigation should involve thyroid function testing to
Order History rule out hypothyroidism, and assessment of the uvula Reference: Normal Values
and tonsils to rule out mechanical obstruction that
Learning Goals might be amenable to ENT surgery. Diagnosis of sleep Haematology
Question Filters apnoea is made via overnight oximetry. The mainstay of
management is weight loss, with the addition of Immunoglobulins
overnight CPAP (continuous positive airway pressure).
Biochemistry
Security
Diabetes
Change Password 1944
Sign Out Endocrinology

Blood gases

CSF

© 2011 PasTest Ltd | About Us | Contact Us | Help

file:///E|/Shakhawan/Respiratory%20S/211a.htm[3/13/2012 4:06:11 PM]


MyPasTest

Main Navigation
Question Browser: MRCP 1
Home
Subscriptions Question Browser Exam Builder Saved Exams
» MRCP 1 Session Progress
• Question Browser
Questions Correct 5
• Timed Test A 42-year-old cotton worker from Southern India is staying
with her brother in London . While here, they pay for a Questions Incorrect 1
• Mock Exam
private medical consultation. She reports what sounds like Questions Total 6
• Past Papers a work-related illness. During the first hour at work after
Questions Percentage 83 %
• Random Questions the weekend she reports severe shortness of breath, cough
and chest tightness, this appears to gradually ease during More
• My Performance
the week, only to return after the next weekend off.
• Media Bank
Which diagnosis best fits this clinical picture?
• New Multimedia
Reference: Normal Values
Online Extras Occupational asthma
Haematology
Library Chronic obstructive pulmonary disease
Community Byssinosis Immunoglobulins
Help Berylliosis Biochemistry
PasTest Store Functional shortness of breath Diabetes

1945 Endocrinology
My Account
Blood gases
Profile
CSF
Newsletters
My Career and Exams
Order History
Learning Goals
Question Filters

Security

Change Password
Sign Out

© 2011 PasTest Ltd | About Us | Contact Us | Help

file:///E|/Shakhawan/Respiratory%20S/212.htm[3/13/2012 4:06:12 PM]


MyPasTest

Main Navigation
Question Browser: MRCP 1
Home
Subscriptions Question Browser Exam Builder Saved Exams
» MRCP 1 Session Progress
• Question Browser
Questions Correct 5
• Timed Test A 42-year-old cotton worker from Southern India is staying
with her brother in London . While here, they pay for a Questions Incorrect 2
• Mock Exam
private medical consultation. She reports what sounds like Questions Total 7
• Past Papers a work-related illness. During the first hour at work after
Questions Percentage 71 %
• Random Questions the weekend she reports severe shortness of breath, cough
and chest tightness, this appears to gradually ease during More
• My Performance
the week, only to return after the next weekend off.
• Media Bank
Which diagnosis best fits this clinical picture?
• New Multimedia
Online Extras Occupational asthma Your answer
Library Chronic obstructive pulmonary
Community disease
Help Byssinosis Correct answer
PasTest Store Berylliosis
Functional shortness of breath

My Account

Profile
Newsletters Byssinosis is now rare in the UK as the number of
My Career and Exams cotton mills has declined, but it still occurs in areas of
cotton-cloth production such as in India. It is Reference: Normal Values
Order History
characteristically described as shortness of breath,
Learning Goals cough and chest tightness that begins on the first day Haematology
Question Filters of the working week and eases during the remainder of
the working week. It is thought to be due to endotoxin Immunoglobulins
released by bacteria existing in cotton dust within the
mill. There are usually no demonstrable chest X-ray Biochemistry
Security
changes. It is thought that more severe changes in Diabetes
Change Password some patients may be related to associated asthma or
cigarette smoking. Endocrinology
Sign Out
Blood gases
1945 CSF

© 2011 PasTest Ltd | About Us | Contact Us | Help

file:///E|/Shakhawan/Respiratory%20S/212a.htm[3/13/2012 4:06:14 PM]


MyPasTest

Main Navigation
Question Browser: MRCP 1
Home
Subscriptions Question Browser Exam Builder Saved Exams
» MRCP 1 Session Progress
• Question Browser
Questions Correct 5
• Timed Test A 72-year-old former coal-miner visits you for review. He
reports having frequently worked at the coal face in Questions Incorrect 2
• Mock Exam
cramped conditions, with exposure to a large volume of Questions Total 7
• Past Papers coal dust. He says he has had increasing symptoms of
Questions Percentage 71 %
• Random Questions cough and shortness of breath over the past few years, but
continues to smoke 10–15 cigarettes per day. His chest X- More
• My Performance
ray reveals a large number of small round opacities within
• Media Bank the lung fields, with almost complete obscurement of
• New Multimedia normal lung markings. Which diagnosis fits best with this
clinical picture? Reference: Normal Values
Online Extras
Chronic obstructive pulmonary disease Haematology
Library
Community Asthma Immunoglobulins
Help Category 1 pneumoconiosis Biochemistry
PasTest Store Category 2 pneumoconiosis
Diabetes
Category 3 pneumoconiosis
Endocrinology
My Account
1946 Blood gases
Profile
CSF
Newsletters
My Career and Exams
Order History
Learning Goals
Question Filters

Security

Change Password
Sign Out

© 2011 PasTest Ltd | About Us | Contact Us | Help

file:///E|/Shakhawan/Respiratory%20S/213.htm[3/13/2012 4:06:15 PM]


MyPasTest

Main Navigation
Question Browser: MRCP 1
Home
Subscriptions Question Browser Exam Builder Saved Exams
» MRCP 1 Session Progress
• Question Browser
Questions Correct 5
• Timed Test A 72-year-old former coal-miner visits you for review. He
reports having frequently worked at the coal face in Questions Incorrect 3
• Mock Exam
cramped conditions, with exposure to a large volume of Questions Total 8
• Past Papers coal dust. He says he has had increasing symptoms of
Questions Percentage 62 %
• Random Questions cough and shortness of breath over the past few years, but
continues to smoke 10–15 cigarettes per day. His chest X- More
• My Performance
ray reveals a large number of small round opacities within
• Media Bank the lung fields, with almost complete obscurement of
• New Multimedia normal lung markings. Which diagnosis fits best with this
clinical picture?
Online Extras
Library Chronic obstructive pulmonary
disease
Community
Asthma
Help
Category 1 pneumoconiosis
PasTest Store
Category 2 pneumoconiosis Your answer
Category 3 pneumoconiosis Correct answer
My Account

Profile
Newsletters
My Career and Exams Although he may have some smoking-related COPD, his
Order History chest X-ray is diagnostic in the presence of his Reference: Normal Values
occupational history of category 3 pneumoconiosis.
Learning Goals Category 2 is less severe, with a number of opacities Haematology
Question Filters but normal lung markings still visible; while category 1
is the least severe, with fewer opacities and normal lung Immunoglobulins
markings clearly visible. The relevance is the
Biochemistry
Security percentage progression to progressive massive fibrosis
(category 1 virtually never, category 2 around 7%, Diabetes
Change Password category 3 around 30%). There is a government
Sign Out compensation scheme for pneumoconiosis, payments Endocrinology
are the norm for category 2 or 3, but category 1
payments are more contentious and may depend on the Blood gases
presence of other conditions contributing to morbidity CSF
such as COPD.

1946

© 2011 PasTest Ltd | About Us | Contact Us | Help

file:///E|/Shakhawan/Respiratory%20S/213a.htm[3/13/2012 4:06:16 PM]


MyPasTest

Main Navigation
Question Browser: MRCP 1
Home
Subscriptions Question Browser Exam Builder Saved Exams
» MRCP 1 Session Progress
• Question Browser
Questions Correct 5
• Timed Test A 69-year-old former coal-miner is referred to you by the
on-call team. There is a smoking history and he has been Questions Incorrect 3
• Mock Exam
managed by his GP for COPD. He has been admitted with Questions Total 8
• Past Papers dyspnoea that is now so bad that he is unable to manage
Questions Percentage 62 %
• Random Questions at home and cannot walk from the chair to the bathroom.
There is a cough productive of black sputum. Lung function More
• My Performance
tests show a mixed restrictive and obstructive picture. A
• Media Bank chest X-ray shows marked changes with massive fibrotic
• New Multimedia masses predominantly in the upper lobes. There are also
changes consistent with lung destruction and emphysema. Reference: Normal Values
Online Extras His rheumatoid factor is positive. Which diagnosis fits best
with this clinical picture? Haematology
Library
Community Immunoglobulins
Progressive Massive Fibrosis (PMF)
Help Biochemistry
Chronic obstructive pulmonary disease
PasTest Store Diabetes
Tuberculosis
Asthma Endocrinology
My Account
Category 1 pneumoconiosis
Blood gases
Profile
1947 CSF
Newsletters
My Career and Exams
Order History
Learning Goals
Question Filters

Security

Change Password
Sign Out

© 2011 PasTest Ltd | About Us | Contact Us | Help

file:///E|/Shakhawan/Respiratory%20S/214.htm[3/13/2012 4:06:18 PM]


MyPasTest

Main Navigation
Question Browser: MRCP 1
Home
Subscriptions Question Browser Exam Builder Saved Exams
» MRCP 1 Session Progress
• Question Browser
Questions Correct 6
• Timed Test A 69-year-old former coal-miner is referred to you by the
on-call team. There is a smoking history and he has been Questions Incorrect 3
• Mock Exam
managed by his GP for COPD. He has been admitted with Questions Total 9
• Past Papers dyspnoea that is now so bad that he is unable to manage
Questions Percentage 66 %
• Random Questions at home and cannot walk from the chair to the bathroom.
There is a cough productive of black sputum. Lung function More
• My Performance
tests show a mixed restrictive and obstructive picture. A
• Media Bank chest X-ray shows marked changes with massive fibrotic
• New Multimedia masses predominantly in the upper lobes. There are also
changes consistent with lung destruction and emphysema.
Online Extras His rheumatoid factor is positive. Which diagnosis fits best
Library with this clinical picture?
Community
Progressive Massive Fibrosis (PMF) Your answer
Help
Chronic obstructive pulmonary disease
PasTest Store
Tuberculosis
Asthma
My Account Category 1 pneumoconiosis
Profile
Newsletters
My Career and Exams
Order History PMF is associated with fibrotic masses in the apices, Reference: Normal Values
sometimes up to 10 cm in diameter. There are also
Learning Goals emphysematous changes. Usually there is a history of Haematology
Question Filters dust inhalation such as coal dust, and rheumatoid factor
and antinuclear antibody are often positive. There is a Immunoglobulins
mixed obstructive and restrictive lung defect with
Biochemistry
Security reduced transfer factor. PMF may rapidly progress, even
in the absence of further dust exposure, leading to Diabetes
Change Password respiratory failure and eventually death. Category 2
Sign Out pneumoconiosis progresses to PMF in around 7% of Endocrinology
cases, the rate of progression of category 3
pneumoconiosis is much higher, at around 30%. Blood gases

CSF

1947

© 2011 PasTest Ltd | About Us | Contact Us | Help

file:///E|/Shakhawan/Respiratory%20S/214a.htm[3/13/2012 4:06:19 PM]


MyPasTest

Main Navigation
Question Browser: MRCP 1
Home
Subscriptions Question Browser Exam Builder Saved Exams
» MRCP 1 Session Progress
• Question Browser
Questions Correct 6
• Timed Test A 67-year-old man who suffers from recurrent ventricular
tachycardia has been taking amiodarone 200 mg daily for 5 Questions Incorrect 3
• Mock Exam
years, as well as phenytoin for epilepsy and NSAIDs for Questions Total 9
• Past Papers arthritis. Over the past few months he has suffered
Questions Percentage 66 %
• Random Questions progressive shortness of breath on exercise. Lung function
testing reveals a restrictive picture and his chest X-ray is More
• My Performance
suggestive of diffuse lung fibrosis.
• Media Bank
• New Multimedia Which diagnosis best fits this clinical picture?
Reference: Normal Values
Online Extras Phenytoin-induced lung fibrosis
Haematology
Library NSAID-induced lung fibrosis
Community Immunoglobulins
Amiodarone-induced lung fibrosis
Help Cardiac failure Biochemistry
PasTest Store Cryptogenic fibrosing alveolitis Diabetes

1948 Endocrinology
My Account
Blood gases
Profile
CSF
Newsletters
My Career and Exams
Order History
Learning Goals
Question Filters

Security

Change Password
Sign Out

© 2011 PasTest Ltd | About Us | Contact Us | Help

file:///E|/Shakhawan/Respiratory%20S/215.htm[3/13/2012 4:06:21 PM]


MyPasTest

Main Navigation
Question Browser: MRCP 1
Home
Subscriptions Question Browser Exam Builder Saved Exams
» MRCP 1 Session Progress
• Question Browser
Questions Correct 7
• Timed Test A 67-year-old man who suffers from recurrent ventricular
tachycardia has been taking amiodarone 200 mg daily for 5 Questions Incorrect 3
• Mock Exam
years, as well as phenytoin for epilepsy and NSAIDs for Questions Total 10
• Past Papers arthritis. Over the past few months he has suffered
Questions Percentage 70 %
• Random Questions progressive shortness of breath on exercise. Lung function
testing reveals a restrictive picture and his chest X-ray is More
• My Performance
suggestive of diffuse lung fibrosis.
• Media Bank
• New Multimedia Which diagnosis best fits this clinical picture?

Online Extras Phenytoin-induced lung fibrosis


Library NSAID-induced lung fibrosis
Community Amiodarone-induced lung fibrosis Your answer
Help Cardiac failure
PasTest Store Cryptogenic fibrosing alveolitis

My Account

Profile
Amiodarone may be related to diffuse lung fibrosis.
Newsletters
NSAIDs may cause asthma or pulmonary eosinophilia,
My Career and Exams phenytoin is also characteristically related to pulmonary
Order History eosinophilia. Other drugs related to lung fibrosis include Reference: Normal Values
nitrofurantoin, hexamethonium and a number of
Learning Goals cytotoxic agents. Corticosteroids may be of value in Haematology
Question Filters some patients, but withdrawal of the amiodarone, if
possible, is advised. In this case, other therapies such Immunoglobulins
as an implantable defibrillator may be considered.
Biochemistry
Security
Diabetes
Change Password 1948
Sign Out Endocrinology

Blood gases

CSF

© 2011 PasTest Ltd | About Us | Contact Us | Help

file:///E|/Shakhawan/Respiratory%20S/215a.htm[3/13/2012 4:06:22 PM]


MyPasTest

Main Navigation
Question Browser: MRCP 1
Home
Subscriptions Question Browser Exam Builder Saved Exams
» MRCP 1 Session Progress
• Question Browser
Questions Correct 0
• Timed Test A 38-year-old woman presents with recurrent chest
infections. For some time she has noticed that her nails are Questions Incorrect 0
• Mock Exam
yellow and misshapen and that she often has oedematous Questions Total 0
• Past Papers legs after standing all day in the shop where she works.
Questions Percentage 0%
• Random Questions Examination reveals evidence of lower lobe consolidation
and possible pleural effusion. This is confirmed on chest X- More
• My Performance
ray. Her nails are very abnormal, thickened and yellow and
• Media Bank she has bilateral lymphoedema affecting her legs. Which
• New Multimedia diagnosis fits best with this clinical picture?
Reference: Normal Values
Online Extras Bronchiectasis
Haematology
Library Asthma
Community Immunoglobulins
Underlying bronchial carcinoma
Help COPD Biochemistry
PasTest Store Yellow-nail Syndrome Diabetes

1949 Endocrinology
My Account
Blood gases
Profile
CSF
Newsletters
My Career and Exams
Order History
Learning Goals
Question Filters

Security

Change Password
Sign Out

© 2011 PasTest Ltd | About Us | Contact Us | Help

file:///E|/Shakhawan/Respiratory%20S/216.htm[3/13/2012 4:06:24 PM]


MyPasTest

Main Navigation
Question Browser: MRCP 1
Home
Subscriptions Question Browser Exam Builder Saved Exams
» MRCP 1 Session Progress
• Question Browser
Questions Correct 1
• Timed Test A 38-year-old woman presents with recurrent chest
infections. For some time she has noticed that her nails are Questions Incorrect 0
• Mock Exam
yellow and misshapen and that she often has oedematous Questions Total 1
• Past Papers legs after standing all day in the shop where she works.
Questions Percentage 100 %
• Random Questions Examination reveals evidence of lower lobe consolidation
and possible pleural effusion. This is confirmed on chest X- More
• My Performance
ray. Her nails are very abnormal, thickened and yellow and
• Media Bank she has bilateral lymphoedema affecting her legs. Which
• New Multimedia diagnosis fits best with this clinical picture?

Online Extras Bronchiectasis


Library Asthma
Community Underlying bronchial carcinoma
Help COPD
PasTest Store Yellow-nail Syndrome Your answer

My Account

Profile
Yellow-nail syndrome is an abnormality of lymphatic
Newsletters
drainage associated with recurrent bronchiectasis, small
My Career and Exams bilateral pleural effusions, lymphoedema and grossly
Order History thickened, yellow nails. Her X-ray changes are chronic Reference: Normal Values
and associated with bronchiectasis. Management is with
Learning Goals antibiotics to treat episodes of acute infection and Haematology
Question Filters regular follow-up.
Immunoglobulins
Biochemistry
Security 1949
Diabetes
Change Password
Sign Out Endocrinology

Blood gases

CSF

© 2011 PasTest Ltd | About Us | Contact Us | Help

file:///E|/Shakhawan/Respiratory%20S/216a.htm[3/13/2012 4:06:25 PM]


MyPasTest

Main Navigation
Question Browser: MRCP 1
Home
Subscriptions Question Browser Exam Builder Saved Exams
» MRCP 1 Session Progress
• Question Browser
Questions Correct 1
• Timed Test A patient with small-cell lung cancer has a serum sodium
concentration of 121 mmol/l. The patient is asymptomatic. Questions Incorrect 0
• Mock Exam
What is the most appropriate therapy? Questions Total 1
• Past Papers
Questions Percentage 100 %
• Random Questions Fluid restriction
• My Performance Glucocorticoids More
• Media Bank Start chemotherapy
• New Multimedia Start radiotherapy
Reference: Normal Values
Online Extras Hypertonic fluid infusion
Haematology
Library
2005
Community Immunoglobulins
Help Biochemistry
PasTest Store Diabetes
Endocrinology
My Account
Blood gases
Profile
CSF
Newsletters
My Career and Exams
Order History
Learning Goals
Question Filters

Security

Change Password
Sign Out

© 2011 PasTest Ltd | About Us | Contact Us | Help

file:///E|/Shakhawan/Respiratory%20S/217.htm[3/13/2012 4:06:27 PM]


MyPasTest

Main Navigation
Question Browser: MRCP 1
Home
Subscriptions Question Browser Exam Builder Saved Exams
» MRCP 1 Session Progress
• Question Browser
Questions Correct 2
• Timed Test A patient with small-cell lung cancer has a serum sodium
concentration of 121 mmol/l. The patient is asymptomatic. Questions Incorrect 0
• Mock Exam
What is the most appropriate therapy? Questions Total 2
• Past Papers
Questions Percentage 100 %
• Random Questions Fluid restriction Your answer
• My Performance Glucocorticoids More
• Media Bank Start chemotherapy
• New Multimedia Start radiotherapy
Online Extras Hypertonic fluid infusion
Library
Community
Help
PasTest Store The continued secretion of vasopressin (ADH) in an
amount in excess of the body’s needs leads to
overhydration in both the intracellular and
My Account extracellular compartments. This is called ‘the
syndrome of inappropriate antidiuretic hormone
Profile secretion’ (SIADH). The cerebral oedema resulting
Newsletters from water intoxication causes drowsiness, lethargy,
irritability, mental confusion and disorientation, with
My Career and Exams seizures and coma being the most profound features.
Order History Peripheral oedema is remarkably rare. The patient is Reference: Normal Values
usually asymptomatic until the sodium level falls
Learning Goals Haematology
below 120 mmol/l and the hyponatraemia is dilutional
Question Filters in type with a low serum osmolality. Urine osmolality
Immunoglobulins
usually exceeds 300 mOsmol/kg.
The commonest cancer causing this syndrome is Biochemistry
Security
small-cell cancer, where it is clinically obvious in 10% Diabetes
Change Password of cases, with subclinical involvement detectable by a
water-loading test in more than 50%. Restriction of Endocrinology
Sign Out
fluid to a daily intake of 700–1000 ml redresses the
hyponatraemia. Also, demethylchlortetracycline Blood gases
(demeclocycline) 600–1200 mg daily is often
CSF
effective. Infusion of hypertonic saline is hazardous,
often precipitating cardiac failure or cerebral oedema.

2005

© 2011 PasTest Ltd | About Us | Contact Us | Help

file:///E|/Shakhawan/Respiratory%20S/217a.htm[3/13/2012 4:06:28 PM]


MyPasTest

Main Navigation
Question Browser: MRCP 1
Home
Subscriptions Question Browser Exam Builder Saved Exams
» MRCP 1 Session Progress
• Question Browser
Questions Correct 2
• Timed Test A 26-year-old man presents with fever, headache and a
non-productive cough. The chest X-ray shows increased Questions Incorrect 0
• Mock Exam
interstitial markings. The laboratory examination shows an Questions Total 2
• Past Papers elevated LDH and anaemia with the presence of cold
Questions Percentage 100 %
• Random Questions agglutinins. What is the most appropriate treatment?
• My Performance More
Erythromycin
• Media Bank
Piperacillin
• New Multimedia
Clindamycin Reference: Normal Values
Online Extras
Ampicillin Haematology
Library
Imipenem
Community Immunoglobulins
Help 2006 Biochemistry
PasTest Store Diabetes
Endocrinology
My Account
Blood gases
Profile
CSF
Newsletters
My Career and Exams
Order History
Learning Goals
Question Filters

Security

Change Password
Sign Out

© 2011 PasTest Ltd | About Us | Contact Us | Help

file:///E|/Shakhawan/Respiratory%20S/218.htm[3/13/2012 4:06:30 PM]


MyPasTest

Main Navigation
Question Browser: MRCP 1
Home
Subscriptions Question Browser Exam Builder Saved Exams
» MRCP 1 Session Progress
• Question Browser
Questions Correct 3
• Timed Test A 26-year-old man presents with fever, headache and a
non-productive cough. The chest X-ray shows increased Questions Incorrect 0
• Mock Exam
interstitial markings. The laboratory examination shows an Questions Total 3
• Past Papers elevated LDH and anaemia with the presence of cold
Questions Percentage 100 %
• Random Questions agglutinins. What is the most appropriate treatment?
• My Performance More
Erythromycin Your answer
• Media Bank
Piperacillin
• New Multimedia
Clindamycin
Online Extras
Ampicillin
Library
Imipenem
Community
Help
PasTest Store

This patient presents with mycoplasma pneumonia.


My Account With regard to treatment, the pathogen lacks a cell
wall and hence is not susceptible to penicillin,
Profile cephalosporins or other cell-wall active antibiotics.
Newsletters Macrolide antibiotics (eg erythromycin or
clarithromycin) are the treatment of choice for
My Career and Exams mycoplasma and other atypical pneumonias. The most
Order History frequent side-effects are nausea, vomiting and Reference: Normal Values
diarrhoea. It should be kept in mind that treatment is
Learning Goals Haematology
likely to be most effective when given over a long
Question Filters rather than short time, suboptimal doses are avoided
Immunoglobulins
and compliance is strict.
Acute cold autoimmune haemolytic anaemia is Biochemistry
Security
commonly seen in adolescents and young adults Diabetes
Change Password following infection with mycoplasma. Haemolysis
occurs approximately 1–2 weeks following infection Endocrinology
Sign Out
and is most commonly associated with a rise in
polyclonal anti-IgM antibodies with mycoplasma Blood gases
pneumonia. The typical patient is usually a young
CSF
adult who experiences a respiratory tract infection
accompanied by headache, myalgia, cough and fever,
and with a chest X-ray that shows
bronchopneumonia. The cough is often non-
productive, but when sputum is obtained it is mucoid,
shows predominantly mononuclear cells and no
dominant organism. A characteristic feature is the
relatively high frequency of extrapulmonary
complications such as rash, neurological syndromes
(aseptic meningitis, encephalitis, neuropathies),
myocarditis, pericarditis and haemolytic anaemia. The
diagnosis should be suspected in those patients with a
relatively mild form of pneumonia, particularly in
previously healthy young adults.

2006

© 2011 PasTest Ltd | About Us | Contact Us | Help

file:///E|/Shakhawan/Respiratory%20S/218a.htm[3/13/2012 4:06:31 PM]


MyPasTest

Main Navigation
Question Browser: MRCP 1
Home
Subscriptions Question Browser Exam Builder Saved Exams
» MRCP 1 Session Progress
• Question Browser
Questions Correct 3
• Timed Test A 24-year-old man with HIV and a CD4+ lymphocyte count
Questions Incorrect 0
• Mock Exam of 150/ μl has been complaining of gradually worsening
dyspnoea associated with a non-productive cough and Questions Total 3
• Past Papers
fever for the last 2 weeks. A chest X-ray shows bilateral Questions Percentage 100 %
• Random Questions diffuse ground-glass opacities. What is the most
• My Performance appropriate therapy? More
• Media Bank Glucocorticoids
• New Multimedia Erythromycin Reference: Normal Values
Online Extras Cefaclor
Haematology
Library Ampicillin
Community Immunoglobulins
Co-trimoxazole
Help Biochemistry
PasTest Store 2007
Diabetes
Endocrinology
My Account
Blood gases
Profile
CSF
Newsletters
My Career and Exams
Order History
Learning Goals
Question Filters

Security

Change Password
Sign Out

© 2011 PasTest Ltd | About Us | Contact Us | Help

file:///E|/Shakhawan/Respiratory%20S/219.htm[3/13/2012 4:06:33 PM]


MyPasTest

Main Navigation
Question Browser: MRCP 1
Home
Subscriptions Question Browser Exam Builder Saved Exams
» MRCP 1 Session Progress
• Question Browser
Questions Correct 4
• Timed Test A 24-year-old man with HIV and a CD4+ lymphocyte count
Questions Incorrect 0
• Mock Exam of 150/ μl has been complaining of gradually worsening
dyspnoea associated with a non-productive cough and Questions Total 4
• Past Papers
fever for the last 2 weeks. A chest X-ray shows bilateral Questions Percentage 100 %
• Random Questions diffuse ground-glass opacities. What is the most
• My Performance appropriate therapy? More
• Media Bank Glucocorticoids
• New Multimedia Erythromycin
Online Extras Cefaclor
Library Ampicillin
Community Co-trimoxazole Your answer
Help
PasTest Store

My Account High-dose co-trimoxazole (120 mg/kg daily in divided


doses) for 3 weeks is the first-line treatment for
Profile
pneumocystis pneumonia. Oral therapy is often
Newsletters adequate, but in moderate and severe cases the drug
My Career and Exams should be given intravenously. Pneumocystis jiroveci
(formerly called P. carinii) pneumonia typically
Order History presents with gradually increasing dyspnoea and Reference: Normal Values
Learning Goals cough over weeks, but sometimes as an acute illness
Haematology
with rapid deterioration over a few days. Although the
Question Filters chest X-ray usually shows diffuse ground-glass Immunoglobulins
opacities, which strongly suggests the diagnosis, it
sometimes shows nodular opacities, lobar Biochemistry
Security consolidation, or a normal film.
Diabetes
Change Password Cystic abnormalities and spontaneous
Sign Out pneumothoraces in patients with known or suspected Endocrinology
HIV infection are usually caused by Pneumocystis
jiroveci pneumonia. However, Pneumocystis jiroveci Blood gases
pneumonia is unlikely in a patient who has had a CSF
CD4+ cell count above 200 cells/mm 3 in the
preceding 2 months in the absence of other HIV-
associated symptoms. Approximately 90% of patients
with Pneumocystis jiroveci pneumonia have an
elevated serum lactic dehydrogenase, but this may
occur with other pulmonary diseases.

All HIV patients with a CD4 count below 200/mm3


should receive effective prophylaxis with low-dose
suppressive therapy to prevent Pneumocystis jiroveci
pneumonia. Co-trimoxazole is the preferred agent.
Dapsone and inhaled pentamidine are also used.

2007

© 2011 PasTest Ltd | About Us | Contact Us | Help

file:///E|/Shakhawan/Respiratory%20S/219a.htm[3/13/2012 4:06:34 PM]


MyPasTest

Main Navigation
Question Browser: MRCP 1
Home
Subscriptions Question Browser Exam Builder Saved Exams
» MRCP 1 Session Progress
• Question Browser
Questions Correct 4
• Timed Test A 36-year-old lorry driver who smokes heavily presents
with a 2-day history of cough associated with fever. He Questions Incorrect 0
• Mock Exam
also complains of right-sided chest pain on inspiration. On Questions Total 4
• Past Papers examination he is slightly cyanosed. His temperature is 38
Questions Percentage 100 %
• Random Questions °C, respiratory rate 38/min, BP 100/70 mmHg and pulse
130/min. He has basal crepitations and dullness to More
• My Performance
percussion at the right lung base. What is the most
• Media Bank important next step in confirming the diagnosis?
• New Multimedia
ESR (Erythrocyte sedimentation rate) Reference: Normal Values
Online Extras
d-Dimer Haematology
Library
Chest X-ray
Community Immunoglobulins
Sputum sample
Help Biochemistry
Blood cultures
PasTest Store Diabetes
2008
Endocrinology
My Account
Blood gases
Profile
CSF
Newsletters
My Career and Exams
Order History
Learning Goals
Question Filters

Security

Change Password
Sign Out

© 2011 PasTest Ltd | About Us | Contact Us | Help

file:///E|/Shakhawan/Respiratory%20S/220.htm[3/13/2012 4:06:36 PM]


MyPasTest

Main Navigation
Question Browser: MRCP 1
Home
Subscriptions Question Browser Exam Builder Saved Exams
» MRCP 1 Session Progress
• Question Browser
Questions Correct 4
• Timed Test A 36-year-old lorry driver who smokes heavily presents
with a 2-day history of cough associated with fever. He Questions Incorrect 1
• Mock Exam
also complains of right-sided chest pain on inspiration. On Questions Total 5
• Past Papers examination he is slightly cyanosed. His temperature is 38
Questions Percentage 80 %
• Random Questions °C, respiratory rate 38/min, BP 100/70 mmHg and pulse
130/min. He has basal crepitations and dullness to More
• My Performance
percussion at the right lung base. What is the most
• Media Bank important next step in confirming the diagnosis?
• New Multimedia
ESR (Erythrocyte sedimentation
Online Extras rate)
Library d-Dimer
Community Chest X-ray Correct answer
Help Sputum sample Your answer
PasTest Store Blood cultures

My Account

Profile
The classic presentation of pneumonia is of a cough and
Newsletters
fever with the variable presence of sputum production,
My Career and Exams dyspnoea and pleurisy. Most patients have
Order History constitutional symptoms such as malaise, fatigue and Reference: Normal Values
asthenia, and many also have gastrointestinal
Learning Goals symptoms. Examination of the lung might reveal Haematology
Question Filters decreased vesicular breath sounds, localised foci of
crepitations, dullness to percussion and sometimes a Immunoglobulins
bronchial wheeze. The chest X-ray is a pivotal test for
Biochemistry
Security the confirmation of pneumonia.
Diabetes
Change Password
Sign Out 2008 Endocrinology

Blood gases

CSF

© 2011 PasTest Ltd | About Us | Contact Us | Help

file:///E|/Shakhawan/Respiratory%20S/220a.htm[3/13/2012 4:06:37 PM]


MyPasTest

Main Navigation
Question Browser: MRCP 1
Home
Subscriptions Question Browser Exam Builder Saved Exams
» MRCP 1 Session Progress
• Question Browser
Questions Correct 4
• Timed Test A 67-year-old patient with lung cancer complains of a
cough and has difficulty breathing. He has swelling of his Questions Incorrect 1
• Mock Exam
face, neck, upper body and arms. What is the most likely Questions Total 5
• Past Papers diagnosis?
Questions Percentage 80 %
• Random Questions
• My Performance Side-effects from radiotherapy More
• Media Bank Allergic reaction

• New Multimedia Superior vena cava syndrome


Reference: Normal Values
Online Extras Side-effect from chemotherapy
Acute myocardial infarction Haematology
Library
Community Immunoglobulins
2009
Help Biochemistry
PasTest Store Diabetes
Endocrinology
My Account
Blood gases
Profile
CSF
Newsletters
My Career and Exams
Order History
Learning Goals
Question Filters

Security

Change Password
Sign Out

© 2011 PasTest Ltd | About Us | Contact Us | Help

file:///E|/Shakhawan/Respiratory%20S/221.htm[3/13/2012 4:06:39 PM]


MyPasTest

Main Navigation
Question Browser: MRCP 1
Home
Subscriptions Question Browser Exam Builder Saved Exams
» MRCP 1 Session Progress
• Question Browser
Questions Correct 5
• Timed Test A 67-year-old patient with lung cancer complains of a
cough and has difficulty breathing. He has swelling of his Questions Incorrect 1
• Mock Exam
face, neck, upper body and arms. What is the most likely Questions Total 6
• Past Papers diagnosis?
Questions Percentage 83 %
• Random Questions
• My Performance Side-effects from radiotherapy More
• Media Bank Allergic reaction

• New Multimedia Superior vena cava syndrome Your answer

Online Extras Side-effect from chemotherapy

Library Acute myocardial infarction

Community
Help
PasTest Store
Superior vena cava syndrome (SVCS) is a collection of
symptoms caused by the partial blockage of the vein
My Account that carries blood from the head, neck, chest and arms
to the heart. Symptoms that may indicate this
Profile syndrome include difficulty breathing, coughing, and
swelling of the face, neck, upper body and arms. In rare
Newsletters
instances, patients may complain of hoarseness, chest
My Career and Exams pain, difficulty swallowing and coughing up blood.
Order History Physical signs of SVCS include swelling of the neck or Reference: Normal Values
chest veins, collection of fluid in the face or arms and
Learning Goals rapid breathing. The syndrome of superior vena cava Haematology
Question Filters obstruction is relieved in about 80% of cases, but
usually requires a more conventional course of 5–10 Immunoglobulins
fractions of radiotherapy. Some centres now use SVC
Biochemistry
Security stents for symptom relief, placed by an interventional
radiologist. Diabetes
Change Password
Sign Out Endocrinology
2009
Blood gases

CSF

© 2011 PasTest Ltd | About Us | Contact Us | Help

file:///E|/Shakhawan/Respiratory%20S/221a.htm[3/13/2012 4:06:40 PM]


MyPasTest

Main Navigation
Question Browser: MRCP 1
Home
Subscriptions Question Browser Exam Builder Saved Exams
» MRCP 1 Session Progress
• Question Browser
Questions Correct 5
• Timed Test A patient with tuberculosis was initially treated with
streptomycin, which was later changed to a combination of Questions Incorrect 1
• Mock Exam
isoniazid, rifampicin, pyrazinamide and ethambutol. Questions Total 6
• Past Papers Abnormal liver functions are noted on this, his follow-up,
Questions Percentage 83 %
• Random Questions visit. Which drug is most likely to be responsible?
• My Performance More
Streptomycin
• Media Bank
Ethambutol
• New Multimedia
Rifampicin Reference: Normal Values
Online Extras
Pyrazinamide Haematology
Library
Isoniazid
Community Immunoglobulins
Help 2010 Biochemistry
PasTest Store Diabetes
Endocrinology
My Account
Blood gases
Profile
CSF
Newsletters
My Career and Exams
Order History
Learning Goals
Question Filters

Security

Change Password
Sign Out

© 2011 PasTest Ltd | About Us | Contact Us | Help

file:///E|/Shakhawan/Respiratory%20S/222.htm[3/13/2012 4:06:42 PM]


MyPasTest

Main Navigation
Question Browser: MRCP 1
Home
Subscriptions Question Browser Exam Builder Saved Exams
» MRCP 1 Session Progress
• Question Browser
Questions Correct 5
• Timed Test A patient with tuberculosis was initially treated with
streptomycin, which was later changed to a combination of Questions Incorrect 2
• Mock Exam
isoniazid, rifampicin, pyrazinamide and ethambutol. Questions Total 7
• Past Papers Abnormal liver functions are noted on this, his follow-up,
Questions Percentage 71 %
• Random Questions visit. Which drug is most likely to be responsible?
• My Performance More
Streptomycin
• Media Bank
Ethambutol
• New Multimedia
Rifampicin Your answer
Online Extras
Pyrazinamide
Library
Isoniazid Correct answer
Community
Help
PasTest Store

Although antituberculosis therapy is remarkably well


My Account tolerated and almost always given to ambulant
patients, drug toxicities do exist. The most serious
Profile adverse drug reaction during tuberculosis treatment is
Newsletters liver toxicity, which may occur in up to 5–10% of
treated patients.
My Career and Exams
Isoniazid, rifampicin and pyrazinamide are all
Order History Reference: Normal Values
associated with liver toxicity. Use of these agents
Learning Goals together increases the risk of a reaction. Isoniazid Haematology
Question Filters causes more hepatotoxicity than rifampicin or
pyrazinamide, and is the agent most frequently Immunoglobulins
implicated when reactions occur. Isoniazid can
produce an idiosyncratic hepatocellular injury, Biochemistry
Security
manifested by elevated liver enzymes and clinical Diabetes
Change Password hepatitis. Elevation of transaminases does not always
portend the development of hepatitis, but may serve Endocrinology
Sign Out
as an important signal to anticipate clinical toxicity.
Blood gases
The development of signs and symptoms of hepatitis,
such as abdominal pain, nausea, vomiting or jaundice, CSF
requires immediate discontinuation of isoniazid, as
continuing treatment may result in death from hepatic
failure. Risk factors for developing isoniazid
hepatotoxicity include increasing age, chronic liver
disease, alcohol abuse, daily dosing of isoniazid and
the use of other hepatotoxic drugs, including
rifampicin. In addition, people with a slow isoniazid-
acetylation genotype are significantly more likely to
develop hepatotoxicity from the drug than
intermediate or rapid acetylators.

2010

© 2011 PasTest Ltd | About Us | Contact Us | Help

file:///E|/Shakhawan/Respiratory%20S/222a.htm[3/13/2012 4:06:44 PM]


MyPasTest

Main Navigation
Question Browser: MRCP 1
Home
Subscriptions Question Browser Exam Builder Saved Exams
» MRCP 1 Session Progress
• Question Browser
Questions Correct 5
• Timed Test Which is the most common malignant tumour found
in the lung? Questions Incorrect 2
• Mock Exam
Questions Total 7
• Past Papers
Carcinoid tumour Questions Percentage 71 %
• Random Questions
Squamous-cell carcinoma of the bronchus
• My Performance More
Metastatic carcinoma
• Media Bank
Adenocarcinoma of the bronchus
• New Multimedia
Oat-cell carcinoma Reference: Normal Values
Online Extras
2011 Haematology
Library
Community Immunoglobulins
Help Biochemistry
PasTest Store Diabetes
Endocrinology
My Account
Blood gases
Profile
CSF
Newsletters
My Career and Exams
Order History
Learning Goals
Question Filters

Security

Change Password
Sign Out

© 2011 PasTest Ltd | About Us | Contact Us | Help

file:///E|/Shakhawan/Respiratory%20S/223.htm[3/13/2012 4:06:45 PM]


MyPasTest

Main Navigation
Question Browser: MRCP 1
Home
Subscriptions Question Browser Exam Builder Saved Exams
» MRCP 1 Session Progress
• Question Browser
Questions Correct 6
• Timed Test Which is the most common malignant tumour found
Questions Incorrect 2
• Mock Exam in the lung?
Questions Total 8
• Past Papers
Carcinoid tumour Questions Percentage 75 %
• Random Questions
Squamous-cell carcinoma of the
• My Performance bronchus More
• Media Bank Metastatic carcinoma Your answer
• New Multimedia Adenocarcinoma of the bronchus
Online Extras Oat-cell carcinoma
Library
Community
Help
PasTest Store Malignant metastases to the lung may present as a
solitary enlarging nodule, as multiple nodules, or with
diffuse lymphatic involvement. Solitary metastasis
My Account represents some 10% of round lesions in general, but
some 70% of round lesions in patients with a known
Profile malignancy. Colorectal cancer is reported to be the
Newsletters commonest tumour of origin. Diagnosis can usually be
secured by percutaneous CT-guided biopsy. In rare
My Career and Exams cases, surgical excision may prolong survival or result
Order History in cure, depending on the state of the primary tumour Reference: Normal Values
and the likelihood of other occult metastases. In
Learning Goals Haematology
general, the longer the interval between resection of
Question Filters the primary tumour and the appearance of the
Immunoglobulins
metastases, the better the prognosis.
Multiple metastases range enormously in size and Biochemistry
Security
number, from cannon balls to miliary shadowing, and Diabetes
Change Password may be accompanied by hilar lymphadenopathy or
pleural effusion. Breast, colon, renal and lung Endocrinology
Sign Out
primaries are probably the commonest underlying
tumours, but other tumours amenable to Blood gases
chemotherapy, such as testicular cancer and
CSF
choriocarcinoma, and also sarcomas, occur. Diagnosis
may be achieved by cytology or histology on various
samples from the pleura or lung and can occasionally
be made from cytology on expectoration or induced
sputum. Tumours that are suitable for chemotherapy
(eg choriocarcinoma) or endocrine manipulation (eg
breast) need to be recognised.
Solitary or multiple Kaposi’s sarcoma is a feature of
AIDS, and can involve the bronchi and pleura as well
as lung tissue. Lymphangitis carcinomatosa is most
commonly due to breast and primary lung tumours
(usually adenocarcinomas). Patients can be
asymptomatic when the disease is first suspected on
the basis of a radiograph showing diffusely increased
interstitial markings accompanied by Kerley B lines,
hilar lymphadenopathy or pleural effusion. Although
diagnosis may be established by cytology from
sputum or pleural fluid, it often requires a
bronchoscopic or transbronchial lung biopsy. Later,
progressive and severe breathlessness with
hypoxaemia often develops, and requires vigorous
palliative relief with opiate and oxygen administration.
Occasionally, metastases, presenting as haemoptysis,
may be confined to a bronchus and not be visible on a

file:///E|/Shakhawan/Respiratory%20S/223a.htm[3/13/2012 4:06:47 PM]


MyPasTest

plain chest X-ray. Renal carcinoma and malignant


melanoma are recorded causes. Diagnosis requires
bronchoscopy, and radiotherapy is usually effective in
controlling the haemoptysis.

2011

© 2011 PasTest Ltd | About Us | Contact Us | Help

file:///E|/Shakhawan/Respiratory%20S/223a.htm[3/13/2012 4:06:47 PM]


MyPasTest

Main Navigation
Question Browser: MRCP 1
Home
Subscriptions Question Browser Exam Builder Saved Exams
» MRCP 1 Session Progress
• Question Browser
Questions Correct 6
• Timed Test A patient with cystic fibrosis presents with a severe
infective exacerbation. Questions Incorrect 2
• Mock Exam
Questions Total 8
• Past Papers What is the most likely pathogen?
Questions Percentage 75 %
• Random Questions
Streptococcus pyogenes More
• My Performance
Streptococcus pneumoniae
• Media Bank
Pseudomonas aeruginosa
• New Multimedia
Klebsiella pneumoniae Reference: Normal Values
Online Extras
An atypical Mycobacteria sp Haematology
Library
Community 2012 Immunoglobulins
Help Biochemistry
PasTest Store Diabetes
Endocrinology
My Account
Blood gases
Profile
CSF
Newsletters
My Career and Exams
Order History
Learning Goals
Question Filters

Security

Change Password
Sign Out

© 2011 PasTest Ltd | About Us | Contact Us | Help

file:///E|/Shakhawan/Respiratory%20S/224.htm[3/13/2012 4:06:48 PM]


MyPasTest

Main Navigation
Question Browser: MRCP 1
Home
Subscriptions Question Browser Exam Builder Saved Exams
» MRCP 1 Session Progress
• Question Browser
Questions Correct 7
• Timed Test A patient with cystic fibrosis presents with a severe
infective exacerbation. Questions Incorrect 2
• Mock Exam
Questions Total 9
• Past Papers What is the most likely pathogen?
Questions Percentage 77 %
• Random Questions
Streptococcus pyogenes More
• My Performance
Streptococcus pneumoniae
• Media Bank
Pseudomonas aeruginosa Your answer
• New Multimedia
Klebsiella pneumoniae
Online Extras
An atypical Mycobacteria sp
Library
Community
Help
PasTest Store
Pseudomonas aeruginosa is the commonest colonising
organism in patients with cystic fibrosis after the age of
My Account 10 years, with a reported prevalence varying between
40 and 80%. Enhanced adherence to the airways of
Profile patients with cystic fibrosis promotes colonisation, but
prior antibiotic treatment may play a part. No particular
Newsletters phage type predominates, but siblings with cystic
My Career and Exams fibrosis often carry the same type, and environmental
sources have been identified in cystic fibrosis centres, Reference: Normal Values
Order History
dentistry equipment, hydrotherapy pools and nebulisers.
Learning Goals After some months or years of colonisation, Haematology
Question Filters pseudomonas produces mucoid alginate as a protective
biofilm and the organisms live in mucoid microcolonies. Immunoglobulins
This mucoid variant is associated with a worse
prognosis and greater antibiotic resistance. Most Biochemistry
Security
colonising types of Pseudomonas aeruginosa are Diabetes
Change Password sensitive to antibiotics at first, but over the years and in
association with antibiotic treatment, multiple resistance Endocrinology
Sign Out
to most antibiotics (except colistin (eg Colomycin))
develops. Blood gases

CSF
2012

© 2011 PasTest Ltd | About Us | Contact Us | Help

file:///E|/Shakhawan/Respiratory%20S/224a.htm[3/13/2012 4:06:50 PM]


MyPasTest

Main Navigation
Question Browser: MRCP 1
Home
Subscriptions Question Browser Exam Builder Saved Exams
» MRCP 1 Session Progress
• Question Browser
Questions Correct 7
• Timed Test A 16-year-old girl presents to A&E with a severe asthma
attack. What is the most important therapy to relieve her Questions Incorrect 2
• Mock Exam
bronchoconstriction? Questions Total 9
• Past Papers
Questions Percentage 77 %
• Random Questions Propranolol
• My Performance Salbutamol More
• Media Bank Oxygen
• New Multimedia Glucocorticosteroids
Reference: Normal Values
Online Extras Cromoglycate
Haematology
Library
2013
Community Immunoglobulins
Help Biochemistry
PasTest Store Diabetes
Endocrinology
My Account
Blood gases
Profile
CSF
Newsletters
My Career and Exams
Order History
Learning Goals
Question Filters

Security

Change Password
Sign Out

© 2011 PasTest Ltd | About Us | Contact Us | Help

file:///E|/Shakhawan/Respiratory%20S/225.htm[3/13/2012 4:06:51 PM]


MyPasTest

Main Navigation
Question Browser: MRCP 1
Home
Subscriptions Question Browser Exam Builder Saved Exams
» MRCP 1 Session Progress
• Question Browser
Questions Correct 8
• Timed Test A 16-year-old girl presents to A&E with a severe asthma
attack. What is the most important therapy to relieve her Questions Incorrect 2
• Mock Exam
bronchoconstriction? Questions Total 10
• Past Papers
Questions Percentage 80 %
• Random Questions Propranolol
• My Performance Salbutamol Your answer More
• Media Bank Oxygen
• New Multimedia Glucocorticosteroids
Online Extras Cromoglycate
Library
Community
Help
PasTest Store The actions of b-agonists in asthma are the result of
stimulation of b -adrenoreceptors that are located in the
airways, on airway epithelium, and in submucosal
My Account glands, airway and vascular smooth muscle. b -
Adrenergic receptors in the airways are entirely b 2 ,
Profile with the exception of some β 1 -receptors on
Newsletters submucosal glands. β 2 -Agonists can influence airway
My Career and Exams function through several mechanisms:
Order History Reference: Normal Values
relaxation of bronchial smooth muscle by direct
Learning Goals effect on b 2 -receptors Haematology
Question Filters inhibition of mast-cell mediator release, and
enhanced mucociliary clearance. Immunoglobulins
Biochemistry
Security Inhalation of a β 2 -agonist by a patient with asthma
increases airway calibre and reduces airway Diabetes
Change Password
hyperresponsiveness. β 2 -Agonists also cause
Sign Out Endocrinology
tachycardia and increased cardiac output, systemic
vasodilatation and increased muscle blood flow. The Blood gases
tachycardia and increased cardiac output are the results
of both stimulation of cardiac β -adrenoreceptors and a CSF
reflex response to peripheral vasodilatation. In addition,
β 2 -agonists cause tremor and have metabolic actions,
of which hypokalaemia is probably the only potentially
important clinical effect.

2013

© 2011 PasTest Ltd | About Us | Contact Us | Help

file:///E|/Shakhawan/Respiratory%20S/225a.htm[3/13/2012 4:06:53 PM]


MyPasTest

Main Navigation
Question Browser: MRCP 1
Home
Subscriptions Question Browser Exam Builder Saved Exams
» MRCP 1 Session Progress
• Question Browser
Questions Correct 8
• Timed Test A 30-year-old shepherd presents with a 1-week history of
headaches, rhinitis and pharyngitis associated with high Questions Incorrect 2
• Mock Exam
fever and you elicit a systolic murmur on examination. Questions Total 10
• Past Papers Three days ago he noticed a painful cough and blood-
Questions Percentage 80 %
• Random Questions stained phlegm. What is the most likely diagnosis?
• My Performance More
Tuberculosis
• Media Bank
Borreliosis
• New Multimedia
Brucellosis Reference: Normal Values
Online Extras
Q-fever Haematology
Library
Pulmonary embolism
Community Immunoglobulins
Help 2014 Biochemistry
PasTest Store Diabetes
Endocrinology
My Account
Blood gases
Profile
CSF
Newsletters
My Career and Exams
Order History
Learning Goals
Question Filters

Security

Change Password
Sign Out

© 2011 PasTest Ltd | About Us | Contact Us | Help

file:///E|/Shakhawan/Respiratory%20S/226.htm[3/13/2012 4:06:54 PM]


MyPasTest

Main Navigation
Question Browser: MRCP 1
Home
Subscriptions Question Browser Exam Builder Saved Exams
» MRCP 1 Session Progress
• Question Browser
Questions Correct 9
• Timed Test A 30-year-old shepherd presents with a 1-week history of
headaches, rhinitis and pharyngitis associated with high Questions Incorrect 2
• Mock Exam
fever and you elicit a systolic murmur on examination. Questions Total 11
• Past Papers Three days ago he noticed a painful cough and blood-
Questions Percentage 81 %
• Random Questions stained phlegm. What is the most likely diagnosis?
• My Performance More
Tuberculosis
• Media Bank
Borreliosis
• New Multimedia
Brucellosis
Online Extras
Q-fever Your answer
Library
Pulmonary embolism
Community
Help
PasTest Store

Q-fever is a zoonotic disease caused by Coxiella


My Account burnetii, a species of bacteria that is distributed
globally. In 1999, Q-fever became a notifiable disease
Profile in the United States but reporting is not required in
Newsletters many other countries. Because the disease is
underreported, scientists cannot reliably assess how
My Career and Exams many cases of Q-fever have actually occurred world-
Order History wide. Many human infections are unapparent. Cattle, Reference: Normal Values
sheep and goats are the primary reservoirs of Coxiella
Learning Goals Haematology
burnetii. Only about one-half of all people infected
Question Filters with Coxiella burnetii show signs of clinical illness.
Immunoglobulins
Most acute cases of Q-fever begin with a sudden
onset of one or more of the following: high fevers, Biochemistry
Security
severe headache, general malaise, myalgia, confusion, Diabetes
Change Password sore throat, chills, sweats, non-productive cough,
nausea, vomiting, diarrhoea, abdominal pain and Endocrinology
Sign Out
chest pain. Fever usually lasts for 1–2 weeks. Weight
loss can occur and persist for some time. Between 30 Blood gases
and 50% of patients with a symptomatic infection will
CSF
develop pneumonia. Additionally, the majority of
patients have abnormal results on liver function tests
and some will develop hepatitis. Confirming a
diagnosis of Q-fever requires serological testing to
detect the presence of Coxiella burnetii.
In general, most patients will recover to good health
within several months with no treatment. Only 1–2%
of people with acute Q-fever die of the disease.
Doxycycline is the treatment of choice for acute Q-
fever.

2014

© 2011 PasTest Ltd | About Us | Contact Us | Help

file:///E|/Shakhawan/Respiratory%20S/226a.htm[3/13/2012 4:06:56 PM]


MyPasTest

Main Navigation
Question Browser: MRCP 1
Home
Subscriptions Question Browser Exam Builder Saved Exams
» MRCP 1 Session Progress
• Question Browser
Questions Correct 9
• Timed Test A 25-year-old patient with epilepsy aspirated gastric juice
after a seizure (Mendelson syndrome). What is the most Questions Incorrect 2
• Mock Exam
likely clinical symptom? Questions Total 11
• Past Papers
Questions Percentage 81 %
• Random Questions Bradypnoea
• My Performance Paradoxical breathing More
• Media Bank Bronchospasm
• New Multimedia Laryngospasm
Reference: Normal Values
Online Extras Tenderness to percussion
Haematology
Library
2015
Community Immunoglobulins
Help Biochemistry
PasTest Store Diabetes
Endocrinology
My Account
Blood gases
Profile
CSF
Newsletters
My Career and Exams
Order History
Learning Goals
Question Filters

Security

Change Password
Sign Out

© 2011 PasTest Ltd | About Us | Contact Us | Help

file:///E|/Shakhawan/Respiratory%20S/227.htm[3/13/2012 4:06:57 PM]


MyPasTest

Main Navigation
Question Browser: MRCP 1
Home
Subscriptions Question Browser Exam Builder Saved Exams
» MRCP 1 Session Progress
• Question Browser
Questions Correct 10
• Timed Test A 25-year-old patient with epilepsy aspirated gastric juice
after a seizure (Mendelson syndrome). What is the most Questions Incorrect 2
• Mock Exam
likely clinical symptom? Questions Total 12
• Past Papers
Questions Percentage 83 %
• Random Questions Bradypnoea
• My Performance Paradoxical breathing More
• Media Bank Bronchospasm Your answer
• New Multimedia Laryngospasm
Online Extras Tenderness to percussion
Library
Community
Help
PasTest Store Mendelson’s syndrome is an acute pneumonia caused by
regurgitation of stomach contents and aspiration of
chemical material, usually gastric juices. It can cause
My Account severe bronchospasm. The pneumonia develops rapidly,
and within hours the patient may become tachypnoeic,
Profile hypoxic and febrile. There is minimal sputum. The
condition often follows anaesthesia when the gag reflex
Newsletters
is depressed.
My Career and Exams
Order History Reference: Normal Values
2015
Learning Goals Haematology
Question Filters
Immunoglobulins
Biochemistry
Security
Diabetes
Change Password
Sign Out Endocrinology

Blood gases

CSF

© 2011 PasTest Ltd | About Us | Contact Us | Help

file:///E|/Shakhawan/Respiratory%20S/227a.htm[3/13/2012 4:06:59 PM]


MyPasTest

Main Navigation
Question Browser: MRCP 1
Home
Subscriptions Question Browser Exam Builder Saved Exams
» MRCP 1 Session Progress
• Question Browser
Questions Correct 10
• Timed Test The 18-year-old son of an immigrant from Bangladesh who
recently came to the UK has been complaining of tiredness, Questions Incorrect 2
• Mock Exam
weight loss and generally not feeling well for the last Questions Total 12
• Past Papers month. He presents to his GP because of haemoptysis,
Questions Percentage 83 %
• Random Questions especially in the morning. What is the most likely
diagnosis? More
• My Performance
• Media Bank Pneumothorax
• New Multimedia Pulmonary embolism Reference: Normal Values
Online Extras Lung cancer
Haematology
Library Pulmonary tuberculosis
Community Asthma Immunoglobulins
Help Biochemistry
2016
PasTest Store Diabetes
Endocrinology
My Account
Blood gases
Profile
CSF
Newsletters
My Career and Exams
Order History
Learning Goals
Question Filters

Security

Change Password
Sign Out

© 2011 PasTest Ltd | About Us | Contact Us | Help

file:///E|/Shakhawan/Respiratory%20S/228.htm[3/13/2012 4:07:01 PM]


MyPasTest

Main Navigation
Question Browser: MRCP 1
Home
Subscriptions Question Browser Exam Builder Saved Exams
» MRCP 1 Session Progress
• Question Browser
Questions Correct 11
• Timed Test The 18-year-old son of an immigrant from Bangladesh who
recently came to the UK has been complaining of tiredness, Questions Incorrect 2
• Mock Exam
weight loss and generally not feeling well for the last Questions Total 13
• Past Papers month. He presents to his GP because of haemoptysis,
Questions Percentage 84 %
• Random Questions especially in the morning. What is the most likely
diagnosis? More
• My Performance
• Media Bank Pneumothorax
• New Multimedia Pulmonary embolism
Online Extras Lung cancer
Library Pulmonary tuberculosis Your answer
Community Asthma
Help
PasTest Store

My Account Typically tuberculosis affects young adults, with a peak


incidence in those aged 25–44 years. The dynamics of
Profile tuberculosis within a particular country or region,
however, reflects both historical trends in tuberculosis
Newsletters
transmission and current risk factors and practices of
My Career and Exams disease control. In Western Europe, for example,
Order History tuberculosis is seen in two demographic groups: elderly Reference: Normal Values
native Europeans who were presumably infected many
Learning Goals years ago and who experience reactivation of latent Haematology
Question Filters infections as they age or become immunocompromised,
and younger immigrants from high-incidence countries Immunoglobulins
in the developing world.
Biochemistry
Security
Diabetes
Change Password 2016
Sign Out Endocrinology

Blood gases

CSF

© 2011 PasTest Ltd | About Us | Contact Us | Help

file:///E|/Shakhawan/Respiratory%20S/228a.htm[3/13/2012 4:07:02 PM]


MyPasTest

Main Navigation
Question Browser: MRCP 1
Home
Subscriptions Question Browser Exam Builder Saved Exams
» MRCP 1 Session Progress
• Question Browser
Questions Correct 11
• Timed Test A 30-year-old asthmatic patient has the following drug
regimen: regular inhaled corticosteroids, regular inhaled Questions Incorrect 2
• Mock Exam
long-acting β 2 -agonists (salmeterol) and inhaled short- Questions Total 13
• Past Papers
acting β 2 -agonists when required. Although her Questions Percentage 84 %
• Random Questions
compliance is good, her symptoms are still not satisfactorily
• My Performance More
controlled. What is the next step in her therapy?
• Media Bank
Antibiotics
• New Multimedia
Salbutamol nebuliser Reference: Normal Values
Online Extras
Oral steroids Haematology
Library
Oral cromoglycate Immunoglobulins
Community
Oral leukotriene-receptor antagonists
Help Biochemistry
PasTest Store 2017 Diabetes
Endocrinology
My Account
Blood gases
Profile
CSF
Newsletters
My Career and Exams
Order History
Learning Goals
Question Filters

Security

Change Password
Sign Out

© 2011 PasTest Ltd | About Us | Contact Us | Help

file:///E|/Shakhawan/Respiratory%20S/229.htm[3/13/2012 4:07:04 PM]


MyPasTest

Main Navigation
Question Browser: MRCP 1
Home
Subscriptions Question Browser Exam Builder Saved Exams
» MRCP 1 Session Progress
• Question Browser
Questions Correct 12
• Timed Test A 30-year-old asthmatic patient has the following drug
regimen: regular inhaled corticosteroids, regular inhaled Questions Incorrect 2
• Mock Exam
long-acting β 2 -agonists (salmeterol) and inhaled short- Questions Total 14
• Past Papers
acting β 2 -agonists when required. Although her Questions Percentage 85 %
• Random Questions
compliance is good, her symptoms are still not satisfactorily
• My Performance More
controlled. What is the next step in her therapy?
• Media Bank
Antibiotics
• New Multimedia
Salbutamol nebuliser
Online Extras
Oral steroids
Library
Oral cromoglycate
Community
Oral leukotriene-receptor antagonists Your answer
Help
PasTest Store

My Account
Occasional-relief bronchodilators are indicated in step 1
Profile in the treatment of chronic asthma, if relief is only
needed once daily and if there are no night-time
Newsletters symptoms or there is no impaired lung function. Regular
My Career and Exams inhaled corticosteroids are added in step 2. Regular
inhaled long-acting β 2 -agonists (salmeterol) are added Reference: Normal Values
Order History
in step 3. Oral leukotriene-receptor antagonists can also
Learning Goals be added in step 3 if the asthma is still not controlled. Haematology
Question Filters
Immunoglobulins
2017 Biochemistry
Security
Diabetes
Change Password
Sign Out Endocrinology

Blood gases

CSF

© 2011 PasTest Ltd | About Us | Contact Us | Help

file:///E|/Shakhawan/Respiratory%20S/229a.htm[3/13/2012 4:07:05 PM]


MyPasTest

Main Navigation
Question Browser: MRCP 1
Home
Subscriptions Question Browser Exam Builder Saved Exams
» MRCP 1 Session Progress
• Question Browser
Questions Correct 12
• Timed Test A 30-year-old asthmatic patient has the following drug
regimen: regular inhaled corticosteroids, regular inhaled Questions Incorrect 2
• Mock Exam
long-acting β 2 -agonists (salmeterol), oral leukotriene- Questions Total 14
• Past Papers
receptor antagonists and inhaled short-acting β 2 -agonists Questions Percentage 85 %
• Random Questions
when required. Although her compliance is good, her
• My Performance More
symptoms are still not satisfactorily controlled. What is the
• Media Bank next step in her therapy?
• New Multimedia
Oral steroids Reference: Normal Values
Online Extras
Antibiotics
Haematology
Library
Switch to nebuliser
Community Immunoglobulins
Oral cromoglycate
Help Biochemistry
Oral theophylline
PasTest Store Diabetes
2018
Endocrinology
My Account
Blood gases
Profile
CSF
Newsletters
My Career and Exams
Order History
Learning Goals
Question Filters

Security

Change Password
Sign Out

© 2011 PasTest Ltd | About Us | Contact Us | Help

file:///E|/Shakhawan/Respiratory%20S/230.htm[3/13/2012 4:07:07 PM]


MyPasTest

Main Navigation
Question Browser: MRCP 1
Home
Subscriptions Question Browser Exam Builder Saved Exams
» MRCP 1 Session Progress
• Question Browser
Questions Correct 1
• Timed Test A 30-year-old asthmatic patient has the following drug
regimen: regular inhaled corticosteroids, regular inhaled Questions Incorrect 0
• Mock Exam
long-acting β 2 -agonists (salmeterol), oral leukotriene- Questions Total 1
• Past Papers
receptor antagonists and inhaled short-acting β 2 -agonists Questions Percentage 100 %
• Random Questions
when required. Although her compliance is good, her
• My Performance More
symptoms are still not satisfactorily controlled. What is the
• Media Bank next step in her therapy?
• New Multimedia
Oral steroids
Online Extras
Antibiotics
Library
Switch to nebuliser
Community
Oral cromoglycate
Help
Oral theophylline Your answer
PasTest Store

My Account

Profile Theophylline is indicated in step 4 in the treatment of


chronic asthma when symptoms are still not controlled.
Newsletters Theophylline improves lung function and symptoms;
My Career and Exams however, side-effects can occur more commonly.
Order History Reference: Normal Values
Learning Goals 2018 Haematology
Question Filters
Immunoglobulins
Biochemistry
Security
Diabetes
Change Password
Sign Out Endocrinology

Blood gases

CSF

© 2011 PasTest Ltd | About Us | Contact Us | Help

file:///E|/Shakhawan/Respiratory%20S/230a.htm[3/13/2012 4:07:08 PM]


MyPasTest

Main Navigation
Question Browser: MRCP 1
Home
Subscriptions Question Browser Exam Builder Saved Exams
» MRCP 1 Session Progress
• Question Browser
Questions Correct 1
• Timed Test A 24-year-old woman is brought to casualty with thorax
injuries after a road traffic accident. Her chest X-ray shows Questions Incorrect 0
• Mock Exam
multiple rib fractures and a right-sided shadow suggestive Questions Total 1
• Past Papers of a haemothorax.
Questions Percentage 100 %
• Random Questions
What is the next step in her management? More
• My Performance
• Media Bank Blood transfusion
• New Multimedia Intubation and ventilation
Reference: Normal Values
Online Extras CT thorax
Haematology
Library Intercostal drain insertion
Community Surgical referral for thoracotomy Immunoglobulins
Help Biochemistry
2019
PasTest Store Diabetes
Endocrinology
My Account
Blood gases
Profile
CSF
Newsletters
My Career and Exams
Order History
Learning Goals
Question Filters

Security

Change Password
Sign Out

© 2011 PasTest Ltd | About Us | Contact Us | Help

file:///E|/Shakhawan/Respiratory%20S/231.htm[3/13/2012 4:07:10 PM]


MyPasTest

Main Navigation
Question Browser: MRCP 1
Home
Subscriptions Question Browser Exam Builder Saved Exams
» MRCP 1 Session Progress
• Question Browser
Questions Correct 2
• Timed Test A 24-year-old woman is brought to casualty with thorax
injuries after a road traffic accident. Her chest X-ray shows Questions Incorrect 0
• Mock Exam
multiple rib fractures and a right-sided shadow suggestive Questions Total 2
• Past Papers of a haemothorax.
Questions Percentage 100 %
• Random Questions
What is the next step in her management? More
• My Performance
• Media Bank Blood transfusion
• New Multimedia Intubation and ventilation
Online Extras CT thorax
Library Intercostal drain insertion Your answer
Community Surgical referral for thoracotomy
Help
PasTest Store

My Account A haemothorax is the result of bleeding into the


pleural space, and is arbitrarily diagnosed on the basis
Profile of having a haematocrit more than half that of
Newsletters peripheral blood. This distinguishes it from a blood-
stained effusion, which can be associated with a
My Career and Exams number of different pathological processes. The vast
Order History majority of haemothoraces are associated with Reference: Normal Values
penetrating or non-penetrating trauma, including
Learning Goals iatrogenic procedures such as central venous Haematology
Question Filters catheterisation. Bleeding usually results from
parenchymal laceration or damage to intercostal Immunoglobulins
vessels. A pneumothorax is present in a high Biochemistry
Security proportion of patients.
Diabetes
Change Password The treatment of choice is to insert a large intercostal
drain (28–32 French), allowing evacuation of blood Endocrinology
Sign Out and reducing the incidence of a subsequent
fibrothorax. If this reveals continued bleeding, a Blood gases
thoracotomy may be required. Surgery is not
indicated simply to remove any residual blood clots CSF
since in a majority of patients there is spontaneous
lysis with no residual damage.

2019

© 2011 PasTest Ltd | About Us | Contact Us | Help

file:///E|/Shakhawan/Respiratory%20S/231a.htm[3/13/2012 4:07:11 PM]


MyPasTest

Main Navigation
Question Browser: MRCP 1
Home
Subscriptions Question Browser Exam Builder Saved Exams
» MRCP 1 Session Progress
• Question Browser
Questions Correct 2
• Timed Test A 55-year-old man presents with increasing shortness of
breath. He has been working in the sandblasting industry Questions Incorrect 0
• Mock Exam
and exposed to quartz particles. What is the most likely Questions Total 2
• Past Papers diagnosis?
Questions Percentage 100 %
• Random Questions
• My Performance Asbestosis More
• Media Bank Sarcoidosis

• New Multimedia Silicosis


Reference: Normal Values
Online Extras Eosinophilic pneumonitis
Asthma Haematology
Library
Community Immunoglobulins
2020
Help Biochemistry
PasTest Store Diabetes
Endocrinology
My Account
Blood gases
Profile
CSF
Newsletters
My Career and Exams
Order History
Learning Goals
Question Filters

Security

Change Password
Sign Out

© 2011 PasTest Ltd | About Us | Contact Us | Help

file:///E|/Shakhawan/Respiratory%20S/232.htm[3/13/2012 4:07:13 PM]


MyPasTest

Main Navigation
Question Browser: MRCP 1
Home
Subscriptions Question Browser Exam Builder Saved Exams
» MRCP 1 Session Progress
• Question Browser
Questions Correct 3
• Timed Test A 55-year-old man presents with increasing shortness of
breath. He has been working in the sandblasting industry Questions Incorrect 0
• Mock Exam
and exposed to quartz particles. What is the most likely Questions Total 3
• Past Papers diagnosis?
Questions Percentage 100 %
• Random Questions
• My Performance Asbestosis More
• Media Bank Sarcoidosis

• New Multimedia Silicosis Your answer

Online Extras Eosinophilic pneumonitis

Library Asthma

Community
Help
PasTest Store
Silicosis is a fibrotic disease of the lungs due to
inhalation of crystalline silicon dioxide, usually in the
My Account form of quartz. Silicosis may affect anyone involved in
quarrying, carving, mining, tunnelling, grinding or
Profile sandblasting, if the dust generated contains quartz.
Newsletters Between 50 and 60 cases are diagnosed in the UK
each year, generally in people involved in the
My Career and Exams production of slate or granite, among miners cutting
Order History through rock and in fettlers in foundries. Crystalline Reference: Normal Values
silica is present in the earth’s crust usually as quartz,
Learning Goals Haematology
although other forms such as crystobalite and
Question Filters tridymite occur occasionally. All are extremely toxic to
Immunoglobulins
macrophages. Quartz seems to be most toxic when
freshly fractured, suggesting that its surface Biochemistry
Security properties are important in toxicity.
Diabetes
Change Password Silicosis presents a spectrum of clinical appearances
depending on the circumstances in which it is Endocrinology
Sign Out
contracted. The most severe, acute silicosis, may be
acquired after very heavy exposure over just months, Blood gases
such as sandblasting without respiratory protection.
CSF
These patients become intensely breathless and die
within months. The X-ray shows appearances
resembling pulmonary oedema. Less heavy exposure
causes progressively less dramatic symptoms, ranging
from a progressive upper lobe fibrosis with slowly
increasing exertional dyspnoea over several years
(accelerated silicosis) to radiographic nodular change
similar to coal-worker’s pneumoconiosis unassociated
with any symptoms or physical signs. The latter type
of silicosis is the most common, and is usually
associated with exposure to dust containing 10–30%
silica over a prolonged period. Simple nodular silicosis
differs from coal-worker’s pneumoconiosis in that the
lesions tend to be larger (3–5 mm) and that it is
progressive even after dust exposure ceases. Lesions
increase in size and become more profuse. Moreover,
extensive simple silicosis may be associated with
some restriction of lung volumes. Accelerated silicosis
and progressive massive fibrosis cause lung restriction
and lead to cor pulmonale and cardiorespiratory
failure.

2020

file:///E|/Shakhawan/Respiratory%20S/232a.htm[3/13/2012 4:07:14 PM]


MyPasTest

Main Navigation
Question Browser: MRCP 1
Home
Subscriptions Question Browser Exam Builder Saved Exams
» MRCP 1 Session Progress
• Question Browser
Questions Correct 3
• Timed Test A 17-year-old boy has been complaining of shortness of
breath for the last 2 days. On examination bronchial Questions Incorrect 0
• Mock Exam
breathing is heard over the right lower lobe. What is the Questions Total 3
• Past Papers most likely diagnosis for this clinical finding?
Questions Percentage 100 %
• Random Questions
• My Performance Pneumothorax More
• Media Bank Pneumonia

• New Multimedia Asthma


Reference: Normal Values
Online Extras Emphysema
Chronic obstructive pulmonary disease Haematology
Library
Community Immunoglobulins
2021
Help Biochemistry
PasTest Store Diabetes
Endocrinology
My Account
Blood gases
Profile
CSF
Newsletters
My Career and Exams
Order History
Learning Goals
Question Filters

Security

Change Password
Sign Out

© 2011 PasTest Ltd | About Us | Contact Us | Help

file:///E|/Shakhawan/Respiratory%20S/233.htm[3/13/2012 4:07:16 PM]


MyPasTest

Main Navigation
Question Browser: MRCP 1
Home
Subscriptions Question Browser Exam Builder Saved Exams
» MRCP 1 Session Progress
• Question Browser
Questions Correct 4
• Timed Test A 17-year-old boy has been complaining of shortness of
breath for the last 2 days. On examination bronchial Questions Incorrect 0
• Mock Exam
breathing is heard over the right lower lobe. What is the Questions Total 4
• Past Papers most likely diagnosis for this clinical finding?
Questions Percentage 100 %
• Random Questions
• My Performance Pneumothorax More
• Media Bank Pneumonia Your answer

• New Multimedia Asthma

Online Extras Emphysema

Library Chronic obstructive pulmonary disease

Community
Help
PasTest Store
Bronchial breathing is heard over an airless lung, such
as in consolidation, atelectasis or dense fibrosis. There
My Account is some resemblance to the sounds heard over the
normal trachea, but, by comparison with normal
Profile breath sounds, bronchial breathing is higher in pitch
Newsletters and more blowing in quality. It does not have to be
loud. Bronchial breath sounds are classically heard
My Career and Exams throughout both inspiration and expiration. Very quiet
Order History breath sounds are heard over hyperinflated lungs, as Reference: Normal Values
in emphysema, or when breath sounds are prevented
Learning Goals Haematology
from reaching the chest wall by a layer of air, fluid or
Question Filters fibrosis.
Immunoglobulins
The classic presentation of pneumonia is of a cough
and fever with the variable presence of sputum Biochemistry
Security
production, dyspnoea and pleurisy. Most patients Diabetes
Change Password have constitutional symptoms such as malaise,
fatigue and asthenia, and many also have Endocrinology
Sign Out
gastrointestinal symptoms. Although patients with
pneumonia usually possess these characteristic clinical Blood gases
features, there can be major differences in
CSF
presentation based on the host and the aetiological
agent.

2021

© 2011 PasTest Ltd | About Us | Contact Us | Help

file:///E|/Shakhawan/Respiratory%20S/233a.htm[3/13/2012 4:07:17 PM]


MyPasTest

Main Navigation
Question Browser: MRCP 1
Home
Subscriptions Question Browser Exam Builder Saved Exams
» MRCP 1 Session Progress
• Question Browser
Questions Correct 4
• Timed Test A young man has been in a car accident and has sustained
a trauma to his thorax. He has a tear in his right main Questions Incorrect 0
• Mock Exam
bronchus without any obvious fractures. Questions Total 4
• Past Papers
What is the most likely finding? Questions Percentage 100 %
• Random Questions
• My Performance More
Pulmonary oedema
• Media Bank
Pulsus paradoxus
• New Multimedia
Atelectasis Reference: Normal Values
Online Extras
Expiratory stridor
Haematology
Library
Wheezing
Community Immunoglobulins
Help 2022 Biochemistry
PasTest Store Diabetes
Endocrinology
My Account
Blood gases
Profile
CSF
Newsletters
My Career and Exams
Order History
Learning Goals
Question Filters

Security

Change Password
Sign Out

© 2011 PasTest Ltd | About Us | Contact Us | Help

file:///E|/Shakhawan/Respiratory%20S/234.htm[3/13/2012 4:07:19 PM]


MyPasTest

Main Navigation
Question Browser: MRCP 1
Home
Subscriptions Question Browser Exam Builder Saved Exams
» MRCP 1 Session Progress
• Question Browser
Questions Correct 5
• Timed Test A young man has been in a car accident and has sustained
Questions Incorrect 0
• Mock Exam a trauma to his thorax. He has a tear in his right main
bronchus without any obvious fractures. Questions Total 5
• Past Papers
Questions Percentage 100 %
• Random Questions What is the most likely finding?
• My Performance More
Pulmonary oedema
• Media Bank
Pulsus paradoxus
• New Multimedia
Atelectasis Your answer
Online Extras
Expiratory stridor
Library
Wheezing
Community
Help
PasTest Store
Atelectasis is when part of, or all of, one lung collapses,
My Account preventing normal oxygen absorption. Breath sounds
are quietened over areas of atelectasis. During
Profile expiration normal breath sounds rapidly fade out,
probably due to the decreasing air-flow rate. Atelectasis
Newsletters can be caused by:
My Career and Exams
an obstruction of an air passage by a thick mucus
Order History Reference: Normal Values
plug from an infection or other disease, such as
Learning Goals cystic fibrosis Haematology
tumours in the air passages
Question Filters
tumours or blood vessles outside the air Immunoglobulins
passages, causing pressure on the airways
inhaled objects, such as peanuts or small toys Biochemistry
Security
prolonged chest or abdomincal surgery under
Diabetes
Change Password general anaesthetic
chest injury or fractured ribs, and Endocrinology
Sign Out a penetrating wound.
Blood gases

CSF
2022

© 2011 PasTest Ltd | About Us | Contact Us | Help

file:///E|/Shakhawan/Respiratory%20S/234a.htm[3/13/2012 4:07:20 PM]


MyPasTest

Main Navigation
Question Browser: MRCP 1
Home
Subscriptions Question Browser Exam Builder Saved Exams
» MRCP 1 Session Progress
• Question Browser
Questions Correct 5
• Timed Test Within a few weeks several patients in a geriatric ward
contracted a legionella pneumonia which has been Questions Incorrect 0
• Mock Exam
successfully treated with macrolide antibiotics. Questions Total 5
• Past Papers
What was the most likely source of those nosocomial Questions Percentage 100 %
• Random Questions
infections? More
• My Performance
• Media Bank Droplet transmission from staff
• New Multimedia Insufficient hand disinfection
Reference: Normal Values
Online Extras Contaminated instruments
Haematology
Library Contaminated warm-water source
Community Contaminated medication boxes Immunoglobulins
Help Biochemistry
2023
PasTest Store Diabetes
Endocrinology
My Account
Blood gases
Profile
CSF
Newsletters
My Career and Exams
Order History
Learning Goals
Question Filters

Security

Change Password
Sign Out

© 2011 PasTest Ltd | About Us | Contact Us | Help

file:///E|/Shakhawan/Respiratory%20S/235.htm[3/13/2012 4:07:22 PM]


MyPasTest

Main Navigation
Question Browser: MRCP 1
Home
Subscriptions Question Browser Exam Builder Saved Exams
» MRCP 1 Session Progress
• Question Browser
Questions Correct 6
• Timed Test Within a few weeks several patients in a geriatric ward
contracted a legionella pneumonia which has been Questions Incorrect 0
• Mock Exam
successfully treated with macrolide antibiotics. Questions Total 6
• Past Papers
What was the most likely source of those nosocomial Questions Percentage 100 %
• Random Questions
infections? More
• My Performance
• Media Bank Droplet transmission from staff
• New Multimedia Insufficient hand disinfection
Online Extras Contaminated instruments
Library Contaminated warm-water source Your answer
Community Contaminated medication boxes
Help
PasTest Store

My Account Legionellae have been found in small numbers in water


distribution systems, through which they can colonise
Profile man-made habitats, again as part of the biofilm from
which they are shed into the water. Factors that
Newsletters encourage colonisation and multiplication are
My Career and Exams temperature (20–45 °C) and stagnation. The most
common sites in buildings in which legionellae have Reference: Normal Values
Order History
been found are hot-water calorifiers and storage tanks.
Learning Goals Piped water, especially hot water from the calorifiers in Haematology
Question Filters large buildings and industrial complexes with long runs
of pipework, is a potential source of infection. Other Immunoglobulins
well-recognised sources include:
Biochemistry
Security
recirculating water in air-conditioning and cooling
Diabetes
Change Password systems
whirlpool spas and other warm-water baths Endocrinology
Sign Out decorative fountains, and
nebulisers and humidifier reservoirs of hospital Blood gases
netilation machines if topped up with
contaminated tap water. CSF

Dissemination of infection is by contaminated water


droplets (aerosol), which are inhaled. To cause infection
the droplets must be of a size (less than 5 mm in
diameter) that can reach the alveoli of the lungs. Taps
and shower-heads produce very localised aerosols,
whereas the water droplets (drift) contained in the
airstream released from a cooling tower may be carried
a considerable distance and expose a greater number of
people to risk.

2023

© 2011 PasTest Ltd | About Us | Contact Us | Help

file:///E|/Shakhawan/Respiratory%20S/235a.htm[3/13/2012 4:07:23 PM]


MyPasTest

Main Navigation
Question Browser: MRCP 1
Home
Subscriptions Question Browser Exam Builder Saved Exams
» MRCP 1 Session Progress
• Question Browser
Questions Correct 6
• Timed Test A 50-year-old woman patient presents with increasing
dyspnoea. She is obese, smokes and takes oestrogens for Questions Incorrect 0
• Mock Exam
menopausal symptoms. On examination you find clinical, Questions Total 6
• Past Papers electrocardial and radiological findings of a right-sided
Questions Percentage 100 %
• Random Questions heart failure without signs of left ventricular failure. What is
the most likely cause for the cor pulmonale? More
• My Performance
• Media Bank Asthma
• New Multimedia Recurrent pneumonias Reference: Normal Values
Online Extras Recurrent small pulmonary embolisms
Haematology
Library Bronchiectasis
Community Carcinoma of the lung Immunoglobulins
Help Biochemistry
2024
PasTest Store Diabetes
Endocrinology
My Account
Blood gases
Profile
CSF
Newsletters
My Career and Exams
Order History
Learning Goals
Question Filters

Security

Change Password
Sign Out

© 2011 PasTest Ltd | About Us | Contact Us | Help

file:///E|/Shakhawan/Respiratory%20S/236.htm[3/13/2012 4:07:25 PM]


MyPasTest

Main Navigation
Question Browser: MRCP 1
Home
Subscriptions Question Browser Exam Builder Saved Exams
» MRCP 1 Session Progress
• Question Browser
Questions Correct 7
• Timed Test A 50-year-old woman patient presents with increasing
dyspnoea. She is obese, smokes and takes oestrogens for Questions Incorrect 0
• Mock Exam
menopausal symptoms. On examination you find clinical, Questions Total 7
• Past Papers electrocardial and radiological findings of a right-sided
Questions Percentage 100 %
• Random Questions heart failure without signs of left ventricular failure. What is
the most likely cause for the cor pulmonale? More
• My Performance
• Media Bank Asthma
• New Multimedia Recurrent pneumonias
Online Extras Recurrent small pulmonary embolisms Your answer
Library Bronchiectasis
Community Carcinoma of the lung
Help
PasTest Store

My Account Pulmonary hypertension develops as the result of a


critical reduction of cross-sectional area for blood
Profile flow. The relatively high flow through non-occluded
Newsletters vessels may cause secondary hypertensive changes in
the resistive or precapillary vessels. Patients with
My Career and Exams mean pulmonary artery pressures over 30 mmHg
Order History have a 5-year survival rate of 30%, but this rate falls Reference: Normal Values
to 10% if the mean pulmonary artery pressure is over
Learning Goals Haematology
50 mmHg.
Question Filters
Most patients do not have an obvious history of Immunoglobulins
venous thrombosis or pulmonary embolism. Dyspnoea
is present in virtually all, but in the early stages may Biochemistry
Security
occur only on exertion. Compensatory right Diabetes
Change Password ventricular hypertrophy develops, and there may be a
period of months to years when symptoms remain Endocrinology
Sign Out
stable, but ultimately right ventricular function
deteriorates. Symptoms then include worsening Blood gases
dyspnoea, fatigue, presyncope, syncope (rarely),
CSF
pleuritic pain, angina-like pain, abdominal swelling
and peripheral oedema. Signs of respiratory and right
ventricular failure develop, with cyanosis, grossly
elevated jugular venous pulse, palpable right
ventricular heave, right ventricular gallop, loud P2,
hepatomegaly, ascites and peripheral oedema. Bruits
may be heard over the pulmonary arteries.
Ventilation–perfusion lung scans typically show one or
more mismatched segmental or larger perfusion
defects, with most patients having several bilateral
mismatched perfusion defects. Pulmonary
angiography is the most definitive diagnostic test. It
shows narrowed segmental pulmonary arteries,
sometimes accompanied by post-stenotic dilatation,
irregularity of the intima, luminal narrowing of the
central arteries and oddly shaped vessels. Pulmonary
fibre-optic angioscopy is useful to define surgical
accessibility.

2024

file:///E|/Shakhawan/Respiratory%20S/236a.htm[3/13/2012 4:07:26 PM]


MyPasTest

Main Navigation
Question Browser: MRCP 1
Home
Subscriptions Question Browser Exam Builder Saved Exams
» MRCP 1 Session Progress
• Question Browser
Questions Correct 7
• Timed Test A 50-year-old man has been referred by his GP because of
a long-standing history of persistent cough productive of Questions Incorrect 0
• Mock Exam
mucopurulent sputum. The patient was treated several Questions Total 7
• Past Papers times for recurrent chest infections. What is the most
Questions Percentage 100 %
• Random Questions appropriate diagnostic step to confirm the diagnosis?
• My Performance More
Bronchoscopy
• Media Bank
Chest X-ray
• New Multimedia
Ventilation–perfusion scan Reference: Normal Values
Online Extras
High-resolution CT Haematology
Library
Spirometry
Community Immunoglobulins
Help 2025 Biochemistry
PasTest Store Diabetes
Endocrinology
My Account
Blood gases
Profile
CSF
Newsletters
My Career and Exams
Order History
Learning Goals
Question Filters

Security

Change Password
Sign Out

© 2011 PasTest Ltd | About Us | Contact Us | Help

file:///E|/Shakhawan/Respiratory%20S/237.htm[3/13/2012 4:07:28 PM]


MyPasTest

Main Navigation
Question Browser: MRCP 1
Home
Subscriptions Question Browser Exam Builder Saved Exams
» MRCP 1 Session Progress
• Question Browser
Questions Correct 8
• Timed Test A 50-year-old man has been referred by his GP because of
a long-standing history of persistent cough productive of Questions Incorrect 0
• Mock Exam
mucopurulent sputum. The patient was treated several Questions Total 8
• Past Papers times for recurrent chest infections. What is the most
Questions Percentage 100 %
• Random Questions appropriate diagnostic step to confirm the diagnosis?
• My Performance More
Bronchoscopy
• Media Bank
Chest X-ray
• New Multimedia
Ventilation–perfusion scan
Online Extras
High-resolution CT Your answer
Library
Spirometry
Community
Help
PasTest Store

The ‘gold standard’ for the diagnosis of bronchiectasis


My Account is thin-section, high-resolution CT (HRCT) of the
chest, which has replaced the more invasive
Profile investigation of bronchography.
Newsletters The diagnostic criteria for bronchiectasis on HRCT
My Career and Exams depend on finding both dilatation and thickening of
the affected bronchi, dilatation being present if the
Order History Reference: Normal Values
internal diameter of the bronchus is greater than the
Learning Goals diameter of its accompanying pulmonary artery. The Haematology
Question Filters classic appearance of a cross-section of a thick-walled
dilated bronchus next to the accompanying pulmonary Immunoglobulins
artery is the ‘signet ring’ sign. Bronchial dilatation is
also recognised when airways are seen in longitudinal Biochemistry
Security
section on CT and there is a failure of tapering as the Diabetes
Change Password bronchus courses towards the periphery.
Sign Out The chest X-ray may be normal in at least 50% of Endocrinology
patients with CT or bronchographic evidence of Blood gases
bronchiectasis. If it is abnormal the findings relate to
thickened and dilated bronchi, which produce tramline CSF
opacities and ring shadows. Retained mucus may be
seen as tubular opacities, and there may be
associated volume loss of the affected lobe.

2025

© 2011 PasTest Ltd | About Us | Contact Us | Help

file:///E|/Shakhawan/Respiratory%20S/237a.htm[3/13/2012 4:07:29 PM]


MyPasTest

Main Navigation
Question Browser: MRCP 1
Home
Subscriptions Question Browser Exam Builder Saved Exams
» MRCP 1 Session Progress
• Question Browser
Questions Correct 8
• Timed Test A 35-year-old woman has been complaining of a 3-month
history of increasing shortness of breath. The clinical Questions Incorrect 0
• Mock Exam
examination is unremarkable, the chest X-ray has a diffuse Questions Total 8
• Past Papers reticulonodular appearance and there is hilar
Questions Percentage 100 %
• Random Questions lymphadenopathy. A transbronchial biopsy shows widened
alveolar septae and granulomas without necrosis. What is More
• My Performance
the most likely diagnosis?
• Media Bank
• New Multimedia Tuberculosis
Reference: Normal Values
Online Extras Pneumonia due to fungi
Mycoplasma pneumonia Haematology
Library
Community Sarcoidosis Immunoglobulins
Help Bronchiectasis Biochemistry
PasTest Store 2026 Diabetes
Endocrinology
My Account
Blood gases
Profile
CSF
Newsletters
My Career and Exams
Order History
Learning Goals
Question Filters

Security

Change Password
Sign Out

© 2011 PasTest Ltd | About Us | Contact Us | Help

file:///E|/Shakhawan/Respiratory%20S/238.htm[3/13/2012 4:07:31 PM]


MyPasTest

Main Navigation
Question Browser: MRCP 1
Home
Subscriptions Question Browser Exam Builder Saved Exams
» MRCP 1 Session Progress
• Question Browser
Questions Correct 9
• Timed Test A 35-year-old woman has been complaining of a 3-month
history of increasing shortness of breath. The clinical Questions Incorrect 0
• Mock Exam
examination is unremarkable, the chest X-ray has a diffuse Questions Total 9
• Past Papers reticulonodular appearance and there is hilar
Questions Percentage 100 %
• Random Questions lymphadenopathy. A transbronchial biopsy shows widened
alveolar septae and granulomas without necrosis. What is More
• My Performance
the most likely diagnosis?
• Media Bank
• New Multimedia Tuberculosis
Online Extras Pneumonia due to fungi
Library Mycoplasma pneumonia
Community Sarcoidosis Your answer
Help Bronchiectasis
PasTest Store

My Account
Respiratory involvement has been described in more
Profile than 90% of patients with sarcoidosis. The lung
involvement includes both the lymph nodes and the
Newsletters
lung parenchyma. Scadding and Wurm independently
My Career and Exams described four stages of the chest X-ray:
Order History Reference: Normal Values
stage 1 – hilar adenopathy alone
Learning Goals stage 2 – adenopathy and parenchymal disease Haematology
Question Filters stage 3 – parenchymal disease alone
stage 4 – fibrosis. Immunoglobulins
Biochemistry
Security The interstitial disease usually has a diffuse
reticulonodular appearance, but confluent patches of Diabetes
Change Password disease (alveolar sarcoidosis) have been described.
Sign Out Fibrotic changes due to sarcoidosis are usually Endocrinology
present in the upper lobe, with retraction. The staging
system has proved useful in standardising reports of Blood gases
the pulmonary level of involvement. It has also CSF
proved a useful prognostic measure. Patients with
stage 1 disease have a 90% rate of resolution within
2–3 years, while patients with stage 3 disease
possess only a 30% chance of resolution. However, it
does not predict the degree of extrapulmonary
disease.
The non-caseating granuloma is the characteristic
pathological feature of sarcoidosis. The centre of the
granuloma includes macrophages and giant cells,
which are of the Langerhans type and can contain
over 10 nuclei. This core of cells is surrounded by two
rings of lymphocytes: the larger inner component of
CD4 helper cells, while the outer ring can comprise
CD8 suppressor cells. The granulomas tend to be well
formed, and in lung biopsies are often well
demarcated from normal tissue. The central area will
occasionally contain a Schaumann body, formed of
crystallised material (calcium phosphate). This is
different in appearance from foreign bodies or
caseation, which can be seen in other granulomatous
diseases. Occasionally the granuloma will have a
necrotic area, but the majority of the granulomas do
not.

file:///E|/Shakhawan/Respiratory%20S/238a.htm[3/13/2012 4:07:32 PM]


MyPasTest

Main Navigation
Question Browser: MRCP 1
Home
Subscriptions Question Browser Exam Builder Saved Exams
» MRCP 1 Session Progress
• Question Browser
Questions Correct 9
• Timed Test A 50-year-old man patient was referred by his GP because
of a long-standing history of persistent cough productive of Questions Incorrect 0
• Mock Exam
mucopurulent sputum. He also noticed increasing shortness Questions Total 9
• Past Papers of breath. The patient has been treated several times for
Questions Percentage 100 %
• Random Questions recurrent chest infections. What is the most likely
diagnosis? More
• My Performance
• Media Bank Lung cancer
• New Multimedia Bronchiectasis Reference: Normal Values
Online Extras Chronic cardiac failure
Haematology
Library Extrinsic allergic alveolitis
Community Asthma Immunoglobulins
Help Biochemistry
2027
PasTest Store Diabetes
Endocrinology
My Account
Blood gases
Profile
CSF
Newsletters
My Career and Exams
Order History
Learning Goals
Question Filters

Security

Change Password
Sign Out

© 2011 PasTest Ltd | About Us | Contact Us | Help

file:///E|/Shakhawan/Respiratory%20S/239.htm[3/13/2012 4:07:34 PM]


MyPasTest

Main Navigation
Question Browser: MRCP 1
Home
Subscriptions Question Browser Exam Builder Saved Exams
» MRCP 1 Session Progress
• Question Browser
Questions Correct 10
• Timed Test A 50-year-old man patient was referred by his GP because
of a long-standing history of persistent cough productive of Questions Incorrect 0
• Mock Exam
mucopurulent sputum. He also noticed increasing shortness Questions Total 10
• Past Papers of breath. The patient has been treated several times for
Questions Percentage 100 %
• Random Questions recurrent chest infections. What is the most likely
diagnosis? More
• My Performance
• Media Bank Lung cancer
• New Multimedia Bronchiectasis Your answer
Online Extras Chronic cardiac failure
Library Extrinsic allergic alveolitis
Community Asthma
Help
PasTest Store

My Account Bronchiectasis should be suspected when there is a


history of persistent cough productive of
Profile mucopurulent or purulent sputum throughout the
Newsletters year. Patients have frequently been treated for
recurrent chest infections and labelled as bronchitic,
My Career and Exams often despite the absence of a history of smoking.
Order History Patients may produce mucoid sputum early in their Reference: Normal Values
disease, developing purulent sputum when they suffer
Learning Goals Haematology
an exacerbation associated with a viral upper
Question Filters respiratory tract infection. Such exacerbations may be
Immunoglobulins
associated with pleuritic chest pain, haemoptysis,
fever and sometimes wheeze. Biochemistry
Security
Those presenting as adults often recall a chesty cough Diabetes
Change Password or wheezy bronchitis associated with upper respiratory
tract infections in childhood, followed by complete Endocrinology
Sign Out
resolution of symptoms in the teens and early adult
life before these return after a viral trigger. Upper Blood gases
respiratory tract symptoms such as nasal drip are
CSF
common, and in about 30% of cases there is a history
of chronic sinusitis. Patients with bronchiectasis also
suffer from undue tiredness, which many find more
troublesome than the productive cough.

2027

© 2011 PasTest Ltd | About Us | Contact Us | Help

file:///E|/Shakhawan/Respiratory%20S/239a.htm[3/13/2012 4:07:35 PM]


MyPasTest

Main Navigation
Question Browser: MRCP 1
Home
Subscriptions Question Browser Exam Builder Saved Exams
» MRCP 1 Session Progress
• Question Browser
Questions Correct 10
• Timed Test A 45-year-old woman presents with progressive fibrosing
alveolitis. When performing lung function tests, which Questions Incorrect 0
• Mock Exam
parameter would be expected to be normal? Questions Total 10
• Past Papers
Questions Percentage 100 %
• Random Questions Total lung capacity
• My Performance Forced expiratory volume in 1 second/forced vital More
• Media Bank capacity (FEV 1 /FVC) ratio

• New Multimedia Vital capacity


Reference: Normal Values
Online Extras Carbon monoxide transfer factor
Forced vital capacity Haematology
Library
Community Immunoglobulins
2028
Help Biochemistry
PasTest Store Diabetes
Endocrinology
My Account
Blood gases
Profile
CSF
Newsletters
My Career and Exams
Order History
Learning Goals
Question Filters

Security

Change Password
Sign Out

© 2011 PasTest Ltd | About Us | Contact Us | Help

file:///E|/Shakhawan/Respiratory%20S/240.htm[3/13/2012 4:07:37 PM]


MyPasTest

Main Navigation
Question Browser: MRCP 1
Home
Subscriptions Question Browser Exam Builder Saved Exams
» MRCP 1 Session Progress
• Question Browser
Questions Correct 11
• Timed Test A 45-year-old woman presents with progressive fibrosing
Questions Incorrect 0
• Mock Exam alveolitis. When performing lung function tests, which
parameter would be expected to be normal? Questions Total 11
• Past Papers
Questions Percentage 100 %
• Random Questions Total lung capacity
• My Performance Forced expiratory volume in 1 Your answer More
• Media Bank second/forced vital capacity (FEV 1 /FVC)
ratio
• New Multimedia
Vital capacity
Online Extras
Carbon monoxide transfer factor
Library
Forced vital capacity
Community
Help
PasTest Store

Fibrosing alveolitis is characterised by a restrictive


My Account ventilatory defect of mechanical function resulting in
reduced pulmonary compliance, vital capacity and
Profile
total lung capacity. Residual volume is usually
Newsletters decreased unless there is coincident airflow
My Career and Exams obstruction due to cigarette smoking, and lung recoil
pressure is increased.
Order History Reference: Normal Values
Carbon monoxide transfer factor (DLco, a measure of
Learning Goals diffusion capacity) is reduced and may be the only Haematology
Question Filters abnormality in early disease. In most patients the gas
transfer measurement adjusted for alveolar volume Immunoglobulins
(Kco) is also reduced, but less so than DLco,
Biochemistry
Security indicating that the capacity to exchange gas is
impaired in a lung that has not been destroyed. If Diabetes
Change Password there is significant coexisting emphysema, lung
Sign Out volumes will be well preserved in the face of a Endocrinology
disproportionately depressed gas transfer
measurement in both DLco and Kco. Gas transfer is Blood gases
reduced by both the emphysematous and the CSF
fibrosing processes, whereas lung volumes will tend to
be increased by emphysema but reduced by fibrosis.
These two opposing influences result in relatively
normal-sized lungs radiographically and
physiologically.
The FEV1 /FVC ratio is < 70% in obstructive lung
disease and > 70% in restrictive lung disease.

2028

© 2011 PasTest Ltd | About Us | Contact Us | Help

file:///E|/Shakhawan/Respiratory%20S/240a.htm[3/13/2012 4:07:38 PM]


MyPasTest

Main Navigation
Question Browser: MRCP 1
Home
Subscriptions Question Browser Exam Builder Saved Exams
» MRCP 1 Session Progress
• Question Browser
Questions Correct 11
• Timed Test An 18-year-old young woman presents to casualty with
acute onset of shortness of breath, dizziness, tingling in Questions Incorrect 0
• Mock Exam
both hands and chest pain. She has presented to casualty Questions Total 11
• Past Papers with the same symptoms three times in the last month.
Questions Percentage 100 %
• Random Questions The clinical examination and chest X-ray are both normal.
What is the most likely diagnosis? More
• My Performance
• Media Bank Tuberculosis
• New Multimedia Chlamydia pneumonia Reference: Normal Values
Online Extras Mycoplasma pneumonia
Haematology
Library Hyperventilation syndrome
Community Pulmonary embolism Immunoglobulins
Help Biochemistry
2029
PasTest Store Diabetes
Endocrinology
My Account
Blood gases
Profile
CSF
Newsletters
My Career and Exams
Order History
Learning Goals
Question Filters

Security

Change Password
Sign Out

© 2011 PasTest Ltd | About Us | Contact Us | Help

file:///E|/Shakhawan/Respiratory%20S/241.htm[3/13/2012 4:07:40 PM]


MyPasTest

Main Navigation
Question Browser: MRCP 1
Home
Subscriptions Question Browser Exam Builder Saved Exams
» MRCP 1 Session Progress
• Question Browser
Questions Correct 12
• Timed Test An 18-year-old young woman presents to casualty with
acute onset of shortness of breath, dizziness, tingling in Questions Incorrect 0
• Mock Exam
both hands and chest pain. She has presented to casualty Questions Total 12
• Past Papers with the same symptoms three times in the last month.
Questions Percentage 100 %
• Random Questions The clinical examination and chest X-ray are both normal.
What is the most likely diagnosis? More
• My Performance
• Media Bank Tuberculosis
• New Multimedia Chlamydia pneumonia
Online Extras Mycoplasma pneumonia
Library Hyperventilation syndrome Your answer
Community Pulmonary embolism
Help
PasTest Store

My Account Psychogenic breathlessness is diagnosed by exclusion,


although there may be clues in the history and
Profile examination. The sighing and irregular breathing will
Newsletters be readily noticeable to a keen observer. Associated
complaints directly related to the hyperventilation are
My Career and Exams paraesthesia in the hands (and, perhaps, feet),
Order History dizziness and collapse. Non-specific features, such as Reference: Normal Values
fatigue, insomnia, weakness or chest pains, may all
Learning Goals Haematology
be part of the syndrome.
Question Filters
Depression and anxiety may both be aspects of the Immunoglobulins
underlying psychiatric state. By definition, in pure
psychogenic breathlessness, the chest X-ray and lung Biochemistry
Security
function are normal. However, some patients may Diabetes
Change Password develop breathlessness because they have been told
they have a shadow on a lung, or through anxiety Endocrinology
Sign Out
exhibit a degree of breathlessness disproportionate to
a mild functional abnormality. The latter patients tend Blood gases
to have an obsessional personality or may be looking
CSF
for compensation for supposed lung damage due to
injury or occupation.

2029

© 2011 PasTest Ltd | About Us | Contact Us | Help

file:///E|/Shakhawan/Respiratory%20S/241a.htm[3/13/2012 4:07:41 PM]


MyPasTest

Main Navigation
Question Browser: MRCP 1
Home
Subscriptions Question Browser Exam Builder Saved Exams
» MRCP 1 Session Progress
• Question Browser
Questions Correct 12
• Timed Test A 40-year-old man presents with a 2-month history of
cough and breathlessness. He has also noted haemoptysis, Questions Incorrect 0
• Mock Exam
which he says has gradually worsened. On examination he Questions Total 12
• Past Papers has bilateral basal crepitations. His chest X-ray shows
Questions Percentage 100 %
• Random Questions diffuse shadowing. He has moderate renal failure. What is
the most likely diagnosis? More
• My Performance
• Media Bank Tuberculosis
• New Multimedia Goodpasture’s disease Reference: Normal Values
Online Extras Bronchial carcinoma
Haematology
Library Legionella pneumonia
Community Pulmonary embolism Immunoglobulins
Help Biochemistry
2030
PasTest Store Diabetes
Endocrinology
My Account
Blood gases
Profile
CSF
Newsletters
My Career and Exams
Order History
Learning Goals
Question Filters

Security

Change Password
Sign Out

© 2011 PasTest Ltd | About Us | Contact Us | Help

file:///E|/Shakhawan/Respiratory%20S/242.htm[3/13/2012 4:07:43 PM]


MyPasTest

Main Navigation
Question Browser: MRCP 1
Home
Subscriptions Question Browser Exam Builder Saved Exams
» MRCP 1 Session Progress
• Question Browser
Questions Correct 13
• Timed Test A 40-year-old man presents with a 2-month history of
Questions Incorrect 0
• Mock Exam cough and breathlessness. He has also noted haemoptysis,
which he says has gradually worsened. On examination he Questions Total 13
• Past Papers has bilateral basal crepitations. His chest X-ray shows Questions Percentage 100 %
• Random Questions diffuse shadowing. He has moderate renal failure. What is
the most likely diagnosis? More
• My Performance
• Media Bank Tuberculosis
• New Multimedia Goodpasture’s disease Your answer
Online Extras Bronchial carcinoma
Library Legionella pneumonia
Community Pulmonary embolism
Help
PasTest Store

My Account Goodpasture described a man with renal failure,


glomerulonephritis and pulmonary haemorrhage.
Profile Goodpasture’s disease consists of diffuse pulmonary
Newsletters haemorrhage and glomerulonephritis with linear
deposition of antibody (90% of which are directed
My Career and Exams against the a-3 chain of type-IV collagen) along the
Order History glomerular basement membrane. In practice, Reference: Normal Values
glomerulonephritis proves to be a much commoner
Learning Goals Haematology
threat to survival than lung haemorrhage, and the
Question Filters diagnosis of Goodpasture’s disease is reached more
Immunoglobulins
conveniently by serological testing (for anti-GBM
antibodies) and from kidney rather than lung biopsy. Biochemistry
Security
In some cases, however, lung disease dominates the Diabetes
Change Password clinical picture, where the majority of patients are
male smokers and some report a recent exposure to Endocrinology
Sign Out
volatile hydrocarbons; case reports have additionally
identified recent exposure to chlorine and smoked Blood gases
cocaine. This suggests that when there is
CSF
susceptibility, inhaled toxic agents enhance pulmonary
endothelial damage and thus allows the initiation of
autoimmunity or the ready access of existing
autoantibody to basement membrane.
Respiratory presentation is with cough, breathlessness
and haemoptysis, which is intermittent and ranges
from occasional streaks to massive fatal bleeding.
Systemic symptoms of fever, joint pains or weight
loss are unusual. The chest X-ray shows patchy or
diffuse shadowing due to intra-alveolar blood, usually
resolving over the course of 2 weeks unless there is
further bleeding. At the time of bleeding there may be
arterial hypoxaemia and reduced lung volumes. Serial
measurement of TLco can be used to monitor
progression, and prolonged bleeding may lead to iron-
deficiency anaemia. The carbon monoxide (CO)
diffusing capacity or transfer factor (TLco) is widely
used as a simple test of the integrity of the alveolar
capillary membrane and of the overall gas-exchanging
function of the lungs. It has good sensitivity but poor
specificity, as impairment can result from a variety of
pathological processes. Renal function may be normal
initially but then deteriorates over days to weeks.
Steroids, other immunosuppressant drugs

file:///E|/Shakhawan/Respiratory%20S/242a.htm[3/13/2012 4:07:44 PM]


MyPasTest

(cyclophosphamide in particular) and plasmapheresis


are all used (in some circumstances) to control renal
disease, and are additionally helpful in treating
pulmonary haemorrhage. Patients should not smoke
and should avoid hydrocarbon exposure.

2030

© 2011 PasTest Ltd | About Us | Contact Us | Help

file:///E|/Shakhawan/Respiratory%20S/242a.htm[3/13/2012 4:07:44 PM]


MyPasTest

Main Navigation
Question Browser: MRCP 1
Home
Subscriptions Question Browser Exam Builder Saved Exams
» MRCP 1 Session Progress
• Question Browser
Questions Correct 13
• Timed Test A patient with asthma complains of worsening of her
symptoms and increasing shortness of breath. She was Questions Incorrect 0
• Mock Exam
recently started on a new medication which she feels may Questions Total 13
• Past Papers have caused the problem.
Questions Percentage 100 %
• Random Questions
What is this medication most likely to be? More
• My Performance
• Media Bank Salbutamol
• New Multimedia Timolol eye drops
Reference: Normal Values
Online Extras Hormone replacement therapy
Haematology
Library Ferrous sulphate
Community Leukotriene antagonists Immunoglobulins
Help Biochemistry
2031
PasTest Store Diabetes
Endocrinology
My Account
Blood gases
Profile
CSF
Newsletters
My Career and Exams
Order History
Learning Goals
Question Filters

Security

Change Password
Sign Out

© 2011 PasTest Ltd | About Us | Contact Us | Help

file:///E|/Shakhawan/Respiratory%20S/243.htm[3/13/2012 4:07:46 PM]


MyPasTest

Main Navigation
Question Browser: MRCP 1
Home
Subscriptions Question Browser Exam Builder Saved Exams
» MRCP 1 Session Progress
• Question Browser
Questions Correct 14
• Timed Test A patient with asthma complains of worsening of her
symptoms and increasing shortness of breath. She was Questions Incorrect 0
• Mock Exam
recently started on a new medication which she feels may Questions Total 14
• Past Papers have caused the problem.
Questions Percentage 100 %
• Random Questions
What is this medication most likely to be? More
• My Performance
• Media Bank Salbutamol
• New Multimedia Timolol eye drops Your answer
Online Extras Hormone replacement therapy
Library Ferrous sulphate
Community Leukotriene antagonists
Help
PasTest Store

My Account Although the adverse effects of oral or systemic β-


blockers are well recognised, those of ophthalmic
Profile preparations are easily overlooked. Timolol, which is
Newsletters commonly used in eye drops for the treatment of
glaucoma, is a potent non-selective β-blocker. Its use
My Career and Exams has frequently been associated with worsening asthma.
Order History The ophthalmic formulation of the newer β-blocker Reference: Normal Values
Learning Goals betaxolol appears to be less dangerous, but should be
Haematology
avoided in patients with asthma unless no suitable
Question Filters alternative is available . Immunoglobulins
Biochemistry
Security
2031
Diabetes
Change Password
Sign Out Endocrinology

Blood gases

CSF

© 2011 PasTest Ltd | About Us | Contact Us | Help

file:///E|/Shakhawan/Respiratory%20S/243a.htm[3/13/2012 4:07:47 PM]


MyPasTest

Main Navigation
Question Browser: MRCP 1
Home
Subscriptions Question Browser Exam Builder Saved Exams
» MRCP 1 Session Progress
• Question Browser
Questions Correct 14
• Timed Test A 40-year-old man presents with a 2-month history of
cough and breathlessness. He has also noticed Questions Incorrect 0
• Mock Exam
haemoptysis, which he says has gradually worsened. On Questions Total 14
• Past Papers examination he has bilateral basal crepitations. His chest X-
Questions Percentage 100 %
• Random Questions ray shows diffuse shadowing. He has moderate renal
failure. He is previously completely well and holds down a More
• My Performance
job as a solicitor.
• Media Bank
Which investigation would be most useful in
• New Multimedia
obtaining the diagnosis? Reference: Normal Values
Online Extras
CT thorax Haematology
Library
Community Ventilation–perfusion scan Immunoglobulins
Help Bronchoscopy Biochemistry
PasTest Store Renal biopsy
Diabetes
Sputum sample
Endocrinology
My Account 2032
Blood gases
Profile
CSF
Newsletters
My Career and Exams
Order History
Learning Goals
Question Filters

Security

Change Password
Sign Out

© 2011 PasTest Ltd | About Us | Contact Us | Help

file:///E|/Shakhawan/Respiratory%20S/244.htm[3/13/2012 4:07:49 PM]


MyPasTest

Main Navigation
Question Browser: MRCP 1
Home
Subscriptions Question Browser Exam Builder Saved Exams
» MRCP 1 Session Progress
• Question Browser
Questions Correct 15
• Timed Test A 40-year-old man presents with a 2-month history of
cough and breathlessness. He has also noticed Questions Incorrect 0
• Mock Exam
haemoptysis, which he says has gradually worsened. On Questions Total 15
• Past Papers examination he has bilateral basal crepitations. His chest X-
Questions Percentage 100 %
• Random Questions ray shows diffuse shadowing. He has moderate renal
failure. He is previously completely well and holds down a More
• My Performance
job as a solicitor.
• Media Bank
Which investigation would be most useful in
• New Multimedia
obtaining the diagnosis?
Online Extras
Library CT thorax
Community Ventilation–perfusion scan
Help Bronchoscopy
PasTest Store Renal biopsy Your answer
Sputum sample

My Account

Profile
Newsletters
Goodpasture’s disease consists of diffuse pulmonary
My Career and Exams haemorrhage and glomerulonephritis with linear
Order History deposition of antibody (90% of which are directed Reference: Normal Values
against the α-3 chain of type-IV collagen) along the
Learning Goals Haematology
glomerular basement membrane.
Question Filters
Serological testing for anti-GBM antibodies and ANCA Immunoglobulins
(antineutrophil cytoplasmic antibody) is crucial for
confirming the diagnosis, and a renal biopsy is almost Biochemistry
Security
always warranted. Some healthy individuals exposed Diabetes
Change Password to inhaled oils, hydrocarbons or solvents may have
borderline raised anti-GBM antibody levels. Anti-GBM Endocrinology
Sign Out
antibodies have also been detected in HIV-negative
patients with pneumocystis pneumonia. Blood gases

CSF

2032

© 2011 PasTest Ltd | About Us | Contact Us | Help

file:///E|/Shakhawan/Respiratory%20S/244a.htm[3/13/2012 4:07:50 PM]


MyPasTest

Main Navigation
Question Browser: MRCP 1
Home
Subscriptions Question Browser Exam Builder Saved Exams
» MRCP 1 Session Progress
• Question Browser
Questions Correct 15
• Timed Test A 55-year-old patient with systemic sclerosis has been
referred by her rheumatologist because of increasing Questions Incorrect 0
• Mock Exam
shortness of breath and occasional dry cough. On Questions Total 15
• Past Papers examination fine crackles are heard at the lung bases. What
Questions Percentage 100 %
• Random Questions is the most likely cause for her symptoms?
• My Performance More
Chest infection
• Media Bank
Pulmonary embolism
• New Multimedia
Fibrosing alveolitis Reference: Normal Values
Online Extras
Tuberculosis Haematology
Library
Late asthma
Community Immunoglobulins
Help 2033 Biochemistry
PasTest Store Diabetes
Endocrinology
My Account
Blood gases
Profile
CSF
Newsletters
My Career and Exams
Order History
Learning Goals
Question Filters

Security

Change Password
Sign Out

© 2011 PasTest Ltd | About Us | Contact Us | Help

file:///E|/Shakhawan/Respiratory%20S/245.htm[3/13/2012 4:07:52 PM]


MyPasTest

Main Navigation
Question Browser: MRCP 1
Home
Subscriptions Question Browser Exam Builder Saved Exams
» MRCP 1 Session Progress
• Question Browser
Questions Correct 16
• Timed Test A 55-year-old patient with systemic sclerosis has been
referred by her rheumatologist because of increasing Questions Incorrect 0
• Mock Exam
shortness of breath and occasional dry cough. On Questions Total 16
• Past Papers examination fine crackles are heard at the lung bases. What
Questions Percentage 100 %
• Random Questions is the most likely cause for her symptoms?
• My Performance More
Chest infection
• Media Bank
Pulmonary embolism
• New Multimedia
Fibrosing alveolitis Your answer
Online Extras
Tuberculosis
Library
Late asthma
Community
Help
PasTest Store
Lung disease may be the first manifestation of systemic
My Account sclerosis, and dyspnoea occurs in roughly 55% of
patients with this condition. Cough is a less frequently
Profile reported symptom, but when it occurs is dry and non-
productive. Pleuritic chest pain is uncommon. A history
Newsletters
of Raynaud’s phenomenon is often present. Digital
My Career and Exams clubbing is rare because of the poor vasculature of the
Order History digits. Fine crackles are heard at the bases and are of a Reference: Normal Values
‘Velcro’ character. Lung fibrosis occurs more commonly
Learning Goals in patients with the Scl 70, anti-DNA topoisomerase Haematology
Question Filters autoantibody. Chest X-ray series have identified diffuse
lung disease in up to 67% of patients. Oesophageal Immunoglobulins
dilatation may be seen on chest X-ray and is almost
Biochemistry
Security universally present on high-resolution computed
tomography. Diabetes
Change Password
Sign Out Endocrinology
2033
Blood gases

CSF

© 2011 PasTest Ltd | About Us | Contact Us | Help

file:///E|/Shakhawan/Respiratory%20S/245a.htm[3/13/2012 4:07:53 PM]


MyPasTest

Main Navigation
Question Browser: MRCP 1
Home
Subscriptions Question Browser Exam Builder Saved Exams
» MRCP 1 Session Progress
• Question Browser
Questions Correct 16
• Timed Test A patient with lung cancer underwent radiotherapy 2 weeks
ago, he now complains of a severe dry cough, increasing Questions Incorrect 0
• Mock Exam
dyspnoea and tachypnoea. On examination there are local Questions Total 16
• Past Papers crepitations, his chest X-ray shows hazy consolidation that
Questions Percentage 100 %
• Random Questions correspond to the irradiation field. What is the most likely
diagnosis? More
• My Performance
• Media Bank Pulmonary embolism
• New Multimedia Mycoplasma pneumonia Reference: Normal Values
Online Extras Radiation pneumonitis
Haematology
Library Legionella pneumonia
Community Cancer progression Immunoglobulins
Help Biochemistry
2034
PasTest Store Diabetes
Endocrinology
My Account
Blood gases
Profile
CSF
Newsletters
My Career and Exams
Order History
Learning Goals
Question Filters

Security

Change Password
Sign Out

© 2011 PasTest Ltd | About Us | Contact Us | Help

file:///E|/Shakhawan/Respiratory%20S/246.htm[3/13/2012 4:07:55 PM]


MyPasTest

Main Navigation
Question Browser: MRCP 1
Home
Subscriptions Question Browser Exam Builder Saved Exams
» MRCP 1 Session Progress
• Question Browser
Questions Correct 17
• Timed Test A patient with lung cancer underwent radiotherapy 2 weeks
ago, he now complains of a severe dry cough, increasing Questions Incorrect 0
• Mock Exam
dyspnoea and tachypnoea. On examination there are local Questions Total 17
• Past Papers crepitations, his chest X-ray shows hazy consolidation that
Questions Percentage 100 %
• Random Questions correspond to the irradiation field. What is the most likely
diagnosis? More
• My Performance
• Media Bank Pulmonary embolism
• New Multimedia Mycoplasma pneumonia
Online Extras Radiation pneumonitis Your answer
Library Legionella pneumonia
Community Cancer progression
Help
PasTest Store

My Account Symptoms begin within a few weeks of radiotherapy


and may persist for weeks or months. They occur in
Profile 10–30% of patients following radiotherapy for lung
Newsletters cancer. A cough (which can be severe and may
produce thick sputum) and breathlessness are the
My Career and Exams principal symptoms, but may be accompanied by fever
Order History of variable degree. Reference: Normal Values
Learning Goals On examination there may be tachypnoea, cyanosis in Haematology
Question Filters severe disease, and local crepitations. Telangiectases,
the result of cutaneous radiation damage, are often Immunoglobulins
observed in the overlying skin. The most
characteristic X-ray feature is an area of hazy Biochemistry
Security
consolidation demarcated by a sharp margin (crossing Diabetes
Change Password anatomical pulmonary planes) that corresponds to the
limits of the irradiation field, though additional effects Endocrinology
Sign Out
are usually detectable beyond these boundaries.
Computed tomography provides the best means of Blood gases
early identification, ground-glass attenuation and
CSF
interalveolar septal thickening being the early
characteristic features. Dense local fibrosis may
develop up to a year or two after radiation, and
magnetic resonance imaging may be required to allow
differentiation from tumour recurrence.

2034

© 2011 PasTest Ltd | About Us | Contact Us | Help

file:///E|/Shakhawan/Respiratory%20S/246a.htm[3/13/2012 4:07:56 PM]


MyPasTest

Main Navigation
Question Browser: MRCP 1
Home
Subscriptions Question Browser Exam Builder Saved Exams
» MRCP 1 Session Progress
• Question Browser
Questions Correct 17
• Timed Test A 29-year-old woman noticed shortness of breath and a
dry cough while jogging last winter. She now wakes up Questions Incorrect 0
• Mock Exam
twice a week at 4 o’clock in the morning with a Questions Total 17
• Past Papers troublesome cough. What is the most likely cause?
Questions Percentage 100 %
• Random Questions
• My Performance Cardiac insufficiency More
• Media Bank Mycoplasma pneumonia

• New Multimedia Asthma


Reference: Normal Values
Online Extras Bronchiectasis
Extrinsic allergic alveolitis Haematology
Library
Community Immunoglobulins
2035
Help Biochemistry
PasTest Store Diabetes
Endocrinology
My Account
Blood gases
Profile
CSF
Newsletters
My Career and Exams
Order History
Learning Goals
Question Filters

Security

Change Password
Sign Out

© 2011 PasTest Ltd | About Us | Contact Us | Help

file:///E|/Shakhawan/Respiratory%20S/247.htm[3/13/2012 4:07:58 PM]


MyPasTest

Main Navigation
Question Browser: MRCP 1
Home
Subscriptions Question Browser Exam Builder Saved Exams
» MRCP 1 Session Progress
• Question Browser
Questions Correct 17
• Timed Test A 29-year-old woman noticed shortness of breath and a
dry cough while jogging last winter. She now wakes up Questions Incorrect 1
• Mock Exam
twice a week at 4 o’clock in the morning with a Questions Total 18
• Past Papers troublesome cough. What is the most likely cause?
Questions Percentage 94 %
• Random Questions
• My Performance Cardiac insufficiency More
• Media Bank Mycoplasma pneumonia

• New Multimedia Asthma Correct answer

Online Extras Bronchiectasis

Library Extrinsic allergic alveolitis Your answer

Community
Help
PasTest Store
The symptoms of asthma are non-specific: shortness of
breath, wheezing, chest tightness and cough. These are
My Account manifestations of airway narrowing (which is usually
variable in severity over short periods, but can be
Profile persistent) and of airway hyperresponsiveness. Asthma
as the cause of these symptoms is suggested by the
Newsletters
variability in their severity and distinguished by their
My Career and Exams periodicity (such as daily, weekly, monthly, or
Order History seasonal), their provocation by specific (such as Reference: Normal Values
allergen) and non-specific stimuli and their reversibility
Learning Goals with bronchodilators or corticosteroids. Patients with Haematology
Question Filters asthma can be categorised, at any one time, by
whether their symptoms are intermittent or persistent, Immunoglobulins
and by the severity of their symptoms and underlying
Biochemistry
Security airway narrowing measured by lung function tests (even
those with mild asthma can develop severe asthma): Diabetes
Change Password
Mild, intermittent asthma – symptoms occur less Endocrinology
Sign Out
than weekly, with normal or near-normal lung
function between episodes. Blood gases
Mild persistent asthma – symptoms occur more
CSF
than weekly but less than daily, with normal or
near-normal lung function between episodes.
Moderate persistent asthma – symptoms occur
daily, with mild to moderate variable airflow
limitation.
Severe persistent asthma – symptoms occur daily
and interfere with normal activities. There is
frequent nocturnal waking and moderate to
severe variable airflow limitation.
Severe asthma – severe distressing symptoms
prevent sleep. Severe airflow limitation responds
poorly to inhaled bronchodilators and can be life-
threatening.

2035

© 2011 PasTest Ltd | About Us | Contact Us | Help

file:///E|/Shakhawan/Respiratory%20S/247a.htm[3/13/2012 4:07:59 PM]


MyPasTest

Main Navigation
Question Browser: MRCP 1
Home
Subscriptions Question Browser Exam Builder Saved Exams
» MRCP 1 Session Progress
• Question Browser
Questions Correct 17
• Timed Test A patient with lung cancer underwent radiotherapy 2 weeks
ago. He now complains of a severe dry cough, increasing Questions Incorrect 1
• Mock Exam
dyspnoea and tachypnoea. On examination there are local Questions Total 18
• Past Papers crepitations and his chest X-ray shows hazy consolidation
Questions Percentage 94 %
• Random Questions that corresponds to the irradiation field. What is the most
important therapy to relieve his symptoms? More
• My Performance
• Media Bank Anticoagulation
• New Multimedia Restart radiotherapy Reference: Normal Values
Online Extras Corticosteroids
Haematology
Library Antibiotics
Community Salbutamol inhaler Immunoglobulins
Help Biochemistry
2036
PasTest Store Diabetes
Endocrinology
My Account
Blood gases
Profile
CSF
Newsletters
My Career and Exams
Order History
Learning Goals
Question Filters

Security

Change Password
Sign Out

© 2011 PasTest Ltd | About Us | Contact Us | Help

file:///E|/Shakhawan/Respiratory%20S/248.htm[3/13/2012 4:08:01 PM]


MyPasTest

Main Navigation
Question Browser: MRCP 1
Home
Subscriptions Question Browser Exam Builder Saved Exams
» MRCP 1 Session Progress
• Question Browser
Questions Correct 18
• Timed Test A patient with lung cancer underwent radiotherapy 2 weeks
ago. He now complains of a severe dry cough, increasing Questions Incorrect 1
• Mock Exam
dyspnoea and tachypnoea. On examination there are local Questions Total 19
• Past Papers crepitations and his chest X-ray shows hazy consolidation
Questions Percentage 94 %
• Random Questions that corresponds to the irradiation field. What is the most
important therapy to relieve his symptoms? More
• My Performance
• Media Bank Anticoagulation
• New Multimedia Restart radiotherapy
Online Extras Corticosteroids Your answer
Library Antibiotics
Community Salbutamol inhaler
Help
PasTest Store

My Account This patient presents with radiation pneumonitis.


Symptoms begin within a few weeks of radiotherapy
Profile and may persist for weeks or months. They occur in
Newsletters 10–30% of patients following radiotherapy for lung
cancer. A cough (which can be severe and may
My Career and Exams produce thick sputum) and breathlessness are the
Order History principal symptoms, but may be accompanied by fever Reference: Normal Values
of variable degree.
Learning Goals Haematology
Question Filters In cases where symptoms are slight, no specific
treatment is needed. However, in more severe Immunoglobulins
disease, corticosteroids produce relief during the
acute phase in most patients. Any response to Biochemistry
Security
corticosteroids occurs within 3–4 days, with clinical Diabetes
Change Password and radiographic improvement, and treatment should
be continued for 3–4 weeks before tapering and Endocrinology
Sign Out
stopping. Corticosteroids do not, however, influence
the extent of subsequent pulmonary fibrosis. Blood gases
Symptomatic relief of cough and hypoxaemia by an
CSF
opioid antitussive and oxygen supplementation may
also be needed. Prevention offers the best means of
control, and further development of methods to
detect undue susceptibility and early disease may
prove to be valuable in developing safer fractionating
protocols.

2036

© 2011 PasTest Ltd | About Us | Contact Us | Help

file:///E|/Shakhawan/Respiratory%20S/248a.htm[3/13/2012 4:08:02 PM]


MyPasTest

Main Navigation
Question Browser: MRCP 1
Home
Subscriptions Question Browser Exam Builder Saved Exams
» MRCP 1 Session Progress
• Question Browser
Questions Correct 18
• Timed Test A 29-year-old woman noticed shortness of breath and dry
cough while jogging last winter. She now wakes up twice a Questions Incorrect 1
• Mock Exam
week at 4 o’clock in the morning with a troublesome cough. Questions Total 19
• Past Papers What is the next step in the diagnosis?
Questions Percentage 94 %
• Random Questions
• My Performance Chest X-ray More
• Media Bank CT thorax

• New Multimedia Measurement of airflow limitation


Reference: Normal Values
Online Extras Bronchoscopy
d-Dimer Haematology
Library
Community Immunoglobulins
2037
Help Biochemistry
PasTest Store Diabetes
Endocrinology
My Account
Blood gases
Profile
CSF
Newsletters
My Career and Exams
Order History
Learning Goals
Question Filters

Security

Change Password
Sign Out

© 2011 PasTest Ltd | About Us | Contact Us | Help

file:///E|/Shakhawan/Respiratory%20S/249.htm[3/13/2012 4:08:04 PM]


MyPasTest

Main Navigation
Question Browser: MRCP 1
Home
Subscriptions Question Browser Exam Builder Saved Exams
» MRCP 1 Session Progress
• Question Browser
Questions Correct 19
• Timed Test A 29-year-old woman noticed shortness of breath and dry
cough while jogging last winter. She now wakes up twice a Questions Incorrect 1
• Mock Exam
week at 4 o’clock in the morning with a troublesome cough. Questions Total 20
• Past Papers What is the next step in the diagnosis?
Questions Percentage 95 %
• Random Questions
• My Performance Chest X-ray More
• Media Bank CT thorax

• New Multimedia Measurement of airflow limitation Your answer

Online Extras Bronchoscopy

Library d-Dimer

Community
Help
PasTest Store
Asthma is usually diagnosed by the demonstration of
airflow limitation that varies spontaneously over short
My Account periods of time, or which reverses after inhalation of
short-acting β-agonists or, over a more prolonged
Profile
period, in response to an inhaled or oral
Newsletters corticosteroid.
My Career and Exams The most clinically useful measurements of airflow
Order History limitation are: (a) forced expiratory volume in 1 s Reference: Normal Values
(FEV 1 ), which may be expressed as a proportion of
Learning Goals Haematology
the forced vital capacity (FVC) as FEV1 /FVC%, and
Question Filters (b) peak expiratory flow rate (PEFR). Both tests Immunoglobulins
require the patient to provide a reproducible maximal
forced expiratory manoeuvre using tested and Biochemistry
Security validated equipment. FEV1 has the advantage of a
Diabetes
Change Password visible tracing of the expelled volume of air over time,
which allows the observer to determine whether Endocrinology
Sign Out reproducible maximal forced expiratory manoeuvres
have been made. PEFR testing does not provide this Blood gases
opportunity. However, peak-flow meters employed to
measure PEFR, unlike spirometers required to CSF
measure FEV1 , can be used regularly by patients to
monitor their lung function, indicating the need for
altered treatment at an early stage.
Whether abnormalities of FEV1 and PEFR should be
expressed in absolute or proportional terms remains
to be decided. Expression as an absolute difference
from the average value anticipated for an individual of
given age, gender and height has more physiological
validity. However, the majority of lung function
laboratories in the UK continue to define values of
FEV1 or PEFR of 20% or more below the mean
predicted value as abnormal.

2037

© 2011 PasTest Ltd | About Us | Contact Us | Help

file:///E|/Shakhawan/Respiratory%20S/249a.htm[3/13/2012 4:08:05 PM]


MyPasTest

Main Navigation
Question Browser: MRCP 1
Home
Subscriptions Question Browser Exam Builder Saved Exams
» MRCP 1 Session Progress
• Question Browser
Questions Correct 19
• Timed Test A 64-year-old mechanic and lifelong smoker noticed
haemoptysis a few days after he had a cold. Clinical Questions Incorrect 1
• Mock Exam
examination is unremarkable. His chest X-ray shows Questions Total 20
• Past Papers bilateral hilar enlargement and mediastinal widening. What
Questions Percentage 95 %
• Random Questions is the most likely diagnosis?
• My Performance More
Tuberculosis
• Media Bank
Bronchial carcinoma
• New Multimedia
Lymphoma Reference: Normal Values
Online Extras
Hilar metastases Haematology
Library
Lung abscess
Community Immunoglobulins
Help 2038 Biochemistry
PasTest Store Diabetes
Endocrinology
My Account
Blood gases
Profile
CSF
Newsletters
My Career and Exams
Order History
Learning Goals
Question Filters

Security

Change Password
Sign Out

© 2011 PasTest Ltd | About Us | Contact Us | Help

file:///E|/Shakhawan/Respiratory%20S/250.htm[3/13/2012 4:08:07 PM]


MyPasTest

Main Navigation
Question Browser: MRCP 1
Home
Subscriptions Question Browser Exam Builder Saved Exams
» MRCP 1 Session Progress
• Question Browser
Questions Correct 20
• Timed Test A 64-year-old mechanic and lifelong smoker noticed
Questions Incorrect 1
• Mock Exam haemoptysis a few days after he had a cold. Clinical
examination is unremarkable. His chest X-ray shows Questions Total 21
• Past Papers bilateral hilar enlargement and mediastinal widening. What Questions Percentage 95 %
• Random Questions is the most likely diagnosis?
• My Performance More
Tuberculosis
• Media Bank
Bronchial carcinoma Your answer
• New Multimedia
Lymphoma
Online Extras
Hilar metastases
Library
Lung abscess
Community
Help
PasTest Store

The value of the chest X-ray in the diagnosis and


My Account management of pulmonary neoplasm needs no
emphasis. No initial examination is complete without
Profile a lateral film. Coned views of the ribs may help where
Newsletters rib invasion is suspected clinically. However, the
finding of a normal X-ray of the chest does not
My Career and Exams exclude bronchial carcinoma as patients presenting
Order History with haemoptysis and a normal chest X-ray are Reference: Normal Values
sometimes found to have a central tumour on
Learning Goals Haematology
bronchoscopy.
Question Filters
The common appearance of a tumour arising from the Immunoglobulins
main central airways (70% of all cases) is
enlargement of one or other hilum. Even experienced Biochemistry
Security
observers sometimes have difficulty in deciding Diabetes
Change Password whether or not a hilar shadow is enlarged, and if
there is any suspicion, investigation by bronchoscopy Endocrinology
Sign Out
and/or CT should be pursued.
Blood gases
Consolidation and collapse distal to the tumour may
have occurred by the time the patient presents, with CSF
the tumour itself often being obscured in the process.
Collapse of the left lower lobe is often hard to
identify, as is a tumour situated behind the heart.
Apically located masses or superior sulcus tumours
(Pancoast tumours) may be misdiagnosed as pleural
caps, and patients often have a long history of pain in
the distribution of the brachial nerve roots. Loss of
the head of the first, second or third rib is not
unusual. The mediastinum may be widened by
enlarged nodes. Involvement of the phrenic nerve
may lead to paralysis and elevation of the
hemidiaphragm, which then moves paradoxically on
sniffing.
Tumour spreading to the pleura causes effusion, but
such an abnormality may be secondary to infection
beyond obstruction caused by a central tumour. The
ribs and spine should be carefully examined for the
presence of metastases. Spread of tumour from
mediastinal nodes peripherally along the lymphatics
gives the characteristic appearance of lymphangitis
carcinomatosa – bilateral hilar enlargement with
streaky shadows fanning out into the lung fields on
either side. Rarely, localised obstructive emphysema
may be observed.

file:///E|/Shakhawan/Respiratory%20S/250a.htm[3/13/2012 4:08:08 PM]


MyPasTest

Main Navigation
Question Browser: MRCP 1
Home
Subscriptions Question Browser Exam Builder Saved Exams
» MRCP 1 Session Progress
• Question Browser
Questions Correct 20
• Timed Test A 64-year-old mechanic and lifelong smoker noticed
haemoptysis a few days after he had a cold. Clinical Questions Incorrect 1
• Mock Exam
examination is unremarkable. His chest X-ray shows Questions Total 21
• Past Papers bilateral hilar enlargement and mediastinal widening. What
Questions Percentage 95 %
• Random Questions is the next step in obtaining the diagnosis?
• My Performance More
CT thorax
• Media Bank
Bronchoscopy
• New Multimedia
Ventilation–perfusion scan Reference: Normal Values
Online Extras
Sputum sample Haematology
Library
d-Dimer
Community Immunoglobulins
Help 2039 Biochemistry
PasTest Store Diabetes
Endocrinology
My Account
Blood gases
Profile
CSF
Newsletters
My Career and Exams
Order History
Learning Goals
Question Filters

Security

Change Password
Sign Out

© 2011 PasTest Ltd | About Us | Contact Us | Help

file:///E|/Shakhawan/Respiratory%20S/251.htm[3/13/2012 4:08:10 PM]


MyPasTest

Main Navigation
Question Browser: MRCP 1
Home
Subscriptions Question Browser Exam Builder Saved Exams
» MRCP 1 Session Progress
• Question Browser
Questions Correct 21
• Timed Test A 64-year-old mechanic and lifelong smoker noticed
haemoptysis a few days after he had a cold. Clinical Questions Incorrect 1
• Mock Exam
examination is unremarkable. His chest X-ray shows Questions Total 22
• Past Papers bilateral hilar enlargement and mediastinal widening. What
Questions Percentage 95 %
• Random Questions is the next step in obtaining the diagnosis?
• My Performance More
CT thorax Your answer
• Media Bank
Bronchoscopy
• New Multimedia
Ventilation–perfusion scan
Online Extras
Sputum sample
Library
d-Dimer
Community
Help
PasTest Store
Where a chest X-ray has been requested in primary or
My Account secondary care and is incidentally suggestive of lung
cancer, a second copy of the radiologist's report should
Profile be sent to a designated member of the lung cancer
multi-disciplinary team (MDT), usually the chest
Newsletters
physician. The MDT should have a mechanism in place
My Career and Exams to follow up these reports to enable the patient's GP to
Order History have a management plan in place. Patients with known Reference: Normal Values
or suspected lung cancer should be offered a contrast-
Learning Goals enhanced chest CT scan to further the diagnosis and Haematology
Question Filters stage the disease. The scan should also include the liver
and adrenals. Chest CT should be performed before an Immunoglobulins
intended fibreoptic bronchoscopy or any other biopsy
Biochemistry
Security procedure.
Diabetes
Change Password
Sign Out 2039 Endocrinology

Blood gases

CSF

© 2011 PasTest Ltd | About Us | Contact Us | Help

file:///E|/Shakhawan/Respiratory%20S/251a.htm[3/13/2012 4:08:11 PM]


MyPasTest

Main Navigation
Question Browser: MRCP 1
Home
Subscriptions Question Browser Exam Builder Saved Exams
» MRCP 1 Session Progress
• Question Browser
Questions Correct 21
• Timed Test A 65-year-old patient with new-onset chronic obstructive
pulmonary disease ( COPD) asks you about his prognosis. Questions Incorrect 1
• Mock Exam
Questions Total 22
• Past Papers Which of the following single tests is the most
important predictor of survival in patients with Questions Percentage 95 %
• Random Questions
COPD? More
• My Performance
• Media Bank Blood gases
• New Multimedia Chest X-ray
Reference: Normal Values
Online Extras FEV1
Haematology
Library Exercise tolerance
Community Immunoglobulins
ECG
Help Biochemistry
2040
PasTest Store Diabetes
Endocrinology
My Account
Blood gases
Profile
CSF
Newsletters
My Career and Exams
Order History
Learning Goals
Question Filters

Security

Change Password
Sign Out

© 2011 PasTest Ltd | About Us | Contact Us | Help

file:///E|/Shakhawan/Respiratory%20S/252.htm[3/13/2012 4:08:13 PM]


MyPasTest

Main Navigation
Question Browser: MRCP 1
Home
Subscriptions Question Browser Exam Builder Saved Exams
» MRCP 1 Session Progress
• Question Browser
Questions Correct 21
• Timed Test A 65-year-old patient with new-onset chronic obstructive
pulmonary disease ( COPD) asks you about his prognosis. Questions Incorrect 2
• Mock Exam
Questions Total 23
• Past Papers Which of the following single tests is the most
important predictor of survival in patients with Questions Percentage 91 %
• Random Questions
COPD? More
• My Performance
• Media Bank Blood gases Your answer
• New Multimedia Chest X-ray
Online Extras FEV1 Correct answer
Library Exercise tolerance
Community ECG
Help
PasTest Store

My Account The strongest predictors of survival in patients with


COPD are age and baseline FEV1 . Less than 50% of
Profile patients whose FEV1 has fallen to 30% of predicted are
Newsletters alive 5 years later. There is an even stronger
My Career and Exams relationship between survival and the
postbronchodilator, rather than prebronchodilator,
Order History FEV1 . Other unfavourable prognostic factors include Reference: Normal Values
Learning Goals severe hypoxaemia, raised pulmonary arterial pressure Haematology
and low carbon monoxide transfer, which become
Question Filters
apparent in patients with severe disease. Factors Immunoglobulins
favouring improved survival are stopping smoking and a
large bronchodilator response. A reduced FEV1 is also Biochemistry
Security
an important additional risk factor for lung cancer, Diabetes
Change Password independent of age or cigarette smoking.
Sign Out Endocrinology

Blood gases
2040
CSF

© 2011 PasTest Ltd | About Us | Contact Us | Help

file:///E|/Shakhawan/Respiratory%20S/252a.htm[3/13/2012 4:08:14 PM]


MyPasTest

Main Navigation
Question Browser: MRCP 1
Home
Subscriptions Question Browser Exam Builder Saved Exams
» MRCP 1 Session Progress
• Question Browser
Questions Correct 21
• Timed Test Which treatment improves the long-term prognosis in
patients with chronic obstructive pulmonary disease Questions Incorrect 2
• Mock Exam
(COPD)? Questions Total 23
• Past Papers
Questions Percentage 91 %
• Random Questions Inhaled steroids
• My Performance Oral steroids More
• Media Bank Inhaled β2 -agonists
• New Multimedia Long-term domiciliary oxygen therapy Reference: Normal Values
Online Extras Theophylline
Haematology
Library
Community 2041 Immunoglobulins
Help Biochemistry
PasTest Store Diabetes
Endocrinology
My Account
Blood gases
Profile
CSF
Newsletters
My Career and Exams
Order History
Learning Goals
Question Filters

Security

Change Password
Sign Out

© 2011 PasTest Ltd | About Us | Contact Us | Help

file:///E|/Shakhawan/Respiratory%20S/253.htm[3/13/2012 4:08:16 PM]


MyPasTest

Main Navigation
Question Browser: MRCP 1
Home
Subscriptions Question Browser Exam Builder Saved Exams
» MRCP 1 Session Progress
• Question Browser
Questions Correct 22
• Timed Test Which treatment improves the long-term prognosis in
patients with chronic obstructive pulmonary disease Questions Incorrect 2
• Mock Exam
(COPD)? Questions Total 24
• Past Papers
Questions Percentage 91 %
• Random Questions Inhaled steroids
• My Performance Oral steroids More
• Media Bank Inhaled β2 -agonists
• New Multimedia Long-term domiciliary oxygen therapy Your answer
Online Extras Theophylline
Library
Community
Help
PasTest Store The only treatment that improves the long-term
prognosis in patients with COPD is long-term
domiciliary oxygen therapy, given for at least 15 h per
My Account day, as shown by two multicentre trials: one
Profile conducted by the Medical Research Council (MRC) in
the UK, and the other by the Nocturnal Oxygen
Newsletters Therapy Trial (NOTT).
My Career and Exams The reasons for the improvement in survival with
Order History oxygen therapy in patients with COPD are still Reference: Normal Values
uncertain, but are not clearly related to improvements
Learning Goals in pulmonary haemodynamics. As usual, survival is Haematology
Question Filters related to the level of pulmonary arterial hypertension
in patients who receive long-term oxygen therapy. In Immunoglobulins
the MRC trial, there was no significant improvement in Biochemistry
Security pulmonary arterial pressure following oxygen therapy,
but the increase of 3 mmHg per year in pulmonary Diabetes
Change Password arterial pressure in the control group did not occur in
Sign Out those who were treated. Overnight oxygen therapy, Endocrinology
which abolishes nocturnal desaturation, also
Blood gases
decreases pulmonary arterial pressure.
In addition to the improvement in survival, a number CSF
of studies have examined other effects of
supplementary oxygen therapy. The effects of oxygen
therapy on breathlessness remain unclear. However,
several studies have shown that oxygen therapy can
lead to an improvement in exercise endurance in
patients with COPD, associated with a reduction in
ventilation at a given submaximal work rate, and an
improvement in walking distance and in the ability to
perform daily activities.
Assessment of patients taking part in the NOTT study
showed that they have marked disturbances in mood
and quality of life: after 6 months of oxygen therapy,
42% of patients showed evidence of an improvement
in cognitive function, but little change in mood or
quality of life.
As in all studies of patients with COPD, the FEV1 is the
strongest predictor of survival in patients receiving
long-term oxygen therapy, but this does not influence
the decline in FEV1 . Long-term oxygen therapy has
been shown to affect the polycythaemia that occurs in
patients with chronic hypoxaemia, by reducing both
the haematocrit and the red cell mass. The clinical

file:///E|/Shakhawan/Respiratory%20S/253a.htm[3/13/2012 4:08:18 PM]


MyPasTest

relevance of these haematological changes produced


by oxygen therapy remains unclear.
Continued cigarette smoking should be a relative
contraindication to long-term oxygen therapy. There
are three forms of domiciliary supplemental oxygen
therapy:

1) long-term controlled oxygen therapy for at least 15h


per day in patients with chronic respiratory failure

2) portable oxygen therapy for exercise-related


hypoxaemia, and

3) short-burst oxygen therapy, as a palliative treatment


for the temporary relief of symptoms.

2041

© 2011 PasTest Ltd | About Us | Contact Us | Help

file:///E|/Shakhawan/Respiratory%20S/253a.htm[3/13/2012 4:08:18 PM]


MyPasTest

Main Navigation
Question Browser: MRCP 1
Home
Subscriptions Question Browser Exam Builder Saved Exams
» MRCP 1 Session Progress
• Question Browser
Questions Correct 22
• Timed Test People with coal-workers’ pneumoconiosis are predisposed
to developing which disease? Questions Incorrect 2
• Mock Exam
Questions Total 24
• Past Papers
Tuberculosis Questions Percentage 91 %
• Random Questions
Carcinoma of the lung
• My Performance More
Progressive massive fibrosis
• Media Bank
Silicosis
• New Multimedia
Heart failure Reference: Normal Values
Online Extras
2042 Haematology
Library
Community Immunoglobulins
Help Biochemistry
PasTest Store Diabetes
Endocrinology
My Account
Blood gases
Profile
CSF
Newsletters
My Career and Exams
Order History
Learning Goals
Question Filters

Security

Change Password
Sign Out

© 2011 PasTest Ltd | About Us | Contact Us | Help

file:///E|/Shakhawan/Respiratory%20S/254.htm[3/13/2012 4:08:20 PM]


MyPasTest

Main Navigation
Question Browser: MRCP 1
Home
Subscriptions Question Browser Exam Builder Saved Exams
» MRCP 1 Session Progress
• Question Browser
Questions Correct 23
• Timed Test People with coal-workers’ pneumoconiosis are predisposed
to developing which disease? Questions Incorrect 2
• Mock Exam
Questions Total 25
• Past Papers
Tuberculosis Questions Percentage 92 %
• Random Questions
Carcinoma of the lung
• My Performance More
Progressive massive fibrosis Your answer
• Media Bank
Silicosis
• New Multimedia
Heart failure
Online Extras
Library
Community
Help Simple coal-worker’s pneumoconiosis causes no
PasTest Store symptoms or physical signs, nor any important
physiological abnormality. The danger associated with
simple pneumoconiosis is that it is a predisposition to
My Account progressive massive fibrosis, a risk directly related to
the profusion of simple pneumoconiosis on the X-ray.
Profile
Progressive massive fibrosis may occur during working
Newsletters life or appear for the first time after (sometimes many
My Career and Exams years after) dust exposure ceases, even when there is
no apparent simple pneumoconiosis on the X-ray.
Order History Reference: Normal Values
Progressive massive fibrosis usually causes a mixture
Learning Goals of restriction of lung volumes and, owing to Haematology
Question Filters associated emphysema, airflow obstruction.
Ultimately, it may lead to cor pulmonale and death. Immunoglobulins
However, the rate of progression is variable. In
general, the earlier that progressive massive fibrosis Biochemistry
Security
develops in a person’s life, the more rapidly it is Diabetes
Change Password progressive and thus the greater is the threat to
health. Endocrinology
Sign Out
A patient with progressive massive fibrosis may Blood gases
complain of shortness of breath and symptoms of cor
pulmonale. An unusual, but pathognomonic symptom CSF
is melanoptysis – the expectoration of the black
contents of a cavitated lesion. Haemoptysis and finger
clubbing suggest lung cancer and should not be
attributed to pneumoconiosis. Abnormal signs in the
chest, if present, relate to the presence of bullae,
although sometimes lobar collapse can occur.
Coal-worker’s pneumoconiosis is not associated with
an increased risk of tuberculosis or lung cancer,
although obviously these diseases can be seen in coal
miners and should be suspected if an unusual
progression of radiological changes occurs. The
association between pneumoconiosis and emphysema
has been controversial, but there is now clear
evidence of a parallel association between dust
exposure and two effects – pneumoconiosis and
airflow obstruction. The more dust that a miner has
been exposed to, the greater are his risks of
pneumoconiosis on the one hand, and productive
cough, reduction in forced expiratory volume in 1 s
(FEV 1 ), and the presence of centriacinar emphysema
on the other. Of course, the latter risks are also
related to cigarette smoking, and the effect of dust
exposure is additive.

file:///E|/Shakhawan/Respiratory%20S/254a.htm[3/13/2012 4:08:21 PM]


MyPasTest

Main Navigation
Question Browser: MRCP 1
Home
Subscriptions Question Browser Exam Builder Saved Exams
» MRCP 1 Session Progress
• Question Browser
Questions Correct 23
• Timed Test A 16-year-old boy is brought to casualty after a fire in his
parents' house. He inhaled a lot of smoke and has a hoarse Questions Incorrect 2
• Mock Exam
voice, stridor and burned nasal hairs. Due to deterioration Questions Total 25
• Past Papers in his peak flow rate and arterial blood gases, he has been
Questions Percentage 92 %
• Random Questions intubated, ventilated and transferred to the intensive care
unit where his condition is now stable. More
• My Performance
• Media Bank In terms of investigation, what is the most important next
step in assessing this boy’s condition?
• New Multimedia
Reference: Normal Values
Online Extras CT thorax
Haematology
Library Chest X-ray
Community Bronchoscopy Immunoglobulins
Help Ventilation–perfusion scan Biochemistry
PasTest Store Echocardiography Diabetes

2043 Endocrinology
My Account
Blood gases
Profile
CSF
Newsletters
My Career and Exams
Order History
Learning Goals
Question Filters

Security

Change Password
Sign Out

© 2011 PasTest Ltd | About Us | Contact Us | Help

file:///E|/Shakhawan/Respiratory%20S/255.htm[3/13/2012 4:08:23 PM]


MyPasTest

Main Navigation
Question Browser: MRCP 1
Home
Subscriptions Question Browser Exam Builder Saved Exams
» MRCP 1 Session Progress
• Question Browser
Questions Correct 23
• Timed Test A 16-year-old boy is brought to casualty after a fire in his
parents' house. He inhaled a lot of smoke and has a hoarse Questions Incorrect 3
• Mock Exam
voice, stridor and burned nasal hairs. Due to deterioration Questions Total 26
• Past Papers in his peak flow rate and arterial blood gases, he has been
Questions Percentage 88 %
• Random Questions intubated, ventilated and transferred to the intensive care
unit where his condition is now stable. More
• My Performance
• Media Bank In terms of investigation, what is the most important next
step in assessing this boy’s condition?
• New Multimedia
Online Extras CT thorax Your answer
Library Chest X-ray
Community Bronchoscopy Correct answer
Help Ventilation–perfusion scan
PasTest Store Echocardiography

My Account

Profile
Newsletters Inhalation of hot smoke can burn the upper airways
and contributes significantly to deaths due to fires.
My Career and Exams Upper airway obstruction due to heat injury and
Order History mucosal swelling usually develops within 24 h of Reference: Normal Values
exposure, but stenosis due to scarring can develop
Learning Goals later. A hoarse voice, stridor, severe conjunctivitis, Haematology
Question Filters burnt nasal hairs and falling peak flow all suggest
significant upper airway damage. Bronchoscopy is Immunoglobulins
then the best tool to establish whether there is Biochemistry
Security significant oedema or mucosal ulceration obstructing
the airways. Diabetes
Change Password
Sign Out Endocrinology

Blood gases
2043
CSF

© 2011 PasTest Ltd | About Us | Contact Us | Help

file:///E|/Shakhawan/Respiratory%20S/255a.htm[3/13/2012 4:08:24 PM]


MyPasTest

Main Navigation
Question Browser: MRCP 1
Home
Subscriptions Question Browser Exam Builder Saved Exams
» MRCP 1 Session Progress
• Question Browser
Questions Correct 23
• Timed Test A 29-year-old woman noticed shortness of breath and dry
cough while jogging last winter. She now wakes up twice a Questions Incorrect 3
• Mock Exam
week at 4 o’clock in the morning with a troublesome cough. Questions Total 26
• Past Papers On examination there is scattered wheeze and her peak
Questions Percentage 88 %
• Random Questions flow is 460 (530 predicted).
• My Performance More
What is the most appropriate therapy?
• Media Bank
• New Multimedia Salbutamol inhaler
Reference: Normal Values
Online Extras Salbutamol inhaler and inhaled steroids
Oral steroids Haematology
Library
Community Ampicillin Immunoglobulins
Help Theophylline
Biochemistry
PasTest Store 2044 Diabetes
Endocrinology
My Account
Blood gases
Profile
CSF
Newsletters
My Career and Exams
Order History
Learning Goals
Question Filters

Security

Change Password
Sign Out

© 2011 PasTest Ltd | About Us | Contact Us | Help

file:///E|/Shakhawan/Respiratory%20S/256.htm[3/13/2012 4:08:26 PM]


MyPasTest

Main Navigation
Question Browser: MRCP 1
Home
Subscriptions Question Browser Exam Builder Saved Exams
» MRCP 1 Session Progress
• Question Browser
Questions Correct 23
• Timed Test A 29-year-old woman noticed shortness of breath and dry
cough while jogging last winter. She now wakes up twice a Questions Incorrect 4
• Mock Exam
week at 4 o’clock in the morning with a troublesome cough. Questions Total 27
• Past Papers On examination there is scattered wheeze and her peak
Questions Percentage 85 %
• Random Questions flow is 460 (530 predicted).
• My Performance More
What is the most appropriate therapy?
• Media Bank
• New Multimedia Salbutamol inhaler Your answer

Online Extras Salbutamol inhaler and inhaled Correct answer


steroids
Library
Oral steroids
Community
Ampicillin
Help
Theophylline
PasTest Store

My Account

Profile This patient requires step 2 of the British Thoracic


Society (BTS) guidelines for asthma therapy (regular
Newsletters preventer therapy). Inhaled steroids should be
My Career and Exams considered for patients with any of the following:
Order History exacerbations of asthma in the last 2 years Reference: Normal Values
Learning Goals using inhaled β2 -agonists three times a week or Haematology
Question Filters more
being symptomatic three times a week or more, Immunoglobulins
or waking one night a week. Biochemistry
Security
Diabetes
Change Password 2044
Sign Out Endocrinology

Blood gases

CSF

© 2011 PasTest Ltd | About Us | Contact Us | Help

file:///E|/Shakhawan/Respiratory%20S/256a.htm[3/13/2012 4:08:27 PM]


MyPasTest

Main Navigation
Question Browser: MRCP 1
Home
Subscriptions Question Browser Exam Builder Saved Exams
» MRCP 1 Session Progress
• Question Browser
Questions Correct 23
• Timed Test A 48-year-old woman presented with increasing shortness
of breath. She also reported that her abdominal girth had Questions Incorrect 4
• Mock Exam
increased over the past few months but put this down to Questions Total 27
• Past Papers good living. On examination her GP noticed a right-sided
Questions Percentage 85 %
• Random Questions pleural effusion and a very large mass in her right pelvis.
Signs of ascites were also present. CA-125, hCG, CEA and More
• My Performance
a-fetoprotein were unremarkable. A few weeks later a
• Media Bank large ovarian fibroma was removed. Some 6 months later
• New Multimedia she remains well.
Reference: Normal Values
Online Extras Whichdiagnosis best fits with this picture?
Haematology
Library
Ovarian carcinomatosis
Community Immunoglobulins
Heart failure
Help Biochemistry
Cirrhosis
PasTest Store Diabetes
Meigs' syndrome
Rheumatoid arthritis Endocrinology
My Account
Blood gases
Profile 2185
CSF
Newsletters
My Career and Exams
Order History
Learning Goals
Question Filters

Security

Change Password
Sign Out

© 2011 PasTest Ltd | About Us | Contact Us | Help

file:///E|/Shakhawan/Respiratory%20S/257.htm[3/13/2012 4:08:29 PM]


MyPasTest

Main Navigation
Question Browser: MRCP 1
Home
Subscriptions Question Browser Exam Builder Saved Exams
» MRCP 1 Session Progress
• Question Browser
Questions Correct 24
• Timed Test A 48-year-old woman presented with increasing shortness
of breath. She also reported that her abdominal girth had Questions Incorrect 4
• Mock Exam
increased over the past few months but put this down to Questions Total 28
• Past Papers good living. On examination her GP noticed a right-sided
Questions Percentage 85 %
• Random Questions pleural effusion and a very large mass in her right pelvis.
Signs of ascites were also present. CA-125, hCG, CEA and More
• My Performance
a-fetoprotein were unremarkable. A few weeks later a
• Media Bank large ovarian fibroma was removed. Some 6 months later
• New Multimedia she remains well.

Online Extras Whichdiagnosis best fits with this picture?


Library
Ovarian carcinomatosis
Community
Heart failure
Help
Cirrhosis
PasTest Store
Meigs' syndrome Your answer
Rheumatoid arthritis
My Account

Profile
Newsletters
My Career and Exams This syndrome is characterised by the presence of a
benign ovarian fibroma, associated with ascites and a Reference: Normal Values
Order History
right-sided pleural effusion. The average age at
Learning Goals presentation is 48 years. Meigs' syndrome is associated Haematology
Question Filters with 0.004% of ovarian tumours. The aetiology of the
pleural effusion is thought to be related to the size of Immunoglobulins
fibroma, leading to accumulated peritoneal ascites that
flows into the pleural cavity via the lymphatics or via Biochemistry
Security
abdominal–pleural communication (via the foramen of Diabetes
Change Password Bochdalek). Removal of the ovarian mass is associated
with resolution of the ascites and pleural effusion and Endocrinology
Sign Out
an excellent prognosis.
Blood gases

CSF
2185

© 2011 PasTest Ltd | About Us | Contact Us | Help

file:///E|/Shakhawan/Respiratory%20S/257a.htm[3/13/2012 4:08:30 PM]


MyPasTest

Main Navigation
Question Browser: MRCP 1
Home
Subscriptions Question Browser Exam Builder Saved Exams
» MRCP 1 Session Progress
• Question Browser
Questions Correct 24
• Timed Test An adolescent girl has chronic cough and recurrent
respiratory infections over the past two to three Questions Incorrect 4
• Mock Exam
years. Questions Total 28
• Past Papers
Which one of the following pieces of clinical Questions Percentage 85 %
• Random Questions
information in her history would point most strongly More
• My Performance to the development of bronchiectasis?
• Media Bank
• New Multimedia Pale stools and low weight
Reference: Normal Values
Online Extras History of wheeze
Previous whooping cough in early childhood Haematology
Library
Community Serum precipitins to Aspergillus fumigatus Immunoglobulins
Help Pepperpot calcification on chest X-ray (CXR)
Biochemistry
PasTest Store 3384 Diabetes
Endocrinology
My Account
Blood gases
Profile
CSF
Newsletters
My Career and Exams
Order History
Learning Goals
Question Filters

Security

Change Password
Sign Out

© 2011 PasTest Ltd | About Us | Contact Us | Help

file:///E|/Shakhawan/Respiratory%20S/258.htm[3/13/2012 4:08:32 PM]


MyPasTest

Main Navigation
Question Browser: MRCP 1
Home
Subscriptions Question Browser Exam Builder Saved Exams
» MRCP 1 Session Progress
• Question Browser
Questions Correct 24
• Timed Test An adolescent girl has chronic cough and recurrent
respiratory infections over the past two to three Questions Incorrect 5
• Mock Exam
years. Questions Total 29
• Past Papers
Which one of the following pieces of clinical Questions Percentage 82 %
• Random Questions
information in her history would point most strongly More
• My Performance to the development of bronchiectasis?
• Media Bank
• New Multimedia Pale stools and low weight Your answer

Online Extras History of wheeze

Library Previous whooping cough in early Correct answer


childhood
Community
Serum precipitins to Aspergillus
Help fumigatus
PasTest Store Pepperpot calcification on chest X-
ray (CXR)

My Account

Profile
Newsletters Cystic fibrosis, Kartagener's syndrome (situs inversus
My Career and Exams and sinusitis, associated with non-motile cilia) and
previous whooping cough are all associated with Reference: Normal Values
Order History
bronchiectasis. Aspergillus precipitins suggest allergic
Learning Goals bronchopulmonary aspergillosis and underlying asthma. Haematology
Question Filters Pepperpot calcification on CXR suggests previous
varicella infection, which rarely gives rise to further Immunoglobulins
symptoms.
Biochemistry
Security
Diabetes
Change Password 3384
Sign Out Endocrinology

Blood gases

CSF

© 2011 PasTest Ltd | About Us | Contact Us | Help

file:///E|/Shakhawan/Respiratory%20S/258a.htm[3/13/2012 4:08:33 PM]


MyPasTest

Main Navigation
Question Browser: MRCP 1
Home
Subscriptions Question Browser Exam Builder Saved Exams
» MRCP 1 Session Progress
• Question Browser
Questions Correct 0
• Timed Test A 62-year-old man has cryptogenic fibrosing alveolitis
(CFA). Which of the following lung function abnormalities Questions Incorrect 0
• Mock Exam
would be typical of CFA? Questions Total 0
• Past Papers
Questions Percentage 0%
• Random Questions Obstructive lung defect
• My Performance Narrowing of the alveolar–arterial (A–a) gradient on More
• Media Bank exercise

• New Multimedia Increased gas transfer factor


Reference: Normal Values
Online Extras Increased lung elastic recoil
Decreased forced expiratory (FEV 1 /FVC) ratio Haematology
Library
Community Immunoglobulins
3385
Help Biochemistry
PasTest Store Diabetes
Endocrinology
My Account
Blood gases
Profile
CSF
Newsletters
My Career and Exams
Order History
Learning Goals
Question Filters

Security

Change Password
Sign Out

© 2011 PasTest Ltd | About Us | Contact Us | Help

file:///E|/Shakhawan/Respiratory%20S/259.htm[3/13/2012 4:08:35 PM]


MyPasTest

Main Navigation
Question Browser: MRCP 1
Home
Subscriptions Question Browser Exam Builder Saved Exams
» MRCP 1 Session Progress
• Question Browser
Questions Correct 1
• Timed Test A 62-year-old man has cryptogenic fibrosing alveolitis
(CFA). Which of the following lung function abnormalities Questions Incorrect 0
• Mock Exam
would be typical of CFA? Questions Total 1
• Past Papers
Questions Percentage 100 %
• Random Questions Obstructive lung defect
• My Performance Narrowing of the alveolar–arterial (A– More
• Media Bank a) gradient on exercise

• New Multimedia Increased gas transfer factor

Online Extras Increased lung elastic recoil Your answer

Library Decreased forced expiratory


(FEV 1 /FVC) ratio
Community
Help
PasTest Store

Typical lung function abnormalities in CFA include a


My Account decreased FEV1 (forced expiratory volume 1) and FVC
(forced vital capacity) with an increased ratio of
Profile FEV1 /FVC ie a restrictive ratio. Gas transfer factor
Newsletters (TLCO or DLCO) is reduced, as is gas transfer coefficient
My Career and Exams (KCO). Arterial oxygen desaturation on exercise is an
early feature and this is reflected in widening of the A–a
Order History gradient on exercise. CFA lungs are stiff ie poorly Reference: Normal Values
Learning Goals compliant, and therefore if compliance is low then Haematology
elastic recoil will be high as the two are inversely
Question Filters
related. Immunoglobulins
Biochemistry
Security
3385 Diabetes
Change Password
Sign Out Endocrinology

Blood gases

CSF

© 2011 PasTest Ltd | About Us | Contact Us | Help

file:///E|/Shakhawan/Respiratory%20S/259a.htm[3/13/2012 4:08:36 PM]


MyPasTest

Main Navigation
Question Browser: MRCP 1
Home
Subscriptions Question Browser Exam Builder Saved Exams
» MRCP 1 Session Progress
• Question Browser
Questions Correct 1
• Timed Test A 60-year-old man develops a small pneumothorax after a
computerised tomography (CT)-guided biopsy of a left Questions Incorrect 0
• Mock Exam
upper lobe mass. He is asymptomatic. Questions Total 1
• Past Papers
The most appropriate management of his Questions Percentage 100 %
• Random Questions
pneumothorax should be: More
• My Performance
• Media Bank Aspiration
• New Multimedia Conservative only
Reference: Normal Values
Online Extras Intercostal tube drainage
Haematology
Library Intercostal tube drainage and high-pressure suction
Community Surgical pleurodesis Immunoglobulins
Help Biochemistry
3386
PasTest Store Diabetes
Endocrinology
My Account
Blood gases
Profile
CSF
Newsletters
My Career and Exams
Order History
Learning Goals
Question Filters

Security

Change Password
Sign Out

© 2011 PasTest Ltd | About Us | Contact Us | Help

file:///E|/Shakhawan/Respiratory%20S/260.htm[3/13/2012 4:08:38 PM]


MyPasTest

Main Navigation
Question Browser: MRCP 1
Home
Subscriptions Question Browser Exam Builder Saved Exams
» MRCP 1 Session Progress
• Question Browser
Questions Correct 1
• Timed Test A 60-year-old man develops a small pneumothorax after a
computerised tomography (CT)-guided biopsy of a left Questions Incorrect 1
• Mock Exam
upper lobe mass. He is asymptomatic. Questions Total 2
• Past Papers
The most appropriate management of his Questions Percentage 50 %
• Random Questions
pneumothorax should be: More
• My Performance
• Media Bank Aspiration Your answer
• New Multimedia Conservative only Correct answer
Online Extras Intercostal tube drainage
Library Intercostal tube drainage and high-
Community pressure suction

Help Surgical pleurodesis

PasTest Store

My Account
Treatment of pneumothorax remains difficult. In this
Profile case the patient has a small pneumothorax and is well.
He may also be a surgical candidate if biopsy confirms a
Newsletters pulmonary malignancy and tube drainage may make
My Career and Exams this technically more difficult. In this case assuming the
patient remains well a conservative approach with Reference: Normal Values
Order History
observation only is appropriate. If the patient
Learning Goals deteriorates then aspiration +/– intercostal tube Haematology
Question Filters drainage would be the next step.
Immunoglobulins
Biochemistry
Security 3386
Diabetes
Change Password
Sign Out Endocrinology

Blood gases

CSF

© 2011 PasTest Ltd | About Us | Contact Us | Help

file:///E|/Shakhawan/Respiratory%20S/260a.htm[3/13/2012 4:08:39 PM]


MyPasTest

Main Navigation
Question Browser: MRCP 1
Home
Subscriptions Question Browser Exam Builder Saved Exams
» MRCP 1 Session Progress
• Question Browser
Questions Correct 1
• Timed Test Which of the following statements best fits with the
features of spontaneous pneumothorax? Questions Incorrect 1
• Mock Exam
Questions Total 2
• Past Papers
Cigarette smoking is not a risk factor Questions Percentage 50 %
• Random Questions
Simple aspiration is preferred to intercostals tube
• My Performance drainage as an initial procedure where the rim is 2cm More
• Media Bank and the patient has pain

• New Multimedia Recurrence rates following first pneumothorax are as


high as 75% Reference: Normal Values
Online Extras
Risk is greatest during the sixth decade of life Haematology
Library
The male : female ratio is approximately 2 : 1
Community Immunoglobulins
Help 3387 Biochemistry
PasTest Store Diabetes
Endocrinology
My Account
Blood gases
Profile
CSF
Newsletters
My Career and Exams
Order History
Learning Goals
Question Filters

Security

Change Password
Sign Out

© 2011 PasTest Ltd | About Us | Contact Us | Help

file:///E|/Shakhawan/Respiratory%20S/261.htm[3/13/2012 4:08:41 PM]


MyPasTest

Main Navigation
Question Browser: MRCP 1
Home
Subscriptions Question Browser Exam Builder Saved Exams
» MRCP 1 Session Progress
• Question Browser
Questions Correct 2
• Timed Test Which of the following statements best fits with the
features of spontaneous pneumothorax? Questions Incorrect 1
• Mock Exam
Questions Total 3
• Past Papers
Cigarette smoking is not a risk factor Questions Percentage 66 %
• Random Questions
Simple aspiration is preferred to Your answer
• My Performance intercostals tube drainage as an initial More
• Media Bank procedure where the rim is 2cm and the
patient has pain
• New Multimedia
Recurrence rates following first
Online Extras pneumothorax are as high as 75%
Library Risk is greatest during the sixth decade
Community of life
Help The male : female ratio is
approximately 2 : 1
PasTest Store

My Account

Profile Spontaneous pneumothorax tends to occur in young


male smokers, who are usually well and have no
Newsletters
obvious underlying pulmonary disease. Recurrence rates
My Career and Exams are extremely high with studies reporting rates of 30–
Order History 50%. Secondary pneumothorax affects patients with Reference: Normal Values
any underlying pulmonary disease such as chronic
Learning Goals obstructive pulmonary disease (COPD), histiocytosis X Haematology
Question Filters and pulmonary fibrosis.
Immunoglobulins
Biochemistry
Security 3387
Diabetes
Change Password
Sign Out Endocrinology

Blood gases

CSF

© 2011 PasTest Ltd | About Us | Contact Us | Help

file:///E|/Shakhawan/Respiratory%20S/261a.htm[3/13/2012 4:08:42 PM]


MyPasTest

Main Navigation
Question Browser: MRCP 1
Home
Subscriptions Question Browser Exam Builder Saved Exams
» MRCP 1 Session Progress
• Question Browser
Questions Correct 2
• Timed Test A 49-year-old man with unexplained cough undergoes
bronchoscopy and transbronchial biopsy. Histology shows Questions Incorrect 1
• Mock Exam
evidence of neutrophil infiltrate, granulomas are absent. Questions Total 3
• Past Papers Which is the most likely diagnosis?
Questions Percentage 66 %
• Random Questions
• My Performance Histiocytosis X More
• Media Bank Histoplasmosis

• New Multimedia Churg–Strauss syndrome


Reference: Normal Values
Online Extras Berylliosis
Polyarteritis nodosa (PAN) Haematology
Library
Community Immunoglobulins
3388
Help Biochemistry
PasTest Store Diabetes
Endocrinology
My Account
Blood gases
Profile
CSF
Newsletters
My Career and Exams
Order History
Learning Goals
Question Filters

Security

Change Password
Sign Out

© 2011 PasTest Ltd | About Us | Contact Us | Help

file:///E|/Shakhawan/Respiratory%20S/262.htm[3/13/2012 4:08:44 PM]


MyPasTest

Main Navigation
Question Browser: MRCP 1
Home
Subscriptions Question Browser Exam Builder Saved Exams
» MRCP 1 Session Progress
• Question Browser
Questions Correct 2
• Timed Test A 49-year-old man with unexplained cough undergoes
bronchoscopy and transbronchial biopsy. Histology shows Questions Incorrect 2
• Mock Exam
evidence of neutrophil infiltrate, granulomas are absent. Questions Total 4
• Past Papers Which is the most likely diagnosis?
Questions Percentage 50 %
• Random Questions
• My Performance Histiocytosis X More
• Media Bank Histoplasmosis

• New Multimedia Churg–Strauss syndrome

Online Extras Berylliosis Your answer

Library Polyarteritis nodosa (PAN) Correct answer

Community
Help
PasTest Store
Histiocytosis X (eosinophilic granuloma) is a disease of
young adults with widespread nodules in the lungs.
My Account Fibrosis appears early and may progress to respiratory
failure and death. Histoplasmosis is the commonest
Profile fungal infection in North America and has both acute
(usually self-limiting) and chronic (those with previous
Newsletters
lung disease) forms. Churg–Strauss syndrome is
My Career and Exams characterised by a prodromal phase of asthma, a
Order History second phase of tissue and peripheral eosinophilia and Reference: Normal Values
a final phase of systemic vasculitis. PAN affects
Learning Goals medium-sized arteries (cf small arteries in Churg– Haematology
Question Filters Strauss syndrome) and the infiltrate is composed of
neutrophils but granulomas are absent. Berylliosis has Immunoglobulins
features similar to sarcoid.
Biochemistry
Security
Diabetes
Change Password 3388
Sign Out Endocrinology

Blood gases

CSF

© 2011 PasTest Ltd | About Us | Contact Us | Help

file:///E|/Shakhawan/Respiratory%20S/262a.htm[3/13/2012 4:08:45 PM]


MyPasTest

Main Navigation
Question Browser: MRCP 1
Home
Subscriptions Question Browser Exam Builder Saved Exams
» MRCP 1 Session Progress
• Question Browser
Questions Correct 2
• Timed Test A 43-year-old woman has a pleural effusion and evidence
of fibrosis on CXR. She has noticed a facial rash on sun Questions Incorrect 2
• Mock Exam
exposure that has worsened over the past few months. Questions Total 4
• Past Papers Which is the most likely diagnosis?
Questions Percentage 50 %
• Random Questions
• My Performance Systemic lupus erythematosus More
• Media Bank Adenocarcinoma lung

• New Multimedia Wegener's granulomatosis


Reference: Normal Values
Online Extras Progressive massive fibrosis
Actinomycosis Haematology
Library
Community Immunoglobulins
3389
Help Biochemistry
PasTest Store Diabetes
Endocrinology
My Account
Blood gases
Profile
CSF
Newsletters
My Career and Exams
Order History
Learning Goals
Question Filters

Security

Change Password
Sign Out

© 2011 PasTest Ltd | About Us | Contact Us | Help

file:///E|/Shakhawan/Respiratory%20S/263.htm[3/13/2012 4:08:47 PM]


MyPasTest

Main Navigation
Question Browser: MRCP 1
Home
Subscriptions Question Browser Exam Builder Saved Exams
» MRCP 1 Session Progress
• Question Browser
Questions Correct 3
• Timed Test A 43-year-old woman has a pleural effusion and evidence
of fibrosis on CXR. She has noticed a facial rash on sun Questions Incorrect 2
• Mock Exam
exposure that has worsened over the past few months. Questions Total 5
• Past Papers Which is the most likely diagnosis?
Questions Percentage 60 %
• Random Questions
• My Performance Systemic lupus erythematosus Your answer More
• Media Bank Adenocarcinoma lung

• New Multimedia Wegener's granulomatosis

Online Extras Progressive massive fibrosis

Library Actinomycosis

Community
Help
PasTest Store
Direct pulmonary involvement in systemic lupus
erythematosus (SLE) occurs in 30% (pleuropericarditis,
My Account atelectasis, pneumonitis, raised hemidiaphragms and
pulmonary fibrosis). Autoimmune screen with
Profile antinuclear antibody (ANA) and anti-dsDNA antibodies
may be useful in confirmation of the diagnosis. The
Newsletters
other stems are all associated with cavitating lung
My Career and Exams lesions. Wegener's granulomatosis is a necrotising
Order History vasculitis characterised by granulomas in the upper and Reference: Normal Values
lower respiratory tracts and a focal glomerulonephritis.
Learning Goals Progressive massive fibrosis is a complicated coal Haematology
Question Filters workers’ pneumoconiosis where pulmonary nodules
coalesce and cavitate. Actinomycosis is a chronic Immunoglobulins
granulomatous disorder produced by a Gram-positive
Biochemistry
Security anaerobe. Any lung cancer may cavitate if it outgrows
its blood supply but it is typically squamous cell Diabetes
Change Password carcinoma, which cavitates.
Sign Out Endocrinology

Blood gases
3389
CSF

© 2011 PasTest Ltd | About Us | Contact Us | Help

file:///E|/Shakhawan/Respiratory%20S/263a.htm[3/13/2012 4:08:48 PM]


MyPasTest

Main Navigation
Question Browser: MRCP 1
Home
Subscriptions Question Browser Exam Builder Saved Exams
» MRCP 1 Session Progress
• Question Browser
Questions Correct 0
• Timed Test A 62-year-old man with a heavy smoking history presents
with a mass on CXR. Questions Incorrect 0
• Mock Exam
Questions Total 0
• Past Papers Which of the following clinical features might still
permit curative surgical resection for bronchial Questions Percentage 0%
• Random Questions
carcinoma? More
• My Performance
• Media Bank Ipsilateral malignant pleural effusion
• New Multimedia Invasive superior vena caval (SVC) obstruction
Reference: Normal Values
Online Extras FVC <1.5 pre-op
Haematology
Library Left recurrent laryngeal nerve (RLN) palsy
Community Hypercalcaemia Immunoglobulins
Help Biochemistry
3390
PasTest Store Diabetes
Endocrinology
My Account
Blood gases
Profile
CSF
Newsletters
My Career and Exams
Order History
Learning Goals
Question Filters

Security

Change Password
Sign Out

© 2011 PasTest Ltd | About Us | Contact Us | Help

file:///E|/Shakhawan/Respiratory%20S/264.htm[3/13/2012 4:08:50 PM]


MyPasTest

Main Navigation
Question Browser: MRCP 1
Home
Subscriptions Question Browser Exam Builder Saved Exams
» MRCP 1 Session Progress
• Question Browser
Questions Correct 1
• Timed Test A 62-year-old man with a heavy smoking history presents
with a mass on CXR. Questions Incorrect 0
• Mock Exam
Questions Total 1
• Past Papers Which of the following clinical features might still
permit curative surgical resection for bronchial Questions Percentage 100 %
• Random Questions
carcinoma? More
• My Performance
• Media Bank Ipsilateral malignant pleural effusion
• New Multimedia Invasive superior vena caval (SVC)
Online Extras obstruction

Library FVC <1.5 pre-op

Community Left recurrent laryngeal nerve (RLN)


palsy
Help
Hypercalcaemia Your answer
PasTest Store

My Account

Profile Malignant pleural involvement is a sign of inoperability,


as is invasive SVC obstruction (occurs in 4% of cases,
Newsletters usually with small cell) and left RLN palsy (signifies
My Career and Exams mediastinal nodes). FVC of less than 1.5 essentially
means there is unlikely to be sufficient reserve after Reference: Normal Values
Order History
pneumonectomy. Hypercalcaemia may be due to bony
Learning Goals secondaries or tumour production of parathyroid Haematology
Question Filters hormone (PTH)-like peptide, the latter of which may be
operable. Immunoglobulins
Biochemistry
Security
3390 Diabetes
Change Password
Sign Out Endocrinology

Blood gases

CSF

© 2011 PasTest Ltd | About Us | Contact Us | Help

file:///E|/Shakhawan/Respiratory%20S/264a.htm[3/13/2012 4:08:51 PM]


MyPasTest

Main Navigation
Question Browser: MRCP 1
Home
Subscriptions Question Browser Exam Builder Saved Exams
» MRCP 1 Session Progress
• Question Browser
Questions Correct 1
• Timed Test A 22-year-old student attends the clinic for his 6-week
check after admission with pneumonia. Which of the Questions Incorrect 0
• Mock Exam
following complications of pneumonia is most likely to be a Questions Total 1
• Past Papers chronic rather than an acute complication of the infective
Questions Percentage 100 %
• Random Questions process?
• My Performance More
Bronchiectasis
• Media Bank
Bronchopleural fistula
• New Multimedia
Empyema Reference: Normal Values
Online Extras
Lung abscess Haematology
Library
Organising pneumonia
Community Immunoglobulins
Help 3391 Biochemistry
PasTest Store Diabetes
Endocrinology
My Account
Blood gases
Profile
CSF
Newsletters
My Career and Exams
Order History
Learning Goals
Question Filters

Security

Change Password
Sign Out

© 2011 PasTest Ltd | About Us | Contact Us | Help

file:///E|/Shakhawan/Respiratory%20S/265.htm[3/13/2012 4:08:53 PM]


MyPasTest

Main Navigation
Question Browser: MRCP 1
Home
Subscriptions Question Browser Exam Builder Saved Exams
» MRCP 1 Session Progress
• Question Browser
Questions Correct 2
• Timed Test A 22-year-old student attends the clinic for his 6-week
check after admission with pneumonia. Which of the Questions Incorrect 0
• Mock Exam
following complications of pneumonia is most likely to be a Questions Total 2
• Past Papers chronic rather than an acute complication of the infective
Questions Percentage 100 %
• Random Questions process?
• My Performance More
Bronchiectasis Your answer
• Media Bank
Bronchopleural fistula
• New Multimedia
Empyema
Online Extras
Lung abscess
Library
Organising pneumonia
Community
Help
PasTest Store
The acute complications of pneumonia may be obvious
My Account at presentation or may have a delayed presentation
(empyema, abscess, fistula and organising pneumonia).
Profile The diagnosis of bronchiectasis can only be made
subsequent to the illness as temporary or reversible
Newsletters
bronchial dilatation may be seen during the acute
My Career and Exams illness.
Order History Reference: Normal Values
Learning Goals 3391 Haematology
Question Filters
Immunoglobulins
Biochemistry
Security
Diabetes
Change Password
Sign Out Endocrinology

Blood gases

CSF

© 2011 PasTest Ltd | About Us | Contact Us | Help

file:///E|/Shakhawan/Respiratory%20S/265a.htm[3/13/2012 4:08:54 PM]


MyPasTest

Main Navigation
Question Browser: MRCP 1
Home
Subscriptions Question Browser Exam Builder Saved Exams
» MRCP 1 Session Progress
• Question Browser
Questions Correct 2
• Timed Test A 37-year-old man presents with increasing breathlessness
and is diagnosed after investigation with emphysema. Questions Incorrect 0
• Mock Exam
Questions Total 2
• Past Papers Which of the following conditions is most likely to
predispose to the development of emphysema? Questions Percentage 100 %
• Random Questions
• My Performance More
Allergic bronchopulmonary aspergillosis
• Media Bank
Hypogammaglobulinaemia
• New Multimedia
Childhood bronchiolitis Reference: Normal Values
Online Extras
Bronchial adenoma
Haematology
Library
Crohn’s disease
Community Immunoglobulins
Help 3392 Biochemistry
PasTest Store Diabetes
Endocrinology
My Account
Blood gases
Profile
CSF
Newsletters
My Career and Exams
Order History
Learning Goals
Question Filters

Security

Change Password
Sign Out

© 2011 PasTest Ltd | About Us | Contact Us | Help

file:///E|/Shakhawan/Respiratory%20S/266.htm[3/13/2012 4:08:56 PM]


MyPasTest

Main Navigation
Question Browser: MRCP 1
Home
Subscriptions Question Browser Exam Builder Saved Exams
» MRCP 1 Session Progress
• Question Browser
Questions Correct 3
• Timed Test A 37-year-old man presents with increasing breathlessness
and is diagnosed after investigation with emphysema. Questions Incorrect 0
• Mock Exam
Questions Total 3
• Past Papers Which of the following conditions is most likely to
predispose to the development of emphysema? Questions Percentage 100 %
• Random Questions
• My Performance More
Allergic bronchopulmonary aspergillosis
• Media Bank
Hypogammaglobulinaemia
• New Multimedia
Childhood bronchiolitis Your answer
Online Extras
Bronchial adenoma
Library
Crohn’s disease
Community
Help
PasTest Store
MacLeod’s syndrome is unilateral emphysema following
My Account childhood bronchiolitis. Emphysema is most commonly
related to smoking, although it may also be associated
Profile to a 1 -antitrypsin deficiency. The other stems are
Newsletters associated with bronchiectasis. Bronchiectasis may
occur as a consequence of bronchial obstruction either
My Career and Exams by intrinsic or extrinsic pathology. Infections such as
Order History tuberculosis and previous pneumonia may lead to Reference: Normal Values
bronchiectasis. Congenital syndrome such as cystic
Learning Goals fibrosis, immotile cilia syndrome and Kartagener’s Haematology
Question Filters syndrome may also be implicated in the development of
bronchiectasis. Inflammatory bowel disease has also Immunoglobulins
been associated with development of bronchiectasis. Biochemistry
Security
Diabetes
Change Password 3392
Sign Out Endocrinology

Blood gases

CSF

© 2011 PasTest Ltd | About Us | Contact Us | Help

file:///E|/Shakhawan/Respiratory%20S/266a.htm[3/13/2012 4:08:57 PM]


MyPasTest

Main Navigation
Question Browser: MRCP 1
Home
Subscriptions Question Browser Exam Builder Saved Exams
» MRCP 1 Session Progress
• Question Browser
Questions Correct 3
• Timed Test Which of the following treatments have been shown
to prolong life in patients with COPD? Questions Incorrect 0
• Mock Exam
Questions Total 3
• Past Papers
Anticholinergics Questions Percentage 100 %
• Random Questions
b 2 -agonists
• My Performance More
Inhaled corticosteroids
• Media Bank
Long-term oxygen therapy
• New Multimedia
Lung volume reduction surgery Reference: Normal Values
Online Extras
3393 Haematology
Library
Community Immunoglobulins
Help Biochemistry
PasTest Store Diabetes
Endocrinology
My Account
Blood gases
Profile
CSF
Newsletters
My Career and Exams
Order History
Learning Goals
Question Filters

Security

Change Password
Sign Out

© 2011 PasTest Ltd | About Us | Contact Us | Help

file:///E|/Shakhawan/Respiratory%20S/267.htm[3/13/2012 4:08:59 PM]


MyPasTest

Main Navigation
Question Browser: MRCP 1
Home
Subscriptions Question Browser Exam Builder Saved Exams
» MRCP 1 Session Progress
• Question Browser
Questions Correct 4
• Timed Test Which of the following treatments have been shown
Questions Incorrect 0
• Mock Exam to prolong life in patients with COPD?
Questions Total 4
• Past Papers
Anticholinergics Questions Percentage 100 %
• Random Questions
b 2 -agonists
• My Performance More
Inhaled corticosteroids
• Media Bank
Long-term oxygen therapy Your answer
• New Multimedia
Lung volume reduction surgery
Online Extras
Library
Community
Help The only treatment shown to prolong life in COPD is
PasTest Store long-term oxygen therapy. It must be taken in excess
of 16 hrs or more per day. It is thought that by
relieving hypoxia there is a reduction in pulmonary
My Account arterial pressure and hence prolongation of life. Suitable
candidates are patients with an FEV1 < 1.5 litres and a
Profile p a (O2 ) < 7.3 kPa when well on two occasions some
Newsletters weeks apart.
My Career and Exams
Order History Reference: Normal Values
3393
Learning Goals Haematology
Question Filters
Immunoglobulins
Biochemistry
Security
Diabetes
Change Password
Sign Out Endocrinology

Blood gases

CSF

© 2011 PasTest Ltd | About Us | Contact Us | Help

file:///E|/Shakhawan/Respiratory%20S/267a.htm[3/13/2012 4:09:00 PM]


MyPasTest

Main Navigation
Question Browser: MRCP 1
Home
Subscriptions Question Browser Exam Builder Saved Exams
» MRCP 1 Session Progress
• Question Browser
Questions Correct 4
• Timed Test A 64-year-old cigarette smoker presents with increasing
breathlessness. Examination confirms basal crackles in the Questions Incorrect 0
• Mock Exam
absence of finger clubbing. His CXR shows widespread Questions Total 4
• Past Papers pleural plaques with additional reticular shadowing at the
Questions Percentage 100 %
• Random Questions bases. Lung function tests confirm mild restriction with a
low TLCO. Which of the following is likely to be true? More
• My Performance
• Media Bank Inhaled steroids may prevent disease progression
• New Multimedia Open lung biopsy is mandatory to establish a Reference: Normal Values
Online Extras histological diagnosis
Oral corticosteroids should lead to a significant Haematology
Library
improvement in lung function
Community Immunoglobulins
The condition is slowly progressive
Help Biochemistry
The most likely diagnosis is extrinsic allergic alveolitis
PasTest Store Diabetes
3394
Endocrinology
My Account
Blood gases
Profile
CSF
Newsletters
My Career and Exams
Order History
Learning Goals
Question Filters

Security

Change Password
Sign Out

© 2011 PasTest Ltd | About Us | Contact Us | Help

file:///E|/Shakhawan/Respiratory%20S/268.htm[3/13/2012 4:09:02 PM]


MyPasTest

Main Navigation
Question Browser: MRCP 1
Home
Subscriptions Question Browser Exam Builder Saved Exams
» MRCP 1 Session Progress
• Question Browser
Questions Correct 5
• Timed Test A 64-year-old cigarette smoker presents with increasing
breathlessness. Examination confirms basal crackles in the Questions Incorrect 0
• Mock Exam
absence of finger clubbing. His CXR shows widespread Questions Total 5
• Past Papers pleural plaques with additional reticular shadowing at the
Questions Percentage 100 %
• Random Questions bases. Lung function tests confirm mild restriction with a
low TLCO. Which of the following is likely to be true? More
• My Performance
• Media Bank Inhaled steroids may prevent disease
• New Multimedia progression
Online Extras Open lung biopsy is mandatory to
establish a histological diagnosis
Library
Oral corticosteroids should lead to a
Community significant improvement in lung function
Help The condition is slowly progressive Your answer
PasTest Store The most likely diagnosis is extrinsic
allergic alveolitis
My Account

Profile
Newsletters
Pulmonary asbestosis usually affects men with a history
My Career and Exams of heavy asbestos exposure. It presents with
Order History progressive breathlessness and is resistant to treatment Reference: Normal Values
with immunosuppressive therapy. The changes typically
Learning Goals affect the lower lobes and CT scanning can be useful in Haematology
Question Filters detecting disease. Biopsy is not mandatory as the
diagnosis can be made on clinical and radiological Immunoglobulins
grounds. The only advice is to avoid further exposure
Biochemistry
Security and to stop smoking (>50´ risk of lung cancer if a
smoker and asbestos exposure compared to exposure Diabetes
Change Password to neither). Clubbing occurs in 43% cases of asbestosis.
Sign Out Endocrinology

Blood gases
3394
CSF

© 2011 PasTest Ltd | About Us | Contact Us | Help

file:///E|/Shakhawan/Respiratory%20S/268a.htm[3/13/2012 4:09:03 PM]


MyPasTest

Main Navigation
Question Browser: MRCP 1
Home
Subscriptions Question Browser Exam Builder Saved Exams
» MRCP 1 Session Progress
• Question Browser
Questions Correct 5
• Timed Test A 78-year-old White woman presents with left upper lobe
cavitating consolidation and sputum samples confirm the Questions Incorrect 0
• Mock Exam
presence of Mycobacterium tuberculosis, which is fully Questions Total 5
• Past Papers sensitive. There is no previous history of treatment for
Questions Percentage 100 %
• Random Questions tuberculosis (TB). The most appropriate antibiotic regimen
is? More
• My Performance
• Media Bank Bacillus Calmette–Guerin (BCG) vaccination
• New Multimedia Rifampicin/clarithromycin 12 months Reference: Normal Values
Online Extras Rifampicin/ethambutol 12 months
Haematology
Library Rifampicin/isoniazid/pyrazinamide/ethambutol 2
Community months then rifampicin/isoniazid 4 months Immunoglobulins
Help Rifampicin/isoniazid/pyrazinamide 4 months then Biochemistry
rifampicin/isoniazid 8 months
PasTest Store Diabetes
3395
Endocrinology
My Account
Blood gases
Profile
CSF
Newsletters
My Career and Exams
Order History
Learning Goals
Question Filters

Security

Change Password
Sign Out

© 2011 PasTest Ltd | About Us | Contact Us | Help

file:///E|/Shakhawan/Respiratory%20S/269.htm[3/13/2012 4:09:05 PM]


MyPasTest

Main Navigation
Question Browser: MRCP 1
Home
Subscriptions Question Browser Exam Builder Saved Exams
» MRCP 1 Session Progress
• Question Browser
Questions Correct 6
• Timed Test A 78-year-old White woman presents with left upper lobe
cavitating consolidation and sputum samples confirm the Questions Incorrect 0
• Mock Exam
presence of Mycobacterium tuberculosis, which is fully sensitive. Questions Total 6
• Past Papers There is no previous history of treatment for tuberculosis (TB).
Questions Percentage 100 %
• Random Questions The most appropriate antibiotic regimen is?
• My Performance More
Bacillus Calmette–Guerin (BCG) vaccination
• Media Bank
Rifampicin/clarithromycin 12 months
• New Multimedia
Rifampicin/ethambutol 12 months
Online Extras
Your answer
Library Rifampicin/isoniazid/pyrazinamide/ethambutol
Community 2 months then rifampicin/isoniazid 4 months
Help Rifampicin/isoniazid/pyrazinamide 4 months
then rifampicin/isoniazid 8 months
PasTest Store

My Account

Profile In a white patient with pulmonary Mycobacterium


tuberculosis that is fully sensitive, and not previously treated,
Newsletters
the use of four drugs for 2 months
My Career and Exams (rifampicin/isoniazid/pyrazinamide/ethambutol) and then two
Order History drugs (rifampicin/isoniazid) for 4 months is perfectly Reference: Normal Values
acceptable. Indications for using a four-drug regimen include
Learning Goals previously treated tuberculosis, HIV risk factor(s) or being Haematology
Question Filters non-White.
Immunoglobulins

Biochemistry
Security 3395
Diabetes
Change Password
Sign Out Endocrinology

Blood gases

CSF

© 2011 PasTest Ltd | About Us | Contact Us | Help

file:///E|/Shakhawan/Respiratory%20S/269a.htm[3/13/2012 4:09:07 PM]


MyPasTest

Main Navigation
Question Browser: MRCP 1
Home
Subscriptions Question Browser Exam Builder Saved Exams
» MRCP 1 Session Progress
• Question Browser
Questions Correct 6
• Timed Test A 48-year-old woman presents with a pleural effusion. You
perform a diagnostic pleural aspiration. Which of the Questions Incorrect 0
• Mock Exam
following is true regarding the results that you receive from Questions Total 6
• Past Papers the laboratory?
Questions Percentage 100 %
• Random Questions
• My Performance Low levels of salivary amylase suggest oesophageal More
rupture
• Media Bank
Heavy blood staining effectively excludes pulmonary
• New Multimedia embolic disease Reference: Normal Values
Online Extras The presence of antinuclear factor is virtually
diagnostic of scleroderma Haematology
Library
Community High glucose levels occur in rheumatoid arthritis Immunoglobulins
Help An eosinophilia makes malignancy less likely Biochemistry
PasTest Store 3396 Diabetes
Endocrinology
My Account
Blood gases
Profile
CSF
Newsletters
My Career and Exams
Order History
Learning Goals
Question Filters

Security

Change Password
Sign Out

© 2011 PasTest Ltd | About Us | Contact Us | Help

file:///E|/Shakhawan/Respiratory%20S/270.htm[3/13/2012 4:09:08 PM]


MyPasTest

Main Navigation
Question Browser: MRCP 1
Home
Subscriptions Question Browser Exam Builder Saved Exams
» MRCP 1 Session Progress
• Question Browser
Questions Correct 6
• Timed Test A 48-year-old woman presents with a pleural effusion. You
perform a diagnostic pleural aspiration. Which of the Questions Incorrect 1
• Mock Exam
following is true regarding the results that you receive from Questions Total 7
• Past Papers the laboratory?
Questions Percentage 85 %
• Random Questions
• My Performance Low levels of salivary amylase More
suggest oesophageal rupture
• Media Bank
Heavy blood staining effectively Your answer
• New Multimedia excludes pulmonary embolic disease
Online Extras The presence of antinuclear factor is
Library virtually diagnostic of scleroderma
Community High glucose levels occur in
rheumatoid arthritis
Help
An eosinophilia makes malignancy Correct answer
PasTest Store less likely

My Account

Profile
Heavily blood-stained fluid in the absence of trauma
Newsletters
suggests pulmonary infarction or malignancy. Low
My Career and Exams glucose levels occur in rheumatoid arthritis, TB,
Order History empyema and malignancy. Pleural fluid eosinophilia Reference: Normal Values
(>10%) makes malignancy and TB less likely, and
Learning Goals suggests air in the pleural cavity. High levels of salivary Haematology
Question Filters amylase suggest oesophageal rupture. Presence of
antinuclear factor is virtually diagnostic of SLE not Immunoglobulins
scleroderma.
Biochemistry
Security
Diabetes
Change Password 3396
Sign Out Endocrinology

Blood gases

CSF

© 2011 PasTest Ltd | About Us | Contact Us | Help

file:///E|/Shakhawan/Respiratory%20S/270a.htm[3/13/2012 4:09:10 PM]


MyPasTest

Main Navigation
Question Browser: MRCP 1
Home
Subscriptions Question Browser Exam Builder Saved Exams
» MRCP 1 Session Progress
• Question Browser
Questions Correct 6
• Timed Test Which of the following statements fits best with α1 -
Questions Incorrect 1
• Mock Exam antitrypsin (AAT) deficiency?
Questions Total 7
• Past Papers
AAT deficiency is never associated with asthma and Questions Percentage 85 %
• Random Questions bronchiectasis
• My Performance More
AAT is an autosomal co-dominant condition
• Media Bank Emphysema with a late presentation is the major
• New Multimedia respiratory complication
Reference: Normal Values
Online Extras Severe deficiency affects the lungs and kidney
predominantly Haematology
Library
The main function of AAT is to neutralise eosinophil Immunoglobulins
Community
elastase in the lung
Help Biochemistry
PasTest Store 3397
Diabetes
Endocrinology
My Account
Blood gases
Profile
CSF
Newsletters
My Career and Exams
Order History
Learning Goals
Question Filters

Security

Change Password
Sign Out

© 2011 PasTest Ltd | About Us | Contact Us | Help

file:///E|/Shakhawan/Respiratory%20S/271.htm[3/13/2012 4:09:11 PM]


MyPasTest

Main Navigation
Question Browser: MRCP 1
Home
Subscriptions Question Browser Exam Builder Saved Exams
» MRCP 1 Session Progress
• Question Browser
Questions Correct 6
• Timed Test Which of the following statements fits best with α1 -
Questions Incorrect 2
• Mock Exam antitrypsin (AAT) deficiency?
Questions Total 8
• Past Papers
AAT deficiency is never associated Questions Percentage 75 %
• Random Questions with asthma and bronchiectasis
• My Performance More
AAT is an autosomal co-dominant Correct answer
• Media Bank condition
• New Multimedia Emphysema with a late presentation
is the major respiratory complication
Online Extras
Severe deficiency affects the lungs
Library
and kidney predominantly
Community
The main function of AAT is to Your answer
Help neutralise eosinophil elastase in the
PasTest Store lung

My Account

Profile AAT deficiency is an autosomal co-dominant condition


Newsletters with the gene located on the long arm of chromosome
14. AAT neutralises neutrophil elastase thereby
My Career and Exams
preventing lung destruction. Deficiency (Pi ZZ
Order History homozygotes) results in premature emphysema. Reference: Normal Values
Learning Goals Asthma and bronchiectasis do occur but less frequently
than emphysema. Haematology
Question Filters
Immunoglobulins

3397 Biochemistry
Security
Diabetes
Change Password
Sign Out Endocrinology

Blood gases

CSF

© 2011 PasTest Ltd | About Us | Contact Us | Help

file:///E|/Shakhawan/Respiratory%20S/271a.htm[3/13/2012 4:09:13 PM]


MyPasTest

Main Navigation
Question Browser: MRCP 1
Home
Subscriptions Question Browser Exam Builder Saved Exams
» MRCP 1
• Question Browser
• Timed Test A 29-year-old breathless Afro-Caribbean woman is referred by the ophthalmologists
• Mock Exam with anterior uveitis and a suspected diagnosis of sarcoidosis.
• Past Papers Which of the following clinical features is most strongly associated with
• Random Questions sarcoidosis?
• My Performance
Obstructive defect on spirometry
• Media Bank
A slowly worsening picture of breathlessness with no periods of improvement
• New Multimedia
Bronchoalveolar lavage shows an eosinophilia
Online Extras
Positive Mantoux test
Library
Decreased gas transfer factor (TLCO) with decreased gas transfer coefficient
Community (KCO)
Help
3398
PasTest Store

My Account

Profile
Newsletters
My Career and Exams
Order History
Learning Goals
Question Filters

Security

Change Password
Sign Out

file:///E|/Shakhawan/Respiratory%20S/272.htm[3/13/2012 4:09:14 PM]


MyPasTest

Main Navigation
Question Browser: MRCP 1
Home
Subscriptions Question Browser Exam Builder Saved Exams
» MRCP 1 Session Progress
• Question Browser
Questions Correct 7
• Timed Test A 29-year-old breathless Afro-Caribbean woman is referred
by the ophthalmologists with anterior uveitis and a Questions Incorrect 2
• Mock Exam
suspected diagnosis of sarcoidosis. Questions Total 9
• Past Papers
Which of the following clinical features is most Questions Percentage 77 %
• Random Questions
strongly associated with sarcoidosis? More
• My Performance
• Media Bank Obstructive defect on spirometry
• New Multimedia A slowly worsening picture of
Online Extras breathlessness with no periods of
improvement
Library
Bronchoalveolar lavage shows an
Community eosinophilia
Help Positive Mantoux test
PasTest Store Decreased gas transfer factor (TLCO) Your answer
with decreased gas transfer coefficient
(KCO)
My Account

Profile
Newsletters
My Career and Exams Spirometry usually shows a restrictive defect.
Spontaneous remission of respiratory symptoms is not Reference: Normal Values
Order History
uncommon and may afford a ‘wait-and-see’ policy to be
Learning Goals adopted at the outset before embarking on Haematology
Question Filters immunosuppressive therapy. Bronchoalveolar lavage
typically shows a lymphocytosis. Mantoux or Heaf Immunoglobulins
testing is usually negative and reflects cutaneous
anergy. Normal serum angiotensin-converting enzyme Biochemistry
Security
(ACE) level does not exclude sarcoidosis but when Diabetes
Change Password elevated, may reflect disease activity. Decreased gas
transfer factor (TLCO) accompanied by elevated gas Endocrinology
Sign Out
transfer coefficient (KCO) is characteristic of
extrathoracic restriction not intrapulmonary restriction: Blood gases
both are usually decreased in intrapulmonary
CSF
sarcoidosis.

3398

© 2011 PasTest Ltd | About Us | Contact Us | Help

file:///E|/Shakhawan/Respiratory%20S/272a.htm[3/13/2012 4:09:16 PM]


MyPasTest

Main Navigation
Question Browser: MRCP 1
Home
Subscriptions Question Browser Exam Builder Saved Exams
» MRCP 1 Session Progress
• Question Browser
Questions Correct 7
• Timed Test A 45-year-old man who races pigeons becomes breathless.
Which of the following features is suggestive of extrinsic Questions Incorrect 2
• Mock Exam
allergic alveolitis? Questions Total 9
• Past Papers
Questions Percentage 77 %
• Random Questions Almost immediate onset after exposure
• My Performance Eosinophilia of sputum More
• Media Bank Positive skinprick testing
• New Multimedia Type 1 hypersensitivity reaction
Reference: Normal Values
Online Extras Circulating IgG precipitins
Haematology
Library
3399
Community Immunoglobulins
Help Biochemistry
PasTest Store Diabetes
Endocrinology
My Account
Blood gases
Profile
CSF
Newsletters
My Career and Exams
Order History
Learning Goals
Question Filters

Security

Change Password
Sign Out

© 2011 PasTest Ltd | About Us | Contact Us | Help

file:///E|/Shakhawan/Respiratory%20S/273.htm[3/13/2012 4:09:17 PM]


MyPasTest

Main Navigation
Question Browser: MRCP 1
Home
Subscriptions Question Browser Exam Builder Saved Exams
» MRCP 1 Session Progress
• Question Browser
Questions Correct 7
• Timed Test A 45-year-old man who races pigeons becomes breathless.
Which of the following features is suggestive of extrinsic Questions Incorrect 3
• Mock Exam
allergic alveolitis? Questions Total 10
• Past Papers
Questions Percentage 70 %
• Random Questions Almost immediate onset after
• My Performance exposure More
• Media Bank Eosinophilia of sputum Your answer

• New Multimedia Positive skinprick testing

Online Extras Type 1 hypersensitivity reaction

Library Circulating IgG precipitins Correct answer

Community
Help
PasTest Store
Extrinsic allergic alveolitis (EAA) is characterised by type
3 (immune-complex) and type 4 (cell-mediated)
My Account hypersensitivity reactions to inhaled antigen(s). Despite
its name, EAA is not allergic and therefore items
Profile associated with allergy tend to be false in EAA eg
wheeze, immediate symptoms (not a type 1 reaction),
Newsletters
raised IgE, positive skinprick test, eosinophilia of blood
My Career and Exams or sputum. In the acute form fever, cough and marked
Order History shortness of breath (SOB) occurs between 4–6 h after Reference: Normal Values
exposure. Subacutely there is weight loss and fatigue
Learning Goals and in the chronic form exertional SOB and pulmonary Haematology
Question Filters fibrosis (typically upper lobe). The CXR shows fine
reticular/nodular shadowing progressing eventually to a Immunoglobulins
fibrotic pattern with shrunken lungs.
Biochemistry
Security
Diabetes
Change Password 3399
Sign Out Endocrinology

Blood gases

CSF

© 2011 PasTest Ltd | About Us | Contact Us | Help

file:///E|/Shakhawan/Respiratory%20S/273a.htm[3/13/2012 4:09:19 PM]


MyPasTest

Main Navigation
Question Browser: MRCP 1
Home
Subscriptions Question Browser Exam Builder Saved Exams
» MRCP 1 Session Progress
• Question Browser
Questions Correct 7
• Timed Test The parents (both cystic fibrosis gene carriers) of a child
with cystic fibrosis (CF) come to see you for advice after Questions Incorrect 3
• Mock Exam
reading about CF on the Internet. Which of the following Questions Total 10
• Past Papers bits of information from their Internet printout is correct?
Questions Percentage 70 %
• Random Questions
• My Performance The gene defect is a mutation on chromosome 6 More
• Media Bank There is evidence of pulmonary disease at birth

• New Multimedia <50% of patients survive to adulthood


Reference: Normal Values
Online Extras Burkholderia cepacia is a significant pathogen
Probability of a further child being affected is 50% Haematology
Library
Community Immunoglobulins
3400
Help Biochemistry
PasTest Store Diabetes
Endocrinology
My Account
Blood gases
Profile
CSF
Newsletters
My Career and Exams
Order History
Learning Goals
Question Filters

Security

Change Password
Sign Out

© 2011 PasTest Ltd | About Us | Contact Us | Help

file:///E|/Shakhawan/Respiratory%20S/274.htm[3/13/2012 4:09:20 PM]


MyPasTest

Main Navigation
Question Browser: MRCP 1
Home
Subscriptions Question Browser Exam Builder Saved Exams
» MRCP 1 Session Progress
• Question Browser
Questions Correct 8
• Timed Test The parents (both cystic fibrosis gene carriers) of a child
with cystic fibrosis (CF) come to see you for advice after Questions Incorrect 3
• Mock Exam
reading about CF on the Internet. Which of the following Questions Total 11
• Past Papers bits of information from their Internet printout is correct?
Questions Percentage 72 %
• Random Questions
• My Performance The gene defect is a mutation on More
chromosome 6
• Media Bank
There is evidence of pulmonary disease
• New Multimedia at birth
Online Extras <50% of patients survive to adulthood
Library Burkholderia cepacia is a significant Your answer
Community pathogen
Help Probability of a further child being
affected is 50%
PasTest Store

My Account

Profile The gene defect in cystic fibrosis is a mutation on the


long arm of chromosome 7. Pulmonary disease develops
Newsletters
within the first few months. The 80% survival curve for
My Career and Exams the 1990s approaches 30 years. Burkholderia cepacia is
Order History seen in severe lung disease and is often associated with Reference: Normal Values
a rapid deterioration. Pneumothorax is seen in up to 5%
Learning Goals of patients over 10 years of age and approximately Haematology
Question Filters 50% recur. The probability of a child born to two
carriers being affected is 25% as CF is an autosomal Immunoglobulins
recessive condition.
Biochemistry
Security
Diabetes
Change Password 3400
Sign Out Endocrinology

Blood gases

CSF

© 2011 PasTest Ltd | About Us | Contact Us | Help

file:///E|/Shakhawan/Respiratory%20S/274a.htm[3/13/2012 4:09:22 PM]


MyPasTest

Main Navigation
Question Browser: MRCP 1
Home
Subscriptions Question Browser Exam Builder Saved Exams
» MRCP 1 Session Progress
• Question Browser
Questions Correct 8
• Timed Test Which of the following investigations is most specific
to allergic bronchopulmonary aspergillosis (ABPA)? Questions Incorrect 3
• Mock Exam
Questions Total 11
• Past Papers
Positive history of exposure to Aspergillus Questions Percentage 72 %
• Random Questions
An early positive skinprick test for Aspergillus
• My Performance fumigatus More
• Media Bank CT evidence of proximal bronchiectasis
• New Multimedia Positive precipitins for Aspergillus
Reference: Normal Values
Online Extras Upper zone fibrosis
Haematology
Library
3401
Community Immunoglobulins
Help Biochemistry
PasTest Store Diabetes
Endocrinology
My Account
Blood gases
Profile
CSF
Newsletters
My Career and Exams
Order History
Learning Goals
Question Filters

Security

Change Password
Sign Out

© 2011 PasTest Ltd | About Us | Contact Us | Help

file:///E|/Shakhawan/Respiratory%20S/275.htm[3/13/2012 4:09:23 PM]


MyPasTest

Main Navigation
Question Browser: MRCP 1
Home
Subscriptions Question Browser Exam Builder Saved Exams
» MRCP 1 Session Progress
• Question Browser
Questions Correct 8
• Timed Test Which of the following investigations is most specific
to allergic bronchopulmonary aspergillosis (ABPA)? Questions Incorrect 4
• Mock Exam
Questions Total 12
• Past Papers
Positive history of exposure to Questions Percentage 66 %
• Random Questions Aspergillus
• My Performance An early positive skinprick test for Correct answer More
• Media Bank Aspergillus fumigatus

• New Multimedia CT evidence of proximal


bronchiectasis
Online Extras
Positive precipitins for Aspergillus
Library
Upper zone fibrosis Your answer
Community
Help
PasTest Store
ABPA is characterised by wheezy breathlessness, serum
My Account eosinophilia and pulmonary infiltrates on X-ray. Lobar
collapse may also be caused by mucus plugging.
Profile Exposure history is very difficult to elicit. Precipitins
(IgG) are more usual with an aspergilloma but may be
Newsletters
positive in ABPI or in up to 10% of patients with
My Career and Exams asthma. Positive skinprick tests reflect antigen-specific
Order History IgE. Reference: Normal Values
Learning Goals Haematology
Question Filters 3401
Immunoglobulins
Biochemistry
Security
Diabetes
Change Password
Sign Out Endocrinology

Blood gases

CSF

© 2011 PasTest Ltd | About Us | Contact Us | Help

file:///E|/Shakhawan/Respiratory%20S/275a.htm[3/13/2012 4:09:25 PM]


MyPasTest

Main Navigation
Question Browser: MRCP 1
Home
Subscriptions Question Browser Exam Builder Saved Exams
» MRCP 1 Session Progress
• Question Browser
Questions Correct 8
• Timed Test A 58-year-old man is referred by his GP with probable
obstructive sleep apnoea (OSA). Which of the following Questions Incorrect 4
• Mock Exam
features is most strongly associated with OSA? Questions Total 12
• Past Papers
Questions Percentage 66 %
• Random Questions Normal blood pressure
• My Performance Normal oxygen saturations at night More
• Media Bank Epworth sleepiness score of 6
• New Multimedia Body mass index of 26
Reference: Normal Values
Online Extras Daytime somnolence
Haematology
Library
3402
Community Immunoglobulins
Help Biochemistry
PasTest Store Diabetes
Endocrinology
My Account
Blood gases
Profile
CSF
Newsletters
My Career and Exams
Order History
Learning Goals
Question Filters

Security

Change Password
Sign Out

© 2011 PasTest Ltd | About Us | Contact Us | Help

file:///E|/Shakhawan/Respiratory%20S/276.htm[3/13/2012 4:09:26 PM]


MyPasTest

Main Navigation
Question Browser: MRCP 1
Home
Subscriptions Question Browser Exam Builder Saved Exams
» MRCP 1 Session Progress
• Question Browser
Questions Correct 9
• Timed Test A 58-year-old man is referred by his GP with probable
obstructive sleep apnoea (OSA). Which of the following Questions Incorrect 4
• Mock Exam
features is most strongly associated with OSA? Questions Total 13
• Past Papers
Questions Percentage 69 %
• Random Questions Normal blood pressure
• My Performance Normal oxygen saturations at night More
• Media Bank Epworth sleepiness score of 6
• New Multimedia Body mass index of 26
Online Extras Daytime somnolence Your answer
Library
Community
Help
PasTest Store OSA is associated with obesity (of which hypothyroidism
is a cause) and sedatives such as alcohol. It is
associated with hypertension. The disorder is most
My Account marked during REM sleep. Epworth sleepiness score can
be 0–24 (the higher the score, the more sleepy the
Profile patient is) and 6/24 is low. If conservative measures
fail then non-invasive ventilation is most appropriate, if
Newsletters
tolerated.
My Career and Exams
Order History Reference: Normal Values
3402
Learning Goals Haematology
Question Filters
Immunoglobulins
Biochemistry
Security
Diabetes
Change Password
Sign Out Endocrinology

Blood gases

CSF

© 2011 PasTest Ltd | About Us | Contact Us | Help

file:///E|/Shakhawan/Respiratory%20S/276a.htm[3/13/2012 4:09:28 PM]


MyPasTest

Main Navigation
Question Browser: MRCP 1
Home
Subscriptions Question Browser Exam Builder Saved Exams
» MRCP 1 Session Progress
• Question Browser
Questions Correct 9
• Timed Test An elderly gentleman with a history of atrial fibrillation and
cardiac failure presents to the chest clinic with increasing Questions Incorrect 4
• Mock Exam
effort breathlessness. A CXR shows bilateral upper lobe Questions Total 13
• Past Papers fibrosis.
Questions Percentage 69 %
• Random Questions
Which of the following is the most likely diagnosis? More
• My Performance
• Media Bank Asbestosis
• New Multimedia Bronchopneumonia
Reference: Normal Values
Online Extras Multiple pulmonary emboli
Haematology
Library Sarcoidosis
Community Worsening cardiac failure Immunoglobulins
Help Biochemistry
3403
PasTest Store Diabetes
Endocrinology
My Account
Blood gases
Profile
CSF
Newsletters
My Career and Exams
Order History
Learning Goals
Question Filters

Security

Change Password
Sign Out

© 2011 PasTest Ltd | About Us | Contact Us | Help

file:///E|/Shakhawan/Respiratory%20S/277.htm[3/13/2012 4:09:29 PM]


MyPasTest

Main Navigation
Question Browser: MRCP 1
Home
Subscriptions Question Browser Exam Builder Saved Exams
» MRCP 1 Session Progress
• Question Browser
Questions Correct 9
• Timed Test An elderly gentleman with a history of atrial fibrillation and
cardiac failure presents to the chest clinic with increasing Questions Incorrect 5
• Mock Exam
effort breathlessness. A CXR shows bilateral upper lobe Questions Total 14
• Past Papers fibrosis.
Questions Percentage 64 %
• Random Questions
Which of the following is the most likely diagnosis? More
• My Performance
• Media Bank Asbestosis Your answer
• New Multimedia Bronchopneumonia
Online Extras Multiple pulmonary emboli
Library Sarcoidosis Correct answer
Community Worsening cardiac failure
Help
PasTest Store

My Account With upper lobe fibrosis there are five main causes you
should think of: TB, extrinsic allergic alveolitis,
Profile sarcoidosis, ankylosing spondylitis and ABPA. Asbestosis
causes lower zone shadowing.
Newsletters
My Career and Exams
Order History 3403 Reference: Normal Values
Learning Goals Haematology
Question Filters
Immunoglobulins
Biochemistry
Security
Diabetes
Change Password
Sign Out Endocrinology

Blood gases

CSF

© 2011 PasTest Ltd | About Us | Contact Us | Help

file:///E|/Shakhawan/Respiratory%20S/277a.htm[3/13/2012 4:09:31 PM]


MyPasTest

Main Navigation
Question Browser: MRCP 1
Home
Subscriptions Question Browser Exam Builder Saved Exams
» MRCP 1 Session Progress
• Question Browser
Questions Correct 9
• Timed Test A 34-year-old woman, who is a non-smoker, is found to
have moderate hypoxaemia on blood gases with a p a (O2 ) Questions Incorrect 5
• Mock Exam
of 7.9 kPa. Lung function tests confirm normal lung Questions Total 14
• Past Papers
volumes but a reduced TLCO at 45% of predicted. The most Questions Percentage 64 %
• Random Questions likely cause for this is:
• My Performance More
Pulmonary AV malformation
• Media Bank
Acute asthma
• New Multimedia
Emphysema Reference: Normal Values
Online Extras
Goodpasture’s syndrome Haematology
Library
Polycythaemia Immunoglobulins
Community
Help 3404 Biochemistry
PasTest Store Diabetes
Endocrinology
My Account
Blood gases
Profile
CSF
Newsletters
My Career and Exams
Order History
Learning Goals
Question Filters

Security

Change Password
Sign Out

© 2011 PasTest Ltd | About Us | Contact Us | Help

file:///E|/Shakhawan/Respiratory%20S/278.htm[3/13/2012 4:09:32 PM]


MyPasTest

Main Navigation
Question Browser: MRCP 1
Home
Subscriptions Question Browser Exam Builder Saved Exams
» MRCP 1 Session Progress
• Question Browser
Questions Correct 10
• Timed Test A 34-year-old woman, who is a non-smoker, is found to
Questions Incorrect 5
• Mock Exam have moderate hypoxaemia on blood gases with a p a (O2 )
of 7.9 kPa. Lung function tests confirm normal lung Questions Total 15
• Past Papers
volumes but a reduced TLCO at 45% of predicted. The most Questions Percentage 66 %
• Random Questions likely cause for this is:
• My Performance More
Pulmonary AV malformation Your answer
• Media Bank
Acute asthma
• New Multimedia
Emphysema
Online Extras
Goodpasture’s syndrome
Library
Polycythaemia
Community
Help
PasTest Store

Pulmonary AV malformations cause right to left shunts


My Account so reducing TLCO values and provoking hypoxaemia.
Emphysema also causes reduced TLCO but is usually
Profile associated with an elevated residual volume and would
Newsletters be unusual in a young non-smoker. Polycythaemia and
pulmonary haemorrhage syndromes usually cause
My Career and Exams elevation of TLCO. During acute asthma lung volumes
Order History will be decreased, particularly FEV1 . Reference: Normal Values
Learning Goals Haematology
Question Filters 3404 Immunoglobulins
Biochemistry
Security
Diabetes
Change Password
Sign Out Endocrinology

Blood gases

CSF

© 2011 PasTest Ltd | About Us | Contact Us | Help

file:///E|/Shakhawan/Respiratory%20S/278a.htm[3/13/2012 4:09:34 PM]


MyPasTest

Main Navigation
Question Browser: MRCP 1
Home
Subscriptions Question Browser Exam Builder Saved Exams
» MRCP 1 Session Progress
• Question Browser
Questions Correct 10
• Timed Test A 64-year-old man is brought to the Accident and
Questions Incorrect 5
• Mock Exam Emergency Department by his wife with drowsiness and
confusion. He has a history of COPD and attends the chest Questions Total 15
• Past Papers clinic. He had been commenced on antibiotics by his Questions Percentage 66 %
• Random Questions general practitioner 2 days earlier for an exacerbation of his
COPD. More
• My Performance
• Media Bank Which of the following blood gases (on 2 litres
• New Multimedia O 2 /min) fit best with this mans condition?
Reference: Normal Values
Online Extras
pH 7.14 p a (CO 2 ) 7.3p a (O2 ) 9.1 bicarbonate 14 Haematology
Library
pH 7.24 p a (CO 2 ) 9.3 p a (O2 ) 8.1 bicarbonate 34
Community Immunoglobulins
pH 7.38 p a (CO 2 ) 8.3p a (O2 ) 8.1 bicarbonate 38
Help Biochemistry
PasTest Store pH 7.38 p a (CO 2 ) 5.3 p a (O2 ) 8.1 bicarbonate 30
Diabetes
pH 7.54 p a (CO 2 ) 3.3 p a (O2 ) 9.1 bicarbonate 24
Endocrinology
My Account
3405 Blood gases
Profile
CSF
Newsletters
My Career and Exams
Order History
Learning Goals
Question Filters

Security

Change Password
Sign Out

© 2011 PasTest Ltd | About Us | Contact Us | Help

file:///E|/Shakhawan/Respiratory%20S/279.htm[3/13/2012 4:09:35 PM]


MyPasTest

Main Navigation
Question Browser: MRCP 1
Home
Subscriptions Question Browser Exam Builder Saved Exams
» MRCP 1 Session Progress
• Question Browser
Questions Correct 11
• Timed Test A 64-year-old man is brought to the Accident and
Emergency Department by his wife with drowsiness and Questions Incorrect 5
• Mock Exam
confusion. He has a history of COPD and attends the chest Questions Total 16
• Past Papers clinic. He had been commenced on antibiotics by his
Questions Percentage 68 %
• Random Questions general practitioner 2 days earlier for an exacerbation of his
COPD. More
• My Performance
• Media Bank Which of the following blood gases (on 2 litres
• New Multimedia O 2 /min) fit best with this mans condition?

Online Extras
pH 7.14 p a (CO 2 ) 7.3p a (O2 ) 9.1
Library bicarbonate 14
Community pH 7.24 p a (CO 2 ) 9.3 p a (O2 ) 8.1 Your answer
Help bicarbonate 34
PasTest Store pH 7.38 p a (CO 2 ) 8.3p a (O2 ) 8.1
bicarbonate 38
My Account pH 7.38 p a (CO 2 ) 5.3 p a (O2 ) 8.1
bicarbonate 30
Profile
pH 7.54 p a (CO 2 ) 3.3 p a (O2 ) 9.1
Newsletters
bicarbonate 24
My Career and Exams
Order History Reference: Normal Values
Learning Goals Haematology
Question Filters This patient has acute on chronic respiratory acidosis.
Immunoglobulins
Respiratory acidosis will have a raised p a (CO 2 ) and H +
concentration. The elevation of the bicarbonate reflects Biochemistry
Security
renal buffering from his chronic respiratory failure. In Diabetes
Change Password acute respiratory acidosis every 1 kPa rise in CO2
Sign Out produces 6 nmol/l of hydrogen ion and a 1 mmol/l Endocrinology
increase in bicarbonate, whereas when chronic, the
Blood gases
increase in hydrogen ions per kPa rise in carbon dioxide
falls to about 2.5 nmol/l. In this question ‘B’ is the best CSF
response as it best encapsulates the clinical data
provided.

3405

© 2011 PasTest Ltd | About Us | Contact Us | Help

file:///E|/Shakhawan/Respiratory%20S/279a.htm[3/13/2012 4:09:37 PM]


MyPasTest

Main Navigation
Question Browser: MRCP 1
Home
Subscriptions Question Browser Exam Builder Saved Exams
» MRCP 1 Session Progress
• Question Browser
Questions Correct 11
• Timed Test A 28-year-old woman presents to Accident and Emergency
with an acute asthmatic attack. Which of the following lung Questions Incorrect 5
• Mock Exam
function abnormalities is she likely to have? Questions Total 16
• Past Papers
Questions Percentage 68 %
• Random Questions Increased residual volume
• My Performance Increased forced expiratory ratio More
• Media Bank Increased forced vital capacity
• New Multimedia Increased airway conductance
Reference: Normal Values
Online Extras Increased gas transfer factor
Haematology
Library
3406
Community Immunoglobulins
Help Biochemistry
PasTest Store Diabetes
Endocrinology
My Account
Blood gases
Profile
CSF
Newsletters
My Career and Exams
Order History
Learning Goals
Question Filters

Security

Change Password
Sign Out

© 2011 PasTest Ltd | About Us | Contact Us | Help

file:///E|/Shakhawan/Respiratory%20S/280.htm[3/13/2012 4:09:38 PM]


MyPasTest

Main Navigation
Question Browser: MRCP 1
Home
Subscriptions Question Browser Exam Builder Saved Exams
» MRCP 1 Session Progress
• Question Browser
Questions Correct 12
• Timed Test A 28-year-old woman presents to Accident and Emergency
with an acute asthmatic attack. Which of the following lung Questions Incorrect 5
• Mock Exam
function abnormalities is she likely to have? Questions Total 17
• Past Papers
Questions Percentage 70 %
• Random Questions Increased residual volume Your answer
• My Performance Increased forced expiratory ratio More
• Media Bank Increased forced vital capacity
• New Multimedia Increased airway conductance
Online Extras Increased gas transfer factor
Library
Community
Help
PasTest Store The classical abnormalities are reduced FEV1 and
reduced FVC with a decrease in forced expiratory
(FEV 1 /FVC) ratio. Because of gas trapping there is an
My Account increase in residual volume and an increase in total lung
capacity, but the ratio of residual volume (RV) to total
Profile lung capacity (TLC) is increased. TLC = vital capacity
Newsletters (VC) + RV. Airway conductance (reciprocal of airway
resistance) is decreased in acute asthma. Gas transfer
My Career and Exams
would be difficult to measure in acute asthma but may
Order History be elevated in stable asthma where there may be Reference: Normal Values
Learning Goals chronic hyperinflation giving rise to a greater surface
area for blood/gas interfacing. Haematology
Question Filters
Immunoglobulins

3406 Biochemistry
Security
Diabetes
Change Password
Sign Out Endocrinology

Blood gases

CSF

© 2011 PasTest Ltd | About Us | Contact Us | Help

file:///E|/Shakhawan/Respiratory%20S/280a.htm[3/13/2012 4:09:40 PM]


MyPasTest

Main Navigation
Question Browser: MRCP 1
Home
Subscriptions Question Browser Exam Builder Saved Exams
» MRCP 1 Session Progress
• Question Browser
Questions Correct 12
• Timed Test An arterial blood sample from a 48-year-old male patient
with progressive dyspnoea has an oxygen tension of 8.5 Questions Incorrect 5
• Mock Exam
kPa (11.3-12.6), and a carbon dioxide tension of 8.5kPa Questions Total 17
• Past Papers (4.7-6.0). Given these blood gas results, what is the most
Questions Percentage 70 %
• Random Questions likely diagnosis?
• My Performance More
Pulmonary embolus
• Media Bank
Aspirin overdose
• New Multimedia
Ankylosing spondylitis Reference: Normal Values
Online Extras
Viral pneumonitis Haematology
Library
Lobar pneumonia
Community Immunoglobulins
Help 3407 Biochemistry
PasTest Store Diabetes
Endocrinology
My Account
Blood gases
Profile
CSF
Newsletters
My Career and Exams
Order History
Learning Goals
Question Filters

Security

Change Password
Sign Out

© 2011 PasTest Ltd | About Us | Contact Us | Help

file:///E|/Shakhawan/Respiratory%20S/281.htm[3/13/2012 4:09:41 PM]


MyPasTest

Main Navigation
Question Browser: MRCP 1
Home
Subscriptions Question Browser Exam Builder Saved Exams
» MRCP 1 Session Progress
• Question Browser
Questions Correct 13
• Timed Test An arterial blood sample from a 48-year-old male patient
with progressive dyspnoea has an oxygen tension of 8.5 Questions Incorrect 5
• Mock Exam
kPa (11.3-12.6), and a carbon dioxide tension of 8.5kPa Questions Total 18
• Past Papers (4.7-6.0). Given these blood gas results, what is the most
Questions Percentage 72 %
• Random Questions likely diagnosis?
• My Performance More
Pulmonary embolus
• Media Bank
Aspirin overdose
• New Multimedia
Ankylosing spondylitis Your answer
Online Extras
Viral pneumonitis
Library
Lobar pneumonia
Community
Help
PasTest Store
This is type II respiratory failure. p a (CO 2 ) rises due to
My Account hypoventilation and this can occur acutely due to drug
overdose or more chronically with gross obesity,
Profile kyphoscoliosis (and similar musculoskeletal disorders)
Newsletters and end-stage muscle, neurological and airways
disorders. Lobar pneumonia causes hypoxia but a
My Career and Exams normal/low p a (CO 2 ) (type I respiratory failure) unless
Order History exhaustion supervenes when the patient may start to Reference: Normal Values
Learning Goals retain carbon dioxide.
Haematology
Question Filters
Immunoglobulins
3407
Biochemistry
Security
Diabetes
Change Password
Sign Out Endocrinology

Blood gases

CSF

© 2011 PasTest Ltd | About Us | Contact Us | Help

file:///E|/Shakhawan/Respiratory%20S/281a.htm[3/13/2012 4:09:43 PM]


MyPasTest

Main Navigation
Question Browser: MRCP 1
Home
Subscriptions Question Browser Exam Builder Saved Exams
» MRCP 1 Session Progress
• Question Browser
Questions Correct 13
• Timed Test A 36-year-old woman is under follow up for recurrent
pulmonary thrombo-embolic disease. Which of the following Questions Incorrect 5
• Mock Exam
features is she most likely to have? Questions Total 18
• Past Papers
Questions Percentage 72 %
• Random Questions Quiet P 2
• My Performance More
Matched ventilation perfusion defect
• Media Bank Paroxysmal dyspnoea
• New Multimedia Increased transfer factor Reference: Normal Values
Online Extras Widening of the alveolar–arterial (A–a) gradient on
exercise Haematology
Library
Community Immunoglobulins
3408
Help Biochemistry
PasTest Store Diabetes
Endocrinology
My Account
Blood gases
Profile
CSF
Newsletters
My Career and Exams
Order History
Learning Goals
Question Filters

Security

Change Password
Sign Out

© 2011 PasTest Ltd | About Us | Contact Us | Help

file:///E|/Shakhawan/Respiratory%20S/282.htm[3/13/2012 4:09:44 PM]


MyPasTest

Main Navigation
Question Browser: MRCP 1
Home
Subscriptions Question Browser Exam Builder Saved Exams
» MRCP 1 Session Progress
• Question Browser
Questions Correct 14
• Timed Test A 36-year-old woman is under follow up for recurrent
pulmonary thrombo-embolic disease. Which of the following Questions Incorrect 5
• Mock Exam
features is she most likely to have? Questions Total 19
• Past Papers
Questions Percentage 73 %
• Random Questions Quiet P 2
• My Performance More
Matched ventilation perfusion defect
• Media Bank Paroxysmal dyspnoea
• New Multimedia Increased transfer factor
Online Extras Widening of the alveolar–arterial (A–a) Your answer
Library gradient on exercise
Community
Help
PasTest Store
Recurrent pulmonary emboli (PE) should always be
considered in cases of progressive SOB with no obvious
My Account cause. Possible clues include pulmonary hypertension,
right ventricular enlargement, hypoxia with a low
Profile p a (CO 2 ) and a low transfer factor. Widening of the
Newsletters lveolar–arterial (A–a) gradient on exercise is likely to
be found. Mismatched defects are classical of PE.
My Career and Exams
Order History Reference: Normal Values
Learning Goals 3408
Haematology
Question Filters
Immunoglobulins
Biochemistry
Security
Diabetes
Change Password
Sign Out Endocrinology

Blood gases

CSF

© 2011 PasTest Ltd | About Us | Contact Us | Help

file:///E|/Shakhawan/Respiratory%20S/282a.htm[3/13/2012 4:09:46 PM]


MyPasTest

Main Navigation
Question Browser: MRCP 1
Home
Subscriptions Question Browser Exam Builder Saved Exams
» MRCP 1 Session Progress
• Question Browser
Questions Correct 14
• Timed Test You review a 72-year-old man with severe chronic
obstructive pulmonary disease (COPD), who asks about the Questions Incorrect 5
• Mock Exam
provision of oxygen therapy at home. Questions Total 19
• Past Papers
In which of the following have randomised controlled Questions Percentage 73 %
• Random Questions
trials shown that long-term oxygen therapy (LTOT) More
• My Performance reduces mortality?
• Media Bank
• New Multimedia Asthma
Reference: Normal Values
Online Extras Cor pulmonale caused by chronic airflow obstruction
Cryptogenic fibrosing alveolitis Haematology
Library
Community Cystic fibrosis Immunoglobulins
Help Pulmonary sarcoidosis
Biochemistry
PasTest Store 3878 Diabetes
Endocrinology
My Account
Blood gases
Profile
CSF
Newsletters
My Career and Exams
Order History
Learning Goals
Question Filters

Security

Change Password
Sign Out

© 2011 PasTest Ltd | About Us | Contact Us | Help

file:///E|/Shakhawan/Respiratory%20S/283.htm[3/13/2012 4:09:47 PM]


MyPasTest

Main Navigation
Question Browser: MRCP 1
Home
Subscriptions Question Browser Exam Builder Saved Exams
» MRCP 1 Session Progress
• Question Browser
Questions Correct 15
• Timed Test You review a 72-year-old man with severe chronic
obstructive pulmonary disease (COPD), who asks about the Questions Incorrect 5
• Mock Exam
provision of oxygen therapy at home. Questions Total 20
• Past Papers
In which of the following have randomised controlled Questions Percentage 75 %
• Random Questions
trials shown that long-term oxygen therapy (LTOT) More
• My Performance reduces mortality?
• Media Bank
• New Multimedia Asthma

Online Extras Cor pulmonale caused by chronic Your answer


airflow obstruction
Library
Cryptogenic fibrosing alveolitis
Community
Cystic fibrosis
Help
Pulmonary sarcoidosis
PasTest Store

My Account

Profile Two controlled studies (in a mostly male population)


indicated that life can be prolonged by the continuous
Newsletters delivery of 2 litres per minute of oxygen via nasal
My Career and Exams prongs to achieve saturations of greater than 90% for a
large proportion of the day and night. Improvements in Reference: Normal Values
Order History
pulmonary artery hypertension were obtained in
Learning Goals patients who wore oxygen for more than 15 hours per Haematology
Question Filters day, but mortality was only improved in patients who
wore oxygen for more than 19 hours per day. Long- Immunoglobulins
term oxygen therapy (LTOT) should therefore be
considered in patients with chronic obstructive Biochemistry
Security
pulmonary disease (COPD) and a forced expiratory Diabetes
Change Password volume 1 (FEV1) of less than 1.5 litres, an arterial
partial pressure of oxygen (PaO 2 ) of less than 7.3 kPa, Endocrinology
Sign Out
and carboxyhaemoglobin of less than 3%. Although
Blood gases
oxygen cylinders may be provided for intermittent use
by patients for the relief of symptoms of breathlessness, CSF
they have no effect on prognosis.

3878

© 2011 PasTest Ltd | About Us | Contact Us | Help

file:///E|/Shakhawan/Respiratory%20S/283a.htm[3/13/2012 4:09:49 PM]


MyPasTest

Main Navigation
Question Browser: MRCP 1
Home
Subscriptions Question Browser Exam Builder Saved Exams
» MRCP 1 Session Progress
• Question Browser
Questions Correct 15
• Timed Test A 60-year-old man presents with inspiratory stridor with a
chest radiograph that reveals compression of the trachea Questions Incorrect 5
• Mock Exam
by a retrosternal goitre. Questions Total 20
• Past Papers
Questions Percentage 75 %
• Random Questions Which of the following investigations is the most
useful to assess the severity of his airway More
• My Performance
obstruction?
• Media Bank
• New Multimedia Flow/volume loop
Reference: Normal Values
Online Extras Forced expiratory volume
Forced vital capacity Haematology
Library
Community Peak expiratory flow rate Immunoglobulins
Help Residual volume Biochemistry
PasTest Store 3879 Diabetes
Endocrinology
My Account
Blood gases
Profile
CSF
Newsletters
My Career and Exams
Order History
Learning Goals
Question Filters

Security

Change Password
Sign Out

© 2011 PasTest Ltd | About Us | Contact Us | Help

file:///E|/Shakhawan/Respiratory%20S/284.htm[3/13/2012 4:09:50 PM]


MyPasTest

Main Navigation
Question Browser: MRCP 1
Home
Subscriptions Question Browser Exam Builder Saved Exams
» MRCP 1 Session Progress
• Question Browser
Questions Correct 16
• Timed Test A 60-year-old man presents with inspiratory stridor with a
chest radiograph that reveals compression of the trachea Questions Incorrect 5
• Mock Exam
by a retrosternal goitre. Questions Total 21
• Past Papers
Questions Percentage 76 %
• Random Questions Which of the following investigations is the most
useful to assess the severity of his airway More
• My Performance
obstruction?
• Media Bank
• New Multimedia Flow/volume loop Your answer
Online Extras Forced expiratory volume
Library Forced vital capacity
Community Peak expiratory flow rate
Help Residual volume
PasTest Store

My Account
Flow/volume loop examination is the best way to
Profile ascertain the effects of extrathoracic tracheal
obstruction. Patients with retrosternal goitre show
Newsletters
proportionally more reduction in inspiratory flow rate
My Career and Exams compared with expiratory flow rate. The test is useful to
Order History provide differentiation from patients who have severe Reference: Normal Values
airflow limitation or intrathoracic large airway
Learning Goals obstruction. Where there is considerable inspiratory Haematology
Question Filters airflow restriction, partial thyroidectomy may be
considered. Immunoglobulins
Biochemistry
Security
3879 Diabetes
Change Password
Sign Out Endocrinology

Blood gases

CSF

© 2011 PasTest Ltd | About Us | Contact Us | Help

file:///E|/Shakhawan/Respiratory%20S/284a.htm[3/13/2012 4:09:52 PM]


MyPasTest

Main Navigation
Question Browser: MRCP 1
Home
Subscriptions Question Browser Exam Builder Saved Exams
» MRCP 1 Session Progress
• Question Browser
Questions Correct 16
• Timed Test An 18-year-old woman presents with an acute exacerbation
of asthma associated with a chest infection. She is unable Questions Incorrect 5
• Mock Exam
to complete a sentence and her peak flow rate was 35% of Questions Total 21
• Past Papers her normal level. She is treated with high-flow oxygen,
Questions Percentage 76 %
• Random Questions nebulised bronchodilators, and has been taking oral
corticosteroids for two days before admission, but her More
• My Performance
condition has not improved.
• Media Bank
Which of the following treatments, given
• New Multimedia
intravenously, would be the most appropriate for this Reference: Normal Values
Online Extras patient?
Haematology
Library
Aminophylline
Community Immunoglobulins
Augmentin
Help Biochemistry
Hydrocortisone
PasTest Store Diabetes
Magnesium
Salbutamol Endocrinology
My Account
Blood gases
Profile 3880
CSF
Newsletters
My Career and Exams
Order History
Learning Goals
Question Filters

Security

Change Password
Sign Out

© 2011 PasTest Ltd | About Us | Contact Us | Help

file:///E|/Shakhawan/Respiratory%20S/285.htm[3/13/2012 4:09:53 PM]


MyPasTest

Main Navigation
Question Browser: MRCP 1
Home
Subscriptions Question Browser Exam Builder Saved Exams
» MRCP 1 Session Progress
• Question Browser
Questions Correct 17
• Timed Test An 18-year-old woman presents with an acute exacerbation
of asthma associated with a chest infection. She is unable Questions Incorrect 5
• Mock Exam
to complete a sentence and her peak flow rate was 35% of Questions Total 22
• Past Papers her normal level. She is treated with high-flow oxygen,
Questions Percentage 77 %
• Random Questions nebulised bronchodilators, and has been taking oral
corticosteroids for two days before admission, but her More
• My Performance
condition has not improved.
• Media Bank
Which of the following treatments, given
• New Multimedia
intravenously, would be the most appropriate for this
Online Extras patient?
Library
Aminophylline
Community
Augmentin
Help
Hydrocortisone
PasTest Store
Magnesium Your answer
Salbutamol
My Account

Profile
Newsletters
My Career and Exams Magnesium therapy given intravenously has been
shown to reduce rates of admission to the intensive Reference: Normal Values
Order History
therapy unit (ITU) in patients who suffer an acute
Learning Goals exacerbation of asthma. Aminophylline and salbutamol Haematology
Question Filters given intravenously are no longer recommended
because of an increased risk of cardiac arrythmias. Immunoglobulins
Hydrocortisone is, of course, part of the therapeutic
regimen, but takes a number of hours to have an Biochemistry
Security
impact on symptoms, even when given as an Diabetes
Change Password intravenous dose. Crucial to this patient’s management
is early assessment by an intensivist, as failure to Endocrinology
Sign Out
respond to medical therapy is likely to necessitate ITU
admission and possible intubation. Blood gases

CSF
3880

© 2011 PasTest Ltd | About Us | Contact Us | Help

file:///E|/Shakhawan/Respiratory%20S/285a.htm[3/13/2012 4:09:55 PM]


MyPasTest

Main Navigation
Question Browser: MRCP 1
Home
Subscriptions Question Browser Exam Builder Saved Exams
» MRCP 1 Session Progress
• Question Browser
Questions Correct 0
• Timed Test An 18-year-old woman presents with an acute pulmonary
embolism in the ninth week of pregnancy. No obvious Questions Incorrect 0
• Mock Exam
factors contributed to the risk of pulmonary embolus. Questions Total 0
• Past Papers
What is the most appropriate treatment for this patient Questions Percentage 0%
• Random Questions
throughout her pregnancy? More
• My Performance
• Media Bank Aspirin
• New Multimedia Intravenous unfractionated heparin
Reference: Normal Values
Online Extras Subcutaneous low molecular weight heparin
Haematology
Library Subcutaneous unfractionated heparin
Community Warfarin Immunoglobulins
Help Biochemistry
3882
PasTest Store Diabetes
Endocrinology
My Account
Blood gases
Profile
CSF
Newsletters
My Career and Exams
Order History
Learning Goals
Question Filters

Security

Change Password
Sign Out

© 2011 PasTest Ltd | About Us | Contact Us | Help

file:///E|/Shakhawan/Respiratory%20S/286.htm[3/13/2012 4:09:56 PM]


MyPasTest

Main Navigation
Question Browser: MRCP 1
Home
Subscriptions Question Browser Exam Builder Saved Exams
» MRCP 1 Session Progress
• Question Browser
Questions Correct 1
• Timed Test An 18-year-old woman presents with an acute pulmonary
embolism in the ninth week of pregnancy. No obvious Questions Incorrect 0
• Mock Exam
factors contributed to the risk of pulmonary embolus. Questions Total 1
• Past Papers
What is the most appropriate treatment for this patient Questions Percentage 100 %
• Random Questions
throughout her pregnancy? More
• My Performance
• Media Bank Aspirin
• New Multimedia Intravenous unfractionated heparin
Online Extras Subcutaneous low molecular weight Your answer
Library heparin

Community Subcutaneous unfractionated heparin

Help Warfarin

PasTest Store

My Account
Warfarin is contraindicated in pregnancy because of the
Profile risk of teratogenicity. It would be impractical to treat
this woman for the remaining 31 weeks of her
Newsletters pregnancy with intravenous heparin. This leaves daily
My Career and Exams injections of subcutaneous low molecular weight heparin
as the best treatment option. Warfarin might be an Reference: Normal Values
Order History
option during the middle weeks of pregnancy (13-36
Learning Goals weeks), but practically it would probably be better to Haematology
Question Filters stick with low molecular weight heparin for the duration.
Aspirin is an inadequate treatment for pulmonary Immunoglobulins
embolus. Post-pregnancy, it seems sensible to screen
this woman for inherited coagulopathy because of her Biochemistry
Security
young age and lack of predisposing factors apart from Diabetes
Change Password the pregnancy.
Sign Out Endocrinology

3882 Blood gases

CSF

© 2011 PasTest Ltd | About Us | Contact Us | Help

file:///E|/Shakhawan/Respiratory%20S/286a.htm[3/13/2012 4:09:58 PM]


MyPasTest

Main Navigation
Question Browser: MRCP 1
Home
Subscriptions Question Browser Exam Builder Saved Exams
» MRCP 1 Session Progress
• Question Browser
Questions Correct 1
• Timed Test A 40-year-old man presents with a long history of
productive cough breathlessness. He had complained of Questions Incorrect 0
• Mock Exam
halitosis, exacerbations that consisted of productive Questions Total 1
• Past Papers sputum, chest pain, and occasional haemoptysis.
Questions Percentage 100 %
• Random Questions Examination in the clinic reveals bilateral inspiratory
crackles. More
• My Performance
• Media Bank Which of following treatments is likely to best
decrease the frequency of his exacerbations?
• New Multimedia
Reference: Normal Values
Online Extras Cyclical antibiotic therapy
Haematology
Library Inhaled corticosteroids
Community Nebulised bronchodilators Immunoglobulins
Help Postural drainage Biochemistry
PasTest Store Surgical resection Diabetes

3902 Endocrinology
My Account
Blood gases
Profile
CSF
Newsletters
My Career and Exams
Order History
Learning Goals
Question Filters

Security

Change Password
Sign Out

© 2011 PasTest Ltd | About Us | Contact Us | Help

file:///E|/Shakhawan/Respiratory%20S/287.htm[3/13/2012 4:09:59 PM]


MyPasTest

Main Navigation
Question Browser: MRCP 1
Home
Subscriptions Question Browser Exam Builder Saved Exams
» MRCP 1 Session Progress
• Question Browser
Questions Correct 1
• Timed Test A 40-year-old man presents with a long history of
productive cough breathlessness. He had complained of Questions Incorrect 1
• Mock Exam
halitosis, exacerbations that consisted of productive Questions Total 2
• Past Papers sputum, chest pain, and occasional haemoptysis.
Questions Percentage 50 %
• Random Questions Examination in the clinic reveals bilateral inspiratory
crackles. More
• My Performance
• Media Bank Which of following treatments is likely to best
decrease the frequency of his exacerbations?
• New Multimedia
Online Extras Cyclical antibiotic therapy Your answer
Library Inhaled corticosteroids
Community Nebulised bronchodilators
Help Postural drainage Correct answer
PasTest Store Surgical resection

My Account

Profile
Postural drainage is the mainstay of treatment for
Newsletters bronchiectasis. Patients should be trained by
My Career and Exams physiotherapists to maintain a position in which the lobe
to be drained is uppermost at least three times per day Reference: Normal Values
Order History
for 10–20 minutes. Rotating antibiotics are favoured by
Learning Goals some clinicians, but their value has not been proved Haematology
Question Filters categorically in randomised controlled studies.
Bronchodilators may help those patients who have Immunoglobulins
some degree of reversible airflow obstruction. It is rare
for bronchiectasis to be sufficiently localised to permit Biochemistry
Security
surgical resection. Diabetes
Change Password
Sign Out Endocrinology
3902
Blood gases

CSF

© 2011 PasTest Ltd | About Us | Contact Us | Help

file:///E|/Shakhawan/Respiratory%20S/287a.htm[3/13/2012 4:10:01 PM]


MyPasTest

Main Navigation
Question Browser: MRCP 1
Home
Subscriptions Question Browser Exam Builder Saved Exams
» MRCP 1 Session Progress
• Question Browser
Questions Correct 1
• Timed Test A man came in with breathlessness and anterior chest pain.
Chest X-ray showed 50% pneumothorax on the right side Questions Incorrect 1
• Mock Exam
with midline shift away from the side of the pneumothorax. Questions Total 2
• Past Papers His PR was 85 and BP was 110/70.
Questions Percentage 50 %
• Random Questions
What is the next appropriate management? More
• My Performance
• Media Bank Intravenous cannula on the second intercostal space
• New Multimedia mid-clavicular line
Reference: Normal Values
Online Extras Needle aspiration
Chest-drain insertion Haematology
Library
Community Repeat CXR after a few hours Immunoglobulins
Help Chest-drain insertion under radiology control
Biochemistry
PasTest Store 5191 Diabetes
Endocrinology
My Account
Blood gases
Profile
CSF
Newsletters
My Career and Exams
Order History
Learning Goals
Question Filters

Security

Change Password
Sign Out

© 2011 PasTest Ltd | About Us | Contact Us | Help

file:///E|/Shakhawan/Respiratory%20S/288.htm[3/13/2012 4:10:02 PM]


MyPasTest

Main Navigation
Question Browser: MRCP 1
Home
Subscriptions Question Browser Exam Builder Saved Exams
» MRCP 1 Session Progress
• Question Browser
Questions Correct 1
• Timed Test A man came in with breathlessness and anterior chest pain.
Chest X-ray showed 50% pneumothorax on the right side Questions Incorrect 2
• Mock Exam
with midline shift away from the side of the pneumothorax. Questions Total 3
• Past Papers His PR was 85 and BP was 110/70.
Questions Percentage 33 %
• Random Questions
What is the next appropriate management? More
• My Performance
• Media Bank Intravenous cannula on the second Correct answer
• New Multimedia intercostal space mid-clavicular line

Online Extras Needle aspiration

Library Chest-drain insertion Your answer

Community Repeat CXR after a few hours

Help Chest-drain insertion under


radiology control
PasTest Store

My Account

Profile The answer is (a). This man has a large pneumothorax


with mediastinal shift and significant symptoms. In the
Newsletters presence of midline shift, the most appropriate
My Career and Exams management would be needle decompression, with
placement of an intravenous cannula in the second Reference: Normal Values
Order History
intercostal space. This should be followed later by
Learning Goals placement of a formal chest drain. However, simple Haematology
Question Filters pneumothorax can be managed with needle aspiration
in the first instance, and this is the course of Immunoglobulins
management recommended by recent British Thoracic
Society guidelines. Biochemistry
Security
Diabetes
Change Password
Sign Out 5191 Endocrinology

Blood gases

CSF

© 2011 PasTest Ltd | About Us | Contact Us | Help

file:///E|/Shakhawan/Respiratory%20S/288a.htm[3/13/2012 4:10:04 PM]


MyPasTest

Main Navigation
Question Browser: MRCP 1
Home
Subscriptions Question Browser Exam Builder Saved Exams
» MRCP 1 Session Progress
• Question Browser
Questions Correct 1
• Timed Test A 38-year-old man presents with emphysema. He is a non-
smoker. He also has abnormal liver function tests, and his Questions Incorrect 2
• Mock Exam
liver biopsy reveals evidence of cirrhosis. You suspect Questions Total 3
• Past Papers alpha-1 antitrypsin deficiency.
Questions Percentage 33 %
• Random Questions
What is the genotype that fits best with this clinical More
• My Performance picture?
• Media Bank
• New Multimedia PiMM
Reference: Normal Values
Online Extras PiMZ
PiSZ Haematology
Library
Community PiZZ Immunoglobulins
Help PiSS
Biochemistry
PasTest Store 5192 Diabetes
Endocrinology
My Account
Blood gases
Profile
CSF
Newsletters
My Career and Exams
Order History
Learning Goals
Question Filters

Security

Change Password
Sign Out

© 2011 PasTest Ltd | About Us | Contact Us | Help

file:///E|/Shakhawan/Respiratory%20S/289.htm[3/13/2012 4:10:05 PM]


MyPasTest

Main Navigation
Question Browser: MRCP 1
Home
Subscriptions Question Browser Exam Builder Saved Exams
» MRCP 1 Session Progress
• Question Browser
Questions Correct 2
• Timed Test A 38-year-old man presents with emphysema. He is a non-
Questions Incorrect 2
• Mock Exam smoker. He also has abnormal liver function tests, and his
liver biopsy reveals evidence of cirrhosis. You suspect Questions Total 4
• Past Papers alpha-1 antitrypsin deficiency. Questions Percentage 50 %
• Random Questions
What is the genotype that fits best with this clinical More
• My Performance picture?
• Media Bank
• New Multimedia PiMM

Online Extras PiMZ

Library PiSZ

Community PiZZ Your answer

Help PiSS

PasTest Store

My Account
This man has severe disease, which is most commonly
Profile associated with the PiZZ genotype. Such a patient may
even present with liver disease in childhood and require
Newsletters eventual transplantation. Around 10–15% of patients
My Career and Exams with PiZZ genotype develop cirrhosis by the age of 50
years, and 75% develop respiratory problems. Reference: Normal Values
Order History
Heterozygotes are less likely to develop liver disease,
Learning Goals and tend to develop less severe lung disease, although Haematology
Question Filters the PiMZ or PiSZ genotypes (heterozygotic state) may
accelerate emphysema associated with smoking. Levels Immunoglobulins
of α1-antitrypsin correlate with genotype; in
Biochemistry
Security homozygotes they run at around 10% of the normal
level, while heterozygotes have levels at around 60% of Diabetes
Change Password normal. Best prognosis from α1-antitrypsin deficiency is
Sign Out seen in non-smoking patients; for this reason, advise Endocrinology
patients to refrain from smoking at all costs. Blood gases

CSF
5192

© 2011 PasTest Ltd | About Us | Contact Us | Help

file:///E|/Shakhawan/Respiratory%20S/289a.htm[3/13/2012 4:10:07 PM]


MyPasTest

Main Navigation
Question Browser: MRCP 1
Home
Subscriptions Question Browser Exam Builder Saved Exams
» MRCP 1 Session Progress
• Question Browser
Questions Correct 2
• Timed Test A 62-year-old man with a history of smoking has been
followed up for breathlessness with productive cough. His Questions Incorrect 2
• Mock Exam
spirometry results are as follows: 5 years ago, FEV1 /FVC Questions Total 4
• Past Papers
90%; currently FEV1 /FVC 50%. Questions Percentage 50 %
• Random Questions
• My Performance Which of the following is the most important More
treatment?
• Media Bank
• New Multimedia Smoking-cessation advice
Reference: Normal Values
Online Extras Start salbutamol
Haematology
Library Start prednisolone
Community Start inhaled steroids Immunoglobulins
Help Start inhaled anticholinergics Biochemistry
PasTest Store Diabetes
5193
Endocrinology
My Account
Blood gases
Profile
CSF
Newsletters
My Career and Exams
Order History
Learning Goals
Question Filters

Security

Change Password
Sign Out

© 2011 PasTest Ltd | About Us | Contact Us | Help

file:///E|/Shakhawan/Respiratory%20S/290.htm[3/13/2012 4:10:09 PM]


MyPasTest

Main Navigation
Question Browser: MRCP 1
Home
Subscriptions Question Browser Exam Builder Saved Exams
» MRCP 1 Session Progress
• Question Browser
Questions Correct 3
• Timed Test A 62-year-old man with a history of smoking has been
followed up for breathlessness with productive cough. His Questions Incorrect 2
• Mock Exam
spirometry results are as follows: 5 years ago, FEV1 /FVC Questions Total 5
• Past Papers
90%; currently FEV1 /FVC 50%. Questions Percentage 60 %
• Random Questions
• My Performance Which of the following is the most important More
treatment?
• Media Bank
• New Multimedia Smoking-cessation advice Your answer
Online Extras Start salbutamol
Library Start prednisolone
Community Start inhaled steroids
Help Start inhaled anticholinergics
PasTest Store

My Account
The answer is (a). This man has severe chronic
Profile obstructive pulmonary disease and has deteriorated
Newsletters significantly during the past five years. Currently the
only intervention proven to affect mortality is smoking
My Career and Exams cessation. A trial of prednisolone therapy may still be
Order History useful, with pre- and postspirometry to assess Reference: Normal Values
reversibility of bronchial obstruction, and likely response
Learning Goals Haematology
to inhaled bronchodilator and inhaled steroid therapy.
Question Filters Recently, inhaled long-acting cholinergics and inhaled
Immunoglobulins
combination inhalers containing both long-acting β2 -
agonist and high-dose inhaled steroid have been shown Biochemistry
Security
to affect lung function positively. The TORCH study has
Diabetes
Change Password also shown a positive effect on exacerbations with
respect to the combination of LABA and inhaled Endocrinology
Sign Out corticosteroid.
Blood gases

CSF
5193

© 2011 PasTest Ltd | About Us | Contact Us | Help

file:///E|/Shakhawan/Respiratory%20S/290a.htm[3/13/2012 4:10:10 PM]


MyPasTest

Main Navigation
Question Browser: MRCP 1
Home
Subscriptions Question Browser Exam Builder Saved Exams
» MRCP 1 Session Progress
• Question Browser
Questions Correct 3
• Timed Test You review a 58-year-old man who has been admitted with
pneumonia. He is a heavy smoker, and chest X-ray Questions Incorrect 2
• Mock Exam
confirms that he has an obstructing tumour, revealed on Questions Total 5
• Past Papers bronchoscopy to be a squamous cell carcinoma of the
Questions Percentage 60 %
• Random Questions bronchus. Which of the following features would be a
contraindication to surgery? More
• My Performance
• Media Bank FEV1 1.8 litres
• New Multimedia Paraneoplastic neuropathy Reference: Normal Values
Online Extras Hypercalcaemia
Haematology
Library Spread to involve the C8, T1 and T2 nerve roots
Community Immunoglobulins
Gas transfer reduced by 33%
Help Biochemistry
PasTest Store 5194
Diabetes
Endocrinology
My Account
Blood gases
Profile
CSF
Newsletters
My Career and Exams
Order History
Learning Goals
Question Filters

Security

Change Password
Sign Out

© 2011 PasTest Ltd | About Us | Contact Us | Help

file:///E|/Shakhawan/Respiratory%20S/291.htm[3/13/2012 4:10:12 PM]


MyPasTest

Main Navigation
Question Browser: MRCP 1
Home
Subscriptions Question Browser Exam Builder Saved Exams
» MRCP 1 Session Progress
• Question Browser
Questions Correct 1
• Timed Test You review a 58-year-old man who has been admitted with
pneumonia. He is a heavy smoker, and chest X-ray Questions Incorrect 0
• Mock Exam
confirms that he has an obstructing tumour, revealed on Questions Total 1
• Past Papers bronchoscopy to be a squamous cell carcinoma of the
Questions Percentage 100 %
• Random Questions bronchus. Which of the following features would be a
contraindication to surgery? More
• My Performance
• Media Bank FEV1 1.8 litres
• New Multimedia Paraneoplastic neuropathy
Online Extras Hypercalcaemia
Library Spread to involve the C8, T1 and T2 Your answer
Community nerve roots
Help Gas transfer reduced by 33%
PasTest Store

My Account
The answer is (d). Spread to involve the C8, T1 and T2
Profile nerve roots occurs by rib-erosion by tumour to involve
Newsletters the lower roots of the brachial plexus. Known as
Pancoast’s tumour, this causes severe pain in the
My Career and Exams shoulder and down the inner aspect of the arm on the
Order History affected side. An FEV1 of less than 1.5 litres is not Reference: Normal Values
Learning Goals compatible with an active life following pneumonectomy,
as is a reduction in the gas-transfer test of more than Haematology
Question Filters 50%. Paraneoplastic phenomena such as Immunoglobulins
hypercalcaemia or neuropathy may not necessarily
preclude successful surgical resection of tumour; but in Biochemistry
Security total only 20–25% of all cases of bronchial carcinoma
are said to be suitable for surgery. Diabetes
Change Password
Sign Out Endocrinology
5194 Blood gases

CSF

© 2011 PasTest Ltd | About Us | Contact Us | Help

file:///E|/Shakhawan/Respiratory%20S/291a.htm[3/13/2012 4:10:13 PM]


MyPasTest

Main Navigation
Question Browser: MRCP 1
Home
Subscriptions Question Browser Exam Builder Saved Exams
» MRCP 1 Session Progress
• Question Browser
Questions Correct 1
• Timed Test You review a 56-year-old man with chronic obstructive
pulmonary disease, who is suffering an acute exacerbation. Questions Incorrect 0
• Mock Exam
Questions Total 1
• Past Papers Which of the following features would suggest
suitability for non-invasive ventilation with BIPAP? Questions Percentage 100 %
• Random Questions
• My Performance More
Profound hypoxia
• Media Bank
Hypercapnia without profound hypoxia
• New Multimedia
Confusion Reference: Normal Values
Online Extras
Cryptogenic fibrosing alveolitis
Haematology
Library
Decreased conscious level
Community Immunoglobulins
Help 5195 Biochemistry
PasTest Store Diabetes
Endocrinology
My Account
Blood gases
Profile
CSF
Newsletters
My Career and Exams
Order History
Learning Goals
Question Filters

Security

Change Password
Sign Out

© 2011 PasTest Ltd | About Us | Contact Us | Help

file:///E|/Shakhawan/Respiratory%20S/292.htm[3/13/2012 4:10:15 PM]


MyPasTest

Main Navigation
Question Browser: MRCP 1
Home
Subscriptions Question Browser Exam Builder Saved Exams
» MRCP 1 Session Progress
• Question Browser
Questions Correct 1
• Timed Test You review a 56-year-old man with chronic obstructive
pulmonary disease, who is suffering an acute exacerbation. Questions Incorrect 1
• Mock Exam
Questions Total 2
• Past Papers Which of the following features would suggest
suitability for non-invasive ventilation with BIPAP? Questions Percentage 50 %
• Random Questions
• My Performance More
Profound hypoxia Your answer
• Media Bank
Hypercapnia without profound Correct answer
• New Multimedia hypoxia
Online Extras Confusion
Library Cryptogenic fibrosing alveolitis
Community Decreased conscious level
Help
PasTest Store

My Account The answer is (b). Non-invasive ventilation (NIV) is


suitable for patients who are conscious, cooperative and
Profile able to protect their airway. Continuous positive airway
pressure (CPAP) or bilevel positive airway pressure
Newsletters (BIPAP) is applied via a tight-fitting nose- or face-mask.
My Career and Exams NIV may be particularly useful in a patient suffering
acute hypercapnic respiratory failure associated with Reference: Normal Values
Order History
chronic obstructive pulmonary disease, if the patient is
Learning Goals not profoundly hypoxic. NIV may also be an option in a Haematology
Question Filters patient who has mild hypoxia associated with
community-acquired pneumonia. There is a reduced Immunoglobulins
risk of ventilator-associated pneumonia, and improved
patient comfort and preservation of airway protection Biochemistry
Security
mechanisms. If there is profound hypoxia then this Diabetes
Change Password would drive you more towards ventilation.
Sign Out Endocrinology

5195 Blood gases

CSF

© 2011 PasTest Ltd | About Us | Contact Us | Help

file:///E|/Shakhawan/Respiratory%20S/292a.htm[3/13/2012 4:10:16 PM]


MyPasTest

Main Navigation
Question Browser: MRCP 1
Home
Subscriptions Question Browser Exam Builder Saved Exams
» MRCP 1 Session Progress
• Question Browser
Questions Correct 1
• Timed Test An obese man with history of hypertension presented with
history of daytime sleepiness which can come on at the Questions Incorrect 1
• Mock Exam
drop of a hat, disturbed nights sleep and associated vivid Questions Total 2
• Past Papers dreams. His wife says he is restless at night and snores
Questions Percentage 50 %
• Random Questions occasionally. He is overweight and has a collar size of 18.5.
• My Performance More
What is the diagnosis?
• Media Bank
• New Multimedia Obstructive sleep apnoea
Reference: Normal Values
Online Extras Central sleep apnoea
Narcolepsy Haematology
Library
Community Simple obesity Immunoglobulins
Help Complex partial seizures
Biochemistry
PasTest Store 5196 Diabetes
Endocrinology
My Account
Blood gases
Profile
CSF
Newsletters
My Career and Exams
Order History
Learning Goals
Question Filters

Security

Change Password
Sign Out

© 2011 PasTest Ltd | About Us | Contact Us | Help

file:///E|/Shakhawan/Respiratory%20S/293.htm[3/13/2012 4:10:18 PM]


MyPasTest

Main Navigation
Question Browser: MRCP 1
Home
Subscriptions Question Browser Exam Builder Saved Exams
» MRCP 1 Session Progress
• Question Browser
Questions Correct 2
• Timed Test An obese man with history of hypertension presented with
history of daytime sleepiness which can come on at the Questions Incorrect 1
• Mock Exam
drop of a hat, disturbed nights sleep and associated vivid Questions Total 3
• Past Papers dreams. His wife says he is restless at night and snores
Questions Percentage 66 %
• Random Questions occasionally. He is overweight and has a collar size of 18.5.
• My Performance More
What is the diagnosis?
• Media Bank
• New Multimedia Obstructive sleep apnoea Your answer

Online Extras Central sleep apnoea

Library Narcolepsy

Community Simple obesity

Help Complex partial seizures

PasTest Store

My Account
The answer is (a). Obstructive sleep apnoea (OSA)
Profile occurs most commonly in overweight middle-aged men
and is said to affect in total some 1–2% of the
Newsletters population. Signs of OSA include loud snoring, daytime
My Career and Exams sleepiness, unrefreshed sleep, restless sleep, morning
headaches, nocturnal choking and reduced libido. Reference: Normal Values
Order History
Important predisposing factors include pharyngeal
Learning Goals encroachment (obesity, acromegaly and enlarged Haematology
Question Filters tonsils), nasal obstruction (due to nasal deformities,
rhinitis, polyps or adenoids) and use of respiratory Immunoglobulins
depressant drugs such as alcohol, sedatives or strong
analgesics. Chronic sleep apnoea eventually leads to Biochemistry
Security
pulmonary hypertension, systemic hypertension and Diabetes
Change Password eventually, if untreated, right heart failure. The answer
represents a balance of probabilities, given the neck Endocrinology
Sign Out
size, OSA is much more likely than narcolepsy.
Blood gases

CSF
5196

© 2011 PasTest Ltd | About Us | Contact Us | Help

file:///E|/Shakhawan/Respiratory%20S/293a.htm[3/13/2012 4:10:19 PM]


MyPasTest

Main Navigation
Question Browser: MRCP 1
Home
Subscriptions Question Browser Exam Builder Saved Exams
» MRCP 1 Session Progress
• Question Browser
Questions Correct 2
• Timed Test A 72-year-old farmer presented with a history of massive
Questions Incorrect 1
• Mock Exam haemoptysis. He has a 50-year smoking history and takes
inhaled long-acting β2-agonists and steroids. Chest X-ray Questions Total 3
• Past Papers
showed a left upper lobe mass that was smooth. Questions Percentage 66 %
• Random Questions Investigation: positive Aspergillus fumigatus precipitin. He
• My Performance was transfused and resuscitated. In the meantime, he had More
another massive haemoptysis but was maintaining a
• Media Bank systolic blood pressure of 145 mmHg, and a pulse of
• New Multimedia 85/min.
Reference: Normal Values
Online Extras What is the next appropriate management?
Haematology
Library
Lobar resection Immunoglobulins
Community
Angiography and embolisation
Help Biochemistry
Further fluid resuscitation with transfusion
PasTest Store Diabetes
Commence amphotericin B therapy
Endocrinology
My Account Nebulised β -agonist therapy to reduce
bronchospasm Blood gases
Profile
5197 CSF
Newsletters
My Career and Exams
Order History
Learning Goals
Question Filters

Security

Change Password
Sign Out

© 2011 PasTest Ltd | About Us | Contact Us | Help

file:///E|/Shakhawan/Respiratory%20S/294.htm[3/13/2012 4:10:21 PM]


MyPasTest

Main Navigation
Question Browser: MRCP 1
Home
Subscriptions Question Browser Exam Builder Saved Exams
» MRCP 1 Session Progress
• Question Browser
Questions Correct 1
• Timed Test A 72-year-old farmer presented with a history of massive
haemoptysis. He has a 50-year smoking history and takes Questions Incorrect 0
• Mock Exam
inhaled long-acting β2-agonists and steroids. Chest X-ray Questions Total 1
• Past Papers
showed a left upper lobe mass that was smooth. Questions Percentage 100 %
• Random Questions Investigation: positive Aspergillus fumigatus precipitin. He
• My Performance was transfused and resuscitated. In the meantime, he had More
another massive haemoptysis but was maintaining a
• Media Bank systolic blood pressure of 145 mmHg, and a pulse of
• New Multimedia 85/min.
Online Extras What is the next appropriate management?
Library
Lobar resection
Community
Angiography and embolisation Your answer
Help
Further fluid resuscitation with
PasTest Store
transfusion
Commence amphotericin B therapy
My Account
Nebulised β -agonist therapy to reduce
Profile bronchospasm
Newsletters
My Career and Exams
Order History Reference: Normal Values
The answer is (b). With such a long smoking history,
Learning Goals Haematology
this patient is likely to have significant chronic
Question Filters obstructive pulmonary disease. In this respect, although
Immunoglobulins
he has a large aspergilloma, it is probably worth first
trying angiography and embolisation, with lobar Biochemistry
Security resection as the intervention of next resort. Case series
from tertiary referral centres suggest that the bleeding Diabetes
Change Password site can be identified and bleeding controlled in up to
Sign Out 90% of cases. Poor prognostic factors for haemoptysis Endocrinology
include increasing age, pre-existing lung or cardiac Blood gases
disease, respiratory compromise, hypoxia, ongoing
haemoptysis and shock. Respiratory causes of CSF
haemoptysis include bronchial carcinoma,
bronchiectasis, infections including tuberculosis and
aspergillus, trauma, vasculitis and arteriovenous
malformation. Systemic vasculitides may also cause
haemoptysis; these include systemic lupus
erythematosus (SLE), Wegener’s granulomatosis,
Goodpasture’s syndrome and polyarteritis nodosa.
Cardiovascular causes include pulmonary embolus,
mitral stenosis and left ventricular failure.

5197

© 2011 PasTest Ltd | About Us | Contact Us | Help

file:///E|/Shakhawan/Respiratory%20S/294a.htm[3/13/2012 4:10:22 PM]


MyPasTest

Main Navigation
Question Browser: MRCP 1
Home
Subscriptions Question Browser Exam Builder Saved Exams
» MRCP 1 Session Progress
• Question Browser
Questions Correct 1
• Timed Test A patient from a chemical factory presents with cough and
is found to have lung carcinoma. He was quite sure that it Questions Incorrect 0
• Mock Exam
was work-related. Questions Total 1
• Past Papers
What is the most likely cause? Questions Percentage 100 %
• Random Questions
• My Performance More
Coal dust
• Media Bank
Isocyanates
• New Multimedia
Polyvinyl chloride Reference: Normal Values
Online Extras
Passive smoking
Haematology
Library
Aromatic amines
Community Immunoglobulins
Help 5198 Biochemistry
PasTest Store Diabetes
Endocrinology
My Account
Blood gases
Profile
CSF
Newsletters
My Career and Exams
Order History
Learning Goals
Question Filters

Security

Change Password
Sign Out

© 2011 PasTest Ltd | About Us | Contact Us | Help

file:///E|/Shakhawan/Respiratory%20S/295.htm[3/13/2012 4:10:24 PM]


MyPasTest

Main Navigation
Question Browser: MRCP 1
Home
Subscriptions Question Browser Exam Builder Saved Exams
» MRCP 1 Session Progress
• Question Browser
Questions Correct 2
• Timed Test A patient from a chemical factory presents with cough and
is found to have lung carcinoma. He was quite sure that it Questions Incorrect 0
• Mock Exam
was work-related. Questions Total 2
• Past Papers
What is the most likely cause? Questions Percentage 100 %
• Random Questions
• My Performance More
Coal dust
• Media Bank
Isocyanates Your answer
• New Multimedia
Polyvinyl chloride
Online Extras
Passive smoking
Library
Aromatic amines
Community
Help
PasTest Store
The answer is (b). Isocyanates are a recognised risk
My Account factor for the development of non-small cell lung
cancer. Other aetiological factors include smoking,
Profile asbestos exposure, and exposure to polycyclic
hydrocarbons or the products of coal burning. Exposure
Newsletters to isocyanate fumes occurs in chemical workers,
My Career and Exams particularly those who work in the rubber industry, and
is most likely to be associated with squamous cell Reference: Normal Values
Order History
carcinoma of the bronchus. Most squamous cell
Learning Goals carcinomas present as obstructive lesions, which Haematology
Question Filters themselves present as infection. Ten per cent of cases
present with cavitating lesions, but widespread Immunoglobulins
metastases occur relatively late. Aromatic amines are
associated with bladder carcinoma, and exposure to Biochemistry
Security
vinyl chloride is associated with angiosarcoma of the Diabetes
Change Password liver.
Sign Out Endocrinology

5198 Blood gases

CSF

© 2011 PasTest Ltd | About Us | Contact Us | Help

file:///E|/Shakhawan/Respiratory%20S/295a.htm[3/13/2012 4:10:25 PM]


MyPasTest

Main Navigation
Question Browser: MRCP 1
Home
Subscriptions Question Browser Exam Builder Saved Exams
» MRCP 1 Session Progress
• Question Browser
Questions Correct 2
• Timed Test Which cell type produces surfactant?
Questions Incorrect 0
• Mock Exam
Alveolar macrophage Questions Total 2
• Past Papers
Endothelial cell Questions Percentage 100 %
• Random Questions
Goblet cell More
• My Performance
Type I pneumocyte
• Media Bank
Type II pneumocyte
• New Multimedia
Reference: Normal Values
Online Extras 5506
Haematology
Library
Community Immunoglobulins
Help Biochemistry
PasTest Store Diabetes
Endocrinology
My Account
Blood gases
Profile
CSF
Newsletters
My Career and Exams
Order History
Learning Goals
Question Filters

Security

Change Password
Sign Out

© 2011 PasTest Ltd | About Us | Contact Us | Help

file:///E|/Shakhawan/Respiratory%20S/296.htm[3/13/2012 4:10:27 PM]


MyPasTest

Main Navigation
Question Browser: MRCP 1
Home
Subscriptions Question Browser Exam Builder Saved Exams
» MRCP 1 Session Progress
• Question Browser
Questions Correct 3
• Timed Test Which cell type produces surfactant?
Questions Incorrect 0
• Mock Exam
Alveolar macrophage Questions Total 3
• Past Papers
Endothelial cell Questions Percentage 100 %
• Random Questions
Goblet cell More
• My Performance
Type I pneumocyte
• Media Bank
Type II pneumocyte Your answer
• New Multimedia
Online Extras
Library
Community
Phospholipid molecules that reduce surface tension in
Help the alveolar air–liquid interface are called surfactant.
PasTest Store Surfactant is produced in conjunction with proteins from
alveolar type II epithelial cells (Type II pneumocyte)
lying free in the alveolar spaces against alveolar walls;
My Account this material reduces surface tension severalfold.

Profile
Newsletters 5506

My Career and Exams


Order History Reference: Normal Values
Learning Goals Haematology
Question Filters
Immunoglobulins
Biochemistry
Security
Diabetes
Change Password
Sign Out Endocrinology

Blood gases

CSF

© 2011 PasTest Ltd | About Us | Contact Us | Help

file:///E|/Shakhawan/Respiratory%20S/296a.htm[3/13/2012 4:10:28 PM]


MyPasTest

Main Navigation
Question Browser: MRCP 1
Home
Subscriptions Question Browser Exam Builder Saved Exams
» MRCP 1 Session Progress
• Question Browser
Questions Correct 3
• Timed Test Which of the following is a poor prognostic factor in patients
suffering from pneumonia? Questions Incorrect 0
• Mock Exam
Questions Total 3
• Past Papers
White cell count (WCC) 17 × 10 9 /l Questions Percentage 100 %
• Random Questions
Blood pressure 110/70mmHg More
• My Performance
Respiratory rate 35/min
• Media Bank
Rigors
• New Multimedia
Temperature 39°C Reference: Normal Values
Online Extras
Haematology
Library 5552
Community Immunoglobulins
Help Biochemistry
PasTest Store Diabetes
Endocrinology
My Account
Blood gases
Profile
CSF
Newsletters
My Career and Exams
Order History
Learning Goals
Question Filters

Security

Change Password
Sign Out

© 2011 PasTest Ltd | About Us | Contact Us | Help

file:///E|/Shakhawan/Respiratory%20S/297.htm[3/13/2012 4:10:30 PM]


MyPasTest

Main Navigation
Question Browser: MRCP 1
Home
Subscriptions Question Browser Exam Builder Saved Exams
» MRCP 1 Session Progress
• Question Browser
Questions Correct 4
• Timed Test Which of the following is a poor prognostic factor in patients
suffering from pneumonia? Questions Incorrect 0
• Mock Exam
Questions Total 4
• Past Papers
White cell count (WCC) 17 × 10 9 /l Questions Percentage 100 %
• Random Questions
Blood pressure 110/70mmHg More
• My Performance
Respiratory rate 35/min Your answer
• Media Bank
Rigors
• New Multimedia
Temperature 39°C
Online Extras
Library
Community
Help The following are poor prognostic factors in patient with
PasTest Store pneumonia:

confusion: abbreviated mental test score of 8 or


My Account less

Profile urea greater than 7 mmol/l


Newsletters respiratory rate greater than 30/min
My Career and Exams
blood pressure: systolic less than 90 mmHg
Order History Reference: Normal Values
and/or diastolic less than 60mmHg.
Learning Goals Haematology
Question Filters
5552 Immunoglobulins
Biochemistry
Security
Diabetes
Change Password
Sign Out Endocrinology

Blood gases

CSF

© 2011 PasTest Ltd | About Us | Contact Us | Help

file:///E|/Shakhawan/Respiratory%20S/297a.htm[3/13/2012 4:10:31 PM]


MyPasTest

Main Navigation
Question Browser: MRCP 1
Home
Subscriptions Question Browser Exam Builder Saved Exams
» MRCP 1 Session Progress
• Question Browser
Questions Correct 4
• Timed Test Surgical resection in carcinoma of the lung is
absolutely contraindicated in the presence of which Questions Incorrect 0
• Mock Exam
one of the following? Questions Total 4
• Past Papers
Questions Percentage 100 %
• Random Questions Adenocarcinoma
• My Performance Forced expiratory volume in 1 s (FEV 1 ) 25% of More
• Media Bank predicted
• New Multimedia Pulmonary artery involvement
Reference: Normal Values
Online Extras Ischaemic heart disease
Superior vena cava obstruction Haematology
Library
Community Immunoglobulins
5559
Help Biochemistry
PasTest Store Diabetes
Endocrinology
My Account
Blood gases
Profile
CSF
Newsletters
My Career and Exams
Order History
Learning Goals
Question Filters

Security

Change Password
Sign Out

© 2011 PasTest Ltd | About Us | Contact Us | Help

file:///E|/Shakhawan/Respiratory%20S/298.htm[3/13/2012 4:10:33 PM]


MyPasTest

Main Navigation
Question Browser: MRCP 1
Home
Subscriptions Question Browser Exam Builder Saved Exams
» MRCP 1 Session Progress
• Question Browser
Questions Correct 4
• Timed Test Surgical resection in carcinoma of the lung is
absolutely contraindicated in the presence of which Questions Incorrect 1
• Mock Exam
one of the following? Questions Total 5
• Past Papers
Questions Percentage 80 %
• Random Questions Adenocarcinoma
• My Performance Forced expiratory volume in 1 s Correct answer More
• Media Bank (FEV 1 ) 25% of predicted

• New Multimedia Pulmonary artery involvement


Online Extras Ischaemic heart disease
Library Superior vena cava obstruction Your answer
Community
Help
PasTest Store
Pulmonary function assessment does not provide clear-
cut answers to the question of operability, but there are
My Account some simple rules for performing a thoracotomy. The
physician should keep in mind that the extent of
Profile resection can be determined only at operation, and
Newsletters pneumonectomy may be needed. The functional criteria
for pneumonectomy are a FEV1 of >1.5 l, and equal to
My Career and Exams
>50% of the observed forced vital capacity, plus a
Order History partial pressure of arterial CO2 (pa(CO 2 )) that is Reference: Normal Values
Learning Goals normal at rest. SVC obstruction may be extrinsic, as Haematology
such there is still a very small chance that resection
Question Filters
may be possible. Immunoglobulins
Biochemistry
Security
5559 Diabetes
Change Password
Sign Out Endocrinology

Blood gases

CSF

© 2011 PasTest Ltd | About Us | Contact Us | Help

file:///E|/Shakhawan/Respiratory%20S/298a.htm[3/13/2012 4:10:34 PM]


MyPasTest

Main Navigation
Question Browser: MRCP 1
Home
Subscriptions Question Browser Exam Builder Saved Exams
» MRCP 1 Session Progress
• Question Browser
Questions Correct 4
• Timed Test What are the NICE indications for home oxygen?
Questions Incorrect 1
• Mock Exam
Low forced expiratory volume in 1 s (FEV 1 ) Questions Total 5
• Past Papers
Low p(CO 2 ) Questions Percentage 80 %
• Random Questions
• My Performance Cor pulmonale More
• Media Bank Low p(O2 ) during exacerbations
• New Multimedia Ischaemic heart disease
Reference: Normal Values
Online Extras
5566 Haematology
Library
Community Immunoglobulins
Help Biochemistry
PasTest Store Diabetes
Endocrinology
My Account
Blood gases
Profile
CSF
Newsletters
My Career and Exams
Order History
Learning Goals
Question Filters

Security

Change Password
Sign Out

© 2011 PasTest Ltd | About Us | Contact Us | Help

file:///E|/Shakhawan/Respiratory%20S/299.htm[3/13/2012 4:10:36 PM]


MyPasTest

Main Navigation
Question Browser: MRCP 1
Home
Subscriptions Question Browser Exam Builder Saved Exams
» MRCP 1 Session Progress
• Question Browser
Questions Correct 4
• Timed Test What are the NICE indications for home oxygen?
Questions Incorrect 2
• Mock Exam
Low forced expiratory volume in 1 s Questions Total 6
• Past Papers
(FEV 1 ) Questions Percentage 66 %
• Random Questions
Low p(CO 2 )
• My Performance More
Cor pulmonale Correct answer
• Media Bank
Low p(O2 ) during exacerbations Your answer
• New Multimedia
Online Extras Ischaemic heart disease

Library
Community
Help
The National Institute for Clinical Excellence
PasTest Store (NICE)states that the following are indications for
considering long-term oxygen therapy (LTOT): patients
with a p a (O2 ) of less than 7.3 kPa when stable, or
My Account
p a (O2 ) of 7.3–8.0 kPa when stable and also an
Profile additional risk feature like secondary polycythaemia,
Newsletters nocturnal hypoxaemia, peripheral oedema or pulmonary
hypertension. Oxygen should be used for at least 15 h a
My Career and Exams day.
Order History Reference: Normal Values
Learning Goals Haematology
5566
Question Filters
Immunoglobulins
Biochemistry
Security
Diabetes
Change Password
Sign Out Endocrinology

Blood gases

CSF

© 2011 PasTest Ltd | About Us | Contact Us | Help

file:///E|/Shakhawan/Respiratory%20S/299a.htm[3/13/2012 4:10:37 PM]


MyPasTest

Main Navigation
Question Browser: MRCP 1
Home
Subscriptions Question Browser Exam Builder Saved Exams
» MRCP 1 Session Progress
• Question Browser
Questions Correct 4
• Timed Test A homeless alcoholic has been admitted to hospital with
increasing shortness of breath, sweats and purulent Questions Incorrect 2
• Mock Exam
sputum. His casualty records indicate that he was admitted Questions Total 6
• Past Papers in an unconscious state a few days ago but self-discharged
Questions Percentage 66 %
• Random Questions when he woke up. On examination there is pyrexia and
dullness at the apex associated with bronchial breathing. More
• My Performance
His chest x-ray shows right upper lobe consolidation.
• Media Bank
What is the most likely cause?
• New Multimedia
Reference: Normal Values
Online Extras Aspiration
Haematology
Library Chlamydia
Community Legionella Immunoglobulins
Help Mycoplasma Biochemistry
PasTest Store Tuberculosis Diabetes

5578 Endocrinology
My Account
Blood gases
Profile
CSF
Newsletters
My Career and Exams
Order History
Learning Goals
Question Filters

Security

Change Password
Sign Out

© 2011 PasTest Ltd | About Us | Contact Us | Help

file:///E|/Shakhawan/Respiratory%20S/300.htm[3/13/2012 4:10:39 PM]


MyPasTest

Main Navigation
Question Browser: MRCP 1
Home
Subscriptions Question Browser Exam Builder Saved Exams
» MRCP 1 Session Progress
• Question Browser
Questions Correct 5
• Timed Test A homeless alcoholic has been admitted to hospital with
increasing shortness of breath, sweats and purulent Questions Incorrect 2
• Mock Exam
sputum. His casualty records indicate that he was admitted Questions Total 7
• Past Papers in an unconscious state a few days ago but self-discharged
Questions Percentage 71 %
• Random Questions when he woke up. On examination there is pyrexia and
dullness at the apex associated with bronchial breathing. More
• My Performance
His chest x-ray shows right upper lobe consolidation.
• Media Bank
What is the most likely cause?
• New Multimedia
Online Extras Aspiration Your answer
Library Chlamydia
Community Legionella
Help Mycoplasma
PasTest Store Tuberculosis

My Account

Profile
The typical patient with aspiration pneumonia has
Newsletters gingival crevice disease combined with a predisposition
My Career and Exams for aspiration that is usually due to a suppressed level
of consciousness or dysphagia. The clinical presentation Reference: Normal Values
Order History
is usually more subtle than that for pneumococcal
Learning Goals pneumonia, in that the infection evolves over a period Haematology
Question Filters of many days, weeks, or even months. Chest
radiographs usually show infection in a dependent Immunoglobulins
segment (usually the superior segments of the lower
lobes or posterior segments of the upper lobes, since Biochemistry
Security
these are dependent in the recumbent position), fever, Diabetes
Change Password sputum that is often putrid, and evidence of chronic
disease with weight loss or anaemia is also found. Endocrinology
Sign Out
Blood gases
5578 CSF

© 2011 PasTest Ltd | About Us | Contact Us | Help

file:///E|/Shakhawan/Respiratory%20S/300a.htm[3/13/2012 4:10:40 PM]


MyPasTest

Main Navigation
Question Browser: MRCP 1
Home
Subscriptions Question Browser Exam Builder Saved Exams
» MRCP 1 Session Progress
• Question Browser
Questions Correct 0
• Timed Test A 26-year-old patient admitted with suspected pneumonia
and an abnormal chest X-ray mentions to the attending Questions Incorrect 0
• Mock Exam
physician that he has an azygous lobe. Where would you Questions Total 0
• Past Papers visualize the azygous lobe on an anterior-posterior chest X-
Questions Percentage 0%
• Random Questions ray?
• My Performance More
Right upper zone
• Media Bank
Right middle zone
• New Multimedia
Right lower zone Reference: Normal Values
Online Extras
Left upper zone Haematology
Library
Left lower zone
Community Immunoglobulins
Help 5582 Biochemistry
PasTest Store Diabetes
Endocrinology
My Account
Blood gases
Profile
CSF
Newsletters
My Career and Exams
Order History
Learning Goals
Question Filters

Security

Change Password
Sign Out

© 2011 PasTest Ltd | About Us | Contact Us | Help

file:///E|/Shakhawan/Respiratory%20S/301.htm[3/13/2012 4:10:42 PM]


MyPasTest

Main Navigation
Question Browser: MRCP 1
Home
Subscriptions Question Browser Exam Builder Saved Exams
» MRCP 1 Session Progress
• Question Browser
Questions Correct 0
• Timed Test A 26-year-old patient admitted with suspected pneumonia
and an abnormal chest X-ray mentions to the attending Questions Incorrect 1
• Mock Exam
physician that he has an azygous lobe. Where would you Questions Total 1
• Past Papers visualize the azygous lobe on an anterior-posterior chest X-
Questions Percentage 0%
• Random Questions ray?
• My Performance More
Right upper zone Correct answer
• Media Bank
Right middle zone Your answer
• New Multimedia
Right lower zone
Online Extras
Left upper zone
Library
Left lower zone
Community
Help
PasTest Store
An azygous lobe is seen in about 0.5% of routine chest
My Account X-rays and is a normal variant. It is seen as a ‘reverse
comma sign’ behind the medial end of the right clavicle.
Profile
Newsletters
5582
My Career and Exams
Order History Reference: Normal Values
Learning Goals Haematology
Question Filters
Immunoglobulins
Biochemistry
Security
Diabetes
Change Password
Sign Out Endocrinology

Blood gases

CSF

© 2011 PasTest Ltd | About Us | Contact Us | Help

file:///E|/Shakhawan/Respiratory%20S/301a.htm[3/13/2012 4:10:43 PM]


MyPasTest

Main Navigation
Question Browser: MRCP 1
Home
Subscriptions Question Browser Exam Builder Saved Exams
» MRCP 1 Session Progress
• Question Browser
Questions Correct 0
• Timed Test Surgical resection in carcinoma of the lung is
absolutely contraindicated in the presence of which Questions Incorrect 1
• Mock Exam
one of the following? Questions Total 1
• Past Papers
Questions Percentage 0%
• Random Questions Adenocarcinoma
• My Performance Forced expiratory volume in 1 s (FEV 1 ) = 1.4 l More
• Media Bank Mediastinal lymph nodes < 1cm diameter
• New Multimedia Paraneoplastic syndrome Reference: Normal Values
Online Extras Severe pulmonary hypertension
Haematology
Library
5584 Immunoglobulins
Community
Help Biochemistry
PasTest Store Diabetes
Endocrinology
My Account
Blood gases
Profile
CSF
Newsletters
My Career and Exams
Order History
Learning Goals
Question Filters

Security

Change Password
Sign Out

© 2011 PasTest Ltd | About Us | Contact Us | Help

file:///E|/Shakhawan/Respiratory%20S/302.htm[3/13/2012 4:10:45 PM]


MyPasTest

Main Navigation
Question Browser: MRCP 1
Home
Subscriptions Question Browser Exam Builder Saved Exams
» MRCP 1 Session Progress
• Question Browser
Questions Correct 1
• Timed Test Surgical resection in carcinoma of the lung is
absolutely contraindicated in the presence of which Questions Incorrect 1
• Mock Exam
one of the following? Questions Total 2
• Past Papers
Questions Percentage 50 %
• Random Questions Adenocarcinoma
• My Performance Forced expiratory volume in 1 s (FEV 1 ) Your answer More
• Media Bank = 1.4 l
• New Multimedia Mediastinal lymph nodes < 1cm
diameter
Online Extras
Paraneoplastic syndrome
Library
Severe pulmonary hypertension
Community
Help
PasTest Store

Pulmonary function assessment does not provide clear-


My Account cut answers to the question of operability, but there are
some simple rules for performing a thoracotomy. The
Profile physician should keep in mind that the extent of
Newsletters resection can be determined only at operation, and
pneumonectomy may be needed. The functional criteria
My Career and Exams for pneumonectomy are an FEV1 of >1.5 l, and equal to
Order History >50% of the observed forced vital capacity plus a Reference: Normal Values
Learning Goals partial pressure of arterial CO2 p a (CO 2 ) that is normal
Haematology
at rest.
Question Filters
Immunoglobulins

5584 Biochemistry
Security
Diabetes
Change Password
Sign Out Endocrinology

Blood gases

CSF

© 2011 PasTest Ltd | About Us | Contact Us | Help

file:///E|/Shakhawan/Respiratory%20S/302a.htm[3/13/2012 4:10:46 PM]


MyPasTest

Main Navigation
Question Browser: MRCP 1
Home
Subscriptions Question Browser Exam Builder Saved Exams
» MRCP 1 Session Progress
• Question Browser
Questions Correct 1
• Timed Test A 40-year-old male complains of increasing shortness of
breath, his chest X-ray shows an elevated hemidiaphragm Questions Incorrect 1
• Mock Exam
on the left side, no other abnormalities are seen. Questions Total 2
• Past Papers
What is the most likely investigation to elucidate the Questions Percentage 50 %
• Random Questions
mechanical reason for his shortness of breath? More
• My Performance
• Media Bank CT Thorax
• New Multimedia Magnetic resonance imaging (MRI) scan
Reference: Normal Values
Online Extras Echocardiography
Haematology
Library Fluoroscopy
Community Electrocardiogram (ECG) Immunoglobulins
Help Biochemistry
5606
PasTest Store Diabetes
Endocrinology
My Account
Blood gases
Profile
CSF
Newsletters
My Career and Exams
Order History
Learning Goals
Question Filters

Security

Change Password
Sign Out

© 2011 PasTest Ltd | About Us | Contact Us | Help

file:///E|/Shakhawan/Respiratory%20S/303.htm[3/13/2012 4:10:48 PM]


MyPasTest

Main Navigation
Question Browser: MRCP 1
Home
Subscriptions Question Browser Exam Builder Saved Exams
» MRCP 1 Session Progress
• Question Browser
Questions Correct 2
• Timed Test A 40-year-old male complains of increasing shortness of
breath, his chest X-ray shows an elevated hemidiaphragm Questions Incorrect 1
• Mock Exam
on the left side, no other abnormalities are seen. Questions Total 3
• Past Papers
What is the most likely investigation to elucidate the Questions Percentage 66 %
• Random Questions
mechanical reason for his shortness of breath? More
• My Performance
• Media Bank CT Thorax
• New Multimedia Magnetic resonance imaging (MRI)
Online Extras scan

Library Echocardiography

Community Fluoroscopy Your answer

Help Electrocardiogram (ECG)

PasTest Store

My Account
The diagnosis of unilateral paralysis, suggested by
Profile asymmetric elevation of the affected hemidiaphragm on
X-ray, can be confirmed by fluoroscopy. During a forced
Newsletters inspiratory maneuver (the ‘sniff’ test), the unaffected
My Career and Exams hemidiaphragm descends forcefully, increasing intra-
abdominal pressure and pushing the paralysed Reference: Normal Values
Order History
hemidiaphragm cephalad (paradoxical motion).
Learning Goals Fluoroscopy is inaccurate for the diagnosis of bilateral Haematology
Question Filters paralysis. Whilst MRI may demonstrate a structural
defect, it isn’t a dynamic investigation. Immunoglobulins
Biochemistry
Security
5606 Diabetes
Change Password
Sign Out Endocrinology

Blood gases

CSF

© 2011 PasTest Ltd | About Us | Contact Us | Help

file:///E|/Shakhawan/Respiratory%20S/303a.htm[3/13/2012 4:10:49 PM]


MyPasTest

Main Navigation
Question Browser: MRCP 1
Home
Subscriptions Question Browser Exam Builder Saved Exams
» MRCP 1 Session Progress
• Question Browser
Questions Correct 2
• Timed Test A 40-year-old male complains of increasing shortness of
breath, his chest X-ray shows an elevated hemidiaphragm Questions Incorrect 1
• Mock Exam
on the left side. Questions Total 3
• Past Papers
Which of the following features would be seen during Questions Percentage 66 %
• Random Questions
the sniff test? More
• My Performance
• Media Bank Decreased vital capacity in a supine position
• New Multimedia Decreased vital capacity while standing
Reference: Normal Values
Online Extras Paradoxical hemidiaphragm movement
Haematology
Library Decreased transfer coefficient (Kco)
Community Increasedtransfer coefficient (Kco) Immunoglobulins
Help Biochemistry
5607
PasTest Store Diabetes
Endocrinology
My Account
Blood gases
Profile
CSF
Newsletters
My Career and Exams
Order History
Learning Goals
Question Filters

Security

Change Password
Sign Out

© 2011 PasTest Ltd | About Us | Contact Us | Help

file:///E|/Shakhawan/Respiratory%20S/304.htm[3/13/2012 4:10:51 PM]


MyPasTest

Main Navigation
Question Browser: MRCP 1
Home
Subscriptions Question Browser Exam Builder Saved Exams
» MRCP 1 Session Progress
• Question Browser
Questions Correct 3
• Timed Test A 40-year-old male complains of increasing shortness of
breath, his chest X-ray shows an elevated hemidiaphragm Questions Incorrect 1
• Mock Exam
on the left side. Questions Total 4
• Past Papers
Which of the following features would be seen during Questions Percentage 75 %
• Random Questions
the sniff test? More
• My Performance
• Media Bank Decreased vital capacity in a supine
• New Multimedia position

Online Extras Decreased vital capacity while standing

Library Paradoxical hemidiaphragm movement Your answer

Community Decreased transfer coefficient (Kco)

Help Increasedtransfer coefficient (Kco)

PasTest Store

My Account
The diagnosis of unilateral paralysis, suggested by
Profile asymmetric elevation of the affected hemidiaphragm on
X-ray, can be confirmed by fluoroscopy. During a forced
Newsletters inspiratory manoeuvre (the ‘sniff’ test), the unaffected
My Career and Exams hemidiaphragm descends forcefully, increasing intra-
abdominal pressure and pushing the paralysed Reference: Normal Values
Order History
hemidiaphragm cephalad (paradoxical motion).
Learning Goals Haematology
Question Filters
5607 Immunoglobulins
Biochemistry
Security
Diabetes
Change Password
Sign Out Endocrinology

Blood gases

CSF

© 2011 PasTest Ltd | About Us | Contact Us | Help

file:///E|/Shakhawan/Respiratory%20S/304a.htm[3/13/2012 4:10:52 PM]


MyPasTest

Main Navigation
Question Browser: MRCP 1
Home
Subscriptions Question Browser Exam Builder Saved Exams
» MRCP 1 Session Progress
• Question Browser
Questions Correct 3
• Timed Test A 42-year-old patient was diagnosed with mediastinal
tuberculosis 3 weeks ago and commenced on treatment. He Questions Incorrect 1
• Mock Exam
presents with worsening breathlessness and stridor. His Questions Total 4
• Past Papers chest X-ray shows mediastinal lymph nodes compressing
Questions Percentage 75 %
• Random Questions the carina area, whilst he is distressed he is alert and
orientated and maintaining his oxygenation. You give him More
• My Performance
an injection of hydrocortisone
• Media Bank
What is the next step in the investigation and/or
• New Multimedia
management? Reference: Normal Values
Online Extras
Urgent computed tomography (CT) scan Haematology
Library
Community Tracheostomy Immunoglobulins
Help Heparin Biochemistry
PasTest Store Ethambutol
Diabetes
Broad spectrum antibiotics
Endocrinology
My Account 5608
Blood gases
Profile
CSF
Newsletters
My Career and Exams
Order History
Learning Goals
Question Filters

Security

Change Password
Sign Out

© 2011 PasTest Ltd | About Us | Contact Us | Help

file:///E|/Shakhawan/Respiratory%20S/305.htm[3/13/2012 4:10:54 PM]


MyPasTest

Main Navigation
Question Browser: MRCP 1
Home
Subscriptions Question Browser Exam Builder Saved Exams
» MRCP 1 Session Progress
• Question Browser
Questions Correct 4
• Timed Test A 42-year-old patient was diagnosed with mediastinal
tuberculosis 3 weeks ago and commenced on treatment. He Questions Incorrect 1
• Mock Exam
presents with worsening breathlessness and stridor. His Questions Total 5
• Past Papers chest X-ray shows mediastinal lymph nodes compressing
Questions Percentage 80 %
• Random Questions the carina area, whilst he is distressed he is alert and
orientated and maintaining his oxygenation. You give him More
• My Performance
an injection of hydrocortisone
• Media Bank
What is the next step in the investigation and/or
• New Multimedia
management?
Online Extras
Library Urgent computed tomography (CT) Your answer
scan
Community
Tracheostomy
Help
Heparin
PasTest Store
Ethambutol
Broad spectrum antibiotics
My Account

Profile
Newsletters
My Career and Exams The CT scan is the most appropriate investigation. On
enhanced CT scans, nodes larger than 2 cm in diameter Reference: Normal Values
Order History
invariably show central areas of low attenuation and
Learning Goals peripheral rim enhancement. Enhanced walls are usually Haematology
Question Filters irregular in thickness. CT scan is useful prior to
commencing steroid therapy, a) to confirm the degree Immunoglobulins
of airway compression and b) to assess the response to
chemotherapy. A number of commentators recommend Biochemistry
Security
rapid initiation of steroid therapy, and this would seem Diabetes
Change Password sensible prior to progression to CT scanning.
Sign Out Endocrinology

5608 Blood gases

CSF

© 2011 PasTest Ltd | About Us | Contact Us | Help

file:///E|/Shakhawan/Respiratory%20S/305a.htm[3/13/2012 4:10:56 PM]


MyPasTest

Main Navigation
Question Browser: MRCP 1
Home
Subscriptions Question Browser Exam Builder Saved Exams
» MRCP 1 Session Progress
• Question Browser
Questions Correct 4
• Timed Test A patient presents with shortness of breath. His transfer
coefficient (KCO) is 160% of predicted. What is the most Questions Incorrect 1
• Mock Exam
likely cause? Questions Total 5
• Past Papers
Questions Percentage 80 %
• Random Questions Fibrosing alveolitis
• My Performance Asthma More
• Media Bank Pulmonary haemorrhage
• New Multimedia Pulmonary embolus
Reference: Normal Values
Online Extras Pneumonia
Haematology
Library
5609
Community Immunoglobulins
Help Biochemistry
PasTest Store Diabetes
Endocrinology
My Account
Blood gases
Profile
CSF
Newsletters
My Career and Exams
Order History
Learning Goals
Question Filters

Security

Change Password
Sign Out

© 2011 PasTest Ltd | About Us | Contact Us | Help

file:///E|/Shakhawan/Respiratory%20S/306.htm[3/13/2012 4:10:57 PM]


MyPasTest

Main Navigation
Question Browser: MRCP 1
Home
Subscriptions Question Browser Exam Builder Saved Exams
» MRCP 1 Session Progress
• Question Browser
Questions Correct 5
• Timed Test A patient presents with shortness of breath. His transfer
coefficient (KCO) is 160% of predicted. What is the most Questions Incorrect 1
• Mock Exam
likely cause? Questions Total 6
• Past Papers
Questions Percentage 83 %
• Random Questions Fibrosing alveolitis
• My Performance Asthma More
• Media Bank Pulmonary haemorrhage Your answer
• New Multimedia Pulmonary embolus
Online Extras Pneumonia
Library
Community
Help
PasTest Store The transfer coefficient (Kco), which is obtained along
with the TLco, represents the uptake of carbon
monoxide per litre of effective alveolar volume (VA),
My Account that is, Kco = TLco /VA. To a large extent, Kco allows
correction for any real or effective reduction of alveolar
Profile volume, tending to be normal after lung resection,
when both TLco and VA are reduced approximately to
Newsletters
the same degree. In some conditions Kco may increase.
My Career and Exams The latter usually results from an increase in red blood
Order History cells in the lungs due to greater blood flow, Reference: Normal Values
haemorrhage, or polycythaemia. Kco is similarly
Learning Goals increased in these conditions, as it is if, at full inflation, Haematology
Question Filters the density of pulmonary capillaries per unit alveolar
volume is greater than normal. This occurs most Immunoglobulins
commonly in patients with extrapulmonary volume
Biochemistry
Security restriction, when the density of pulmonary capillaries is
unusually high in relation to the (restricted) lung Diabetes
Change Password volume at which the measurement is made.
Sign Out Endocrinology

Blood gases
5609
CSF

© 2011 PasTest Ltd | About Us | Contact Us | Help

file:///E|/Shakhawan/Respiratory%20S/306a.htm[3/13/2012 4:10:58 PM]


MyPasTest

Main Navigation
Question Browser: MRCP 1
Home
Subscriptions Question Browser Exam Builder Saved Exams
» MRCP 1 Session Progress
• Question Browser
Questions Correct 5
• Timed Test In which pulmonary disease is the alveolar structure
preserved? Questions Incorrect 1
• Mock Exam
Questions Total 6
• Past Papers
Asthma Questions Percentage 83 %
• Random Questions
Pneumonia
• My Performance More
Pulmonary fibrosis
• Media Bank
Allergic alveolitis
• New Multimedia
Toxic alveolitis Reference: Normal Values
Online Extras
5610 Haematology
Library
Community Immunoglobulins
Help Biochemistry
PasTest Store Diabetes
Endocrinology
My Account
Blood gases
Profile
CSF
Newsletters
My Career and Exams
Order History
Learning Goals
Question Filters

Security

Change Password
Sign Out

© 2011 PasTest Ltd | About Us | Contact Us | Help

file:///E|/Shakhawan/Respiratory%20S/307.htm[3/13/2012 4:11:00 PM]


MyPasTest

Main Navigation
Question Browser: MRCP 1
Home
Subscriptions Question Browser Exam Builder Saved Exams
» MRCP 1 Session Progress
• Question Browser
Questions Correct 6
• Timed Test In which pulmonary disease is the alveolar structure
preserved? Questions Incorrect 1
• Mock Exam
Questions Total 7
• Past Papers
Asthma Your answer Questions Percentage 85 %
• Random Questions
Pneumonia
• My Performance More
Pulmonary fibrosis
• Media Bank
Allergic alveolitis
• New Multimedia
Toxic alveolitis
Online Extras
Library
Community
Help Asthma is a chronic inflammatory disease of the
PasTest Store bronchial airways, which is characterised by a
desquamative eosinophilic bronchitis. The defining
clinical characteristics of asthma are reversible airway
My Account narrowing and increased airway responsiveness to non-
specific provocative stimuli. The alveolar functional
Profile structure is preserved.
Newsletters
My Career and Exams 5610
Order History Reference: Normal Values
Learning Goals Haematology
Question Filters
Immunoglobulins
Biochemistry
Security
Diabetes
Change Password
Sign Out Endocrinology

Blood gases

CSF

© 2011 PasTest Ltd | About Us | Contact Us | Help

file:///E|/Shakhawan/Respiratory%20S/307a.htm[3/13/2012 4:11:01 PM]


MyPasTest

Main Navigation
Question Browser: MRCP 1
Home
Subscriptions Question Browser Exam Builder Saved Exams
» MRCP 1 Session Progress
• Question Browser
Questions Correct 6
• Timed Test You are asked to see a 32-year-old immigrant who
complains of chronic cough and weight loss over the past Questions Incorrect 1
• Mock Exam
few months. Examination of sputum reveals acid and Questions Total 7
• Past Papers alcohol fast bacilli (AAFBs) and tuberculosis is confirmed.
Questions Percentage 85 %
• Random Questions You elect to begin treatment with isoniazid, rifampicin,
ethambutol and pyrazinamide as he is from an area where More
• My Performance
high levels of drug resistance are present.
• Media Bank
• New Multimedia Which of the following blood tests is most desirable
before starting therapy? Reference: Normal Values
Online Extras
Liver function testing Haematology
Library
Community Serum calcium Immunoglobulins
Help Platelet count Biochemistry
PasTest Store Clotting
Diabetes
Haemoglobin
Endocrinology
My Account
8260 Blood gases
Profile
CSF
Newsletters
My Career and Exams
Order History
Learning Goals
Question Filters

Security

Change Password
Sign Out

© 2011 PasTest Ltd | About Us | Contact Us | Help

file:///E|/Shakhawan/Respiratory%20S/308.htm[3/13/2012 4:11:03 PM]


MyPasTest

Main Navigation
Question Browser: MRCP 1
Home
Subscriptions Question Browser Exam Builder Saved Exams
» MRCP 1 Session Progress
• Question Browser
Questions Correct 7
• Timed Test You are asked to see a 32-year-old immigrant who
complains of chronic cough and weight loss over the past Questions Incorrect 1
• Mock Exam
few months. Examination of sputum reveals acid and Questions Total 8
• Past Papers alcohol fast bacilli (AAFBs) and tuberculosis is confirmed.
Questions Percentage 87 %
• Random Questions You elect to begin treatment with isoniazid, rifampicin,
ethambutol and pyrazinamide as he is from an area where More
• My Performance
high levels of drug resistance are present.
• Media Bank
• New Multimedia Which of the following blood tests is most desirable
before starting therapy?
Online Extras
Library Liver function testing Your answer
Community Serum calcium
Help Platelet count
PasTest Store Clotting
Haemoglobin
My Account

Profile
Newsletters
Both isoniazid and rifampicin may be associated with
My Career and Exams significant hepatic dysfunction. In particular, severe and
Order History sometimes fatal hepatitis has been seen with use of Reference: Normal Values
isoniazid. Particular problems occur in slow acetylators
Learning Goals who have markedly elevated serum isoniazid levels. In Haematology
Question Filters patients with existing liver dysfunction, rifampicin and
isoniazid should only be used in cases of absolute Immunoglobulins
clinical necessity. Even then, dose reduction of
Biochemistry
Security rifampicin is recommended and initial weekly monitoring
of liver function tests should be carried out. Diabetes
Change Password
Sign Out Endocrinology
8260
Blood gases

CSF

© 2011 PasTest Ltd | About Us | Contact Us | Help

file:///E|/Shakhawan/Respiratory%20S/308a.htm[3/13/2012 4:11:04 PM]


MyPasTest

Main Navigation
Question Browser: MRCP 1
Home
Subscriptions Question Browser Exam Builder Saved Exams
» MRCP 1 Session Progress
• Question Browser
Questions Correct 7
• Timed Test A 68-year-old man who has a long history of smoking
presents to the Emergency Department with worsening Questions Incorrect 1
• Mock Exam
shortness of breath. His general health has deteriorated Questions Total 8
• Past Papers over the past few months and recently he has been
Questions Percentage 87 %
• Random Questions prescribed a salbutamol inhaler by his GP for cough and
wheezing, particularly on exercise and at night. On More
• My Performance
examination in the Emergency Department he is lip pursing
• Media Bank and has considerable wheeze on auscultation of the chest.
• New Multimedia He is pyrexial at 37.8°C and has purulent sputum. He can
only manage a peak flow of 150 l/min. Arterial blood gas Reference: Normal Values
Online Extras sampling reveals a p a (O2 ) of 7.2 kPa.
Haematology
Library
Community Which of the following would be the most appropriate Immunoglobulins
choice for antibiotic therapy in this man?
Help Biochemistry
PasTest Store Clarithromycin 500 mg po bd
Diabetes
Penicillin V 500 mg po qds
Endocrinology
My Account Metronidazole 500 mg po tds
Cefotaxime 1 g iv tds Blood gases
Profile
Ciprofloxacin 500 mg po bd CSF
Newsletters
My Career and Exams 8262
Order History
Learning Goals
Question Filters

Security

Change Password
Sign Out

© 2011 PasTest Ltd | About Us | Contact Us | Help

file:///E|/Shakhawan/Respiratory%20S/309.htm[3/13/2012 4:11:06 PM]


MyPasTest

Main Navigation
Question Browser: MRCP 1
Home
Subscriptions Question Browser Exam Builder Saved Exams
» MRCP 1 Session Progress
• Question Browser
Questions Correct 7
• Timed Test A 68-year-old man who has a long history of smoking
presents to the Emergency Department with worsening Questions Incorrect 2
• Mock Exam
shortness of breath. His general health has deteriorated Questions Total 9
• Past Papers over the past few months and recently he has been
Questions Percentage 77 %
• Random Questions prescribed a salbutamol inhaler by his GP for cough and
wheezing, particularly on exercise and at night. On More
• My Performance
examination in the Emergency Department he is lip pursing
• Media Bank and has considerable wheeze on auscultation of the chest.
• New Multimedia He is pyrexial at 37.8°C and has purulent sputum. He can
only manage a peak flow of 150 l/min. Arterial blood gas
Online Extras sampling reveals a p a (O2 ) of 7.2 kPa.
Library
Community Which of the following would be the most appropriate
choice for antibiotic therapy in this man?
Help
PasTest Store Clarithromycin 500 mg po bd Correct answer
Penicillin V 500 mg po qds
My Account Metronidazole 500 mg po tds
Cefotaxime 1 g iv tds Your answer
Profile
Ciprofloxacin 500 mg po bd
Newsletters
My Career and Exams
Order History Reference: Normal Values
Learning Goals Haematology
This man has an exacerbation of chronic pulmonary
Question Filters disease (COPD). The most common causative pathogens
Immunoglobulins
are Haemophilus influenzae and Moraxella catarrhalis.
The most appropriate antibiotic therefore would be a Biochemistry
Security macrolide such as clarithromycin. It should be noted
however that in patients taking theophyllines, Diabetes
Change Password concomitant use of macrolides might increase serum
Sign Out aminophylline levels. Other important acute treatment Endocrinology
in this case would be repeated nebulisation with Blood gases
salbutamol and ipratropium 4–6 hourly, oxygen
therapy, and oral steroids. Non-invasive ventilation may CSF
be considered where the pH is 7.3 or less and the
patient is not improving after 4 h of medical therapy.

8262

© 2011 PasTest Ltd | About Us | Contact Us | Help

file:///E|/Shakhawan/Respiratory%20S/309a.htm[3/13/2012 4:11:07 PM]


MyPasTest

Main Navigation
Question Browser: MRCP 1
Home
Subscriptions Question Browser Exam Builder Saved Exams
» MRCP 1 Session Progress
• Question Browser
Questions Correct 7
• Timed Test A 67-year-old man consults his doctor complaining of a
painful mouth and increasing difficulty eating. He has a Questions Incorrect 2
• Mock Exam
past history of smoking and has chronic pulmonary disease Questions Total 9
• Past Papers (COPD). Medication history includes use of
Questions Percentage 77 %
• Random Questions fluticasone/salmeterol combination inhaler and omeprazole
for indigestion. On examination he has a body mass index More
• My Performance
(BMI) of 29 and looks well. There is extensive stomatitis
• Media Bank and pharyngitis on examination of the oropharynx, with
• New Multimedia white plaques on examination of the tongue. Full blood
count, U&E, liver function test (LFT) and viscosity are Reference: Normal Values
Online Extras normal.
Haematology
Library
Which of the following would be the most appropriate Immunoglobulins
Community management in this case?
Help Biochemistry
PasTest Store Arrange urgent upper gastrointestinal (GI) endoscopy
Diabetes
Arrange urgent barium swallow
Stop his inhaled steroids Endocrinology
My Account
Advise him to rinse the mouth each time he uses his Blood gases
Profile inhaler and use a spacer device and review him in a
month CSF
Newsletters
My Career and Exams Increase his dose of omeprazole
Order History
8263
Learning Goals
Question Filters

Security

Change Password
Sign Out

© 2011 PasTest Ltd | About Us | Contact Us | Help

file:///E|/Shakhawan/Respiratory%20S/310.htm[3/13/2012 4:11:09 PM]


MyPasTest

Main Navigation
Question Browser: MRCP 1
Home
Subscriptions Question Browser Exam Builder Saved Exams
» MRCP 1 Session Progress
• Question Browser
Questions Correct 8
• Timed Test A 67-year-old man consults his doctor complaining of a
painful mouth and increasing difficulty eating. He has a Questions Incorrect 2
• Mock Exam
past history of smoking and has chronic pulmonary disease Questions Total 10
• Past Papers (COPD). Medication history includes use of
Questions Percentage 80 %
• Random Questions fluticasone/salmeterol combination inhaler and omeprazole
for indigestion. On examination he has a body mass index More
• My Performance
(BMI) of 29 and looks well. There is extensive stomatitis
• Media Bank and pharyngitis on examination of the oropharynx, with
• New Multimedia white plaques on examination of the tongue. Full blood
count, U&E, liver function test (LFT) and viscosity are
Online Extras normal.
Library
Which of the following would be the most appropriate
Community management in this case?
Help
PasTest Store Arrange urgent upper gastrointestinal
(GI) endoscopy
Arrange urgent barium swallow
My Account
Stop his inhaled steroids
Profile Advise him to rinse the mouth each Your answer
Newsletters time he uses his inhaler and use a
spacer device and review him in a
My Career and Exams month
Order History Increase his dose of omeprazole Reference: Normal Values
Learning Goals Haematology
Question Filters
Immunoglobulins
Biochemistry
Security This man almost certainly has oro-pharyngeal and
oesophageal candidiasis from inadequate hygiene after Diabetes
Change Password using his inhaler. He should be taught adequate inhaler
Sign Out technique and urged to rinse the mouth after each use. Endocrinology
The lack of weight loss and normal investigations
makes underlying malignancy unlikely, although a Blood gases
symptom checkin 1 month is still advisable. Resistant CSF
symptoms may be managed with oral nystatin or a
course of fluconazole.

8263

© 2011 PasTest Ltd | About Us | Contact Us | Help

file:///E|/Shakhawan/Respiratory%20S/310a.htm[3/13/2012 4:11:10 PM]


MyPasTest

Main Navigation
Question Browser: MRCP 1
Home
Subscriptions Question Browser Exam Builder Saved Exams
» MRCP 1 Session Progress
• Question Browser
Questions Correct 8
• Timed Test A 29-year-old woman is admitted complaining of
intermittent pleuritic chest pain and shortness of breath. Questions Incorrect 2
• Mock Exam
She is using the oral contraceptive pill. Other past history Questions Total 10
• Past Papers of note includes recent separation from a violent man
Questions Percentage 80 %
• Random Questions partner and a recent attack of shingles, which has now
resolved. On examination her pulse is 74/min and regular, More
• My Performance
with blood pressure of 124/72 mmHg. Saturations are 98%
• Media Bank on air and she does not de-saturate on exercise.
• New Multimedia
Arterial blood gasses reveal: Reference: Normal Values
Online Extras
· pH 7.48 Haematology
Library
Community · p a (O2 ) 11.1 kPa Immunoglobulins
Help Biochemistry
· p a (CO 2 ) 3.1 kPa
PasTest Store Diabetes
· D-dimers are negative
Endocrinology
My Account · free thyroxine (T4) is 17.1 pmol/l (10–22)
Blood gases
Profile · chest X-ray is unremarkable
CSF
Newsletters
Which of the following represents the most likely diagnosis
My Career and Exams in this case?
Order History
Hyperthyroidism
Learning Goals
Chronic thrombo-embolic disease
Question Filters
Chickenpox pneumonitis
Hyperventilation syndrome
Security
Diffuse interstitial lung disease
Change Password
Sign Out 10699

© 2011 PasTest Ltd | About Us | Contact Us | Help

file:///E|/Shakhawan/Respiratory%20S/311.htm[3/13/2012 4:11:12 PM]


MyPasTest

Main Navigation
Question Browser: MRCP 1
Home
Subscriptions Question Browser Exam Builder Saved Exams
» MRCP 1 Session Progress
• Question Browser
Questions Correct 9
• Timed Test A 29-year-old woman is admitted complaining of
intermittent pleuritic chest pain and shortness of breath. Questions Incorrect 2
• Mock Exam
She is using the oral contraceptive pill. Other past history Questions Total 11
• Past Papers of note includes recent separation from a violent man
Questions Percentage 81 %
• Random Questions partner and a recent attack of shingles, which has now
resolved. On examination her pulse is 74/min and regular, More
• My Performance
with blood pressure of 124/72 mmHg. Saturations are 98%
• Media Bank on air and she does not de-saturate on exercise.
• New Multimedia
Arterial blood gasses reveal:
Online Extras
· pH 7.48
Library
Community · p a (O2 ) 11.1 kPa
Help
· p a (CO 2 ) 3.1 kPa
PasTest Store
· D-dimers are negative
My Account · free thyroxine (T4) is 17.1 pmol/l (10–22)
Profile · chest X-ray is unremarkable
Newsletters
Which of the following represents the most likely diagnosis
My Career and Exams in this case?
Order History Reference: Normal Values
Hyperthyroidism
Learning Goals Haematology
Chronic thrombo-embolic disease
Question Filters
Chickenpox pneumonitis Immunoglobulins

Hyperventilation syndrome Your answer Biochemistry


Security
Diffuse interstitial lung disease Diabetes
Change Password
Sign Out Endocrinology

Blood gases
The blood gases and presentation fit this scenario well. CSF
Diagnosis is made if no cardiac or respiratory cause can
be found for the breathlessness, as exemplified by the
normal physical examination, chest X-ray and D-dimers
seen here. The blood gas picture also supports the
diagnosis of hyperventilation syndrome. In addition,
thyrotoxicosis as a precipitant for attacks can be ruled
out, the free T4 here being in the normal range. Many
patients often gain benefit from a short course of
physiotherapy from a therapist experienced in managing
the syndrome. While diffuse interstitial lung disease is a
differential diagnosis here, given that there is a history
of significant life events that may have triggered the
attacks, hyperventilation syndrome is more likely.

10699

© 2011 PasTest Ltd | About Us | Contact Us | Help

file:///E|/Shakhawan/Respiratory%20S/311a.htm[3/13/2012 4:11:13 PM]


MyPasTest

Main Navigation
Question Browser: MRCP 1
Home
Subscriptions Question Browser Exam Builder Saved Exams
» MRCP 1 Session Progress
• Question Browser
Questions Correct 9
• Timed Test You are asked to see a 57-year-old smoker, who complains
of shortness of breath some 7 days after a total hip Questions Incorrect 2
• Mock Exam
replacement. On examination he is obese and has a Questions Total 11
• Past Papers swollen left leg, and he is visibly short of breath. There
Questions Percentage 81 %
• Random Questions appears to be increased prominence of vascular markings
at the right hilum on chest X-ray. His calculated alveolar– More
• My Performance
arterial (A–a) gradient is 4.5 kPa.
• Media Bank
Which of the following fits best with his diagnosis?
• New Multimedia
Reference: Normal Values
Online Extras Postoperative pneumonia
Haematology
Library Pulmonary embolus
Community Hyperventilation syndrome Immunoglobulins
Help Atelectasis Biochemistry
PasTest Store Pneumothorax Diabetes

10700 Endocrinology
My Account
Blood gases
Profile
CSF
Newsletters
My Career and Exams
Order History
Learning Goals
Question Filters

Security

Change Password
Sign Out

© 2011 PasTest Ltd | About Us | Contact Us | Help

file:///E|/Shakhawan/Respiratory%20S/312.htm[3/13/2012 4:11:15 PM]


MyPasTest

Main Navigation
Question Browser: MRCP 1
Home
Subscriptions Question Browser Exam Builder Saved Exams
» MRCP 1 Session Progress
• Question Browser
Questions Correct 10
• Timed Test You are asked to see a 57-year-old smoker, who complains
of shortness of breath some 7 days after a total hip Questions Incorrect 2
• Mock Exam
replacement. On examination he is obese and has a Questions Total 12
• Past Papers swollen left leg, and he is visibly short of breath. There
Questions Percentage 83 %
• Random Questions appears to be increased prominence of vascular markings
at the right hilum on chest X-ray. His calculated alveolar– More
• My Performance
arterial (A–a) gradient is 4.5 kPa.
• Media Bank
Which of the following fits best with his diagnosis?
• New Multimedia
Online Extras Postoperative pneumonia
Library Pulmonary embolus Your answer
Community Hyperventilation syndrome
Help Atelectasis
PasTest Store Pneumothorax

My Account

Profile
The alveolar–arterial ( A–a) gradient is affected
Newsletters primarily by V/Q mismatch and shunting. This situation
My Career and Exams occurs in pulmonary embolism and in pneumonia and
given the clinical history and findings, pulmonary Reference: Normal Values
Order History
embolus is the most likely diagnosis here. The alveolar–
Learning Goals arterial (A–a) gradient is calculated with the following Haematology
Question Filters equation
Immunoglobulins
p A(O2 ) – (p a (O2 ) + p a (CO 2 )/0.8)
Biochemistry
Security
p A(O2 ) = alveolar oxygen. Diabetes
Change Password
Calculated by: (100 – 7) ´ % inspired O 2 Endocrinology
Sign Out
eg for 21% O 2 , the p A(O2 ) would be 20 kPa Blood gases

CSF
p a (O2 ) and p a (CO 2 ) are arterial O 2 and CO2 levels.

This illustrates how the ‘normal’ alveolar–arterial ( A–a)


gradient also varies according to level of inspired
oxygen. This means that if you use a nomogram for
calculating the alveolar–arterial (A–a) gradient, you
must make sure you are looking at the correct slope
line for the patient’s inspired oxygen.

10700

© 2011 PasTest Ltd | About Us | Contact Us | Help

file:///E|/Shakhawan/Respiratory%20S/312a.htm[3/13/2012 4:11:17 PM]


MyPasTest

Main Navigation
Question Browser: MRCP 1
Home
Subscriptions Question Browser Exam Builder Saved Exams
» MRCP 1 Session Progress
• Question Browser
Questions Correct 10
• Timed Test A 42-year-old man presents to his GP with symptoms of
lower respiratory tract infection. This fails to clear after 2 Questions Incorrect 2
• Mock Exam
weeks of oral antibiotics and unfortunately chest X-ray Questions Total 12
• Past Papers reveals a suspicious mass in the central region of the right
Questions Percentage 83 %
• Random Questions lung. At bronchoscopy the tumour is noted to be
particularly vascular. Histology reveals small polygonal cells More
• My Performance
with a finely granular eosinophilic cytoplasm, and the nuclei
• Media Bank are small and round. There is no evidence of tumour
• New Multimedia metastasis.
Reference: Normal Values
Online Extras Which of the following represents the most likely
diagnosis in this case? Haematology
Library
Community Immunoglobulins
Small-cell carcinoma of the bronchus
Help Carcinoid tumour of the bronchus Biochemistry
PasTest Store Squamous-cell carcinoma of the bronchus Diabetes
Alveolar carcinoma Endocrinology
My Account Adenocarcinoma of the bronchus
Blood gases
Profile
10701 CSF
Newsletters
My Career and Exams
Order History
Learning Goals
Question Filters

Security

Change Password
Sign Out

© 2011 PasTest Ltd | About Us | Contact Us | Help

file:///E|/Shakhawan/Respiratory%20S/313.htm[3/13/2012 4:11:18 PM]


MyPasTest

Main Navigation
Question Browser: MRCP 1
Home
Subscriptions Question Browser Exam Builder Saved Exams
» MRCP 1 Session Progress
• Question Browser
Questions Correct 11
• Timed Test A 42-year-old man presents to his GP with symptoms of
lower respiratory tract infection. This fails to clear after 2 Questions Incorrect 2
• Mock Exam
weeks of oral antibiotics and unfortunately chest X-ray Questions Total 13
• Past Papers reveals a suspicious mass in the central region of the right
Questions Percentage 84 %
• Random Questions lung. At bronchoscopy the tumour is noted to be
particularly vascular. Histology reveals small polygonal cells More
• My Performance
with a finely granular eosinophilic cytoplasm, and the nuclei
• Media Bank are small and round. There is no evidence of tumour
• New Multimedia metastasis.

Online Extras Which of the following represents the most likely


Library diagnosis in this case?

Community Small-cell carcinoma of the bronchus


Help Carcinoid tumour of the bronchus Your answer
PasTest Store Squamous-cell carcinoma of the
bronchus
My Account Alveolar carcinoma
Adenocarcinoma of the bronchus
Profile
Newsletters
My Career and Exams
Order History The histological picture seen here, particularly with Reference: Normal Values
respect to granular eosinophilic staining of the
Learning Goals cytoplasm is highly suggestive of a carcinoid tumour. Haematology
Question Filters Around 80–90% of tumours develop in a bronchus of
subsegmental size or greater, and hence patients often Immunoglobulins
present with bronchial obstruction leading to lower
Biochemistry
Security respiratory tract infection. Bronchial carcinoid is thought
to derive from stem cells of the bronchial epithelium Diabetes
Change Password known as Kulchitsky cells. Bronchoscopic tumour
Sign Out resection is not recommended, and total surgical Endocrinology
resection should be attempted if there is no evidence of
metastases. Trials of laser resection have been mooted Blood gases
for palliation where metastases exist. CSF

10701

© 2011 PasTest Ltd | About Us | Contact Us | Help

file:///E|/Shakhawan/Respiratory%20S/313a.htm[3/13/2012 4:11:20 PM]


MyPasTest

Main Navigation
Question Browser: MRCP 1
Home
Subscriptions Question Browser Exam Builder Saved Exams
» MRCP 1 Session Progress
• Question Browser
Questions Correct 11
• Timed Test A 36-year-old man attends with his wife for review at the
GP surgery. His wife complains that he snores excessively, Questions Incorrect 2
• Mock Exam
regularly stops breathing during the night and now has a Questions Total 13
• Past Papers habit of falling asleep in the daytime, sometimes even in
Questions Percentage 84 %
• Random Questions the middle of a conversation. He is noted to have a neck
collar size of 18 inches. More
• My Performance
• Media Bank To which of the following conditions does his underlying
problem predispose?
• New Multimedia
Reference: Normal Values
Online Extras Hypotension
Haematology
Library Osteomalacia
Community Stroke Immunoglobulins
Help Osteoporosis Biochemistry
PasTest Store Hypoglycaemia Diabetes

14959 Endocrinology
My Account
Blood gases
Profile
CSF
Newsletters
My Career and Exams
Order History
Learning Goals
Question Filters

Security

Change Password
Sign Out

© 2011 PasTest Ltd | About Us | Contact Us | Help

file:///E|/Shakhawan/Respiratory%20S/314.htm[3/13/2012 4:11:21 PM]


MyPasTest

Main Navigation
Question Browser: MRCP 1
Home
Subscriptions Question Browser Exam Builder Saved Exams
» MRCP 1 Session Progress
• Question Browser
Questions Correct 12
• Timed Test A 36-year-old man attends with his wife for review at the
GP surgery. His wife complains that he snores excessively, Questions Incorrect 2
• Mock Exam
regularly stops breathing during the night and now has a Questions Total 14
• Past Papers habit of falling asleep in the daytime, sometimes even in
Questions Percentage 85 %
• Random Questions the middle of a conversation. He is noted to have a neck
collar size of 18 inches. More
• My Performance
• Media Bank To which of the following conditions does his underlying
problem predispose?
• New Multimedia
Online Extras Hypotension
Library Osteomalacia
Community Stroke Your answer
Help Osteoporosis
PasTest Store Hypoglycaemia

My Account

Profile
This patient has symptoms of sleep apnoea, and a
Newsletters collar size of greater than 17 inches is strongly
My Career and Exams associated with this condition. Problems linked to sleep
apnoea include hypertension and hyperglycaemia. This Reference: Normal Values
Order History
may be due in part due to direct sympathetic
Learning Goals stimulation associated with periodic night-time arousal. Haematology
Question Filters Over the longer term, chronic elevated blood pressure
levels are linked to increased stroke risk. These patients Immunoglobulins
are insulin resistant and have increased long term risk
of Type 2 diabetes due to obesity, prior to the Biochemistry
Security
development of diabetes they have a relatively Diabetes
Change Password increased bone mineral density. After the development
of diabetes this diminishes. Treatment of sleep apnoea Endocrinology
Sign Out
involves aggressive attempts at weight loss, coupled
with night time CPAP. Blood gases

CSF
14959

© 2011 PasTest Ltd | About Us | Contact Us | Help

file:///E|/Shakhawan/Respiratory%20S/314a.htm[3/13/2012 4:11:23 PM]


MyPasTest

Main Navigation
Question Browser: MRCP 1
Home
Subscriptions Question Browser Exam Builder Saved Exams
» MRCP 1 Session Progress
• Question Browser
Questions Correct 12
• Timed Test You review a 71-year-old former boilermaker. He has
suffered increasing shortness of breath and vague right Questions Incorrect 2
• Mock Exam
sided chest pain over the past few months. He has a Questions Total 14
• Past Papers history of smoking 30 cigarettes per day for 30 years. On
Questions Percentage 85 %
• Random Questions examination he appears to have a large right sided pleural
effusion. This is confirmed on chest x-ray. Pleural fluid More
• My Performance
evaluation reveals this to be an exudate and you suspect an
• Media Bank underlying mesothelioma.
• New Multimedia
Which of the following fit best with the Reference: Normal Values
Online Extras pathophysiology of mesothelioma?
Haematology
Library
Cigarette smoking is a known primary risk factor Immunoglobulins
Community
Simian SV 40 is the most common cause
Help Biochemistry
Loss of one copy of chromosome 22 is the most
PasTest Store Diabetes
common karyotypic change in mesothelioma cell lines
Drainage of pleural effusion is never associated with Endocrinology
My Account tumour seeding along the track
Blood gases
Profile Surgical cure is possible in 30% of cases
CSF
Newsletters 18606
My Career and Exams
Order History
Learning Goals
Question Filters

Security

Change Password
Sign Out

© 2011 PasTest Ltd | About Us | Contact Us | Help

file:///E|/Shakhawan/Respiratory%20S/315.htm[3/13/2012 4:11:24 PM]


MyPasTest

Main Navigation
Question Browser: MRCP 1
Home
Subscriptions Question Browser Exam Builder Saved Exams
» MRCP 1 Session Progress
• Question Browser
Questions Correct 12
• Timed Test You review a 71-year-old former boilermaker. He has
suffered increasing shortness of breath and vague right Questions Incorrect 3
• Mock Exam
sided chest pain over the past few months. He has a Questions Total 15
• Past Papers history of smoking 30 cigarettes per day for 30 years. On
Questions Percentage 80 %
• Random Questions examination he appears to have a large right sided pleural
effusion. This is confirmed on chest x-ray. Pleural fluid More
• My Performance
evaluation reveals this to be an exudate and you suspect an
• Media Bank underlying mesothelioma.
• New Multimedia
Which of the following fit best with the
Online Extras pathophysiology of mesothelioma?
Library
Cigarette smoking is a known
Community
primary risk factor
Help
Simian SV 40 is the most common Your answer
PasTest Store cause
Loss of one copy of chromosome 22 Correct answer
is the most common karyotypic
My Account
change in mesothelioma cell lines
Profile Drainage of pleural effusion is never
Newsletters associated with tumour seeding along
the track
My Career and Exams
Surgical cure is possible in 30% of
Order History cases Reference: Normal Values
Learning Goals Haematology
Question Filters
Immunoglobulins
Biochemistry
Security Exposure to asbestosis is the primary risk factor
associated with the development of mesothelioma. Diabetes
Change Password SV40 is a possible co-carcinogen, although exposure to
Sign Out SV40 alone is not thought to be associated with Endocrinology
mesothelioma. Surgical cure is usually not possible,
although pleurectomy or pneumonectomy may bring Blood gases
relief of symptoms. The role of cigarette smoking has CSF
been under much debate; a categorical link to increased
risk of mesothelioma on top of asbestos exposure has
not been proven.

18606

© 2011 PasTest Ltd | About Us | Contact Us | Help

file:///E|/Shakhawan/Respiratory%20S/315a.htm[3/13/2012 4:11:26 PM]


MyPasTest

Main Navigation
Question Browser: MRCP 1
Home
Subscriptions Question Browser Exam Builder Saved Exams
» MRCP 1 Session Progress
• Question Browser
Questions Correct 12
• Timed Test You review a 35-year-old woman with progressively
increased shortness of breath and lethargy after the birth Questions Incorrect 3
• Mock Exam
of her 1st child. You send her for some pulmonary function Questions Total 15
• Past Papers tests, including measurement of gas transfer.
Questions Percentage 80 %
• Random Questions
Which of the following gases is usually used for More
• My Performance
measurement of gas transfer?
• Media Bank
• New Multimedia Carbon dioxide
Reference: Normal Values
Online Extras Oxygen
Methane Haematology
Library
Community Carbon monoxide Immunoglobulins
Help Nitrous oxide Biochemistry
PasTest Store 18607 Diabetes
Endocrinology
My Account
Blood gases
Profile
CSF
Newsletters
My Career and Exams
Order History
Learning Goals
Question Filters

Security

Change Password
Sign Out

© 2011 PasTest Ltd | About Us | Contact Us | Help

file:///E|/Shakhawan/Respiratory%20S/316.htm[3/13/2012 4:11:27 PM]


MyPasTest

Main Navigation
Question Browser: MRCP 1
Home
Subscriptions Question Browser Exam Builder Saved Exams
» MRCP 1 Session Progress
• Question Browser
Questions Correct 13
• Timed Test You review a 35-year-old woman with progressively
increased shortness of breath and lethargy after the birth Questions Incorrect 3
• Mock Exam
of her 1st child. You send her for some pulmonary function Questions Total 16
• Past Papers tests, including measurement of gas transfer.
Questions Percentage 81 %
• Random Questions
Which of the following gases is usually used for More
• My Performance
measurement of gas transfer?
• Media Bank
• New Multimedia Carbon dioxide
Online Extras Oxygen
Library Methane
Community Carbon monoxide Your answer
Help Nitrous oxide
PasTest Store

My Account
The DLCO (or diffusing capacity for carbon monoxide) is
Profile the standard method for measuring gas transfer from
alveoli to red blood cells. DLCO is measured by looking
Newsletters
at end-expiratory levels of CO after inspiring a small
My Career and Exams amount and breath holding. It is adjusted for
Order History haematocrit and alveolar volume. Conditions which Reference: Normal Values
affect the pulmonary vasculature and those which lead
Learning Goals to pulmonary fibrosis affect the DLCO. Haematology
Question Filters
Immunoglobulins
18607 Biochemistry
Security
Diabetes
Change Password
Sign Out Endocrinology

Blood gases

CSF

© 2011 PasTest Ltd | About Us | Contact Us | Help

file:///E|/Shakhawan/Respiratory%20S/316a.htm[3/13/2012 4:11:29 PM]


MyPasTest

Main Navigation
Question Browser: MRCP 1
Home
Subscriptions Question Browser Exam Builder Saved Exams
» MRCP 1 Session Progress
• Question Browser
Questions Correct 13
• Timed Test A 54-year-old man with a 40 pack year smoking history
presents to the clinic complaining of a chronic cough and Questions Incorrect 3
• Mock Exam
haemoptysis. He has lost 4kg in weight recently. He has an Questions Total 16
• Past Papers abnormal chest x-ray consistent with bronchial carcinoma.
Questions Percentage 81 %
• Random Questions
Investigations; More
• My Performance
• Media Bank Hb 11.0 g/dl
• New Multimedia
WCC 6.1 x10 9 /l Reference: Normal Values
Online Extras
Library PLT 352 x10 9 /l Haematology

Community ESR 65 mm/hr Immunoglobulins


Help Na + 132 mmol/l Biochemistry
PasTest Store
K+ 3.9 mmol/l Diabetes

Creatinine 130 μmol/l Endocrinology


My Account
Blood gases
Profile Bronchoscopy with transbronchial biopsy reveals
adenocarcinoma of the bronchus CSF
Newsletters
My Career and Exams You arrange a CT thorax.
Order History
Which of the following would tend to rule out the
Learning Goals possibility of a surgical cure?
Question Filters
FEV1 1.6
Superior vena caval obstruction
Security
Malignant pleural effusion
Change Password
Ipisilateral mediastinal lymph node involvement
Sign Out
Horner’s syndrome

18608

© 2011 PasTest Ltd | About Us | Contact Us | Help

file:///E|/Shakhawan/Respiratory%20S/317.htm[3/13/2012 4:11:30 PM]


MyPasTest

Main Navigation
Question Browser: MRCP 1
Home
Subscriptions Question Browser Exam Builder Saved Exams
» MRCP 1 Session Progress
• Question Browser
Questions Correct 14
• Timed Test A 54-year-old man with a 40 pack year smoking history
presents to the clinic complaining of a chronic cough and Questions Incorrect 3
• Mock Exam
haemoptysis. He has lost 4kg in weight recently. He has an Questions Total 17
• Past Papers abnormal chest x-ray consistent with bronchial carcinoma.
Questions Percentage 82 %
• Random Questions
Investigations; More
• My Performance
• Media Bank Hb 11.0 g/dl
• New Multimedia
WCC 6.1 x10 9 /l
Online Extras
Library PLT 352 x10 9 /l

Community ESR 65 mm/hr


Help Na + 132 mmol/l
PasTest Store
K+ 3.9 mmol/l
Creatinine 130 μmol/l
My Account

Profile Bronchoscopy with transbronchial biopsy reveals


adenocarcinoma of the bronchus
Newsletters
My Career and Exams You arrange a CT thorax.
Order History Reference: Normal Values
Which of the following would tend to rule out the
Learning Goals possibility of a surgical cure? Haematology
Question Filters
FEV1 1.6 Immunoglobulins

Superior vena caval obstruction Biochemistry


Security
Malignant pleural effusion Your answer Diabetes
Change Password
Ipisilateral mediastinal lymph node
Sign Out Endocrinology
involvement
Horner’s syndrome Blood gases

CSF

Pleural effusion implies pleural involvement; hence this


precludes a surgical cure. FEV1 of less than 1.5
precludes surgery for bronchial carcinoma as not
enough lung reserve would be present afterwards.
Contralateral lymph node involvement implies distant
spread and again precludes surgery. Horner’s syndrome
is due to local nerve involvement and therefore does
not exclude successful surgical excision of the primary
tumour. Metastasis is usual at time of presentation in
small cell carcinoma; therefore patients are usually
managed with a combination of radio- and
chemotherapy.

18608

© 2011 PasTest Ltd | About Us | Contact Us | Help

file:///E|/Shakhawan/Respiratory%20S/317a.htm[3/13/2012 4:11:32 PM]


MyPasTest

Main Navigation
Question Browser: MRCP 1
Home
Subscriptions Question Browser Exam Builder Saved Exams
» MRCP 1 Session Progress
• Question Browser
Questions Correct 14
• Timed Test A 71-year-old woman with a 30 pack year history of
smoking presents with rapidly worsening shortness of Questions Incorrect 3
• Mock Exam
breath. Her daughter has brought her to the hospital and is Questions Total 17
• Past Papers concerned as despite having antibiotics from the GP her
Questions Percentage 82 %
• Random Questions cough has worsened and she now seems very tired and
lethargic. Normal medication includes salbutamol and More
• My Performance
atrovent inhalers. On examination she looks weary and has
• Media Bank a cough productive of yellow green sputum, there are
• New Multimedia coarse crackles and wheeze on auscultation of the chest.
Reference: Normal Values
Online Extras Her blood gases on 28% O 2 :
Haematology
Library
Community pH 7.25 Immunoglobulins
Help PO2 7.4kPa (11.3-12.6) Biochemistry
PasTest Store PCO 2 Diabetes
8.9kPa (4.7-6.0)

HCO3 - 35mmol/l (20-28) Endocrinology


My Account
BE +9 Blood gases
Profile
CSF
Newsletters What is the next most appropriate management
My Career and Exams step?

Order History Non-invasive ventilation


Learning Goals Intubation and ventilation
Question Filters IV doxapram
Increase inspired oxygen
Security Decrease inspired oxygen
Change Password
18609
Sign Out

© 2011 PasTest Ltd | About Us | Contact Us | Help

file:///E|/Shakhawan/Respiratory%20S/318.htm[3/13/2012 4:11:33 PM]


MyPasTest

Main Navigation
Question Browser: MRCP 1
Home
Subscriptions Question Browser Exam Builder Saved Exams
» MRCP 1 Session Progress
• Question Browser
Questions Correct 14
• Timed Test A 71-year-old woman with a 30 pack year history of
Questions Incorrect 4
• Mock Exam smoking presents with rapidly worsening shortness of
breath. Her daughter has brought her to the hospital and is Questions Total 18
• Past Papers concerned as despite having antibiotics from the GP her Questions Percentage 77 %
• Random Questions cough has worsened and she now seems very tired and
lethargic. Normal medication includes salbutamol and More
• My Performance
atrovent inhalers. On examination she looks weary and has
• Media Bank a cough productive of yellow green sputum, there are
• New Multimedia coarse crackles and wheeze on auscultation of the chest.

Online Extras Her blood gases on 28% O 2 :


Library
Community pH 7.25
Help PO2 7.4kPa (11.3-12.6)
PasTest Store PCO 2 8.9kPa (4.7-6.0)

HCO3 - 35mmol/l (20-28)


My Account
BE +9
Profile
Newsletters What is the next most appropriate management
My Career and Exams step?

Order History Non-invasive ventilation Correct answer Reference: Normal Values


Learning Goals Intubation and ventilation Haematology
Question Filters IV doxapram Your answer Immunoglobulins
Increase inspired oxygen
Biochemistry
Security Decrease inspired oxygen
Diabetes
Change Password
Sign Out Endocrinology

Blood gases
This patient has a long history of COPD and is
deteriorating. Options include intubation and ventilation CSF
or non-invasive ventilation normally using BIPAP (bi-
level positive airway pressure). Intubation and
ventilation may not be the best option in this case as
weaning can often be difficult in CO2 retainers. IV
doxapram would only be used if the patient was not
suitable for either intubation or non-invasive ventilation.
Decreasing inspired oxygen would lead to worsening
hypoxia, increasing it may drive worsening CO2
retention.

18609

© 2011 PasTest Ltd | About Us | Contact Us | Help

file:///E|/Shakhawan/Respiratory%20S/318a.htm[3/13/2012 4:11:35 PM]


MyPasTest

Main Navigation
Question Browser: MRCP 1
Home
Subscriptions Question Browser Exam Builder Saved Exams
» MRCP 1 Session Progress
• Question Browser
Questions Correct 14
• Timed Test A 48-year-old non-smoking lady with asthma managed
with seretide 50 2 puffs BD and a BMI of 35 presents with Questions Incorrect 4
• Mock Exam
3 months progressive shortness of breath. Her lung Questions Total 18
• Past Papers function tests are as follows:
Questions Percentage 77 %
• Random Questions
FEV1 2.2 (87% predicted) Post salbutamol 2.3 More
• My Performance
FVC 3.4 (82% predicted) Post salbutamol 3.5
• Media Bank Transfer coefficient 55%
• New Multimedia
What is the most likely diagnosis? Reference: Normal Values
Online Extras
Haematology
Library Pulmonary Embolism
Community Asthma Immunoglobulins
Help Alveolar haemorrhage Biochemistry
PasTest Store Obesity Diabetes
Chronic obstructive pulmonary disease
Endocrinology
My Account
18610 Blood gases
Profile
CSF
Newsletters
My Career and Exams
Order History
Learning Goals
Question Filters

Security

Change Password
Sign Out

© 2011 PasTest Ltd | About Us | Contact Us | Help

file:///E|/Shakhawan/Respiratory%20S/319.htm[3/13/2012 4:11:36 PM]


MyPasTest

Main Navigation
Question Browser: MRCP 1
Home
Subscriptions Question Browser Exam Builder Saved Exams
» MRCP 1 Session Progress
• Question Browser
Questions Correct 14
• Timed Test A 48-year-old non-smoking lady with asthma managed
with seretide 50 2 puffs BD and a BMI of 35 presents with Questions Incorrect 5
• Mock Exam
3 months progressive shortness of breath. Her lung Questions Total 19
• Past Papers function tests are as follows:
Questions Percentage 73 %
• Random Questions
FEV1 2.2 (87% predicted) Post salbutamol 2.3 More
• My Performance
FVC 3.4 (82% predicted) Post salbutamol 3.5
• Media Bank Transfer coefficient 55%
• New Multimedia
What is the most likely diagnosis?
Online Extras
Library Pulmonary Embolism Correct answer
Community Asthma
Help Alveolar haemorrhage
PasTest Store Obesity Your answer
Chronic obstructive pulmonary
My Account disease

Profile
Newsletters
My Career and Exams This woman has morbid obesity and because of this is
Order History at risk of pulmonary emboli. Her FEV1 and FVC are Reference: Normal Values
Learning Goals relatively preserved; at least some of the reduction
seen will be due to her obesity. She is a non-smoker so Haematology
Question Filters that COPD is unlikely, and there is no history of Immunoglobulins
haemoptysis or other symptoms suggestive of a
condition associated with pulmonary haemorrhage. The Biochemistry
Security absence of chest pain and the slow increase in
shortness of breath suggests that she has suffered Diabetes
Change Password
multiple small pulmonary emboli.
Sign Out Endocrinology

Blood gases
18610
CSF

© 2011 PasTest Ltd | About Us | Contact Us | Help

file:///E|/Shakhawan/Respiratory%20S/319a.htm[3/13/2012 4:11:38 PM]


MyPasTest

Main Navigation
Question Browser: MRCP 1
Home
Subscriptions Question Browser Exam Builder Saved Exams
» MRCP 1 Session Progress
• Question Browser
Questions Correct 14
• Timed Test A 19-year-old woman has recently started work as an
apprentice in a carpentry factory. She reports cough and Questions Incorrect 5
• Mock Exam
wheeze which worsens as the working week progresses. Questions Total 19
• Past Papers Unusually though, she seemed free of symptoms when she
Questions Percentage 73 %
• Random Questions joined some friends on a week’s holiday to Spain, and
certainly she is better at weekends. Her peak flow in the GP More
• My Performance
surgery is 450 (predicted 510).
• Media Bank
• New Multimedia You suspect that she may have occupational asthma;
which of the following is the most appropriate way to Reference: Normal Values
Online Extras diagnose it?
Haematology
Library
Trial of 30mg prednisolone PO for 1 week Immunoglobulins
Community
Outpatient spirometry
Help Biochemistry
Peak flow diary
PasTest Store Diabetes
Trial of salbutamol inhaler
Patch testing Endocrinology
My Account
Blood gases
Profile 18611
CSF
Newsletters
My Career and Exams
Order History
Learning Goals
Question Filters

Security

Change Password
Sign Out

© 2011 PasTest Ltd | About Us | Contact Us | Help

file:///E|/Shakhawan/Respiratory%20S/320.htm[3/13/2012 4:11:40 PM]


MyPasTest

Main Navigation
Question Browser: MRCP 1
Home
Subscriptions Question Browser Exam Builder Saved Exams
» MRCP 1 Session Progress
• Question Browser
Questions Correct 14
• Timed Test A 19-year-old woman has recently started work as an
apprentice in a carpentry factory. She reports cough and Questions Incorrect 6
• Mock Exam
wheeze which worsens as the working week progresses. Questions Total 20
• Past Papers Unusually though, she seemed free of symptoms when she
Questions Percentage 70 %
• Random Questions joined some friends on a week’s holiday to Spain, and
certainly she is better at weekends. Her peak flow in the GP More
• My Performance
surgery is 450 (predicted 510).
• Media Bank
• New Multimedia You suspect that she may have occupational asthma;
which of the following is the most appropriate way to
Online Extras diagnose it?
Library
Trial of 30mg prednisolone PO for 1 Your answer
Community
week
Help
Outpatient spirometry
PasTest Store
Peak flow diary Correct answer
Trial of salbutamol inhaler
My Account Patch testing
Profile
Newsletters
My Career and Exams
Order History This woman should measure morning and evening peak Reference: Normal Values
flows for a few weeks, taking note of which days she is
Learning Goals at work and which are weekends. It is likely that her Haematology
Question Filters peak flows will be significantly better during the
weekend period and that she would show a degree of Immunoglobulins
airway obstruction first seen in her peak flow
Biochemistry
Security measurement on a Monday evening if she works from
Monday to Friday. Exposure to wood dust, flux, plastics, Diabetes
Change Password solvents, animal allergens and fungi may all contribute
Sign Out to occupational asthma. The best treatment is to Endocrinology
remove the patient from the allergen; hence it is crucial
to make the diagnosis. Post diagnosis of occupational Blood gases
asthma it may be appropriate to consider patch testing CSF
to decide on which potential allergen is the most likely
cause of her symptoms.

18611

© 2011 PasTest Ltd | About Us | Contact Us | Help

file:///E|/Shakhawan/Respiratory%20S/320a.htm[3/13/2012 4:11:41 PM]


MyPasTest

Main Navigation
Question Browser: MRCP 1
Home
Subscriptions Question Browser Exam Builder Saved Exams
» MRCP 1 Session Progress
• Question Browser
Questions Correct 14
• Timed Test A 40-year-old obese lady with a history of asthma presents
with progressive breathlessness. On examination she is has Questions Incorrect 6
• Mock Exam
a raised JVP, ankle oedema, right parasternal heave and a Questions Total 20
• Past Papers murmur consistent with tricuspid regurgitation. Chest
Questions Percentage 70 %
• Random Questions auscultation is clear. You understand from her husband
that she has suffered progressively worsening symptoms More
• My Performance
over the past year, he has noticed that she has had
• Media Bank particular problems with snoring and stopping breathing at
• New Multimedia night since he met her.
Reference: Normal Values
Online Extras ABG results;
Haematology
Library
Community pH 7.39 Immunoglobulins
Help PO2 7.3kPa (11.3-12.6) Biochemistry
PasTest Store PCO 2 3.9kPa (4.7-6.0) Diabetes
Endocrinology
My Account What is the most likely diagnosis?
Blood gases
Profile Primary pulmonary hypertension
CSF
Newsletters Chronic asthma
My Career and Exams Secondary pulmonary hypertension
Order History Atrial septal defect
Learning Goals Pulmonary fibrosis
Question Filters
18612

Security

Change Password
Sign Out

© 2011 PasTest Ltd | About Us | Contact Us | Help

file:///E|/Shakhawan/Respiratory%20S/321.htm[3/13/2012 4:11:43 PM]


MyPasTest

Main Navigation
Question Browser: MRCP 1
Home
Subscriptions Question Browser Exam Builder Saved Exams
» MRCP 1 Session Progress
• Question Browser
Questions Correct 1
• Timed Test A 40-year-old obese lady with a history of asthma presents
with progressive breathlessness. On examination she is has Questions Incorrect 0
• Mock Exam
a raised JVP, ankle oedema, right parasternal heave and a Questions Total 1
• Past Papers murmur consistent with tricuspid regurgitation. Chest
Questions Percentage 100 %
• Random Questions auscultation is clear. You understand from her husband
that she has suffered progressively worsening symptoms More
• My Performance
over the past year, he has noticed that she has had
• Media Bank particular problems with snoring and stopping breathing at
• New Multimedia night since he met her.

Online Extras ABG results;


Library
Community pH 7.39

Help PO2 7.3kPa (11.3-12.6)


PasTest Store PCO 2 3.9kPa (4.7-6.0)

My Account What is the most likely diagnosis?

Profile Primary pulmonary hypertension


Newsletters Chronic asthma
My Career and Exams Secondary pulmonary hypertension Your answer
Order History Atrial septal defect Reference: Normal Values
Learning Goals Pulmonary fibrosis Haematology
Question Filters
Immunoglobulins
Biochemistry
Security
The history of snoring is suggestive of sleep apnoea. Diabetes
Change Password Sleep apnoea is known to result in chronic nocturnal
Sign Out hypoxia, which in turn leads to pulmonary artery Endocrinology
vasoconstriction. Eventually cor pulmonale is the result,
which is seen here. Treatment for this woman is likely Blood gases
to involve aggressive attempts to lose weight and CSF
nocturnal ventilatory support, probably with the use of a
CPAP or BIPAP machine.

18612

© 2011 PasTest Ltd | About Us | Contact Us | Help

file:///E|/Shakhawan/Respiratory%20S/321a.htm[3/13/2012 4:11:44 PM]


MyPasTest

Main Navigation
Question Browser: MRCP 1
Home
Subscriptions Question Browser Exam Builder Saved Exams
» MRCP 1 Session Progress
• Question Browser
Questions Correct 1
• Timed Test A 51-year-old-life-long smoker, who has worked for many
years in a shipyard presents with a few months history of Questions Incorrect 0
• Mock Exam
increasing breathlessness. On examination he has a BP of Questions Total 1
• Past Papers 145/85 mmHg and a pulse of 75/minute; his BMI is 31 and
Questions Percentage 100 %
• Random Questions he also appears to have finger clubbing. Auscultation of the
chest reveals bibasal inspiratory crackles. More
• My Performance
• Media Bank CXR is reported as showing evidence of pleural plaques
• New Multimedia Pulmonary function testing reveals a mixed
obstructive/restrictive picture Reference: Normal Values
Online Extras
Which of the following is the most likely cause of his Haematology
Library
breathlessness? Immunoglobulins
Community
Help COPD Biochemistry
PasTest Store Obesity Diabetes
Asbestos related pleural plaques
Endocrinology
My Account Asbestosis
Blood gases
Profile Idiopathic pulmonary fibrosis
CSF
Newsletters 18613
My Career and Exams
Order History
Learning Goals
Question Filters

Security

Change Password
Sign Out

© 2011 PasTest Ltd | About Us | Contact Us | Help

file:///E|/Shakhawan/Respiratory%20S/322.htm[3/13/2012 4:11:46 PM]


MyPasTest

Main Navigation
Question Browser: MRCP 1
Home
Subscriptions Question Browser Exam Builder Saved Exams
» MRCP 1 Session Progress
• Question Browser
Questions Correct 2
• Timed Test A 51-year-old-life-long smoker, who has worked for many
years in a shipyard presents with a few months history of Questions Incorrect 0
• Mock Exam
increasing breathlessness. On examination he has a BP of Questions Total 2
• Past Papers 145/85 mmHg and a pulse of 75/minute; his BMI is 31 and
Questions Percentage 100 %
• Random Questions he also appears to have finger clubbing. Auscultation of the
chest reveals bibasal inspiratory crackles. More
• My Performance
• Media Bank CXR is reported as showing evidence of pleural plaques
• New Multimedia Pulmonary function testing reveals a mixed
obstructive/restrictive picture
Online Extras
Library Which of the following is the most likely cause of his
breathlessness?
Community
Help COPD
PasTest Store Obesity
Asbestos related pleural plaques
My Account Asbestosis Your answer

Profile Idiopathic pulmonary fibrosis

Newsletters
My Career and Exams
Order History Reference: Normal Values
Shipbuilding, car manufacture, boiler making and
Learning Goals plumbing industries are all associated with risk of Haematology
Question Filters asbestosis exposure. Whilst he does indeed have
asbestos related pleural plaques seen on CXR, plaques Immunoglobulins
themselves are not thought to contribute to
Biochemistry
Security breathlessness. In contrast the findings on auscultation
of the chest, coupled with the finger clubbing and the Diabetes
Change Password restrictive component seen on pulmonary function
Sign Out testing point towards asbestos related pulmonary Endocrinology
fibrosis, or asbestosis.
Blood gases

CSF
18613

© 2011 PasTest Ltd | About Us | Contact Us | Help

file:///E|/Shakhawan/Respiratory%20S/322a.htm[3/13/2012 4:11:47 PM]


MyPasTest

Main Navigation
Question Browser: MRCP 1
Home
Subscriptions Question Browser Exam Builder Saved Exams
» MRCP 1 Session Progress
• Question Browser
Questions Correct 2
• Timed Test A 54-year-old shipyard worker with a 30 pack year
smoking history presents to the GP with increasing Questions Incorrect 0
• Mock Exam
shortness of breath. He rarely consults the doctor, but now Questions Total 2
• Past Papers he is unable to walk up the road to the shops. His BP is
Questions Percentage 100 %
• Random Questions 150/80 mmHg, with a pulse of 75/minute, in atrial
fibrillation. On auscultation of the chest there is pronounced More
• My Performance
wheeze interspersed with occasional coarse crackles. His
• Media Bank FEV1 /FVC is 60%, with a KCO of 55%. CXR shows slightly
• New Multimedia increased lung markings, but nil else of note.
Reference: Normal Values
Online Extras
Which of the following is the most likely diagnosis?
Haematology
Library
Community Idiopathic pulmonary fibrosis Immunoglobulins
Help Asthma Biochemistry
PasTest Store COPD
Diabetes
Asbestosis
Endocrinology
My Account Left ventricular failure
Blood gases
Profile 18614
CSF
Newsletters
My Career and Exams
Order History
Learning Goals
Question Filters

Security

Change Password
Sign Out

© 2011 PasTest Ltd | About Us | Contact Us | Help

file:///E|/Shakhawan/Respiratory%20S/323.htm[3/13/2012 4:11:49 PM]


MyPasTest

Main Navigation
Question Browser: MRCP 1
Home
Subscriptions Question Browser Exam Builder Saved Exams
» MRCP 1 Session Progress
• Question Browser
Questions Correct 3
• Timed Test A 54-year-old shipyard worker with a 30 pack year
smoking history presents to the GP with increasing Questions Incorrect 0
• Mock Exam
shortness of breath. He rarely consults the doctor, but now Questions Total 3
• Past Papers he is unable to walk up the road to the shops. His BP is
Questions Percentage 100 %
• Random Questions 150/80 mmHg, with a pulse of 75/minute, in atrial
fibrillation. On auscultation of the chest there is pronounced More
• My Performance
wheeze interspersed with occasional coarse crackles. His
• Media Bank FEV1 /FVC is 60%, with a KCO of 55%. CXR shows slightly
• New Multimedia increased lung markings, but nil else of note.
Online Extras
Which of the following is the most likely diagnosis?
Library
Community Idiopathic pulmonary fibrosis

Help Asthma

PasTest Store COPD Your answer


Asbestosis

My Account Left ventricular failure

Profile
Newsletters
My Career and Exams In the presence of a positive smoking history and a
Order History clearly obstructive picture on spirometry, the most likely Reference: Normal Values
diagnosis is COPD. Whilst he is likely to have had
Learning Goals Haematology
exposure to asbestosis, his CXR would appear to make
Question Filters this an unlikely diagnosis. Initial treatment of COPD
Immunoglobulins
centres on the use of short acting beta-2 agonists or
anti-cholinergics. More severe disease is managed with Biochemistry
Security either long-acting anti-cholinergics or long-acting beta-
2 agonist and high dose inhaled steroid combined Diabetes
Change Password treatment.
Sign Out Endocrinology

Blood gases
18614
CSF

© 2011 PasTest Ltd | About Us | Contact Us | Help

file:///E|/Shakhawan/Respiratory%20S/323a.htm[3/13/2012 4:11:50 PM]


MyPasTest

Main Navigation
Question Browser: MRCP 1
Home
Subscriptions Question Browser Exam Builder Saved Exams
» MRCP 1 Session Progress
• Question Browser
Questions Correct 3
• Timed Test A 40-year-old farmer’s wife presents to the GP complaining
of increasing shortness of breath, worsening cough with Questions Incorrect 0
• Mock Exam
mucus plugs and wheeze. She was diagnosed with asthma Questions Total 3
• Past Papers some years ago and is managed with combination steroid
Questions Percentage 100 %
• Random Questions and long acting beta agonist (LABA) therapy. She has used
2 courses of oral corticosteroids in the past 3 months. On More
• My Performance
examination there is wheeze on auscultation of the chest.
• Media Bank
• New Multimedia Investigations;
Reference: Normal Values
Online Extras
Hb 13.0 g/dl Haematology
Library
9
WCC 7.0 x10 /l (Increase in eosinophils) Immunoglobulins
Community
Help PLT 235 x10 9 /l Biochemistry
PasTest Store IgE increased
Diabetes
Skin prick positive for aspergillus
Endocrinology
My Account Na + 139 mmol/l
Blood gases
Profile K+ 4.2 mmol/l
CSF
Newsletters Creatinine 100 μmol/l
My Career and Exams Scattered pulmonary infiltrates, no focal
Order History CXR lesion
Learning Goals
Which of the following is the most likely diagnosis?
Question Filters
Invasive pulmonary aspergillosis
Security Aspergilloma

Change Password Uncomplicated asthma

Sign Out Allergic Bronchopulmonary Aspergillosis


Cryptogenic fibrosing alveolitis

18676

© 2011 PasTest Ltd | About Us | Contact Us | Help

file:///E|/Shakhawan/Respiratory%20S/324.htm[3/13/2012 4:11:52 PM]


MyPasTest

Main Navigation
Question Browser: MRCP 1
Home
Subscriptions Question Browser Exam Builder Saved Exams
» MRCP 1 Session Progress
• Question Browser
Questions Correct 4
• Timed Test A 40-year-old farmer’s wife presents to the GP complaining
of increasing shortness of breath, worsening cough with Questions Incorrect 0
• Mock Exam
mucus plugs and wheeze. She was diagnosed with asthma Questions Total 4
• Past Papers some years ago and is managed with combination steroid
Questions Percentage 100 %
• Random Questions and long acting beta agonist (LABA) therapy. She has used
2 courses of oral corticosteroids in the past 3 months. On More
• My Performance
examination there is wheeze on auscultation of the chest.
• Media Bank
• New Multimedia Investigations;

Online Extras
Hb 13.0 g/dl
Library
WCC 7.0 x10 9 /l (Increase in eosinophils)
Community
Help PLT 235 x10 9 /l
PasTest Store IgE increased
Skin prick positive for aspergillus
My Account Na + 139 mmol/l
Profile K+ 4.2 mmol/l
Newsletters Creatinine 100 μmol/l
My Career and Exams Scattered pulmonary infiltrates, no focal
Order History CXR lesion Reference: Normal Values
Learning Goals Haematology
Which of the following is the most likely diagnosis?
Question Filters
Immunoglobulins
Invasive pulmonary aspergillosis
Biochemistry
Security Aspergilloma
Uncomplicated asthma Diabetes
Change Password
Sign Out Allergic Bronchopulmonary Your answer Endocrinology
Aspergillosis
Blood gases
Cryptogenic fibrosing alveolitis
CSF

ABPA often presents in patients with asthma that have


great difficulty controlling their disease, manifesting
with mucus plugs or even haemoptysis. It is defined by
a number of criteria including clinical deterioration in
asthma symptoms, raised IgE levels, positive aspergillus
serology and pulmonary infiltrates on CXR.
Corticosteroids are the primary therapeutic agent for
ABPA, although the addition of itraconazole may
improve the resolution of symptoms.

18676

© 2011 PasTest Ltd | About Us | Contact Us | Help

file:///E|/Shakhawan/Respiratory%20S/324a.htm[3/13/2012 4:11:53 PM]


MyPasTest

Main Navigation
Question Browser: MRCP 1
Home
Subscriptions Question Browser Exam Builder Saved Exams
» MRCP 1 Session Progress
• Question Browser
Questions Correct 4
• Timed Test A 38-year-old man presents to the GP clinic complaining of
shortness of breath. He has a history of smoking 10 Questions Incorrect 0
• Mock Exam
cigarettes per day and is obese. Other history of note Questions Total 4
• Past Papers includes hypertension, for which he is treated with atenolol
Questions Percentage 100 %
• Random Questions 50mg daily.
• My Performance More
PEFR is 540L/min (predicted is 600)
• Media Bank FEV1/FVC is 90% predicted
• New Multimedia FVC falls when measured supine versus standing up
Reference: Normal Values
Online Extras Which of the following is the most likely diagnosis?
Haematology
Library
Obesity related changes in PFTS Immunoglobulins
Community
COPD
Help Biochemistry
Asthma
PasTest Store Diabetes
Early fibrotic lung disease
Atenolol related obstructive lung picture Endocrinology
My Account
Blood gases
Profile 18677
CSF
Newsletters
My Career and Exams
Order History
Learning Goals
Question Filters

Security

Change Password
Sign Out

© 2011 PasTest Ltd | About Us | Contact Us | Help

file:///E|/Shakhawan/Respiratory%20S/325.htm[3/13/2012 4:11:55 PM]


MyPasTest

Main Navigation
Question Browser: MRCP 1
Home
Subscriptions Question Browser Exam Builder Saved Exams
» MRCP 1 Session Progress
• Question Browser
Questions Correct 5
• Timed Test A 38-year-old man presents to the GP clinic complaining of
shortness of breath. He has a history of smoking 10 Questions Incorrect 0
• Mock Exam
cigarettes per day and is obese. Other history of note Questions Total 5
• Past Papers includes hypertension, for which he is treated with atenolol
Questions Percentage 100 %
• Random Questions 50mg daily.
• My Performance More
PEFR is 540L/min (predicted is 600)
• Media Bank FEV1/FVC is 90% predicted
• New Multimedia FVC falls when measured supine versus standing up

Online Extras Which of the following is the most likely diagnosis?


Library
Obesity related changes in PFTS Your answer
Community
COPD
Help
Asthma
PasTest Store
Early fibrotic lung disease
Atenolol related obstructive lung
My Account picture
Profile
Newsletters
My Career and Exams
Order History This man has small reductions in the PEFR and the Reference: Normal Values
FEV1/FVC ratio, and the FVC ratio falls when measured
Learning Goals in the supine position. The most likely explanation is Haematology
Question Filters increased abdominal fat leading to a functional
reduction in FVC. Treatment of choice is aggressive Immunoglobulins
weight reduction, and he should also be encouraged to
Biochemistry
Security stop smoking. Finally, atenolol may not be the most
appropriate choice in a young man, if there is a Diabetes
Change Password suggestion of erectile dysfunction, a switch to an
Sign Out alternative such as an ACE inhibitor should be Endocrinology
considered.
Blood gases

CSF
18677

© 2011 PasTest Ltd | About Us | Contact Us | Help

file:///E|/Shakhawan/Respiratory%20S/325a.htm[3/13/2012 4:11:57 PM]


MyPasTest

Main Navigation
Question Browser: MRCP 1
Home
Subscriptions Question Browser Exam Builder Saved Exams
» MRCP 1 Session Progress
• Question Browser
Questions Correct 5
• Timed Test A 59-year-old woman with severe rheumatoid arthritis
presents to the respiratory clinic with worsening shortness Questions Incorrect 0
• Mock Exam
of breath. Her rheumatoid arthritis has been present for 17 Questions Total 5
• Past Papers years and she is now managed with a methotrexate based
Questions Percentage 100 %
• Random Questions regime. Other medical history of note includes
hypertension, for which she is treated with ramipril 10mg More
• My Performance
daily. On examination she has evidence of severe
• Media Bank rheumatoid joint disease. Crackles are heard on
• New Multimedia auscultation of the chest.
Reference: Normal Values
Online Extras Investigations;
Haematology
Library
Community Hb 11.0 g/dl Immunoglobulins
Help WCC 4.8 x10 9 /l Biochemistry
PasTest Store PLT 345 x10 9 /l Diabetes

Na + 139 mmol/l Endocrinology


My Account
K+ 4.5 mmol/l Blood gases
Profile
Creatinine 140 μmol/l CSF
Newsletters
My Career and Exams CXR – Patchy consolidation, small pulmonary nodules, small
Order History bilateral pleural effusions
CT Thorax – Patchy ground glass opacities
Learning Goals (peribronchovascular region), bronchial wall thickening and
Question Filters areas of bronchial dilatation, centrilobular pulmonary
nodules
PFTS – Restrictive pattern, reduced DLCO, fall in
Security oxygenation on exercise

Change Password Which of the following is the most likely diagnosis?


Sign Out
Chronic eosinophilic pneumonia (CEP)
Methotrexate related pulmonary fibrosis
Bronchiolitis obliterans organising pneumonia (BOOP)
Cryptogenic organising pneumonia (COP)
Idiopathic pulmonary fibrosis

18678

© 2011 PasTest Ltd | About Us | Contact Us | Help

file:///E|/Shakhawan/Respiratory%20S/326.htm[3/13/2012 4:11:58 PM]


MyPasTest

Main Navigation
Question Browser: MRCP 1
Home
Subscriptions Question Browser Exam Builder Saved Exams
» MRCP 1 Session Progress
• Question Browser
Questions Correct 6
• Timed Test A 59-year-old woman with severe rheumatoid arthritis
presents to the respiratory clinic with worsening shortness Questions Incorrect 0
• Mock Exam
of breath. Her rheumatoid arthritis has been present for 17 Questions Total 6
• Past Papers years and she is now managed with a methotrexate based
Questions Percentage 100 %
• Random Questions regime. Other medical history of note includes
hypertension, for which she is treated with ramipril 10mg More
• My Performance
daily. On examination she has evidence of severe
• Media Bank rheumatoid joint disease. Crackles are heard on
• New Multimedia auscultation of the chest.

Online Extras Investigations;


Library
Community Hb 11.0 g/dl

Help WCC 4.8 x10 9 /l


PasTest Store PLT 345 x10 9 /l

Na + 139 mmol/l
My Account
K+ 4.5 mmol/l
Profile
Creatinine 140 μmol/l
Newsletters
My Career and Exams CXR – Patchy consolidation, small pulmonary nodules, small
Order History bilateral pleural effusions Reference: Normal Values
CT Thorax – Patchy ground glass opacities
Learning Goals (peribronchovascular region), bronchial wall thickening and Haematology
Question Filters areas of bronchial dilatation, centrilobular pulmonary
nodules Immunoglobulins
PFTS – Restrictive pattern, reduced DLCO, fall in Biochemistry
Security oxygenation on exercise
Diabetes
Change Password Which of the following is the most likely diagnosis?
Sign Out Endocrinology
Chronic eosinophilic pneumonia (CEP)
Blood gases
Methotrexate related pulmonary
fibrosis CSF
Bronchiolitis obliterans organising Your answer
pneumonia (BOOP)
Cryptogenic organising pneumonia
(COP)
Idiopathic pulmonary fibrosis

BOOP is known to occur in patients with rheumatoid


arthritis and other connective tissue disorders, and is
associated with the typical radiographic and pulmonary
function test picture seen here. CEP on the other hand
is associated with an obstructive picture on pulmonary
function testing. Corticosteroids are the treatment of
choice for BOOP, although relapse is said to occur when
steroids are withdrawn in around 30% of cases.

18678

file:///E|/Shakhawan/Respiratory%20S/326a.htm[3/13/2012 4:12:00 PM]


MyPasTest

Main Navigation
Question Browser: MRCP 1
Home
Subscriptions Question Browser Exam Builder Saved Exams
» MRCP 1 Session Progress
• Question Browser
Questions Correct 6
• Timed Test A 42-year-old man with a history of asthma presents to
the GP with shortness of breath and a cough productive of Questions Incorrect 0
• Mock Exam
green/yellow blood stained sputum. On examination he has Questions Total 6
• Past Papers saturations of 91% and bilateral bronchial breathing at both
Questions Percentage 100 %
• Random Questions bases. He also has reactivation of a cold sore on his upper
lip. More
• My Performance
• Media Bank Investigations;
• New Multimedia
Hb 12.9 g/dl Reference: Normal Values
Online Extras
WCC 15.2 x10 9 /l Haematology
Library
Community PLT 305 x10 9 /l Immunoglobulins
Help Biochemistry
Na + 134 mmol/l
PasTest Store Diabetes
K+ 5.4 mmol/l
Creatinine 130 μmol/l Endocrinology
My Account
ESR 55 mm/hr Blood gases
Profile
CXR Bilateral lower lobe pneumonia CSF
Newsletters
My Career and Exams Which of the following is the most likely causative
Order History organism?
Learning Goals Legionella pneumophilia
Question Filters Klebsiella
Staphylococcus aureus
Security Streptococcus pneumoniae
Change Password Mycoplasma pneumoniae
Sign Out
18744

© 2011 PasTest Ltd | About Us | Contact Us | Help

file:///E|/Shakhawan/Respiratory%20S/327.htm[3/13/2012 4:12:01 PM]


MyPasTest

Main Navigation
Question Browser: MRCP 1
Home
Subscriptions Question Browser Exam Builder Saved Exams
» MRCP 1 Session Progress
• Question Browser
Questions Correct 7
• Timed Test A 42-year-old man with a history of asthma presents to
the GP with shortness of breath and a cough productive of Questions Incorrect 0
• Mock Exam
green/yellow blood stained sputum. On examination he has Questions Total 7
• Past Papers saturations of 91% and bilateral bronchial breathing at both
Questions Percentage 100 %
• Random Questions bases. He also has reactivation of a cold sore on his upper
lip. More
• My Performance
• Media Bank Investigations;
• New Multimedia
Hb 12.9 g/dl
Online Extras
Library WCC 15.2 x10 9 /l
Community PLT 305 x10 9 /l
Help
Na + 134 mmol/l
PasTest Store
K+ 5.4 mmol/l
Creatinine 130 μmol/l
My Account
ESR 55 mm/hr
Profile
CXR Bilateral lower lobe pneumonia
Newsletters
My Career and Exams Which of the following is the most likely causative
Order History organism? Reference: Normal Values
Learning Goals Legionella pneumophilia Haematology
Question Filters Klebsiella Immunoglobulins
Staphylococcus aureus
Biochemistry
Security Streptococcus pneumoniae Your answer
Diabetes
Change Password Mycoplasma pneumoniae
Sign Out Endocrinology

Blood gases

CSF
Streptococcal pneumonia Is most commonly associated
with re-activation of herpes simplex virus. The clinical
picture seen here with evidence of bilateral
consolidation also fits with this diagnosis. Most
guidelines for the management of streptococcal
pneumonia recommend combination therapy with a
penicillin and macrolide. This patient should be referred
for hospital assessment, the saturation of 91%
suggesting the possibility that he may rapidly
deteriorate.

18744

© 2011 PasTest Ltd | About Us | Contact Us | Help

file:///E|/Shakhawan/Respiratory%20S/327a.htm[3/13/2012 4:12:03 PM]


MyPasTest

Main Navigation
Question Browser: MRCP 1
Home
Subscriptions Question Browser Exam Builder Saved Exams
» MRCP 1 Session Progress
• Question Browser
Questions Correct 7
• Timed Test A 38-year-old woman presents to the Emergency room
Questions Incorrect 0
• Mock Exam with severe asthma. She is visiting the area on holiday, but
her partner informs you that she has suffered 3 admissions Questions Total 7
• Past Papers to the intensive care unit at home and has brittle asthma Questions Percentage 100 %
• Random Questions poorly controlled with maximum dose seretide, theophylline
and intermittent oral corticosteroids. On examination she More
• My Performance
looks tired and has a respiratory rate of 21/min. There is
• Media Bank marked wheeze and poor air entry on auscultation of the
• New Multimedia chest.
Reference: Normal Values
Online Extras Investigations;
Haematology
Library
Hb 13.1 g/dl Immunoglobulins
Community
WCC 6.1 x10 9 /l
Help Biochemistry
PLT 245 x10 9 /l
PasTest Store Na + 140 mmol/l Diabetes
K + 3.9 Endocrinology
My Account Creatinine 100 µmol/l
ECG – Sinus tachycardia 104 BPM Blood gases
Profile Sats 94% on O 2 on admission
PEFR 160 l/min CSF
Newsletters
My Career and Exams You arrange back to back nebulisers and ask the SHO to
Order History repeat some arterial blood gasses afterwards.
Learning Goals
Which of the following would be most worrying when
Question Filters you review the results?

Normal pH
Security
Raised lactate
Change Password Normal bicarbonate
Sign Out Normal CO2
Slightly low O 2

20460

© 2011 PasTest Ltd | About Us | Contact Us | Help

file:///E|/Shakhawan/Respiratory%20S/328.htm[3/13/2012 4:12:05 PM]


MyPasTest

Main Navigation
Question Browser: MRCP 1
Home
Subscriptions Question Browser Exam Builder Saved Exams
» MRCP 1 Session Progress
• Question Browser
Questions Correct 7
• Timed Test A 38-year-old woman presents to the Emergency room
with severe asthma. She is visiting the area on holiday, but Questions Incorrect 1
• Mock Exam
her partner informs you that she has suffered 3 admissions Questions Total 8
• Past Papers to the intensive care unit at home and has brittle asthma
Questions Percentage 87 %
• Random Questions poorly controlled with maximum dose seretide, theophylline
and intermittent oral corticosteroids. On examination she More
• My Performance
looks tired and has a respiratory rate of 21/min. There is
• Media Bank marked wheeze and poor air entry on auscultation of the
• New Multimedia chest.

Online Extras Investigations;


Library
Hb 13.1 g/dl
Community
WCC 6.1 x10 9 /l
Help
PLT 245 x10 9 /l
PasTest Store Na + 140 mmol/l
K + 3.9
My Account Creatinine 100 µmol/l
ECG – Sinus tachycardia 104 BPM
Profile Sats 94% on O 2 on admission
Newsletters PEFR 160 l/min
My Career and Exams You arrange back to back nebulisers and ask the SHO to
Order History repeat some arterial blood gasses afterwards. Reference: Normal Values
Learning Goals Haematology
Which of the following would be most worrying when
Question Filters you review the results?
Immunoglobulins
Normal pH Biochemistry
Security
Raised lactate Your answer
Diabetes
Change Password Normal bicarbonate
Sign Out Endocrinology
Normal CO2 Correct answer
Blood gases
Slightly low O 2
CSF

A normal CO2 in the face of inadequate respiratory


effort may indicate increasing fatigue. As such a normal
CO2 in the presence of a slightly reduced O 2 may be an
indication for mechanical ventilation. Patients may
deteriorate rapidly, so that frequent reassessment and
early involvement of an intensivist in the management
of acute severe asthma is strongly recommended.

20460

© 2011 PasTest Ltd | About Us | Contact Us | Help

file:///E|/Shakhawan/Respiratory%20S/328a.htm[3/13/2012 4:12:06 PM]


MyPasTest

Main Navigation
Question Browser: MRCP 1
Home
Subscriptions Question Browser Exam Builder Saved Exams
» MRCP 1 Session Progress
• Question Browser
Questions Correct 7
• Timed Test A 71-year-old man who has a 40 pack year smoking
history presents to the GP with shortness of breath and Questions Incorrect 1
• Mock Exam
bilateral ankle swelling. On examination he has a BP of Questions Total 8
• Past Papers 145/90 mmHg, a plethoric face and bilateral coarse wheeze
Questions Percentage 87 %
• Random Questions on auscultation of the chest. There is pitting oedema
affecting both ankles. More
• My Performance
• Media Bank Which of the following has proven mortality benefit in
• New Multimedia this condition?
Reference: Normal Values
Online Extras Digoxin
Haematology
Library Long term oxygen therapy (LTOT)
Community Immunoglobulins
Bisoprolol
Help Ramipril Biochemistry
PasTest Store Ipratropium Diabetes

20775 Endocrinology
My Account
Blood gases
Profile
CSF
Newsletters
My Career and Exams
Order History
Learning Goals
Question Filters

Security

Change Password
Sign Out

© 2011 PasTest Ltd | About Us | Contact Us | Help

file:///E|/Shakhawan/Respiratory%20S/329.htm[3/13/2012 4:12:08 PM]


MyPasTest

Main Navigation
Question Browser: MRCP 1
Home
Subscriptions Question Browser Exam Builder Saved Exams
» MRCP 1 Session Progress
• Question Browser
Questions Correct 8
• Timed Test A 71-year-old man who has a 40 pack year smoking
history presents to the GP with shortness of breath and Questions Incorrect 1
• Mock Exam
bilateral ankle swelling. On examination he has a BP of Questions Total 9
• Past Papers 145/90 mmHg, a plethoric face and bilateral coarse wheeze
Questions Percentage 88 %
• Random Questions on auscultation of the chest. There is pitting oedema
affecting both ankles. More
• My Performance
• Media Bank Which of the following has proven mortality benefit in
• New Multimedia this condition?

Online Extras Digoxin


Library Long term oxygen therapy (LTOT) Your answer
Community Bisoprolol
Help Ramipril
PasTest Store Ipratropium

My Account

Profile
Digoxin, whilst offering relief of symptoms in severe
Newsletters
heart failure, does not offer mortality benefit. Bisoprolol
My Career and Exams and ramipril offer benefit in left ventricular failure. This
Order History patient has COPD with cor pulmonale, right ventricular Reference: Normal Values
failure. Both the NHLB and the MRC trials did show a
Learning Goals mortality benefit of LTOT in COPD. Whilst LTOT does Haematology
Question Filters double the chances of survival however, it must be used
for around 16-19hrs per day, and poor patient Immunoglobulins
compliance often means the demands of treatment can’t
Biochemistry
Security be met. Whilst useful for relief of COPD symptoms,
ipratropium has no impact on outcomes. Diabetes
Change Password
Sign Out Endocrinology
20775
Blood gases

CSF

© 2011 PasTest Ltd | About Us | Contact Us | Help

file:///E|/Shakhawan/Respiratory%20S/329a.htm[3/13/2012 4:12:09 PM]


MyPasTest

Main Navigation
Question Browser: MRCP 1
Home
Subscriptions Question Browser Exam Builder Saved Exams
» MRCP 1 Session Progress
• Question Browser
Questions Correct 8
• Timed Test A 63-year-old man is admitted with a severe cough
Questions Incorrect 1
• Mock Exam productive of rusty coloured sputum that he has had for the
past 4 days. On admission he is pyrexial 38.5 o C, has a BP Questions Total 9
• Past Papers
of 105/65 mmHg, and a pulse of 105/min. His respiratory Questions Percentage 88 %
• Random Questions rate is 29/min. There is extensive bronchial breathing over
• My Performance the lower right lung field. More
• Media Bank Which of the following in his history, examinations or
• New Multimedia investigations would be the worst prognostic factor
for pneumonia? Reference: Normal Values
Online Extras
Haematology
Library Respiratory rate 29/min
Community Age 63 Immunoglobulins
Help BP 105/65 mmHg Biochemistry
PasTest Store Urea 8.2 mmol/l Diabetes
o
Pyrexia 38.5 C Endocrinology
My Account
20776 Blood gases
Profile
CSF
Newsletters
My Career and Exams
Order History
Learning Goals
Question Filters

Security

Change Password
Sign Out

© 2011 PasTest Ltd | About Us | Contact Us | Help

file:///E|/Shakhawan/Respiratory%20S/330.htm[3/13/2012 4:12:11 PM]


MyPasTest

Main Navigation
Question Browser: MRCP 1
Home
Subscriptions Question Browser Exam Builder Saved Exams
» MRCP 1 Session Progress
• Question Browser
Questions Correct 9
• Timed Test A 63-year-old man is admitted with a severe cough
productive of rusty coloured sputum that he has had for the Questions Incorrect 1
• Mock Exam
past 4 days. On admission he is pyrexial 38.5 o C, has a BP Questions Total 10
• Past Papers
of 105/65 mmHg, and a pulse of 105/min. His respiratory Questions Percentage 90 %
• Random Questions rate is 29/min. There is extensive bronchial breathing over
• My Performance the lower right lung field. More
• Media Bank Which of the following in his history, examinations or
• New Multimedia investigations would be the worst prognostic factor
for pneumonia?
Online Extras
Library Respiratory rate 29/min
Community Age 63
Help BP 105/65 mmHg
PasTest Store Urea 8.2 mmol/l Your answer
Pyrexia 38.5 o C
My Account

Profile
Newsletters
My Career and Exams The CURB-65 criteria were developed by the British
Thoracic Society to aid in prognostic evaluation of
Order History pneumonia. The criteria considered include new Reference: Normal Values
Learning Goals confusion, age = 65, urea >7 mmol/l, BP <90 systolic
Haematology
or <60diastolic, and respiratoryrate = 30/min. Presence
Question Filters of 3 or more of these factors is said to classify a severe Immunoglobulins
pneumonia. Patients with 1 or 0 criteria are at
significantly less risk, and this has been used to drive Biochemistry
Security
potential patient identification for out of hospital
management of community pneumonia. Diabetes
Change Password
Sign Out Endocrinology

20776 Blood gases

CSF

© 2011 PasTest Ltd | About Us | Contact Us | Help

file:///E|/Shakhawan/Respiratory%20S/330a.htm[3/13/2012 4:12:12 PM]


MyPasTest

Main Navigation
Question Browser: MRCP 1
Home
Subscriptions Question Browser Exam Builder Saved Exams
» MRCP 1 Session Progress
• Question Browser
Questions Correct 9
• Timed Test A 70-year-old woman with a history of rheumatoid arthritis
comes to the clinic for review. Most recently she has been Questions Incorrect 1
• Mock Exam
suffering from increased shortness of breath. She takes Questions Total 10
• Past Papers diclofenac and methotrexate for her arthritis. Other history
Questions Percentage 90 %
• Random Questions of note includes smoking of 10 cigarettes per day. On
examination her BP is 145/82 mmHg, she is mildly clubbed. More
• My Performance
On auscultation there are inspiratory crackles throughout
• Media Bank both lung fields.
• New Multimedia
Investigations Reference: Normal Values
Online Extras
Haematology
Library Hb 12.2 g/dl
Community Immunoglobulins
WCC 5.6 x10 9 /l
Help Biochemistry
PLT 200 x10 9 /l
PasTest Store Diabetes
Na + 139 mmol/l
Endocrinology
My Account K+ 4.9 mmol/l
Blood gases
Profile Creatinine 139 μmol/l
CSF
Newsletters Anti-GBM antibody negative
My Career and Exams FEV1 84%
Order History FVC 81%
Learning Goals KCO reduced
Question Filters pO2 7.8 kPa
pCO 2 3.5 kPa
Security

Change Password What is the most likely diagnosis?


Sign Out
Asthma
COPD
Methotrexate pneumonitis
PE
Haemorrhage

20777

© 2011 PasTest Ltd | About Us | Contact Us | Help

file:///E|/Shakhawan/Respiratory%20S/331.htm[3/13/2012 4:12:14 PM]


MyPasTest

Main Navigation
Question Browser: MRCP 1
Home
Subscriptions Question Browser Exam Builder Saved Exams
» MRCP 1 Session Progress
• Question Browser
Questions Correct 10
• Timed Test A 70-year-old woman with a history of rheumatoid arthritis
comes to the clinic for review. Most recently she has been Questions Incorrect 1
• Mock Exam
suffering from increased shortness of breath. She takes Questions Total 11
• Past Papers diclofenac and methotrexate for her arthritis. Other history
Questions Percentage 90 %
• Random Questions of note includes smoking of 10 cigarettes per day. On
examination her BP is 145/82 mmHg, she is mildly clubbed. More
• My Performance
On auscultation there are inspiratory crackles throughout
• Media Bank both lung fields.
• New Multimedia
Investigations
Online Extras
Library Hb 12.2 g/dl
Community
WCC 5.6 x10 9 /l
Help
PLT 200 x10 9 /l
PasTest Store
Na + 139 mmol/l

My Account K+ 4.9 mmol/l

Profile Creatinine 139 μmol/l


Newsletters Anti-GBM antibody negative
My Career and Exams FEV1 84%
Order History FVC 81% Reference: Normal Values
Learning Goals KCO reduced Haematology
Question Filters pO2 7.8 kPa Immunoglobulins
pCO 2 3.5 kPa Biochemistry
Security
Diabetes
Change Password What is the most likely diagnosis?
Sign Out Endocrinology
Asthma
Blood gases
COPD
Methotrexate pneumonitis Your answer CSF

PE
Haemorrhage

The lung function picture is consistent with pulmonary


fibrosis, and the reduced KCO and anti-GBM negativity
make pulmonary haemorrhage less likely. Whilst she
smokes, FEV1 is not disproportionately reduced;
therefore fibrosis rather than obstruction is the more
likely cause. Other drugs which may result in pulmonary
fibrosis include bleomycin, busulphan, amiodarone,
gold, penicillamine, crack cocaine, and heroin.
Management includes cessation of methotrexate, and
cessation of smoking, supplemental oxygen therapy and
corticosteroids may also be of value.

20777

file:///E|/Shakhawan/Respiratory%20S/331a.htm[3/13/2012 4:12:15 PM]


MyPasTest

Main Navigation
Question Browser: MRCP 1
Home
Subscriptions Question Browser Exam Builder Saved Exams
» MRCP 1 Session Progress
• Question Browser
Questions Correct 10
• Timed Test A female who is a known alcoholic was rescued from a
burning house. She has no burns and appears clinically Questions Incorrect 1
• Mock Exam
well, although she has suffered some smoke inhalation. On Questions Total 11
• Past Papers
examination her temperature is 34 o C, she is Questions Percentage 90 %
• Random Questions haemodynamically stable with a BP of 122/72 mmHg. Her
• My Performance sats are measured at 96% on pulse oximetry. More
• Media Bank Investigations - ABG results;
• New Multimedia
Reference: Normal Values
Online Extras PaO2 12 kPa
Haematology
Library PaCO 2 2.6 kPa
Community Immunoglobulins
pH normal
Help Biochemistry
Bicarbonate normal
PasTest Store Diabetes
SpO 2 92%
Endocrinology
My Account What is the cause for the apparent hypoxia seen
onSpO2 in the blood gas sample? Blood gases
Profile
CSF
Newsletters Poor peripheral circulation
My Career and Exams Respiratory alkalosis
Order History Pulmonary embolism
Learning Goals Carbon monoxide poisoning
Question Filters Pulmonary haemorrhage

20778
Security

Change Password
Sign Out

© 2011 PasTest Ltd | About Us | Contact Us | Help

file:///E|/Shakhawan/Respiratory%20S/332.htm[3/13/2012 4:12:17 PM]


MyPasTest

Main Navigation
Question Browser: MRCP 1
Home
Subscriptions Question Browser Exam Builder Saved Exams
» MRCP 1 Session Progress
• Question Browser
Questions Correct 11
• Timed Test A female who is a known alcoholic was rescued from a
burning house. She has no burns and appears clinically Questions Incorrect 1
• Mock Exam
well, although she has suffered some smoke inhalation. On Questions Total 12
• Past Papers
examination her temperature is 34 o C, she is Questions Percentage 91 %
• Random Questions haemodynamically stable with a BP of 122/72 mmHg. Her
• My Performance sats are measured at 96% on pulse oximetry. More
• Media Bank Investigations - ABG results;
• New Multimedia
Online Extras PaO2 12 kPa
Library PaCO 2 2.6 kPa
Community pH normal
Help
Bicarbonate normal
PasTest Store
SpO 2 92%

My Account What is the cause for the apparent hypoxia seen


onSpO2 in the blood gas sample?
Profile
Newsletters Poor peripheral circulation
My Career and Exams Respiratory alkalosis
Order History Pulmonary embolism Reference: Normal Values
Learning Goals Carbon monoxide poisoning Your answer Haematology
Question Filters Pulmonary haemorrhage Immunoglobulins
Biochemistry
Security
Diabetes
Change Password
Data suggests that around 1/8th of episodes of carbon Endocrinology
Sign Out monoxide poisoning are related to domestic fires. It is
likely if this woman had consumed alcohol on the night Blood gases
of the fire then she may have been exposed to smoke
inhalation for a significant length of time before she CSF
realised. PaO2 levels remain normal, (or elevated on
ABG measurement when patients are receiving inhaled
oxygen as here). Oxygen saturation on blood gas
measurement may be reduced however due to
preferential binding of Hb to carbon monoxide. With
respect to pulse oximetry, most machines cannot
differentiate between CO and O 2 binding, for this reason
don't be lulled into a false sense of security by a normal
pulse oximetry result.

20778

© 2011 PasTest Ltd | About Us | Contact Us | Help

file:///E|/Shakhawan/Respiratory%20S/332a.htm[3/13/2012 4:12:19 PM]


MyPasTest

Main Navigation
Question Browser: MRCP 1
Home
Subscriptions Question Browser Exam Builder Saved Exams
» MRCP 1 Session Progress
• Question Browser
Questions Correct 11
• Timed Test A 38-year-old nurse who has had a positive tuberculin skin
test comes to you for advice. She had been in contact with Questions Incorrect 1
• Mock Exam
a patient who had pulmonary tuberculosis some 6 days Questions Total 12
• Past Papers earlier, and has not received a BCG vaccination in the past.
Questions Percentage 91 %
• Random Questions She is well and her CXR is normal. She has started a course
of isoniazid and rifampicin. More
• My Performance
• Media Bank Which of the following is the most appropriate
• New Multimedia occupational health advice?
Reference: Normal Values
Online Extras Continue to work as normal
Haematology
Library Stay off work for 2 weeks while she is on the initial
Community prophylactic isoniazid course Immunoglobulins
Help Stay off work and have a repeat CXR in 6 weeks Biochemistry
PasTest Store Stay off work for 6 weeks
Diabetes
Continue isoniazid and rifampicin for at least 3
months Endocrinology
My Account
Blood gases
Profile 20779
CSF
Newsletters
My Career and Exams
Order History
Learning Goals
Question Filters

Security

Change Password
Sign Out

© 2011 PasTest Ltd | About Us | Contact Us | Help

file:///E|/Shakhawan/Respiratory%20S/333.htm[3/13/2012 4:12:20 PM]


MyPasTest

Main Navigation
Question Browser: MRCP 1
Home
Subscriptions Question Browser Exam Builder Saved Exams
» MRCP 1 Session Progress
• Question Browser
Questions Correct 11
• Timed Test A 38-year-old nurse who has had a positive tuberculin skin
test comes to you for advice. She had been in contact with Questions Incorrect 2
• Mock Exam
a patient who had pulmonary tuberculosis some 6 days Questions Total 13
• Past Papers earlier, and has not received a BCG vaccination in the past.
Questions Percentage 84 %
• Random Questions She is well and her CXR is normal. She has started a course
of isoniazid and rifampicin. More
• My Performance
• Media Bank Which of the following is the most appropriate
• New Multimedia occupational health advice?

Online Extras Continue to work as normal Your answer


Library Stay off work for 2 weeks while she
Community is on the initial prophylactic isoniazid
course
Help
Stay off work and have a repeat
PasTest Store
CXR in 6 weeks
Stay off work for 6 weeks
My Account Continue isoniazid and rifampicin for Correct answer
Profile at least 3 months

Newsletters
My Career and Exams
Order History Reference: Normal Values
NICE guideline 33 suggests that health care workers
Learning Goals with a positive tuberculin test after exposure to Haematology
Question Filters tuberculosis should be treated with isoniazid and
rifampicin combined for 3 months, or isoniazid alone for Immunoglobulins
a period of 6 months. The positive tuberculin test raises
Biochemistry
Security the possibility of latent TB infection in the healthcare
worker, which may be associated with not Diabetes
Change Password inconsiderable risk of infection for patients.
Sign Out Endocrinology

Blood gases
20779
CSF

© 2011 PasTest Ltd | About Us | Contact Us | Help

file:///E|/Shakhawan/Respiratory%20S/333a.htm[3/13/2012 4:12:22 PM]


MyPasTest

Main Navigation
Question Browser: MRCP 1
Home
Subscriptions Question Browser Exam Builder Saved Exams
» MRCP 1 Session Progress
• Question Browser
Questions Correct 11
• Timed Test A 38-year-old patient with a history of asthma presents
with weakness of right hand and plantar flexion of his left Questions Incorrect 2
• Mock Exam
foot. His asthma is managed with salmeterol fluticasone Questions Total 13
• Past Papers combination inhaler. On examination his BP is 152/91
Questions Percentage 84 %
• Random Questions mmHg. He has polyphonic wheeze on auscultation of the
chest. More
• My Performance
• Media Bank Investigations
• New Multimedia
Hb 13.2 g/dl Reference: Normal Values
Online Extras
WCC 8.2 x10 9 /l (raised eosinophils) Haematology
Library
Community PLT 180 x10 9 /l Immunoglobulins
Help Biochemistry
Na + 139 mmol/l
PasTest Store Diabetes
K+ 4.3 mmol/l
Creatinine 149 μmol/l Endocrinology
My Account
Urine dipstick blood +, protein + Blood gases
Profile
CSF
Newsletters Which of the following is the most appropriate
My Career and Exams autoantibody to test for?

Order History p-ANCA


Learning Goals ANA
Question Filters c-ANCA
Anti-ds DNA
Security Anti-SM antibody
Change Password
20780
Sign Out

© 2011 PasTest Ltd | About Us | Contact Us | Help

file:///E|/Shakhawan/Respiratory%20S/334.htm[3/13/2012 4:12:23 PM]


MyPasTest

Main Navigation
Question Browser: MRCP 1
Home
Subscriptions Question Browser Exam Builder Saved Exams
» MRCP 1 Session Progress
• Question Browser
Questions Correct 11
• Timed Test A 38-year-old patient with a history of asthma presents
with weakness of right hand and plantar flexion of his left Questions Incorrect 3
• Mock Exam
foot. His asthma is managed with salmeterol fluticasone Questions Total 14
• Past Papers combination inhaler. On examination his BP is 152/91
Questions Percentage 78 %
• Random Questions mmHg. He has polyphonic wheeze on auscultation of the
chest. More
• My Performance
• Media Bank Investigations
• New Multimedia
Hb 13.2 g/dl
Online Extras
Library WCC 8.2 x10 9 /l (raised eosinophils)
Community PLT 180 x10 9 /l
Help
Na + 139 mmol/l
PasTest Store
K+ 4.3 mmol/l
Creatinine 149 μmol/l
My Account
Urine dipstick blood +, protein +
Profile
Newsletters Which of the following is the most appropriate
My Career and Exams autoantibody to test for?

Order History p-ANCA Correct answer Reference: Normal Values


Learning Goals ANA Haematology
Question Filters c-ANCA Your answer Immunoglobulins
Anti-ds DNA
Biochemistry
Security Anti-SM antibody
Diabetes
Change Password
Sign Out Endocrinology

Blood gases
70% of patients with Churg Strauss have a positive p-
ANCA result. Churg Strauss presents with symptoms of CSF
asthma, and more than 7/10 patients suffer from
mononeuritis multiplex in addition. Renal involvement
leads to blood and proteinuria, hypertension and raised
creatinine. The raised eosinophil count here is also
typical of Churg Strauss. Treatment consists of
corticosteroids +/- additional therapy with
cyclophosphamide. Without treatment, the 5 year
survival rate for Churg Strauss is around 25%, with
appropriate therapy this rises to above 60%.

20780

© 2011 PasTest Ltd | About Us | Contact Us | Help

file:///E|/Shakhawan/Respiratory%20S/334a.htm[3/13/2012 4:12:25 PM]


MyPasTest

Main Navigation
Question Browser: MRCP 1
Home
Subscriptions Question Browser Exam Builder Saved Exams
» MRCP 1 Session Progress
• Question Browser
Questions Correct 11
• Timed Test A 74-year-old man presents to the clinic complaining of
increased shortness of breath. He has smoked 40 cigarettes Questions Incorrect 3
• Mock Exam
per day for the past 30 years. Medications include Questions Total 14
• Past Papers diclofenac and paracetamol for joint pain.
Questions Percentage 78 %
• Random Questions
He comes to the clinic with a CXR from his GP which shows More
• My Performance
a spiculated left hilar mass.
• Media Bank
• New Multimedia What would be the expected respiratory findings
over the abnormal area? Reference: Normal Values
Online Extras
Decreased breath sounds Haematology
Library
Community Hyper-resonance Immunoglobulins
Help Whispering pectoriloquy Biochemistry
PasTest Store Polyphonic wheeze
Diabetes
Inspiratory crackles
Endocrinology
My Account
20781 Blood gases
Profile
CSF
Newsletters
My Career and Exams
Order History
Learning Goals
Question Filters

Security

Change Password
Sign Out

© 2011 PasTest Ltd | About Us | Contact Us | Help

file:///E|/Shakhawan/Respiratory%20S/335.htm[3/13/2012 4:12:27 PM]


MyPasTest

Main Navigation
Question Browser: MRCP 1
Home
Subscriptions Question Browser Exam Builder Saved Exams
» MRCP 1 Session Progress
• Question Browser
Questions Correct 11
• Timed Test A 74-year-old man presents to the clinic complaining of
increased shortness of breath. He has smoked 40 cigarettes Questions Incorrect 4
• Mock Exam
per day for the past 30 years. Medications include Questions Total 15
• Past Papers diclofenac and paracetamol for joint pain.
Questions Percentage 73 %
• Random Questions
He comes to the clinic with a CXR from his GP which shows More
• My Performance
a spiculated left hilar mass.
• Media Bank
• New Multimedia What would be the expected respiratory findings
over the abnormal area?
Online Extras
Library Decreased breath sounds Your answer
Community Hyper-resonance
Help Whispering pectoriloquy Correct answer
PasTest Store Polyphonic wheeze
Inspiratory crackles
My Account

Profile
Newsletters
Whispering pectoriloquy is a sign of consolidation. It is
My Career and Exams likely that due both to the tumour and locally trapped
Order History secretions that you would hear whispering pectoriloquy Reference: Normal Values
over the region. Polyphonic wheeze could indicate
Learning Goals obstructive lung disease, and inspiratory crackles would Haematology
Question Filters be associated with pulmonary fibrosis or heart failure. It
is very likely this man has bronchial carcinoma, and a Immunoglobulins
endo-bronchial biopsy may be the easiest way to obtain
Biochemistry
Security a histological diagnosis.
Diabetes
Change Password
Sign Out 20781 Endocrinology

Blood gases

CSF

© 2011 PasTest Ltd | About Us | Contact Us | Help

file:///E|/Shakhawan/Respiratory%20S/335a.htm[3/13/2012 4:12:28 PM]


MyPasTest

Main Navigation
Question Browser: MRCP 1
Home
Subscriptions Question Browser Exam Builder Saved Exams
» MRCP 1 Session Progress
• Question Browser
Questions Correct 11
• Timed Test A 62-year-old man who had previously undergone
treatment for tuberculosis some 8 years ago presents via Questions Incorrect 4
• Mock Exam
his GP complaining of haemoptysis. He also says that over Questions Total 15
• Past Papers the past 3 months he has suffered night sweats on a few
Questions Percentage 73 %
• Random Questions occasions each week and a chronic cough. He smokes 10
cigarettes oer day. On examination he is mildly pyrexial More
• My Performance
37.4°C and his BP is 142/89 mmHg. Auscultation of the
• Media Bank chest reveals evidence of consolidation affecting the right
• New Multimedia upper lobe.
Reference: Normal Values
Online Extras Investigations;
Haematology
Library
Community Hb 11.9 g/dl Immunoglobulins
Help WCC 11.1 x10 9 /l Biochemistry
PasTest Store PLT 190 x10 9 /l Diabetes

Na + 138 mmol/l Endocrinology


My Account
K+ 4.8 mmol/l Blood gases
Profile
Creatinine 105 μmol/l CSF
Newsletters
My Career and Exams CXR Right upper lobe cavitating lesion

Order History Aspergillus precipitins +ve

Learning Goals
Which of the following is the most likely diagnosis?
Question Filters
Allergic bronchopulmonary aspergillosis

Security Aspergilloma
Invasive aspergillosis
Change Password
Reactivated tuberculosis
Sign Out
Lung cancer

20928

© 2011 PasTest Ltd | About Us | Contact Us | Help

file:///E|/Shakhawan/Respiratory%20S/336.htm[3/13/2012 4:12:30 PM]


MyPasTest

Main Navigation
Question Browser: MRCP 1
Home
Subscriptions Question Browser Exam Builder Saved Exams
» MRCP 1 Session Progress
• Question Browser
Questions Correct 12
• Timed Test A 62-year-old man who had previously undergone
treatment for tuberculosis some 8 years ago presents via Questions Incorrect 4
• Mock Exam
his GP complaining of haemoptysis. He also says that over Questions Total 16
• Past Papers the past 3 months he has suffered night sweats on a few
Questions Percentage 75 %
• Random Questions occasions each week and a chronic cough. He smokes 10
cigarettes oer day. On examination he is mildly pyrexial More
• My Performance
37.4°C and his BP is 142/89 mmHg. Auscultation of the
• Media Bank chest reveals evidence of consolidation affecting the right
• New Multimedia upper lobe.

Online Extras Investigations;


Library
Community Hb 11.9 g/dl

Help WCC 11.1 x10 9 /l


PasTest Store PLT 190 x10 9 /l

Na + 138 mmol/l
My Account
K+ 4.8 mmol/l
Profile
Creatinine 105 μmol/l
Newsletters
My Career and Exams CXR Right upper lobe cavitating lesion

Order History Aspergillus precipitins +ve Reference: Normal Values


Learning Goals Haematology
Which of the following is the most likely diagnosis?
Question Filters
Immunoglobulins
Allergic bronchopulmonary aspergillosis
Aspergilloma Your answer Biochemistry
Security
Invasive aspergillosis Diabetes
Change Password
Reactivated tuberculosis Endocrinology
Sign Out
Lung cancer
Blood gases

CSF

Aspergilloma is known to present in patients who have


suffered previous cavitating lung disease, such as
tuberculosis. Invasive aspergillosis however tends to
occur in patients who are immunocompromised either
due to immunosupressive therapy , haematological
malignancy or HIV. Surgical resection is successful, as
long as the patient’s pre-operative lung function is
adequate to tolerate the procedure. Other options
include chronic long term therapy with anti-fungals such
as intraconazole, or locally delivered amphotericin B.
Where bleeding becomes severe, selective bronchial
artery embolisation may be considered.

20928

© 2011 PasTest Ltd | About Us | Contact Us | Help

file:///E|/Shakhawan/Respiratory%20S/336a.htm[3/13/2012 4:12:32 PM]


MyPasTest

Main Navigation
Question Browser: MRCP 1
Home
Subscriptions Question Browser Exam Builder Saved Exams
» MRCP 1 Session Progress
• Question Browser
Questions Correct 12
• Timed Test A 50-year-old retired boiler man with shortness of breath
comes to the clinic. The GP wonders if he has obstructive Questions Incorrect 4
• Mock Exam
lung disease and the man arrives with his spirometry Questions Total 16
• Past Papers results. They are shown below.
Questions Percentage 75 %
• Random Questions Investigations;
• My Performance More
<td ">Post salbutamol
• Media Bank FEV1 1.86 (predicted 3.0)
• New Multimedia
FVC 2.5 (predicted 2.8) Reference: Normal Values
Online Extras
FEV1 Haematology
Library 2.0
Community FVC 2.7 Immunoglobulins
Help Transfer factor 55% Biochemistry
PasTest Store Diabetes
Which of the following is the most likely diagnosis?
Endocrinology
My Account Asthma
Blood gases
Profile Emphysema
CSF
Newsletters Pulmonary fibrosis
My Career and Exams Pulmonary embolism
Order History Asbestos related pleural plaque disease
Learning Goals
20929
Question Filters

Security

Change Password
Sign Out

© 2011 PasTest Ltd | About Us | Contact Us | Help

file:///E|/Shakhawan/Respiratory%20S/337.htm[3/13/2012 4:12:34 PM]


MyPasTest

Main Navigation
Question Browser: MRCP 1
Home
Subscriptions Question Browser Exam Builder Saved Exams
» MRCP 1 Session Progress
• Question Browser
Questions Correct 13
• Timed Test A 50-year-old retired boiler man with shortness of breath
comes to the clinic. The GP wonders if he has obstructive Questions Incorrect 4
• Mock Exam
lung disease and the man arrives with his spirometry Questions Total 17
• Past Papers results. They are shown below.
Questions Percentage 76 %
• Random Questions Investigations;
• My Performance More
<td ">Post salbutamol
• Media Bank FEV1 1.86 (predicted 3.0)
• New Multimedia
FVC 2.5 (predicted 2.8)
Online Extras
Library FEV1 2.0
Community FVC 2.7
Help Transfer factor 55%
PasTest Store
Which of the following is the most likely diagnosis?

My Account Asthma
Profile Emphysema Your answer
Newsletters Pulmonary fibrosis
My Career and Exams Pulmonary embolism
Order History Asbestos related pleural plaque disease Reference: Normal Values
Learning Goals Haematology
Question Filters
Immunoglobulins

FVC is only slightly reduced, but FEV1 /FVC is just under Biochemistry
Security
75%. Transfer factor is also reduced at 55%. With
Diabetes
Change Password reversibility of less than 10% post salbutamol, the most
likely diagnosis is COPD. Stopping smoking is the key Endocrinology
Sign Out initial measure, with trials also indicating that high dose
inhaled steroids combined with long acting beta 2 Blood gases
agonists may reduce exacerbation rates and improve
quality of life versus other options. CSF

20929

© 2011 PasTest Ltd | About Us | Contact Us | Help

file:///E|/Shakhawan/Respiratory%20S/337a.htm[3/13/2012 4:12:36 PM]


MyPasTest

Main Navigation
Question Browser: MRCP 1
Home
Subscriptions Question Browser Exam Builder Saved Exams
» MRCP 1 Session Progress
• Question Browser
Questions Correct 13
• Timed Test A 71-year-old man with a 40 pack year smoking history is
admitted with an exacerbation of COPD. He is usually Questions Incorrect 4
• Mock Exam
independent at home and manages with no intervention or Questions Total 17
• Past Papers social support. He has been unwell for the past 3 days with
Questions Percentage 76 %
• Random Questions increasing cough productive of purulent sputum. On
admission he appears to be significantly hypoxic and More
• My Performance
hypercapnic, with a pO2 of 7.1, and pCO 2 of 5.9. He is very
• Media Bank short of breath, but struggles to tell you that he does not
• New Multimedia want to be ventilated. Unfortunately he becomes
increasingly tired and becomes unresponsive some 30 mins Reference: Normal Values
Online Extras
later.
Haematology
Library
Community Which of the following is the most appropriate plan Immunoglobulins
with respect to his further management?
Help Biochemistry
PasTest Store Continue on antibiotics only
Diabetes
Contact hospital legal team to proceed to intubate
Endocrinology
My Account Ask the family to consent for him to be intubated
Intubate and act on the best interests of the patient, Blood gases
Profile whilst informing the relatives CSF
Newsletters Withdraw ALL treatment
My Career and Exams
Order History 21079

Learning Goals
Question Filters

Security

Change Password
Sign Out

© 2011 PasTest Ltd | About Us | Contact Us | Help

file:///E|/Shakhawan/Respiratory%20S/338.htm[3/13/2012 4:12:38 PM]


MyPasTest

Main Navigation
Question Browser: MRCP 1
Home
Subscriptions Question Browser Exam Builder Saved Exams
» MRCP 1 Session Progress
• Question Browser
Questions Correct 13
• Timed Test A 71-year-old man with a 40 pack year smoking history is
admitted with an exacerbation of COPD. He is usually Questions Incorrect 5
• Mock Exam
independent at home and manages with no intervention or Questions Total 18
• Past Papers social support. He has been unwell for the past 3 days with
Questions Percentage 72 %
• Random Questions increasing cough productive of purulent sputum. On
admission he appears to be significantly hypoxic and More
• My Performance
hypercapnic, with a pO2 of 7.1, and pCO 2 of 5.9. He is very
• Media Bank short of breath, but struggles to tell you that he does not
• New Multimedia want to be ventilated. Unfortunately he becomes
increasingly tired and becomes unresponsive some 30 mins
Online Extras
later.
Library
Community Which of the following is the most appropriate plan
with respect to his further management?
Help
PasTest Store Continue on antibiotics only
Contact hospital legal team to
proceed to intubate
My Account
Ask the family to consent for him to Your answer
Profile be intubated
Newsletters Intubate and act on the best Correct answer
interests of the patient, whilst
My Career and Exams
informing the relatives
Order History Reference: Normal Values
Withdraw ALL treatment
Learning Goals Haematology
Question Filters
Immunoglobulins
Biochemistry
Security This man had a reasonable level of functioning prior to
his acute exacerbation; as such he may well recover Diabetes
Change Password with appropriate intervention. If he had a written
Sign Out advanced directive, properly witnessed, while he was Endocrinology
well, then it would not be possible to consider Blood gases
intervention if he wished for it not to happen. On the
other hand, he has significant hypoxia now, and may CSF
not be able to give a rational decision with respect to
his further treatment. The family should not have the
final decision with respect to intubation, the decision on
management is yours, but you should of course keep
the family closely informed with respect to treatment.

21079

© 2011 PasTest Ltd | About Us | Contact Us | Help

file:///E|/Shakhawan/Respiratory%20S/338a.htm[3/13/2012 4:12:39 PM]


MyPasTest

Main Navigation
Question Browser: MRCP 1
Home
Subscriptions Question Browser Exam Builder Saved Exams
» MRCP 1 Session Progress
• Question Browser
Questions Correct 13
• Timed Test A 25-year-old smoker of 5 cigarettes per day comes to the
clinic complaining of recurrent haemoptysis that he has Questions Incorrect 5
• Mock Exam
suffered from for the past 2 years. He has been treated for Questions Total 18
• Past Papers intermittent cough and respiratory infections over the past
Questions Percentage 72 %
• Random Questions few years. On examination he looks a little thin but
otherwise well. Respiratory examination raises the More
• My Performance
suggestion of left upper lobe collapse. There are no other
• Media Bank abnormal findings.
• New Multimedia
Investigations; Reference: Normal Values
Online Extras
Haematology
Library Hb 11.9 g/dl
Community Immunoglobulins
WCC 5.9 x10 9 /l
Help Biochemistry
PLT 187 x10 9 /l
PasTest Store Diabetes
Na + 141 mmol/l
Endocrinology
My Account K+ 4.2 mmol/l
Blood gases
Profile Creatinine 110 μmol/l
CSF
Newsletters CXR left upper lobe collapse
My Career and Exams
Which of the following is the most likely diagnosis?
Order History
Learning Goals Inhaled foreign body
Question Filters Bronchiectasis
Bronchial carcinoma
Security Left upper lobe pneumonia

Change Password Bronchial carcinoid

Sign Out 21080

© 2011 PasTest Ltd | About Us | Contact Us | Help

file:///E|/Shakhawan/Respiratory%20S/339.htm[3/13/2012 4:12:41 PM]


MyPasTest

Main Navigation
Question Browser: MRCP 1
Home
Subscriptions Question Browser Exam Builder Saved Exams
» MRCP 1 Session Progress
• Question Browser
Questions Correct 13
• Timed Test A 25-year-old smoker of 5 cigarettes per day comes to the
clinic complaining of recurrent haemoptysis that he has Questions Incorrect 6
• Mock Exam
suffered from for the past 2 years. He has been treated for Questions Total 19
• Past Papers intermittent cough and respiratory infections over the past
Questions Percentage 68 %
• Random Questions few years. On examination he looks a little thin but
otherwise well. Respiratory examination raises the More
• My Performance
suggestion of left upper lobe collapse. There are no other
• Media Bank abnormal findings.
• New Multimedia
Investigations;
Online Extras
Library Hb 11.9 g/dl
Community
WCC 5.9 x10 9 /l
Help
PLT 187 x10 9 /l
PasTest Store
Na + 141 mmol/l

My Account K+ 4.2 mmol/l

Profile Creatinine 110 μmol/l


Newsletters CXR left upper lobe collapse
My Career and Exams
Which of the following is the most likely diagnosis?
Order History Reference: Normal Values
Learning Goals Inhaled foreign body Your answer Haematology
Question Filters Bronchiectasis
Immunoglobulins
Bronchial carcinoma
Biochemistry
Security Left upper lobe pneumonia
Bronchial carcinoid Correct answer Diabetes
Change Password
Sign Out Endocrinology

Blood gases

CSF
Recurrent haemoptysis with segmental collapse is a
typical presentation associated with bronchial carcinoid.
No features suggest that there is active infection, which
makes both left upper lobe pneumonia and
bronchiectasis unlikely. An inhaled foreign body would
be more likely to lie in the right main bronchus and as
such does not fit with the clinical scenario here. Given
the fact that this patient is relatively well, and has
limited chest disease, bronchial carcinoid is much more
likely than a carcinoma. The prolonged clinical course,
without features of carcinoid syndrome is typical of a
bronchial carcinoid tumour.

21080

© 2011 PasTest Ltd | About Us | Contact Us | Help

file:///E|/Shakhawan/Respiratory%20S/339a.htm[3/13/2012 4:12:43 PM]


MyPasTest

Main Navigation
Question Browser: MRCP 1
Home
Subscriptions Question Browser Exam Builder Saved Exams
» MRCP 1 Session Progress
• Question Browser
Questions Correct 13
• Timed Test A 17-year-girl who has known cystic fibrosis presents with
increasing cough productive of purulent sputum. She has Questions Incorrect 6
• Mock Exam
had 3 previous admissions to hospital with exacerbations Questions Total 19
• Past Papers over the past 4 years. So far she is maintaining her weight
Questions Percentage 68 %
• Random Questions and is able to continue her studies at school. On
• My Performance examination she is pyrexial 37.8 o C, her BP is 120/72 More
mmHg, pulse 90/min, regular. She has bilateral crackles
• Media Bank and wheeze; the crackles are particularly increased at the
• New Multimedia left base.
Reference: Normal Values
Online Extras
Investigations; Haematology
Library
Community Hb 12.0 g/dl Immunoglobulins
Help WCC 13.1 x10 9 /l Biochemistry
PasTest Store Diabetes
PLT 181 x10 9 /l

Na + 141 mmol/l Endocrinology


My Account
K+ Blood gases
4.9 mmol/l
Profile
CSF
Newsletters Creatinine 110 μmol/l
My Career and Exams CRP 71 mg/l
Order History
Which of the following is the most appropriate initial
Learning Goals antimicrobial treatment?
Question Filters
Piperacillin plus tazobactam (Tazocin)
Ciprofloxacin
Security
Benzylpenicillin
Change Password
Erythromycin
Sign Out
Clindamycin

21081

© 2011 PasTest Ltd | About Us | Contact Us | Help

file:///E|/Shakhawan/Respiratory%20S/340.htm[3/13/2012 4:12:45 PM]


MyPasTest

Main Navigation
Question Browser: MRCP 1
Home
Subscriptions Question Browser Exam Builder Saved Exams
» MRCP 1 Session Progress
• Question Browser
Questions Correct 14
• Timed Test A 17-year-girl who has known cystic fibrosis presents with
increasing cough productive of purulent sputum. She has Questions Incorrect 6
• Mock Exam
had 3 previous admissions to hospital with exacerbations Questions Total 20
• Past Papers over the past 4 years. So far she is maintaining her weight
Questions Percentage 70 %
• Random Questions and is able to continue her studies at school. On
• My Performance examination she is pyrexial 37.8 o C, her BP is 120/72 More
mmHg, pulse 90/min, regular. She has bilateral crackles
• Media Bank and wheeze; the crackles are particularly increased at the
• New Multimedia left base.
Online Extras
Investigations;
Library
Community Hb 12.0 g/dl
Help WCC 13.1 x10 9 /l
PasTest Store
PLT 181 x10 9 /l

Na + 141 mmol/l
My Account
K+ 4.9 mmol/l
Profile
Newsletters Creatinine 110 μmol/l
My Career and Exams CRP 71 mg/l
Order History Reference: Normal Values
Which of the following is the most appropriate initial
Learning Goals antimicrobial treatment? Haematology
Question Filters
Piperacillin plus tazobactam (Tazocin) Your answer Immunoglobulins

Ciprofloxacin Biochemistry
Security
Benzylpenicillin Diabetes
Change Password
Erythromycin
Sign Out Endocrinology
Clindamycin
Blood gases

CSF

The combination of piperacillin and tazobactam


(Tazocin) and is indicated for the treatment of cystic
fibrosis exacerbation. The usual dose for adults is 4.5g
of Tazocin given every 8hrs, rash and fever may be
seen in association with Tazocin administration more
often in patients with cystic fibrosis versus others. The
usual treatment duration is 7-10 days in total.
Ciprofloxacin is no longer widely recommended for
exacerbations of CF because resistance rates of up to
30% have been shown in some case series.

21081

© 2011 PasTest Ltd | About Us | Contact Us | Help

file:///E|/Shakhawan/Respiratory%20S/340a.htm[3/13/2012 4:12:47 PM]


MyPasTest

Main Navigation
Question Browser: MRCP 1
Home
Subscriptions Question Browser Exam Builder Saved Exams
» MRCP 1 Session Progress
• Question Browser
Questions Correct 14
• Timed Test A 28-year-old woman comes to the clinic for review 4
weeks after discharge from the ward following an asthma Questions Incorrect 6
• Mock Exam
attack. She was diagnosed with asthma aged 7 and has Questions Total 20
• Past Papers been taking inhaled beclomethasone 800mcg and
Questions Percentage 70 %
• Random Questions salbutamol PRN since then. She tells you that since
discharge she has been using her salbutamol 3 times per More
• My Performance
day and has been waking 2-3 times per night with
• Media Bank coughing. On examination her peak flow is 340 (predicted is
• New Multimedia 570). She has scattered wheeze throughout both lung fields
on examination. Good inhaler technique with volumatic as Reference: Normal Values
Online Extras assessed in the clinic.
Haematology
Library
Investigations; Immunoglobulins
Community
Help Hb 13.1 g/dl Biochemistry
PasTest Store 9 Diabetes
WCC 6.5 x10 /l

PLT 231 x10 9 /l Endocrinology


My Account
ESR 12 mm/hr Blood gases
Profile
Na + 140 mmol/l CSF
Newsletters
My Career and Exams K+ 4.9 mmol/l
Order History Creatinine 110 μmol/l
Learning Goals CXR No evidence of consolidation
Question Filters
Which of the following is the most appropriate
management plan for her?
Security
Change her to 800mcg of fluticasone
Change Password
Add low dose oral theophylline to her regimen
Sign Out
Add oral monteleukast to her regimen
Add BD inhaled salmeterol to her regimen
Add omalizumab

21254

© 2011 PasTest Ltd | About Us | Contact Us | Help

file:///E|/Shakhawan/Respiratory%20S/341.htm[3/13/2012 4:12:48 PM]


MyPasTest

Main Navigation
Question Browser: MRCP 1
Home
Subscriptions Question Browser Exam Builder Saved Exams
» MRCP 1 Session Progress
• Question Browser
Questions Correct 15
• Timed Test A 28-year-old woman comes to the clinic for review 4
weeks after discharge from the ward following an asthma Questions Incorrect 6
• Mock Exam
attack. She was diagnosed with asthma aged 7 and has Questions Total 21
• Past Papers been taking inhaled beclomethasone 800mcg and
Questions Percentage 71 %
• Random Questions salbutamol PRN since then. She tells you that since
discharge she has been using her salbutamol 3 times per More
• My Performance
day and has been waking 2-3 times per night with
• Media Bank coughing. On examination her peak flow is 340 (predicted is
• New Multimedia 570). She has scattered wheeze throughout both lung fields
on examination. Good inhaler technique with volumatic as
Online Extras assessed in the clinic.
Library
Investigations;
Community
Help Hb 13.1 g/dl
PasTest Store
WCC 6.5 x10 9 /l

PLT 231 x10 9 /l


My Account
ESR 12 mm/hr
Profile
Na + 140 mmol/l
Newsletters
My Career and Exams K+ 4.9 mmol/l
Order History Creatinine 110 μmol/l Reference: Normal Values
Learning Goals CXR No evidence of consolidation Haematology
Question Filters
Which of the following is the most appropriate Immunoglobulins
management plan for her? Biochemistry
Security
Change her to 800mcg of fluticasone Diabetes
Change Password
Add low dose oral theophylline to her Endocrinology
Sign Out regimen
Add oral monteleukast to her regimen Blood gases

Add BD inhaled salmeterol to her Your answer CSF


regimen
Add omalizumab

British Thoracic Society (BTS) guidelines recommend


the addition of a long acting inhaled beta agonist to
therapy before further uptitrating inhaled corticosteroids
above this level. Oral monteleukast is an option after
addition of salmeterol. Another option is maximisation
of inhaled corticosteroids, as is the addition of a
theophylline preparation. Omalizumab, an anti-IgE
antibody is reserved for specialist therapy in patients
with severe allergic asthma in whom other treatments
have been unsuccessful.

21254

file:///E|/Shakhawan/Respiratory%20S/341a.htm[3/13/2012 4:12:50 PM]


MyPasTest

Main Navigation
Question Browser: MRCP 1
Home
Subscriptions Question Browser Exam Builder Saved Exams
» MRCP 1 Session Progress
• Question Browser
Questions Correct 15
• Timed Test A 32-year-old man with a 2 year history of asthma
diagnosed and managed by his GP now presents with Questions Incorrect 6
• Mock Exam
weight loss, abdominal pain diarrhoea, and petechial skin Questions Total 21
• Past Papers changes. During the past week he has become increasingly
Questions Percentage 71 %
• Random Questions lethargic with associated feelings of nausea. Other past
history of note includes an episode of foot drop which More
• My Performance
occurred some 3 months ago.
• Media Bank
• New Multimedia Investigations;
Reference: Normal Values
Online Extras
Hb 11.0 g/dl Haematology
Library
9
WCC 10.0 x10 /l (raised eosinophils) Immunoglobulins
Community
Help PLT 190 x10 9 /l Biochemistry
PasTest Store Na + 141 mmol/l Diabetes
K+ 5.7 mmol/l Endocrinology
My Account
Creatinine 234 μmol/l Blood gases
Profile ESR 71 mm/hr
CSF
Newsletters Rheumatoid factor +
My Career and Exams CXR Bilateral pulmonary infiltrates
Order History
Urine blood ++, protein ++
Learning Goals
Question Filters Which of the following is the most likely positive
antibody finding in this case?

Security Anticardiolipin

Change Password C-ANCA

Sign Out Antimitochondrial


Anti GBM
Anti-myeloperoxidase

21264

© 2011 PasTest Ltd | About Us | Contact Us | Help

file:///E|/Shakhawan/Respiratory%20S/342.htm[3/13/2012 4:12:51 PM]


MyPasTest

Main Navigation
Question Browser: MRCP 1
Home
Subscriptions Question Browser Exam Builder Saved Exams
» MRCP 1 Session Progress
• Question Browser
Questions Correct 16
• Timed Test A 32-year-old man with a 2 year history of asthma
diagnosed and managed by his GP now presents with Questions Incorrect 6
• Mock Exam
weight loss, abdominal pain diarrhoea, and petechial skin Questions Total 22
• Past Papers changes. During the past week he has become increasingly
Questions Percentage 72 %
• Random Questions lethargic with associated feelings of nausea. Other past
history of note includes an episode of foot drop which More
• My Performance
occurred some 3 months ago.
• Media Bank
• New Multimedia Investigations;

Online Extras
Hb 11.0 g/dl
Library
WCC 10.0 x10 9 /l (raised eosinophils)
Community
Help PLT 190 x10 9 /l
PasTest Store Na + 141 mmol/l

K+ 5.7 mmol/l
My Account
Creatinine 234 μmol/l
Profile ESR 71 mm/hr
Newsletters Rheumatoid factor +
My Career and Exams CXR Bilateral pulmonary infiltrates
Order History Reference: Normal Values
Urine blood ++, protein ++
Learning Goals Haematology
Question Filters Which of the following is the most likely positive
antibody finding in this case? Immunoglobulins
Biochemistry
Security Anticardiolipin
C-ANCA Diabetes
Change Password
Sign Out Antimitochondrial Endocrinology
Anti GBM Blood gases
Anti-myeloperoxidase Your answer
CSF

The clinical picture seen here is very suggestive of


Churg-Strauss syndrome, which is commonly
associated with anti-myeloperoxidase antibodies. Raised
eosinophil count is seen in 90%+ of patients with
advanced disease. Skin biopsy reveals small vessel
arteriopathy with granuloma formation and is a
diagnostic investigation of choice. High dose
methylprednisolone +/- cyclophosphamide is the
treatment of choice for this condition.

21264

© 2011 PasTest Ltd | About Us | Contact Us | Help

file:///E|/Shakhawan/Respiratory%20S/342a.htm[3/13/2012 4:12:53 PM]


MyPasTest

Main Navigation
Question Browser: MRCP 1
Home
Subscriptions Question Browser Exam Builder Saved Exams
» MRCP 1 Session Progress
• Question Browser
Questions Correct 16
• Timed Test You are referred a 68-year-old man who smokes 40
Questions Incorrect 6
• Mock Exam cigarettes per day and has suffered from a chronic cough
for the past 6 months, increasingly associated with Questions Total 22
• Past Papers haemoptysis. He also has a dull ache on the left side of his Questions Percentage 72 %
• Random Questions chest, and his CXR reveals a left hilar mass suspicious of
bronchial carcinoma. You are considering radical More
• My Performance
radiotherapy in this man.
• Media Bank
Which of the following is a relative contraindication
• New Multimedia
to radical radiotherapy? Reference: Normal Values
Online Extras
SVC obstruction Haematology
Library
Community Tumour adjacent to the hilum Immunoglobulins
Help Malignant pleural effusion Biochemistry
PasTest Store Adenocarcinoma
Diabetes
FEV1 < 60%
Endocrinology
My Account
21326 Blood gases
Profile
CSF
Newsletters
My Career and Exams
Order History
Learning Goals
Question Filters

Security

Change Password
Sign Out

© 2011 PasTest Ltd | About Us | Contact Us | Help

file:///E|/Shakhawan/Respiratory%20S/343.htm[3/13/2012 4:12:55 PM]


MyPasTest

Main Navigation
Question Browser: MRCP 1
Home
Subscriptions Question Browser Exam Builder Saved Exams
» MRCP 1 Session Progress
• Question Browser
Questions Correct 17
• Timed Test You are referred a 68-year-old man who smokes 40
cigarettes per day and has suffered from a chronic cough Questions Incorrect 6
• Mock Exam
for the past 6 months, increasingly associated with Questions Total 23
• Past Papers haemoptysis. He also has a dull ache on the left side of his
Questions Percentage 73 %
• Random Questions chest, and his CXR reveals a left hilar mass suspicious of
bronchial carcinoma. You are considering radical More
• My Performance
radiotherapy in this man.
• Media Bank
Which of the following is a relative contraindication
• New Multimedia
to radical radiotherapy?
Online Extras
Library SVC obstruction
Community Tumour adjacent to the hilum
Help Malignant pleural effusion Your answer
PasTest Store Adenocarcinoma
FEV1 < 60%

My Account

Profile
Newsletters
It was previously thought that patients with FEV1 <50%
My Career and Exams were at particular risk from post radiotherapy
Order History pneumonitis, although it now appears that some Reference: Normal Values
patients enrolled in radical radiotherapy trials with
Learning Goals Haematology
severe disease actually showed a small improvement in
Question Filters lung function. SVC obstruction and tumour adjacency to
Immunoglobulins
the hilum may increase surgical difficultly, but actually
targeted radiotherapy may not be a problem in the Biochemistry
Security majority of patients. Studies have however shown that
presence of malignant pleural effusion is predictive of Diabetes
Change Password poor outcome in conjunction with radical radiotherapy.
Sign Out Endocrinology
Acta Oncologica, Volume 31, Issue 5 1992 , pages 555
Blood gases
– 561
http://erj.ersjournals.com/cgi/reprint/34/1/17.pdf CSF

21326

© 2011 PasTest Ltd | About Us | Contact Us | Help

file:///E|/Shakhawan/Respiratory%20S/343a.htm[3/13/2012 4:12:56 PM]


MyPasTest

Main Navigation
Question Browser: MRCP 1
Home
Subscriptions Question Browser Exam Builder Saved Exams
» MRCP 1 Session Progress
• Question Browser
Questions Correct 17
• Timed Test A 62-year-old woman is admitted with confusion and
increased respiratory rate. She has been managed by her Questions Incorrect 6
• Mock Exam
GP for shortness of breath and is taking ramipril and Questions Total 23
• Past Papers indapamide for hypertension and has a salbutamol inhaler.
Questions Percentage 73 %
• Random Questions She came to the Emergency room with her daughter
because of concerns that she was getting worse. On More
• My Performance
examination her BP is 112/62 mmHg, she has a pyrexia of
• Media Bank 37.4 o C. Pulse is 75/min and regular and heart sounds are
• New Multimedia normal. Auscultation of the chest reveals scattered crackles
and wheeze. Reference: Normal Values
Online Extras
Haematology
Library Investigations;
Community Immunoglobulins
Hb 13.1 g/dl
Help Biochemistry
PasTest Store WCC 9.2 x10 9 /l
Diabetes
PLT 201 x10 9 /l
Endocrinology
My Account Na + 138 mmol/l
Blood gases
Profile K+ 4.5 mmol/l
CSF
Newsletters Bicarbonate 22 mmol/l
My Career and Exams Creatinine 130 µmol/l
Order History
pO2 9.1 kPa
Learning Goals
pCO 2 7.2 kPa
Question Filters
pH 7.2

Security Which of the following is the most likely diagnosis?


Change Password
Acute respiratory acidosis
Sign Out
Acute on chronic respiratory acidosis
Metabolic acidosis
Mixed metabolic and respiratory acidosis
Chronic respiratory acidosis

21327

© 2011 PasTest Ltd | About Us | Contact Us | Help

file:///E|/Shakhawan/Respiratory%20S/344.htm[3/13/2012 4:12:58 PM]


MyPasTest

Main Navigation
Question Browser: MRCP 1
Home
Subscriptions Question Browser Exam Builder Saved Exams
» MRCP 1 Session Progress
• Question Browser
Questions Correct 17
• Timed Test A 62-year-old woman is admitted with confusion and
increased respiratory rate. She has been managed by her Questions Incorrect 7
• Mock Exam
GP for shortness of breath and is taking ramipril and Questions Total 24
• Past Papers indapamide for hypertension and has a salbutamol inhaler.
Questions Percentage 70 %
• Random Questions She came to the Emergency room with her daughter
because of concerns that she was getting worse. On More
• My Performance
examination her BP is 112/62 mmHg, she has a pyrexia of
• Media Bank 37.4 o C. Pulse is 75/min and regular and heart sounds are
• New Multimedia normal. Auscultation of the chest reveals scattered crackles
and wheeze.
Online Extras
Library Investigations;
Community
Hb 13.1 g/dl
Help
PasTest Store WCC 9.2 x10 9 /l

PLT 201 x10 9 /l


My Account Na + 138 mmol/l
Profile K+ 4.5 mmol/l
Newsletters Bicarbonate 22 mmol/l
My Career and Exams Creatinine 130 µmol/l
Order History Reference: Normal Values
pO2 9.1 kPa
Learning Goals Haematology
pCO 2 7.2 kPa
Question Filters
Immunoglobulins
pH 7.2
Biochemistry
Security Which of the following is the most likely diagnosis?
Diabetes
Change Password
Acute respiratory acidosis Correct answer Endocrinology
Sign Out
Acute on chronic respiratory acidosis
Blood gases
Metabolic acidosis
CSF
Mixed metabolic and respiratory Your answer
acidosis
Chronic respiratory acidosis

This woman is hypercapnic with decreased pH. This has


occurred too quickly for metabolic compensation to
occur via renal bicarbonate reabsorption, as this takes
3-5 days to occur. As such it is an acute event such as
a COPD exacerbation that is most likely to have led to
her deterioration in symptoms. Therefore aggressive
management is likely to return her to a reasonable level
of function.

21327

© 2011 PasTest Ltd | About Us | Contact Us | Help

file:///E|/Shakhawan/Respiratory%20S/344a.htm[3/13/2012 4:12:59 PM]


MyPasTest

Main Navigation
Question Browser: MRCP 1
Home
Subscriptions Question Browser Exam Builder Saved Exams
» MRCP 1 Session Progress
• Question Browser
Questions Correct 17
• Timed Test A 29-year-old woman with brittle asthma is admitted to the
Emergency room with a viral exacerbation of her asthma. Questions Incorrect 7
• Mock Exam
Her usual peak flow is around 490 l/min, and she is Questions Total 24
• Past Papers managed with a high dose seretide inhaler. On examination
Questions Percentage 70 %
• Random Questions her BP is 145/80 mmHg, pulse is 105/min, regular. She has
a respiratory rate of 40/min and looks exhausted. On More
• My Performance
auscultation you can hear wheeze and decreased air entry.
• Media Bank Her peak flow is measured at 180 l/min.
• New Multimedia
Investigations; Reference: Normal Values
Online Extras
Hb 13.1 g/dl Haematology
Library
Community WCC 8.1 x10 9 /l Immunoglobulins
Help Biochemistry
PLT 249 x10 9 /l
PasTest Store Diabetes
Na + 141 mmol/l
Endocrinology
My Account K+ 3.9 mmol/l
Creatinine 110 µmol/l Blood gases
Profile
PaO2 10.5 kPa CSF
Newsletters
My Career and Exams pCO 2 6.4 kPa
Order History
Her peak flow has not improved 30 mins after admission,
Learning Goals
despite salbutamol and atrovent nebulisers and IV
Question Filters hydrocortisone. You arrange review by the ITU registrar.

Whist you are waiting for her visit, which of the


Security following is the most appropriate next management
step?
Change Password
Sign Out IV aminophylline
IV salbutamol
IV magnesium
Inhaled helium oxygen mixture
NIPPV

21366

© 2011 PasTest Ltd | About Us | Contact Us | Help

file:///E|/Shakhawan/Respiratory%20S/345.htm[3/13/2012 4:13:01 PM]


MyPasTest

Main Navigation
Question Browser: MRCP 1
Home
Subscriptions Question Browser Exam Builder Saved Exams
» MRCP 1 Session Progress
• Question Browser
Questions Correct 18
• Timed Test A 29-year-old woman with brittle asthma is admitted to the
Emergency room with a viral exacerbation of her asthma. Questions Incorrect 7
• Mock Exam
Her usual peak flow is around 490 l/min, and she is Questions Total 25
• Past Papers managed with a high dose seretide inhaler. On examination
Questions Percentage 72 %
• Random Questions her BP is 145/80 mmHg, pulse is 105/min, regular. She has
a respiratory rate of 40/min and looks exhausted. On More
• My Performance
auscultation you can hear wheeze and decreased air entry.
• Media Bank Her peak flow is measured at 180 l/min.
• New Multimedia
Investigations;
Online Extras
Library Hb 13.1 g/dl
Community WCC 8.1 x10 9 /l
Help
PLT 249 x10 9 /l
PasTest Store
Na + 141 mmol/l

My Account K+ 3.9 mmol/l


Creatinine 110 µmol/l
Profile
PaO2 10.5 kPa
Newsletters
My Career and Exams pCO 2 6.4 kPa
Order History Reference: Normal Values
Her peak flow has not improved 30 mins after admission,
Learning Goals Haematology
despite salbutamol and atrovent nebulisers and IV
Question Filters hydrocortisone. You arrange review by the ITU registrar.
Immunoglobulins
Whist you are waiting for her visit, which of the Biochemistry
Security following is the most appropriate next management
step? Diabetes
Change Password
Sign Out IV aminophylline Endocrinology
IV salbutamol Blood gases
IV magnesium Your answer CSF
Inhaled helium oxygen mixture
NIPPV

A Cochrane meta-analysis showed some benefit from IV


magnesium in acute asthma, and for this reason it is
recommended in the British asthma guideline update
from 2008 in patients who have failed to improve on
salbutamol, atrovent and IV corticosteroids. Her pCO 2 is
just outside the upper limit of normal, for this reason
she requires urgent ITU admission. IV aminophylline is
not recommended routinely and should only be used in
specific patients after consultation with senior staff.

21366

© 2011 PasTest Ltd | About Us | Contact Us | Help

file:///E|/Shakhawan/Respiratory%20S/345a.htm[3/13/2012 4:13:02 PM]


MyPasTest

Main Navigation
Question Browser: MRCP 1
Home
Subscriptions Question Browser Exam Builder Saved Exams
» MRCP 1 Session Progress
• Question Browser
Questions Correct 18
• Timed Test A 29-year-old IV heroin abuser is admitted to the
Emergency department with a severe cough, fever and Questions Incorrect 7
• Mock Exam
rigors. He says that he has suffered progressively Questions Total 25
• Past Papers increasing shortness of breath on exertion over the past
Questions Percentage 72 %
• Random Questions few days. On examination he has a pyrexia of 37.9 o C, he
• My Performance has a BP of 122/75 mmHg and a BMI of 17. There are mild More
crackles and wheeze on auscultation of the chest.
• Media Bank
• New Multimedia Investigations;
Reference: Normal Values
Online Extras
Hb 10.9 g/dl Haematology
Library
9
WCC 6.1 x10 /l Immunoglobulins
Community
PLT 245 x109/l
Help Biochemistry
PasTest Store Na + 141 mmol/l
Diabetes
K+ 4.0 mmol/l Endocrinology
My Account Creatinine 130 µmol/l
Blood gases
Profile LDH 420 U/l (70-250)
CSF
Newsletters Sats 92% on air, 89% after walk test
My Career and Exams CXR Diffuse bilateral infiltrates
Order History
Which of the following is the most likely diagnosis?
Learning Goals
Question Filters Tuberculosis
Endocarditis
Security Klebsiella pneumoniae pneumonia
Change Password Pneumocystis jiroveci pneumonia
Sign Out Staphylococcus aureus pneumonia

21367

© 2011 PasTest Ltd | About Us | Contact Us | Help

file:///E|/Shakhawan/Respiratory%20S/346.htm[3/13/2012 4:13:04 PM]


MyPasTest

Main Navigation
Question Browser: MRCP 1
Home
Subscriptions Question Browser Exam Builder Saved Exams
» MRCP 1 Session Progress
• Question Browser
Questions Correct 19
• Timed Test A 29-year-old IV heroin abuser is admitted to the
Emergency department with a severe cough, fever and Questions Incorrect 7
• Mock Exam
rigors. He says that he has suffered progressively Questions Total 26
• Past Papers increasing shortness of breath on exertion over the past
Questions Percentage 73 %
• Random Questions few days. On examination he has a pyrexia of 37.9 o C, he
• My Performance has a BP of 122/75 mmHg and a BMI of 17. There are mild More
crackles and wheeze on auscultation of the chest.
• Media Bank
• New Multimedia Investigations;
Online Extras
Hb 10.9 g/dl
Library
WCC 6.1 x10 9 /l
Community
PLT 245 x109/l
Help
PasTest Store Na + 141 mmol/l

K+ 4.0 mmol/l
My Account Creatinine 130 µmol/l
Profile LDH 420 U/l (70-250)
Newsletters Sats 92% on air, 89% after walk test
My Career and Exams CXR Diffuse bilateral infiltrates
Order History Reference: Normal Values
Which of the following is the most likely diagnosis?
Learning Goals Haematology
Question Filters Tuberculosis Immunoglobulins
Endocarditis
Biochemistry
Security Klebsiella pneumoniae pneumonia
Diabetes
Change Password Pneumocystis jiroveci pneumonia Your answer
Sign Out Staphylococcus aureus pneumonia Endocrinology

Blood gases

CSF
The clinical picture seen here, with relatively little to
find on auscultation, but with chest x-ray changes and
desaturation on exercise is very typical of pneumocystis
jiroveci. This may be diagnosed on the basis of a
sputum sample, although bronchio-alveolar lavage may
be required to obtain a suitable sample, the yield for
BAL samples is over 90%. Co-trimoxazole or
pentamidine are both proven treatments for the
condition. Given the possible diagnosis he should be
screened for HIV.

21367

© 2011 PasTest Ltd | About Us | Contact Us | Help

file:///E|/Shakhawan/Respiratory%20S/346a.htm[3/13/2012 4:13:05 PM]


MyPasTest

Main Navigation
Question Browser: MRCP 1
Home
Subscriptions Question Browser Exam Builder Saved Exams
» MRCP 1 Session Progress
• Question Browser
Questions Correct 19
• Timed Test You review a 67-year-old man with COPD. He has smoked
30 cigarettes per day for around the past 40 years. Questions Incorrect 7
• Mock Exam
Pulmonary function tests indicate that he has a Questions Total 26
• Past Papers predominantly emphysematous picture.
Questions Percentage 73 %
• Random Questions
Which of the following is the most important factor in More
• My Performance airflow limitation in severe emphysema?
• Media Bank
• New Multimedia Smooth muscle contraction
Reference: Normal Values
Online Extras Large airways obstruction
Mucosal oedema Haematology
Library
Community Loss of elastic recoil Immunoglobulins
Help Mucus plugging
Biochemistry
PasTest Store 21368 Diabetes
Endocrinology
My Account
Blood gases
Profile
CSF
Newsletters
My Career and Exams
Order History
Learning Goals
Question Filters

Security

Change Password
Sign Out

© 2011 PasTest Ltd | About Us | Contact Us | Help

file:///E|/Shakhawan/Respiratory%20S/347.htm[3/13/2012 4:13:07 PM]


MyPasTest

Main Navigation
Question Browser: MRCP 1
Home
Subscriptions Question Browser Exam Builder Saved Exams
» MRCP 1 Session Progress
• Question Browser
Questions Correct 20
• Timed Test You review a 67-year-old man with COPD. He has smoked
30 cigarettes per day for around the past 40 years. Questions Incorrect 7
• Mock Exam
Pulmonary function tests indicate that he has a Questions Total 27
• Past Papers predominantly emphysematous picture.
Questions Percentage 74 %
• Random Questions
Which of the following is the most important factor in More
• My Performance airflow limitation in severe emphysema?
• Media Bank
• New Multimedia Smooth muscle contraction

Online Extras Large airways obstruction

Library Mucosal oedema

Community Loss of elastic recoil Your answer

Help Mucus plugging

PasTest Store

My Account Emphysema is characterised by focal destruction limited


to the airspaces distal to terminal bronchioles. When
Profile the disease is severe, it is loss of elastic recoil which
drives airflow limitation. Although airflow limitation is
Newsletters
virtually irreversible, the small inflammatory component
My Career and Exams may respond to high dose inhaled corticosteroids.
Order History Reference: Normal Values
21368
Learning Goals Haematology
Question Filters
Immunoglobulins
Biochemistry
Security
Diabetes
Change Password
Sign Out Endocrinology

Blood gases

CSF

© 2011 PasTest Ltd | About Us | Contact Us | Help

file:///E|/Shakhawan/Respiratory%20S/347a.htm[3/13/2012 4:13:09 PM]


MyPasTest

Main Navigation
Question Browser: MRCP 1
Home
Subscriptions Question Browser Exam Builder Saved Exams
» MRCP 1 Session Progress
• Question Browser
Questions Correct 20
• Timed Test A 56-year-old alcoholic is admitted to the Emergency
department with a pyrexia and cough productive of green Questions Incorrect 7
• Mock Exam
sputum. He admits to drinking around 8 pints of cider per Questions Total 27
• Past Papers day and an unspecified amount of gin. He has been
Questions Percentage 74 %
• Random Questions admitted on a number of occasions with decreased
consciousness and once previously with aspiration More
• My Performance
pneumonia. On examination he is pyrexial 38.2 o C, with a
• Media Bank BP of 105/60 mmHg, and has bilateral coarse breath
• New Multimedia sounds and crackles.
Reference: Normal Values
Online Extras Investigations;
Haematology
Library
Community Hb 12.1 g/dl Immunoglobulins
Help WCC 12.9 x10 9 /l Biochemistry
PasTest Store PLT 245 x10 9 /l Diabetes

Na + 140 mmol/l Endocrinology


My Account
K+ 4.5 mmol/l Blood gases
Profile
Creatinine 140 µmol/l CSF
Newsletters
CXR Cavitating lesions involving the upper lobes
My Career and Exams
Order History Which of the following is the most likely diagnosis?
Learning Goals
Tuberculosis
Question Filters
Mycoplasma
Legionella
Security
Pneumococcus
Change Password
Klebsiella
Sign Out
21369

© 2011 PasTest Ltd | About Us | Contact Us | Help

file:///E|/Shakhawan/Respiratory%20S/348.htm[3/13/2012 4:13:10 PM]


MyPasTest

Main Navigation
Question Browser: MRCP 1
Home
Subscriptions Question Browser Exam Builder Saved Exams
» MRCP 1 Session Progress
• Question Browser
Questions Correct 21
• Timed Test A 56-year-old alcoholic is admitted to the Emergency
department with a pyrexia and cough productive of green Questions Incorrect 7
• Mock Exam
sputum. He admits to drinking around 8 pints of cider per Questions Total 28
• Past Papers day and an unspecified amount of gin. He has been
Questions Percentage 75 %
• Random Questions admitted on a number of occasions with decreased
consciousness and once previously with aspiration More
• My Performance
pneumonia. On examination he is pyrexial 38.2 o C, with a
• Media Bank BP of 105/60 mmHg, and has bilateral coarse breath
• New Multimedia sounds and crackles.
Online Extras Investigations;
Library
Community Hb 12.1 g/dl

Help WCC 12.9 x10 9 /l


PasTest Store PLT 245 x10 9 /l

Na + 140 mmol/l
My Account
K+ 4.5 mmol/l
Profile
Creatinine 140 µmol/l
Newsletters
CXR Cavitating lesions involving the upper lobes
My Career and Exams
Order History Which of the following is the most likely diagnosis? Reference: Normal Values
Learning Goals Haematology
Tuberculosis
Question Filters
Mycoplasma Immunoglobulins

Legionella Biochemistry
Security
Pneumococcus Diabetes
Change Password
Klebsiella Your answer
Sign Out Endocrinology

Blood gases

CSF
Klebsiella pneumonia appears to occur with increased
frequency in patients with a history of alcoholism and
the typical picture is one of cavitating lesions
predominantly affecting the upper lobes as is seen here.
Third generation cephalosporins or quinolones are used
as standard therapy for Klebsiella infection.
Unfortunately klebsiella carries a mortality rate of up to
50%, patients who respond to therapy are at increased
risk of lung abscess formation.

21369

© 2011 PasTest Ltd | About Us | Contact Us | Help

file:///E|/Shakhawan/Respiratory%20S/348a.htm[3/13/2012 4:13:12 PM]


MyPasTest

Main Navigation
Question Browser: MRCP 1
Home
Subscriptions Question Browser Exam Builder Saved Exams
» MRCP 1 Session Progress
• Question Browser
Questions Correct 21
• Timed Test A 32-year-old woman is admitted to the Emergency room
with a severe cough and shortness of breath. She has been Questions Incorrect 7
• Mock Exam
unwell for a few days with a cough and sore throat and Questions Total 28
• Past Papers now says she is coughing purulent sputum which is rust
Questions Percentage 75 %
• Random Questions coloured/ blood stained. She has a history of asthma which
is usually managed with a seretide 50 inhaler. On More
• My Performance
examination she is pyrexial 38.2 o C and has a BP of 110/82
• Media Bank mmHg. Her pulse is 95/min and regular. She has a
• New Multimedia respiratory rate of 30/min and coarse inspiratory crackles
to the mid zone on the right hand side. There are marked Reference: Normal Values
Online Extras
cold sores affecting her upper lip. Haematology
Library
Investigations; Immunoglobulins
Community
Help Hb 12.1 g/dl Biochemistry
PasTest Store 9 Diabetes
WCC 14.3 x10 /l

PLT 202 x10 9 /l Endocrinology


My Account
Na + 139 mmol/l Blood gases
Profile
K+ CSF
Newsletters 4.5 mmol/l
My Career and Exams Creatinine 179 µmol/l
Order History CRP 170 mg/l
Learning Goals PO2 9.1 kPa
Question Filters PCO 2 4.3 kPa

Security Which of the following is the most likely cause of her


underlying pneumonia?
Change Password
Sign Out Staphylococcus aureus
Streptococcus pneumoniae
Klebsiella pneumoniae
Mycoplasma pneumoniae
Chlamydia pneumoniae

22426

© 2011 PasTest Ltd | About Us | Contact Us | Help

file:///E|/Shakhawan/Respiratory%20S/349.htm[3/13/2012 4:13:13 PM]


MyPasTest

Main Navigation
Question Browser: MRCP 1
Home
Subscriptions Question Browser Exam Builder Saved Exams
» MRCP 1 Session Progress
• Question Browser
Questions Correct 22
• Timed Test A 32-year-old woman is admitted to the Emergency room
with a severe cough and shortness of breath. She has been Questions Incorrect 7
• Mock Exam
unwell for a few days with a cough and sore throat and Questions Total 29
• Past Papers now says she is coughing purulent sputum which is rust
Questions Percentage 75 %
• Random Questions coloured/ blood stained. She has a history of asthma which
is usually managed with a seretide 50 inhaler. On More
• My Performance
examination she is pyrexial 38.2 o C and has a BP of 110/82
• Media Bank mmHg. Her pulse is 95/min and regular. She has a
• New Multimedia respiratory rate of 30/min and coarse inspiratory crackles
to the mid zone on the right hand side. There are marked
Online Extras
cold sores affecting her upper lip.
Library
Investigations;
Community
Help Hb 12.1 g/dl
PasTest Store
WCC 14.3 x10 9 /l

PLT 202 x10 9 /l


My Account
Na + 139 mmol/l
Profile
Newsletters K+ 4.5 mmol/l
My Career and Exams Creatinine 179 µmol/l
Order History CRP 170 mg/l Reference: Normal Values
Learning Goals PO2 9.1 kPa Haematology
Question Filters PCO 2 4.3 kPa Immunoglobulins
Biochemistry
Security Which of the following is the most likely cause of her
underlying pneumonia? Diabetes
Change Password
Sign Out Staphylococcus aureus Endocrinology
Streptococcus pneumoniae Your answer Blood gases
Klebsiella pneumoniae CSF
Mycoplasma pneumoniae
Chlamydia pneumoniae

The history is very typical of community acquired


pneumonia, and the clinical findings and investigations
are also consistent with this. Herpes labialis is usually
associated with S. pneumoniae infection.
Staphylococcus aureus pneumonia would normally be
considered to be a post-influenza phenomenon and
both Klebsiella and Staphylococcus are associated with
empyema formation and cavitating lung lesions. Usual
management involves the combination of both a
penicillin such as amoxicillin and a macrolide such as
clarithromycin.

22426

file:///E|/Shakhawan/Respiratory%20S/349a.htm[3/13/2012 4:13:15 PM]


MyPasTest

Main Navigation
Question Browser: MRCP 1
Home
Subscriptions Question Browser Exam Builder Saved Exams
» MRCP 1 Session Progress
• Question Browser
Questions Correct 22
• Timed Test A 19-year-old woman presents with sudden onset of left
sided pleuritic chest pain. She has no previous respiratory Questions Incorrect 7
• Mock Exam
history of note and her only medication is the oral Questions Total 29
• Past Papers contraceptive pill. Her respiratory rate is 12/min, she is not
Questions Percentage 75 %
• Random Questions cyanotic and her oxygen sats are 98% on room air.
• My Performance More
Investigations reveal;
• Media Bank
CXR – 1.2cm rim of air consistent with a pneumothorax
• New Multimedia abutting the wall of the left lung. Reference: Normal Values
Online Extras
Which of the following is the most appropriate Haematology
Library management plan?
Community Immunoglobulins
Help Admit for observation
Biochemistry
PasTest Store Needle aspiration
Diabetes
Insert a formal chest drain with suction
Discharge and review in outpatients Endocrinology
My Account
Insert a formal chest drain without suction Blood gases
Profile
CSF
Newsletters 22461
My Career and Exams
Order History
Learning Goals
Question Filters

Security

Change Password
Sign Out

© 2011 PasTest Ltd | About Us | Contact Us | Help

file:///E|/Shakhawan/Respiratory%20S/350.htm[3/13/2012 4:13:16 PM]


MyPasTest

Main Navigation
Question Browser: MRCP 1
Home
Subscriptions Question Browser Exam Builder Saved Exams
» MRCP 1 Session Progress
• Question Browser
Questions Correct 22
• Timed Test A 19-year-old woman presents with sudden onset of left sided pleuritic chest pain. She has no previous respiratory
history of note and her only medication is the oral contraceptive pill. Her respiratory rate is 12/min, she is not Questions Incorrect 8
• Mock Exam
cyanotic and her oxygen sats are 98% on room air. Questions Total 30
• Past Papers
Investigations reveal; Questions Percentage 73 %
• Random Questions
• My Performance More
CXR – 1.2cm rim of air consistent with a pneumothorax abutting the wall of the left lung.
• Media Bank
Which of the following is the most appropriate management plan?
• New Multimedia
Online Extras Admit for observation Your answer
Library Needle aspiration
Community Insert a formal chest drain with suction
Help Discharge and review in outpatients Correct answer
PasTest Store Insert a formal chest drain without suction

My Account

Profile
The answer is discharge and review in outpatients with a repeat CXR. BTS guidelines on the management of
Newsletters pneumothorax with minimal symptoms indicate that discharge with appropriate follow up is appropriate. It
My Career and Exams should be stressed upon patients however that if they suffer any increase in breathlessness at all, then they
should return to the hospital immediately. Where there is breathlessness or the rim of air is greater than 2cm
Order History then aspiration is recommended. Reference: Normal Values
Learning Goals Haematology
http://www.brit-
Question Filters thoracic.org.uk/Portals/0/Clinical%20Information/Pleural%20Disease/Guidelines/PleuralDiseaseSpontaneous.pdf Immunoglobulins

Biochemistry
Security
22461
Diabetes
Change Password
Sign Out Endocrinology

Blood gases

CSF

© 2011 PasTest Ltd | About Us | Contact Us | Help

file:///E|/Shakhawan/Respiratory%20S/350a.htm[3/13/2012 4:13:18 PM]


MyPasTest

Main Navigation
Question Browser: MRCP 1
Home
Subscriptions Question Browser Exam Builder Saved Exams
» MRCP 1 Session Progress
• Question Browser
Questions Correct 22
• Timed Test A 67-year-old retired boilermaker is referred to the
respiratory clinic complaining of shortness of breath. He Questions Incorrect 8
• Mock Exam
also has a persistent dry cough which he tells you is non- Questions Total 30
• Past Papers productive of sputum. His GP diagnosed him with asthma
Questions Percentage 73 %
• Random Questions and prescribed steroid and salbutamol inhalers, but he tells
you this seems to have made little difference to his More
• My Performance
symptoms. On examination he looks tired, his BMI is 23,
• Media Bank his pulse 85/min and his BP is 142/84 mmHg. His heart
• New Multimedia sounds are normal. Respiratory examination reveals end
expiratory crackles to the mid-zones on both sides. He is Reference: Normal Values
Online Extras also clubbed.
Haematology
Library
Investigations;
Community Immunoglobulins
Help Hb 13.4 g/dl Biochemistry
PasTest Store WCC 7.9 x10 9 /l
Diabetes
PLT 301 x10 9 /l Endocrinology
My Account
Na + 138 mmol/l Blood gases
Profile
K+ 4.9 mmol/l CSF
Newsletters
My Career and Exams Creatinine 131 µmol/l

Order History FEV1 60% predicted


Learning Goals FVC 70% predicted
Question Filters Calcified pleural plaques, areas of interstitial
CXR fibrosis

Security Which of the following is the next best step to


Change Password confirm the diagnosis?

Sign Out TLCO


High resolution CT
Transbronchial biopsy
Bronchioalveolar lavage
Serum ACE

22462

© 2011 PasTest Ltd | About Us | Contact Us | Help

file:///E|/Shakhawan/Respiratory%20S/351.htm[3/13/2012 4:13:19 PM]


MyPasTest

Main Navigation
Question Browser: MRCP 1
Home
Subscriptions Question Browser Exam Builder Saved Exams
» MRCP 1 Session Progress
• Question Browser
Questions Correct 23
• Timed Test A 67-year-old retired boilermaker is referred to the
respiratory clinic complaining of shortness of breath. He Questions Incorrect 8
• Mock Exam
also has a persistent dry cough which he tells you is non- Questions Total 31
• Past Papers productive of sputum. His GP diagnosed him with asthma
Questions Percentage 74 %
• Random Questions and prescribed steroid and salbutamol inhalers, but he tells
you this seems to have made little difference to his More
• My Performance
symptoms. On examination he looks tired, his BMI is 23,
• Media Bank his pulse 85/min and his BP is 142/84 mmHg. His heart
• New Multimedia sounds are normal. Respiratory examination reveals end
expiratory crackles to the mid-zones on both sides. He is
Online Extras also clubbed.
Library
Investigations;
Community
Help Hb 13.4 g/dl
PasTest Store WCC 7.9 x10 9 /l

PLT 301 x10 9 /l


My Account
Na + 138 mmol/l
Profile
K+ 4.9 mmol/l
Newsletters
My Career and Exams Creatinine 131 µmol/l

Order History FEV1 60% predicted Reference: Normal Values


Learning Goals FVC 70% predicted Haematology
Question Filters Calcified pleural plaques, areas of interstitial
Immunoglobulins
CXR fibrosis
Biochemistry
Security Which of the following is the next best step to
confirm the diagnosis? Diabetes
Change Password
Sign Out Endocrinology
TLCO
High resolution CT Your answer Blood gases

Transbronchial biopsy CSF


Bronchioalveolar lavage
Serum ACE

There is strong evidence from the history, clinical


examination, and investigations with which we are
provided that this patient may have asbestosis.
Transbronchial biopsy is very unlikely to yield a
significant enough quantity of tissue to confirm the
diagnosis, and samples may be subject to crush
damage, making histology difficult to interpret. We are
given the history of his previous occupation as a boiler
maker, meaning that asbestos exposure is likely to be
an important feature in the underlying diagnosis. Serum
ACE may be useful if sarcoidosis is suspected but there
are no real pointers to this diagnosis here. High
resolution CT is the most useful investigation, allowing
clear delineation of pleural plaques and demonstration
of pulmonary fibrosis.

22462

file:///E|/Shakhawan/Respiratory%20S/351a.htm[3/13/2012 4:13:21 PM]


MyPasTest

Main Navigation
Question Browser: MRCP 1
Home
Subscriptions Question Browser Exam Builder Saved Exams
» MRCP 1 Session Progress
• Question Browser
Questions Correct 23
• Timed Test A 72-year-old man comes to the clinic with increased
shortness of breath and wheeze. His GP has tried him on a Questions Incorrect 8
• Mock Exam
regular salbutamol inhaler, but this has had little measured Questions Total 31
• Past Papers effect on his symptoms. Unfortunately he continues to
Questions Percentage 74 %
• Random Questions smoke 30 cigarettes per day and has done for the past 50
years. On examination his BP is 155/85 mmHg and his BMI More
• My Performance
is 21. He has respiratory changes consistent with COPD
• Media Bank including coarse wheeze throughout both lung fields and
• New Multimedia crackles.
Reference: Normal Values
Online Extras Investigations;
Haematology
Library
Hb 12.3 g/dl Immunoglobulins
Community
9 /l
Help WCC 6.7 x10 Biochemistry
PasTest Store PLT 201 x10 9 /l Diabetes
Na + 139 mmol/l Endocrinology
My Account
K+ 4.9 mmol/l Blood gases
Profile
Creatinine 130 µmol/l CSF
Newsletters
FEV1 50% of predicted
My Career and Exams
Transfer factor reduced
Order History
Learning Goals Which of the following most closely correlates with
Question Filters mortality in COPD?

FEV1
Security
PEFR
Change Password Residual volume
Sign Out Total lung capacity
Transfer factor

22528

© 2011 PasTest Ltd | About Us | Contact Us | Help

file:///E|/Shakhawan/Respiratory%20S/352.htm[3/13/2012 4:13:23 PM]


MyPasTest

Main Navigation
Question Browser: MRCP 1
Home
Subscriptions Question Browser Exam Builder Saved Exams
» MRCP 1 Session Progress
• Question Browser
Questions Correct 24
• Timed Test A 72-year-old man comes to the clinic with increased
shortness of breath and wheeze. His GP has tried him on a Questions Incorrect 8
• Mock Exam
regular salbutamol inhaler, but this has had little measured Questions Total 32
• Past Papers effect on his symptoms. Unfortunately he continues to
Questions Percentage 75 %
• Random Questions smoke 30 cigarettes per day and has done for the past 50
years. On examination his BP is 155/85 mmHg and his BMI More
• My Performance
is 21. He has respiratory changes consistent with COPD
• Media Bank including coarse wheeze throughout both lung fields and
• New Multimedia crackles.

Online Extras Investigations;


Library
Hb 12.3 g/dl
Community
Help WCC 6.7 x10 9 /l
PasTest Store PLT 201 x10 9 /l

Na + 139 mmol/l
My Account
K+ 4.9 mmol/l
Profile
Creatinine 130 µmol/l
Newsletters
FEV1 50% of predicted
My Career and Exams
Transfer factor reduced
Order History Reference: Normal Values
Learning Goals Which of the following most closely correlates with Haematology
Question Filters mortality in COPD?
Immunoglobulins
FEV1 Your answer Biochemistry
Security
PEFR
Diabetes
Change Password Residual volume
Sign Out Endocrinology
Total lung capacity
Transfer factor Blood gases

CSF

FEV1 is the factor which correlates most closely with


mortality in COPD. Other factors have been built in to
the BODE index, an index of mortality in COPD
developed by the MRC. This also incorporates BMI,
dyspnoea, and exercise capacity as evidenced by a 6
minute walk test. The BODE index can also be used to
estimate risk of hospitalisation. Stopping smoking will
have the biggest impact on this man’s prognosis, with
regular short acting anti-cholinergics and beta-agonists
useful for symptomatic relief. Given that he has severe
COPD as evidenced by his FEV1 , a trial of high dose
inhaled corticosteroid and beta agonist therapy may be
appropriate.

22528

© 2011 PasTest Ltd | About Us | Contact Us | Help

file:///E|/Shakhawan/Respiratory%20S/352a.htm[3/13/2012 4:13:24 PM]

Das könnte Ihnen auch gefallen